Sunteți pe pagina 1din 527

Republic of the Philippines

SUPREME COURT
Manila
EN BANC
G.R No. 187167

August 16, 2011

PROF. MERLIN M. MAGALLONA, AKBAYAN PARTY-LIST REP. RISA HONTIVEROS, PROF.


HARRY C. ROQUE, JR., AND UNIVERSITY OF THE PHILIPPINES COLLEGE OF LAW
STUDENTS, ALITHEA BARBARA ACAS, VOLTAIRE ALFERES, CZARINA MAY ALTEZ,
FRANCIS ALVIN ASILO, SHERYL BALOT, RUBY AMOR BARRACA, JOSE JAVIER BAUTISTA,
ROMINA BERNARDO, VALERIE PAGASA BUENAVENTURA, EDAN MARRI CAETE, VANN
ALLEN DELA CRUZ, RENE DELORINO, PAULYN MAY DUMAN, SHARON ESCOTO, RODRIGO
FAJARDO III, GIRLIE FERRER, RAOULLE OSEN FERRER, CARLA REGINA GREPO, ANNA
MARIE CECILIA GO, IRISH KAY KALAW, MARY ANN JOY LEE, MARIA LUISA MANALAYSAY,
MIGUEL RAFAEL MUSNGI, MICHAEL OCAMPO, JAKLYN HANNA PINEDA, WILLIAM
RAGAMAT, MARICAR RAMOS, ENRIK FORT REVILLAS, JAMES MARK TERRY RIDON,
JOHANN FRANTZ RIVERA IV, CHRISTIAN RIVERO, DIANNE MARIE ROA, NICHOLAS
SANTIZO, MELISSA CHRISTINA SANTOS, CRISTINE MAE TABING, VANESSA ANNE TORNO,
MARIA ESTER VANGUARDIA, and MARCELINO VELOSO III, Petitioners,
vs.
HON. EDUARDO ERMITA, IN HIS CAPACITY AS EXECUTIVE SECRETARY, HON. ALBERTO
ROMULO, IN HIS CAPACITY AS SECRETARY OF THE DEPARTMENT OF FOREIGN AFFAIRS,
HON. ROLANDO ANDAYA, IN HIS CAPACITY AS SECRETARY OF THE DEPARTMENT OF
BUDGET AND MANAGEMENT, HON. DIONY VENTURA, IN HIS CAPACITY AS
ADMINISTRATOR OF THE NATIONAL MAPPING & RESOURCE INFORMATION AUTHORITY,
and HON. HILARIO DAVIDE, JR., IN HIS CAPACITY AS REPRESENTATIVE OF THE
PERMANENT MISSION OF THE REPUBLIC OF THE PHILIPPINES TO THE UNITED
NATIONS,Respondents.
DECISION
CARPIO, J.:
The Case
This original action for the writs of certiorari and prohibition assails the constitutionality of Republic
Act No. 95221(RA 9522) adjusting the countrys archipelagic baselines and classifying the baseline
regime of nearby territories.
The Antecedents
In 1961, Congress passed Republic Act No. 3046 (RA 3046) 2 demarcating the maritime baselines of
the Philippines as an archipelagic State.3 This law followed the framing of the Convention on the
Territorial Sea and the Contiguous Zone in 1958 (UNCLOS I),4 codifying, among others, the
sovereign right of States parties over their "territorial sea," the breadth of which, however, was left
undetermined. Attempts to fill this void during the second round of negotiations in Geneva in 1960
(UNCLOS II) proved futile. Thus, domestically, RA 3046 remained unchanged for nearly five

decades, save for legislation passed in 1968 (Republic Act No. 5446 [RA 5446]) correcting
typographical errors and reserving the drawing of baselines around Sabah in North Borneo.
In March 2009, Congress amended RA 3046 by enacting RA 9522, the statute now under scrutiny.
The change was prompted by the need to make RA 3046 compliant with the terms of the United
Nations Convention on the Law of the Sea (UNCLOS III),5 which the Philippines ratified on 27
February 1984.6 Among others, UNCLOS III prescribes the water-land ratio, length, and contour of
baselines of archipelagic States like the Philippines7 and sets the deadline for the filing of application
for the extended continental shelf.8 Complying with these requirements, RA 9522 shortened one
baseline, optimized the location of some basepoints around the Philippine archipelago and classified
adjacent territories, namely, the Kalayaan Island Group (KIG) and the Scarborough Shoal, as
"regimes of islands" whose islands generate their own applicable maritime zones.
Petitioners, professors of law, law students and a legislator, in their respective capacities as
"citizens, taxpayers or x x x legislators,"9 as the case may be, assail the constitutionality of RA 9522
on two principal grounds, namely: (1) RA 9522 reduces Philippine maritime territory, and logically,
the reach of the Philippine states sovereign power, in violation of Article 1 of the 1987
Constitution,10 embodying the terms of the Treaty of Paris11 and ancillary treaties,12 and (2) RA 9522
opens the countrys waters landward of the baselines to maritime passage by all vessels and
aircrafts, undermining Philippine sovereignty and national security, contravening the countrys
nuclear-free policy, and damaging marine resources, in violation of relevant constitutional
provisions.13
In addition, petitioners contend that RA 9522s treatment of the KIG as "regime of islands" not only
results in the loss of a large maritime area but also prejudices the livelihood of subsistence
fishermen.14 To buttress their argument of territorial diminution, petitioners facially attack RA 9522 for
what it excluded and included its failure to reference either the Treaty of Paris or Sabah and its use
of UNCLOS IIIs framework of regime of islands to determine the maritime zones of the KIG and the
Scarborough Shoal.
Commenting on the petition, respondent officials raised threshold issues questioning (1) the
petitions compliance with the case or controversy requirement for judicial review grounded on
petitioners alleged lack of locus standiand (2) the propriety of the writs of certiorari and prohibition to
assail the constitutionality of RA 9522. On the merits, respondents defended RA 9522 as the
countrys compliance with the terms of UNCLOS III, preserving Philippine territory over the KIG or
Scarborough Shoal. Respondents add that RA 9522 does not undermine the countrys security,
environment and economic interests or relinquish the Philippines claim over Sabah.
Respondents also question the normative force, under international law, of petitioners assertion that
what Spain ceded to the United States under the Treaty of Paris were the islands and all the
waters found within the boundaries of the rectangular area drawn under the Treaty of Paris.
We left unacted petitioners prayer for an injunctive writ.
The Issues
The petition raises the following issues:
1. Preliminarily
1. Whether petitioners possess locus standi to bring this suit; and

2. Whether the writs of certiorari and prohibition are the proper remedies to assail the
constitutionality of RA 9522.
2. On the merits, whether RA 9522 is unconstitutional.
The Ruling of the Court
On the threshold issues, we hold that (1) petitioners possess locus standi to bring this suit as
citizens and (2) the writs of certiorari and prohibition are proper remedies to test the constitutionality
of RA 9522. On the merits, we find no basis to declare RA 9522 unconstitutional.
On the Threshold Issues
Petitioners Possess Locus
Standi as Citizens
Petitioners themselves undermine their assertion of locus standi as legislators and taxpayers
because the petition alleges neither infringement of legislative prerogative 15 nor misuse of public
funds,16 occasioned by the passage and implementation of RA 9522. Nonetheless, we recognize
petitioners locus standi as citizens with constitutionally sufficient interest in the resolution of the
merits of the case which undoubtedly raises issues of national significance necessitating urgent
resolution. Indeed, owing to the peculiar nature of RA 9522, it is understandably difficult to find other
litigants possessing "a more direct and specific interest" to bring the suit, thus satisfying one of the
requirements for granting citizenship standing.17
The Writs of Certiorari and Prohibition
Are Proper Remedies to Test
the Constitutionality of Statutes
In praying for the dismissal of the petition on preliminary grounds, respondents seek a strict
observance of the offices of the writs of certiorari and prohibition, noting that the writs cannot issue
absent any showing of grave abuse of discretion in the exercise of judicial, quasi-judicial or
ministerial powers on the part of respondents and resulting prejudice on the part of petitioners. 18
Respondents submission holds true in ordinary civil proceedings. When this Court exercises its
constitutional power of judicial review, however, we have, by tradition, viewed the writs of certiorari
and prohibition as proper remedial vehicles to test the constitutionality of statutes, 19 and indeed, of
acts of other branches of government.20 Issues of constitutional import are sometimes crafted out of
statutes which, while having no bearing on the personal interests of the petitioners, carry such
relevance in the life of this nation that the Court inevitably finds itself constrained to take cognizance
of the case and pass upon the issues raised, non-compliance with the letter of procedural rules
notwithstanding. The statute sought to be reviewed here is one such law.
RA 9522 is Not Unconstitutional
RA 9522 is a Statutory Tool
to Demarcate the Countrys
Maritime Zones and Continental
Shelf Under UNCLOS III, not to
Delineate Philippine Territory
Petitioners submit that RA 9522 "dismembers a large portion of the national territory" 21 because it
discards the pre-UNCLOS III demarcation of Philippine territory under the Treaty of Paris and related

treaties, successively encoded in the definition of national territory under the 1935, 1973 and 1987
Constitutions. Petitioners theorize that this constitutional definition trumps any treaty or statutory
provision denying the Philippines sovereign control over waters, beyond the territorial sea
recognized at the time of the Treaty of Paris, that Spain supposedly ceded to the United States.
Petitioners argue that from the Treaty of Paris technical description, Philippine sovereignty over
territorial waters extends hundreds of nautical miles around the Philippine archipelago, embracing
the rectangular area delineated in the Treaty of Paris.22
Petitioners theory fails to persuade us.
UNCLOS III has nothing to do with the acquisition (or loss) of territory. It is a multilateral treaty
regulating, among others, sea-use rights over maritime zones (i.e., the territorial waters [12 nautical
miles from the baselines], contiguous zone [24 nautical miles from the baselines], exclusive
economic zone [200 nautical miles from the baselines]), and continental shelves that UNCLOS III
delimits.23 UNCLOS III was the culmination of decades-long negotiations among United Nations
members to codify norms regulating the conduct of States in the worlds oceans and submarine
areas, recognizing coastal and archipelagic States graduated authority over a limited span of waters
and submarine lands along their coasts.
On the other hand, baselines laws such as RA 9522 are enacted by UNCLOS III States parties to
mark-out specific basepoints along their coasts from which baselines are drawn, either straight or
contoured, to serve as geographic starting points to measure the breadth of the maritime zones and
continental shelf. Article 48 of UNCLOS III on archipelagic States like ours could not be any clearer:
Article 48. Measurement of the breadth of the territorial sea, the contiguous zone, the exclusive
economic zone and the continental shelf. The breadth of the territorial sea, the contiguous zone,
the exclusive economic zone and the continental shelf shall be measured from archipelagic
baselines drawn in accordance with article 47. (Emphasis supplied)
Thus, baselines laws are nothing but statutory mechanisms for UNCLOS III States parties to delimit
with precision the extent of their maritime zones and continental shelves. In turn, this gives notice to
the rest of the international community of the scope of the maritime space and submarine areas
within which States parties exercise treaty-based rights, namely, the exercise of sovereignty over
territorial waters (Article 2), the jurisdiction to enforce customs, fiscal, immigration, and sanitation
laws in the contiguous zone (Article 33), and the right to exploit the living and non-living resources in
the exclusive economic zone (Article 56) and continental shelf (Article 77).
Even under petitioners theory that the Philippine territory embraces the islands and all the
waters within the rectangular area delimited in the Treaty of Paris, the baselines of the Philippines
would still have to be drawn in accordance with RA 9522 because this is the only way to draw the
baselines in conformity with UNCLOS III. The baselines cannot be drawn from the boundaries or
other portions of the rectangular area delineated in the Treaty of Paris, but from the "outermost
islands and drying reefs of the archipelago."24
UNCLOS III and its ancillary baselines laws play no role in the acquisition, enlargement or, as
petitioners claim, diminution of territory. Under traditional international law typology, States acquire
(or conversely, lose) territory through occupation, accretion, cession and prescription, 25 not by
executing multilateral treaties on the regulations of sea-use rights or enacting statutes to comply with
the treatys terms to delimit maritime zones and continental shelves. Territorial claims to land
features are outside UNCLOS III, and are instead governed by the rules on general international
law.26

RA 9522s Use of the Framework


of Regime of Islands to Determine the
Maritime Zones of the KIG and the
Scarborough Shoal, not Inconsistent
with the Philippines Claim of Sovereignty
Over these Areas
Petitioners next submit that RA 9522s use of UNCLOS IIIs regime of islands framework to draw the
baselines, and to measure the breadth of the applicable maritime zones of the KIG, "weakens our
territorial claim" over that area.27 Petitioners add that the KIGs (and Scarborough Shoals) exclusion
from the Philippine archipelagic baselines results in the loss of "about 15,000 square nautical miles
of territorial waters," prejudicing the livelihood of subsistence fishermen. 28 A comparison of the
configuration of the baselines drawn under RA 3046 and RA 9522 and the extent of maritime space
encompassed by each law, coupled with a reading of the text of RA 9522 and its congressional
deliberations, vis--vis the Philippines obligations under UNCLOS III, belie this view.
1avvphi1

The configuration of the baselines drawn under RA 3046 and RA 9522 shows that RA 9522 merely
followed the basepoints mapped by RA 3046, save for at least nine basepoints that RA 9522 skipped
to optimize the location of basepoints and adjust the length of one baseline (and thus comply with
UNCLOS IIIs limitation on the maximum length of baselines). Under RA 3046, as under RA 9522,
the KIG and the Scarborough Shoal lie outside of the baselines drawn around the Philippine
archipelago. This undeniable cartographic fact takes the wind out of petitioners argument branding
RA 9522 as a statutory renunciation of the Philippines claim over the KIG, assuming that baselines
are relevant for this purpose.
Petitioners assertion of loss of "about 15,000 square nautical miles of territorial waters" under RA
9522 is similarly unfounded both in fact and law. On the contrary, RA 9522, by optimizing the location
of basepoints, increasedthe Philippines total maritime space (covering its internal waters, territorial
sea and exclusive economic zone) by 145,216 square nautical miles, as shown in the table below: 29

Extent of maritime
area using RA 3046,
as amended, taking
into account the
Treaty of Paris
delimitation (in
square nautical
miles)

Extent of maritime
area using RA 9522,
taking into account
UNCLOS III (in
square nautical
miles)

Internal or
archipelagic
waters

166,858

171,435

Territorial Sea

274,136

32,106

Exclusive
Economic Zone
TOTAL

382,669
440,994

586,210

Thus, as the map below shows, the reach of the exclusive economic zone drawn under RA 9522
even extends way beyond the waters covered by the rectangular demarcation under the Treaty of
Paris. Of course, where there are overlapping exclusive economic zones of opposite or adjacent
States, there will have to be a delineation of maritime boundaries in accordance with UNCLOS III. 30

Further, petitioners argument that the KIG now lies outside Philippine territory because the baselines
that RA 9522 draws do not enclose the KIG is negated by RA 9522 itself. Section 2 of the law
commits to text the Philippines continued claim of sovereignty and jurisdiction over the KIG and the
Scarborough Shoal:
SEC. 2. The baselines in the following areas over which the Philippines likewise exercises
sovereignty and jurisdiction shall be determined as "Regime of Islands" under the Republic of the
Philippines consistent with Article 121 of the United Nations Convention on the Law of the Sea
(UNCLOS):

a) The Kalayaan Island Group as constituted under Presidential Decree No. 1596 and
b) Bajo de Masinloc, also known as Scarborough Shoal. (Emphasis supplied)
Had Congress in RA 9522 enclosed the KIG and the Scarborough Shoal as part of the Philippine
archipelago, adverse legal effects would have ensued. The Philippines would have committed a
breach of two provisions of UNCLOS III. First, Article 47 (3) of UNCLOS III requires that "[t]he
drawing of such baselines shall not depart to any appreciable extent from the general configuration
of the archipelago." Second, Article 47 (2) of UNCLOS III requires that "the length of the baselines
shall not exceed 100 nautical miles," save for three per cent (3%) of the total number of baselines
which can reach up to 125 nautical miles.31
Although the Philippines has consistently claimed sovereignty over the KIG 32 and the Scarborough
Shoal for several decades, these outlying areas are located at an appreciable distance from the
nearest shoreline of the Philippine archipelago,33 such that any straight baseline loped around them
from the nearest basepoint will inevitably "depart to an appreciable extent from the general
configuration of the archipelago."
The principal sponsor of RA 9522 in the Senate, Senator Miriam Defensor-Santiago, took pains to
emphasize the foregoing during the Senate deliberations:
What we call the Kalayaan Island Group or what the rest of the world call[] the Spratlys and the
Scarborough Shoal are outside our archipelagic baseline because if we put them inside our
baselines we might be accused of violating the provision of international law which states: "The
drawing of such baseline shall not depart to any appreciable extent from the general configuration of
the archipelago." So sa loob ng ating baseline, dapat magkalapit ang mga islands. Dahil malayo ang
Scarborough Shoal, hindi natin masasabing malapit sila sa atin although we are still allowed by
international law to claim them as our own.
This is called contested islands outside our configuration. We see that our archipelago is defined by
the orange line which [we] call[] archipelagic baseline. Ngayon, tingnan ninyo ang maliit na circle
doon sa itaas, that is Scarborough Shoal, itong malaking circle sa ibaba, that is Kalayaan Group or
the Spratlys. Malayo na sila sa ating archipelago kaya kung ilihis pa natin ang dating archipelagic
baselines para lamang masama itong dalawang circles, hindi na sila magkalapit at baka hindi na
tatanggapin ng United Nations because of the rule that it should follow the natural configuration of
the archipelago.34 (Emphasis supplied)
Similarly, the length of one baseline that RA 3046 drew exceeded UNCLOS IIIs limits. The need to
shorten this baseline, and in addition, to optimize the location of basepoints using current maps,
became imperative as discussed by respondents:
1avvphi1

[T]he amendment of the baselines law was necessary to enable the Philippines to draw the outer
limits of its maritime zones including the extended continental shelf in the manner provided by Article
47 of [UNCLOS III]. As defined by R.A. 3046, as amended by R.A. 5446, the baselines suffer from
some technical deficiencies, to wit:
1. The length of the baseline across Moro Gulf (from Middle of 3 Rock Awash to Tongquil
Point) is 140.06 nautical miles x x x. This exceeds the maximum length allowed under Article
47(2) of the [UNCLOS III], which states that "The length of such baselines shall not exceed
100 nautical miles, except that up to 3 per cent of the total number of baselines enclosing
any archipelago may exceed that length, up to a maximum length of 125 nautical miles."

2. The selection of basepoints is not optimal. At least 9 basepoints can be skipped or deleted
from the baselines system. This will enclose an additional 2,195 nautical miles of water.
3. Finally, the basepoints were drawn from maps existing in 1968, and not established by
geodetic survey methods. Accordingly, some of the points, particularly along the west coasts
of Luzon down to Palawan were later found to be located either inland or on water, not on
low-water line and drying reefs as prescribed by Article 47. 35
Hence, far from surrendering the Philippines claim over the KIG and the Scarborough Shoal,
Congress decision to classify the KIG and the Scarborough Shoal as "Regime[s] of Islands under
the Republic of the Philippines consistent with Article 121" 36 of UNCLOS III manifests the Philippine
States responsible observance of its pacta sunt servanda obligation under UNCLOS III. Under
Article 121 of UNCLOS III, any "naturally formed area of land, surrounded by water, which is above
water at high tide," such as portions of the KIG, qualifies under the category of "regime of islands,"
whose islands generate their own applicable maritime zones.37
Statutory Claim Over Sabah under
RA 5446 Retained
Petitioners argument for the invalidity of RA 9522 for its failure to textualize the Philippines claim
over Sabah in North Borneo is also untenable. Section 2 of RA 5446, which RA 9522 did not repeal,
keeps open the door for drawing the baselines of Sabah:
Section 2. The definition of the baselines of the territorial sea of the Philippine Archipelago as
provided in this Actis without prejudice to the delineation of the baselines of the territorial sea
around the territory of Sabah, situated in North Borneo, over which the Republic of the
Philippines has acquired dominion and sovereignty. (Emphasis supplied)
UNCLOS III and RA 9522 not
Incompatible with the Constitutions
Delineation of Internal Waters
As their final argument against the validity of RA 9522, petitioners contend that the law
unconstitutionally "converts" internal waters into archipelagic waters, hence subjecting these waters
to the right of innocent and sea lanes passage under UNCLOS III, including overflight. Petitioners
extrapolate that these passage rights indubitably expose Philippine internal waters to nuclear and
maritime pollution hazards, in violation of the Constitution. 38
Whether referred to as Philippine "internal waters" under Article I of the Constitution 39 or as
"archipelagic waters" under UNCLOS III (Article 49 [1]), the Philippines exercises sovereignty over
the body of water lying landward of the baselines, including the air space over it and the submarine
areas underneath. UNCLOS III affirms this:
Article 49. Legal status of archipelagic waters, of the air space over archipelagic waters and of their
bed and subsoil.
1. The sovereignty of an archipelagic State extends to the waters enclosed by the
archipelagic baselines drawn in accordance with article 47, described as archipelagic
waters, regardless of their depth or distance from the coast.

2. This sovereignty extends to the air space over the archipelagic waters, as well as to
their bed and subsoil, and the resources contained therein.
xxxx
4. The regime of archipelagic sea lanes passage established in this Part shall not in other
respects affect the status of the archipelagic waters, including the sea lanes, or the
exercise by the archipelagic State of its sovereignty over such waters and their air
space, bed and subsoil, and the resources contained therein. (Emphasis supplied)
The fact of sovereignty, however, does not preclude the operation of municipal and international law
norms subjecting the territorial sea or archipelagic waters to necessary, if not marginal, burdens in
the interest of maintaining unimpeded, expeditious international navigation, consistent with the
international law principle of freedom of navigation. Thus, domestically, the political branches of the
Philippine government, in the competent discharge of their constitutional powers, may pass
legislation designating routes within the archipelagic waters to regulate innocent and sea lanes
passage.40 Indeed, bills drawing nautical highways for sea lanes passage are now pending in
Congress.41
In the absence of municipal legislation, international law norms, now codified in UNCLOS III, operate
to grant innocent passage rights over the territorial sea or archipelagic waters, subject to the treatys
limitations and conditions for their exercise.42 Significantly, the right of innocent passage is a
customary international law,43 thus automatically incorporated in the corpus of Philippine law.44 No
modern State can validly invoke its sovereignty to absolutely forbid innocent passage that is
exercised in accordance with customary international law without risking retaliatory measures from
the international community.
The fact that for archipelagic States, their archipelagic waters are subject to both the right of
innocent passage and sea lanes passage45 does not place them in lesser footing vis-vis continental coastal States which are subject, in their territorial sea, to the right of innocent
passage and the right of transit passage through international straits. The imposition of these
passage rights through archipelagic waters under UNCLOS III was a concession by archipelagic
States, in exchange for their right to claim all the waters landward of their baselines,regardless of
their depth or distance from the coast, as archipelagic waters subject to their territorial sovereignty.
More importantly, the recognition of archipelagic States archipelago and the waters enclosed by
their baselines as one cohesive entity prevents the treatment of their islands as separate islands
under UNCLOS III.46 Separate islands generate their own maritime zones, placing the waters
between islands separated by more than 24 nautical miles beyond the States territorial sovereignty,
subjecting these waters to the rights of other States under UNCLOS III. 47
Petitioners invocation of non-executory constitutional provisions in Article II (Declaration of
Principles and State Policies)48 must also fail. Our present state of jurisprudence considers the
provisions in Article II as mere legislative guides, which, absent enabling legislation, "do not embody
judicially enforceable constitutional rights x x x."49 Article II provisions serve as guides in formulating
and interpreting implementing legislation, as well as in interpreting executory provisions of the
Constitution. Although Oposa v. Factoran50 treated the right to a healthful and balanced ecology
under Section 16 of Article II as an exception, the present petition lacks factual basis to substantiate
the claimed constitutional violation. The other provisions petitioners cite, relating to the protection of
marine wealth (Article XII, Section 2, paragraph 251 ) and subsistence fishermen (Article XIII, Section
752 ), are not violated by RA 9522.

In fact, the demarcation of the baselines enables the Philippines to delimit its exclusive economic
zone, reserving solely to the Philippines the exploitation of all living and non-living resources within
such zone. Such a maritime delineation binds the international community since the delineation is in
strict observance of UNCLOS III. If the maritime delineation is contrary to UNCLOS III, the
international community will of course reject it and will refuse to be bound by it.
UNCLOS III favors States with a long coastline like the Philippines. UNCLOS III creates a sui
generis maritime space the exclusive economic zone in waters previously part of the high seas.
UNCLOS III grants new rights to coastal States to exclusively exploit the resources found within this
zone up to 200 nautical miles.53 UNCLOS III, however, preserves the traditional freedom of
navigation of other States that attached to this zone beyond the territorial sea before UNCLOS III.
RA 9522 and the Philippines Maritime Zones
Petitioners hold the view that, based on the permissive text of UNCLOS III, Congress was not bound
to pass RA 9522.54 We have looked at the relevant provision of UNCLOS III 55 and we find petitioners
reading plausible. Nevertheless, the prerogative of choosing this option belongs to Congress, not to
this Court. Moreover, the luxury of choosing this option comes at a very steep price. Absent an
UNCLOS III compliant baselines law, an archipelagic State like the Philippines will find itself devoid
of internationally acceptable baselines from where the breadth of its maritime zones and continental
shelf is measured. This is recipe for a two-fronted disaster: first, it sends an open invitation to the
seafaring powers to freely enter and exploit the resources in the waters and submarine areas around
our archipelago; and second, it weakens the countrys case in any international dispute over
Philippine maritime space. These are consequences Congress wisely avoided.
The enactment of UNCLOS III compliant baselines law for the Philippine archipelago and adjacent
areas, as embodied in RA 9522, allows an internationally-recognized delimitation of the breadth of
the Philippines maritime zones and continental shelf. RA 9522 is therefore a most vital step on the
part of the Philippines in safeguarding its maritime zones, consistent with the Constitution and our
national interest.
WHEREFORE, we DISMISS the petition.
SO ORDERED.

Republic of the Philippines


SUPREME COURT
Manila
EN BANC
G.R. No. 183591

October 14, 2008

THE PROVINCE OF NORTH COTABATO, duly represented by GOVERNOR JESUS SACDALAN


and/or VICE-GOVERNOR EMMANUEL PIOL, for and in his own behalf, petitioners,
vs.
THE GOVERNMENT OF THE REPUBLIC OF THE PHILIPPINES PEACE PANEL ON
ANCESTRAL DOMAIN (GRP), represented by SEC. RODOLFO GARCIA, ATTY. LEAH
ARMAMENTO, ATTY. SEDFREY CANDELARIA, MARK RYAN SULLIVAN and/or GEN.
HERMOGENES ESPERON, JR., the latter in his capacity as the present and duly-appointed
Presidential Adviser on the Peace Process (OPAPP) or the so-called Office of the Presidential
Adviser on the Peace Process, respondents.
x--------------------------------------------x
G.R. No. 183752

October 14, 2008

CITY GOVERNMENT OF ZAMBOANGA, as represented by HON. CELSO L. LOBREGAT, City


Mayor of Zamboanga, and in his personal capacity as resident of the City of Zamboanga, Rep.
MA. ISABELLE G. CLIMACO, District 1, and Rep. ERICO BASILIO A. FABIAN, District 2, City of
Zamboanga, petitioners,
vs.
THE GOVERNMENT OF THE REPUBLIC OF THE PHILIPPINES PEACE NEGOTIATING PANEL
(GRP), as represented by RODOLFO C. GARCIA, LEAH ARMAMENTO, SEDFREY
CANDELARIA, MARK RYAN SULLIVAN and HERMOGENES ESPERON, in his capacity as the
Presidential Adviser on Peace Process,respondents.
x--------------------------------------------x
G.R. No. 183893

October 14, 2008

THE CITY OF ILIGAN, duly represented by CITY MAYOR LAWRENCE LLUCH CRUZ, petitioner,
vs.
THE GOVERNMENT OF THE REPUBLIC OF THE PHILIPPINES PEACE PANEL ON
ANCESTRAL DOMAIN (GRP), represented by SEC. RODOLFO GARCIA, ATTY. LEAH
ARMAMENTO, ATTY. SEDFREY CANDELARIA, MARK RYAN SULLIVAN; GEN. HERMOGENES
ESPERON, JR., in his capacity as the present and duly appointed Presidential Adviser on the
Peace Process; and/or SEC. EDUARDO ERMITA, in his capacity as Executive
Secretary. respondents.
x--------------------------------------------x

G.R. No. 183951

October 14, 2008

THE PROVINCIAL GOVERNMENT OF ZAMBOANGA DEL NORTE, as represented by HON.


ROLANDO E. YEBES, in his capacity as Provincial Governor, HON. FRANCIS H. OLVIS, in his
capacity as Vice-Governor and Presiding Officer of the Sangguniang Panlalawigan, HON.
CECILIA JALOSJOS CARREON, Congresswoman, 1st Congressional District, HON. CESAR G.
JALOSJOS, Congressman, 3rdCongressional District, and Members of the Sangguniang
Panlalawigan of the Province of Zamboanga del Norte, namely, HON. SETH FREDERICK P.
JALOSJOS, HON. FERNANDO R. CABIGON, JR., HON. ULDARICO M. MEJORADA II, HON.
EDIONAR M. ZAMORAS, HON. EDGAR J. BAGUIO, HON. CEDRIC L. ADRIATICO, HON.
FELIXBERTO C. BOLANDO, HON. JOSEPH BRENDO C. AJERO, HON. NORBIDEIRI B.
EDDING, HON. ANECITO S. DARUNDAY, HON. ANGELICA J. CARREON and HON.
LUZVIMINDA E. TORRINO, petitioners,
vs.
THE GOVERNMENT OF THE REPUBLIC OF THE PHILIPPINES PEACE NEGOTIATING PANEL
[GRP], as represented by HON. RODOLFO C. GARCIA and HON. HERMOGENES ESPERON, in
his capacity as the Presidential Adviser of Peace Process, respondents.
x--------------------------------------------x
G.R. No. 183962

October 14, 2008

ERNESTO M. MACEDA, JEJOMAR C. BINAY, and AQUILINO L. PIMENTEL III, petitioners,


vs.
THE GOVERNMENT OF THE REPUBLIC OF THE PHILIPPINES PEACE NEGOTIATING PANEL,
represented by its Chairman RODOLFO C. GARCIA, and the MORO ISLAMIC LIBERATION
FRONT PEACE NEGOTIATING PANEL, represented by its Chairman MOHAGHER
IQBAL, respondents.
x--------------------------------------------x
FRANKLIN M. DRILON and ADEL ABBAS TAMANO, petitioners-in-intervention.
x--------------------------------------------x
SEN. MANUEL A. ROXAS, petitioners-in-intervention.
x--------------------------------------------x
MUNICIPALITY OF LINAMON duly represented by its Municipal Mayor NOEL N.
DEANO, petitioners-in-intervention,
x--------------------------------------------x
THE CITY OF ISABELA, BASILAN PROVINCE, represented by MAYOR CHERRYLYN P.
SANTOS-AKBAR,petitioners-in-intervention.
x--------------------------------------------x

THE PROVINCE OF SULTAN KUDARAT, rep. by HON. SUHARTO T. MANGUDADATU, in his


capacity as Provincial Governor and a resident of the Province of Sultan Kudarat, petitioner-inintervention.
x-------------------------------------------x
RUY ELIAS LOPEZ, for and in his own behalf and on behalf of Indigenous Peoples in
Mindanao Not Belonging to the MILF, petitioner-in-intervention.
x--------------------------------------------x
CARLO B. GOMEZ, GERARDO S. DILIG, NESARIO G. AWAT, JOSELITO C. ALISUAG and
RICHALEX G. JAGMIS, as citizens and residents of Palawan, petitioners-in-intervention.
x--------------------------------------------x
MARINO RIDAO and KISIN BUXANI, petitioners-in-intervention.
x--------------------------------------------x
MUSLIM LEGAL ASSISTANCE FOUNDATION, INC (MUSLAF), respondent-in-intervention.
x--------------------------------------------x
MUSLIM MULTI-SECTORAL MOVEMENT FOR PEACE & DEVELOPMENT
(MMMPD), respondent-in-intervention.
x--------------------------------------------x
DECISION
CARPIO MORALES, J.:
Subject of these consolidated cases is the extent of the powers of the President in pursuing the
peace process.While the facts surrounding this controversy center on the armed conflict in Mindanao
between the government and the Moro Islamic Liberation Front (MILF), the legal issue involved has
a bearing on all areas in the country where there has been a long-standing armed conflict. Yet again,
the Court is tasked to perform a delicate balancing act. It must uncompromisingly delineate the
bounds within which the President may lawfully exercise her discretion, but it must do so in strict
adherence to the Constitution, lest its ruling unduly restricts the freedom of action vested by that
same Constitution in the Chief Executive precisely to enable her to pursue the peace process
effectively.
I. FACTUAL ANTECEDENTS OF THE PETITIONS
On August 5, 2008, the Government of the Republic of the Philippines (GRP) and the MILF, through
the Chairpersons of their respective peace negotiating panels, were scheduled to sign a
Memorandum of Agreement on the Ancestral Domain (MOA-AD) Aspect of the GRP-MILF Tripoli
Agreement on Peace of 2001 in Kuala Lumpur, Malaysia.

The MILF is a rebel group which was established in March 1984 when, under the leadership of the
late Salamat Hashim, it splintered from the Moro National Liberation Front (MNLF) then headed by
Nur Misuari, on the ground, among others, of what Salamat perceived to be the manipulation of the
MNLF away from an Islamic basis towards Marxist-Maoist orientations.1
The signing of the MOA-AD between the GRP and the MILF was not to materialize, however, for
upon motion of petitioners, specifically those who filed their cases before the scheduled signing of
the MOA-AD, this Court issued a Temporary Restraining Order enjoining the GRP from signing the
same.
The MOA-AD was preceded by a long process of negotiation and the concluding of several prior
agreements between the two parties beginning in 1996, when the GRP-MILF peace negotiations
began. On July 18, 1997, the GRP and MILF Peace Panels signed the Agreement on General
Cessation of Hostilities. The following year, they signed the General Framework of Agreement of
Intent on August 27, 1998.
The Solicitor General, who represents respondents, summarizes the MOA-AD by stating that the
same contained, among others, the commitment of the parties to pursue peace negotiations, protect
and respect human rights, negotiate with sincerity in the resolution and pacific settlement of the
conflict, and refrain from the use of threat or force to attain undue advantage while the peace
negotiations on the substantive agenda are on-going.2
Early on, however, it was evident that there was not going to be any smooth sailing in the GRP-MILF
peace process. Towards the end of 1999 up to early 2000, the MILF attacked a number of
municipalities in Central Mindanao and, in March 2000, it took control of the town hall of Kauswagan,
Lanao del Norte.3 In response, then President Joseph Estrada declared and carried out an "all-outwar" against the MILF.
When President Gloria Macapagal-Arroyo assumed office, the military offensive against the MILF
was suspended and the government sought a resumption of the peace talks. The MILF, according to
a leading MILF member, initially responded with deep reservation, but when President Arroyo asked
the Government of Malaysia through Prime Minister Mahathir Mohammad to help convince the MILF
to return to the negotiating table, the MILF convened its Central Committee to seriously discuss the
matter and, eventually, decided to meet with the GRP.4
The parties met in Kuala Lumpur on March 24, 2001, with the talks being facilitated by the Malaysian
government, the parties signing on the same date the Agreement on the General Framework for the
Resumption of Peace Talks Between the GRP and the MILF. The MILF thereafter suspended all its
military actions.5
Formal peace talks between the parties were held in Tripoli, Libya from June 20-22, 2001, the
outcome of which was the GRP-MILF Tripoli Agreement on Peace (Tripoli Agreement 2001)
containing the basic principles and agenda on the following aspects of the
negotiation: Security Aspect, Rehabilitation Aspect, and Ancestral Domain Aspect. With regard to
the Ancestral Domain Aspect, the parties in Tripoli Agreement 2001 simply agreed "that the same be
discussed further by the Parties in their next meeting."
A second round of peace talks was held in Cyberjaya, Malaysia on August 5-7, 2001 which ended
with the signing of the Implementing Guidelines on the Security Aspect of the Tripoli Agreement 2001
leading to a ceasefire status between the parties. This was followed by the Implementing Guidelines
on the Humanitarian Rehabilitation and Development Aspects of the Tripoli Agreement 2001, which

was signed on May 7, 2002 at Putrajaya, Malaysia. Nonetheless, there were many incidence of
violence between government forces and the MILF from 2002 to 2003.
Meanwhile, then MILF Chairman Salamat Hashim passed away on July 13, 2003 and he was
replaced by Al Haj Murad, who was then the chief peace negotiator of the MILF. Murad's position as
chief peace negotiator was taken over by Mohagher Iqbal. 6
In 2005, several exploratory talks were held between the parties in Kuala Lumpur, eventually leading
to the crafting of the draft MOA-AD in its final form, which, as mentioned, was set to be signed last
August 5, 2008.
II. STATEMENT OF THE PROCEEDINGS
Before the Court is what is perhaps the most contentious "consensus" ever embodied in an
instrument - the MOA-AD which is assailed principally by the present petitions bearing docket
numbers 183591, 183752, 183893, 183951 and 183962.
Commonly impleaded as respondents are the GRP Peace Panel on Ancestral Domain 7 and the
Presidential Adviser on the Peace Process (PAPP) Hermogenes Esperon, Jr.
On July 23, 2008, the Province of North Cotabato8 and Vice-Governor Emmanuel Piol filed a
petition, docketed as G.R. No. 183591, for Mandamus and Prohibition with Prayer for the Issuance
of Writ of Preliminary Injunction and Temporary Restraining Order.9 Invoking the right to information
on matters of public concern, petitioners seek to compel respondents to disclose and furnish them
the complete and official copies of the MOA-AD including its attachments, and to prohibit the slated
signing of the MOA-AD, pending the disclosure of the contents of the MOA-AD and the holding of a
public consultation thereon. Supplementarily, petitioners pray that the MOA-AD be declared
unconstitutional.10
This initial petition was followed by another one, docketed as G.R. No. 183752, also for Mandamus
and Prohibition11 filed by the City of Zamboanga,12 Mayor Celso Lobregat, Rep. Ma. Isabelle Climaco
and Rep. Erico Basilio Fabian who likewise pray for similar injunctive reliefs. Petitioners herein
moreover pray that the City of Zamboanga be excluded from the Bangsamoro Homeland and/or
Bangsamoro Juridical Entity and, in the alternative, that the MOA-AD be declared null and void.
By Resolution of August 4, 2008, the Court issued a Temporary Restraining Order commanding and
directing public respondents and their agents to cease and desist from formally signing the MOAAD.13 The Court also required the Solicitor General to submit to the Court and petitioners the official
copy of the final draft of the MOA-AD,14 to which she complied.15
Meanwhile, the City of Iligan16 filed a petition for Injunction and/or Declaratory Relief, docketed
as G.R. No. 183893, praying that respondents be enjoined from signing the MOA-AD or, if the same
had already been signed, from implementing the same, and that the MOA-AD be declared
unconstitutional. Petitioners herein additionally implead Executive Secretary Eduardo Ermita as
respondent.
The Province of Zamboanga del Norte,17 Governor Rolando Yebes, Vice-Governor Francis Olvis,
Rep. Cecilia Jalosjos-Carreon, Rep. Cesar Jalosjos, and the members 18 of the Sangguniang
Panlalawigan of Zamboanga del Norte filed on August 15, 2008 a petition for Certiorari, Mandamus
and Prohibition,19 docketed as G.R. No. 183951. They pray, inter alia, that the MOA-AD be declared

null and void and without operative effect, and that respondents be enjoined from executing the
MOA-AD.
On August 19, 2008, Ernesto Maceda, Jejomar Binay, and Aquilino Pimentel III filed a petition for
Prohibition,20docketed as G.R. No. 183962, praying for a judgment prohibiting and permanently
enjoining respondents from formally signing and executing the MOA-AD and or any other agreement
derived therefrom or similar thereto, and nullifying the MOA-AD for being unconstitutional and illegal.
Petitioners herein additionally implead as respondent the MILF Peace Negotiating Panel
represented by its Chairman Mohagher Iqbal.
Various parties moved to intervene and were granted leave of court to file their petitions-/commentsin-intervention. Petitioners-in-Intervention include Senator Manuel A. Roxas, former Senate
President Franklin Drilon and Atty. Adel Tamano, the City of Isabela21 and Mayor Cherrylyn SantosAkbar, the Province of Sultan Kudarat22 and Gov. Suharto Mangudadatu, the Municipality of Linamon
in Lanao del Norte,23 Ruy Elias Lopez of Davao City and of the Bagobo tribe, Sangguniang
Panlungsod member Marino Ridao and businessman Kisin Buxani, both of Cotabato City; and
lawyers Carlo Gomez, Gerardo Dilig, Nesario Awat, Joselito Alisuag, Richalex Jagmis, all of Palawan
City. The Muslim Legal Assistance Foundation, Inc. (Muslaf) and the Muslim Multi-Sectoral
Movement for Peace and Development (MMMPD) filed their respective Comments-in-Intervention.
By subsequent Resolutions, the Court ordered the consolidation of the petitions. Respondents filed
Comments on the petitions, while some of petitioners submitted their respective Replies.
Respondents, by Manifestation and Motion of August 19, 2008, stated that the Executive
Department shall thoroughly review the MOA-AD and pursue further negotiations to address the
issues hurled against it, and thus moved to dismiss the cases. In the succeeding exchange of
pleadings, respondents' motion was met with vigorous opposition from petitioners.
The cases were heard on oral argument on August 15, 22 and 29, 2008 that tackled the following
principal issues:
1. Whether the petitions have become moot and academic
(i) insofar as the mandamus aspect is concerned, in view of the disclosure of official
copies of the final draft of the Memorandum of Agreement (MOA); and
(ii) insofar as the prohibition aspect involving the Local Government Units is
concerned, if it is considered that consultation has become fait accompli with the
finalization of the draft;
2. Whether the constitutionality and the legality of the MOA is ripe for adjudication;
3. Whether respondent Government of the Republic of the Philippines Peace Panel
committed grave abuse of discretion amounting to lack or excess of jurisdiction when it
negotiated and initiated the MOA vis--vis ISSUES Nos. 4 and 5;
4. Whether there is a violation of the people's right to information on matters of public
concern (1987 Constitution, Article III, Sec. 7) under a state policy of full disclosure of all its
transactions involving public interest (1987 Constitution, Article II, Sec. 28) including public
consultation under Republic Act No. 7160 (LOCAL GOVERNMENT CODE OF 1991)[;]

If it is in the affirmative, whether prohibition under Rule 65 of the 1997 Rules of Civil
Procedure is an appropriate remedy;
5. Whether by signing the MOA, the Government of the Republic of the Philippines would be
BINDING itself
a) to create and recognize the Bangsamoro Juridical Entity (BJE) as a separate
state, or a juridical, territorial or political subdivision not recognized by law;
b) to revise or amend the Constitution and existing laws to conform to the MOA;
c) to concede to or recognize the claim of the Moro Islamic Liberation Front for
ancestral domain in violation of Republic Act No. 8371 (THE INDIGENOUS
PEOPLES RIGHTS ACT OF 1997), particularly Section 3(g) & Chapter VII
(DELINEATION, RECOGNITION OF ANCESTRAL DOMAINS)[;]
If in the affirmative, whether the Executive Branch has the authority to so bind the
Government of the Republic of the Philippines;
6. Whether the inclusion/exclusion of the Province of North Cotabato, Cities of Zamboanga,
Iligan and Isabela, and the Municipality of Linamon, Lanao del Norte in/from the areas
covered by the projected Bangsamoro Homeland is a justiciable question; and
7. Whether desistance from signing the MOA derogates any prior valid commitments of the
Government of the Republic of the Philippines.24
The Court, thereafter, ordered the parties to submit their respective Memoranda. Most of the parties
submitted their memoranda on time.
III. OVERVIEW OF THE MOA-AD
As a necessary backdrop to the consideration of the objections raised in the subject five petitions
and six petitions-in-intervention against the MOA-AD, as well as the two comments-in-intervention in
favor of the MOA-AD, the Court takes an overview of the MOA.
The MOA-AD identifies the Parties to it as the GRP and the MILF.
Under the heading "Terms of Reference" (TOR), the MOA-AD includes not only four earlier
agreements between the GRP and MILF, but also two agreements between the GRP and the MNLF:
the 1976 Tripoli Agreement, and the Final Peace Agreement on the Implementation of the 1976
Tripoli Agreement, signed on September 2, 1996 during the administration of President Fidel Ramos.
The MOA-AD also identifies as TOR two local statutes - the organic act for the Autonomous Region
in Muslim Mindanao (ARMM)25 and the Indigenous Peoples Rights Act (IPRA),26 and several
international law instruments - the ILO Convention No. 169 Concerning Indigenous and Tribal
Peoples in Independent Countries in relation to the UN Declaration on the Rights of the Indigenous
Peoples, and the UN Charter, among others.
The MOA-AD includes as a final TOR the generic category of "compact rights entrenchment
emanating from the regime of dar-ul-mua'hada (or territory under compact) and dar-ul-sulh (or
territory under peace agreement) that partakes the nature of a treaty device."

During the height of the Muslim Empire, early Muslim jurists tended to see the world through a
simple dichotomy: there was the dar-ul-Islam (the Abode of Islam) and dar-ul-harb (the Abode
of War). The first referred to those lands where Islamic laws held sway, while the second denoted
those lands where Muslims were persecuted or where Muslim laws were outlawed or
ineffective.27 This way of viewing the world, however, became more complex through the centuries as
the Islamic world became part of the international community of nations.
As Muslim States entered into treaties with their neighbors, even with distant States and intergovernmental organizations, the classical division of the world into dar-ul-Islam and dar-ulharb eventually lost its meaning. New terms were drawn up to describe novel ways of perceiving
non-Muslim territories. For instance, areas like dar-ul-mua'hada (land of compact) and dar-ulsulh (land of treaty) referred to countries which, though under a secular regime, maintained peaceful
and cooperative relations with Muslim States, having been bound to each other by treaty or
agreement. Dar-ul-aman (land of order), on the other hand, referred to countries which, though not
bound by treaty with Muslim States, maintained freedom of religion for Muslims. 28
It thus appears that the "compact rights entrenchment" emanating from the regime of dar-ulmua'hada and dar-ul-sulh simply refers to all other agreements between the MILF and the Philippine
government - the Philippines being the land of compact and peace agreement - that partake of the
nature of a treaty device, "treaty" being broadly defined as "any solemn agreement in writing that
sets out understandings, obligations, and benefits for both parties which provides for a framework
that elaborates the principles declared in the [MOA-AD]."29
The MOA-AD states that the Parties "HAVE AGREED AND ACKNOWLEDGED AS FOLLOWS," and
starts with its main body.
The main body of the MOA-AD is divided into four strands, namely, Concepts and Principles,
Territory, Resources, and Governance.
A. CONCEPTS AND PRINCIPLES
This strand begins with the statement that it is "the birthright of all Moros and all Indigenous peoples
of Mindanao to identify themselves and be accepted as Bangsamoros.'" It defines "Bangsamoro
people" as the natives or original inhabitants of Mindanao and its adjacent islands including
Palawan and the Sulu archipelago at the time of conquest or colonization, and their
descendants whether mixed or of full blood, including their spouses.30
Thus, the concept of "Bangsamoro," as defined in this strand of the MOA-AD, includes not only
"Moros" as traditionally understood even by Muslims,31 but all indigenous peoples of Mindanao and
its adjacent islands. The MOA-AD adds that the freedom of choice of indigenous peoples shall be
respected. What this freedom of choice consists in has not been specifically defined.
The MOA-AD proceeds to refer to the "Bangsamoro homeland," the ownership of which is vested
exclusively in the Bangsamoro people by virtue of their prior rights of occupation.32 Both parties to
the MOA-AD acknowledge that ancestral domain does not form part of the public domain.33
The Bangsamoro people are acknowledged as having the right to self-governance, which right is
said to be rooted on ancestral territoriality exercised originally under the suzerain authority of their
sultanates and the Pat a Pangampong ku Ranaw. The sultanates were described as states or
"karajaan/kadatuan" resembling a body politic endowed with all the elements of a nation-state in the
modern sense.34

The MOA-AD thus grounds the right to self-governance of the Bangsamoro people on the past
suzerain authority of the sultanates. As gathered, the territory defined as the Bangsamoro homeland
was ruled by several sultanates and, specifically in the case of the Maranao, by the Pat a
Pangampong ku Ranaw, a confederation of independent principalities (pangampong) each ruled by
datus and sultans, none of whom was supreme over the others. 35
The MOA-AD goes on to describe the Bangsamoro people as "the First Nation' with defined territory
and with a system of government having entered into treaties of amity and commerce with foreign
nations."
The term "First Nation" is of Canadian origin referring to the indigenous peoples of that territory,
particularly those known as Indians. In Canada, each of these indigenous peoples is equally entitled
to be called "First Nation," hence, all of them are usually described collectively by the plural "First
Nations."36 To that extent, the MOA-AD, by identifying the Bangsamoro people as "the First Nation" suggesting its exclusive entitlement to that designation - departs from the Canadian usage of the
term.
The MOA-AD then mentions for the first time the "Bangsamoro Juridical Entity" (BJE) to which it
grants the authority and jurisdiction over the Ancestral Domain and Ancestral Lands of the
Bangsamoro.37
B. TERRITORY
The territory of the Bangsamoro homeland is described as the land mass as well as the maritime,
terrestrial, fluvial and alluvial domains, including the aerial domain and the atmospheric space above
it, embracing the Mindanao-Sulu-Palawan geographic region.38
More specifically, the core of the BJE is defined as the present geographic area of the ARMM - thus
constituting the following areas: Lanao del Sur, Maguindanao, Sulu, Tawi-Tawi, Basilan, and Marawi
City. Significantly, this core also includes certain municipalities of Lanao del Norte that voted for
inclusion in the ARMM in the 2001 plebiscite.39
Outside of this core, the BJE is to cover other provinces, cities, municipalities and barangays, which
are grouped into two categories, Category A and Category B. Each of these areas is to be subjected
to a plebiscite to be held on different dates, years apart from each other. Thus, Category A areas are
to be subjected to a plebiscite not later than twelve (12) months following the signing of the MOAAD.40 Category B areas, also called "Special Intervention Areas," on the other hand, are to be
subjected to a plebiscite twenty-five (25) years from the signing of a separate agreement - the
Comprehensive Compact.41
The Parties to the MOA-AD stipulate that the BJE shall have jurisdiction over all natural resources
within its "internal waters," defined as extending fifteen (15) kilometers from the coastline of the BJE
area;42 that the BJE shall also have "territorial waters," which shall stretch beyond the BJE internal
waters up to the baselines of the Republic of the Philippines (RP) south east and south west of
mainland Mindanao; and that within these territorialwaters, the BJE and the "Central
Government" (used interchangeably with RP) shall exercise joint jurisdiction, authority and
management over all natural resources.43 Notably, the jurisdiction over the internal waters is not
similarly described as "joint."
The MOA-AD further provides for the sharing of minerals on the territorial waters between the
Central Government and the BJE, in favor of the latter, through production sharing and economic
cooperation agreement.44 The activities which the Parties are allowed to conduct on

the territorial waters are enumerated, among which are the exploration and utilization of natural
resources, regulation of shipping and fishing activities, and the enforcement of police and safety
measures.45 There is no similar provision on the sharing of minerals and allowed activities with
respect to the internal waters of the BJE.
C. RESOURCES
The MOA-AD states that the BJE is free to enter into any economic cooperation and trade relations
with foreign countries and shall have the option to establish trade missions in those countries. Such
relationships and understandings, however, are not to include aggression against the GRP. The BJE
may also enter into environmental cooperation agreements.46
The external defense of the BJE is to remain the duty and obligation of the Central Government. The
Central Government is also bound to "take necessary steps to ensure the BJE's participation in
international meetings and events" like those of the ASEAN and the specialized agencies of the UN.
The BJE is to be entitled to participate in Philippine official missions and delegations for the
negotiation of border agreements or protocols for environmental protection and equitable sharing of
incomes and revenues involving the bodies of water adjacent to or between the islands forming part
of the ancestral domain.47
With regard to the right of exploring for, producing, and obtaining all potential sources of energy,
petroleum, fossil fuel, mineral oil and natural gas, the jurisdiction and control thereon is to be vested
in the BJE "as the party having control within its territorial jurisdiction." This right carries
the proviso that, "in times of national emergency, when public interest so requires," the Central
Government may, for a fixed period and under reasonable terms as may be agreed upon by both
Parties, assume or direct the operation of such resources.48
The sharing between the Central Government and the BJE of total production pertaining to natural
resources is to be 75:25 in favor of the BJE.49
The MOA-AD provides that legitimate grievances of the Bangsamoro people arising from any unjust
dispossession of their territorial and proprietary rights, customary land tenures, or their
marginalization shall be acknowledged. Whenever restoration is no longer possible, reparation is to
be in such form as mutually determined by the Parties.50
The BJE may modify or cancel the forest concessions, timber licenses, contracts or agreements,
mining concessions, Mineral Production and Sharing Agreements (MPSA), Industrial Forest
Management Agreements (IFMA), and other land tenure instruments granted by the Philippine
Government, including those issued by the present ARMM.51
D. GOVERNANCE
The MOA-AD binds the Parties to invite a multinational third-party to observe and monitor the
implementation of the Comprehensive Compact. This compact is to embody the "details for the
effective enforcement" and "the mechanisms and modalities for the actual implementation" of the
MOA-AD. The MOA-AD explicitly provides that the participation of the third party shall not in any way
affect the status of the relationship between the Central Government and the BJE. 52
The "associative" relationship
between the Central Government
and the BJE

The MOA-AD describes the relationship of the Central Government and the BJE as "associative,"
characterizedby shared authority and responsibility. And it states that the structure of governance is
to be based on executive, legislative, judicial, and administrative institutions with defined powers and
functions in the Comprehensive Compact.
The MOA-AD provides that its provisions requiring "amendments to the existing legal framework"
shall take effect upon signing of the Comprehensive Compact and upon effecting the aforesaid
amendments, with due regard to the non-derogation of prior agreements and within the stipulated
timeframe to be contained in the Comprehensive Compact. As will be discussed later, much of
the present controversy hangs on the legality of this provision.
The BJE is granted the power to build, develop and maintain its own institutions inclusive of civil
service, electoral, financial and banking, education, legislation, legal, economic, police and internal
security force, judicial system and correctional institutions, the details of which shall be discussed in
the negotiation of the comprehensive compact.
As stated early on, the MOA-AD was set to be signed on August 5, 2008 by Rodolfo Garcia and
Mohagher Iqbal, Chairpersons of the Peace Negotiating Panels of the GRP and the MILF,
respectively. Notably, the penultimate paragraph of the MOA-AD identifies the signatories as "the
representatives of the Parties," meaning the GRP and MILF themselves, and not merely of the
negotiating panels.53 In addition, the signature page of the MOA-AD states that it is "WITNESSED
BY" Datuk Othman Bin Abd Razak, Special Adviser to the Prime Minister of Malaysia, "ENDORSED
BY" Ambassador Sayed Elmasry, Adviser to Organization of the Islamic Conference (OIC) Secretary
General and Special Envoy for Peace Process in Southern Philippines, and SIGNED "IN THE
PRESENCE OF" Dr. Albert G. Romulo, Secretary of Foreign Affairs of RP and Dato' Seri Utama Dr.
Rais Bin Yatim, Minister of Foreign Affairs, Malaysia, all of whom were scheduled to sign the
Agreement last August 5, 2008.
Annexed to the MOA-AD are two documents containing the respective lists cum maps of the
provinces, municipalities, and barangays under Categories A and B earlier mentioned in the
discussion on the strand on TERRITORY.
IV. PROCEDURAL ISSUES
A. RIPENESS
The power of judicial review is limited to actual cases or controversies. 54 Courts decline to issue
advisory opinions or to resolve hypothetical or feigned problems, or mere academic questions. 55 The
limitation of the power of judicial review to actual cases and controversies defines the role assigned
to the judiciary in a tripartite allocation of power, to assure that the courts will not intrude into areas
committed to the other branches of government.56
An actual case or controversy involves a conflict of legal rights, an assertion of opposite legal claims,
susceptible of judicial resolution as distinguished from a hypothetical or abstract difference or
dispute. There must be a contrariety of legal rights that can be interpreted and enforced on the basis
of existing law and jurisprudence.57The Court can decide the constitutionality of an act or treaty only
when a proper case between opposing parties is submitted for judicial determination. 58
Related to the requirement of an actual case or controversy is the requirement of ripeness. A
question is ripe for adjudication when the act being challenged has had a direct adverse effect on
the individual challenging it.59 For a case to be considered ripe for adjudication, it is a prerequisite
that something had then been accomplished or performed by either branch before a court may come

into the picture,60 and the petitioner must allege the existence of an immediate or threatened injury to
itself as a result of the challenged action.61 He must show that he has sustained or is immediately in
danger of sustaining some direct injury as a result of the act complained of. 62
The Solicitor General argues that there is no justiciable controversy that is ripe for judicial review in
the present petitions, reasoning that
The unsigned MOA-AD is simply a list of consensus points subject to further negotiations
and legislative enactments as well as constitutional processes aimed at attaining a final
peaceful agreement. Simply put, the MOA-AD remains to be a proposal that does not
automatically create legally demandable rights and obligations until the list of operative acts
required have been duly complied with. x x x
xxxx
In the cases at bar, it is respectfully submitted that this Honorable Court has no authority to
pass upon issues based on hypothetical or feigned constitutional problems or interests
with no concrete bases. Considering the preliminary character of the MOA-AD, there are no
concrete acts that could possibly violate petitioners' and intervenors' rights since the acts
complained of are mere contemplated steps toward the formulation of a final peace
agreement. Plainly, petitioners and intervenors' perceived injury, if at all, is merely imaginary
and illusory apart from being unfounded and based on mere conjectures. (Underscoring
supplied)
The Solicitor General cites63 the following provisions of the MOA-AD:
TERRITORY
xxxx
2. Toward this end, the Parties enter into the following stipulations:
xxxx
d. Without derogating from the requirements of prior agreements, the Government stipulates
to conduct and deliver, using all possible legal measures, within twelve (12) months following
the signing of the MOA-AD, a plebiscite covering the areas as enumerated in the list and
depicted in the map as Category A attached herein (the "Annex"). The Annex constitutes an
integral part of this framework agreement. Toward this end, the Parties shall endeavor to
complete the negotiations and resolve all outstanding issues on the Comprehensive
Compact within fifteen (15) months from the signing of the MOA-AD.
xxxx
GOVERNANCE
xxxx
7. The Parties agree that mechanisms and modalities for the actual implementation of this
MOA-AD shall be spelt out in the Comprehensive Compact to mutually take such steps to
enable it to occur effectively.

Any provisions of the MOA-AD requiring amendments to the existing legal framework shall
come into forceupon the signing of a Comprehensive Compact and upon effecting the
necessary changes to the legal framework with due regard to non-derogation of prior
agreements and within the stipulated timeframe to be contained in the Comprehensive
Compact.64 (Underscoring supplied)
The Solicitor General's arguments fail to persuade.
Concrete acts under the MOA-AD are not necessary to render the present controversy ripe.
In Pimentel, Jr. v. Aguirre,65 this Court held:
x x x [B]y the mere enactment of the questioned law or the approval of the challenged action,
the dispute is said to have ripened into a judicial controversy even without any other overt
act. Indeed, even a singular violation of the Constitution and/or the law is enough to awaken
judicial duty.
xxxx
By the same token, when an act of the President, who in our constitutional scheme is a
coequal of Congress, is seriously alleged to have infringed the Constitution and the laws x x
x settling the dispute becomes the duty and the responsibility of the courts. 66
In Santa Fe Independent School District v. Doe,67 the United States Supreme Court held that the
challenge to the constitutionality of the school's policy allowing student-led prayers and speeches
before games was ripe for adjudication, even if no public prayer had yet been led under the policy,
because the policy was being challenged as unconstitutional on its face. 68
That the law or act in question is not yet effective does not negate ripeness. For example, in New
York v. United States,69 decided in 1992, the United States Supreme Court held that the action by the
State of New York challenging the provisions of the Low-Level Radioactive Waste Policy Act was ripe
for adjudication even if the questioned provision was not to take effect until January 1, 1996,
because the parties agreed that New York had to take immediate action to avoid the provision's
consequences.70
The present petitions pray for Certiorari,71 Prohibition, and Mandamus. Certiorari and Prohibition are
remedies granted by law when any tribunal, board or officer has acted, in the case of certiorari, or is
proceeding, in the case of prohibition, without or in excess of its jurisdiction or with grave abuse of
discretion amounting to lack or excess of jurisdiction. 72 Mandamus is a remedy granted by law when
any tribunal, corporation, board, officer or person unlawfully neglects the performance of an act
which the law specifically enjoins as a duty resulting from an office, trust, or station, or unlawfully
excludes another from the use or enjoyment of a right or office to which such other is
entitled.73 Certiorari, Mandamus and Prohibition are appropriate remedies to raise constitutional
issues and to review and/or prohibit/nullify, when proper, acts of legislative and executive officials. 74
The authority of the GRP Negotiating Panel is defined by Executive Order No. 3 (E.O. No. 3), issued
on February 28, 2001.75 The said executive order requires that "[t]he government's policy framework
for peace, including the systematic approach and the administrative structure for carrying out the
comprehensive peace process x x x be governed by this Executive Order." 76
The present petitions allege that respondents GRP Panel and PAPP Esperon drafted the terms of
the MOA-AD without consulting the local government units or communities affected, nor informing

them of the proceedings. As will be discussed in greater detail later, such omission, by itself,
constitutes a departure by respondents from their mandate under E.O. No. 3.
Furthermore, the petitions allege that the provisions of the MOA-AD violate the Constitution. The
MOA-AD provides that "any provisions of the MOA-AD requiring amendments to the existing legal
framework shall come into force upon the signing of a Comprehensive Compact and upon effecting
the necessary changes to the legal framework," implying an amendment of the Constitution to
accommodate the MOA-AD. This stipulation, in effect,guaranteed to the MILF the amendment of the
Constitution. Such act constitutes another violation of its authority. Again, these points will be
discussed in more detail later.
As the petitions allege acts or omissions on the part of respondent that exceed their authority, by
violating their duties under E.O. No. 3 and the provisions of the Constitution and statutes, the
petitions make a prima facie case for Certiorari, Prohibition, and Mandamus, and an actual case or
controversy ripe for adjudication exists. When an act of a branch of government is seriously
alleged to have infringed the Constitution, it becomes not only the right but in fact the duty of
the judiciary to settle the dispute.77
B. LOCUS STANDI
For a party to have locus standi, one must allege "such a personal stake in the outcome of the
controversy as to assure that concrete adverseness which sharpens the presentation of issues upon
which the court so largely depends for illumination of difficult constitutional questions." 78
Because constitutional cases are often public actions in which the relief sought is likely to affect
other persons, a preliminary question frequently arises as to this interest in the constitutional
question raised.79
When suing as a citizen, the person complaining must allege that he has been or is about to be
denied some right or privilege to which he is lawfully entitled or that he is about to be subjected to
some burdens or penalties by reason of the statute or act complained of. 80 When the issue concerns
a public right, it is sufficient that the petitioner is a citizen and has an interest in the execution of the
laws.81
For a taxpayer, one is allowed to sue where there is an assertion that public funds are illegally
disbursed or deflected to an illegal purpose, or that there is a wastage of public funds through the
enforcement of an invalid or unconstitutional law.82 The Court retains discretion whether or not to
allow a taxpayer's suit.83
In the case of a legislator or member of Congress, an act of the Executive that injures the institution
of Congress causes a derivative but nonetheless substantial injury that can be questioned by
legislators. A member of the House of Representatives has standing to maintain inviolate the
prerogatives, powers and privileges vested by the Constitution in his office. 84
An organization may be granted standing to assert the rights of its members,85 but the mere
invocation by theIntegrated Bar of the Philippines or any member of the legal profession of the duty
to preserve the rule of law does not suffice to clothe it with standing. 86
As regards a local government unit (LGU), it can seek relief in order to protect or vindicate an
interest of its own, and of the other LGUs.87

Intervenors, meanwhile, may be given legal standing upon showing of facts that satisfy the
requirements of the law authorizing intervention,88 such as a legal interest in the matter in litigation,
or in the success of either of the parties.
In any case, the Court has discretion to relax the procedural technicality on locus standi, given the
liberal attitude it has exercised, highlighted in the case of David v. Macapagal-Arroyo,89 where
technicalities of procedure were brushed aside, the constitutional issues raised being of paramount
public interest or of transcendental importance deserving the attention of the Court in view of their
seriousness, novelty and weight as precedents.90 The Court's forbearing stance on locus standi on
issues involving constitutional issues has for its purpose the protection of fundamental rights.
In not a few cases, the Court, in keeping with its duty under the Constitution to determine whether
the other branches of government have kept themselves within the limits of the Constitution and the
laws and have not abused the discretion given them, has brushed aside technical rules of
procedure.91
In the petitions at bar, petitioners Province of North Cotabato (G.R. No. 183591) Province of
Zamboanga del Norte (G.R. No. 183951), City of Iligan (G.R. No. 183893) and City of
Zamboanga (G.R. No. 183752) and petitioners-in-intervention Province of Sultan Kudarat, City of
Isabela and Municipality of Linamon havelocus standi in view of the direct and substantial injury
that they, as LGUs, would suffer as their territories, whether in whole or in part, are to be included in
the intended domain of the BJE. These petitioners allege that they did not vote for their inclusion in
the ARMM which would be expanded to form the BJE territory. Petitioners' legal standing is thus
beyond doubt.
In G.R. No. 183962, petitioners Ernesto Maceda, Jejomar Binay and Aquilino Pimentel III would
have no standing as citizens and taxpayers for their failure to specify that they would be denied
some right or privilege or there would be wastage of public funds. The fact that they are a former
Senator, an incumbent mayor of Makati City, and a resident of Cagayan de Oro, respectively, is of no
consequence. Considering their invocation of the transcendental importance of the issues at hand,
however, the Court grants them standing.
Intervenors Franklin Drilon and Adel Tamano, in alleging their standing as taxpayers, assert that
government funds would be expended for the conduct of an illegal and unconstitutional plebiscite to
delineate the BJE territory. On that score alone, they can be given legal standing. Their allegation
that the issues involved in these petitions are of "undeniable transcendental importance" clothes
them with added basis for their personality to intervene in these petitions.
With regard to Senator Manuel Roxas, his standing is premised on his being a member of the
Senate and a citizen to enforce compliance by respondents of the public's constitutional right to be
informed of the MOA-AD, as well as on a genuine legal interest in the matter in litigation, or in the
success or failure of either of the parties. He thus possesses the requisite standing as an intervenor.
With respect to Intervenors Ruy Elias Lopez, as a former congressman of the 3rd district of Davao
City, a taxpayer and a member of the Bagobo tribe; Carlo B. Gomez, et al., as members of the IBP
Palawan chapter, citizens and taxpayers; Marino Ridao, as taxpayer, resident and member of
the Sangguniang Panlungsod of Cotabato City; and Kisin Buxani, as taxpayer, they failed to allege
any proper legal interest in the present petitions. Just the same, the Court exercises its discretion to
relax the procedural technicality on locus standigiven the paramount public interest in the issues at
hand.

Intervening respondents Muslim Multi-Sectoral Movement for Peace and Development, an


advocacy group for justice and the attainment of peace and prosperity in Muslim Mindanao;
and Muslim Legal Assistance Foundation Inc., a non-government organization of Muslim lawyers,
allege that they stand to be benefited or prejudiced, as the case may be, in the resolution of the
petitions concerning the MOA-AD, and prays for the denial of the petitions on the grounds therein
stated. Such legal interest suffices to clothe them with standing.
B. MOOTNESS
Respondents insist that the present petitions have been rendered moot with the satisfaction of all the
reliefs prayed for by petitioners and the subsequent pronouncement of the Executive Secretary that
"[n]o matter what the Supreme Court ultimately decides[,] the government will not sign the MOA." 92
In lending credence to this policy decision, the Solicitor General points out that the President had
already disbanded the GRP Peace Panel.93
In David v. Macapagal-Arroyo,94 this Court held that the "moot and academic" principle not being a
magical formula that automatically dissuades courts in resolving a case, it will decide cases,
otherwise moot and academic, if it finds that (a) there is a grave violation of the Constitution; 95 (b) the
situation is of exceptional character and paramount public interest is involved; 96 (c) the constitutional
issue raised requires formulation of controlling principles to guide the bench, the bar, and the
public;97 and (d) the case is capable of repetition yet evading review.98
Another exclusionary circumstance that may be considered is where there is a voluntary cessation of
the activity complained of by the defendant or doer. Thus, once a suit is filed and the doer voluntarily
ceases the challenged conduct, it does not automatically deprive the tribunal of power to hear and
determine the case and does not render the case moot especially when the plaintiff seeks damages
or prays for injunctive relief against the possible recurrence of the violation. 99
The present petitions fall squarely into these exceptions to thus thrust them into the domain of
judicial review. The grounds cited above in David are just as applicable in the present cases as they
were, not only in David, but also in Province of Batangas v. Romulo100 and Manalo v.
Calderon101 where the Court similarly decided them on the merits, supervening events that would
ordinarily have rendered the same moot notwithstanding.
Petitions not mooted
Contrary then to the asseverations of respondents, the non-signing of the MOA-AD and the eventual
dissolution of the GRP Peace Panel did not moot the present petitions. It bears emphasis that the
signing of the MOA-AD did not push through due to the Court's issuance of a Temporary Restraining
Order.
Contrary too to respondents' position, the MOA-AD cannot be considered a mere "list of consensus
points," especially given its nomenclature, the need to have it signed or initialed by all the parties
concerned on August 5, 2008, and the far-reaching Constitutional implications of these
"consensus points," foremost of which is the creation of the BJE.
In fact, as what will, in the main, be discussed, there is a commitment on the part of respondents
to amend and effect necessary changes to the existing legal framework for certain provisions
of the MOA-AD to take effect. Consequently, the present petitions are not confined to the terms
and provisions of the MOA-AD, but to other on-going and future negotiations and agreements

necessary for its realization. The petitions have not, therefore, been rendered moot and academic
simply by the public disclosure of the MOA-AD,102 the manifestation that it will not be signed as well
as the disbanding of the GRP Panel not withstanding.
Petitions are imbued with paramount public interest
There is no gainsaying that the petitions are imbued with paramount public interest, involving a
significant part of the country's territory and the wide-ranging political modifications of affected LGUs.
The assertion that the MOA-AD is subject to further legal enactments including possible
Constitutional amendments more than ever provides impetus for the Court to formulate
controlling principles to guide the bench, the bar, the public and, in this case, the government
and its negotiating entity.
Respondents cite Suplico v. NEDA, et al.103 where the Court did not "pontificat[e] on issues which no
longer legitimately constitute an actual case or controversy [as this] will do more harm than good to
the nation as a whole."
The present petitions must be differentiated from Suplico. Primarily, in Suplico, what was assailed
and eventually cancelled was a stand-alone government procurement contract for a national
broadband network involving a one-time contractual relation between two parties-the government
and a private foreign corporation. As the issues therein involved specific government procurement
policies and standard principles on contracts, the majority opinion in Suplico found nothing
exceptional therein, the factual circumstances being peculiar only to the transactions and parties
involved in the controversy.
The MOA-AD is part of a series of agreements
In the present controversy, the MOA-AD is a significant part of a series of agreements necessary
to carry out the Tripoli Agreement 2001. The MOA-AD which dwells on the Ancestral Domain
Aspect of said Tripoli Agreement is the third such component to be undertaken following the
implementation of the Security Aspect in August 2001 and the Humanitarian, Rehabilitation and
Development Aspect in May 2002.
Accordingly, even if the Executive Secretary, in his Memorandum of August 28, 2008 to the Solicitor
General, has stated that "no matter what the Supreme Court ultimately decides[,] the government
will not sign the MOA[-AD],"mootness will not set in in light of the terms of the Tripoli Agreement
2001.
Need to formulate principles-guidelines
Surely, the present MOA-AD can be renegotiated or another one will be drawn up to carry out the
Ancestral Domain Aspect of the Tripoli Agreement 2001, in another or in any form, which could
contain similar or significantly drastic provisions. While the Court notes the word of the Executive
Secretary that the government "is committed to securing an agreement that is both constitutional and
equitable because that is the only way that long-lasting peace can be assured," it is minded to
render a decision on the merits in the present petitions toformulate controlling principles to guide
the bench, the bar, the public and, most especially, the government in negotiating with the
MILF regarding Ancestral Domain.
Respondents invite the Court's attention to the separate opinion of then Chief Justice Artemio
Panganiban inSanlakas v. Reyes104 in which he stated that the doctrine of "capable of repetition yet

evading review" can override mootness, "provided the party raising it in a proper case has been
and/or continue to be prejudiced or damaged as a direct result of their issuance." They contend that
the Court must have jurisdiction over the subject matter for the doctrine to be invoked.
The present petitions all contain prayers for Prohibition over which this Court exercises original
jurisdiction. While G.R. No. 183893 (City of Iligan v. GRP) is a petition for Injunction and Declaratory
Relief, the Court will treat it as one for Prohibition as it has far reaching implications and raises
questions that need to be resolved.105 At all events, the Court has jurisdiction over most if not the rest
of the petitions.
Indeed, the present petitions afford a proper venue for the Court to again apply the doctrine
immediately referred to as what it had done in a number of landmark cases. 106 There is
a reasonable expectation that petitioners, particularly the Provinces of North Cotabato, Zamboanga
del Norte and Sultan Kudarat, the Cities of Zamboanga, Iligan and Isabela, and the Municipality of
Linamon, will again be subjected to the same problem in the future as respondents' actions are
capable of repetition, in another or any form.
It is with respect to the prayers for Mandamus that the petitions have become moot, respondents
having, by Compliance of August 7, 2008, provided this Court and petitioners with official copies of
the final draft of the MOA-AD and its annexes. Too, intervenors have been furnished, or have
procured for themselves, copies of the MOA-AD.
V. SUBSTANTIVE ISSUES
As culled from the Petitions and Petitions-in-Intervention, there are basically two SUBSTANTIVE
issues to be resolved, one relating to the manner in which the MOA-AD was negotiated and
finalized, the other relating to its provisions, viz:
1. Did respondents violate constitutional and statutory provisions on public consultation and the right
to information when they negotiated and later initialed the MOA-AD?
2. Do the contents of the MOA-AD violate the Constitution and the laws?
ON THE FIRST SUBSTANTIVE ISSUE
Petitioners invoke their constitutional right to information on matters of public concern, as
provided in Section 7, Article III on the Bill of Rights:
Sec. 7. The right of the people to information on matters of public concern shall be
recognized. Access to official records, and to documents, and papers pertaining to official
acts, transactions, or decisions, as well as to government research data used as basis for
policy development, shall be afforded the citizen, subject to such limitations as may be
provided by law.107
As early as 1948, in Subido v. Ozaeta,108 the Court has recognized the statutory right to examine and
inspect public records, a right which was eventually accorded constitutional status.
The right of access to public documents, as enshrined in both the 1973 Constitution and the 1987
Constitution, has been recognized as a self-executory constitutional right. 109

In the 1976 case of Baldoza v. Hon. Judge Dimaano,110 the Court ruled that access to public records
is predicated on the right of the people to acquire information on matters of public concern since,
undoubtedly, in a democracy, the pubic has a legitimate interest in matters of social and political
significance.
x x x The incorporation of this right in the Constitution is a recognition of the fundamental role of free
exchange of information in a democracy. There can be no realistic perception by the public of the
nation's problems, nor a meaningful democratic decision-making if they are denied access to
information of general interest. Information is needed to enable the members of society to cope with
the exigencies of the times. As has been aptly observed: "Maintaining the flow of such information
depends on protection for both its acquisition and its dissemination since, if either process is
interrupted, the flow inevitably ceases." x x x111
In the same way that free discussion enables members of society to cope with the exigencies of
their time, access to information of general interest aids the people in democratic decision-making
by giving them a better perspective of the vital issues confronting the nation112 so that they may be
able to criticize and participate in the affairs of the government in a responsible, reasonable and
effective manner. It is by ensuring an unfettered and uninhibited exchange of ideas among a wellinformed public that a government remains responsive to the changes desired by the people. 113
The MOA-AD is a matter of public concern
That the subject of the information sought in the present cases is a matter of public concern 114 faces
no serious challenge. In fact, respondents admit that the MOA-AD is indeed of public concern.115 In
previous cases, the Court found that the regularity of real estate transactions entered in the Register
of Deeds,116 the need for adequate notice to the public of the various laws, 117 the civil service eligibility
of a public employee,118 the proper management of GSIS funds allegedly used to grant loans to
public officials,119 the recovery of the Marcoses' alleged ill-gotten wealth, 120 and the identity of partylist nominees,121 among others, are matters of public concern. Undoubtedly, the MOA-AD subject of
the present cases is of public concern, involving as it does the sovereignty and territorial
integrity of the State, which directly affects the lives of the public at large.
Matters of public concern covered by the right to information include steps and negotiations leading
to the consummation of the contract. In not distinguishing as to the executory nature or commercial
character of agreements, the Court has categorically ruled:
x x x [T]he right to information "contemplates inclusion of negotiations leading to the
consummation of the transaction." Certainly, a consummated contract is not a
requirement for the exercise of the right to information. Otherwise, the people can never
exercise the right if no contract is consummated, and if one is consummated, it may be too
late for the public to expose its defects.
Requiring a consummated contract will keep the public in the dark until the contract, which
may be grossly disadvantageous to the government or even illegal, becomes fait accompli.
This negates the State policy of full transparency on matters of public concern, a situation
which the framers of the Constitution could not have intended. Such a requirement will
prevent the citizenry from participating in the public discussion of any proposed contract,
effectively truncating a basic right enshrined in the Bill of Rights. We can allow neither an
emasculation of a constitutional right, nor a retreat by the State of its avowed "policy of full
disclosure of all its transactions involving public interest."122 (Emphasis and italics in the
original)

Intended as a "splendid symmetry"123 to the right to information under the Bill of Rights is the policy of
public disclosure under Section 28, Article II of the Constitution reading:
Sec. 28. Subject to reasonable conditions prescribed by law, the State adopts and
implements a policy of full public disclosure of all its transactions involving public interest. 124
The policy of full public disclosure enunciated in above-quoted Section 28 complements the right of
access to information on matters of public concern found in the Bill of Rights. The right to information
guarantees the right of the people to demand information, while Section 28 recognizes the duty of
officialdom to give information even if nobody demands.125
The policy of public disclosure establishes a concrete ethical principle for the conduct of public
affairs in a genuinely open democracy, with the people's right to know as the centerpiece. It is a
mandate of the State to be accountable by following such policy.126 These provisions are vital to the
exercise of the freedom of expression and essential to hold public officials at all times accountable to
the people.127
Whether Section 28 is self-executory, the records of the deliberations of the Constitutional
Commission so disclose:
MR. SUAREZ. And since this is not self-executory, this policy will not be enunciated or will
not be in force and effect until after Congress shall have provided it.
MR. OPLE. I expect it to influence the climate of public ethics immediately but, of course, the
implementing law will have to be enacted by Congress, Mr. Presiding Officer.128
The following discourse, after Commissioner Hilario Davide, Jr., sought clarification on the issue, is
enlightening.
MR. DAVIDE. I would like to get some clarifications on this. Mr. Presiding Officer, did I get the
Gentleman correctly as having said that this is not a self-executing provision? It would
require a legislation by Congress to implement?
MR. OPLE. Yes. Originally, it was going to be self-executing, but I accepted an amendment
from Commissioner Regalado, so that the safeguards on national interest are modified by
the clause "as may be provided by law"
MR. DAVIDE. But as worded, does it not mean that this will immediately take effect and
Congress may provide for reasonable safeguards on the sole ground national interest?
MR. OPLE. Yes. I think so, Mr. Presiding Officer, I said earlier that it should
immediately influence the climate of the conduct of public affairs but, of course,
Congress here may no longer pass a law revoking it, or if this is approved, revoking this
principle, which is inconsistent with this policy.129 (Emphasis supplied)
Indubitably, the effectivity of the policy of public disclosure need not await the passing of a
statute. As Congress cannot revoke this principle, it is merely directed to provide for "reasonable
safeguards." The complete and effective exercise of the right to information necessitates that its
complementary provision on public disclosure derive the same self-executory nature. Since both
provisions go hand-in-hand, it is absurd to say that the broader 130 right to information on matters of
public concern is already enforceable while the correlative duty of the State to disclose its

transactions involving public interest is not enforceable until there is an enabling law.Respondents
cannot thus point to the absence of an implementing legislation as an excuse in not effecting such
policy.
An essential element of these freedoms is to keep open a continuing dialogue or process of
communication between the government and the people. It is in the interest of the State that the
channels for free political discussion be maintained to the end that the government may perceive
and be responsive to the people's will.131Envisioned to be corollary to the twin rights to information
and disclosure is the design for feedback mechanisms.
MS. ROSARIO BRAID. Yes. And lastly, Mr. Presiding Officer, will the people be able to
participate? Will the government provide feedback mechanisms so that the people
can participate and can react where the existing media facilities are not able to
provide full feedback mechanisms to the government? I suppose this will be part of
the government implementing operational mechanisms.
MR. OPLE. Yes. I think through their elected representatives and that is how these courses
take place. There is a message and a feedback, both ways.
xxxx
MS. ROSARIO BRAID. Mr. Presiding Officer, may I just make one last sentence?
I think when we talk about the feedback network, we are not talking about public
officials but also network of private business o[r] community-based organizations that
will be reacting. As a matter of fact, we will put more credence or credibility on the private
network of volunteers and voluntary community-based organizations. So I do not think we
are afraid that there will be another OMA in the making. 132(Emphasis supplied)
The imperative of a public consultation, as a species of the right to information, is evident in the
"marching orders" to respondents. The mechanics for the duty to disclose information and to conduct
public consultation regarding the peace agenda and process is manifestly provided by E.O. No.
3.133 The preambulatory clause of E.O. No. 3 declares that there is a need to further enhance the
contribution of civil society to the comprehensive peace process by institutionalizing the people's
participation.
One of the three underlying principles of the comprehensive peace process is that it "should be
community-based, reflecting the sentiments, values and principles important to all Filipinos" and
"shall be defined not by the government alone, nor by the different contending groups only, but by all
Filipinos as one community."134Included as a component of the comprehensive peace process is
consensus-building and empowerment for peace, which includes "continuing consultations on both
national and local levels to build consensus for a peace agenda and process, and the mobilization
and facilitation of people's participation in the peace process."135
Clearly, E.O. No. 3 contemplates not just the conduct of a plebiscite to effectuate
"continuing" consultations, contrary to respondents' position that plebiscite is "more than
sufficient consultation."136
Further, E.O. No. 3 enumerates the functions and responsibilities of the PAPP, one of which is to
"[c]onductregular dialogues with the National Peace Forum (NPF) and other peace partners to seek
relevant information, comments, recommendations as well as to render appropriate and timely

reports on the progress of the comprehensive peace process."137 E.O. No. 3 mandates the
establishment of the NPF to be "the principal forumfor the PAPP to consult with and seek advi[c]e
from the peace advocates, peace partners and concerned sectors of society on both national and
local levels, on the implementation of the comprehensive peace process, as well as for
government[-]civil society dialogue and consensus-building on peace agenda and initiatives." 138
In fine, E.O. No. 3 establishes petitioners' right to be consulted on the peace agenda, as a
corollary to the constitutional right to information and disclosure.
PAPP Esperon committed grave abuse of discretion
The PAPP committed grave abuse of discretion when he failed to carry out the pertinent
consultation. The furtive process by which the MOA-AD was designed and crafted runs contrary to
and in excess of the legal authority, and amounts to a whimsical, capricious, oppressive, arbitrary
and despotic exercise thereof.
The Court may not, of course, require the PAPP to conduct the consultation in a particular way or
manner. It may, however, require him to comply with the law and discharge the functions within the
authority granted by the President.139
Petitioners are not claiming a seat at the negotiating table, contrary to respondents' retort in justifying
the denial of petitioners' right to be consulted. Respondents' stance manifests the manner by which
they treat the salient provisions of E.O. No. 3 on people's participation. Such disregard of the
express mandate of the President is not much different from superficial conduct toward token
provisos that border on classic lip service.140 It illustrates a gross evasion of positive duty and a
virtual refusal to perform the duty enjoined.
As for respondents' invocation of the doctrine of executive privilege, it is not tenable under the
premises. The argument defies sound reason when contrasted with E.O. No. 3's explicit provisions
on continuing consultation and dialogue on both national and local levels. The executive order
even recognizes the exercise of the public's right even before the GRP makes its official
recommendations or before the government proffers its definite propositions.141 It bear emphasis that
E.O. No. 3 seeks to elicit relevant advice, information, comments and recommendations from the
people through dialogue.
AT ALL EVENTS, respondents effectively waived the defense of executive privilege in view of their
unqualified disclosure of the official copies of the final draft of the MOA-AD. By unconditionally
complying with the Court's August 4, 2008 Resolution, without a prayer for the document's
disclosure in camera, or without a manifestation that it was complying therewith ex abundante ad
cautelam.
Petitioners' assertion that the Local Government Code (LGC) of 1991 declares it a State policy to
"require all national agencies and offices to conduct periodic consultations with appropriate local
government units, non-governmental and people's organizations, and other concerned sectors of the
community before any project or program is implemented in their respective jurisdictions" 142 is welltaken. The LGC chapter on intergovernmental relations puts flesh into this avowed policy:
Prior Consultations Required. - No project or program shall be implemented by government
authoritiesunless the consultations mentioned in Sections 2 (c) and 26 hereof are complied
with, and prior approval of the sanggunian concerned is obtained: Provided, That occupants
in areas where such projects are to be implemented shall not be evicted unless appropriate

relocation sites have been provided, in accordance with the provisions of the
Constitution.143 (Italics and underscoring supplied)
In Lina, Jr. v. Hon. Pao,144 the Court held that the above-stated policy and above-quoted provision of
the LGU apply only to national programs or projects which are to be implemented in a particular local
community. Among the programs and projects covered are those that are critical to the environment
and human ecology including those that may call for the eviction of a particular group of people
residing in the locality where these will be implemented.145 The MOA-AD is one peculiar program
that unequivocally and unilaterally vests ownership of a vast territory to the Bangsamoro
people,146 which could pervasively and drastically result to the diaspora or displacement of a
great number of inhabitants from their total environment.
With respect to the indigenous cultural communities/indigenous peoples (ICCs/IPs), whose interests
are represented herein by petitioner Lopez and are adversely affected by the MOA-AD, the ICCs/IPs
have, under the IPRA, the right to participate fully at all levels of decision-making in matters which
may affect their rights, lives and destinies.147 The MOA-AD, an instrument recognizing ancestral
domain, failed to justify its non-compliance with the clear-cut mechanisms ordained in said
Act,148 which entails, among other things, the observance of the free and prior informed consent of
the ICCs/IPs.
Notably, the IPRA does not grant the Executive Department or any government agency the power to
delineate and recognize an ancestral domain claim by mere agreement or compromise. The
recognition of the ancestral domain is the raison d'etre of the MOA-AD, without which all other
stipulations or "consensus points" necessarily must fail. In proceeding to make a sweeping
declaration on ancestral domain, without complying with the IPRA, which is cited as one of the TOR
of the MOA-AD, respondents clearly transcended the boundaries of their authority. As it
seems, even the heart of the MOA-AD is still subject to necessary changes to the legal framework.
While paragraph 7 on Governance suspends the effectivity of all provisions requiring changes to the
legal framework, such clause is itself invalid, as will be discussed in the following section.
Indeed, ours is an open society, with all the acts of the government subject to public scrutiny and
available always to public cognizance. This has to be so if the country is to remain democratic, with
sovereignty residing in the people and all government authority emanating from them. 149
ON THE SECOND SUBSTANTIVE ISSUE
With regard to the provisions of the MOA-AD, there can be no question that they cannot all be
accommodated under the present Constitution and laws. Respondents have admitted as much in the
oral arguments before this Court, and the MOA-AD itself recognizes the need to amend the existing
legal framework to render effective at least some of its provisions. Respondents, nonetheless,
counter that the MOA-AD is free of any legal infirmity because any provisions therein which are
inconsistent with the present legal framework will not be effective until the necessary changes to that
framework are made. The validity of this argument will be considered later. For now, the Court shall
pass upon how
The MOA-AD is inconsistent with the Constitution and laws as presently worded.
In general, the objections against the MOA-AD center on the extent of the powers conceded therein
to the BJE. Petitioners assert that the powers granted to the BJE exceed those granted to any local
government under present laws, and even go beyond those of the present ARMM. Before assessing
some of the specific powers that would have been vested in the BJE, however, it would be useful to
turn first to a general idea that serves as a unifying link to the different provisions of the MOA-AD,

namely, the international law concept of association. Significantly, the MOA-AD explicitly alludes to
this concept, indicating that the Parties actually framed its provisions with it in mind.
Association is referred to in paragraph 3 on TERRITORY, paragraph 11 on RESOURCES, and
paragraph 4 on GOVERNANCE. It is in the last mentioned provision, however, that the MOA-AD
most clearly uses it to describe the envisioned relationship between the BJE and the Central
Government.
4. The relationship between the Central Government and the Bangsamoro juridical
entity shall beassociative characterized by shared authority and responsibility with a
structure of governance based on executive, legislative, judicial and administrative
institutions with defined powers and functions in the comprehensive compact. A period of
transition shall be established in a comprehensive peace compact specifying the relationship
between the Central Government and the BJE. (Emphasis and underscoring supplied)
The nature of the "associative" relationship may have been intended to be defined more precisely in
the still to be forged Comprehensive Compact. Nonetheless, given that there is a concept of
"association" in international law, and the MOA-AD - by its inclusion of international law instruments
in its TOR- placed itself in an international legal context, that concept of association may be brought
to bear in understanding the use of the term "associative" in the MOA-AD.
Keitner and Reisman state that
[a]n association is formed when two states of unequal power voluntarily establish durable
links. In the basic model, one state, the associate, delegates certain responsibilities to
the other, the principal, while maintaining its international status as a state. Free
associations represent a middle ground between integration and independence. x x
x150 (Emphasis and underscoring supplied)
For purposes of illustration, the Republic of the Marshall Islands and the Federated States of
Micronesia (FSM), formerly part of the U.S.-administered Trust Territory of the Pacific Islands, 151 are
associated states of the U.S. pursuant to a Compact of Free Association. The currency in these
countries is the U.S. dollar, indicating their very close ties with the U.S., yet they issue their own
travel documents, which is a mark of their statehood. Their international legal status as states was
confirmed by the UN Security Council and by their admission to UN membership.
According to their compacts of free association, the Marshall Islands and the FSM generally have
the capacity to conduct foreign affairs in their own name and right, such capacity extending to
matters such as the law of the sea, marine resources, trade, banking, postal, civil aviation, and
cultural relations. The U.S. government, when conducting its foreign affairs, is obligated to consult
with the governments of the Marshall Islands or the FSM on matters which it (U.S. government)
regards as relating to or affecting either government.
In the event of attacks or threats against the Marshall Islands or the FSM, the U.S. government has
the authority and obligation to defend them as if they were part of U.S. territory. The U.S.
government, moreover, has the option of establishing and using military areas and facilities within
these associated states and has the right to bar the military personnel of any third country from
having access to these territories for military purposes.
It bears noting that in U.S. constitutional and international practice, free association is understood as
an international association between sovereigns. The Compact of Free Association is a treaty which
is subordinate to the associated nation's national constitution, and each party may terminate the

association consistent with the right of independence. It has been said that, with the admission of the
U.S.-associated states to the UN in 1990, the UN recognized that the American model of free
association is actually based on an underlying status of independence. 152
In international practice, the "associated state" arrangement has usually been used as a transitional
device of former colonies on their way to full independence. Examples of states that have passed
through the status of associated states as a transitional phase are Antigua, St. Kitts-Nevis-Anguilla,
Dominica, St. Lucia, St. Vincent and Grenada. All have since become independent states. 153
Back to the MOA-AD, it contains many provisions which are consistent with the international legal
concept ofassociation, specifically the following: the BJE's capacity to enter into economic and trade
relations with foreign countries, the commitment of the Central Government to ensure the BJE's
participation in meetings and events in the ASEAN and the specialized UN agencies, and the
continuing responsibility of the Central Government over external defense. Moreover, the BJE's right
to participate in Philippine official missions bearing on negotiation of border agreements,
environmental protection, and sharing of revenues pertaining to the bodies of water adjacent to or
between the islands forming part of the ancestral domain, resembles the right of the governments of
FSM and the Marshall Islands to be consulted by the U.S. government on any foreign affairs matter
affecting them.
These provisions of the MOA indicate, among other things, that the Parties aimed to vest in the
BJE the status of an associated state or, at any rate, a status closely approximating it.
The concept of association is not recognized under the present Constitution
No province, city, or municipality, not even the ARMM, is recognized under our laws as having an
"associative" relationship with the national government. Indeed, the concept implies powers that go
beyond anything ever granted by the Constitution to any local or regional government. It also implies
the recognition of the associated entity as a state. The Constitution, however, does not contemplate
any state in this jurisdiction other than the Philippine State, much less does it provide for a transitory
status that aims to prepare any part of Philippine territory for independence.
Even the mere concept animating many of the MOA-AD's provisions, therefore, already requires for
its validity the amendment of constitutional provisions, specifically the following provisions of Article
X:
SECTION 1. The territorial and political subdivisions of the Republic of the Philippines are
the provinces, cities, municipalities, and barangays. There shall be autonomous
regions in Muslim Mindanao and the Cordilleras as hereinafter provided.
SECTION 15. There shall be created autonomous regions in Muslim Mindanao and in the
Cordilleras consisting of provinces, cities, municipalities, and geographical areas sharing
common and distinctive historical and cultural heritage, economic and social structures, and
other relevant characteristics within the framework of this Constitution and the national
sovereignty as well as territorial integrity of the Republic of the Philippines.
The BJE is a far more powerful
entity than the autonomous region
recognized in the Constitution

It is not merely an expanded version of the ARMM, the status of its relationship with the national
government being fundamentally different from that of the ARMM. Indeed, BJE is a state in all but
name as it meets the criteria of a state laid down in the Montevideo Convention,154 namely,
a permanent population, a defined territory, a government, and a capacity to enter into relations with
other states.
Even assuming arguendo that the MOA-AD would not necessarily sever any portion of Philippine
territory, the spirit animating it - which has betrayed itself by its use of the concept of association
- runs counter to the national sovereignty and territorial integrity of the Republic.
The defining concept underlying the relationship between the national government and the
BJE being itself contrary to the present Constitution, it is not surprising that many of the
specific provisions of the MOA-AD on the formation and powers of the BJE are in conflict
with the Constitution and the laws.
Article X, Section 18 of the Constitution provides that "[t]he creation of the autonomous region shall
be effective when approved by a majority of the votes cast by the constituent units in a plebiscite
called for the purpose, provided that only provinces, cities, and geographic areas voting
favorably in such plebiscite shall be included in the autonomous region." (Emphasis supplied)
As reflected above, the BJE is more of a state than an autonomous region. But even assuming that it
is covered by the term "autonomous region" in the constitutional provision just quoted, the MOA-AD
would still be in conflict with it. Under paragraph 2(c) on TERRITORY in relation to 2(d) and 2(e), the
present geographic area of the ARMM and, in addition, the municipalities of Lanao del Norte which
voted for inclusion in the ARMM during the 2001 plebiscite - Baloi, Munai, Nunungan, Pantar,
Tagoloan and Tangkal - are automatically part of the BJE without need of another plebiscite, in
contrast to the areas under Categories A and B mentioned earlier in the overview. That the present
components of the ARMM and the above-mentioned municipalities voted for inclusion therein in
2001, however, does not render another plebiscite unnecessary under the Constitution, precisely
because what these areas voted for then was their inclusion in the ARMM, not the BJE.
The MOA-AD, moreover, would not
comply with Article X, Section 20 of
the Constitution
since that provision defines the powers of autonomous regions as follows:
SECTION 20. Within its territorial jurisdiction and subject to the provisions of this Constitution
and national laws, the organic act of autonomous regions shall provide for legislative powers
over:
(1) Administrative organization;
(2) Creation of sources of revenues;
(3) Ancestral domain and natural resources;
(4) Personal, family, and property relations;
(5) Regional urban and rural planning development;

(6) Economic, social, and tourism development;


(7) Educational policies;
(8) Preservation and development of the cultural heritage; and
(9) Such other matters as may be authorized by law for the promotion of the general welfare
of the people of the region. (Underscoring supplied)
Again on the premise that the BJE may be regarded as an autonomous region, the MOA-AD would
require an amendment that would expand the above-quoted provision. The mere passage of new
legislation pursuant to sub-paragraph No. 9 of said constitutional provision would not suffice, since
any new law that might vest in the BJE the powers found in the MOA-AD must, itself, comply with
other provisions of the Constitution. It would not do, for instance, to merely pass legislation vesting
the BJE with treaty-making power in order to accommodate paragraph 4 of the strand on
RESOURCES which states: "The BJE is free to enter into any economic cooperation and trade
relations with foreign countries: provided, however, that such relationships and understandings do
not include aggression against the Government of the Republic of the Philippines x x x." Under our
constitutional system, it is only the President who has that power. Pimentel v. Executive
Secretary155 instructs:
In our system of government, the President, being the head of state, is regarded as the sole
organ and authority in external relations and is the country's sole representative with
foreign nations. As the chief architect of foreign policy, the President acts as the country's
mouthpiece with respect to international affairs. Hence, the President is vested with the
authority to deal with foreign states and governments, extend or withhold
recognition, maintain diplomatic relations, enter into treaties, and otherwise transact
the business of foreign relations. In the realm of treaty-making, the President has the
sole authority to negotiate with other states. (Emphasis and underscoring supplied)
Article II, Section 22 of the Constitution must also be amended if the scheme envisioned in
the MOA-AD is to be effected. That constitutional provision states: "The State recognizes and
promotes the rights ofindigenous cultural communities within the framework of national unity and
development." (Underscoring supplied) An associative arrangement does not uphold national unity.
While there may be a semblance of unity because of the associative ties between the BJE and the
national government, the act of placing a portion of Philippine territory in a status which, in
international practice, has generally been a preparation for independence, is certainly not conducive
to national unity.
Besides being irreconcilable with the Constitution, the MOA-AD is also inconsistent with prevailing
statutory law, among which are R.A. No. 9054156 or the Organic Act of the ARMM, and the IPRA.157
Article X, Section 3 of the Organic Act of the ARMM is a bar to the adoption of the definition
of "Bangsamoro people" used in the MOA-AD. Paragraph 1 on Concepts and Principles states:
1. It is the birthright of all Moros and all Indigenous peoples of Mindanao to identify
themselves and be accepted as "Bangsamoros". The Bangsamoro people refers to those
who are natives or original inhabitants of Mindanao and its adjacent islands including
Palawan and the Sulu archipelago at the time of conquest or colonization of its descendants
whether mixed or of full blood. Spouses and their descendants are classified as
Bangsamoro. The freedom of choice of the Indigenous people shall be respected. (Emphasis
and underscoring supplied)

This use of the term Bangsamoro sharply contrasts with that found in the Article X, Section 3 of the
Organic Act, which, rather than lumping together the identities of the Bangsamoro and other
indigenous peoples living in Mindanao, clearly distinguishes between Bangsamoro people and
Tribal peoples, as follows:
"As used in this Organic Act, the phrase "indigenous cultural community" refers to Filipino
citizens residing in the autonomous region who are:
(a) Tribal peoples. These are citizens whose social, cultural and economic conditions
distinguish them from other sectors of the national community; and
(b) Bangsa Moro people. These are citizens who are believers in Islam and who have
retained some or all of their own social, economic, cultural, and political institutions."
Respecting the IPRA, it lays down the prevailing procedure for the delineation and recognition of
ancestral domains. The MOA-AD's manner of delineating the ancestral domain of the Bangsamoro
people is a clear departure from that procedure. By paragraph 1 of Territory, the Parties simply agree
that, subject to the delimitations in the agreed Schedules, "[t]he Bangsamoro homeland and historic
territory refer to the land mass as well as the maritime, terrestrial, fluvial and alluvial domains, and
the aerial domain, the atmospheric space above it, embracing the Mindanao-Sulu-Palawan
geographic region."
Chapter VIII of the IPRA, on the other hand, lays down a detailed procedure, as illustrated in the
following provisions thereof:
SECTION 52. Delineation Process. - The identification and delineation of ancestral domains
shall be done in accordance with the following procedures:
xxxx
b) Petition for Delineation. - The process of delineating a specific perimeter may be initiated
by the NCIP with the consent of the ICC/IP concerned, or through a Petition for Delineation
filed with the NCIP, by a majority of the members of the ICCs/IPs;
c) Delineation Proper. - The official delineation of ancestral domain boundaries including
census of all community members therein, shall be immediately undertaken by the Ancestral
Domains Office upon filing of the application by the ICCs/IPs concerned. Delineation will be
done in coordination with the community concerned and shall at all times include genuine
involvement and participation by the members of the communities concerned;
d) Proof Required. - Proof of Ancestral Domain Claims shall include the testimony of elders
or community under oath, and other documents directly or indirectly attesting to the
possession or occupation of the area since time immemorial by such ICCs/IPs in the concept
of owners which shall be any one (1) of the following authentic documents:
1) Written accounts of the ICCs/IPs customs and traditions;
2) Written accounts of the ICCs/IPs political structure and institution;
3) Pictures showing long term occupation such as those of old improvements, burial
grounds, sacred places and old villages;

4) Historical accounts, including pacts and agreements concerning boundaries


entered into by the ICCs/IPs concerned with other ICCs/IPs;
5) Survey plans and sketch maps;
6) Anthropological data;
7) Genealogical surveys;
8) Pictures and descriptive histories of traditional communal forests and hunting
grounds;
9) Pictures and descriptive histories of traditional landmarks such as mountains,
rivers, creeks, ridges, hills, terraces and the like; and
10) Write-ups of names and places derived from the native dialect of the community.
e) Preparation of Maps. - On the basis of such investigation and the findings of fact based
thereon, the Ancestral Domains Office of the NCIP shall prepare a perimeter map, complete
with technical descriptions, and a description of the natural features and landmarks
embraced therein;
f) Report of Investigation and Other Documents. - A complete copy of the preliminary census
and a report of investigation, shall be prepared by the Ancestral Domains Office of the NCIP;
g) Notice and Publication. - A copy of each document, including a translation in the native
language of the ICCs/IPs concerned shall be posted in a prominent place therein for at least
fifteen (15) days. A copy of the document shall also be posted at the local, provincial and
regional offices of the NCIP, and shall be published in a newspaper of general circulation
once a week for two (2) consecutive weeks to allow other claimants to file opposition thereto
within fifteen (15) days from date of such publication: Provided, That in areas where no such
newspaper exists, broadcasting in a radio station will be a valid substitute: Provided, further,
That mere posting shall be deemed sufficient if both newspaper and radio station are not
available;
h) Endorsement to NCIP. - Within fifteen (15) days from publication, and of the inspection
process, the Ancestral Domains Office shall prepare a report to the NCIP endorsing a
favorable action upon a claim that is deemed to have sufficient proof. However, if the proof is
deemed insufficient, the Ancestral Domains Office shall require the submission of additional
evidence: Provided, That the Ancestral Domains Office shall reject any claim that is deemed
patently false or fraudulent after inspection and verification: Provided, further, That in case of
rejection, the Ancestral Domains Office shall give the applicant due notice, copy furnished all
concerned, containing the grounds for denial. The denial shall be appealable to the NCIP:
Provided, furthermore, That in cases where there are conflicting claims among ICCs/IPs on
the boundaries of ancestral domain claims, the Ancestral Domains Office shall cause the
contending parties to meet and assist them in coming up with a preliminary resolution of the
conflict, without prejudice to its full adjudication according to the section below.
xxxx

To remove all doubts about the irreconcilability of the MOA-AD with the present legal system, a
discussion of not only the Constitution and domestic statutes, but also of international law is in order,
for
Article II, Section 2 of the Constitution states that the Philippines "adopts the generally
accepted principles of international law as part of the law of the land."
Applying this provision of the Constitution, the Court, in Mejoff v. Director of Prisons,158 held that the
Universal Declaration of Human Rights is part of the law of the land on account of which it ordered
the release on bail of a detained alien of Russian descent whose deportation order had not been
executed even after two years. Similarly, the Court in Agustin v. Edu159 applied the aforesaid
constitutional provision to the 1968 Vienna Convention on Road Signs and Signals.
International law has long recognized the right to self-determination of "peoples," understood not
merely as the entire population of a State but also a portion thereof. In considering the question of
whether the people of Quebec had a right to unilaterally secede from Canada, the Canadian
Supreme Court in REFERENCE RE SECESSION OF QUEBEC160 had occasion to acknowledge that
"the right of a people to self-determination is now so widely recognized in international conventions
that the principle has acquired a status beyond convention' and is considered a general principle of
international law."
Among the conventions referred to are the International Covenant on Civil and Political Rights 161 and
the International Covenant on Economic, Social and Cultural Rights 162 which state, in Article 1 of both
covenants, that all peoples, by virtue of the right of self-determination, "freely determine their political
status and freely pursue their economic, social, and cultural development."
The people's right to self-determination should not, however, be understood as extending to a
unilateral right of secession. A distinction should be made between the right of internal and external
self-determination. REFERENCE RE SECESSION OF QUEBEC is again instructive:
"(ii) Scope of the Right to Self-determination
126. The recognized sources of international law establish that the right to selfdetermination of a people is normally fulfilled through internal self-determination - a
people's pursuit of its political, economic, social and cultural development within the
framework of an existing state. A right toexternal self-determination (which in this
case potentially takes the form of the assertion of a right to unilateral secession)
arises in only the most extreme of cases and, even then, under carefully defined
circumstances. x x x
External self-determination can be defined as in the following statement from
the Declaration on Friendly Relations, supra, as
The establishment of a sovereign and independent State, the free association or
integration with an independent State or the emergence into any other political status
freely determined by apeople constitute modes of implementing the right of selfdetermination by that people. (Emphasis added)
127. The international law principle of self-determination has evolved within a
framework of respect for the territorial integrity of existing states. The various
international documents that support the existence of a people's right to self-determination

also contain parallel statements supportive of the conclusion that the exercise of such a right
must be sufficiently limited to prevent threats to an existing state's territorial integrity or the
stability of relations between sovereign states.
x x x x (Emphasis, italics and underscoring supplied)
The Canadian Court went on to discuss the exceptional cases in which the right to external selfdetermination can arise, namely, where a people is under colonial rule, is subject to foreign
domination or exploitation outside a colonial context, and - less definitely but asserted by a number
of commentators - is blocked from the meaningful exercise of its right to internal self-determination.
The Court ultimately held that the population of Quebec had no right to secession, as the same is
not under colonial rule or foreign domination, nor is it being deprived of the freedom to make political
choices and pursue economic, social and cultural development, citing that Quebec is equitably
represented in legislative, executive and judicial institutions within Canada, even occupying
prominent positions therein.
The exceptional nature of the right of secession is further exemplified in the REPORT OF THE
INTERNATIONAL COMMITTEE OF JURISTS ON THE LEGAL ASPECTS OF THE AALAND
ISLANDS QUESTION.163 There, Sweden presented to the Council of the League of Nations the
question of whether the inhabitants of the Aaland Islands should be authorized to determine by
plebiscite if the archipelago should remain under Finnish sovereignty or be incorporated in the
kingdom of Sweden. The Council, before resolving the question, appointed an International
Committee composed of three jurists to submit an opinion on the preliminary issue of whether the
dispute should, based on international law, be entirely left to the domestic jurisdiction of Finland. The
Committee stated the rule as follows:
x x x [I]n the absence of express provisions in international treaties, the right of disposing
of national territory is essentially an attribute of the sovereignty of every State.
Positive International Law does not recognize the right of national groups, as such, to
separate themselves from the State of which they form part by the simple expression
of a wish, any more than it recognizes the right of other States to claim such a
separation. Generally speaking, the grant or refusal of the right to a portion of its
population of determining its own political fate by plebiscite or by some other method,
is, exclusively, an attribute of the sovereignty of every State which is definitively
constituted. A dispute between two States concerning such a question, under normal
conditions therefore, bears upon a question which International Law leaves entirely to the
domestic jurisdiction of one of the States concerned. Any other solution would amount to an
infringement of sovereign rights of a State and would involve the risk of creating difficulties
and a lack of stability which would not only be contrary to the very idea embodied in term
"State," but would also endanger the interests of the international community. If this right is
not possessed by a large or small section of a nation, neither can it be held by the State to
which the national group wishes to be attached, nor by any other State. (Emphasis and
underscoring supplied)
The Committee held that the dispute concerning the Aaland Islands did not refer to a question which
is left by international law to the domestic jurisdiction of Finland, thereby applying the exception
rather than the rule elucidated above. Its ground for departing from the general rule, however, was a
very narrow one, namely, the Aaland Islands agitation originated at a time when Finland was
undergoing drastic political transformation. The internal situation of Finland was, according to the
Committee, so abnormal that, for a considerable time, the conditions required for the formation of a
sovereign State did not exist. In the midst of revolution, anarchy, and civil war, the legitimacy of the
Finnish national government was disputed by a large section of the people, and it had, in fact, been

chased from the capital and forcibly prevented from carrying out its duties. The armed camps and
the police were divided into two opposing forces. In light of these circumstances, Finland was not,
during the relevant time period, a "definitively constituted" sovereign state. The Committee,
therefore, found that Finland did not possess the right to withhold from a portion of its population the
option to separate itself - a right which sovereign nations generally have with respect to their own
populations.
Turning now to the more specific category of indigenous peoples, this term has been used, in
scholarship as well as international, regional, and state practices, to refer to groups with distinct
cultures, histories, and connections to land (spiritual and otherwise) that have been forcibly
incorporated into a larger governing society. These groups are regarded as "indigenous" since they
are the living descendants of pre-invasion inhabitants of lands now dominated by others. Otherwise
stated, indigenous peoples, nations, or communities are culturally distinctive groups that find
themselves engulfed by settler societies born of the forces of empire and conquest. 164 Examples of
groups who have been regarded as indigenous peoples are the Maori of New Zealand and the
aboriginal peoples of Canada.
As with the broader category of "peoples," indigenous peoples situated within states do not have a
general right to independence or secession from those states under international law,165 but they do
have rights amounting to what was discussed above as the right to internal self-determination.
In a historic development last September 13, 2007, the UN General Assembly adopted the United
Nations Declaration on the Rights of Indigenous Peoples (UN DRIP) through General Assembly
Resolution 61/295. The vote was 143 to 4, the Philippines being included among those in favor,
and the four voting against being Australia, Canada, New Zealand, and the U.S. The Declaration
clearly recognized the right of indigenous peoples to self-determination, encompassing the
right to autonomy or self-government, to wit:
Article 3
Indigenous peoples have the right to self-determination. By virtue of that right they freely
determine their political status and freely pursue their economic, social and cultural
development.
Article 4
Indigenous peoples, in exercising their right to self-determination, have the right
to autonomy or self-government in matters relating to their internal and local affairs,
as well as ways and means for financing their autonomous functions.
Article 5
Indigenous peoples have the right to maintain and strengthen their distinct political, legal,
economic, social and cultural institutions, while retaining their right to participate fully, if they
so choose, in the political, economic, social and cultural life of the State.
Self-government, as used in international legal discourse pertaining to indigenous peoples, has been
understood as equivalent to "internal self-determination."166 The extent of self-determination provided
for in the UN DRIP is more particularly defined in its subsequent articles, some of which are quoted
hereunder:

Article 8
1. Indigenous peoples and individuals have the right not to be subjected to forced
assimilation or destruction of their culture.
2. States shall provide effective mechanisms for prevention of, and redress for:
(a) Any action which has the aim or effect of depriving them of their integrity as
distinct peoples, or of their cultural values or ethnic identities;
(b) Any action which has the aim or effect of dispossessing them of their lands,
territories or resources;
(c) Any form of forced population transfer which has the aim or effect of violating or
undermining any of their rights;
(d) Any form of forced assimilation or integration;
(e) Any form of propaganda designed to promote or incite racial or ethnic
discrimination directed against them.
Article 21
1. Indigenous peoples have the right, without discrimination, to the improvement of their
economic and social conditions, including, inter alia, in the areas of education, employment,
vocational training and retraining, housing, sanitation, health and social security.
2. States shall take effective measures and, where appropriate, special measures to ensure
continuing improvement of their economic and social conditions. Particular attention shall be
paid to the rights and special needs of indigenous elders, women, youth, children and
persons with disabilities.
Article 26
1. Indigenous peoples have the right to the lands, territories and resources which they
have traditionally owned, occupied or otherwise used or acquired.
2. Indigenous peoples have the right to own, use, develop and control the lands, territories
and resources that they possess by reason of traditional ownership or other traditional
occupation or use, as well as those which they have otherwise acquired.
3. States shall give legal recognition and protection to these lands, territories and resources.
Such recognition shall be conducted with due respect to the customs, traditions and land
tenure systems of the indigenous peoples concerned.
Article 30
1. Military activities shall not take place in the lands or territories of indigenous peoples,
unless justified by a relevant public interest or otherwise freely agreed with or requested by
the indigenous peoples concerned.

2. States shall undertake effective consultations with the indigenous peoples concerned,
through appropriate procedures and in particular through their representative institutions,
prior to using their lands or territories for military activities.
Article 32
1. Indigenous peoples have the right to determine and develop priorities and strategies for
the development or use of their lands or territories and other resources.
2. States shall consult and cooperate in good faith with the indigenous peoples concerned
through their own representative institutions in order to obtain their free and informed
consent prior to the approval of any project affecting their lands or territories and other
resources, particularly in connection with the development, utilization or exploitation of
mineral, water or other resources.
3. States shall provide effective mechanisms for just and fair redress for any such activities,
and appropriate measures shall be taken to mitigate adverse environmental, economic,
social, cultural or spiritual impact.
Article 37
1. Indigenous peoples have the right to the recognition, observance and enforcement of
treaties, agreements and other constructive arrangements concluded with States or their
successors and to have States honour and respect such treaties, agreements and other
constructive arrangements.
2. Nothing in this Declaration may be interpreted as diminishing or eliminating the rights of
indigenous peoples contained in treaties, agreements and other constructive arrangements.
Article 38
States in consultation and cooperation with indigenous peoples, shall take the appropriate
measures, including legislative measures, to achieve the ends of this Declaration.
Assuming that the UN DRIP, like the Universal Declaration on Human Rights, must now be regarded
as embodying customary international law - a question which the Court need not definitively resolve
here - the obligations enumerated therein do not strictly require the Republic to grant the
Bangsamoro people, through the instrumentality of the BJE, the particular rights and powers
provided for in the MOA-AD. Even the more specific provisions of the UN DRIP are general in scope,
allowing for flexibility in its application by the different States.
There is, for instance, no requirement in the UN DRIP that States now guarantee indigenous
peoples their own police and internal security force. Indeed, Article 8 presupposes that it is the State
which will provide protection for indigenous peoples against acts like the forced dispossession of
their lands - a function that is normally performed by police officers. If the protection of a right so
essential to indigenous people's identity is acknowledged to be the responsibility of the State, then
surely the protection of rights less significant to them as such peoples would also be the duty of
States. Nor is there in the UN DRIP an acknowledgement of the right of indigenous peoples to the
aerial domain and atmospheric space. What it upholds, in Article 26 thereof, is the right of
indigenous peoples to the lands, territories and resources which they have traditionally owned,
occupied or otherwise used or acquired.

Moreover, the UN DRIP, while upholding the right of indigenous peoples to autonomy, does not
obligate States to grant indigenous peoples the near-independent status of an associated state. All
the rights recognized in that document are qualified in Article 46 as follows:
1. Nothing in this Declaration may be interpreted as implying for any State, people, group
or person any right to engage in any activity or to perform any act contrary to the Charter of
the United Nations orconstrued as authorizing or encouraging any action which would
dismember or impair, totally or in part, the territorial integrity or political unity of
sovereign and independent States.
Even if the UN DRIP were considered as part of the law of the land pursuant to Article II, Section 2 of
the Constitution, it would not suffice to uphold the validity of the MOA-AD so as to render its
compliance with other laws unnecessary.
It is, therefore, clear that the MOA-AD contains numerous provisions that cannot be
reconciled with the Constitution and the laws as presently worded. Respondents proffer,
however, that the signing of the MOA-AD alone would not have entailed any violation of law or grave
abuse of discretion on their part, precisely because it stipulates that the provisions thereof
inconsistent with the laws shall not take effect until these laws are amended. They cite paragraph 7
of the MOA-AD strand on GOVERNANCE quoted earlier, but which is reproduced below for
convenience:
7. The Parties agree that the mechanisms and modalities for the actual implementation of
this MOA-AD shall be spelt out in the Comprehensive Compact to mutually take such steps
to enable it to occur effectively.
Any provisions of the MOA-AD requiring amendments to the existing legal framework shall
come into force upon signing of a Comprehensive Compact and upon effecting the
necessary changes to the legal framework with due regard to non derogation of prior
agreements and within the stipulated timeframe to be contained in the Comprehensive
Compact.
Indeed, the foregoing stipulation keeps many controversial provisions of the MOA-AD from coming
into force until the necessary changes to the legal framework are effected. While the word
"Constitution" is not mentioned in the provision now under consideration or anywhere else
in the MOA-AD, the term "legal framework" is certainly broad enough to include the
Constitution.
Notwithstanding the suspensive clause, however, respondents, by their mere act of incorporating in
the MOA-AD the provisions thereof regarding the associative relationship between the BJE and the
Central Government, have already violated the Memorandum of Instructions From The President
dated March 1, 2001, which states that the "negotiations shall be conducted in accordance with x x x
the principles of the sovereignty and territorial integrityof the Republic of the Philippines."
(Emphasis supplied) Establishing an associative relationship between the BJE and the Central
Government is, for the reasons already discussed, a preparation for independence, or worse, an
implicit acknowledgment of an independent status already prevailing.
Even apart from the above-mentioned Memorandum, however, the MOA-AD is defective because
the suspensive clause is invalid, as discussed below.
The authority of the GRP Peace Negotiating Panel to negotiate with the MILF is founded on E.O. No.
3, Section 5(c), which states that there shall be established Government Peace Negotiating Panels

for negotiations with different rebel groups to be "appointed by the President as her official
emissaries to conduct negotiations, dialogues, and face-to-face discussions with rebel groups."
These negotiating panels are to report to the President, through the PAPP on the conduct and
progress of the negotiations.
It bears noting that the GRP Peace Panel, in exploring lasting solutions to the Moro Problem through
its negotiations with the MILF, was not restricted by E.O. No. 3 only to those options available under
the laws as they presently stand. One of the components of a comprehensive peace process, which
E.O. No. 3 collectively refers to as the "Paths to Peace," is the pursuit of social, economic, and
political reforms which may require new legislation or even constitutional amendments. Sec. 4(a) of
E.O. No. 3, which reiterates Section 3(a), of E.O. No. 125, 167 states:
SECTION 4. The Six Paths to Peace. - The components of the comprehensive peace
process comprise the processes known as the "Paths to Peace". These component
processes are interrelated and not mutually exclusive, and must therefore be pursued
simultaneously in a coordinated and integrated fashion. They shall include, but may not be
limited to, the following:
a. PURSUIT OF SOCIAL, ECONOMIC AND POLITICAL REFORMS. This component
involves the vigorous implementation of various policies, reforms, programs and
projects aimed at addressing the root causes of internal armed conflicts and social
unrest. This may require administrative action, new legislation or even constitutional
amendments.
x x x x (Emphasis supplied)
The MOA-AD, therefore, may reasonably be perceived as an attempt of respondents to address,
pursuant to this provision of E.O. No. 3, the root causes of the armed conflict in Mindanao. The E.O.
authorized them to "think outside the box," so to speak. Hence, they negotiated and were set on
signing the MOA-AD that included various social, economic, and political reforms which cannot,
however, all be accommodated within the present legal framework, and which thus would require
new legislation and constitutional amendments.
The inquiry on the legality of the "suspensive clause," however, cannot stop here, because it must
be askedwhether the President herself may exercise the power delegated to the GRP Peace
Panel under E.O. No. 3, Sec. 4(a).
The President cannot delegate a power that she herself does not possess. May the President, in the
course of peace negotiations, agree to pursue reforms that would require new legislation and
constitutional amendments, or should the reforms be restricted only to those solutions which the
present laws allow? The answer to this question requires a discussion of the extent of the
President's power to conduct peace negotiations.
That the authority of the President to conduct peace negotiations with rebel groups is not explicitly
mentioned in the Constitution does not mean that she has no such authority. In Sanlakas v.
Executive Secretary,168 in issue was the authority of the President to declare a state of rebellion - an
authority which is not expressly provided for in the Constitution. The Court held thus:
"In her ponencia in Marcos v. Manglapus, Justice Cortes put her thesis into jurisprudence.
There, the Court, by a slim 8-7 margin, upheld the President's power to forbid the return of
her exiled predecessor. The rationale for the majority's ruling rested on the President's

. . . unstated residual powers which are implied from the grant of executive
power and which are necessary for her to comply with her duties under the
Constitution. The powers of the President are not limited to what are expressly
enumerated in the article on the Executive Department and in scattered
provisions of the Constitution. This is so, notwithstanding the avowed intent of the
members of the Constitutional Commission of 1986 to limit the powers of the
President as a reaction to the abuses under the regime of Mr. Marcos, for the result
was a limitation of specific powers of the President, particularly those relating to the
commander-in-chief clause, but not a diminution of the general grant of executive
power.
Thus, the President's authority to declare a state of rebellion springs in the main from
her powers as chief executive and, at the same time, draws strength from her
Commander-in-Chief powers. x x x (Emphasis and underscoring supplied)
Similarly, the President's power to conduct peace negotiations is implicitly included in her powers as
Chief Executive and Commander-in-Chief. As Chief Executive, the President has the general
responsibility to promote public peace, and as Commander-in-Chief, she has the more specific duty
to prevent and suppress rebellion and lawless violence.169
As the experience of nations which have similarly gone through internal armed conflict will show,
however, peace is rarely attained by simply pursuing a military solution. Oftentimes, changes as farreaching as a fundamental reconfiguration of the nation's constitutional structure is required. The
observations of Dr. Kirsti Samuels are enlightening, to wit:
x x x [T]he fact remains that a successful political and governance transition must form the
core of any post-conflict peace-building mission. As we have observed in Liberia and Haiti
over the last ten years, conflict cessation without modification of the political environment,
even where state-building is undertaken through technical electoral assistance and
institution- or capacity-building, is unlikely to succeed. On average, more than 50 percent of
states emerging from conflict return to conflict. Moreover, a substantial proportion of
transitions have resulted in weak or limited democracies.
The design of a constitution and its constitution-making process can play an important role in
the political and governance transition. Constitution-making after conflict is an opportunity to
create a common vision of the future of a state and a road map on how to get there. The
constitution can be partly a peace agreement and partly a framework setting up the rules by
which the new democracy will operate.170
In the same vein, Professor Christine Bell, in her article on the nature and legal status of peace
agreements, observed that the typical way that peace agreements establish or confirm mechanisms
for demilitarization and demobilization is by linking them to new constitutional
structures addressing governance, elections, and legal and human rights institutions. 171
In the Philippine experience, the link between peace agreements and constitution-making has been
recognized by no less than the framers of the Constitution. Behind the provisions of the Constitution
on autonomous regions172is the framers' intention to implement a particular peace agreement,
namely, the Tripoli Agreement of 1976 between the GRP and the MNLF, signed by then
Undersecretary of National Defense Carmelo Z. Barbero and then MNLF Chairman Nur Misuari.

MR. ROMULO. There are other speakers; so, although I have some more questions, I will
reserve my right to ask them if they are not covered by the other speakers. I have only two
questions.
I heard one of the Commissioners say that local autonomy already exists in the
Muslim region; it is working very well; it has, in fact, diminished a great deal of the
problems. So, my question is: since that already exists, why do we have to go into
something new?
MR. OPLE. May I answer that on behalf of Chairman Nolledo. Commissioner Yusup
Abubakar is right thatcertain definite steps have been taken to implement the provisions
of the Tripoli Agreement with respect to an autonomous region in Mindanao. This is a
good first step, but there is no question that this is merely a partial response to the
Tripoli Agreement itself and to the fuller standard of regional autonomy contemplated
in that agreement, and now by state policy.173(Emphasis supplied)
The constitutional provisions on autonomy and the statutes enacted pursuant to them have, to the
credit of their drafters, been partly successful. Nonetheless, the Filipino people are still faced with
the reality of an on-going conflict between the Government and the MILF. If the President is to be
expected to find means for bringing this conflict to an end and to achieve lasting peace in Mindanao,
then she must be given the leeway to explore, in the course of peace negotiations, solutions that
may require changes to the Constitution for their implementation. Being uniquely vested with the
power to conduct peace negotiations with rebel groups, the President is in a singular position to
know the precise nature of their grievances which, if resolved, may bring an end to hostilities.
The President may not, of course, unilaterally implement the solutions that she considers viable, but
she may not be prevented from submitting them as recommendations to Congress, which could
then, if it is minded, act upon them pursuant to the legal procedures for constitutional amendment
and revision. In particular, Congress would have the option, pursuant to Article XVII, Sections 1 and
3 of the Constitution, to propose the recommended amendments or revision to the people, call a
constitutional convention, or submit to the electorate the question of calling such a convention.
While the President does not possess constituent powers - as those powers may be exercised only
by Congress, a Constitutional Convention, or the people through initiative and referendum - she may
submit proposals for constitutional change to Congress in a manner that does not involve the
arrogation of constituent powers.
In Sanidad v. COMELEC,174 in issue was the legality of then President Marcos' act of directly
submitting proposals for constitutional amendments to a referendum, bypassing the interim National
Assembly which was the body vested by the 1973 Constitution with the power to propose such
amendments. President Marcos, it will be recalled, never convened the interim National Assembly.
The majority upheld the President's act, holding that "the urges of absolute necessity" compelled the
President as the agent of the people to act as he did, there being no interim National Assembly to
propose constitutional amendments. Against this ruling, Justices Teehankee and Muoz Palma
vigorously dissented. The Court's concern at present, however, is not with regard to the point on
which it was then divided in that controversial case, but on that which was not disputed by either
side.
Justice Teehankee's dissent,175 in particular, bears noting. While he disagreed that the President may
directly submit proposed constitutional amendments to a referendum, implicit in his opinion is a
recognition that he would have upheld the President's action along with the majority had the

President convened the interim National Assembly and coursed his proposals through it. Thus
Justice Teehankee opined:
"Since the Constitution provides for the organization of the essential departments of
government, defines and delimits the powers of each and prescribes the manner of the
exercise of such powers, and the constituent power has not been granted to but has been
withheld from the President or Prime Minister, it follows that the President's questioned
decrees proposing and submitting constitutional amendments directly to the people (without
the intervention of the interim National Assembly in whom the power is expressly
vested) are devoid of constitutional and legal basis."176 (Emphasis supplied)
From the foregoing discussion, the principle may be inferred that the President - in the course of
conducting peace negotiations - may validly consider implementing even those policies that require
changes to the Constitution, but she may not unilaterally implement them without the intervention
of Congress, or act in any way as if the assent of that body were assumed as a certainty.
Since, under the present Constitution, the people also have the power to directly propose
amendments through initiative and referendum, the President may also submit her
recommendations to the people, not as a formal proposal to be voted on in a plebiscite similar to
what President Marcos did in Sanidad, but for their independent consideration of whether these
recommendations merit being formally proposed through initiative.
These recommendations, however, may amount to nothing more than the President's suggestions to
the people, for any further involvement in the process of initiative by the Chief Executive may vitiate
its character as a genuine "people's initiative." The only initiative recognized by the Constitution is
that which truly proceeds from the people. As the Court stated in Lambino v. COMELEC:177
"The Lambino Group claims that their initiative is the people's voice.' However, the Lambino
Group unabashedly states in ULAP Resolution No. 2006-02, in the verification of their
petition with the COMELEC, that ULAP maintains its unqualified support to the agenda of
Her Excellency President Gloria Macapagal-Arroyo for constitutional reforms.' The Lambino
Group thus admits that their people's' initiative is an unqualified support to the
agenda' of the incumbent President to change the Constitution. This forewarns the Court to
be wary of incantations of people's voice' or sovereign will' in the present initiative."
It will be observed that the President has authority, as stated in her oath of office, 178 only to preserve
and defend the Constitution. Such presidential power does not, however, extend to allowing her to
change the Constitution, but simply to recommend proposed amendments or revision. As long as
she limits herself to recommending these changes and submits to the proper procedure for
constitutional amendments and revision, her mere recommendation need not be construed as an
unconstitutional act.
The foregoing discussion focused on the President's authority to
propose constitutional amendments, since her authority to propose new legislation is not in
controversy. It has been an accepted practice for Presidents in this jurisdiction to propose new
legislation. One of the more prominent instances the practice is usually done is in the yearly State of
the Nation Address of the President to Congress. Moreover, the annual general appropriations bill
has always been based on the budget prepared by the President, which - for all intents and
purposes - is a proposal for new legislation coming from the President. 179
The "suspensive clause" in the MOA-AD viewed in light of the above-discussed standards

Given the limited nature of the President's authority to propose constitutional amendments,
she cannot guarantee to any third party that the required amendments will eventually be put in
place, nor even be submitted to a plebiscite. The most she could do is submit these proposals as
recommendations either to Congress or the people, in whom constituent powers are vested.
Paragraph 7 on Governance of the MOA-AD states, however, that all provisions thereof which
cannot be reconciled with the present Constitution and laws "shall come into force upon signing of a
Comprehensive Compact and upon effecting the necessary changes to the legal framework." This
stipulation does not bear the marks of a suspensive condition - defined in civil law as a future
and uncertain event - but of a term. It is not a question of whether the necessary changes to the
legal framework will be effected, but when. That there is no uncertainty being contemplated is plain
from what follows, for the paragraph goes on to state that the contemplated changes shall be "with
due regard to non derogation of prior agreements and within the stipulated timeframe to be
contained in the Comprehensive Compact."
Pursuant to this stipulation, therefore, it is mandatory for the GRP to effect the changes to the legal
framework contemplated in the MOA-AD - which changes would include constitutional amendments,
as discussed earlier. It bears noting that,
By the time these changes are put in place, the MOA-AD itself would be counted among the
"prior agreements" from which there could be no derogation.
What remains for discussion in the Comprehensive Compact would merely be the implementing
details for these "consensus points" and, notably, the deadline for effecting the contemplated
changes to the legal framework.
Plainly, stipulation-paragraph 7 on GOVERNANCE is inconsistent with the limits of the
President's authority to propose constitutional amendments, it being a virtual guarantee that
the Constitution and the laws of the Republic of the Philippines will certainly be adjusted to conform
to all the "consensus points" found in the MOA-AD. Hence, it must be struck down
as unconstitutional.
A comparison between the "suspensive clause" of the MOA-AD with a similar provision appearing in
the 1996 final peace agreement between the MNLF and the GRP is most instructive.
As a backdrop, the parties to the 1996 Agreement stipulated that it would be implemented in two
phases. Phase Icovered a three-year transitional period involving the putting up of new
administrative structures through Executive Order, such as the Special Zone of Peace and
Development (SZOPAD) and the Southern Philippines Council for Peace and Development
(SPCPD), while Phase II covered the establishment of the new regional autonomous
government through amendment or repeal of R.A. No. 6734, which was then the Organic Act of the
ARMM.
The stipulations on Phase II consisted of specific agreements on the structure of the expanded
autonomous region envisioned by the parties. To that extent, they are similar to the provisions of the
MOA-AD. There is, however, a crucial difference between the two agreements. While the MOAAD virtually guarantees that the "necessary changes to the legal framework" will be put in
place, the GRP-MNLF final peace agreement states thus: "Accordingly, these provisions [on Phase
II] shall be recommended by the GRP to Congress for incorporation in the amendatory or repealing
law."

Concerns have been raised that the MOA-AD would have given rise to a binding international law
obligation on the part of the Philippines to change its Constitution in conformity thereto, on the
ground that it may be considered either as a binding agreement under international law, or a
unilateral declaration of the Philippine government to the international community that it would grant
to the Bangsamoro people all the concessions therein stated. Neither ground finds sufficient support
in international law, however.
The MOA-AD, as earlier mentioned in the overview thereof, would have included foreign dignitaries
as signatories. In addition, representatives of other nations were invited to witness its signing in
Kuala Lumpur. These circumstances readily lead one to surmise that the MOA-AD would have had
the status of a binding international agreement had it been signed. An examination of the prevailing
principles in international law, however, leads to the contrary conclusion.
The Decision on Challenge to Jurisdiction: Lom Accord Amnesty180 (the Lom Accord case) of the
Special Court of Sierra Leone is enlightening. The Lom Accord was a peace agreement signed on
July 7, 1999 between the Government of Sierra Leone and the Revolutionary United Front (RUF), a
rebel group with which the Sierra Leone Government had been in armed conflict for around eight
years at the time of signing. There were non-contracting signatories to the agreement, among which
were the Government of the Togolese Republic, the Economic Community of West African States,
and the UN.
On January 16, 2002, after a successful negotiation between the UN Secretary-General and the
Sierra Leone Government, another agreement was entered into by the UN and that Government
whereby the Special Court of Sierra Leone was established. The sole purpose of the Special Court,
an international court, was to try persons who bore the greatest responsibility for serious violations of
international humanitarian law and Sierra Leonean law committed in the territory of Sierra Leone
since November 30, 1996.
Among the stipulations of the Lom Accord was a provision for the full pardon of the members of the
RUF with respect to anything done by them in pursuit of their objectives as members of that
organization since the conflict began.
In the Lom Accord case, the Defence argued that the Accord created an internationally
binding obligation not to prosecute the beneficiaries of the amnesty provided therein, citing, among
other things, the participation of foreign dignitaries and international organizations in the finalization
of that agreement. The Special Court, however, rejected this argument, ruling that the Lome Accord
is not a treaty and that it can only create binding obligations and rights between the parties in
municipal law, not in international law. Hence, the Special Court held, it is ineffective in depriving an
international court like it of jurisdiction.
"37. In regard to the nature of a negotiated settlement of an internal armed conflict it is easy
to assume and to argue with some degree of plausibility, as Defence counsel for the
defendants seem to have done, that the mere fact that in addition to the parties to the
conflict, the document formalizing the settlement is signed by foreign heads of state
or their representatives and representatives of international organizations, means the
agreement of the parties is internationalized so as to create obligations in
international law.
xxxx
40. Almost every conflict resolution will involve the parties to the conflict and the mediator or
facilitator of the settlement, or persons or bodies under whose auspices the settlement took

place but who are not at all parties to the conflict, are not contracting parties and who do not
claim any obligation from the contracting parties or incur any obligation from the settlement.
41. In this case, the parties to the conflict are the lawful authority of the State and the
RUF which has no status of statehood and is to all intents and purposes a faction
within the state. The non-contracting signatories of the Lom Agreement were moral
guarantors of the principle that, in the terms of Article XXXIV of the Agreement, "this
peace agreement is implemented with integrity and in good faith by both parties". The
moral guarantors assumed no legal obligation. It is recalled that the UN by its
representative appended, presumably for avoidance of doubt, an understanding of the extent
of the agreement to be implemented as not including certain international crimes.
42. An international agreement in the nature of a treaty must create rights and obligations
regulated by international law so that a breach of its terms will be a breach determined under
international law which will also provide principle means of enforcement. The Lom
Agreement created neither rights nor obligations capable of being regulated by
international law. An agreement such as the Lom Agreement which brings to an end
an internal armed conflict no doubt creates a factual situation of restoration of peace
that the international community acting through the Security Council may take note
of. That, however, will not convert it to an international agreement which creates an
obligation enforceable in international, as distinguished from municipal, law. A breach
of the terms of such a peace agreement resulting in resumption of internal armed conflict or
creating a threat to peace in the determination of the Security Council may indicate a
reversal of the factual situation of peace to be visited with possible legal consequences
arising from the new situation of conflict created. Such consequences such as action by the
Security Council pursuant to Chapter VII arise from the situation and not from the
agreement, nor from the obligation imposed by it. Such action cannot be regarded as a
remedy for the breach. A peace agreement which settles an internal armed conflict
cannot be ascribed the same status as one which settles an international armed
conflict which, essentially, must be between two or more warring States. The Lom
Agreement cannot be characterised as an international instrument. x x x" (Emphasis,
italics and underscoring supplied)
Similarly, that the MOA-AD would have been signed by representatives of States and international
organizations not parties to the Agreement would not have sufficed to vest in it a binding character
under international law.
In another vein, concern has been raised that the MOA-AD would amount to a unilateral declaration
of the Philippine State, binding under international law, that it would comply with all the stipulations
stated therein, with the result that it would have to amend its Constitution accordingly regardless of
the true will of the people. Cited as authority for this view is Australia v. France,181 also known as
the Nuclear Tests Case, decided by the International Court of Justice (ICJ).
In the Nuclear Tests Case, Australia challenged before the ICJ the legality of France's nuclear tests
in the South Pacific. France refused to appear in the case, but public statements from its President,
and similar statements from other French officials including its Minister of Defence, that its 1974
series of atmospheric tests would be its last, persuaded the ICJ to dismiss the case. 182 Those
statements, the ICJ held, amounted to a legal undertaking addressed to the international community,
which required no acceptance from other States for it to become effective.
Essential to the ICJ ruling is its finding that the French government intended to be bound to the
international community in issuing its public statements, viz:

43. It is well recognized that declarations made by way of unilateral acts, concerning legal or
factual situations, may have the effect of creating legal obligations. Declarations of this kind
may be, and often are, very specific. When it is the intention of the State making the
declaration that it should become bound according to its terms, that intention confers
on the declaration the character of a legal undertaking, the State being thenceforth
legally required to follow a course of conduct consistent with the declaration. An
undertaking of this kind, if given publicly, and with an intent to be bound, even though not
made within the context of international negotiations, is binding. In these circumstances,
nothing in the nature of a quid pro quo nor any subsequent acceptance of the declaration,
nor even any reply or reaction from other States, is required for the declaration to take effect,
since such a requirement would be inconsistent with the strictly unilateral nature of the
juridical act by which the pronouncement by the State was made.
44. Of course, not all unilateral acts imply obligation; but a State may choose to take
up a certain position in relation to a particular matter with the intention of being
bound-the intention is to be ascertained by interpretation of the act. When States make
statements by which their freedom of action is to be limited, a restrictive interpretation is
called for.
xxxx
51. In announcing that the 1974 series of atmospheric tests would be the last, the
French Government conveyed to the world at large, including the Applicant, its
intention effectively to terminate these tests. It was bound to assume that other States
might take note of these statements and rely on their being effective. The validity of
these statements and their legal consequences must be considered within the general
framework of the security of international intercourse, and the confidence and trust
which are so essential in the relations among States. It is from the actual substance of
these statements, and from the circumstances attending their making, that the legal
implications of the unilateral act must be deduced. The objects of these statements
are clear and they were addressed to the international community as a whole, and the
Court holds that they constitute an undertaking possessing legal effect. The Court
considers *270 that the President of the Republic, in deciding upon the effective cessation of
atmospheric tests, gave an undertaking to the international community to which his words
were addressed. x x x (Emphasis and underscoring supplied)
As gathered from the above-quoted ruling of the ICJ, public statements of a state representative may
be construed as a unilateral declaration only when the following conditions are present: the
statements were clearly addressed to the international community, the state intended to be bound to
that community by its statements, and that not to give legal effect to those statements would be
detrimental to the security of international intercourse. Plainly, unilateral declarations arise only in
peculiar circumstances.
The limited applicability of the Nuclear Tests Case ruling was recognized in a later case decided by
the ICJ entitled Burkina Faso v. Mali,183 also known as the Case Concerning the Frontier Dispute.
The public declaration subject of that case was a statement made by the President of Mali, in an
interview by a foreign press agency, that Mali would abide by the decision to be issued by a
commission of the Organization of African Unity on a frontier dispute then pending between Mali and
Burkina Faso.

Unlike in the Nuclear Tests Case, the ICJ held that the statement of Mali's President was not a
unilateral act with legal implications. It clarified that its ruling in the Nuclear Tests case rested on the
peculiar circumstances surrounding the French declaration subject thereof, to wit:
40. In order to assess the intentions of the author of a unilateral act, account must be taken
of all the factual circumstances in which the act occurred. For example, in the Nuclear Tests
cases, the Court took the view that since the applicant States were not the only ones
concerned at the possible continuance of atmospheric testing by the French
Government, that Government's unilateral declarations had conveyed to the world at
large, including the Applicant, its intention effectively to terminate these tests (I.C.J.
Reports 1974, p. 269, para. 51; p. 474, para. 53). In the particular circumstances of those
cases, the French Government could not express an intention to be bound otherwise
than by unilateral declarations. It is difficult to see how it could have accepted the
terms of a negotiated solution with each of the applicants without thereby
jeopardizing its contention that its conduct was lawful. The circumstances of the
present case are radically different. Here, there was nothing to hinder the Parties from
manifesting an intention to accept the binding character of the conclusions of the
Organization of African Unity Mediation Commission by the normal method: a formal
agreement on the basis of reciprocity. Since no agreement of this kind was concluded
between the Parties, the Chamber finds that there are no grounds to interpret the declaration
made by Mali's head of State on 11 April 1975 as a unilateral act with legal implications in
regard to the present case. (Emphasis and underscoring supplied)
Assessing the MOA-AD in light of the above criteria, it would not have amounted to a unilateral
declaration on the part of the Philippine State to the international community. The Philippine panel
did not draft the same with the clear intention of being bound thereby to the international community
as a whole or to any State, but only to the MILF. While there were States and international
organizations involved, one way or another, in the negotiation and projected signing of the MOA-AD,
they participated merely as witnesses or, in the case of Malaysia, as facilitator. As held in the Lom
Accord case, the mere fact that in addition to the parties to the conflict, the peace settlement is
signed by representatives of states and international organizations does not mean that the
agreement is internationalized so as to create obligations in international law.
Since the commitments in the MOA-AD were not addressed to States, not to give legal effect to such
commitments would not be detrimental to the security of international intercourse - to the trust and
confidence essential in the relations among States.
In one important respect, the circumstances surrounding the MOA-AD are closer to that of Burkina
Faso wherein, as already discussed, the Mali President's statement was not held to be a binding
unilateral declaration by the ICJ. As in that case, there was also nothing to hinder the Philippine
panel, had it really been its intention to be bound to other States, to manifest that intention by formal
agreement. Here, that formal agreement would have come about by the inclusion in the MOA-AD of
a clear commitment to be legally bound to the international community, not just the MILF, and by an
equally clear indication that the signatures of the participating states-representatives would
constitute an acceptance of that commitment. Entering into such a formal agreement would not have
resulted in a loss of face for the Philippine government before the international community, which
was one of the difficulties that prevented the French Government from entering into a formal
agreement with other countries. That the Philippine panel did not enter into such a formal agreement
suggests that it had no intention to be bound to the international community. On that ground, the
MOA-AD may not be considered a unilateral declaration under international law.

The MOA-AD not being a document that can bind the Philippines under international law
notwithstanding, respondents' almost consummated act of guaranteeing amendments to the legal
framework is, by itself, sufficient to constitute grave abuse of discretion. The grave abuse lies
not in the fact that they considered, as a solution to the Moro Problem, the creation of a state within
a state, but in their brazen willingness toguarantee that Congress and the sovereign Filipino
people would give their imprimatur to their solution. Upholding such an act would amount to
authorizing a usurpation of the constituent powers vested only in Congress, a Constitutional
Convention, or the people themselves through the process of initiative, for the only way that the
Executive can ensure the outcome of the amendment process is through an undue influence or
interference with that process.
The sovereign people may, if it so desired, go to the extent of giving up a portion of its own territory
to the Moros for the sake of peace, for it can change the Constitution in any it wants, so long as the
change is not inconsistent with what, in international law, is known as Jus Cogens.184 Respondents,
however, may not preempt it in that decision.
SUMMARY
The petitions are ripe for adjudication. The failure of respondents to consult the local government
units or communities affected constitutes a departure by respondents from their mandate under E.O.
No. 3. Moreover, respondents exceeded their authority by the mere act of guaranteeing
amendments to the Constitution. Any alleged violation of the Constitution by any branch of
government is a proper matter for judicial review.
As the petitions involve constitutional issues which are of paramount public interest or of
transcendental importance, the Court grants the petitioners, petitioners-in-intervention and
intervening respondents the requisitelocus standi in keeping with the liberal stance adopted in David
v. Macapagal-Arroyo.
Contrary to the assertion of respondents that the non-signing of the MOA-AD and the eventual
dissolution of the GRP Peace Panel mooted the present petitions, the Court finds that the present
petitions provide an exception to the "moot and academic" principle in view of (a) the grave violation
of the Constitution involved; (b) the exceptional character of the situation and paramount public
interest; (c) the need to formulate controlling principles to guide the bench, the bar, and the public;
and (d) the fact that the case is capable of repetition yet evading review.
The MOA-AD is a significant part of a series of agreements necessary to carry out the GRP-MILF
Tripoli Agreement on Peace signed by the government and the MILF back in June 2001. Hence, the
present MOA-AD can be renegotiated or another one drawn up that could contain similar or
significantly dissimilar provisions compared to the original.
The Court, however, finds that the prayers for mandamus have been rendered moot in view of the
respondents' action in providing the Court and the petitioners with the official copy of the final draft of
the MOA-AD and its annexes.
The people's right to information on matters of public concern under Sec. 7, Article III of the
Constitution is insplendid symmetry with the state policy of full public disclosure of all its transactions
involving public interest under Sec. 28, Article II of the Constitution. The right to information
guarantees the right of the people to demand information, while Section 28 recognizes the duty of
officialdom to give information even if nobody demands. The complete and effective exercise of the
right to information necessitates that its complementary provision on public disclosure derive the

same self-executory nature, subject only to reasonable safeguards or limitations as may be provided
by law.
The contents of the MOA-AD is a matter of paramount public concern involving public interest in the
highest order. In declaring that the right to information contemplates steps and negotiations leading
to the consummation of the contract, jurisprudence finds no distinction as to the executory nature or
commercial character of the agreement.
An essential element of these twin freedoms is to keep a continuing dialogue or process of
communication between the government and the people. Corollary to these twin rights is the design
for feedback mechanisms. The right to public consultation was envisioned to be a species of these
public rights.
At least three pertinent laws animate these constitutional imperatives and justify the exercise of the
people's right to be consulted on relevant matters relating to the peace agenda.
One, E.O. No. 3 itself is replete with mechanics for continuing consultations on both national and
local levels and for a principal forum for consensus-building. In fact, it is the duty of the Presidential
Adviser on the Peace Process to conduct regular dialogues to seek relevant information, comments,
advice, and recommendations from peace partners and concerned sectors of society.
Two, Republic Act No. 7160 or the Local Government Code of 1991 requires all national offices to
conduct consultations before any project or program critical to the environment and human ecology
including those that may call for the eviction of a particular group of people residing in such locality,
is implemented therein. The MOA-AD is one peculiar program that unequivocally and unilaterally
vests ownership of a vast territory to the Bangsamoro people, which could pervasively and
drastically result to the diaspora or displacement of a great number of inhabitants from their total
environment.
Three, Republic Act No. 8371 or the Indigenous Peoples Rights Act of 1997 provides for clear-cut
procedure for the recognition and delineation of ancestral domain, which entails, among other things,
the observance of the free and prior informed consent of the Indigenous Cultural
Communities/Indigenous Peoples. Notably, the statute does not grant the Executive Department or
any government agency the power to delineate and recognize an ancestral domain claim by mere
agreement or compromise.
The invocation of the doctrine of executive privilege as a defense to the general right to information
or the specific right to consultation is untenable. The various explicit legal provisions fly in the face of
executive secrecy. In any event, respondents effectively waived such defense after it unconditionally
disclosed the official copies of the final draft of the MOA-AD, for judicial compliance and public
scrutiny.
In sum, the Presidential Adviser on the Peace Process committed grave abuse of discretion when he
failed to carry out the pertinent consultation process, as mandated by E.O. No. 3, Republic Act No.
7160, and Republic Act No. 8371. The furtive process by which the MOA-AD was designed and
crafted runs contrary to and in excess of the legal authority, and amounts to a whimsical, capricious,
oppressive, arbitrary and despotic exercise thereof. It illustrates a gross evasion of positive duty and
a virtual refusal to perform the duty enjoined.
The MOA-AD cannot be reconciled with the present Constitution and laws. Not only its specific
provisions but the very concept underlying them, namely, the associative relationship envisioned

between the GRP and the BJE, areunconstitutional, for the concept presupposes that the
associated entity is a state and implies that the same is on its way to independence.
While there is a clause in the MOA-AD stating that the provisions thereof inconsistent with the
present legal framework will not be effective until that framework is amended, the same does not
cure its defect. The inclusion of provisions in the MOA-AD establishing an associative relationship
between the BJE and the Central Government is, itself, a violation of the Memorandum of
Instructions From The President dated March 1, 2001, addressed to the government peace panel.
Moreover, as the clause is worded, it virtually guarantees that the necessary amendments to the
Constitution and the laws will eventually be put in place. Neither the GRP Peace Panel nor the
President herself is authorized to make such a guarantee. Upholding such an act would amount to
authorizing a usurpation of the constituent powers vested only in Congress, a Constitutional
Convention, or the people themselves through the process of initiative, for the only way that the
Executive can ensure the outcome of the amendment process is through an undue influence or
interference with that process.
While the MOA-AD would not amount to an international agreement or unilateral declaration binding
on the Philippines under international law, respondents' act of guaranteeing amendments is, by itself,
already a constitutional violation that renders the MOA-AD fatally defective.
WHEREFORE, respondents' motion to dismiss is DENIED. The main and intervening petitions are
GIVEN DUE COURSE and hereby GRANTED.
The Memorandum of Agreement on the Ancestral Domain Aspect of the GRP-MILF Tripoli
Agreement on Peace of 2001 is declared contrary to law and the Constitution.
SO ORDERED.

Republic of the Philippines


SUPREME COURT
Manila
EN BANC
G.R. No. 104768

July 21, 2003

REPUBLIC OF THE PHILIPPINES, Petitioner,


vs.
SANDIGANBAYAN, MAJOR GENERAL JOSEPHUS Q. RAMAS and ELIZABETH
DIMAANO, Respondents.
DECISION
CARPIO, J.:

The Case
Before this Court is a petition for review on certiorari seeking to set aside the Resolutions of the
Sandiganbayan (First Division)1 dated 18 November 1991 and 25 March 1992 in Civil Case No.
0037. The first Resolution dismissed petitioners Amended Complaint and ordered the return of the
confiscated items to respondent Elizabeth Dimaano, while the second Resolution denied petitioners
Motion for Reconsideration. Petitioner prays for the grant of the reliefs sought in its Amended
Complaint, or in the alternative, for the remand of this case to the Sandiganbayan (First Division) for
further proceedings allowing petitioner to complete the presentation of its evidence.
Antecedent Facts
Immediately upon her assumption to office following the successful EDSA Revolution, then President
Corazon C. Aquino issued Executive Order No. 1 ("EO No. 1") creating the Presidential Commission
on Good Government ("PCGG"). EO No. 1 primarily tasked the PCGG to recover all ill-gotten wealth
of former President Ferdinand E. Marcos, his immediate family, relatives, subordinates and close
associates. EO No. 1 vested the PCGG with the power "(a) to conduct investigation as may be
necessary in order to accomplish and carry out the purposes of this order" and the power "(h) to
promulgate such rules and regulations as may be necessary to carry out the purpose of this order."
Accordingly, the PCGG, through its then Chairman Jovito R. Salonga, created an AFP Anti-Graft
Board ("AFP Board") tasked to investigate reports of unexplained wealth and corrupt practices by
AFP personnel, whether in the active service or retired.2
Based on its mandate, the AFP Board investigated various reports of alleged unexplained wealth of
respondent Major General Josephus Q. Ramas ("Ramas"). On 27 July 1987, the AFP Board issued
a Resolution on its findings and recommendation on the reported unexplained wealth of Ramas. The
relevant part of the Resolution reads:
III. FINDINGS and EVALUATION:
Evidence in the record showed that respondent is the owner of a house and lot located at 15-Yakan
St., La Vista, Quezon City. He is also the owner of a house and lot located in Cebu City. The lot has
an area of 3,327 square meters.
The value of the property located in Quezon City may be estimated modestly at P700,000.00.
The equipment/items and communication facilities which were found in the premises of Elizabeth
Dimaano and were confiscated by elements of the PC Command of Batangas were all covered by
invoice receipt in the name of CAPT. EFREN SALIDO, RSO Command Coy, MSC, PA. These items
could not have been in the possession of Elizabeth Dimaano if not given for her use by respondent
Commanding General of the Philippine Army.
Aside from the military equipment/items and communications equipment, the raiding team was also
able to confiscate money in the amount of P2,870,000.00 and $50,000 US Dollars in the house of
Elizabeth Dimaano on 3 March 1986.

Affidavits of members of the Military Security Unit, Military Security Command, Philippine Army,
stationed at Camp Eldridge, Los Baos, Laguna, disclosed that Elizabeth Dimaano is the mistress of
respondent. That respondent usually goes and stays and sleeps in the alleged house of Elizabeth
Dimaano in Barangay Tengga, Itaas, Batangas City and when he arrives, Elizabeth Dimaano
embraces and kisses respondent. That on February 25, 1986, a person who rode in a car went to
the residence of Elizabeth Dimaano with four (4) attache cases filled with money and owned by
MGen Ramas.
Sworn statement in the record disclosed also that Elizabeth Dimaano had no visible means of
income and is supported by respondent for she was formerly a mere secretary.
Taking in toto the evidence, Elizabeth Dimaano could not have used the military equipment/items
seized in her house on March 3, 1986 without the consent of respondent, he being the Commanding
General of the Philippine Army. It is also impossible for Elizabeth Dimaano to claim that she owns
the P2,870,000.00 and $50,000 US Dollars for she had no visible source of income.
This money was never declared in the Statement of Assets and Liabilities of respondent. There was
an intention to cover the existence of these money because these are all ill-gotten and unexplained
wealth. Were it not for the affidavits of the members of the Military Security Unit assigned at Camp
Eldridge, Los Baos, Laguna, the existence and ownership of these money would have never been
known.
The Statement of Assets and Liabilities of respondent were also submitted for scrutiny and analysis
by the Boards consultant. Although the amount of P2,870,000.00 and $50,000 US Dollars were not
included, still it was disclosed that respondent has an unexplained wealth of P104,134. 60.
IV. CONCLUSION:
In view of the foregoing, the Board finds that a prima facie case exists against respondent for illgotten and unexplained wealth in the amount of P2,974,134.00 and $50,000 US Dollars.
V. RECOMMENDATION:
Wherefore it is recommended that Maj. Gen. Josephus Q. Ramas (ret.) be prosecuted and tried for
violation of RA 3019, as amended, otherwise known as "Anti-Graft and Corrupt Practices Act" and
RA 1379, as amended, otherwise known as "The Act for the Forfeiture of Unlawfully Acquired
Property."3
Thus, on 1 August 1987, the PCGG filed a petition for forfeiture under Republic Act No. 1379 ("RA
No. 1379") 4against Ramas.
Before Ramas could answer the petition, then Solicitor General Francisco I. Chavez filed an
Amended Complaint naming the Republic of the Philippines ("petitioner"), represented by the PCGG,
as plaintiff and Ramas as defendant. The Amended Complaint also impleaded Elizabeth Dimaano
("Dimaano") as co-defendant.

The Amended Complaint alleged that Ramas was the Commanding General of the Philippine Army
until 1986. On the other hand, Dimaano was a confidential agent of the Military Security Unit,
Philippine Army, assigned as a clerk-typist at the office of Ramas from 1 January 1978 to February
1979. The Amended Complaint further alleged that Ramas "acquired funds, assets and properties
manifestly out of proportion to his salary as an army officer and his other income from legitimately
acquired property by taking undue advantage of his public office and/or using his power, authority
and influence as such officer of the Armed Forces of the Philippines and as a subordinate and close
associate of the deposed President Ferdinand Marcos." 5
The Amended Complaint also alleged that the AFP Board, after a previous inquiry, found reasonable
ground to believe that respondents have violated RA No. 1379. 6 The Amended Complaint prayed for,
among others, the forfeiture of respondents properties, funds and equipment in favor of the State.
Ramas filed an Answer with Special and/or Affirmative Defenses and Compulsory Counterclaim to
the Amended Complaint. In his Answer, Ramas contended that his property consisted only of a
residential house at La Vista Subdivision, Quezon City, valued at P700,000, which was not out of
proportion to his salary and other legitimate income. He denied ownership of any mansion in Cebu
City and the cash, communications equipment and other items confiscated from the house of
Dimaano.
Dimaano filed her own Answer to the Amended Complaint. Admitting her employment as a clerktypist in the office of Ramas from January-November 1978 only, Dimaano claimed ownership of the
monies, communications equipment, jewelry and land titles taken from her house by the Philippine
Constabulary raiding team.
After termination of the pre-trial,7 the court set the case for trial on the merits on 9-11 November
1988.
On 9 November 1988, petitioner asked for a deferment of the hearing due to its lack of preparation
for trial and the absence of witnesses and vital documents to support its case. The court reset the
hearing to 17 and 18 April 1989.
On 13 April 1989, petitioner filed a motion for leave to amend the complaint in order "to charge the
delinquent properties with being subject to forfeiture as having been unlawfully acquired by
defendant Dimaano alone x x x."8
Nevertheless, in an order dated 17 April 1989, the Sandiganbayan proceeded with petitioners
presentation of evidence on the ground that the motion for leave to amend complaint did not state
when petitioner would file the amended complaint. The Sandiganbayan further stated that the
subject matter of the amended complaint was on its face vague and not related to the existing
complaint. The Sandiganbayan also held that due to the time that the case had been pending in
court, petitioner should proceed to present its evidence.
After presenting only three witnesses, petitioner asked for a postponement of the trial.

On 28 September 1989, during the continuation of the trial, petitioner manifested its inability to
proceed to trial because of the absence of other witnesses or lack of further evidence to present.
Instead, petitioner reiterated its motion to amend the complaint to conform to the evidence already
presented or to change the averments to show that Dimaano alone unlawfully acquired the monies
or properties subject of the forfeiture.
The Sandiganbayan noted that petitioner had already delayed the case for over a year mainly
because of its many postponements. Moreover, petitioner would want the case to revert to its
preliminary stage when in fact the case had long been ready for trial. The Sandiganbayan ordered
petitioner to prepare for presentation of its additional evidence, if any.
During the trial on 23 March 1990, petitioner again admitted its inability to present further evidence.
Giving petitioner one more chance to present further evidence or to amend the complaint to conform
to its evidence, the Sandiganbayan reset the trial to 18 May 1990. The Sandiganbayan, however,
hinted that the re-setting was without prejudice to any action that private respondents might take
under the circumstances.
However, on 18 May 1990, petitioner again expressed its inability to proceed to trial because it had
no further evidence to present. Again, in the interest of justice, the Sandiganbayan granted petitioner
60 days within which to file an appropriate pleading. The Sandiganbayan, however, warned
petitioner that failure to act would constrain the court to take drastic action.
Private respondents then filed their motions to dismiss based on Republic v. Migrino. 9 The Court held
in Migrino that the PCGG does not have jurisdiction to investigate and prosecute military officers by
reason of mere position held without a showing that they are "subordinates" of former President
Marcos.
On 18 November 1991, the Sandiganbayan rendered a resolution, the dispositive portion of which
states:
WHEREFORE, judgment is hereby rendered dismissing the Amended Complaint, without
pronouncement as to costs. The counterclaims are likewise dismissed for lack of merit, but the
confiscated sum of money, communications equipment, jewelry and land titles are ordered returned
to Elizabeth Dimaano.
The records of this case are hereby remanded and referred to the Hon. Ombudsman, who has
primary jurisdiction over the forfeiture cases under R.A. No. 1379, for such appropriate action as the
evidence warrants. This case is also referred to the Commissioner of the Bureau of Internal
Revenue for a determination of any tax liability of respondent Elizabeth Dimaano in connection
herewith.
SO ORDERED.
On 4 December 1991, petitioner filed its Motion for Reconsideration.

In answer to the Motion for Reconsideration, private respondents filed a Joint Comment/Opposition
to which petitioner filed its Reply on 10 January 1992.
On 25 March 1992, the Sandiganbayan rendered a Resolution denying the Motion for
Reconsideration.
Ruling of the Sandiganbayan
The Sandiganbayan dismissed the Amended Complaint on the following grounds:
(1.) The actions taken by the PCGG are not in accordance with the rulings of the Supreme
Court in Cruz, Jr. v. Sandiganbayan10 and Republic v. Migrino11 which involve the same
issues.
(2.) No previous inquiry similar to preliminary investigations in criminal cases was conducted
against Ramas and Dimaano.
(3.) The evidence adduced against Ramas does not constitute a prima facie case against
him.
(4.) There was an illegal search and seizure of the items confiscated.
The Issues
Petitioner raises the following issues:
A. RESPONDENT COURT SERIOUSLY ERRED IN CONCLUDING THAT PETITIONERS
EVIDENCE CANNOT MAKE A CASE FOR FORFEITURE AND THAT THERE WAS NO
SHOWING OF CONSPIRACY, COLLUSION OR RELATIONSHIP BY CONSANGUINITY OR
AFFINITY BY AND BETWEEN RESPONDENT RAMAS AND RESPONDENT DIMAANO
NOTWITHSTANDING THE FACT THAT SUCH CONCLUSIONS WERE CLEARLY
UNFOUNDED AND PREMATURE, HAVING BEEN RENDERED PRIOR TO THE
COMPLETION OF THE PRESENTATION OF THE EVIDENCE OF THE PETITIONER.
B. RESPONDENT COURT SERIOUSLY ERRED IN HOLDING THAT THE ACTIONS TAKEN
BY THE PETITIONER, INCLUDING THE FILING OF THE ORIGINAL COMPLAINT AND
THE AMENDED COMPLAINT, SHOULD BE STRUCK OUT IN LINE WITH THE RULINGS
OF THE SUPREME COURT IN CRUZ, JR. v. SANDIGANBAYAN, 194 SCRA 474 AND
REPUBLIC v. MIGRINO, 189 SCRA 289, NOTWITHSTANDING THE FACT THAT:
1. The cases of Cruz, Jr. v. Sandiganbayan, supra, and Republic v. Migrino, supra,
are clearly not applicable to this case;
2. Any procedural defect in the institution of the complaint in Civil Case No. 0037 was
cured and/or waived by respondents with the filing of their respective answers with
counterclaim; and

3. The separate motions to dismiss were evidently improper considering that they
were filed after commencement of the presentation of the evidence of the petitioner
and even before the latter was allowed to formally offer its evidence and rest its case;
C. RESPONDENT COURT SERIOUSLY ERRED IN HOLDING THAT THE ARTICLES AND
THINGS SUCH AS SUMS OF MONEY, COMMUNICATIONS EQUIPMENT, JEWELRY AND
LAND TITLES CONFISCATED FROM THE HOUSE OF RESPONDENT DIMAANO WERE
ILLEGALLY SEIZED AND THEREFORE EXCLUDED AS EVIDENCE.12
The Courts Ruling
First Issue: PCGGs Jurisdiction to Investigate Private Respondents
This case involves a revisiting of an old issue already decided by this Court in Cruz, Jr. v.
Sandiganbayan13 and Republic v. Migrino.14
The primary issue for resolution is whether the PCGG has the jurisdiction to investigate and cause
the filing of a forfeiture petition against Ramas and Dimaano for unexplained wealth under RA No.
1379.
We hold that PCGG has no such jurisdiction.
The PCGG created the AFP Board to investigate the unexplained wealth and corrupt practices of
AFP personnel, whether in the active service or retired.15 The PCGG tasked the AFP Board to make
the necessary recommendations to appropriate government agencies on the action to be taken
based on its findings.16 The PCGG gave this task to the AFP Board pursuant to the PCGGs power
under Section 3 of EO No. 1 "to conduct investigation as may be necessary in order to accomplish
and to carry out the purposes of this order." EO No. 1 gave the PCGG specific responsibilities, to wit:
SEC. 2. The Commission shall be charged with the task of assisting the President in regard to the
following matters:
(a) The recovery of all ill-gotten wealth accumulated by former President Ferdinand E. Marcos, his
immediate family, relatives, subordinates and close associates, whether located in the Philippines or
abroad, including the takeover and sequestration of all business enterprises and entities owned or
controlled by them, during his administration, directly or through nominees, by taking undue
advantage of their public office and/ or using their powers, authority, influence, connections or
relationship.
(b) The investigation of such cases of graft and corruption as the President may assign to the
Commission from time to time.
x x x.
The PCGG, through the AFP Board, can only investigate the unexplained wealth and corrupt
practices of AFP personnel who fall under either of the two categories mentioned in Section 2 of EO

No. 1. These are: (1) AFP personnel who have accumulated ill-gotten wealth during the
administration of former President Marcos by being the latters immediate family, relative,
subordinate or close associate, taking undue advantage of their public office or using their powers,
influence x x x;17 or (2) AFP personnel involved in other cases of graft and corruption provided the
President assigns their cases to the PCGG.18
Petitioner, however, does not claim that the President assigned Ramas case to the PCGG.
Therefore, Ramas case should fall under the first category of AFP personnel before the PCGG could
exercise its jurisdiction over him. Petitioner argues that Ramas was undoubtedly a subordinate of
former President Marcos because of his position as the Commanding General of the Philippine
Army. Petitioner claims that Ramas position enabled him to receive orders directly from his
commander-in-chief, undeniably making him a subordinate of former President Marcos.
We hold that Ramas was not a "subordinate" of former President Marcos in the sense contemplated
under EO No. 1 and its amendments.
Mere position held by a military officer does not automatically make him a "subordinate" as this term
is used in EO Nos. 1, 2, 14 and 14-A absent a showing that he enjoyed close association with former
President Marcos. Migrino discussed this issue in this wise:
A close reading of EO No. 1 and related executive orders will readily show what is contemplated
within the term subordinate. The Whereas Clauses of EO No. 1 express the urgent need to recover
the ill-gotten wealth amassed by former President Ferdinand E. Marcos, his immediate family,
relatives, and close associates both here and abroad.
EO No. 2 freezes all assets and properties in the Philippines in which former President Marcos
and/or his wife, Mrs. Imelda Marcos, their close relatives, subordinates, business associates,
dummies, agents, or nominees have any interest or participation.
Applying the rule in statutory construction known as ejusdem generis that is[W]here general words follow an enumeration of persons or things by words of a particular and
specific meaning, such general words are not to be construed in their widest extent, but are to be
held as applying only to persons or things of the same kind or class as those specifically mentioned
[Smith, Bell & Co, Ltd. vs. Register of Deeds of Davao, 96 Phil. 53, 58, citing Black on Interpretation
of Laws, 2nd Ed., 203].
[T]he term "subordinate" as used in EO Nos. 1 & 2 refers to one who enjoys a close association with
former President Marcos and/or his wife, similar to the immediate family member, relative, and close
associate in EO No. 1 and the close relative, business associate, dummy, agent, or nominee in EO
No. 2.
xxx
It does not suffice, as in this case, that the respondent is or was a government official or employee
during the administration of former President Marcos. There must be a prima facie showing that the

respondent unlawfully accumulated wealth by virtue of his close association or relation with former
Pres. Marcos and/or his wife. (Emphasis supplied)
Ramas position alone as Commanding General of the Philippine Army with the rank of Major
General19 does not suffice to make him a "subordinate" of former President Marcos for purposes of
EO No. 1 and its amendments. The PCGG has to provide a prima facie showing that Ramas was a
close associate of former President Marcos, in the same manner that business associates,
dummies, agents or nominees of former President Marcos were close to him. Such close association
is manifested either by Ramas complicity with former President Marcos in the accumulation of illgotten wealth by the deposed President or by former President Marcos acquiescence in Ramas
own accumulation of ill-gotten wealth if any.
This, the PCGG failed to do.
Petitioners attempt to differentiate the instant case from Migrino does not convince us. Petitioner
argues that unlike in Migrino, the AFP Board Resolution in the instant case states that the AFP Board
conducted the investigation pursuant to EO Nos. 1, 2, 14 and 14-A in relation to RA No. 1379.
Petitioner asserts that there is a presumption that the PCGG was acting within its jurisdiction of
investigating crony-related cases of graft and corruption and that Ramas was truly a subordinate of
the former President. However, the same AFP Board Resolution belies this contention. Although the
Resolution begins with such statement, it ends with the following recommendation:
V. RECOMMENDATION:
Wherefore it is recommended that Maj. Gen. Josephus Q. Ramas (ret.) be prosecuted and tried for
violation of RA 3019, as amended, otherwise known as "Anti-Graft and Corrupt Practices Act" and
RA 1379, as amended, otherwise known as "The Act for the Forfeiture of Unlawfully Acquired
Property."20
Thus, although the PCGG sought to investigate and prosecute private respondents under EO Nos.
1, 2, 14 and 14-A, the result yielded a finding of violation of Republic Acts Nos. 3019 and 1379
without any relation to EO Nos. 1, 2, 14 and 14-A. This absence of relation to EO No. 1 and its
amendments proves fatal to petitioners case. EO No. 1 created the PCGG for a specific and limited
purpose, and necessarily its powers must be construed to address such specific and limited
purpose.
Moreover, the resolution of the AFP Board and even the Amended Complaint do not show that the
properties Ramas allegedly owned were accumulated by him in his capacity as a "subordinate" of
his commander-in-chief. Petitioner merely enumerated the properties Ramas allegedly owned and
suggested that these properties were disproportionate to his salary and other legitimate income
without showing that Ramas amassed them because of his close association with former President
Marcos. Petitioner, in fact, admits that the AFP Board resolution does not contain a finding that
Ramas accumulated his wealth because of his close association with former President Marcos, thus:
10. While it is true that the resolution of the Anti-Graft Board of the New Armed Forces of the
Philippines did not categorically find a prima facie evidence showing that respondent Ramas

unlawfully accumulated wealth by virtue of his close association or relation with former
President Marcos and/or his wife, it is submitted that such omission was not fatal. The
resolution of the Anti-Graft Board should be read in the context of the law creating the same and the
objective of the investigation which was, as stated in the above, pursuant to Republic Act Nos. 3019
and 1379 in relation to Executive Order Nos. 1, 2, 14 and 14-a;21(Emphasis supplied)
Such omission is fatal. Petitioner forgets that it is precisely a prima facie showing that the ill-gotten
wealth was accumulated by a "subordinate" of former President Marcos that vests jurisdiction on
PCGG. EO No. 122 clearly premises the creation of the PCGG on the urgent need to recover all illgotten wealth amassed by former President Marcos, his immediate family, relatives, subordinates
and close associates. Therefore, to say that such omission was not fatal is clearly contrary to the
intent behind the creation of the PCGG.
In Cruz, Jr. v. Sandiganbayan,23 the Court outlined the cases that fall under the jurisdiction of the
PCGG pursuant to EO Nos. 1, 2,24 14,25 14-A:26
A careful reading of Sections 2(a) and 3 of Executive Order No. 1 in relation with Sections 1, 2 and 3
of Executive Order No. 14, shows what the authority of the respondent PCGG to investigate and
prosecute covers:
(a) the investigation and prosecution of the civil action for the recovery of ill-gotten wealth
under Republic Act No. 1379, accumulated by former President Marcos, his immediate
family, relatives, subordinates and close associates, whether located in the Philippines or
abroad, including the take-over or sequestration of all business enterprises and entities
owned or controlled by them, during his administration, directly or through his nominees, by
taking undue advantage of their public office and/or using their powers, authority and
influence, connections or relationships; and
(b) the investigation and prosecution of such offenses committed in the acquisition of said illgotten wealth as contemplated under Section 2(a) of Executive Order No. 1.
However, other violations of the Anti-Graft and Corrupt Practices Act not otherwise falling
under the foregoing categories, require a previous authority of the President for the
respondent PCGG to investigate and prosecute in accordance with Section 2 (b) of Executive
Order No. 1. Otherwise, jurisdiction over such cases is vested in the Ombudsman and other
duly authorized investigating agencies such as the provincial and city prosecutors, their
assistants, the Chief State Prosecutor and his assistants and the state
prosecutors. (Emphasis supplied)
The proper government agencies, and not the PCGG, should investigate and prosecute forfeiture
petitions not falling under EO No. 1 and its amendments. The preliminary investigation of
unexplained wealth amassed on or before 25 February 1986 falls under the jurisdiction of the
Ombudsman, while the authority to file the corresponding forfeiture petition rests with the Solicitor
General.27 The Ombudsman Act or Republic Act No. 6770 ("RA No. 6770") vests in the Ombudsman
the power to conduct preliminary investigation and to file forfeiture proceedings involving
unexplained wealth amassed after 25 February 1986. 28

After the pronouncements of the Court in Cruz, the PCGG still pursued this case despite the
absence of a prima facie finding that Ramas was a "subordinate" of former President Marcos. The
petition for forfeiture filed with the Sandiganbayan should be dismissed for lack of authority by the
PCGG to investigate respondents since there is no prima facie showing that EO No. 1 and its
amendments apply to respondents. The AFP Board Resolution and even the Amended Complaint
state that there are violations of RA Nos. 3019 and 1379. Thus, the PCGG should have
recommended Ramas case to the Ombudsman who has jurisdiction to conduct the preliminary
investigation of ordinary unexplained wealth and graft cases. As stated in Migrino:
[But] in view of the patent lack of authority of the PCGG to investigate and cause the prosecution of
private respondent for violation of Rep. Acts Nos. 3019 and 1379, the PCGG must also be enjoined
from proceeding with the case, without prejudice to any action that may be taken by the proper
prosecutory agency. The rule of law mandates that an agency of government be allowed to exercise
only the powers granted to it.
Petitioners argument that private respondents have waived any defect in the filing of the forfeiture
petition by submitting their respective Answers with counterclaim deserves no merit as well.
Petitioner has no jurisdiction over private respondents. Thus, there is no jurisdiction to waive in the
first place. The PCGG cannot exercise investigative or prosecutorial powers never granted to it.
PCGGs powers are specific and limited. Unless given additional assignment by the President,
PCGGs sole task is only to recover the ill-gotten wealth of the Marcoses, their relatives and
cronies.29 Without these elements, the PCGG cannot claim jurisdiction over a case.
Private respondents questioned the authority and jurisdiction of the PCGG to investigate and
prosecute their cases by filing their Motion to Dismiss as soon as they learned of the pronouncement
of the Court in Migrino. This case was decided on 30 August 1990, which explains why private
respondents only filed their Motion to Dismiss on 8 October 1990. Nevertheless, we have held that
the parties may raise lack of jurisdiction at any stage of the proceeding. 30 Thus, we hold that there
was no waiver of jurisdiction in this case. Jurisdiction is vested by law and not by the parties to an
action.31
Consequently, the petition should be dismissed for lack of jurisdiction by the PCGG to conduct the
preliminary investigation. The Ombudsman may still conduct the proper preliminary investigation for
violation of RA No. 1379, and if warranted, the Solicitor General may file the forfeiture petition with
the Sandiganbayan.32 The right of the State to forfeit unexplained wealth under RA No. 1379 is not
subject to prescription, laches or estoppel.33
Second Issue: Propriety of Dismissal of Case
Before Completion of Presentation of Evidence
Petitioner also contends that the Sandiganbayan erred in dismissing the case before completion of
the presentation of petitioners evidence.
We disagree.

Based on the findings of the Sandiganbayan and the records of this case, we find that petitioner has
only itself to blame for non-completion of the presentation of its evidence. First, this case has been
pending for four years before the Sandiganbayan dismissed it. Petitioner filed its Amended
Complaint on 11 August 1987, and only began to present its evidence on 17 April 1989. Petitioner
had almost two years to prepare its evidence. However, despite this sufficient time, petitioner still
delayed the presentation of the rest of its evidence by filing numerous motions for postponements
and extensions. Even before the date set for the presentation of its evidence, petitioner filed, on 13
April 1989, a Motion for Leave to Amend the Complaint.34 The motion sought "to charge the
delinquent properties (which comprise most of petitioners evidence) with being subject to forfeiture
as having been unlawfully acquired by defendant Dimaano alone x x x."
The Sandiganbayan, however, refused to defer the presentation of petitioners evidence since
petitioner did not state when it would file the amended complaint. On 18 April 1989, the
Sandiganbayan set the continuation of the presentation of evidence on 28-29 September and 9-11
October 1989, giving petitioner ample time to prepare its evidence. Still, on 28 September 1989,
petitioner manifested its inability to proceed with the presentation of its evidence. The
Sandiganbayan issued an Order expressing its view on the matter, to wit:
The Court has gone through extended inquiry and a narration of the above events because this case
has been ready for trial for over a year and much of the delay hereon has been due to the inability of
the government to produce on scheduled dates for pre-trial and for trial documents and witnesses,
allegedly upon the failure of the military to supply them for the preparation of the presentation of
evidence thereon. Of equal interest is the fact that this Court has been held to task in public about its
alleged failure to move cases such as this one beyond the preliminary stage, when, in view of the
developments such as those of today, this Court is now faced with a situation where a case already
in progress will revert back to the preliminary stage, despite a five-month pause where appropriate
action could have been undertaken by the plaintiff Republic. 35
On 9 October 1989, the PCGG manifested in court that it was conducting a preliminary investigation
on the unexplained wealth of private respondents as mandated by RA No. 1379. 36 The PCGG prayed
for an additional four months to conduct the preliminary investigation. The Sandiganbayan granted
this request and scheduled the presentation of evidence on 26-29 March 1990. However, on the
scheduled date, petitioner failed to inform the court of the result of the preliminary investigation the
PCGG supposedly conducted. Again, the Sandiganbayan gave petitioner until 18 May 1990 to
continue with the presentation of its evidence and to inform the court of "what lies ahead insofar as
the status of the case is concerned x x x."37 Still on the date set, petitioner failed to present its
evidence. Finally, on 11 July 1990, petitioner filed its Re-Amended Complaint. 38 The Sandiganbayan
correctly observed that a case already pending for years would revert to its preliminary stage if the
court were to accept the Re-Amended Complaint.
Based on these circumstances, obviously petitioner has only itself to blame for failure to complete
the presentation of its evidence. The Sandiganbayan gave petitioner more than sufficient time to
finish the presentation of its evidence. The Sandiganbayan overlooked petitioners delays and yet
petitioner ended the long-string of delays with the filing of a Re-Amended Complaint, which would
only prolong even more the disposition of the case.

Moreover, the pronouncements of the Court in Migrino and Cruz prompted the Sandiganbayan to
dismiss the case since the PCGG has no jurisdiction to investigate and prosecute the case against
private respondents. This alone would have been sufficient legal basis for the Sandiganbayan to
dismiss the forfeiture case against private respondents.
Thus, we hold that the Sandiganbayan did not err in dismissing the case before completion of the
presentation of petitioners evidence.
Third Issue: Legality of the Search and Seizure
Petitioner claims that the Sandiganbayan erred in declaring the properties confiscated from
Dimaanos house as illegally seized and therefore inadmissible in evidence. This issue bears a
significant effect on petitioners case since these properties comprise most of petitioners evidence
against private respondents. Petitioner will not have much evidence to support its case against
private respondents if these properties are inadmissible in evidence.
On 3 March 1986, the Constabulary raiding team served at Dimaanos residence a search warrant
captioned "Illegal Possession of Firearms and Ammunition." Dimaano was not present during the
raid but Dimaanos cousins witnessed the raid. The raiding team seized the items detailed in the
seizure receipt together with other items not included in the search warrant. The raiding team seized
these items: one baby armalite rifle with two magazines; 40 rounds of 5.56 ammunition; one pistol,
caliber .45; communications equipment, cash consisting of P2,870,000 and US$50,000, jewelry, and
land titles.
Petitioner wants the Court to take judicial notice that the raiding team conducted the search and
seizure "on March 3, 1986 or five days after the successful EDSA revolution." 39 Petitioner argues that
a revolutionary government was operative at that time by virtue of Proclamation No. 1 announcing
that President Aquino and Vice President Laurel were "taking power in the name and by the will of
the Filipino people."40 Petitioner asserts that the revolutionary government effectively withheld the
operation of the 1973 Constitution which guaranteed private respondents exclusionary right.
Moreover, petitioner argues that the exclusionary right arising from an illegal search applies only
beginning 2 February 1987, the date of ratification of the 1987 Constitution. Petitioner contends that
all rights under the Bill of Rights had already reverted to its embryonic stage at the time of the
search. Therefore, the government may confiscate the monies and items taken from Dimaano and
use the same in evidence against her since at the time of their seizure, private respondents did not
enjoy any constitutional right.
Petitioner is partly right in its arguments.
The EDSA Revolution took place on 23-25 February 1986. As succinctly stated in President Aquinos
Proclamation No. 3 dated 25 March 1986, the EDSA Revolution was "done in defiance of the
provisions of the 1973 Constitution."41 The resulting government was indisputably a revolutionary
government bound by no constitution or legal limitations except treaty obligations that the
revolutionary government, as the de jure government in the Philippines, assumed under international
law.

The correct issues are: (1) whether the revolutionary government was bound by the Bill of Rights of
the 1973 Constitution during the interregnum, that is, after the actual and effective take-over of
power by the revolutionary government following the cessation of resistance by loyalist forces up to
24 March 1986 (immediately before the adoption of the Provisional Constitution); and (2) whether
the protection accorded to individuals under the International Covenant on Civil and Political Rights
("Covenant") and the Universal Declaration of Human Rights ("Declaration") remained in effect
during the interregnum.
We hold that the Bill of Rights under the 1973 Constitution was not operative during the interregnum.
However, we rule that the protection accorded to individuals under the Covenant and the Declaration
remained in effect during the interregnum.
During the interregnum, the directives and orders of the revolutionary government were the supreme
law because no constitution limited the extent and scope of such directives and orders. With the
abrogation of the 1973 Constitution by the successful revolution, there was no municipal law higher
than the directives and orders of the revolutionary government. Thus, during the interregnum, a
person could not invoke any exclusionary right under a Bill of Rights because there was neither a
constitution nor a Bill of Rights during the interregnum. As the Court explained in Letter of Associate
Justice Reynato S. Puno:42
A revolution has been defined as "the complete overthrow of the established government in any
country or state by those who were previously subject to it" or as "a sudden, radical and fundamental
change in the government or political system, usually effected with violence or at least some acts of
violence." In Kelsen's book, General Theory of Law and State, it is defined as that which "occurs
whenever the legal order of a community is nullified and replaced by a new order . . . a way not
prescribed by the first order itself."
It was through the February 1986 revolution, a relatively peaceful one, and more popularly known as
the "people power revolution" that the Filipino people tore themselves away from an existing regime.
This revolution also saw the unprecedented rise to power of the Aquino government.
From the natural law point of view, the right of revolution has been defined as "an inherent right of a
people to cast out their rulers, change their policy or effect radical reforms in their system of
government or institutions by force or a general uprising when the legal and constitutional methods
of making such change have proved inadequate or are so obstructed as to be unavailable." It has
been said that "the locus of positive law-making power lies with the people of the state" and from
there is derived "the right of the people to abolish, to reform and to alter any existing form of
government without regard to the existing constitution."
xxx
It is widely known that Mrs. Aquinos rise to the presidency was not due to constitutional
processes; in fact, it was achieved in violation of the provisions of the 1973 Constitution as a
Batasang Pambansa resolution had earlier declared Mr. Marcos as the winner in the 1986
presidential election. Thus it can be said that the organization of Mrs. Aquinos Government which
was met by little resistance and her control of the state evidenced by the appointment of the Cabinet

and other key officers of the administration, the departure of the Marcos Cabinet officials, revamp of
the Judiciary and the Military signaled the point where the legal system then in effect, had
ceased to be obeyed by the Filipino. (Emphasis supplied)
To hold that the Bill of Rights under the 1973 Constitution remained operative during the interregnum
would render void all sequestration orders issued by the Philippine Commission on Good
Government ("PCGG") before the adoption of the Freedom Constitution. The sequestration orders,
which direct the freezing and even the take-over of private property by mere executive issuance
without judicial action, would violate the due process and search and seizure clauses of the Bill of
Rights.
During the interregnum, the government in power was concededly a revolutionary government
bound by no constitution. No one could validly question the sequestration orders as violative of the
Bill of Rights because there was no Bill of Rights during the interregnum. However, upon the
adoption of the Freedom Constitution, the sequestered companies assailed the sequestration orders
as contrary to the Bill of Rights of the Freedom Constitution.
In Bataan Shipyard & Engineering Co. Inc. vs. Presidential Commission on Good
Government,43 petitioner Baseco, while conceding there was no Bill of Rights during the interregnum,
questioned the continued validity of the sequestration orders upon adoption of the Freedom
Constitution in view of the due process clause in its Bill of Rights. The Court ruled that the Freedom
Constitution, and later the 1987 Constitution, expressly recognized the validity of sequestration
orders, thus:
If any doubt should still persist in the face of the foregoing considerations as to the validity and
propriety of sequestration, freeze and takeover orders, it should be dispelled by the fact that these
particular remedies and the authority of the PCGG to issue them have received constitutional
approbation and sanction. As already mentioned, the Provisional or "Freedom" Constitution
recognizes the power and duty of the President to enact "measures to achieve the mandate of the
people to . . . (r)ecover ill-gotten properties amassed by the leaders and supporters of the previous
regime and protect the interest of the people through orders of sequestration or freezing of assets or
accounts." And as also already adverted to, Section 26, Article XVIII of the 1987 Constitution treats
of, and ratifies the "authority to issue sequestration or freeze orders under Proclamation No. 3 dated
March 25, 1986."
The framers of both the Freedom Constitution and the 1987 Constitution were fully aware that the
sequestration orders would clash with the Bill of Rights. Thus, the framers of both constitutions had
to include specific language recognizing the validity of the sequestration orders. The following
discourse by Commissioner Joaquin G. Bernas during the deliberations of the Constitutional
Commission is instructive:
FR. BERNAS: Madam President, there is something schizophrenic about the arguments in defense
of the present amendment.
For instance, I have carefully studied Minister Salongas lecture in the Gregorio Araneta University
Foundation, of which all of us have been given a copy. On the one hand, he argues that everything

the Commission is doing is traditionally legal. This is repeated by Commissioner Romulo also.
Minister Salonga spends a major portion of his lecture developing that argument. On the other hand,
almost as an afterthought, he says that in the end what matters are the results and not the legal
niceties, thus suggesting that the PCGG should be allowed to make some legal shortcuts, another
word for niceties or exceptions.
Now, if everything the PCGG is doing is legal, why is it asking the CONCOM for special protection?
The answer is clear. What they are doing will not stand the test of ordinary due process, hence they
are asking for protection, for exceptions. Grandes malos, grandes remedios, fine, as the saying
stands, but let us not say grandes malos, grande y malos remedios. That is not an allowable
extrapolation. Hence, we should not give the exceptions asked for, and let me elaborate and give
three reasons:
First, the whole point of the February Revolution and of the work of the CONCOM is to hasten
constitutional normalization. Very much at the heart of the constitutional normalization is the full
effectivity of the Bill of Rights. We cannot, in one breath, ask for constitutional normalization and at
the same time ask for a temporary halt to the full functioning of what is at the heart of
constitutionalism. That would be hypocritical; that would be a repetition of Marcosian protestation of
due process and rule of law. The New Society word for that is "backsliding." It is tragic when we
begin to backslide even before we get there.
Second, this is really a corollary of the first. Habits tend to become ingrained. The committee report
asks for extraordinary exceptions from the Bill of Rights for six months after the convening of
Congress, and Congress may even extend this longer.
Good deeds repeated ripen into virtue; bad deeds repeated become vice. What the committee report
is asking for is that we should allow the new government to acquire the vice of disregarding the Bill
of Rights.
Vices, once they become ingrained, become difficult to shed. The practitioners of the vice begin to
think that they have a vested right to its practice, and they will fight tooth and nail to keep the
franchise. That would be an unhealthy way of consolidating the gains of a democratic revolution.
Third, the argument that what matters are the results and not the legal niceties is an argument that is
very disturbing. When it comes from a staunch Christian like Commissioner Salonga, a Minister, and
repeated verbatim by another staunch Christian like Commissioner Tingson, it becomes doubly
disturbing and even discombobulating. The argument makes the PCGG an auctioneer, placing the
Bill of Rights on the auction block. If the price is right, the search and seizure clause will be sold.
"Open your Swiss bank account to us and we will award you the search and seizure clause. You can
keep it in your private safe."
Alternatively, the argument looks on the present government as hostage to the hoarders of hidden
wealth. The hoarders will release the hidden health if the ransom price is paid and the ransom price
is the Bill of Rights, specifically the due process in the search and seizure clauses. So, there is
something positively revolving about either argument. The Bill of Rights is not for sale to the highest
bidder nor can it be used to ransom captive dollars. This nation will survive and grow strong, only if it

would become convinced of the values enshrined in the Constitution of a price that is beyond
monetary estimation.
For these reasons, the honorable course for the Constitutional Commission is to delete all of Section
8 of the committee report and allow the new Constitution to take effect in full vigor. If Section 8 is
deleted, the PCGG has two options. First, it can pursue the Salonga and the Romulo argument
that what the PCGG has been doing has been completely within the pale of the law. If sustained, the
PCGG can go on and should be able to go on, even without the support of Section 8. If not
sustained, however, the PCGG has only one honorable option, it must bow to the majesty of the Bill
of Rights.
The PCGG extrapolation of the law is defended by staunch Christians. Let me conclude with what
another Christian replied when asked to toy around with the law. From his prison cell, Thomas More
said, "I'll give the devil benefit of law for my nations safety sake." I ask the Commission to give the
devil benefit of law for our nations sake. And we should delete Section 8.
Thank you, Madam President. (Emphasis supplied)
Despite the impassioned plea by Commissioner Bernas against the amendment excepting
sequestration orders from the Bill of Rights, the Constitutional Commission still adopted the
amendment as Section 26,44 Article XVIII of the 1987 Constitution. The framers of the Constitution
were fully aware that absent Section 26, sequestration orders would not stand the test of due
process under the Bill of Rights.
Thus, to rule that the Bill of Rights of the 1973 Constitution remained in force during the interregnum,
absent a constitutional provision excepting sequestration orders from such Bill of Rights, would
clearly render all sequestration orders void during the interregnum. Nevertheless, even during the
interregnum the Filipino people continued to enjoy, under the Covenant and the Declaration, almost
the same rights found in the Bill of Rights of the 1973 Constitution.
The revolutionary government, after installing itself as the de jure government, assumed
responsibility for the States good faith compliance with the Covenant to which the Philippines is a
signatory. Article 2(1) of the Covenant requires each signatory State "to respect and to ensure to all
individuals within its territory and subject to its jurisdiction the rights 45 recognized in the present
Covenant." Under Article 17(1) of the Covenant, the revolutionary government had the duty to insure
that "[n]o one shall be subjected to arbitrary or unlawful interference with his privacy, family, home or
correspondence."
The Declaration, to which the Philippines is also a signatory, provides in its Article 17(2) that "[n]o
one shall be arbitrarily deprived of his property." Although the signatories to the Declaration did not
intend it as a legally binding document, being only a declaration, the Court has interpreted the
Declaration as part of the generally accepted principles of international law and binding on the
State.46 Thus, the revolutionary government was also obligated under international law to observe
the rights47 of individuals under the Declaration.

The revolutionary government did not repudiate the Covenant or the Declaration during the
interregnum. Whether the revolutionary government could have repudiated all its obligations under
the Covenant or the Declaration is another matter and is not the issue here. Suffice it to say that the
Court considers the Declaration as part of customary international law, and that Filipinos as human
beings are proper subjects of the rules of international law laid down in the Covenant. The fact is the
revolutionary government did not repudiate the Covenant or the Declaration in the same way it
repudiated the 1973 Constitution. As the de jure government, the revolutionary government could not
escape responsibility for the States good faith compliance with its treaty obligations under
international law.
It was only upon the adoption of the Provisional Constitution on 25 March 1986 that the directives
and orders of the revolutionary government became subject to a higher municipal law that, if
contravened, rendered such directives and orders void. The Provisional Constitution adopted
verbatim the Bill of Rights of the 1973 Constitution. 48 The Provisional Constitution served as a selflimitation by the revolutionary government to avoid abuses of the absolute powers entrusted to it by
the people.
During the interregnum when no constitution or Bill of Rights existed, directives and orders issued by
government officers were valid so long as these officers did not exceed the authority granted them
by the revolutionary government. The directives and orders should not have also violated the
Covenant or the Declaration. In this case, the revolutionary government presumptively sanctioned
the warrant since the revolutionary government did not repudiate it. The warrant, issued by a judge
upon proper application, specified the items to be searched and seized. The warrant is thus valid
with respect to the items specifically described in the warrant.
However, the Constabulary raiding team seized items not included in the warrant. As admitted by
petitioners witnesses, the raiding team confiscated items not included in the warrant, thus:
Direct Examination of Capt. Rodolfo Sebastian
AJ AMORES
Q. According to the search warrant, you are supposed to seize only for weapons. What else, aside
from the weapons, were seized from the house of Miss Elizabeth Dimaano?
A. The communications equipment, money in Philippine currency and US dollars, some jewelries,
land titles, sir.
Q. Now, the search warrant speaks only of weapons to be seized from the house of Elizabeth
Dimaano. Do you know the reason why your team also seized other properties not mentioned in said
search warrant?
A. During the conversation right after the conduct of said raid, I was informed that the reason why
they also brought the other items not included in the search warrant was because the money and
other jewelries were contained in attach cases and cartons with markings "Sony Trinitron", and I

think three (3) vaults or steel safes. Believing that the attach cases and the steel safes were
containing firearms, they forced open these containers only to find out that they contained money.
xxx
Q. You said you found money instead of weapons, do you know the reason why your team seized
this money instead of weapons?
A. I think the overall team leader and the other two officers assisting him decided to bring along also
the money because at that time it was already dark and they felt most secured if they will bring that
because they might be suspected also of taking money out of those items, your Honor.49
Cross-examination
Atty. Banaag
Q. Were you present when the search warrant in connection with this case was applied before the
Municipal Trial Court of Batangas, Branch 1?
A. Yes, sir.
Q. And the search warrant applied for by you was for the search and seizure of five (5) baby armalite
rifles M-16 and five (5) boxes of ammunition?
A. Yes, sir.
xxx
AJ AMORES
Q. Before you applied for a search warrant, did you conduct surveillance in the house of Miss
Elizabeth Dimaano?
A. The Intelligence Operatives conducted surveillance together with the MSU elements, your Honor.
Q. And this party believed there were weapons deposited in the house of Miss Elizabeth Dimaano?
A. Yes, your Honor.
Q. And they so swore before the Municipal Trial Judge?
A. Yes, your Honor.
Q. But they did not mention to you, the applicant for the search warrant, any other properties or
contraband which could be found in the residence of Miss Elizabeth Dimaano?

A. They just gave us still unconfirmed report about some hidden items, for instance, the
communications equipment and money. However, I did not include that in the application for search
warrant considering that we have not established concrete evidence about that. So when
Q. So that when you applied for search warrant, you had reason to believe that only weapons were
in the house of Miss Elizabeth Dimaano?
A. Yes, your Honor.50
xxx
Q. You stated that a .45 caliber pistol was seized along with one armalite rifle M-16 and how many
ammunition?
A. Forty, sir.
Q. And this became the subject of your complaint with the issuing Court, with the fiscals office who
charged Elizabeth Dimaano for Illegal Possession of Firearms and Ammunition?
A. Yes, sir.
Q. Do you know what happened to that case?
A. I think it was dismissed, sir.
Q. In the fiscals office?
A. Yes, sir.
Q. Because the armalite rifle you seized, as well as the .45 caliber pistol had a Memorandum
Receipt in the name of Felino Melegrito, is that not correct?
A. I think that was the reason, sir.
Q. There were other articles seized which were not included in the search warrant, like for instance,
jewelries. Why did you seize the jewelries?
A. I think it was the decision of the overall team leader and his assistant to bring along also the
jewelries and other items, sir. I do not really know where it was taken but they brought along also
these articles. I do not really know their reason for bringing the same, but I just learned that these
were taken because they might get lost if they will just leave this behind.
xxx
Q. How about the money seized by your raiding team, they were not also included in the search
warrant?

A. Yes sir, but I believe they were also taken considering that the money was discovered to be
contained in attach cases. These attach cases were suspected to be containing pistols or other
high powered firearms, but in the course of the search the contents turned out to be money. So the
team leader also decided to take this considering that they believed that if they will just leave the
money behind, it might get lost also.
1wphi1

Q. That holds true also with respect to the other articles that were seized by your raiding team, like
Transfer Certificates of Title of lands?
A. Yes, sir. I think they were contained in one of the vaults that were opened. 51
It is obvious from the testimony of Captain Sebastian that the warrant did not include the monies,
communications equipment, jewelry and land titles that the raiding team confiscated. The search
warrant did not particularly describe these items and the raiding team confiscated them on its own
authority. The raiding team had no legal basis to seize these items without showing that these items
could be the subject of warrantless search and seizure.52 Clearly, the raiding team exceeded its
authority when it seized these items.
The seizure of these items was therefore void, and unless these items are contraband per se, 53 and
they are not, they must be returned to the person from whom the raiding seized them. However, we
do not declare that such person is the lawful owner of these items, merely that the search and
seizure warrant could not be used as basis to seize and withhold these items from the possessor.
We thus hold that these items should be returned immediately to Dimaano.
WHEREFORE, the petition for certiorari is DISMISSED. The questioned Resolutions of the
Sandiganbayan dated 18 November 1991 and 25 March 1992 in Civil Case No. 0037, remanding the
records of this case to the Ombudsman for such appropriate action as the evidence may warrant,
and referring this case to the Commissioner of the Bureau of Internal Revenue for a determination of
any tax liability of respondent Elizabeth Dimaano, are AFFIRMED.
SO ORDERED.

Republic of the Philippines


SUPREME COURT
Manila
EN BANC

G.R. No. 118295 May 2, 1997

WIGBERTO E. TAADA and ANNA DOMINIQUE COSETENG, as members of the Philippine


Senate and as taxpayers; GREGORIO ANDOLANA and JOKER ARROYO as members of the
House of Representatives and as taxpayers; NICANOR P. PERLAS and HORACIO R.
MORALES, both as taxpayers; CIVIL LIBERTIES UNION, NATIONAL ECONOMIC
PROTECTIONISM ASSOCIATION, CENTER FOR ALTERNATIVE DEVELOPMENT INITIATIVES,
LIKAS-KAYANG KAUNLARAN FOUNDATION, INC., PHILIPPINE RURAL RECONSTRUCTION
MOVEMENT, DEMOKRATIKONG KILUSAN NG MAGBUBUKID NG PILIPINAS, INC., and
PHILIPPINE PEASANT INSTITUTE, in representation of various taxpayers and as nongovernmental organizations, petitioners,
vs.
EDGARDO ANGARA, ALBERTO ROMULO, LETICIA RAMOS-SHAHANI, HEHERSON ALVAREZ,
AGAPITO AQUINO, RODOLFO BIAZON, NEPTALI GONZALES, ERNESTO HERRERA, JOSE
LINA, GLORIA. MACAPAGAL-ARROYO, ORLANDO MERCADO, BLAS OPLE, JOHN OSMEA,
SANTANINA RASUL, RAMON REVILLA, RAUL ROCO, FRANCISCO TATAD and FREDDIE
WEBB, in their respective capacities as members of the Philippine Senate who concurred in
the ratification by the President of the Philippines of the Agreement Establishing the World
Trade Organization; SALVADOR ENRIQUEZ, in his capacity as Secretary of Budget and
Management; CARIDAD VALDEHUESA, in her capacity as National Treasurer; RIZALINO
NAVARRO, in his capacity as Secretary of Trade and Industry; ROBERTO SEBASTIAN, in his
capacity as Secretary of Agriculture; ROBERTO DE OCAMPO, in his capacity as Secretary of
Finance; ROBERTO ROMULO, in his capacity as Secretary of Foreign Affairs; and TEOFISTO
T. GUINGONA, in his capacity as Executive Secretary, respondents.

PANGANIBAN, J.:
The emergence on January 1, 1995 of the World Trade Organization, abetted by the membership
thereto of the vast majority of countries has revolutionized international business and economic
relations amongst states. It has irreversibly propelled the world towards trade liberalization and
economic globalization. Liberalization, globalization, deregulation and privatization, the thirdmillennium buzz words, are ushering in a new borderless world of business by sweeping away as
mere historical relics the heretofore traditional modes of promoting and protecting national
economies like tariffs, export subsidies, import quotas, quantitative restrictions, tax exemptions and
currency controls. Finding market niches and becoming the best in specific industries in a marketdriven and export-oriented global scenario are replacing age-old "beggar-thy-neighbor" policies that
unilaterally protect weak and inefficient domestic producers of goods and services. In the words of
Peter Drucker, the well-known management guru, "Increased participation in the world economy has
become the key to domestic economic growth and prosperity."
Brief Historical Background
To hasten worldwide recovery from the devastation wrought by the Second World War, plans for the
establishment of three multilateral institutions inspired by that grand political body, the United
Nations were discussed at Dumbarton Oaks and Bretton Woods. The first was the World Bank
(WB) which was to address the rehabilitation and reconstruction of war-ravaged and later developing

countries; the second, the International Monetary Fund (IMF) which was to deal with currency
problems; and the third, the International Trade Organization (ITO), which was to foster order and
predictability in world trade and to minimize unilateral protectionist policies that invite challenge,
even retaliation, from other states. However, for a variety of reasons, including its non-ratification by
the United States, the ITO, unlike the IMF and WB, never took off. What remained was only GATT
the General Agreement on Tariffs and Trade. GATT was a collection of treaties governing access to
the economies of treaty adherents with no institutionalized body administering the agreements or
dependable system of dispute settlement.
After half a century and several dizzying rounds of negotiations, principally the Kennedy Round, the
Tokyo Round and the Uruguay Round, the world finally gave birth to that administering body the
World Trade Organization with the signing of the "Final Act" in Marrakesh, Morocco and the
ratification of the WTO Agreement by its members. 1
Like many other developing countries, the Philippines joined WTO as a founding member with the
goal, as articulated by President Fidel V. Ramos in two letters to the Senate (infra), of improving
"Philippine access to foreign markets, especially its major trading partners, through the reduction of
tariffs on its exports, particularly agricultural and industrial products." The President also saw in the
WTO the opening of "new opportunities for the services sector . . . , (the reduction of) costs and
uncertainty associated with exporting . . . , and (the attraction of) more investments into the country."
Although the Chief Executive did not expressly mention it in his letter, the Philippines and this is of
special interest to the legal profession will benefit from the WTO system of dispute settlement by
judicial adjudication through the independent WTO settlement bodies called (1) Dispute Settlement
Panels and (2) Appellate Tribunal. Heretofore, trade disputes were settled mainly through
negotiations where solutions were arrived at frequently on the basis of relative bargaining strengths,
and where naturally, weak and underdeveloped countries were at a disadvantage.
The Petition in Brief
Arguing mainly (1) that the WTO requires the Philippines "to place nationals and products of
member-countries on the same footing as Filipinos and local products" and (2) that the WTO
"intrudes, limits and/or impairs" the constitutional powers of both Congress and the Supreme Court,
the instant petition before this Court assails the WTO Agreement for violating the mandate of the
1987 Constitution to "develop a self-reliant and independent national economy effectively controlled
by Filipinos . . . (to) give preference to qualified Filipinos (and to) promote the preferential use of
Filipino labor, domestic materials and locally produced goods."
Simply stated, does the Philippine Constitution prohibit Philippine participation in worldwide trade
liberalization and economic globalization? Does it proscribe Philippine integration into a global
economy that is liberalized, deregulated and privatized? These are the main questions raised in this
petition for certiorari, prohibition andmandamus under Rule 65 of the Rules of Court praying (1) for
the nullification, on constitutional grounds, of the concurrence of the Philippine Senate in the
ratification by the President of the Philippines of the Agreement Establishing the World Trade
Organization (WTO Agreement, for brevity) and (2) for the prohibition of its implementation and
enforcement through the release and utilization of public funds, the assignment of public officials and
employees, as well as the use of government properties and resources by respondent-heads of

various executive offices concerned therewith. This concurrence is embodied in Senate Resolution
No. 97, dated December 14, 1994.
The Facts
On April 15, 1994, Respondent Rizalino Navarro, then Secretary of The Department of Trade and
Industry (Secretary Navarro, for brevity), representing the Government of the Republic of the
Philippines, signed in Marrakesh, Morocco, the Final Act Embodying the Results of the Uruguay
Round of Multilateral Negotiations (Final Act, for brevity).
By signing the Final Act, 2 Secretary Navarro on behalf of the Republic of the Philippines, agreed:
(a) to submit, as appropriate, the WTO Agreement for the consideration of their
respective competent authorities, with a view to seeking approval of the Agreement
in accordance with their procedures; and
(b) to adopt the Ministerial Declarations and Decisions.
On August 12, 1994, the members of the Philippine Senate received a letter dated August 11, 1994
from the President of the Philippines, 3 stating among others that "the Uruguay Round Final Act is
hereby submitted to the Senate for its concurrence pursuant to Section 21, Article VII of the Constitution."
On August 13, 1994, the members of the Philippine Senate received another letter from the
President of the Philippines 4 likewise dated August 11, 1994, which stated among others that "the
Uruguay Round Final Act, the Agreement Establishing the World Trade Organization, the Ministerial
Declarations and Decisions, and the Understanding on Commitments in Financial Services are hereby
submitted to the Senate for its concurrence pursuant to Section 21, Article VII of the Constitution."
On December 9, 1994, the President of the Philippines certified the necessity of the immediate
adoption of P.S. 1083, a resolution entitled "Concurring in the Ratification of the Agreement
Establishing the World Trade Organization." 5
On December 14, 1994, the Philippine Senate adopted Resolution No. 97 which "Resolved, as it is
hereby resolved, that the Senate concur, as it hereby concurs, in the ratification by the President of
the Philippines of the Agreement Establishing the World Trade Organization." 6 The text of the WTO
Agreement is written on pages 137 et seq. of Volume I of the 36-volume Uruguay Round of Multilateral
Trade Negotiations and includes various agreements and associated legal instruments (identified in the
said Agreement as Annexes 1, 2 and 3 thereto and collectively referred to as Multilateral Trade
Agreements, for brevity) as follows:
ANNEX 1
Annex 1A: Multilateral Agreement on Trade in Goods
General Agreement on Tariffs and Trade 1994
Agreement on Agriculture
Agreement on the Application of Sanitary and
Phytosanitary Measures

Agreement on Textiles and Clothing


Agreement on Technical Barriers to Trade
Agreement on Trade-Related Investment Measures
Agreement on Implementation of Article VI of he
General Agreement on Tariffs and Trade
1994
Agreement on Implementation of Article VII of the
General on Tariffs and Trade 1994
Agreement on Pre-Shipment Inspection
Agreement on Rules of Origin
Agreement on Imports Licensing Procedures
Agreement on Subsidies and Coordinating
Measures
Agreement on Safeguards
Annex 1B: General Agreement on Trade in Services and Annexes
Annex 1C: Agreement on Trade-Related Aspects of Intellectual
Property Rights
ANNEX 2
Understanding on Rules and Procedures Governing
the Settlement of Disputes
ANNEX 3
Trade Policy Review Mechanism
On December 16, 1994, the President of the Philippines signed 7 the Instrument of Ratification,
declaring:
NOW THEREFORE, be it known that I, FIDEL V. RAMOS, President of the Republic
of the Philippines, after having seen and considered the aforementioned Agreement
Establishing the World Trade Organization and the agreements and associated legal
instruments included in Annexes one (1), two (2) and three (3) of that Agreement
which are integral parts thereof, signed at Marrakesh, Morocco on 15 April 1994, do
hereby ratify and confirm the same and every Article and Clause thereof.
To emphasize, the WTO Agreement ratified by the President of the Philippines is composed of the
Agreement Proper and "the associated legal instruments included in Annexes one (1), two (2) and
three (3) of that Agreement which are integral parts thereof."
On the other hand, the Final Act signed by Secretary Navarro embodies not only the WTO
Agreement (and its integral annexes aforementioned) but also (1) the Ministerial Declarations and

Decisions and (2) the Understanding on Commitments in Financial Services. In his Memorandum
dated May 13, 1996, 8 the Solicitor General describes these two latter documents as follows:
The Ministerial Decisions and Declarations are twenty-five declarations and
decisions on a wide range of matters, such as measures in favor of least developed
countries, notification procedures, relationship of WTO with the International
Monetary Fund (IMF), and agreements on technical barriers to trade and on dispute
settlement.
The Understanding on Commitments in Financial Services dwell on, among other
things, standstill or limitations and qualifications of commitments to existing nonconforming measures, market access, national treatment, and definitions of nonresident supplier of financial services, commercial presence and new financial
service.
On December 29, 1994, the present petition was filed. After careful deliberation on respondents'
comment and petitioners' reply thereto, the Court resolved on December 12, 1995, to give due
course to the petition, and the parties thereafter filed their respective memoranda. The court also
requested the Honorable Lilia R. Bautista, the Philippine Ambassador to the United Nations
stationed in Geneva, Switzerland, to submit a paper, hereafter referred to as "Bautista Paper," 9 for
brevity, (1) providing a historical background of and (2) summarizing the said agreements.
During the Oral Argument held on August 27, 1996, the Court directed:
(a) the petitioners to submit the (1) Senate Committee Report on the matter in
controversy and (2) the transcript of proceedings/hearings in the Senate; and
(b) the Solicitor General, as counsel for respondents, to file (1) a list of Philippine
treaties signed prior to the Philippine adherence to the WTO Agreement, which
derogate from Philippine sovereignty and (2) copies of the multi-volume WTO
Agreement and other documents mentioned in the Final Act, as soon as possible.
After receipt of the foregoing documents, the Court said it would consider the case submitted for
resolution. In a Compliance dated September 16, 1996, the Solicitor General submitted a printed
copy of the 36-volume Uruguay Round of Multilateral Trade Negotiations, and in another Compliance
dated October 24, 1996, he listed the various "bilateral or multilateral treaties or international
instruments involving derogation of Philippine sovereignty." Petitioners, on the other hand, submitted
their Compliance dated January 28, 1997, on January 30, 1997.
The Issues
In their Memorandum dated March 11, 1996, petitioners summarized the issues as follows:
A. Whether the petition presents a political question or is otherwise not justiciable.

B. Whether the petitioner members of the Senate who participated in the


deliberations and voting leading to the concurrence are estopped from impugning the
validity of the Agreement Establishing the World Trade Organization or of the validity
of the concurrence.
C. Whether the provisions of the Agreement Establishing the World Trade
Organization contravene the provisions of Sec. 19, Article II, and Secs. 10 and 12,
Article XII, all of the 1987 Philippine Constitution.
D. Whether provisions of the Agreement Establishing the World Trade Organization
unduly limit, restrict and impair Philippine sovereignty specifically the legislative
power which, under Sec. 2, Article VI, 1987 Philippine Constitution is "vested in the
Congress of the Philippines";
E. Whether provisions of the Agreement Establishing the World Trade Organization
interfere with the exercise of judicial power.
F. Whether the respondent members of the Senate acted in grave abuse of discretion
amounting to lack or excess of jurisdiction when they voted for concurrence in the
ratification of the constitutionally-infirm Agreement Establishing the World Trade
Organization.
G. Whether the respondent members of the Senate acted in grave abuse of
discretion amounting to lack or excess of jurisdiction when they concurred only in the
ratification of the Agreement Establishing the World Trade Organization, and not with
the Presidential submission which included the Final Act, Ministerial Declaration and
Decisions, and the Understanding on Commitments in Financial Services.
On the other hand, the Solicitor General as counsel for respondents "synthesized the several issues
raised by petitioners into the following": 10
1. Whether or not the provisions of the "Agreement Establishing the World Trade
Organization and the Agreements and Associated Legal Instruments included in
Annexes one (1), two (2) and three (3) of that agreement" cited by petitioners directly
contravene or undermine the letter, spirit and intent of Section 19, Article II and
Sections 10 and 12, Article XII of the 1987 Constitution.
2. Whether or not certain provisions of the Agreement unduly limit, restrict or impair
the exercise of legislative power by Congress.
3. Whether or not certain provisions of the Agreement impair the exercise of judicial
power by this Honorable Court in promulgating the rules of evidence.
4. Whether or not the concurrence of the Senate "in the ratification by the President
of the Philippines of the Agreement establishing the World Trade Organization"
implied rejection of the treaty embodied in the Final Act.

By raising and arguing only four issues against the seven presented by petitioners, the Solicitor
General has effectively ignored three, namely: (1) whether the petition presents a political question
or is otherwise not justiciable; (2) whether petitioner-members of the Senate (Wigberto E. Taada
and Anna Dominique Coseteng) are estopped from joining this suit; and (3) whether the respondentmembers of the Senate acted in grave abuse of discretion when they voted for concurrence in the
ratification of the WTO Agreement. The foregoing notwithstanding, this Court resolved to deal with
these three issues thus:
(1) The "political question" issue being very fundamental and vital, and being a matter that probes
into the very jurisdiction of this Court to hear and decide this case was deliberated upon by the
Court and will thus be ruled upon as the first issue;
(2) The matter of estoppel will not be taken up because this defense is waivable and the
respondents have effectively waived it by not pursuing it in any of their pleadings; in any event, this
issue, even if ruled in respondents' favor, will not cause the petition's dismissal as there are
petitioners other than the two senators, who are not vulnerable to the defense of estoppel; and
(3) The issue of alleged grave abuse of discretion on the part of the respondent senators will be
taken up as an integral part of the disposition of the four issues raised by the Solicitor General.
During its deliberations on the case, the Court noted that the respondents did not question the locus
standi of petitioners. Hence, they are also deemed to have waived the benefit of such issue. They
probably realized that grave constitutional issues, expenditures of public funds and serious
international commitments of the nation are involved here, and that transcendental public interest
requires that the substantive issues be met head on and decided on the merits, rather than skirted or
deflected by procedural matters. 11
To recapitulate, the issues that will be ruled upon shortly are:
(1) DOES THE PETITION PRESENT A JUSTICIABLE CONTROVERSY?
OTHERWISE STATED, DOES THE PETITION INVOLVE A POLITICAL QUESTION
OVER WHICH THIS COURT HAS NO JURISDICTION?
(2) DO THE PROVISIONS OF THE WTO AGREEMENT AND ITS THREE ANNEXES
CONTRAVENE SEC. 19, ARTICLE II, AND SECS. 10 AND 12, ARTICLE XII, OF
THE PHILIPPINE CONSTITUTION?
(3) DO THE PROVISIONS OF SAID AGREEMENT AND ITS ANNEXES LIMIT,
RESTRICT, OR IMPAIR THE EXERCISE OF LEGISLATIVE POWER BY
CONGRESS?
(4) DO SAID PROVISIONS UNDULY IMPAIR OR INTERFERE WITH THE
EXERCISE OF JUDICIAL POWER BY THIS COURT IN PROMULGATING RULES
ON EVIDENCE?

(5) WAS THE CONCURRENCE OF THE SENATE IN THE WTO AGREEMENT AND
ITS ANNEXES SUFFICIENT AND/OR VALID, CONSIDERING THAT IT DID NOT
INCLUDE THE FINAL ACT, MINISTERIAL DECLARATIONS AND DECISIONS, AND
THE UNDERSTANDING ON COMMITMENTS IN FINANCIAL SERVICES?
The First Issue: Does the Court
Have Jurisdiction Over the Controversy?
In seeking to nullify an act of the Philippine Senate on the ground that it contravenes the
Constitution, the petition no doubt raises a justiciable controversy. Where an action of the legislative
branch is seriously alleged to have infringed the Constitution, it becomes not only the right but in fact
the duty of the judiciary to settle the dispute. "The question thus posed is judicial rather than political.
The duty (to adjudicate) remains to assure that the supremacy of the Constitution is upheld." 12 Once
a "controversy as to the application or interpretation of a constitutional provision is raised before this
Court (as in the instant case), it becomes a legal issue which the Court is bound by constitutional
mandate to decide." 13
The jurisdiction of this Court to adjudicate the matters
1987 Constitution, 15 as follows:

14

raised in the petition is clearly set out in the

Judicial power includes the duty of the courts of justice to settle actual controversies
involving rights which are legally demandable and enforceable, and to determine
whether or not there has been a grave abuse of discretion amounting to lack or
excess of jurisdiction on the part of any branch or instrumentality of the government.
The foregoing text emphasizes the judicial department's duty and power to strike down grave abuse
of discretion on the part of any branch or instrumentality of government including Congress. It is an
innovation in our political law. 16 As explained by former Chief Justice Roberto Concepcion, 17 "the
judiciary is the final arbiter on the question of whether or not a branch of government or any of its officials
has acted without jurisdiction or in excess of jurisdiction or so capriciously as to constitute an abuse of
discretion amounting to excess of jurisdiction. This is not only a judicial power but a duty to pass judgment
on matters of this nature."
As this Court has repeatedly and firmly emphasized in many cases, 18 it will not shirk, digress from or
abandon its sacred duty and authority to uphold the Constitution in matters that involve grave abuse of
discretion brought before it in appropriate cases, committed by any officer, agency, instrumentality or
department of the government.
As the petition alleges grave abuse of discretion and as there is no other plain, speedy or adequate
remedy in the ordinary course of law, we have no hesitation at all in holding that this petition should
be given due course and the vital questions raised therein ruled upon under Rule 65 of the Rules of
Court. Indeed, certiorari, prohibition andmandamus are appropriate remedies to raise constitutional
issues and to review and/or prohibit/nullify, when proper, acts of legislative and executive officials.
On this, we have no equivocation.
We should stress that, in deciding to take jurisdiction over this petition, this Court will not review
the wisdom of the decision of the President and the Senate in enlisting the country into the WTO, or

pass upon the merits of trade liberalization as a policy espoused by said international body. Neither
will it rule on the propriety of the government's economic policy of reducing/removing tariffs, taxes,
subsidies, quantitative restrictions, and other import/trade barriers. Rather, it will only exercise its
constitutional duty "to determine whether or not there had been a grave abuse of discretion
amounting to lack or excess of jurisdiction" on the part of the Senate in ratifying the WTO Agreement
and its three annexes.
Second Issue: The WTO Agreement
and Economic Nationalism
This is the lis mota, the main issue, raised by the petition.
Petitioners vigorously argue that the "letter, spirit and intent" of the Constitution mandating
"economic nationalism" are violated by the so-called "parity provisions" and "national treatment"
clauses scattered in various parts not only of the WTO Agreement and its annexes but also in the
Ministerial Decisions and Declarations and in the Understanding on Commitments in Financial
Services.
Specifically, the "flagship" constitutional provisions referred to are Sec 19, Article II, and Secs. 10
and 12, Article XII, of the Constitution, which are worded as follows:
Article II
DECLARATION OF PRINCIPLES
AND STATE POLICIES
xxx xxx xxx
Sec. 19. The State shall develop a self-reliant and independent national economy
effectively controlled by Filipinos.
xxx xxx xxx
Article XII
NATIONAL ECONOMY AND PATRIMONY
xxx xxx xxx
Sec. 10. . . . The Congress shall enact measures that will encourage the formation
and operation of enterprises whose capital is wholly owned by Filipinos.
In the grant of rights, privileges, and concessions covering the national economy and
patrimony, the State shall give preference to qualified Filipinos.
xxx xxx xxx

Sec. 12. The State shall promote the preferential use of Filipino labor, domestic
materials and locally produced goods, and adopt measures that help make them
competitive.
Petitioners aver that these sacred constitutional principles are desecrated by the following WTO
provisions quoted in their memorandum: 19
a) In the area of investment measures related to trade in goods (TRIMS, for brevity):
Article 2
National Treatment and Quantitative Restrictions.
1. Without prejudice to other rights and obligations under GATT 1994,
no Member shall apply any TRIM that is inconsistent with the
provisions of Article II or Article XI of GATT 1994.
2. An illustrative list of TRIMS that are inconsistent with the
obligations of general elimination of quantitative restrictions provided
for in paragraph I of Article XI of GATT 1994 is contained in the Annex
to this Agreement." (Agreement on Trade-Related Investment
Measures, Vol. 27, Uruguay Round, Legal Instruments, p. 22121,
emphasis supplied).
The Annex referred to reads as follows:
ANNEX
Illustrative List
1. TRIMS that are inconsistent with the obligation of national treatment provided for
in paragraph 4 of Article III of GATT 1994 include those which are mandatory or
enforceable under domestic law or under administrative rulings, or compliance with
which is necessary to obtain an advantage, and which require:
(a) the purchase or use by an enterprise of products of domestic
origin or from any domestic source, whether specified in terms of
particular products, in terms of volume or value of products, or in
terms of proportion of volume or value of its local production; or
(b) that an enterprise's purchases or use of imported products be
limited to an amount related to the volume or value of local products
that it exports.
2. TRIMS that are inconsistent with the obligations of general elimination of
quantitative restrictions provided for in paragraph 1 of Article XI of GATT 1994

include those which are mandatory or enforceable under domestic laws or under
administrative rulings, or compliance with which is necessary to obtain an advantage,
and which restrict:
(a) the importation by an enterprise of products used in or related to
the local production that it exports;
(b) the importation by an enterprise of products used in or related to
its local production by restricting its access to foreign exchange
inflows attributable to the enterprise; or
(c) the exportation or sale for export specified in terms of particular
products, in terms of volume or value of products, or in terms of a
preparation of volume or value of its local production. (Annex to the
Agreement on Trade-Related Investment Measures, Vol. 27, Uruguay
Round Legal Documents, p. 22125, emphasis supplied).
The paragraph 4 of Article III of GATT 1994 referred to is quoted as follows:
The products of the territory of any contracting party imported into the
territory of any other contracting party shall be accorded treatment no
less favorable than that accorded to like products of national origin in
respect of laws, regulations and requirements affecting their internal
sale, offering for sale, purchase, transportation, distribution or use,
the provisions of this paragraph shall not prevent the application of
differential internal transportation charges which are based
exclusively on the economic operation of the means of transport and
not on the nationality of the product." (Article III, GATT 1947, as
amended by the Protocol Modifying Part II, and Article XXVI of GATT,
14 September 1948, 62 UMTS 82-84 in relation to paragraph 1(a) of
the General Agreement on Tariffs and Trade 1994, Vol. 1, Uruguay
Round, Legal Instruments p. 177, emphasis supplied).
(b) In the area of trade related aspects of intellectual property rights (TRIPS, for
brevity):
Each Member shall accord to the nationals of other Members
treatment no less favourable than that it accords to its own
nationals with regard to the protection of intellectual property. . . (par.
1 Article 3, Agreement on Trade-Related Aspect of Intellectual
Property rights, Vol. 31, Uruguay Round, Legal Instruments, p. 25432
(emphasis supplied)
(c) In the area of the General Agreement on Trade in Services:
National Treatment

1. In the sectors inscribed in its schedule, and subject to any


conditions and qualifications set out therein, each Member shall
accord to services and service suppliers of any other Member, in
respect of all measures affecting the supply of services, treatment no
less favourable than it accords to its own like services and service
suppliers.
2. A Member may meet the requirement of paragraph I by according
to services and service suppliers of any other Member, either formally
suppliers of any other Member, either formally identical treatment or
formally different treatment to that it accords to its own like services
and service suppliers.
3. Formally identical or formally different treatment shall be
considered to be less favourable if it modifies the conditions of
completion in favour of services or service suppliers of the Member
compared to like services or service suppliers of any other Member.
(Article XVII, General Agreement on Trade in Services, Vol. 28,
Uruguay Round Legal Instruments, p. 22610 emphasis supplied).
It is petitioners' position that the foregoing "national treatment" and "parity provisions" of the WTO
Agreement "place nationals and products of member countries on the same footing as Filipinos and
local products," in contravention of the "Filipino First" policy of the Constitution. They allegedly
render meaningless the phrase "effectively controlled by Filipinos." The constitutional conflict
becomes more manifest when viewed in the context of the clear duty imposed on the Philippines as
a WTO member to ensure the conformity of its laws, regulations and administrative procedures with
its obligations as provided in the annexed agreements. 20 Petitioners further argue that these provisions
contravene constitutional limitations on the role exports play in national development and negate the
preferential treatment accorded to Filipino labor, domestic materials and locally produced goods.
On the other hand, respondents through the Solicitor General counter (1) that such Charter
provisions are not self-executing and merely set out general policies; (2) that these nationalistic
portions of the Constitution invoked by petitioners should not be read in isolation but should be
related to other relevant provisions of Art. XII, particularly Secs. 1 and 13 thereof; (3) that read
properly, the cited WTO clauses do not conflict with Constitution; and (4) that the WTO Agreement
contains sufficient provisions to protect developing countries like the Philippines from the harshness
of sudden trade liberalization.
We shall now discuss and rule on these arguments.
Declaration of Principles
Not Self-Executing
By its very title, Article II of the Constitution is a "declaration of principles and state policies." The
counterpart of this article in the 1935 Constitution 21 is called the "basic political creed of the nation" by
Dean Vicente Sinco. 22 These principles in Article II are not intended to be self-executing principles ready

for enforcement through the courts. 23 They are used by the judiciary as aids or as guides in the exercise
of its power of judicial review, and by the legislature in its enactment of laws. As held in the leading case
of Kilosbayan, Incorporated vs. Morato, 24 the principles and state policies enumerated in Article II and
some sections of Article XII are not "self-executing provisions, the disregard of which can give rise to a
cause of action in the courts. They do not embody judicially enforceable constitutional rights but
guidelines for legislation."

In the same light, we held in Basco vs. Pagcor 25 that broad constitutional principles need legislative
enactments to implement the, thus:
On petitioners' allegation that P.D. 1869 violates Sections 11 (Personal Dignity) 12
(Family) and 13 (Role of Youth) of Article II; Section 13 (Social Justice) of Article XIII
and Section 2 (Educational Values) of Article XIV of the 1987 Constitution, suffice it to
state also that these are merely statements of principles and policies. As such, they
are basically not self-executing, meaning a law should be passed by Congress to
clearly define and effectuate such principles.
In general, therefore, the 1935 provisions were not intended to be
self-executing principles ready for enforcement through the courts.
They were rather directives addressed to the executive and to the
legislature. If the executive and the legislature failed to heed the
directives of the article, the available remedy was not judicial but
political. The electorate could express their displeasure with the
failure of the executive and the legislature through the language of
the ballot. (Bernas, Vol. II, p. 2).
The reasons for denying a cause of action to an alleged infringement of board constitutional
principles are sourced from basic considerations of due process and the lack of judicial authority to
wade "into the uncharted ocean of social and economic policy making." Mr. Justice Florentino P.
Feliciano in his concurring opinion inOposa vs. Factoran, Jr., 26 explained these reasons as follows:
My suggestion is simply that petitioners must, before the trial court, show a more
specific legal right a right cast in language of a significantly lower order of
generality than Article II (15) of the Constitution that is or may be violated by the
actions, or failures to act, imputed to the public respondent by petitioners so that the
trial court can validly render judgment grating all or part of the relief prayed for. To my
mind, the court should be understood as simply saying that such a more specific
legal right or rights may well exist in our corpus of law, considering the general policy
principles found in the Constitution and the existence of the Philippine Environment
Code, and that the trial court should have given petitioners an effective opportunity
so to demonstrate, instead of aborting the proceedings on a motion to dismiss.
It seems to me important that the legal right which is an essential component of a
cause of action be a specific, operable legal right, rather than a constitutional or
statutory policy, for at least two (2) reasons. One is that unless the legal right claimed
to have been violated or disregarded is given specification in operational terms,

defendants may well be unable to defend themselves intelligently and effectively; in


other words, there are due process dimensions to this matter.
The second is a broader-gauge consideration where a specific violation of law or
applicable regulation is not alleged or proved, petitioners can be expected to fall back
on the expanded conception of judicial power in the second paragraph of Section 1
of Article VIII of the Constitution which reads:
Sec. 1. . . .
Judicial power includes the duty of the courts of justice to settle actual
controversies involving rights which are legally demandable and
enforceable, and to determine whether or not there has been a grave
abuse of discretion amounting to lack or excess of jurisdiction on the
part of any branch or instrumentality of the Government. (Emphasis
supplied)
When substantive standards as general as "the right to a balanced and healthy
ecology" and "the right to health" are combined with remedial standards as broad
ranging as "a grave abuse of discretion amounting to lack or excess of jurisdiction,"
the result will be, it is respectfully submitted, to propel courts into the uncharted
ocean of social and economic policy making. At least in respect of the vast area of
environmental protection and management, our courts have no claim to special
technical competence and experience and professional qualification. Where no
specific, operable norms and standards are shown to exist, then the policy making
departments the legislative and executive departments must be given a real
and effective opportunity to fashion and promulgate those norms and standards, and
to implement them before the courts should intervene.
Economic Nationalism Should Be Read with
Other Constitutional Mandates to Attain
Balanced Development of Economy
On the other hand, Secs. 10 and 12 of Article XII, apart from merely laying down general principles
relating to the national economy and patrimony, should be read and understood in relation to the
other sections in said article, especially Secs. 1 and 13 thereof which read:
Sec. 1. The goals of the national economy are a more equitable distribution of
opportunities, income, and wealth; a sustained increase in the amount of goods and
services produced by the nation for the benefit of the people; and an expanding
productivity as the key to raising the quality of life for all especially the
underprivileged.
The State shall promote industrialization and full employment based on sound
agricultural development and agrarian reform, through industries that make full and
efficient use of human and natural resources, and which are competitive in both

domestic and foreign markets. However, the State shall protect Filipino enterprises
against unfair foreign competition and trade practices.
In the pursuit of these goals, all sectors of the economy and all regions of the country
shall be given optimum opportunity to develop. . . .
xxx xxx xxx
Sec. 13. The State shall pursue a trade policy that serves the general welfare and
utilizes all forms and arrangements of exchange on the basis of equality and
reciprocity.
As pointed out by the Solicitor General, Sec. 1 lays down the basic goals of national economic
development, as follows:
1. A more equitable distribution of opportunities, income and wealth;
2. A sustained increase in the amount of goods and services provided by the nation for the benefit of
the people; and
3. An expanding productivity as the key to raising the quality of life for all especially the
underprivileged.
With these goals in context, the Constitution then ordains the ideals of economic nationalism (1) by
expressing preference in favor of qualified Filipinos "in the grant of rights, privileges and concessions
covering the national economy and patrimony" 27 and in the use of "Filipino labor, domestic materials
and locally-produced goods"; (2) by mandating the State to "adopt measures that help make them
competitive; 28 and (3) by requiring the State to "develop a self-reliant and independent national economy
effectively controlled by Filipinos." 29 In similar language, the Constitution takes into account the realities
of the outside world as it requires the pursuit of "a trade policy that serves the general welfare and utilizes
all forms and arrangements of exchange on the basis of equality ad reciprocity"; 30 and speaks of
industries "which are competitive in both domestic and foreign markets" as well as of the protection of
"Filipino enterprises against unfair foreign competition and trade practices."
It is true that in the recent case of Manila Prince Hotel vs. Government Service Insurance System, et
al., 31 this Court held that "Sec. 10, second par., Art. XII of the 1987 Constitution is a mandatory, positive
command which is complete in itself and which needs no further guidelines or implementing laws or rule
for its enforcement. From its very words the provision does not require any legislation to put it in
operation. It is per se judicially enforceable." However, as the constitutional provision itself states, it is
enforceable only in regard to "the grants of rights, privileges and concessions covering national economy
and patrimony" and not to every aspect of trade and commerce. It refers to exceptions rather than the
rule. The issue here is not whether this paragraph of Sec. 10 of Art. XII is self-executing or not. Rather,
the issue is whether, as a rule, there are enough balancing provisions in the Constitution to allow the
Senate to ratify the Philippine concurrence in the WTO Agreement. And we hold that there are.
All told, while the Constitution indeed mandates a bias in favor of Filipino goods, services, labor and
enterprises, at the same time, it recognizes the need for business exchange with the rest of the

world on the bases of equality and reciprocity and limits protection of Filipino enterprises only
against foreign competition and trade practices that are unfair. 32 In other words, the Constitution did
not intend to pursue an isolationist policy. It did not shut out foreign investments, goods and services in
the development of the Philippine economy. While the Constitution does not encourage the unlimited
entry of foreign goods, services and investments into the country, it does not prohibit them either. In fact, it
allows an exchange on the basis of equality and reciprocity, frowning only on foreign competition that
is unfair.
WTO Recognizes Need to
Protect Weak Economies
Upon the other hand, respondents maintain that the WTO itself has some built-in advantages to
protect weak and developing economies, which comprise the vast majority of its members. Unlike in
the UN where major states have permanent seats and veto powers in the Security Council, in the
WTO, decisions are made on the basis of sovereign equality, with each member's vote equal in
weight to that of any other. There is no WTO equivalent of the UN Security Council.
WTO decides by consensus whenever possible, otherwise, decisions of the
Ministerial Conference and the General Council shall be taken by the majority of the
votes cast, except in cases of interpretation of the Agreement or waiver of the
obligation of a member which would require three fourths vote. Amendments would
require two thirds vote in general. Amendments to MFN provisions and the
Amendments provision will require assent of all members. Any member may
withdraw from the Agreement upon the expiration of six months from the date of
notice of withdrawals. 33
Hence, poor countries can protect their common interests more effectively through the WTO than
through one-on-one negotiations with developed countries. Within the WTO, developing countries
can form powerful blocs to push their economic agenda more decisively than outside the
Organization. This is not merely a matter of practical alliances but a negotiating strategy rooted in
law. Thus, the basic principles underlying the WTO Agreement recognize the need of developing
countries like the Philippines to "share in the growth in international trade commensurate with the
needs of their economic development." These basic principles are found in the preamble 34 of the
WTO Agreement as follows:
The Parties to this Agreement,
Recognizing that their relations in the field of trade and economic endeavour should
be conducted with a view to raising standards of living, ensuring full employment and
a large and steadily growing volume of real income and effective demand, and
expanding the production of and trade in goods and services, while allowing for the
optimal use of the world's resources in accordance with the objective of sustainable
development, seeking both to protect and preserve the environment and to enhance
the means for doing so in a manner consistent with their respective needs and
concerns at different levels of economic development,

Recognizing further that there is need for positive efforts designed to ensure that
developing countries, and especially the least developed among them, secure
a share in the growth in international trade commensurate with the needs of their
economic development,
Being desirous of contributing to these objectives by entering into reciprocal and
mutually advantageous arrangements directed to the substantial reduction of tariffs
and other barriers to trade and to the elimination of discriminatory treatment in
international trade relations,
Resolved, therefore, to develop an integrated, more viable and durable multilateral
trading system encompassing the General Agreement on Tariffs and Trade, the
results of past trade liberalization efforts, and all of the results of the Uruguay Round
of Multilateral Trade Negotiations,
Determined to preserve the basic principles and to further the objectives underlying
this multilateral trading system, . . . (emphasis supplied.)
Specific WTO Provisos
Protect Developing Countries
So too, the Solicitor General points out that pursuant to and consistent with the foregoing basic
principles, the WTO Agreement grants developing countries a more lenient treatment, giving their
domestic industries some protection from the rush of foreign competition. Thus, with respect to tariffs
in general, preferential treatment is given to developing countries in terms of the amount of tariff
reduction and the period within which the reduction is to be spread out. Specifically, GATT requires
an average tariff reduction rate of 36% for developed countries to be effected within a period of six
(6) years while developing countries including the Philippines are required to effect an average
tariff reduction of only 24% within ten (10) years.
In respect to domestic subsidy, GATT requires developed countries to reduce domestic support to
agricultural products by 20% over six (6) years, as compared to only 13% for developing countries to
be effected within ten (10) years.
In regard to export subsidy for agricultural products, GATT requires developed countries to reduce
their budgetary outlays for export subsidy by 36% and export volumes receiving export subsidy
by 21% within a period of six (6) years. For developing countries, however, the reduction rate is
only two-thirds of that prescribed for developed countries and a longer period of ten (10) years within
which to effect such reduction.
Moreover, GATT itself has provided built-in protection from unfair foreign competition and trade
practices including anti-dumping measures, countervailing measures and safeguards against import
surges. Where local businesses are jeopardized by unfair foreign competition, the Philippines can
avail of these measures. There is hardly therefore any basis for the statement that under the WTO,
local industries and enterprises will all be wiped out and that Filipinos will be deprived of control of
the economy. Quite the contrary, the weaker situations of developing nations like the Philippines

have been taken into account; thus, there would be no basis to say that in joining the WTO, the
respondents have gravely abused their discretion. True, they have made a bold decision to steer the
ship of state into the yet uncharted sea of economic liberalization. But such decision cannot be set
aside on the ground of grave abuse of discretion, simply because we disagree with it or simply
because we believe only in other economic policies. As earlier stated, the Court in taking jurisdiction
of this case will not pass upon the advantages and disadvantages of trade liberalization as an
economic policy. It will only perform its constitutional duty of determining whether the Senate
committed grave abuse of discretion.
Constitution Does Not
Rule Out Foreign Competition
Furthermore, the constitutional policy of a "self-reliant and independent national economy" 35 does not
necessarily rule out the entry of foreign investments, goods and services. It contemplates neither
"economic seclusion" nor "mendicancy in the international community." As explained by Constitutional
Commissioner Bernardo Villegas, sponsor of this constitutional policy:
Economic self-reliance is a primary objective of a developing country that is keenly
aware of overdependence on external assistance for even its most basic needs. It
does not mean autarky or economic seclusion; rather, it means avoiding mendicancy
in the international community. Independence refers to the freedom from undue
foreign control of the national economy, especially in such strategic industries as in
the development of natural resources and public utilities. 36
The WTO reliance on "most favored nation," "national treatment," and "trade without discrimination"
cannot be struck down as unconstitutional as in fact they are rules of equality and reciprocity that
apply to all WTO members. Aside from envisioning a trade policy based on "equality and
reciprocity," 37 the fundamental law encourages industries that are "competitive in both domestic and
foreign markets," thereby demonstrating a clear policy against a sheltered domestic trade environment,
but one in favor of the gradual development of robust industries that can compete with the best in the
foreign markets. Indeed, Filipino managers and Filipino enterprises have shown capability and tenacity to
compete internationally. And given a free trade environment, Filipino entrepreneurs and managers in
Hongkong have demonstrated the Filipino capacity to grow and to prosper against the best offered under
a policy of laissez faire.
Constitution Favors Consumers,
Not Industries or Enterprises
The Constitution has not really shown any unbalanced bias in favor of any business or enterprise,
nor does it contain any specific pronouncement that Filipino companies should be pampered with a
total proscription of foreign competition. On the other hand, respondents claim that WTO/GATT aims
to make available to the Filipino consumer the best goods and services obtainable anywhere in the
world at the most reasonable prices. Consequently, the question boils down to whether WTO/GATT
will favor the general welfare of the public at large.
Will adherence to the WTO treaty bring this ideal (of favoring the general welfare) to reality?

Will WTO/GATT succeed in promoting the Filipinos' general welfare because it will as promised
by its promoters expand the country's exports and generate more employment?
Will it bring more prosperity, employment, purchasing power and quality products at the most
reasonable rates to the Filipino public?
The responses to these questions involve "judgment calls" by our policy makers, for which they are
answerable to our people during appropriate electoral exercises. Such questions and the answers
thereto are not subject to judicial pronouncements based on grave abuse of discretion.
Constitution Designed to Meet
Future Events and Contingencies
No doubt, the WTO Agreement was not yet in existence when the Constitution was drafted and
ratified in 1987. That does not mean however that the Charter is necessarily flawed in the sense that
its framers might not have anticipated the advent of a borderless world of business. By the same
token, the United Nations was not yet in existence when the 1935 Constitution became effective. Did
that necessarily mean that the then Constitution might not have contemplated a diminution of the
absoluteness of sovereignty when the Philippines signed the UN Charter, thereby effectively
surrendering part of its control over its foreign relations to the decisions of various UN organs like the
Security Council?
It is not difficult to answer this question. Constitutions are designed to meet not only the vagaries of
contemporary events. They should be interpreted to cover even future and unknown circumstances.
It is to the credit of its drafters that a Constitution can withstand the assaults of bigots and infidels but
at the same time bend with the refreshing winds of change necessitated by unfolding events. As one
eminent political law writer and respected jurist 38 explains:
The Constitution must be quintessential rather than superficial, the root and not the
blossom, the base and frame-work only of the edifice that is yet to rise. It is but the
core of the dream that must take shape, not in a twinkling by mandate of our
delegates, but slowly "in the crucible of Filipino minds and hearts," where it will in
time develop its sinews and gradually gather its strength and finally achieve its
substance. In fine, the Constitution cannot, like the goddess Athena, rise full-grown
from the brow of the Constitutional Convention, nor can it conjure by mere fiat an
instant Utopia. It must grow with the society it seeks to re-structure and march apace
with the progress of the race, drawing from the vicissitudes of history the dynamism
and vitality that will keep it, far from becoming a petrified rule, a pulsing, living law
attuned to the heartbeat of the nation.
Third Issue: The WTO Agreement and Legislative Power
The WTO Agreement provides that "(e)ach Member shall ensure the conformity of its laws,
regulations and administrative procedures with its obligations as provided in the annexed
Agreements." 39 Petitioners maintain that this undertaking "unduly limits, restricts and impairs Philippine
sovereignty, specifically the legislative power which under Sec. 2, Article VI of the 1987 Philippine

Constitution is vested in the Congress of the Philippines. It is an assault on the sovereign powers of the
Philippines because this means that Congress could not pass legislation that will be good for our national
interest and general welfare if such legislation will not conform with the WTO Agreement, which not only
relates to the trade in goods . . . but also to the flow of investments and money . . . as well as to a whole
slew of agreements on socio-cultural matters . . . 40

More specifically, petitioners claim that said WTO proviso derogates from the power to tax, which is
lodged in the Congress. 41 And while the Constitution allows Congress to authorize the President to fix
tariff rates, import and export quotas, tonnage and wharfage dues, and other duties or imposts, such
authority is subject to "specified limits and . . . such limitations and restrictions" as Congress may
provide, 42 as in fact it did under Sec. 401 of the Tariff and Customs Code.
Sovereignty Limited by
International Law and Treaties
This Court notes and appreciates the ferocity and passion by which petitioners stressed their
arguments on this issue. However, while sovereignty has traditionally been deemed absolute and allencompassing on the domestic level, it is however subject to restrictions and limitations voluntarily
agreed to by the Philippines, expressly or impliedly, as a member of the family of nations.
Unquestionably, the Constitution did not envision a hermit-type isolation of the country from the rest
of the world. In its Declaration of Principles and State Policies, the Constitution "adopts the generally
accepted principles of international law as part of the law of the land, and adheres to the policy of
peace, equality, justice, freedom, cooperation and amity, with all nations." 43 By the doctrine of
incorporation, the country is bound by generally accepted principles of international law, which are
considered to be automatically part of our own laws. 44 One of the oldest and most fundamental rules in
international law is pacta sunt servanda international agreements must be performed in good faith. "A
treaty engagement is not a mere moral obligation but creates a legally binding obligation on the
parties . . . A state which has contracted valid international obligations is bound to make in its legislations
such modifications as may be necessary to ensure the fulfillment of the obligations undertaken." 45
By their inherent nature, treaties really limit or restrict the absoluteness of sovereignty. By their
voluntary act, nations may surrender some aspects of their state power in exchange for greater
benefits granted by or derived from a convention or pact. After all, states, like individuals, live with
coequals, and in pursuit of mutually covenanted objectives and benefits, they also commonly agree
to limit the exercise of their otherwise absolute rights. Thus, treaties have been used to record
agreements between States concerning such widely diverse matters as, for example, the lease of
naval bases, the sale or cession of territory, the termination of war, the regulation of conduct of
hostilities, the formation of alliances, the regulation of commercial relations, the settling of claims, the
laying down of rules governing conduct in peace and the establishment of international
organizations.46 The sovereignty of a state therefore cannot in fact and in reality be considered absolute.
Certain restrictions enter into the picture: (1) limitations imposed by the very nature of membership in the
family of nations and (2) limitations imposed by treaty stipulations. As aptly put by John F. Kennedy,
"Today, no nation can build its destiny alone. The age of self-sufficient nationalism is over. The age of
interdependence is here." 47
UN Charter and Other Treaties
Limit Sovereignty

Thus, when the Philippines joined the United Nations as one of its 51 charter members, it consented
to restrict its sovereign rights under the "concept of sovereignty as auto-limitation." 47-A Under Article 2
of the UN Charter, "(a)ll members shall give the United Nations every assistance in any action it takes in
accordance with the present Charter, and shall refrain from giving assistance to any state against which
the United Nations is taking preventive or enforcement action." Such assistance includes payment of its
corresponding share not merely in administrative expenses but also in expenditures for the peacekeeping operations of the organization. In its advisory opinion of July 20, 1961, the International Court of
Justice held that money used by the United Nations Emergency Force in the Middle East and in the
Congo were "expenses of the United Nations" under Article 17, paragraph 2, of the UN Charter. Hence, all
its members must bear their corresponding share in such expenses. In this sense, the Philippine
Congress is restricted in its power to appropriate. It is compelled to appropriate funds whether it agrees
with such peace-keeping expenses or not. So too, under Article 105 of the said Charter, the UN and its
representatives enjoy diplomatic privileges and immunities, thereby limiting again the exercise of
sovereignty of members within their own territory. Another example: although "sovereign equality" and
"domestic jurisdiction" of all members are set forth as underlying principles in the UN Charter, such
provisos are however subject to enforcement measures decided by the Security Council for the
maintenance of international peace and security under Chapter VII of the Charter. A final example: under
Article 103, "(i)n the event of a conflict between the obligations of the Members of the United Nations
under the present Charter and their obligations under any other international agreement, their obligation
under the present charter shall prevail," thus unquestionably denying the Philippines as a member
the sovereign power to make a choice as to which of conflicting obligations, if any, to honor.
Apart from the UN Treaty, the Philippines has entered into many other international pacts both
bilateral and multilateral that involve limitations on Philippine sovereignty. These are enumerated
by the Solicitor General in his Compliance dated October 24, 1996, as follows:
(a) Bilateral convention with the United States regarding taxes on income, where the
Philippines agreed, among others, to exempt from tax, income received in the
Philippines by, among others, the Federal Reserve Bank of the United States, the
Export/Import Bank of the United States, the Overseas Private Investment
Corporation of the United States. Likewise, in said convention, wages, salaries and
similar remunerations paid by the United States to its citizens for labor and personal
services performed by them as employees or officials of the United States are
exempt from income tax by the Philippines.
(b) Bilateral agreement with Belgium, providing, among others, for the avoidance of
double taxation with respect to taxes on income.
(c) Bilateral convention with the Kingdom of Sweden for the avoidance of double
taxation.
(d) Bilateral convention with the French Republic for the avoidance of double
taxation.
(e) Bilateral air transport agreement with Korea where the Philippines agreed to
exempt from all customs duties, inspection fees and other duties or taxes aircrafts of

South Korea and the regular equipment, spare parts and supplies arriving with said
aircrafts.
(f) Bilateral air service agreement with Japan, where the Philippines agreed to
exempt from customs duties, excise taxes, inspection fees and other similar duties,
taxes or charges fuel, lubricating oils, spare parts, regular equipment, stores on
board Japanese aircrafts while on Philippine soil.
(g) Bilateral air service agreement with Belgium where the Philippines granted
Belgian air carriers the same privileges as those granted to Japanese and Korean air
carriers under separate air service agreements.
(h) Bilateral notes with Israel for the abolition of transit and visitor visas where the
Philippines exempted Israeli nationals from the requirement of obtaining transit or
visitor visas for a sojourn in the Philippines not exceeding 59 days.
(i) Bilateral agreement with France exempting French nationals from the requirement
of obtaining transit and visitor visa for a sojourn not exceeding 59 days.
(j) Multilateral Convention on Special Missions, where the Philippines agreed that
premises of Special Missions in the Philippines are inviolable and its agents can not
enter said premises without consent of the Head of Mission concerned. Special
Missions are also exempted from customs duties, taxes and related charges.
(k) Multilateral convention on the Law of Treaties. In this convention, the Philippines
agreed to be governed by the Vienna Convention on the Law of Treaties.
(l) Declaration of the President of the Philippines accepting compulsory jurisdiction of
the International Court of Justice. The International Court of Justice has jurisdiction in
all legal disputes concerning the interpretation of a treaty, any question of
international law, the existence of any fact which, if established, would constitute a
breach "of international obligation."
In the foregoing treaties, the Philippines has effectively agreed to limit the exercise of its sovereign
powers of taxation, eminent domain and police power. The underlying consideration in this partial
surrender of sovereignty is the reciprocal commitment of the other contracting states in granting the
same privilege and immunities to the Philippines, its officials and its citizens. The same reciprocity
characterizes the Philippine commitments under WTO-GATT.
International treaties, whether relating to nuclear disarmament, human rights, the
environment, the law of the sea, or trade, constrain domestic political sovereignty
through the assumption of external obligations. But unless anarchy in international
relations is preferred as an alternative, in most cases we accept that the benefits of
the reciprocal obligations involved outweigh the costs associated with any loss of
political sovereignty. (T)rade treaties that structure relations by reference to durable,
well-defined substantive norms and objective dispute resolution procedures reduce

the risks of larger countries exploiting raw economic power to bully smaller countries,
by subjecting power relations to some form of legal ordering. In addition, smaller
countries typically stand to gain disproportionately from trade liberalization. This is
due to the simple fact that liberalization will provide access to a larger set of potential
new trading relationship than in case of the larger country gaining enhanced success
to the smaller country's market. 48
The point is that, as shown by the foregoing treaties, a portion of sovereignty may be waived without
violating the Constitution, based on the rationale that the Philippines "adopts the generally accepted
principles of international law as part of the law of the land and adheres to the policy of . . .
cooperation and amity with all nations."
Fourth Issue: The WTO Agreement and Judicial Power
Petitioners aver that paragraph 1, Article 34 of the General Provisions and Basic Principles of the
Agreement on Trade-Related Aspects of Intellectual Property Rights (TRIPS) 49 intrudes on the power
of the Supreme Court to promulgate rules concerning pleading, practice and procedures. 50
To understand the scope and meaning of Article 34, TRIPS,
follows:

51

it will be fruitful to restate its full text as

Article 34
Process Patents: Burden of Proof
1. For the purposes of civil proceedings in respect of the infringement of the rights of
the owner referred to in paragraph 1 (b) of Article 28, if the subject matter of a patent
is a process for obtaining a product, the judicial authorities shall have the authority to
order the defendant to prove that the process to obtain an identical product is
different from the patented process. Therefore, Members shall provide, in at least
one of the following circumstances, that any identical product when produced without
the consent of the patent owner shall, in the absence of proof to the contrary, be
deemed to have been obtained by the patented process:
(a) if the product obtained by the patented process is new;
(b) if there is a substantial likelihood that the identical product was
made by the process and the owner of the patent has been unable
through reasonable efforts to determine the process actually used.
2. Any Member shall be free to provide that the burden of proof indicated in
paragraph 1 shall be on the alleged infringer only if the condition referred to in
subparagraph (a) is fulfilled or only if the condition referred to in subparagraph (b) is
fulfilled.

3. In the adduction of proof to the contrary, the legitimate interests of defendants in


protecting their manufacturing and business secrets shall be taken into account.
From the above, a WTO Member is required to provide a rule of disputable (not the words "in the
absence of proof to the contrary") presumption that a product shown to be identical to one produced
with the use of a patented process shall be deemed to have been obtained by the (illegal) use of the
said patented process, (1) where such product obtained by the patented product is new, or (2) where
there is "substantial likelihood" that the identical product was made with the use of the said patented
process but the owner of the patent could not determine the exact process used in obtaining such
identical product. Hence, the "burden of proof" contemplated by Article 34 should actually be
understood as the duty of the alleged patent infringer to overthrow such presumption. Such burden,
properly understood, actually refers to the "burden of evidence" (burden of going forward) placed on
the producer of the identical (or fake) product to show that his product was produced without the use
of the patented process.
The foregoing notwithstanding, the patent owner still has the "burden of proof" since, regardless of
the presumption provided under paragraph 1 of Article 34, such owner still has to introduce evidence
of the existence of the alleged identical product, the fact that it is "identical" to the genuine one
produced by the patented process and the fact of "newness" of the genuine product or the fact of
"substantial likelihood" that the identical product was made by the patented process.
The foregoing should really present no problem in changing the rules of evidence as the present law
on the subject, Republic Act No. 165, as amended, otherwise known as the Patent Law, provides a
similar presumption in cases of infringement of patented design or utility model, thus:
Sec. 60. Infringement. Infringement of a design patent or of a patent for utility
model shall consist in unauthorized copying of the patented design or utility model for
the purpose of trade or industry in the article or product and in the making, using or
selling of the article or product copying the patented design or utility model. Identity
or substantial identity with the patented design or utility model shall constitute
evidence of copying. (emphasis supplied)
Moreover, it should be noted that the requirement of Article 34 to provide a disputable presumption
applies only if (1) the product obtained by the patented process in NEW or (2) there is a substantial
likelihood that the identical product was made by the process and the process owner has not been
able through reasonable effort to determine the process used. Where either of these two provisos
does not obtain, members shall be free to determine the appropriate method of implementing the
provisions of TRIPS within their own internal systems and processes.
By and large, the arguments adduced in connection with our disposition of the third issue
derogation of legislative power will apply to this fourth issue also. Suffice it to say that the
reciprocity clause more than justifies such intrusion, if any actually exists. Besides, Article 34 does
not contain an unreasonable burden, consistent as it is with due process and the concept of
adversarial dispute settlement inherent in our judicial system.

So too, since the Philippine is a signatory to most international conventions on patents, trademarks
and copyrights, the adjustment in legislation and rules of procedure will not be substantial. 52
Fifth Issue: Concurrence Only in the WTO Agreement and
Not in Other Documents Contained in the Final Act
Petitioners allege that the Senate concurrence in the WTO Agreement and its annexes but not in
the other documents referred to in the Final Act, namely the Ministerial Declaration and Decisions
and the Understanding on Commitments in Financial Services is defective and insufficient and
thus constitutes abuse of discretion. They submit that such concurrence in the WTO
Agreement alone is flawed because it is in effect a rejection of the Final Act, which in turn was the
document signed by Secretary Navarro, in representation of the Republic upon authority of the
President. They contend that the second letter of the President to the Senate 53 which enumerated
what constitutes the Final Act should have been the subject of concurrence of the Senate.
"A final act, sometimes called protocol de cloture, is an instrument which records the winding up of
the proceedings of a diplomatic conference and usually includes a reproduction of the texts of
treaties, conventions, recommendations and other acts agreed upon and signed by the
plenipotentiaries attending the conference." 54 It is not the treaty itself. It is rather a summary of the
proceedings of a protracted conference which may have taken place over several years. The text of the
"Final Act Embodying the Results of the Uruguay Round of Multilateral Trade Negotiations" is contained in
just one page 55 in Vol. I of the 36-volume Uruguay Round of Multilateral Trade Negotiations. By signing
said Final Act, Secretary Navarro as representative of the Republic of the Philippines undertook:
(a) to submit, as appropriate, the WTO Agreement for the consideration of their
respective competent authorities with a view to seeking approval of the Agreement in
accordance with their procedures; and
(b) to adopt the Ministerial Declarations and Decisions.
The assailed Senate Resolution No. 97 expressed concurrence in exactly what the Final Act
required from its signatories, namely, concurrence of the Senate in the WTO Agreement.
The Ministerial Declarations and Decisions were deemed adopted without need for ratification. They
were approved by the ministers by virtue of Article XXV: 1 of GATT which provides that
representatives of the members can meet "to give effect to those provisions of this Agreement which
invoke joint action, and generally with a view to facilitating the operation and furthering the objectives
of this Agreement." 56
The Understanding on Commitments in Financial Services also approved in Marrakesh does not
apply to the Philippines. It applies only to those 27 Members which "have indicated in their
respective schedules of commitments on standstill, elimination of monopoly, expansion of operation
of existing financial service suppliers, temporary entry of personnel, free transfer and processing of
information, and national treatment with respect to access to payment, clearing systems and
refinancing available in the normal course of business." 57

On the other hand, the WTO Agreement itself expresses what multilateral agreements are deemed
included as its integral parts, 58 as follows:
Article II
Scope of the WTO
1. The WTO shall provide the common institutional frame-work for the conduct of
trade relations among its Members in matters to the agreements and associated
legal instruments included in the Annexes to this Agreement.
2. The Agreements and associated legal instruments included in Annexes 1, 2, and
3, (hereinafter referred to as "Multilateral Agreements") are integral parts of this
Agreement, binding on all Members.
3. The Agreements and associated legal instruments included in Annex 4 (hereinafter
referred to as "Plurilateral Trade Agreements") are also part of this Agreement for
those Members that have accepted them, and are binding on those Members. The
Plurilateral Trade Agreements do not create either obligation or rights for Members
that have not accepted them.
4. The General Agreement on Tariffs and Trade 1994 as specified in annex 1A
(hereinafter referred to as "GATT 1994") is legally distinct from the General
Agreement on Tariffs and Trade, dated 30 October 1947, annexed to the Final Act
adopted at the conclusion of the Second Session of the Preparatory Committee of
the United Nations Conference on Trade and Employment, as subsequently rectified,
amended or modified (hereinafter referred to as "GATT 1947").
It should be added that the Senate was well-aware of what it was concurring in as shown by the
members' deliberation on August 25, 1994. After reading the letter of President Ramos dated August
11, 1994, 59 the senators
of the Republic minutely dissected what the Senate was concurring in, as follows: 60
THE CHAIRMAN: Yes. Now, the question of the validity of the submission came up in
the first day hearing of this Committee yesterday. Was the observation made by
Senator Taada that what was submitted to the Senate was not the agreement on
establishing the World Trade Organization by the final act of the Uruguay Round
which is not the same as the agreement establishing the World Trade Organization?
And on that basis, Senator Tolentino raised a point of order which, however, he
agreed to withdraw upon understanding that his suggestion for an alternative solution
at that time was acceptable. That suggestion was to treat the proceedings of the
Committee as being in the nature of briefings for Senators until the question of the
submission could be clarified.
And so, Secretary Romulo, in effect, is the President submitting a new . . . is he
making a new submission which improves on the clarity of the first submission?

MR. ROMULO: Mr. Chairman, to make sure that it is clear cut and there should be no
misunderstanding, it was his intention to clarify all matters by giving this letter.
THE CHAIRMAN: Thank you.
Can this Committee hear from Senator Taada and later on Senator Tolentino since
they were the ones that raised this question yesterday?
Senator Taada, please.
SEN. TAADA: Thank you, Mr. Chairman.
Based on what Secretary Romulo has read, it would now clearly appear that what is
being submitted to the Senate for ratification is not the Final Act of the Uruguay
Round, but rather the Agreement on the World Trade Organization as well as the
Ministerial Declarations and Decisions, and the Understanding and Commitments in
Financial Services.
I am now satisfied with the wording of the new submission of President Ramos.
SEN. TAADA. . . . of President Ramos, Mr. Chairman.
THE CHAIRMAN. Thank you, Senator Taada. Can we hear from Senator Tolentino?
And after him Senator Neptali Gonzales and Senator Lina.
SEN. TOLENTINO, Mr. Chairman, I have not seen the new submission actually
transmitted to us but I saw the draft of his earlier, and I think it now complies with the
provisions of the Constitution, and with the Final Act itself . The Constitution does not
require us to ratify the Final Act. It requires us to ratify the Agreement which is now
being submitted. The Final Act itself specifies what is going to be submitted to with
the governments of the participants.
In paragraph 2 of the Final Act, we read and I quote:
By signing the present Final Act, the representatives agree: (a) to submit as
appropriate the WTO Agreement for the consideration of the respective competent
authorities with a view to seeking approval of the Agreement in accordance with their
procedures.
In other words, it is not the Final Act that was agreed to be submitted to the
governments for ratification or acceptance as whatever their constitutional
procedures may provide but it is the World Trade Organization Agreement. And if
that is the one that is being submitted now, I think it satisfies both the Constitution
and the Final Act itself .
Thank you, Mr. Chairman.

THE CHAIRMAN. Thank you, Senator Tolentino, May I call on Senator Gonzales.
SEN. GONZALES. Mr. Chairman, my views on this matter are already a matter of
record. And they had been adequately reflected in the journal of yesterday's session
and I don't see any need for repeating the same.
Now, I would consider the new submission as an act ex abudante cautela.
THE CHAIRMAN. Thank you, Senator Gonzales. Senator Lina, do you want to make
any comment on this?
SEN. LINA. Mr. President, I agree with the observation just made by Senator
Gonzales out of the abundance of question. Then the new submission is, I believe,
stating the obvious and therefore I have no further comment to make.
Epilogue
In praying for the nullification of the Philippine ratification of the WTO Agreement, petitioners are
invoking this Court's constitutionally imposed duty "to determine whether or not there has been
grave abuse of discretion amounting to lack or excess of jurisdiction" on the part of the Senate in
giving its concurrence therein via Senate Resolution No. 97. Procedurally, a writ
of certiorari grounded on grave abuse of discretion may be issued by the Court under Rule 65 of the
Rules of Court when it is amply shown that petitioners have no other plain, speedy and adequate
remedy in the ordinary course of law.
By grave abuse of discretion is meant such capricious and whimsical exercise of judgment as is
equivalent to lack of jurisdiction. 61 Mere abuse of discretion is not enough. It must be grave abuse of
discretion as when the power is exercised in an arbitrary or despotic manner by reason of passion or
personal hostility, and must be so patent and so gross as to amount to an evasion of a positive duty or to
a virtual refusal to perform the duty enjoined or to act at all in contemplation of law. 62 Failure on the part
of the petitioner to show grave abuse of discretion will result in the dismissal of the petition. 63
In rendering this Decision, this Court never forgets that the Senate, whose act is under review, is one
of two sovereign houses of Congress and is thus entitled to great respect in its actions. It is itself a
constitutional body independent and coordinate, and thus its actions are presumed regular and done
in good faith. Unless convincing proof and persuasive arguments are presented to overthrow such
presumptions, this Court will resolve every doubt in its favor. Using the foregoing well-accepted
definition of grave abuse of discretion and the presumption of regularity in the Senate's processes,
this Court cannot find any cogent reason to impute grave abuse of discretion to the Senate's
exercise of its power of concurrence in the WTO Agreement granted it by Sec. 21 of Article VII of the
Constitution. 64
It is true, as alleged by petitioners, that broad constitutional principles require the State to develop an
independent national economy effectively controlled by Filipinos; and to protect and/or prefer Filipino
labor, products, domestic materials and locally produced goods. But it is equally true that such
principles while serving as judicial and legislative guides are not in themselves sources of

causes of action. Moreover, there are other equally fundamental constitutional principles relied upon
by the Senate which mandate the pursuit of a "trade policy that serves the general welfare and
utilizes all forms and arrangements of exchange on the basis of equality and reciprocity" and the
promotion of industries "which are competitive in both domestic and foreign markets," thereby
justifying its acceptance of said treaty. So too, the alleged impairment of sovereignty in the exercise
of legislative and judicial powers is balanced by the adoption of the generally accepted principles of
international law as part of the law of the land and the adherence of the Constitution to the policy of
cooperation and amity with all nations.
That the Senate, after deliberation and voting, voluntarily and overwhelmingly gave its consent to the
WTO Agreement thereby making it "a part of the law of the land" is a legitimate exercise of its
sovereign duty and power. We find no "patent and gross" arbitrariness or despotism "by reason of
passion or personal hostility" in such exercise. It is not impossible to surmise that this Court, or at
least some of its members, may even agree with petitioners that it is more advantageous to the
national interest to strike down Senate Resolution No. 97. But that is not a legal reason to attribute
grave abuse of discretion to the Senate and to nullify its decision. To do so would constitute grave
abuse in the exercise of our own judicial power and duty. Ineludably, what the Senate did was a valid
exercise of its authority. As to whether such exercise was wise, beneficial or viable is outside the
realm of judicial inquiry and review. That is a matter between the elected policy makers and the
people. As to whether the nation should join the worldwide march toward trade liberalization and
economic globalization is a matter that our people should determine in electing their policy makers.
After all, the WTO Agreement allows withdrawal of membership, should this be the political desire of
a member.
The eminent futurist John Naisbitt, author of the best seller Megatrends, predicts an Asian
Renaissance 65 where "the East will become the dominant region of the world economically, politically
and culturally in the next century." He refers to the "free market" espoused by WTO as the "catalyst" in
this coming Asian ascendancy. There are at present about 31 countries including China, Russia and
Saudi Arabia negotiating for membership in the WTO. Notwithstanding objections against possible
limitations on national sovereignty, the WTO remains as the only viable structure for multilateral trading
and the veritable forum for the development of international trade law. The alternative to WTO is isolation,
stagnation, if not economic self-destruction. Duly enriched with original membership, keenly aware of the
advantages and disadvantages of globalization with its on-line experience, and endowed with a vision of
the future, the Philippines now straddles the crossroads of an international strategy for economic
prosperity and stability in the new millennium. Let the people, through their duly authorized elected
officers, make their free choice.
WHEREFORE, the petition is DISMISSED for lack of merit.
SO ORDERED.

Republic of the Philippines


SUPREME COURT
Manila
SECOND DIVISION
G.R. No. 155504

June 26, 2009

PROFESSIONAL VIDEO, INC., Petitioner,


vs.
TECHNICAL EDUCATION AND SKILLS DEVELOPMENT AUTHORITY, Respondent.
DECISION
BRION, J.:
We resolve the petition filed by Professional Video, Inc. (PROVI)1 to annul and set aside the
Decision2 of the Court of Appeals (CA) in CA-G.R. SP No. 67599, and its subsequent Order denying
PROVIs motion for reconsideration.3 The assailed CA decision nullified:
a. the Order4 dated July 16, 2001 of the Regional Trial Court (RTC), Pasig City, in Civil Case No.
68527, directing the attachment/garnishment of the properties of respondent Technical Education
and Skills Development Authority (TESDA) amounting to Thirty Five Million Pesos (P35,000,000.00);
and
b. the RTCs August 24, 2001 Order5 denying respondent TESDAs motion to discharge/quash writ of
attachment.
THE FACTUAL BACKGROUND
PROVI is an entity engaged in the sale of high technology equipment, information technology
products and broadcast devices, including the supply of plastic card printing and security facilities.
TESDA is an instrumentality of the government established under Republic Act (R.A.) No. 7796 (the
TESDA Act of 1994) and attached to the Department of Labor and Employment (DOLE) to "develop
and establish a national system of skills standardization, testing, and certification in the country." 6 To
fulfill this mandate, it sought to issue security-printed certification and/or identification polyvinyl
(PVC) cards to trainees who have passed the certification process.
TESDAs Pre-Qualification Bids Award Committee (PBAC) conducted two (2) public biddings on
June 25, 1999 and July 22, 1999 for the printing and encoding of PVC cards. A failure of bidding
resulted in both instances since only two (2) bidders PROVI and Sirex Phils. Corp. submitted
proposals.
Due to the failed bidding, the PBAC recommended that TESDA enter into a negotiated contract with
PROVI. On December 29, 1999, TESDA and PROVI signed and executed their "Contract Agreement
Project: PVC ID Card Issuance" (the Contract Agreement) for the provision of goods and services in
the printing and encoding of PVC cards.7 Under this Contract Agreement, PROVI was to provide
TESDA with the system and equipment compliant with the specifications defined in the Technical
Proposal. In return, TESDA would pay PROVI the amount of Thirty-Nine Million Four Hundred and

Seventy-Five Thousand Pesos (P39,475,000) within fifteen (15) days after TESDAs acceptance of
the contracted goods and services.
On August 24, 2000, TESDA and PROVI executed an "Addendum to the Contract Agreement
Project: PVC ID Card Issuance" (Addendum),8 whose terms bound PROVI to deliver one hundred
percent (100%) of the enumerated supplies to TESDA consisting of five hundred thousand (500,000)
pieces of security foil; five (5) pieces of security die with TESDA seal; five hundred thousand
(500,000) pieces of pre-printed and customized identification cards; one hundred thousand
(100,000) pieces of scannable answer sheets; and five hundred thousand (500,000) customized
TESDA holographic laminate. In addition, PROVI would install and maintain the following equipment:
one (1) unit of Micropoise, two (2) units of card printer, three (3) units of flatbed scanner, one (1) unit
of OMR scanner, one (1) unit of Server, and seven (7) units of personal computer.
TESDA in turn undertook to pay PROVI thirty percent (30%) of the total cost of the supplies within
thirty (30) days after receipt and acceptance of the contracted supplies, with the balance payable
within thirty (30) days after the initial payment.
According to PROVI, it delivered the following items to TESDA on the dates indicated:
Date

Particulars

26 April 2000

48,500 pre-printed cards

P 2,764,500.00

07 June 2000

330,000 pre-printed cards

18,810,000.00

07 August 2000

121,500 pre-printed cards

6,925,500.00

26 April 2000

100,000 scannable answer sheets

600,000.00

06 June 2000

5 Micro-Poise customized die

375,000.00

13 June 2000

35 boxes @ 15,000 imp/box


Custom hologram Foil
Total

Amount

10,000,000.00
P 39,475,000.00

PROVI further alleged that out of TESDAs liability of P39,475,000.00, TESDA paid PROVI
only P3,739,500.00, leaving an outstanding balance of P35,735,500.00, as evidenced by PROVIs
Statement of Account.9 Despite the two demand letters dated March 8 and April 27, 2001 that PROVI
sent TESDA,10 the outstanding balance remained unpaid.
On July 11, 2001, PROVI filed with the RTC a complaint for sum of money with damages against
TESDA. PROVI additionally prayed for the issuance of a writ of preliminary attachment/garnishment
against TESDA. The case was docketed as Civil Case No. 68527. In an Order dated July 16, 2001,
the RTC granted PROVIs prayer and issued a writ of preliminary attachment against the properties
of TESDA not exempt from execution in the amount of P35,000,000.00.11
TESDA responded on July 24, 2001 by filing a Motion to Discharge/Quash the Writ of Attachment,
arguing mainly that public funds cannot be the subject of garnishment.12 The RTC denied TESDAs
motion, and subsequently ordered the manager of the Land Bank of the Philippines to produce
TESDAs bank statement for the garnishment of the covered amount.13

Faced with these rulings, TESDA filed a Petition for Certiorari with the CA to question the RTC
orders, imputing grave abuse of discretion amounting to lack or excess of jurisdiction on the trial
court for issuing a writ of preliminary attachment against TESDAs public funds. 14
The CA set aside the RTCs orders after finding that: (a) TESDAs funds are public in nature and,
therefore, exempt from garnishment; and (b) TESDAs purchase of the PVC cards was a necessary
incident of its governmental function; consequently, it ruled that there was no legal basis for the
issuance of a writ of preliminary attachment/garnishment. 15 The CA subsequently denied PROVIs
motion for reconsideration;16 hence, the present petition.
THE PETITION
The petition submits to this Court the single issue of whether or not the writ of attachment against
TESDA and its funds, to cover PROVIs claim against TESDA, is valid. The issue involves a pure
question of law and requires us to determine whether the CA was correct in ruling that the RTC
gravely abused its discretion in issuing a writ of attachment against TESDA.
PROVI argues that the CA should have dismissed TESDAs petition for certiorari as the RTC did not
commit any grave abuse of discretion when it issued the Orders dated July 16, 2001 and August 24,
2001. According to PROVI, the RTC correctly found that when TESDA entered into a purely
commercial contract with PROVI, TESDA went to the level of an ordinary private citizen and could no
longer use the defense of state immunity from suit. PROVI further contends that it has alleged
sufficient ultimate facts in the affidavit it submitted to support its application for a writ of preliminary
attachment. Lastly, PROVI maintains that sufficient basis existed for the RTCs grant of the writ of
preliminary attachment, since TESDA fraudulently misapplied or embezzled the money earmarked
for the payment of the contracted supplies and services, as evidenced by the Certification as to
Availability of Funds.
TESDA claims that it entered the Contract Agreement and Addendum in the performance of its
governmental function to develop and establish a national system of skills standardization, testing,
and certification; in the performance of this governmental function, TESDA is immune from suit. Even
assuming that it had impliedly consented to be sued by entering into a contract with PROVI, TESDA
posits that the RTC still did not have the power to garnish or attach its funds since these are public
funds. Lastly, TESDA points out that PROVI failed to comply with the elements for the valid issuance
of a writ of preliminary attachment, as set forth in Section 1, Rule 57 of the 1997 Rules of Civil
Procedure.
THE COURTS RULING
We find, as the CA did, that the RTCs questioned order involved a gross misreading of the law and
jurisprudence amounting to action in excess of its jurisdiction. Hence, we resolve to DENY PROVIs
petition for lack of merit.
TESDA is an instrumentality of the government undertaking governmental functions.
R.A. No. 7796 created the Technical Education and Skills Development Authority or TESDA under
the declared "policy of the State to provide relevant, accessible, high quality and efficient technical
education and skills development in support of the development of high quality Filipino middle-level
manpower responsive to and in accordance with Philippine development goals and
priorities."17 TESDA replaced and absorbed the National Manpower and Youth Council, the Bureau of
Technical and Vocational Education and the personnel and functions pertaining to technicalvocational education in the regional offices of the Department of Education, Culture and Sports and

the apprenticeship program of the Bureau of Local Employment of the DOLE. 18 Thus, TESDA is an
unincorporated instrumentality of the government operating under its own charter.
Among others, TESDA is empowered to: approve trade skills standards and trade tests as
established and conducted by private industries; establish and administer a system of accreditation
of both public and private institutions; establish, develop and support the institutions' trainors' training
and/or programs; exact reasonable fees and charges for such tests and trainings conducted, and
retain such earnings for its own use, subject to guidelines promulgated by the Authority; and perform
such other duties and functions necessary to carry out the provisions of the Act, consistent with the
purposes of the creation of TESDA.19
Within TESDAs structure, as provided by R.A. No. 7769, is a Skills Standards and Certification
Office expressly tasked, among others, to develop and establish a national system of skills
standardization, testing and certification in the country; and to conduct research and development on
various occupational areas in order to recommend policies, rules and regulations for effective and
efficient skills standardization, testing and certification system in the country.20 The law likewise
mandates that "[T]here shall be national occupational skills standards to be established by TESDAaccredited industry committees. The TESDA shall develop and implement a certification and
accreditation program in which private groups and trade associations are accredited to conduct
approved trade tests, and the local government units to promote such trade testing activities in their
respective areas in accordance with the guidelines to be set by the TESDA. The Secretary of Labor
and Employment shall determine the occupational trades for mandatory certification. All certificates
relating to the national trade skills testing and certification system shall be issued by the TESDA
through its Secretariat."21
All these measures are undertaken pursuant to the constitutional command that "[T]he State affirms
labor as a primary social economic force," and shall "protect the rights of workers and promote their
welfare";22 that "[T]he State shall protect and promote the right of all citizens to quality education at
all levels, and shall take appropriate steps to make such education accessible to all"; 23 in order "to
afford protection to labor" and "promote full employment and equality of employment opportunities
for all."24
Under these terms, both constitutional and statutory, we do not believe that the role and status of
TESDA can seriously be contested: it is an unincorporated instrumentality of the government,
directly attached to the DOLE through the participation of the Secretary of Labor as its Chairman, for
the performance of governmental functions i.e., the handling of formal and non-formal education
and training, and skills development. As an unincorporated instrumentality operating under a specific
charter, it is equipped with both express and implied powers,25 and all State immunities fully apply to
it.26
TESDA, as an agency of the State, cannot be sued without its consent.
The rule that a state may not be sued without its consent is embodied in Section 3, Article XVI of the
1987 Constitution and has been an established principle that antedates this Constitution. 27 It is as
well a universally recognized principle of international law that exempts a state and its organs from
the jurisdiction of another state.28 The principle is based on the very essence of sovereignty, and on
the practical ground that there can be no legal right as against the authority that makes the law on
which the right depends.29 It also rests on reasons of public policy that public service would be
hindered, and the public endangered, if the sovereign authority could be subjected to law suits at the
instance of every citizen and, consequently, controlled in the uses and dispositions of the means
required for the proper administration of the government. 30

The proscribed suit that the state immunity principle covers takes on various forms, namely: a suit
against the Republic by name; a suit against an unincorporated government agency; a suit against a
government agency covered by a charter with respect to the agencys performance of governmental
functions; and a suit that on its face is against a government officer, but where the ultimate liability
will fall on the government. In the present case, the writ of attachment was issued against a
government agency covered by its own charter. As discussed above, TESDA performs governmental
functions, and the issuance of certifications is a task within its function of developing and
establishing a system of skills standardization, testing, and certification in the country. From the
perspective of this function, the core reason for the existence of state immunity applies i.e., the
public policy reason that the performance of governmental function cannot be hindered or delayed
by suits, nor can these suits control the use and disposition of the means for the performance of
governmental functions. In Providence Washington Insurance Co. v. Republic of the Philippines, 31 we
said:
[A] continued adherence to the doctrine of non-suability is not to be deplored for as against the
inconvenience that may be caused private parties, the loss of governmental efficiency and the
obstacle to the performance of its multifarious functions are far greater if such a fundamental
principle were abandoned and the availability of judicial remedy were not thus restricted. With the
well known propensity on the part of our people to go to court, at the least provocation, the loss of
time and energy required to defend against law suits, in the absence of such a basic principle that
constitutes such an effective obstacle, could very well be imagined.
PROVI argues that TESDA can be sued because it has effectively waived its immunity when it
entered into a contract with PROVI for a commercial purpose. According to PROVI, since the
purpose of its contract with TESDA is to provide identification PVC cards with security seal which
TESDA will thereafter sell to TESDA trainees, TESDA thereby engages in commercial transactions
not incidental to its governmental functions.
TESDAs response to this position is to point out that it is not engaged in business, and there is
nothing in the records to show that its purchase of the PVC cards from PROVI is for a business
purpose. While TESDA admits that it will charge the trainees with a fee for the PVC cards, it claims
that this fee is only to recover their costs and is not intended for profit.
We agree with TESDA. As the appellate court found, the PVC cards purchased by TESDA from
PROVI are meant to properly identify the trainees who passed TESDAs National Skills Certification
Program the program that immediately serves TESDAs mandated function of developing and
establishing a national system of skills standardization, testing, and certification in the
country.32 Aside from the express mention of this function in R.A. No. 7796, the details of this function
are provided under DOLE Administrative Order No. 157, S. 1992, as supplemented by Department
Order Nos. 3 thru 3-F, S. 1994 and Department Order No. 13, S. 1994.33
Admittedly, the certification and classification of trainees may be undertaken in ways other than the
issuance of identification cards, as the RTC stated in its assailed Order.34 How the mandated
certification is to be done, however, lies within the discretion of TESDA as an incident of its
mandated function, and is a properly delegated authority that this Court cannot inquire into, unless
its exercise is attended by grave abuse of discretion.
That TESDA sells the PVC cards to its trainees for a fee does not characterize the transaction as
industrial or business; the sale, expressly authorized by the TESDA Act,35 cannot be considered
separately from TESDAs general governmental functions, as they are undertaken in the discharge
of these functions. Along this line of reasoning, we held in Mobil Philippines v. Customs Arrastre
Services:36

Now, the fact that a non-corporate government entity performs a function proprietary in nature does
not necessarily result in its being suable. If said non-governmental function is undertaken as an
incident to its governmental function, there is no waiver thereby of the sovereign immunity from suit
extended to such government entity.
TESDAs funds are public in character, hence exempt from attachment or garnishment.
Even assuming that TESDA entered into a proprietary contract with PROVI and thereby gave its
implied consent to be sued, TESDAs funds are still public in nature and, thus, cannot be the valid
subject of a writ of garnishment or attachment. Under Section 33 of the TESDA Act, the TESDA
budget for the implementation of the Act shall be included in the annual General Appropriation Act;
hence, TESDA funds, being sourced from the Treasury, are moneys belonging to the government, or
any of its departments, in the hands of public officials.37 We specifically spoke of the limits in dealing
with this fund in Republic v. Villasor38 when we said:
This fundamental postulate underlying the 1935 Constitution is now made explicit in the revised
charter. It is therein expressly provided, The State may not be sued without its consent. A corollary,
both dictated by logic and sound sense, from such a basic concept, is that public funds cannot be
the object of garnishment proceedings even if the consent to be sued had been previously granted
and the state liability adjudged. Thus in the recent case of Commissioner of Public Highways vs. San
Diego, such a well-settled doctrine was restated in the opinion of Justice Teehankee:
The universal rule that where the State gives its consent to be sued by private parties either by
general or special law, it may limit claimant's action 'only up to the completion of proceedings
anterior to the stage of execution' and that the power of the Courts ends when the judgment is
rendered, since government funds and properties may not be seized under writs of execution or
garnishment to satisfy such judgments, is based on obvious considerations of public
policy. Disbursements of public funds must be covered by the corresponding appropriation as
required by law. The functions and public services rendered by the State cannot be allowed to be
paralyzed or disrupted by the diversion of public funds from their legitimate and specific objects, as
appropriated by law. [Emphasis supplied.]
We reiterated this doctrine in Traders Royal Bank v. Intermediate Appellate Court, 39 where we said:
The NMPCs implied consent to be sued notwithstanding, the trial court did not have the power to
garnish NMPC deposits to answer for any eventual judgment against it. Being public funds, the
deposits are not within the reach of any garnishment or attachment proceedings. [Emphasis
supplied.]
As pointed out by TESDA in its Memorandum,40 the garnished funds constitute TESDAs lifeblood
in government parlance, its MOOE41 whose withholding via a writ of attachment, even on a
temporary basis, would paralyze TESDAs functions and services. As well, these funds also include
TESDAs Personal Services funds from which salaries of TESDA personnel are sourced. Again and
for obvious reasons, the release of these funds cannot be delayed.
PROVI has not shown that it is entitled to the writ of attachment.
Even without the benefit of any immunity from suit, the attachment of TESDA funds should not have
been granted, as PROVI failed to prove that TESDA "fraudulently misapplied or converted funds
allocated under the Certificate as to Availability of Funds." Section 1, Rule 57 of the Rules of Court
sets forth the grounds for issuance of a writ of preliminary attachment, as follows:

SECTION 1. Grounds upon which attachment may issue. A plaintiff or any proper party may, at the
commencement of the action or at any time thereafter, have the property of the adverse party
attached as security for the satisfaction of any judgment that may be recovered in the following
cases:
(a) In an action for recovery of a specified amount of money or damages, other than moral
and exemplary, on a cause of action arising from law, contract, quasi-contract, delict or
quasi-delict against a party who is about to depart from the Philippines with intent to defraud
his creditors;
(b) In an action for money or property embezzled or fraudulently misapplied or converted to
his use by a public officer, or an officer of a corporation, or an attorney, factor, broker, agent
or clerk, in the course of his employment as such, or by any other person in a fiduciary
capacity, or for a willful violation of duty;
(c) In an action to recover the possession of property unjustly or fraudulently taken, detained
or converted, when the property or any part thereof, has been concealed, removed or
disposed of to prevent its being found or taken by the applicant or an authorized person;
(d) In an action against a party who has been guilty of fraud in contracting the debt or
incurring the obligation upon which the action is brought, or in concealing or disposing of the
property for the taking, detention or conversion of which the action is brought;
(e) In an action against a party who has removed or disposed of his property, or is about to
do so, with intent to defraud his creditors;
(f) In an action against a party who does not reside and is not found in the Philippines, or on
whom summons may be served by publication. [Emphasis supplied.]
Jurisprudence teaches us that the rule on the issuance of a writ of attachment must be construed
strictly in favor of the defendant. Attachment, a harsh remedy, must be issued only on concrete and
specific grounds and not on general averments merely quoting the words of the pertinent
rules.42 Thus, the applicants affidavit must contain statements clearly showing that the ground relied
upon for the attachment exists.
Section 1(b), Rule 57 of the Rules of Court, that PROVI relied upon, applies only where money or
property has been embezzled or converted by a public officer, an officer of a corporation, or some
other person who took advantage of his fiduciary position or who willfully violated his duty.
PROVI, in this case, never entrusted any money or property to TESDA. While the Contract
Agreement is supported by a Certificate as to Availability of Funds (Certificate) issued by the Chief of
TESDAs Accounting Division, this Certificate does not automatically confer ownership over the funds
to PROVI. Absent any actual disbursement, these funds form part of TESDAs public funds, and
TESDAs failure to pay PROVI the amount stated in the Certificate cannot be construed as an act of
fraudulent misapplication or embezzlement. In this regard, Section 86 of Presidential Decree No.
1445 (The Accounting Code) provides:
Section 86. Certificate showing appropriation to meet contract. Except in a case of a contract for
personal service, for supplies for current consumption or to be carried in stock not exceeding the
estimated consumption for three months, or banking transactions of government-owned or controlled
banks, no contract involving the expenditure of public funds by any government agency shall be

entered into or authorized unless the proper accounting official or the agency concerned shall have
certified to the officer entering into the obligation that funds have been duly appropriated for the
purpose and that the amount necessary to cover the proposed contract for the current fiscal year is
available for expenditure on account thereof, subject to verification by the auditor concerned. The
certification signed by the proper accounting official and the auditor who verified it, shall be attached
to and become an integral part of the proposed contract, and the sum so certified shall not thereafter
be available for expenditure for any other purpose until the obligation of the government agency
concerned under the contract is fully extinguished. [Emphasis supplied.]
By law, therefore, the amount stated in the Certification should be intact and remains devoted to its
purpose since its original appropriation. PROVI can rebut the presumption that necessarily arises
from the cited provision only by evidence to the contrary. No such evidence has been adduced.
Section 1 (d), Rule 57 of the Rules of Court applies where a party is guilty of fraud in contracting a
debt or incurring an obligation, or in concealing or disposing of the property for the taking, detention
or conversion of which the action is brought. In Wee v. Tankiansee, 43 we held that for a writ of
attachment to issue under this Rule, the applicant must sufficiently show the factual circumstances of
the alleged fraud because fraudulent intent cannot be inferred from the debtors mere non-payment
of the debt or failure to comply with his obligation. The affidavit, being the foundation of the writ, must
contain particulars showing how the imputed fraud was committed for the court to decide whether or
not to issue the writ. To reiterate, a writ of attachment can only be granted on concrete and specific
grounds and not on general averments merely quoting the words of the rules. 44
The affidavit filed by PROVI through Elmer Ramiro, its President and Chief Executive Officer, only
contained a general allegation that TESDA had fraudulent misapplied or converted the amount
of P10,975,000.00 that was allotted to it. Clearly, we cannot infer any finding of fraud from PROVIs
vague assertion, and the CA correctly ruled that the lower court acted with grave abuse of discretion
in granting the writ of attachment despite want of any valid ground for its issuance.
1avvphi1

For all these reasons, we support the appellate courts conclusion that no valid ground exists to
support the grant of the writ of attachment against TESDA. The CAs annulment and setting aside of
the Orders of the RTC were therefore fully in order.
WHEREFORE, premises considered, we hereby DENY the petition filed by petitioner Professional
Video, Inc., and AFFIRM the Court of Appeals Decision dated July 23, 2002, and Resolution of
September 27, 2002, in CA-G.R. SP No. 67599. Costs against the petitioner.
SO ORDERED.

Republic of the Philippines


SUPREME COURT
Manila
EN BANC
G.R. No. 154705

June 26, 2003

THE REPUBLIC OF INDONESIA, HIS EXCELLENCY AMBASSADOR SOERATMIN, and


MINISTER COUNSELLOR AZHARI KASIM, Petitioners,
vs.
JAMES VINZON, doing business under the name and style of VINZON TRADE AND
SERVICES,Respondent.
DECISION
AZCUNA, J:
This is a petition for review on certiorari to set aside the Decision of the Court of Appeals dated May
30, 2002 and its Resolution dated August 16, 2002, in CA-G.R. SP No. 66894 entitled "The Republic
of Indonesia, His Excellency Ambassador Soeratmin and Minister Counselor Azhari Kasim v. Hon.
Cesar Santamaria, Presiding Judge, RTC Branch 145, Makati City, and James Vinzon, doing
business under the name and style of Vinzon Trade and Services."
Petitioner, Republic of Indonesia, represented by its Counsellor, Siti Partinah, entered into a
Maintenance Agreement in August 1995 with respondent James Vinzon, sole proprietor of Vinzon
Trade and Services. The Maintenance Agreement stated that respondent shall, for a consideration,
maintain specified equipment at the Embassy Main Building, Embassy Annex Building and the
Wisma Duta, the official residence of petitioner Ambassador Soeratmin. The equipment covered by
the Maintenance Agreement are air conditioning units, generator sets, electrical facilities, water
heaters, and water motor pumps. It is likewise stated therein that the agreement shall be effective for
a period of four years and will renew itself automatically unless cancelled by either party by giving
thirty days prior written notice from the date of expiry.1
Petitioners claim that sometime prior to the date of expiration of the said agreement, or before
August 1999, they informed respondent that the renewal of the agreement shall be at the discretion
of the incoming Chief of Administration, Minister Counsellor Azhari Kasim, who was expected to
arrive in February 2000. When Minister Counsellor Kasim assumed the position of Chief of
Administration in March 2000, he allegedly found respondents work and services unsatisfactory and
not in compliance with the standards set in the Maintenance Agreement. Hence, the Indonesian
Embassy terminated the agreement in a letter dated August 31, 2000. 2 Petitioners claim, moreover,
that they had earlier verbally informed respondent of their decision to terminate the agreement.
On the other hand, respondent claims that the aforesaid termination was arbitrary and unlawful.
Respondent cites various circumstances which purportedly negated petitioners alleged
dissatisfaction over respondents services: (a) in July 2000, Minister Counsellor Kasim still requested
respondent to assign to the embassy an additional full-time worker to assist one of his other workers;
(b) in August 2000, Minister Counsellor Kasim asked respondent to donate a prize, which the latter
did, on the occasion of the Indonesian Independence Day golf tournament; and (c) in a letter dated
August 22, 2000, petitioner Ambassador Soeratmin thanked respondent for sponsoring a prize and
expressed his hope that the cordial relations happily existing between them will continue to prosper
and be strengthened in the coming years.

Hence, on December 15, 2000, respondent filed a complaint 3 against petitioners docketed as Civil
Case No. 18203 in the Regional Trial Court (RTC) of Makati, Branch 145. On February 20, 2001,
petitioners filed a Motion to Dismiss, alleging that the Republic of Indonesia, as a foreign sovereign
State, has sovereign immunity from suit and cannot be sued as a party-defendant in the Philippines.
The said motion further alleged that Ambassador Soeratmin and Minister Counsellor Kasim are
diplomatic agents as defined under the Vienna Convention on Diplomatic Relations and therefore
enjoy diplomatic immunity.4 In turn, respondent filed on March 20, 2001, an Opposition to the said
motion alleging that the Republic of Indonesia has expressly waived its immunity from suit. He based
this claim upon the following provision in the Maintenance Agreement:
"Any legal action arising out of this Maintenance Agreement shall be settled according to the laws of
the Philippines and by the proper court of Makati City, Philippines."
Respondents Opposition likewise alleged that Ambassador Soeratmin and Minister Counsellor
Kasim can be sued and held liable in their private capacities for tortious acts done with malice and
bad faith.5
On May 17, 2001, the trial court denied herein petitioners Motion to Dismiss. It likewise denied the
Motion for Reconsideration subsequently filed.
The trial courts denial of the Motion to Dismiss was brought up to the Court of Appeals by herein
petitioners in a petition for certiorari and prohibition. Said petition, docketed as CA-G.R. SP No.
66894, alleged that the trial court gravely abused its discretion in ruling that the Republic of
Indonesia gave its consent to be sued and voluntarily submitted itself to the laws and jurisdiction of
Philippine courts and that petitioners Ambassador Soeratmin and Minister Counsellor Kasim waived
their immunity from suit.
On May 30, 2002, the Court of Appeals rendered its assailed decision denying the petition for lack of
merit.6 On August 16, 2002, it denied herein petitioners motion for reconsideration. 7
Hence, this petition.
In the case at bar, petitioners raise the sole issue of whether or not the Court of Appeals erred in
sustaining the trial courts decision that petitioners have waived their immunity from suit by using as
its basis the abovementioned provision in the Maintenance Agreement.
The petition is impressed with merit.
International law is founded largely upon the principles of reciprocity, comity, independence, and
equality of States which were adopted as part of the law of our land under Article II, Section 2 of the
1987 Constitution.8 The rule that a State may not be sued without its consent is a necessary
consequence of the principles of independence and equality of States. 9 As enunciated in Sanders v.
Veridiano II,10 the practical justification for the doctrine of sovereign immunity is that there can be no
legal right against the authority that makes the law on which the right depends. In the case of foreign
States, the rule is derived from the principle of the sovereign equality of States, as expressed in the

maxim par in parem non habet imperium. All states are sovereign equals and cannot assert
jurisdiction over one another.11 A contrary attitude would "unduly vex the peace of nations."12
The rules of International Law, however, are neither unyielding nor impervious to change. The
increasing need of sovereign States to enter into purely commercial activities remotely connected
with the discharge of their governmental functions brought about a new concept of sovereign
immunity. This concept, the restrictive theory, holds that the immunity of the sovereign is recognized
only with regard to public acts or acts jure imperii, but not with regard to private acts or acts jure
gestionis.13
In United States v. Ruiz,14 for instance, we held that the conduct of public bidding for the repair of a
wharf at a United States Naval Station is an act jure imperii. On the other hand, we considered as an
act jure gestionis the hiring of a cook in the recreation center catering to American servicemen and
the general public at the John Hay Air Station in Baguio City,15 as well as the bidding for the
operation of barber shops in Clark Air Base in Angeles City.16
Apropos the present case, the mere entering into a contract by a foreign State with a private party
cannot be construed as the ultimate test of whether or not it is an act jure imperii or jure gestionis.
Such act is only the start of the inquiry. Is the foreign State engaged in the regular conduct of a
business? If the foreign State is not engaged regularly in a business or commercial activity, and in
this case it has not been shown to be so engaged, the particular act or transaction must then be
tested by its nature. If the act is in pursuit of a sovereign activity, or an incident thereof, then it is an
act jure imperii.17
Hence, the existence alone of a paragraph in a contract stating that any legal action arising out of
the agreement shall be settled according to the laws of the Philippines and by a specified court of
the Philippines is not necessarily a waiver of sovereign immunity from suit. The aforesaid provision
contains language not necessarily inconsistent with sovereign immunity. On the other hand, such
provision may also be meant to apply where the sovereign party elects to sue in the local courts, or
otherwise waives its immunity by any subsequent act. The applicability of Philippine laws must be
deemed to include Philippine laws in its totality, including the principle recognizing sovereign
immunity. Hence, the proper court may have no proper action, by way of settling the case, except to
dismiss it.
Submission by a foreign state to local jurisdiction must be clear and unequivocal. It must be given
explicitly or by necessary implication. We find no such waiver in this case.
Respondent concedes that the establishment of a diplomatic mission is a sovereign function. On the
other hand, he argues that the actual physical maintenance of the premises of the diplomatic
mission, such as the upkeep of its furnishings and equipment, is no longer a sovereign function of
the State.18
1wphi1

We disagree. There is no dispute that the establishment of a diplomatic mission is an act jure imperii.
A sovereign State does not merely establish a diplomatic mission and leave it at that; the
establishment of a diplomatic mission encompasses its maintenance and upkeep. Hence, the State
may enter into contracts with private entities to maintain the premises, furnishings and equipment of

the embassy and the living quarters of its agents and officials. It is therefore clear that petitioner
Republic of Indonesia was acting in pursuit of a sovereign activity when it entered into a contract
with respondent for the upkeep or maintenance of the air conditioning units, generator sets, electrical
facilities, water heaters, and water motor pumps of the Indonesian Embassy and the official
residence of the Indonesian ambassador.
The Solicitor General, in his Comment, submits the view that, "the Maintenance Agreement was
entered into by the Republic of Indonesia in the discharge of its governmental functions. In such a
case, it cannot be deemed to have waived its immunity from suit." As to the paragraph in the
agreement relied upon by respondent, the Solicitor General states that it "was not a waiver of their
immunity from suit but a mere stipulation that in the event they do waive their immunity, Philippine
laws shall govern the resolution of any legal action arising out of the agreement and the proper court
in Makati City shall be the agreed venue thereof. 19
On the matter of whether or not petitioners Ambassador Soeratmin and Minister Counsellor Kasim
may be sued herein in their private capacities, Article 31 of the Vienna Convention on Diplomatic
Relations provides:
xxx
1. A diplomatic agent shall enjoy immunity from the criminal jurisidiction of the receiving State. He
shall also enjoy immunity from its civil and administrative jurisdiction, except in the case of:
(a) a real action relating to private immovable property situated in the territory of the
receiving State, unless he holds it on behalf of the sending State for the purposes of the
mission;
(b) an action relating to succession in which the diplomatic agent is involved as executor,
administrator, heir or legatee as a private person and not on behalf of the sending State;
(c) an action relating to any professional or commercial activity exercised by the diplomatic
agent in the receiving State outside his official functions.
xxx
The act of petitioners Ambassador Soeratmin and Minister Counsellor Kasim in terminating the
Maintenance Agreement is not covered by the exceptions provided in the abovementioned provision.
The Solicitor General believes that said act may fall under subparagraph (c) thereof, 20 but said
provision clearly applies only to a situation where the diplomatic agent engages in any professional
or commercial activity outside official functions, which is not the case herein.
WHEREFORE, the petition is hereby GRANTED. The decision and resolution of the Court of
Appeals in CA G.R. SP No. 66894 are REVERSED and SET ASIDE and the complaint in Civil Case
No. 18203 against petitioners is DISMISSED.

Republic of the Philippines


SUPREME COURT
Manila
SECOND DIVISION
G.R. No. 115634

April 27, 2000

FELIPE CALUB and RICARDO VALENCIA, DEPARTMENT of ENVIRONMENT and NATURAL


RESOURCES (DENR), CATBALOGAN, SAMAR, petitioners,
vs.
COURT OF APPEALS, MANUELA T. BABALCON, and CONSTANCIO ABUGANDA, respondents.

QUISUMBING, J.:
For review is the decision1 dated May 27, 1994, of the Court of Appeals in CA-G.R. SP No. 29191,
denying the petition filed by herein petitioners for certiorari, prohibition and mandamus, in order to
annul the Order dated May 27, 1992, by the Regional Trial Court of Catbalogan, Samar. Said Order
had denied petitioners' (a) Motion to Dismiss the replevin case filed by herein private respondents,
as well as (b) petitioners Motion for Reconsideration of the Order of said trial court dated April 24,
1992, granting an application for a Writ of replevin.2
The pertinent facts of the case, borne by the records, are as follows:
On January 28, 1992, the Forest Protection and Law Enforcement Team of the Community
Environment and Natural Resources Office (CENRO) of the DENR apprehended two (2) motor
vehicles, described as follows:
1. Motor Vehicle with Plate No. HAK-733 loaded with one thousand and twenty six (1,026)
board feet of illegally sourced lumber valued at P8,544.75, being driven by one Pio Gabon
and owned by [a certain] Jose Vargas.
2. Motor Vehicle with Plate No. FCN-143 loaded with one thousand two hundred twenty four
and ninety seven (1,224.97) board feet of illegally-sourced lumber valued at P9,187.27,
being driven by one Constancio Abuganda and owned by [a certain] Manuela Babalcon. . . . 3
Constancio Abuganda and Pio Gabon, the drivers of the vehicles, failed to present proper
documents and/or licenses. Thus, the apprehending team seized and impounded the vehicles and
its load of lumber at the DENR-PENR (Department of Environment and Natural ResourcesProvincial Environment and Natural Resources) Office in Catbalogan. 4 Seizure receipts were issued
but the drivers refused to accept the receipts.5 Felipe Calub, Provincial Environment and Natural
Resources Officer, then filed before the Provincial Prosecutor's Office in Samar, a criminal complaint
against Abuganda, in Criminal Case No. 3795, for violation of Section 68 [78], Presidential Decree
705 as amended by Executive Order 277, otherwise known as the Revised Forestry Code. 6

On January 31, 1992, the impounded vehicles were forcibly taken by Gabon and Abuganda from the
custody of the DENR, prompting DENR Officer Calub this time to file a criminal complaint for grave
coercion against Gabon and Abuganda. The complaint was, however, dismissed by the Public
Prosecutor.7
On February 11, 1992, one of the two vehicles, with plate number FCN 143, was again apprehended
by a composite team of DENR-CENR in Catbalogan and Philippine Army elements of the 802nd
Infantry Brigade at Barangay Buray, Paranas, Samar. It was again loaded with forest products with
an equivalent volume of 1,005.47 board feet, valued at P10,054.70. Calub duly filed a criminal
complaint against Constancio Abuganda, a certain Abegonia, and several John Does, in Criminal
Case No. 3625, for violation of Section 68 [78], Presidential Decree 705 as amended by Executive
Order 277, otherwise known as the Revised Forestry Code. 8
In Criminal Cases Nos. 3795 and 3625, however, Abegonia and Abuganda were acquitted on the
ground of reasonable doubt. But note the trial court ordered that a copy of the decision be furnished
the Secretary of Justice, in order that the necessary criminal action may be filed against Noe
Pagarao and all other persons responsible for violation of the Revised Forestry Code. For it
appeared that it was Pagarao who chartered the subject vehicle and ordered that cut timber be
loaded on it.9
Subsequently, herein private respondents Manuela Babalcon, the vehicle owner, and Constancio
Abuganda, the driver, filed a complaint for the recovery of possession of the two (2) impounded
vehicles with an application for replevin against herein petitioners before the RTC of Catbalogan.
The trial court granted the application for replevin and issued the corresponding writ in an Order
dated April 24, 1992. 10 Petitioners filed a motion to dismiss which was denied by the trial court. 11
Thus, on June 15, 1992, petitioners filed with the Supreme Court the present Petition for Certiorari,
Prohibition and Mandamus with application for Preliminary Injunction and/or a Temporary
Restraining Order. The Court issued a TRO, enjoining respondent RTC judge from conducting
further proceedings in the civil case for replevin; and enjoining private respondents from taking or
attempting to take the motor vehicles and forest products seized from the custody of the petitioners.
The Court further instructed the petitioners to see to it that the motor vehicles and other forest
products seized are kept in a secured place and protected from deterioration, said property being
in custodia legis and subject to the direct order of the Supreme Court. 12 In a Resolution issued on
September 28, 1992, the Court referred said petition to respondent appellate court for appropriate
disposition. 13
On May 27, 1994, the Court of Appeals denied said petition for lack of merit. It ruled that the mere
seizure of a motor vehicle pursuant to the authority granted by Section 68 [78] of P.D. No. 705 as
amended by E.O. No. 277 does not automatically place said conveyance in custodia legis. According
to the appellate court, such authority of the Department Head of the DENR or his duly authorized
representative to order the confiscation and disposition of illegally obtained forest products and the
conveyance used for that purpose is not absolute and unqualified. It is subject to pertinent laws,
regulations, or policies on that matter, added the appellate court. The DENR Administrative Order
No. 59, series of 1990, is one such regulation, the appellate court said. For it prescribes the
guidelines in the confiscation, forfeiture and disposition of conveyances used in the commission of
offenses penalized under Section 68 [78] of P.D. No. 705 as amended by E.O. No. 277. 14
Additionally, respondent Court of Appeals noted that the petitioners failed to observe the procedure
outlined in DENR Administrative Order No. 59, series of 1990. They were unable to submit a report
of the seizure to the DENR Secretary, to give a written notice to the owner of the vehicle, and to
render a report of their findings and recommendations to the Secretary. Moreover, petitioners' failure

to comply with the procedure laid down by DENR Administrative Order No. 59, series of 1990, was
confirmed by the admission of petitioners' counsel that no confiscation order has been issued prior to
the seizure of the vehicle and the filing of the replevin suit. Therefore, in failing to follow such
procedure, according to the appellate court, the subject vehicles could not be considered in custodia
legis. 15
Respondent Court of Appeals also found no merit in petitioners' claim that private respondents'
complaint for replevin is a suit against the State. Accordingly, petitioners could not shield themselves
under the principle of state immunity as the property sought to be recovered in the instant suit had
not yet been lawfully adjudged forfeited in favor of the government. Moreover, according to
respondent appellate court, there could be no pecuniary liability nor loss of property that could ensue
against the government. It reasoned that a suit against a public officer who acted illegally or beyond
the scope of his authority could not be considered a suit against the State; and that a public officer
might be sued for illegally seizing or withholding the possession of the property of another. 16
Respondent court brushed aside other grounds raised by petitioners based on the claim that the
subject vehicles were validly seized and held in custody because they were contradicted by its own
findings. 17 Their petition was found without merit. 18
Now, before us, the petitioners assign the following errors:

19

(1) THE COURT OF APPEALS ERRED IN HOLDING THAT MERE SEIZURE OF A


CONVEYANCE PURSUANT TO SECTION 68-A [78-A] OF P.D. NO. 705 AS AMENDED BY
EXECUTIVE ORDER 277 DOES NOT PLACE SAID CONVEYANCE IN CUSTODIA LEGIS;
(2) THE COURT OF APPEALS ERRED IN NOT HOLDING THAT THE OPERATIVE ACT
GIVING RISE FOR THE SUBJECT CONVEYANCE TO BE IN CUSTODIA LEGIS IS ITS
LAWFUL SEIZURE BY THE DENR PURSUANT TO SECTION 68-A [78-A] OF P.D. NO. 705,
AS AMENDED BY E.O. NO. 277; AND
(3) THE COURT OF APPEALS ERRED IN HOLDING THAT THE COMPLAINT FOR
REPLEVIN AGAINST THE PETITIONERS IS NOT A SUIT AGAINST THE STATE.
In brief, the pertinent issues for our consideration are:
(1) Whether or not the DENR-seized motor vehicle, with plate number FCN 143, is
in custodia legis.
(2) Whether or not the complaint for the recovery of possession of impounded vehicles, with
an application for replevin, is a suit against the State.
We will now resolve both issues.
The Revised Forestry Code authorizes the DENR to seize all conveyances used in the commission
of an offense in violation of Section 78. Section 78 states:
Sec. 78. Cutting, Gathering, and/or Collecting Timber, or Other Forest Products without
License. Any person who shall cut, gather, collect, remove timber or other forest products
from any forestland, or timber from alienable or disposable public land, or from private land,
without any authority, or possess timber or other forest products without the legal documents

as required under existing forest laws and regulations, shall be punished with the penalties
imposed under Articles 309 and 310 of the Revised Penal Code. . .
The Court shall further order the confiscation in favor of the government of the timber or any
forest products cut, gathered, collected, removed, or possessed, as well as the machinery,
equipment, implements and tools illegally used in the area where the timber or forest
products are found.
This provision makes mere possession of timber or other forest products without the accompanying
legal documents unlawful and punishable with the penalties imposed for the crime of theft, as
prescribed in Articles 309-310 of the Revised Penal Code. In the present case, the subject vehicles
were loaded with forest products at the time of the seizure. But admittedly no permit evidencing
authority to possess and transport said load of forest products was duly presented. These products,
in turn, were deemed illegally sourced. Thus there was a prima facie violation of Section 68 [78] of
the Revised Forestry Code, although as found by the trial court, the persons responsible for said
violation were not the ones charged by the public prosecutor.
The corresponding authority of the DENR to seize all conveyances used in the commission of an
offense in violation of Section 78 of the Revised Forestry Code is pursuant to Sections 78-A and 89
of the same Code. They read as follows:
Sec. 78-A. Administrative Authority of the Department Head or His Duly Authorized
Representative to Order Confiscation. In all cases of violation of this Code or other forest
laws, rules and regulations, the Department Head or his duly authorized representative, may
order the confiscation of any forest products illegally cut, gathered, removed, or possessed
or abandoned, and all conveyances used either by land, water or air in the commission of
the offense and to dispose of the same in accordance with pertinent laws, regulations or
policies on the matter.
Sec. 89. Arrest; Institution of criminal actions. A forest officer or employee of the Bureau
[Department] or any personnel of the Philippine Constabulary/Philippine National Police shall
arrest even without warrant any person who has committed or is committing in his presence
any of the offenses defined in this Chapter. He shall also seize and confiscate, in favor of the
Government, the tools and equipment used in committing the offense. . . [Emphasis
supplied.]
Note that DENR Administrative Order No. 59, series of 1990, implements Sections 78-A and 89 of
the Forestry Code, as follows:
Sec. 2. Conveyances Subject to Confiscation and Forfeiture. All conveyances used in the
transport of any forest product obtained or gathered illegally whether or not covered with
transport documents, found spurious or irregular in accordance with Sec. 68-A [78-A] of P.D.
No. 705, shall be confiscated in favor of the government or disposed of in accordance with
pertinent laws, regulations or policies on the matter.
Sec. 4. Who are Authorized to Seize Conveyance. The Secretary or his duly authorized
representative such as the forest officers and/or natural resources officers, or deputized
officers of the DENR areauthorized to seize said conveyances subject to policies and
guidelines pertinent thereto. Deputized military personnel and officials of other agencies
apprehending illegal logs and other forest products and their conveyances shall notify the
nearest DENR field offices, and turn oversaid forest products and conveyances for proper
action and disposition. In case where the apprehension is made by DENR field officer, the

conveyance shall be deposited with the nearest CENRO/PENRO/RED Office as the case
may be, for safekeeping wherever it is most convenient and secured. [Emphasis supplied.]
Upon apprehension of the illegally-cut timber while being transported without pertinent documents
that could evidence title to or right to possession of said timber, a warrantless seizure of the involved
vehicles and their load was allowed under Section 78 and 89 of the Revised Forestry Code.
Note further that petitioners' failure to observe the procedure outlined in DENR Administrative Order
No. 59, series of 1990 was justifiably explained. Petitioners did not submit a report of the seizure to
the Secretary nor give a written notice to the owner of the vehicle because on the 3rd day following
the seizure, Gabon and Abuganda, drivers of the seized vehicles, forcibly took the impounded
vehicles from the custody of the DENR. Then again, when one of the motor vehicles was
apprehended and impounded for the second time, the petitioners, again were not able to report the
seizure to the DENR Secretary nor give a written notice to the owner of the vehicle because private
respondents immediately went to court and applied for a writ of replevin. The seizure of the vehicles
and their load was done upon their apprehension for a violation of the Revised Forestry Code. It
would be absurd to require a confiscation order or notice and hearing before said seizure could be
effected under the circumstances.
Since there was a violation of the Revised Forestry Code and the seizure was in accordance with
law, in our view the subject vehicles were validly deemed in custodia legis. It could not be subject to
an action for replevin. For it is property lawfully taken by virtue of legal process and considered in the
custody of the law, and not otherwise. 20
In Mamanteo, et. al. v. Deputy Sheriff Magumun, A.M. No. P-98-1264, promulgated on July 28, 1999,
the case involves property to be seized by a Deputy Sheriff in a replevin suit. But said property were
already impounded by the DENR due to violation of forestry laws and, in fact, already forfeited in
favor of the government by order of the DENR. We said that such property was deemed in custodia
legis. The sheriff could not insist on seizing the property already subject of a prior warrant of seizure.
The appropriate action should be for the sheriff to inform the trial court of the situation by way of
partial Sheriff's Return, and wait for the judge's instructions on the proper procedure to be observed.
Note that property that is validly deposited in custodia legis cannot be the subject of a replevin suit.
In Mamanteo v. Deputy Sheriff Magumun, we elucidated further:
. . . the writ of replevin has been repeatedly used by unscrupulous plaintiffs to retrieve their
chattel earlier taken for violation of the Tariff and Customs Code, tax assessment,
attachment or execution. Officers of the court, from the presiding judge to the sheriff, are
implored to be vigilant in their execution of the law otherwise, as in this case, valid seizure
and forfeiture proceedings could easily be undermined by the simple devise of a writ of
replevin. . . 21
On the second issue, is the complaint for the recovery of possession of the two impounded vehicles,
with an application for replevin, a suit against the State?
Well established is the doctrine that the State may not be sued without its consent. 22 And a suit
against a public officer for his official acts is, in effect, a suit against the State if its purpose is to hold
the State ultimately liable. 23However, the protection afforded to public officers by this doctrine
generally applies only to activities within the scope of their authority in good faith and without
willfulness, malice or corruption. 24 In the present case, the acts for which the petitioners are being
called to account were performed by them in the discharge of their official duties. The acts in
question are clearly official in nature. 25 In implementing and enforcing Sections 78-A and 89 of the

Forestry Code through the seizure carried out, petitioners were performing their duties and functions
as officers of the DENR, and did so within the limits of their authority. There was no malice nor bad
faith on their part. Hence, a suit against the petitioners who represent the DENR is a suit against the
State. It cannot prosper without the State's consent.
Given the circumstances in this case, we need not pursue the Office of the Solicitor General's line for
the defense of petitioners concerning exhaustion of administrative remedies. We ought only to recall
that exhaustion must be raised at the earliest time possible, even before filing the answer to the
complaint or pleading asserting a claim, by a motion to dismiss. 26 If not invoked at the proper time,
this ground for dismissal could be deemed waived and the court could take cognizance of the case
and try it. 27
ACCORDINGLY, the Petition is GRANTED, and the assailed Decision of the Court of Appeals in CAG.R. SP No. 29191 is SET ASIDE. Consequently, the Order issued by the Regional Trial Court of
Catbalogan, dated May 27, 1992, and the Writ of replevin issued in the Order dated April 24, 1992,
are ANNULLED. The Sheriff of the Regional Trial Court of Catbalogan, Branch 29, is directed to take
possession of the subject motor vehicle, with plate number FCN 143, for delivery to the custody of
and appropriate disposition by petitioners. Let a copy of this decision be provided the Honorable
Secretary of Justice for his appropriate action, against any and all persons responsible for the
abovecited violation of the Revised Forestry Code.
1wphi1

Costs against private respondents.

1wphi1.nt

SO ORDERED.

Republic of the Philippines


SUPREME COURT
Manila
SECOND DIVISION
G.R. No. 102667

February 23, 2000

AMADO J. LANSANG, petitioner,


vs.
COURT OF APPEALS, GENERAL ASSEMBLY OF THE BLIND, INC., and JOSE
IGLESIAS, respondents.
QUISUMBING, J.:
Before us is a petition to review the decision of the Court of Appeals in C.A. G.R. CV No. 27244,
which set aside the ruling of the Regional Trial Court, Manila, Branch 8, in Civil Case No. 88-43887,
and ordered petitioner Amado J. Lansang to pay private respondent Jose Iglesias P50,000.00 in
moral damages, P10,000.00 in exemplary damages and P5,000.00 in attorney's fees.
Like public streets, public parks are beyond the commerce of man. However, private respondents
were allegedly awarded a "verbal contract of lease" in 1970 by the National Parks Development

Committee (NPDC), a government initiated civic body engaged in the development of national parks,
including Rizal Park,1 but actually administered by high profile civic leaders and journalists. Whoever
in NPDC gave such "verbal" accommodation to private respondents was unclear, for indeed no
document or instrument appears on record to show the grantor of the verbal license to private
respondents to occupy a portion of the government park dedicated to the national hero's memory.
Private respondents were allegedly given office and library space as well as kiosks area selling food
and drinks. One such kiosk was located along T.M. Kalaw St., in front of the Army and Navy Club.
Private respondent General Assembly of the Blind, Inc. (GABI) was to remit to NPDC, 40 percent of
the profits derived from operating the kiosks,2 without again anything shown in the record who
received the share of the profits or how they were used or spent.
With the change of government after the EDSA Revolution, the new Chairman of the NPDC, herein
petitioner, sought to clean up Rizal Park. In a written notice dated February 23, 1988 and received
by private respondents on February 29, 1988, petitioner terminated the so-called verbal agreement
with GABI and demanded that the latter vacate the premises and the kiosks it ran privately within the
public park.3 In another notice dated March 5, 1988, respondents were given until March 8, 1988 to
vacate.4
The latter notice was signed by private respondent Iglesias, GABI president, allegedly to indicate his
conformity to its contents. However, Iglesias, who is totally blind, claims that he was deceived into
signing the notice. He was allegedly told by Ricardo Villanueva, then chief warden of Rizal Park, that
he was merely acknowledging receipt of the notice. Although blind, Iglesias as president was
knowledgeable enough to run GABI as well as its business.
On the day of the supposed eviction, GABI filed an action for damages and injunction in the
Regional Trial Court against petitioner, Villanueva, and "all persons acting on their behalf". 5 The trial
court issued a temporary restraining order on the same day.6
The TRO expired on March 28, 1988. The following day, GABI was finally evicted by NPDC.
GABI's action for damages and injunction was subsequently dismissed by the RTC, ruling that the
complaint was actually directed against the State which could not be sued without its consent.
Moreover, the trial court ruled that GABI could not claim damages under the alleged oral lease
agreement since GABI was a mere accommodation concessionaire. As such, it could only recover
damages upon proof of the profits it could realize from the conclusion. The trial court noted that no
such proof was presented.
1wphi1.nt

On appeal, the Court of Appeals reversed the decision of the trial court.
The Court of Appeals ruled that the mere allegation that a government official is being sued in his
official capacity is not enough to protect such official from liability for acts done without or in excess
of his authority.7 Granting that petitioner had the authority to evict GABI from Rizal Park, "the abusive
and capricious manner in which that authority was exercised amounted to a legal wrong for which he
must now be held liable for damages"8 according to the Court of Appeals.

The Court of Appeals noted that, as the trial court observed, the eviction of GABI came at the heels
of two significant incidents. First, after private respondent Iglesias extended monetary support to
striking workers of the NPDC, and second, after Iglesias sent the Tanodbayan, a letter on November
26, 1987, denouncing alleged graft and corruption in the NPDC.9 These, according to the Court of
Appeals, should not have been taken against GABI, which had been occupying Rizal Park for nearly
20 years. GABI was evicted purportedly for violating its verbal agreement with NPDC. 10 However, the
Court of Appeals pointed out that NPDC failed to present proof of such violation. 11
The Court of Appeals found petitioner liable for damages under Articles 19, 21, and 24 of the Civil
Code.12
The Court of Appeals absolved from liability all other persons impleaded in GABI's complaint since it
appeared that they were merely acting under the orders of petitioner. The new officers of NPDC,
additionally impleaded by GABI, were likewise absolved from liability, absent any showing that they
participated in the acts complained of. Petitioner was ordered to pay private respondent Iglesias
moral and exemplary damages and attorney's fees.
Hence, this petition, in which petitioner raises the following issues:
I. WHETHER OR NOT RESPONDENT COURT ERRED IN NOT HOLDING THAT PRIVATE
RESPONDENTS' COMPLAINT AGAINST PETITIONER, AS CHAIRMAN OF NPDC, AND
HIS CO-DEFENDANTS IN CIVIL CASE NO. 88-43887, IS IN EFFECT A SUIT AGAINST
THE STATE WHICH CANNOT BE SUED WITHOUT ITS CONSENT.
II. WHETHER OR NOT RESPONDENT COURT ERRED IN NOT HOLDING THAT
PETITIONER'S ACT OF TERMINATING RESPONDENT GABI'S CONCESSION IS VALID
AND DONE IN THE LAWFUL PERFORMANCE OF OFFICIAL DUTY.13
Petitioner insists that the complaint filed against him is in reality a complaint against the State, which
could not prosper without the latter's consent. He anchors his argument on the fact that NPDC is a
government agency, and that when he ordered the eviction of GABI, he was acting in his capacity as
chairman of NPDC. Petitioner avers that the mere allegation that he was being sued in his personal
capacity did not remove the case from the coverage of the law of public officers and the doctrine of
state immunity.
Petitioner points out that Iglesias signed the notice of eviction to indicate his conformity thereto. He
contends that as evidence of private respondents' bad faith, they sued petitioner instead of
complying with their undertaking to vacate their library and kiosk at Rizal Park.
Petitioner adds that during the actual eviction, no untoward incident occurred. GABI's properties
were properly inventoried and stored.
According to petitioner, the Court of Appeals' observation that the eviction was prompted by Iglesias'
support for striking NPDC workers and the letter-complaint sent to the Tanodbayan is merely
conjectural.

Finally, petitioner avers that the move to evict GABI and award the spaces it occupied to another
group was an executive policy decision within the discretion of NPDC. GABI's possession of the
kiosks as concessionaire was by mere tolerance of NPDC and, thus, such possession may be
withdrawn at any time, with or without cause.
On the other hand, private respondents aver that petitioner acted beyond the scope of his authority
when he showed malice and bad faith in ordering GABI's ejectment from Rizal Park. Quoting from
the decision of the Court of Appeals, private respondents argue that petitioner is liable for damages
for performing acts "to injure an individual rather than to discharge a public duty." 14
While private respondents recognize the authority of petitioner to terminate the agreement with GABI
"if [the contract] is prejudicial to the interest of the NPDC," 15 they maintain that petitioner's personal
interest, and not that of the NPDC, was the root cause of GABI's ejecment.
The doctrine of state immunity from suit applies to complaints filed against public officials for acts
done in the performance of their duties. The rule is that the suit must be regarded as one against the
state where satisfaction of the judgment against the public official concerned will require the state
itself to perform a positive act, such as appropriation of the amount necessary to pay the damages
awarded to the plaintiff.16
The rule does not apply where the public official is charged in his official capacity for acts that are
unlawful and injurious to the rights of others.17 Public officials are not exempt, in their personal
capacity, from liability arising from acts committed in bad faith.18
Neither does it apply where the public official is clearly being sued not in his official capacity but in
his personal capacity, although the acts complained of may have been committed while he occupied
a public position.
We are convinced that petitioner is being sued not in his capacity as NPDC chairman but in his
personal capacity. The complaint filed by private respondents in the RTC merely identified petitioner
as chairman of the NPDC, but did not categorically state that he is being sued in that capacity.19 Also,
it is evident from paragraph 4 of said complaint that petitioner was sued allegedly for having
personal motives in ordering the ejectment of GABI from Rizal Park.
4. Defendant AMADO J. LANSANG, JR., the Chairman of the National Parks Development
Committee,acting under the spirit of revenge, ill-will, evil motive and personal resentment
against plaintiff JOSE IGLESIAS, served on the plaintiff corporation a letter, dated February
23, 1988 terminating plaintiffs lease agreement with a demand for the plaintiff corporation to
vacate its office premises. . .20 (Emphasis supplied.)
The parties do not dispute that it was petitioner who ordered the ejectment of GABI from their office
and kiosk at Rizal Park. There is also no dispute that petitioner, as chairman of the NPDC which was
the agency tasked to administer Rizal Park, had the authority to terminate the agreement with
GABI21 and order the organization's ejectment. The question now is whether or not petitioner abused
his authority in ordering the ejectment of private respondents.

We find, however, no evidence of such abuse of authority on record. As earlier stated, Rizal Park is
beyond the commerce of man and, thus, could not be the subject of a lease contract. Admittedly,
there was no written contract. That private respondents were allowed to occupy office and kiosk
spaces in the park was only a matter of accommodation by the previous administrator. This being so,
also admittedly, petitioner may validly discontinue the accommodation extended to private
respondents, who may be ejected from the park when necessary. Private respondents cannot and
does not claim a vested right to continue to occupy Rizal Park.
The Court of Appeals awarded private respondent Iglesias moral and exemplary damages and
attorney's fees. However, we find no evidence on record to support Iglesias' claim that he suffered
moral injury as a result of GABI's ejectment from Rizal Park. Absent any satisfactory proof upon
which the Court may base the amount of damages suffered, the award of moral damages cannot be
sustained.22
Neither can we sustain the award of exemplary damages, which may only be awarded in addition to
moral, temperate, liquidated, or compensatory damages. 23 We also disallow the award for attorney's
fees, which can only be recovered per stipulation of the parties, which is absent in this case. There is
no showing that any of the exceptions justifying the award of attorney's fees absent a stipulation is
present in this case.24
WHEREFORE, the instant petition is GRANTED. The decision of the Court of Appeals in CA-G.R.
CV No. 27244 is hereby SET ASIDE, and the DISMISSAL of the complaint for damages by the trial
court for want of merit is AFFIRMED. No costs.
SO ORDERED.

1wphi1.nt

Republic of the Philippines


SUPREME COURT
Manila
THIRD DIVISION
G.R. No. 107271

September 10, 2003

CITY OF CALOOCAN and NORMA M. ABRACIA, petitioners,


vs.
HON. MAURO T. ALLARDE, Presiding Judge of Branch 123, RTC of Caloocan City, ALBERTO
A. CASTILLO, Deputy Sheriff of Branch 123, RTC of Caloocan City, and DELFINA
HERNANDEZ SANTIAGO and PHILIPPINE NATIONAL BANK (PNB), respondents.
CORONA, J.:

Assailed in this petition for certiorari is the decision1 dated August 31, 1992, of the Court of Appeals
in CA G.R. SP No. 27423, ordering the Regional Trial Court of Caloocan City, Branch 123, to
implement an alias writ of execution dated January 16, 1992. The dispositive portion read as follows:
WHEREFORE the petition is hereby granted ordering the Regional Trial Court of Kaloocan
City, Branch 123, to immediately effect the alias writ of execution dated January 16, 1992
without further delay.
Counsel for the respondents are warned that a repetition of their contemptuous act to delay
the execution of a final and executory judgment will be dealt with more severely.
SO ORDERED.2
It is important to state at the outset that the dispute between petitioner and private respondent has
been litigated thrice before this Court: first, in G.R. No. L-39288-89, entitled Heirs of Abelardo
Palomique, et al. vs. Marcial Samson, et al., decided on January 31, 1985; second, in G.R. No.
98366, entitled City Government of Caloocan vs. Court of Appeals, et al., resolved on May 16, 1991,
and third, in G.R. No. 102625, entitled Santiago vs. Sto. Tomas, et al., decided on August 1, 1995.
This is not to mention the numerous concurrent efforts by the City Government of Caloocan to seek
relief from other judicial and quasi-judicial bodies. The present petition for certiorari is the fourth time
we are called upon to resolve the dispute.
The factual and procedural antecedents follow.
Sometime in 1972, Marcial Samson, City Mayor of Caloocan City, through Ordinance No. 1749,
abolished the position of Assistant City Administrator and 17 other positions from the plantilla of the
local government of Caloocan. Then Assistant City Administrator Delfina Hernandez Santiago and
the 17 affected employees of the City Government assailed the legality of the abolition before the
then Court of First Instance (CFI) of Caloocan City, Branch 33.
In 1973, the CFI declared the abolition illegal and ordered the reinstatement of all the dismissed
employees and the payment of their back salaries and other emoluments. The City Government of
Caloocan appealed to the Court of Appeals. Respondent Santiago and her co-parties moved for the
dismissal of the appeal for being dilatory and frivolous but the appellate court denied their motion.
Thus, they elevated the case on certiorari before this Court, docketed as G.R. No. L-39288-89, Heirs
of Abelardo Palomique, et al. vs. Marcial Samson, et al. In our Resolution dated January 31, 1985,
we held that the appellate court "erred in not dismissing the appeal," and "that the appeal of the City
Government of Caloocan was frivolous and dilatory." In due time, the resolution lapsed into finality
and entry of judgment was made on February 27, 1985.
In 1986, the City Government of Caloocan paid respondent Santiago P75,083.37 in partial payment
of her backwages, thereby leaving a balance of P530,761.91. Her co-parties were paid in full.3 In
1987, the City of Caloocan appropriated funds for her unpaid back salaries. This was included in
Supplemental Budget No. 3 for the fiscal year 1987. Surprisingly, however, the City later refused to
release the money to respondent Santiago.
Respondent Santiago exerted effort for the execution of the remainder of the money judgment but
she met stiff opposition from the City Government of Caloocan. On February 12, 1991, Judge Mauro
T. Allarde, RTC of Caloocan City, Branch 123, issued a writ of execution for the payment of the
remainder of respondent Santiagos back salaries and other emoluments. 4

For the second time, the City Government of Caloocan went up to the Court of Appeals and filed a
petition for certiorari, prohibition and injunction to stop the trial court from enforcing the writ of
execution. The CA dismissed the petition and affirmed the order of issuance of the writ of
execution.5 One of the issues raised and resolved therein was the extent to which back salaries and
emoluments were due to respondent Santiago. The appellate court held that she was entitled to her
salaries from October, 1983 to December, 1986.
And for the second time, the City Government of Caloocan appealed to this Court in G.R. No.
98366, City Government of Caloocan vs. Court of Appeals, et al. The petition was dismissed,
through our Resolution of May 16, 1991, for having been filed late and for failure to show any
reversible error on the part of the Court of Appeals. The resolution subsequently attained finality and
the corresponding entry of judgment was made on July 29, 1991.
On motion of private respondent Santiago, Judge Mauro T. Allarde ordered the issuance of an alias
writ of execution on March 3, 1992. The City Government of Caloocan moved to reconsider the
order, insisting in the main that respondent Santiago was not entitled to backwages from 1983 to
1986. The court a quo denied the motion and forthwith issued the alias writ of execution. Unfazed,
the City Government of Caloocan filed a motion to quash the writ, maintaining that the money
judgment sought to be enforced should not have included salaries and allowances for the years
1983-1986. The trial court likewise denied the motion.
On July 27, 1992, Sheriff Alberto A. Castillo levied and sold at public auction one of the motor
vehicles of the City Government of Caloocan, with plate no. SBH-165, for P100,000. The proceeds
of the sale were turned over to respondent Santiago in partial satisfaction of her claim, thereby
leaving a balance of P439,377.14, inclusive of interest. Petitioners filed a motion questioning the
validity of the auction sale of the vehicle with plate no. SBH-165, and a supplemental motion
maintaining that the properties of the municipality were exempt from execution. In his Order dated
October 1, 1992, Judge Allarde denied both motions and directed the sheriff to levy and schedule at
public auction three more vehicles of the City of Caloocan -6</p>
ONE (1) Unit Motor Vehicle (Hunter Station Wagon); Motor No. C-240-199629; Chassis No.
MBB-910369C;
ONE (1) Unit Motor Vehicle (Hunter Series 11-Diesel); Engine No. 4FB1-174328, Chassis
No. MBB-910345C; Plate No. SDL-653;
ONE (1) Unit Motor Vehicle (Hunter Series 11-Diesel); Engine No. 4FB-165196; Chassis No.
MBB 910349C.
All the vehicles, including that previously sold in the auction sale, were owned by the City and
assigned for the use of herein petitioner Norma Abracia, Division Superintendent of Caloocan City,
and other officials of the Division of City Schools.
Meanwhile, the City Government of Caloocan sought clarification from the Civil Service Commission
(CSC) on whether respondent Santiago was considered to have rendered services from 1983-1986
as to be entitled to backwages for that period. In its Resolution No. 91-1124, the CSC ruled in the
negative.
On November 22, 1991, private respondent Santiago challenged the CSC resolution before this
Court in G.R. No. 102625, Santiago vs. Sto. Tomas, et al. On July 8, 1993, we initially dismissed the
petition for lack of merit; however, we reconsidered the dismissal of the petition in our Resolution
dated August 1, 1995, this time ruling in favor of respondent Santiago:

The issue of petitioner Santiagos right to back salaries for the period from October 1983 to
December 1986 having been resolved in G.R. No. 98366 on 16 May 1991, CSC Resolution
No. 91-1124 promulgated later on 24 September 1991 in particular, its ruling on the extent
of backwages due petitioner Santiago was in fact moot and academic at the time of its
promulgation. CSC Resolution No. 91-1124 could not, of course, set aside what had been
judicially decided with finality x x x x the court considers that resort by the City Government
of Caloocan to respondent CSC was but another attempt to deprive petitioner Santiago of
her claim to back salaries x x x and a continuation of the Citys abuse and misuse of the
rules of judicial procedure. The Citys acts have resulted in wasting the precious time and
resources of the courts and respondent CSC. (Underscoring supplied).
On October 5, 1992, the City Council of Caloocan passed Ordinance No. 0134, Series of 1992,
which included the amount of P439,377.14 claimed by respondent Santiago as back salaries, plus
interest.7 Pursuant to the subject ordinance, Judge Allarde issued an order dated November 10,
1992, decreeing that:
WHEREFORE, the City Treasurer (of Caloocan), Norberto Azarcon is hereby ordered to
deliver to this Court within five (5) days from receipt hereof, (a) managers check covering
the amount of P439,378.00 representing the back salaries of petitioner Delfina H. Santiago
in accordance with Ordinance No. 0134 S. 1992 and pursuant to the final and executory
decision in these cases.
Then Caloocan Mayor Macario A. Asistio, Jr., however, refused to sign the check intended as
payment for respondent Santiagos claims. This, despite the fact that he was one of the
signatories of the ordinance authorizing such payment. On April 29, 1993, Judge Allarde
issued another order directing the Acting City Mayor of Caloocan, Reynaldo O. Malonzo, to
sign the check which had been pending before the Office of the Mayor since December 11,
1992. Acting City Mayor Malonzo informed the trial court that "he could not comply with the
order since the subject check was not formally turned over to him by the City Mayor" who
went on official leave of absence on April 15, 1993, and that "he doubted whether he had
authority to sign the same."8
Thus, in an order dated May 7, 1993, Judge Allarde ordered Sheriff Alberto A. Castillo to immediately
garnish the funds of the City Government of Caloocan corresponding to the claim of respondent
Santiago.9 On the same day, Sheriff Alberto A. Castillo served a copy of the Notice of Garnishment
on the Philippine National Bank (PNB), Sangandaan Branch, Caloocan City. When PNB immediately
notified the City of Caloocan of the Notice of Garnishment, the City Treasurer sent a letter-advice
informing PNB that the order of garnishment was "illegal," with a warning that it would hold PNB
liable for any damages which may be caused by the withholding of the funds of the city. PNB opted
to comply with the order of Judge Allarde and released to the Sheriff a managers check amounting
to P439,378. After 21 long years, the claim of private respondent Santiago was finally settled in full.
On June 4, 1993, however, while the instant petition was pending, the City Government of Caloocan
filed yet another motion with this Court, a Motion to Declare in Contempt of Court; to Set Aside the
Garnishment and Administrative Complaint against Judge Allarde, respondent Santiago and PNB.
Subsequently, the City Government of Caloocan filed a Supplemental Petition formally impleading
PNB as a party-respondent in this case.
The instant petition for certiorari is directed this time against the validity of the garnishment of the
funds of the City of Caloocan, as well as the validity of the levy and sale of the motor vehicles
belonging to the City of Caloocan. More specifically, petitioners insist that Judge Allarde gravely
abused his discretion in:

(a) ordering the garnishment of the funds of the City of Caloocan deposited with the PNB,
since it is settled that public funds are beyond the reach of garnishment and even with the
appropriation passed by the City Council, the authority of the Mayor is still needed for the
release of the appropriation;
(b) ordering the levy and sale at public auction of three (3) motor vehicles owned by the City
of Caloocan, which vehicles are necessary for public use and cannot be attached nor sold in
an execution sale to satisfy a money judgment against the City of Caloocan;
(c) peremptorily denying petitioner City of Caloocans urgent motions to vacate and set aside
the auction sale of the motor vehicle with PLATE NO. SBH-165, notwithstanding that the
auction sale by the Sheriff was tainted with serious irregularities, more particularly:
i. non-compliance with the mandatory posting of the notice of sale;
ii. non-observance of the procedure that a sale through public auction has to be
made and consummated at the time of the auction, at the designated place and upon
actual payment of the purchase price by the winning bidder;
iii. violation of Sec. 21, Rule 39 of the Rules of Court to the effect that sale of
personal property capable of manual delivery must be sold within the view of those
attending the sale; and,
iv. the Sheriffs Certificate of Sale contained false narration of facts respecting the
actual time of the public auction;
(d) the enforcement of the levy made by the Sheriff covering the three (3) motor vehicles
based on an alias writ that has long expired.
The petition has absolutely no merit. The trial court committed no grave abuse of discretion
in implementing the alias writ of execution to settle the claim of respondent Santiago, the
satisfaction of which petitioner had been maliciously evading for 21 years.
Petitioner argues that the garnishment of its funds in PNB was invalid inasmuch as these
were public funds and thus exempt from execution. Garnishment is considered a specie of
attachment by means of which the plaintiff seeks to subject to his claim property of the
defendant in the hands of a third person, or money owed by such third person or garnishee
to the defendant.10
The rule is and has always been that all government funds deposited in the PNB or any other official
depositary of the Philippine Government by any of its agencies or instrumentalities, whether by
general or special deposit, remain government funds and may not be subject to garnishment or levy,
in the absence of a corresponding appropriation as required by law: 11
Even though the rule as to immunity of a state from suit is relaxed, the power of the courts
ends when the judgment is rendered. Although the liability of the state has been judicially
ascertained, the state is at liberty to determine for itself whether to pay the judgment or not,
and execution cannot issue on a judgment against the state. Such statutes do not authorize
a seizure of state property to satisfy judgments recovered, and only convey an implication
that the legislature will recognize such judgment as final and make provision for the
satisfaction thereof.12

The rule is based on obvious considerations of public policy. The functions and public services
rendered by the State cannot be allowed to be paralyzed or disrupted by the diversion of public
funds from their legitimate and specific objects, as appropriated by law.13
However, the rule is not absolute and admits of a well-defined exception, that is, when there is a
corresponding appropriation as required by law. Otherwise stated, the rule on the immunity of public
funds from seizure or garnishment does not apply where the funds sought to be levied under
execution are already allocated by law specifically for the satisfaction of the money judgment against
the government. In such a case, the monetary judgment may be legally enforced by judicial
processes.
Thus, in the similar case of Pasay City Government, et al. vs. CFI of Manila, Br. X, et al.,14 where
petitioners challenged the trial courts order garnishing its funds in payment of the contract price for
the construction of the City Hall, we ruled that, while government funds deposited in the PNB are
exempt from execution or garnishment, this rule does not apply if an ordinance has already been
enacted for the payment of the Citys obligations
Upon the issuance of the writ of execution, the petitioner-appellants moved for its quashal
alleging among other things the exemption of the government from execution. This move on
the part of petitioner-appellants is at first glance laudable for all government funds deposited
with the Philippine National Bank by any agency or instrumentality of the government,
whether by way of general or special deposit, remain government funds and may not be
subject to garnishment or levy. But inasmuch as an ordinance has already been enacted
expressly appropriating the amount of P613,096.00 as payment to the respondent-appellee,
then the herein case is covered by the exception to the general rule x x x x
In the instant case, the City Council of Caloocan already approved and passed Ordinance No. 0134,
Series of 1992, allocating the amount of P439,377.14 for respondent Santiagos back salaries plus
interest. Thus this case fell squarely within the exception. For all intents and purposes, Ordinance
No. 0134, Series of 1992, was the "corresponding appropriation as required by law." The sum
indicated in the ordinance for Santiago were deemed automatically segregated from the other
budgetary allocations of the City of Caloocan and earmarked solely for the Citys monetary obligation
to her. The judgment of the trial court could then be validly enforced against such funds.
Indeed, this conclusion is further buttressed by the Certification issued on December 23, 1992 by
Norberto C. Azarcon, City Treasurer of Caloocan:
CERTIFICATION
This is to certify that according to the records available in this Office the claim for backwages
of the HON. JUDGE DELFINA H. SANTIAGO has been properly obligated and can be
collected in accordance with existing accounting and auditing rules and regulations.
This is to certify further that in case the claim is not collected within the present fiscal year,
such claim shall be entered in the books of Accounts Payable and can still be collected in the
next fiscal year x x x x (Underscoring supplied)
Petitioners reliance on Municipality of Makati vs. Court of Appeals, et al.,15 and Commissioner of
Public Highways vs. San Diego,16 does not help their cause.17 Both cases implicitly affirmed that
public funds may be garnished if there is a statute which appropriated the amount so
garnished. Thus, in Municipality of Makati, citing San Diego, we unequivocally held that:

In this jurisdiction, well-settled is the rule that public funds are not subject to levy and
execution, unless otherwise provided by statute x x x x
Similarly, we cannot agree with petitioners argument that the appropriation ordinance of the
City Council did not authorize PNB to release the funds because only the City Mayor could
authorize the release thereof. A valid appropriation of public funds lifts its exemption from
execution. Here, the appropriation passed by the City Council of Caloocan providing for the
payment of backwages to respondent was duly approved and signed by both the council and
then Mayor Macario Asistio, Jr. The mayors signature approving the budget ordinance was
his assent to the appropriation of funds for respondent Santiagos backwages. If he did not
agree with such allocation, he could have vetoed the item pursuant to Section 55 of the
Local Government Code.18 There was no such veto.
In view of the foregoing discourse, we dismiss petitioners unfounded assertion, probably made more
out of sheer ignorance of prevailing jurisprudence than a deliberate attempt to mislead us, that the
rule that "public funds (are) beyond the reach of levy and garnishment is not qualified by any
condition."19
We now come to the issue of the legality of the levy on the three motor vehicles belonging to the City
of Caloocan which petitioners claimed to be exempt from execution, and which levy was based on
an alias writ that had purportedly expired. Suffice it to say that Judge Allarde, in his Order dated
November 10, 1992,20 already lifted the levy on the three vehicles, thereby formally discharging them
from the jurisdiction of the court and turning them over to the City Government of Caloocan:
x x x x the levy of the three (3) vehicles made by Sheriff Alberto Castillo pursuant to the
Orders of this Court dated October 1 and 8, 1992 is hereby lifted and the said Sheriff is
hereby ordered to return the same to the City Government in view of the satisfaction of the
decision in these cases x x x x
It is thus unnecessary for us to discuss a moot issue.
We turn to the third issue raised by petitioners that the auction sale by Sheriff Alberto A.
Castillo of the motor vehicle with plate no. SBH-165 was tainted with serious irregularities.
We need not emphasize that the sheriff enjoys the presumption of regularity in the
performance of the functions of his office. This presumption prevails in the absence of
substantial evidence to the contrary and cannot be overcome by bare and self-serving
allegations. The petitioners failed to convince us that the auction sale conducted by the
sheriff indeed suffered from fatal flaws. No evidence was adduced to prove that the sheriff
had been remiss in the performance of his duties during the public auction sale. Indeed it
would be injudicious for us to assume, as petitioners want us to do, that the sheriff failed to
follow the established procedures governing public auctions.
On the contrary, a review of the records shows that the sheriff complied with the rules on
public auction. The sale of the Citys vehicle was made publicly in front of the Caloocan City
Hall on the date fixed in the notice July 27, 1992. In fact, petitioners in their Motion to
Declare in Contempt of Court; to Set Aside the Garnishment and Administrative Complaint
admitted as much:
On July 27, 1992, by virtue of an alias writ of execution issued by the respondent court, a
vehicle owned by the petitioner xxx was levied and sold at public auction for the amount
of P100,000.00 and which amount was immediately delivered to the private respondent x x x
x21

Hence, petitioners cannot now be heard to impugn the validity of the auction sale.
Petitioners, in desperation, likewise make much of the proceedings before the trial court on October
8, 1992, wherein petitioner Norma Abracia, Superintendent of the Division of City Schools of
Caloocan, was commanded to appear and show cause why she should not be cited in contempt for
delaying the execution of judgment. This was in connection with her failure (or refusal) to surrender
the three motor vehicles assigned to the Division of City Schools to the custody of the sheriff.
Petitioner Abracia, assisted by Mr. Ricardo Nagpacan of the Division of City Schools, appeared
during the hearing but requested a ten-day period within which to refer the matter of contempt to a
counsel of her choice. The request was denied by Judge Allarde in his assailed order dated October
8, 1992. Thus petitioner Abracia claimed, inter alia, that: (a) she was denied due process; (b) the
silence of the order of Judge Allarde on her request for time violated an orderly and faithful recording
of the proceedings, and (c) she was coerced into agreeing to surrender the vehicles.
We do not think so. What violates due process is the absolute lack of opportunity to be heard. That
opportunity, the Court is convinced, was sufficiently accorded to petitioner Abracia. She was notified
of the contempt charge against her; she was effectively assisted by counsel when she appeared
during the hearing on October 8, 1992; and she was afforded ample opportunity to answer and
refute the charge against her. The circumstance that she opted not to avail of her chance to be
heard on that occasion by asking for an extension of time within which to hire a counsel of her
choice, a request denied by the trial court, did not transgress nor deprive her of her right to due
process.
Significantly, during the hearing on October 8, 1992, Mr. Nagpacan manifested in open court that,
after conferring with petitioner Abracia, the latter was "willing to surrender these vehicles into the
custody of the sheriff on the condition that the standing motion (for contempt) be withdrawn."22 Her
decision was made freely and voluntarily, and after conferring with her counsel. Moreover, it was
petitioner Abracia herself who imposed the condition that respondent Santiago should withdraw her
motion for contempt in exchange for her promise to surrender the subject vehicles. Thus, petitioner
Abracias claim that she was coerced into surrendering the vehicles had no basis.
Even assuming ex gratia argumenti that there indeed existed certain legal infirmities in connection
with the assailed orders of Judge Allarde, still, considering the totality of circumstances of this case,
the nullification of the contested orders would be way out of line. For 21 long years, starting 1972
when this controversy started up to 1993 when her claim was fully paid out of the garnished funds of
the City of Caloocan, respondent Santiago was cruelly and unjustly deprived of what was due her. It
would be, at the very least, merciless and unchristian to make private respondent refund the City of
Caloocan the amount already paid to her, only to force her to go through the same nightmare all over
again.
At any rate, of paramount importance to us is that justice has been served. No right of the public was
violated and public interest was preserved.
Finally, we cannot simply pass over in silence the deplorable act of the former Mayor of Caloocan
City in refusing to sign the check in payment of the Citys obligation to private respondent. It was an
open defiance of judicial processes, smacking of political arrogance, and a direct violation of the very
ordinance he himself approved. Our Resolution in G.R. No. 98366, City Government of Caloocan vs.
Court of Appeals, et al., dated May 16, 1991, dismissing the petition of the City of Caloocan assailing
the issuance of a writ of execution by the trial court, already resolved with finality all impediments to
the execution of judgment in this case. Yet, the City Government of Caloocan, in a blatant display of
malice and bad faith, refused to comply with the decision. Now, it has the temerity to come to this

Court once more and continue inflicting injustice on a hapless citizen, as if all the harm and prejudice
it has already heaped upon respondent Santiago are still not enough.
This Court will not condone the repudiation of just obligations contracted by municipal corporations.
On the contrary, we will extend our aid and every judicial facility to any citizen in the enforcement of
just and valid claims against abusive local government units.
WHEREFORE, the petition is hereby DISMISSED for utter lack of merit. The assailed orders of the
trial court dated October 1, 1992, October 8, 1992 and May 7, 1993, respectively, are AFFIRMED.
Petitioners and their counsels are hereby warned against filing any more pleadings in connection
with the issues already resolved with finality herein and in related cases.
Costs against petitioners.

Republic of the Philippines


SUPREME COURT
Manila
FIRST DIVISION
G.R. No. 154411

June 19, 2003

NATIONAL HOUSING AUTHORITY, Petitioner,


vs.
HEIRS OF ISIDRO GUIVELONDO, court of appeals, HON. ISAIAS DICDICAN, Presiding Judge,
Regional Trial Court, Branch 11, Cebu City, and PASCUAL Y. ABORDO, Sheriff, Regional Trial
Court, Branch 11, Cebu City, Respondents.
DECISION
YNARES-SANTIAGO, J.:
On February 23, 1999, petitioner National Housing Authority filed with the Regional Trial Court of
Cebu City, Branch 11, an Amended Complaint for eminent domain against Associacion Benevola de
Cebu, Engracia Urot and the Heirs of Isidro Guivelondo, docketed as Civil Case No. CEB-23386.
Petitioner alleged that defendant Associacion Benevola de Cebu was the claimant/owner of Lot 108C located in the Banilad Estate, Cebu City; that defendant Engracia Urot was the claimant/owner of
Lots Nos. 108-F, 108-I, 108-G, 6019-A and 6013-A, all of the Banilad Estate; that defendant Heirs of
Isidro Guivelondo were the claimants/owners of Cadastral Lot No. 1613-D located at Carreta,
Mabolo, Cebu City; and that the lands are within a blighted urban center which petitioner intends to
develop as a socialized housing project.1

On November 12, 1999, the Heirs of Isidro Guivelondo, respondents herein, filed a Manifestation
stating that they were waiving their objections to petitioners power to expropriate their properties.
Hence, the trial court issued an Order as follows:
WHEREFORE, the Court hereby declares that the plaintiff has a lawful right to expropriate the
properties of the defendants who are heirs of Isidro Guivelondo.
The appointment of commissioners who would ascertain and report to the Court the just
compensation for said properties will be done as soon as the parties shall have submitted to the
Court the names of persons desired by them to be appointed as such commissioners.
SO ORDERED.2
Thereafter, the trial court appointed three Commissioners to ascertain the correct and just
compensation of the properties of respondents. On April 17, 2000, the Commissioners submitted
their report wherein they recommended that the just compensation of the subject properties be fixed
at P11,200.00 per square meter.3 On August 7, 2000, the trial court rendered Partial Judgment
adopting the recommendation of the Commissioners and fixing the just compensation of the lands of
respondent Heirs of Isidro Guivelondo at P11,200.00 per square meter, to wit:
WHEREFORE, in view of the foregoing premises, judgment is hereby rendered by the Court in this
case fixing the just compensation for the lands of the defendants who are the heirs of Isidro
Guivelondo, more particularly Lots Nos. 1, 2, 3, 4, 5, 6, 7, 8, 9, 10, 11, 12, 13, 15, 16, 19, 20, 6016-F,
6016-H, 6016-E and 6016-D of Csd-10219, which were sought to be expropriated by the plaintiff at
P11,200.00 per square meter and ordering the plaintiff to pay to the said defendants the just
compensation for the said lands computed at P11,200.00 per square meter.
IT IS SO ORDERED.4
Petitioner NHA filed two motions for reconsideration dated August 30, 2000 and August 31, 2000,
assailing the inclusion of Lots 12, 13 and 19 as well as the amount of just compensation,
respectively. Respondent Heirs also filed a motion for reconsideration of the Partial Judgment. On
October 11, 2000, the trial court issued an Omnibus Order denying the motion for reconsideration of
respondent Heirs and the August 31, 2000 motion of petitioner, on the ground that the fixing of the
just compensation had adequate basis and support. On the other hand, the trial court granted
petitioners August 30, 2000 motion for reconsideration on the ground that the Commissioners
Report did not include Lots 12, 13 and 19 within its coverage. Thus:
WHEREFORE, in view of the foregoing premises, the Court hereby denies the motion of the heirs of
Isidro Guivelondo (with the exception of Carlota Mercado and Juanita Suemith) for reconsideration
of the partial judgment rendered in this case on August 7, 2000 and plaintiffs motion for
reconsideration of said judgment, dated August 31, 2000.
However, the Court hereby grants the plaintiffs motion for reconsideration of said judgment, dated
August 30, 2000. Accordingly, the judgment rendered in this case on August 7, 2000 is hereby set

aside insofar as it has fixed just compensations for Lots Nos. 12, 13 and 19 of Csd-10219 because
the fixing of said just compensations appears to lack adequate basis.
SO ORDERED.5
Petitioner filed with the Court of Appeals a petition for certiorari, which was docketed as CA-G.R. SP
No. 61746.6Meanwhile, on October 31, 2000, the trial court issued an Entry of Judgment over the
Partial Judgment dated August 7, 2000 as modified by the Omnibus Order dated October 11,
2000.7 Subsequently, respondent Heirs filed a Motion for Execution, which was granted on
November 22, 2000.
On January 31, 2001, the Court of Appeals dismissed the petition for certiorari on the ground that the
Partial Judgment and Omnibus Order became final and executory when petitioner failed to appeal
the same.8
Petitioners Motion for Reconsideration and Urgent Ex-Parte Motion for a Clarificatory Ruling were
denied in a Resolution dated March 18, 2001.9 A petition for review was filed by petitioner with this
Court, which was docketed as G.R. No. 147527. However, the same was denied in a Minute
Resolution dated May 9, 2001 for failure to show that the Court of Appeals committed a reversible
error.10
Petitioner filed a Motion for Reconsideration which was however denied with finality on August 20,
2001.11
Prior to the aforesaid denial of the Motion for Reconsideration, petitioner, on July 16, 2001, filed with
the trial court a Motion to Dismiss Civil Case No. CEB-23386, complaint for eminent domain, alleging
that the implementation of its socialized housing project was rendered impossible by the
unconscionable value of the land sought to be expropriated, which the intended beneficiaries can not
afford.12 The Motion was denied on September 17, 2001, on the ground that the Partial Judgment
had already become final and executory and there was no just and equitable reason to warrant the
dismissal of the case.13 Petitioner filed a Motion for Reconsideration, which was denied in an Order
dated November 20, 2001.14
Petitioner thus filed a petition for certiorari with the Court of Appeals, which was docketed as CAG.R. SP No. 68670, praying for the annulment of the Order of the trial court denying its Motion to
Dismiss and its Motion for Reconsideration.15
On February 5, 2002, the Court of Appeals summarily dismissed the petition. Immediately thereafter,
respondent Sheriff Pascual Y. Abordo of the Regional Trial Court of Cebu City, Branch 11, served on
petitioner a Notice of Levy pursuant to the Writ of Execution issued by the trial court to enforce the
Partial Judgment of August 7, 2000 and the Omnibus Order of October 11, 2000. 16
On February 18, 2002, the Court of Appeals set aside the dismissal of the petition and reinstated the
same.17Thereafter, a temporary restraining order was issued enjoining respondent sheriff to preserve
the status quo.18

On May 27, 2002, respondent sheriff served on the Landbank of the Philippines a Notice of Third
Garnishment against the deposits, moneys and interests of petitioner therein. 19 Subsequently,
respondent sheriff levied on funds and personal properties of petitioner.20
On July 16, 2002, the Court of Appeals rendered the assailed decision dismissing the petition for
certiorari.21
Hence, petitioner filed this petition for review, raising the following issues:
1) WHETHER OR NOT THE STATE CAN BE COMPELLED AND COERCED BY THE
COURTS TO EXERCISE OR CONTINUE WITH THE EXERCISE OF ITS INHERENT
POWER OF EMINENT DOMAIN;
2) WHETHER OR NOT JUDGMENT HAS BECOME FINAL AND EXECUTORY AND IF
ESTOPPEL OR LACHES APPLIES TO GOVERNMENT;
3) WHETHER OR NOT WRITS OF EXECUTION AND GARNISHMENT MAY BE ISSUED
AGAINST THE STATE IN AN EXPROPRIATION WHEREIN THE EXERCISE OF THE
POWER OF EMINENT DOMAIN WILL NOT SERVE PUBLIC USE OR PURPOSE
{APPLICATION OF SUPREME COURT ADMINISTRATIVE CIRCULAR NO. 10-2000}. 22
Respondent Heirs of Isidro Guivelondo filed their Comment, arguing as follows:
I
AS EARLIER UPHELD BY THE HONORABLE COURT, THE JUDGMENT OF THE TRIAL
COURT IS ALREADY FINAL AND EXECUTORY, HENCE, COULD NO LONGER BE
DISTURBED NOR SET ASIDE
II
THE FUNDS AND ASSETS OF THE PETITIONER ARE NOT EXEMPT FROM LEVY AND
GARNISHMENT
III
THE ISSUES RAISED IN THIS SECOND PETITION FOR REVIEW WERE ALREADY
RESOLVED BY THE HONORABLE COURT23
In the early case of City of Manila v. Ruymann,24 the Court was confronted with the question: May the
petitioner, in an action for expropriation, after he has been placed in possession of the property and
before the termination of the action, dismiss the petition? It resolved the issue in the affirmative and
held:
The right of the plaintiff to dismiss an action with the consent of the court is universally recognized
with certain well-defined exceptions. If the plaintiff discovers that the action which he commenced

was brought for the purpose of enforcing a right or a benefit, the advisability or necessity of which he
later discovers no longer exists, or that the result of the action would be different from what he had
intended, then he should be permitted to withdraw his action, subject to the approval of the court.
The plaintiff should not be required to continue the action, subject to some well-defined exceptions,
when it is not to his advantage to do so. Litigation should be discouraged and not encouraged.
Courts should not require parties to litigate when they no longer desire to do so. Courts, in granting
permission to dismiss an action, of course, should always take into consideration the effect which
said dismissal would have upon the rights of the defendant.25
Subsequently, in Metropolitan Water District v. De Los Angeles,26 the Court had occasion to apply the
above-quoted ruling when the petitioner, during the pendency of the expropriation case, resolved
that the land sought to be condemned was no longer necessary in the maintenance and operation of
its system of waterworks. It was held:
It is not denied that the purpose of the plaintiff was to acquire the land in question for a public use.
The fundamental basis then of all actions brought for the expropriation of lands, under the power of
eminent domain, is public use. That being true, the very moment that it appears at any stage of the
proceedings that the expropriation is not for a public use, the action must necessarily fail and should
be dismissed, for the reason that the action cannot be maintained at all except when the
expropriation is for some public use. That must be true even during the pendency of the appeal of at
any other stage of the proceedings. If, for example, during the trial in the lower court, it should be
made to appear to the satisfaction of the court that the expropriation is not for some public use, it
would be the duty and the obligation of the trial court to dismiss the action. And even during the
pendency of the appeal, if it should be made to appear to the satisfaction of the appellate court that
the expropriation is not for public use, then it would become the duty and the obligation of the
appellate court to dismiss it.27
Notably, the foregoing cases refer to the dismissal of an action for eminent domain at the instance of
the plaintiff during the pendency of the case. The rule is different where the case had been decided
and the judgment had already become final and executory.
Expropriation proceedings consists of two stages: first, condemnation of the property after it is
determined that its acquisition will be for a public purpose or public use and, second, the
determination of just compensation to be paid for the taking of private property to be made by the
court with the assistance of not more than three commissioners.28 Thus:
There are two (2) stages in every action for expropriation. The first is concerned with the
determination of the authority of the plaintiff to exercise the power of eminent domain and the
propriety of its exercise in the context of the facts involved in the suit. It ends with an order, if not of
dismissal of the action, "of condemnation declaring that the plaintiff has a lawful right to take the
property sought to be condemned, for the public use or purpose described in the complaint, upon the
payment of just compensation to be determined as of the date of the filing of the complaint." An order
of dismissal, if this be ordained, would be a final one, of course, since it finally disposes of the action
and leaves nothing more to be done by the Court on the merits. So, too, would an order of
condemnation be a final one, for thereafter, as the Rules expressly state, in the proceedings before

the Trial Court, "no objection to the exercise of the right of condemnation (or the propriety thereof)
shall be filed or heard."
The second phase of the eminent domain action is concerned with the determination by the Court of
"the just compensation for the property sought to be taken." This is done by the Court with the
assistance of not more than three (3) commissioners. The order fixing the just compensation on the
basis of the evidence before, and findings of, the commissioners would be final, too. It would finally
dispose of the second stage of the suit, and leave nothing more to be done by the Court regarding
the issue. Obviously, one or another of the parties may believe the order to be erroneous in its
appreciation of the evidence or findings of fact or otherwise. Obviously, too, such a dissatisfied party
may seek a reversal of the order by taking an appeal therefrom. 29
The outcome of the first phase of expropriation proceedings, which is either an order of expropriation
or an order of dismissal, is final since it finally disposes of the case. On the other hand, the second
phase ends with an order fixing the amount of just compensation. Both orders, being final, are
appealable.30 An order of condemnation or dismissal is final, resolving the question of whether or not
the plaintiff has properly and legally exercised its power of eminent domain. 31 Once the first order
becomes final and no appeal thereto is taken, the authority to expropriate and its public use can no
longer be questioned.32
The above rule is based on Rule 67, Section 4 of the 1997 Rules of Civil Procedure, which provides:
Order of expropriation. If the objections to and the defenses against the right of the plaintiff to
expropriate the property are overruled, or when no party appears to defend as required by this Rule,
the court may issue an order of expropriation declaring that the plaintiff has a lawful right to take the
property sought to be expropriated, for the public use or purpose described in the complaint, upon
the payment of just compensation to be determined as of the date of the taking of the property or the
filing of the complaint, whichever came first.
A final order sustaining the right to expropriate the property may be appealed by any party aggrieved
thereby.Such appeal, however, shall not prevent the court from determining the just compensation to
be paid.
After the rendition of such an order, the plaintiff shall not be permitted to dismiss or discontinue the
proceeding except on such terms as the court deems just and equitable. (underscoring ours)
In the case at bar, petitioner did not appeal the Order of the trial court dated December 10, 1999,
which declared that it has a lawful right to expropriate the properties of respondent Heirs of Isidro
Guivelondo. Hence, the Order became final and may no longer be subject to review or reversal in
any court.33 A final and executory decision or order can no longer be disturbed or reopened no matter
how erroneous it may be. Although judicial determinations are not infallible, judicial error should be
corrected through appeals, not through repeated suits on the same claim. 34
Petitioner anchors its arguments on the last paragraph of the above-quoted Rule 67, Section 4. In
essence, it contends that there are just and equitable grounds to allow dismissal or discontinuance
of the expropriation proceedings. More specifically, petitioner alleges that the intended public use

was rendered nugatory by the unreasonable just compensation fixed by the court, which is beyond
the means of the intended beneficiaries of the socialized housing project. The argument is tenuous.
Socialized housing has been recognized as public use for purposes of exercising the power of
eminent domain.
Housing is a basic human need. Shortage in housing is a matter of state concern since it directly and
significantly affects public health, safety, the environment and in sum, the general welfare. The public
character of housing measures does not change because units in housing projects cannot be
occupied by all but only by those who satisfy prescribed qualifications. A beginning has to be made,
for it is not possible to provide housing for all who need it, all at once.
xxx

xxx

xxx

In the light of the foregoing, this Court is satisfied that "socialized housing" falls with the confines of
"public use". xxx xxx xxx. Provisions on economic opportunities inextricably linked with low-cost
housing, or slum clearance, relocation and resettlement, or slum improvement emphasize the public
purpose of the project.35
The public purpose of the socialized housing project is not in any way diminished by the amount of
just compensation that the court has fixed. The need to provide decent housing to the urban poor
dwellers in the locality was not lost by the mere fact that the land cost more than petitioner had
expected. It is worthy to note that petitioner pursued its petition for certiorari with the Court of
Appeals assailing the amount of just compensation and its petition for review with this Court which
eloquently indicates that there still exists a public use for the housing project. It was only after its
appeal and petitions for review were dismissed that petitioner made a complete turn-around and
decided it did not want the property anymore.
Respondent landowners had already been prejudiced by the expropriation case. Petitioner cannot
be permitted to institute condemnation proceedings against respondents only to abandon it later
when it finds the amount of just compensation unacceptable. Indeed, our reprobation in the case of
Cosculluela v. Court of Appeals36 is apropos:
It is arbitrary and capricious for a government agency to initiate expropriation proceedings, seize a
persons property, allow the judgment of the court to become final and executory and then refuse to
pay on the ground that there are no appropriations for the property earlier taken and profitably used.
We condemn in the strongest possible terms the cavalier attitude of government officials who adopt
such a despotic and irresponsible stance.
In order to resolve the issue of the propriety of the garnishment against petitioners funds and
personal properties, there is a need to first determine its true character as a government entity.
Generally, funds and properties of the government cannot be the object of garnishment proceedings
even if the consent to be sued had been previously granted and the state liability adjudged. 37
The universal rule that where the State gives its consent to be sued by private parties either by
general or special law, it may limit claimants action "only up to the completion of proceedings

anterior to the stage of execution" and that the power of the Courts ends when the judgment is
rendered, since government funds and properties may not be seized under writs of execution or
garnishment to satisfy such judgments, is based on obvious considerations of public policy.
Disbursements of public funds must be covered by the corresponding appropriation as required by
law. The functions and public services rendered by the State cannot be allowed to be paralyzed or
disrupted by the diversion of public funds from their legitimate and specific objects, as appropriated
by law.38
However, if the funds belong to a public corporation or a government-owned or controlled
corporation which is clothed with a personality of its own, separate and distinct from that of the
government, then its funds are not exempt from garnishment. 39 This is so because when the
government enters into commercial business, it abandons its sovereign capacity and is to be treated
like any other corporation.40
In the case of petitioner NHA, the matter of whether its funds and properties are exempt from
garnishment has already been resolved squarely against its predecessor, the Peoples Homesite
and Housing Corporation (PHHC), to wit:
The plea for setting aside the notice of garnishment was premised on the funds of the Peoples
Homesite and Housing Corporation deposited with petitioner being "public in character." There was
not even a categorical assertion to that effect. It is only the possibility of its being "public in
character." The tone was thus irresolute, the approach diffident. The premise that the funds cold be
spoken of as public in character may be accepted in the sense that the Peoples Homesite and
Housing Corporation was a government-owned entity. It does not follow though that they were
exempt from garnishment.41
This was reiterated in the subsequent case of Philippine Rock Industries, Inc. v. Board of
Liquidators:42
Having a juridical personality separate and distinct from the government, the funds of such
government-owned and controlled corporations and non-corporate agency, although considered
public in character, are not exempt from garnishment. This doctrine was applied to suits filed against
the Philippine Virginia Tobacco Administration (PNB vs. Pabalan, et al., 83 SCRA 695); the National
Shipyard & Steel Corporation (NASSCO vs. CIR, 118 Phil. 782); the Manila Hotel Company (Manila
Hotel Employees Asso. vs. Manila Hotel Co., 73 Phil. 374); and thePeople's Homesite and Housing
Corporation (PNB vs. CIR, 81 SCRA 314). [emphasis ours]
Hence, it is clear that the funds of petitioner NHA are not exempt from garnishment or execution.
Petitioners prayer for injunctive relief to restrain respondent Sheriff Pascual Abordo from enforcing
the Notice of Levy and Garnishment against its funds and properties must, therefore, be denied.
WHEREFORE, in view of the foregoing, the instant petition for review is DENIED. The decision of
the Court of Appeals in CA-G.R. SP No. 68670, affirming the trial courts Order denying petitioners
Motion to Dismiss the expropriation proceedings in Civil Case No. CEB-23386, is AFFIRMED.
Petitioners prayer for injunctive relief against the levy and garnishment of its funds and personal
properties is DENIED. The Temporary Restraining Order dated January 22, 2003 is LIFTED.

SO ORDERED.

Republic of the Philippines


SUPREME COURT
Manila
THIRD DIVISION
G.R. No. 168289

March 22, 2010

THE MUNICIPALITY OF HAGONOY, BULACAN, represented by the HON. FELIX V. OPLE,


Municipal Mayor, and FELIX V. OPLE, in his personal capacity, Petitioners,
vs.
HON. SIMEON P. DUMDUM, JR., in his capacity as the Presiding Judge of the REGIONAL
TRIAL COURT, BRANCH 7, CEBU CITY; HON. CLERK OF COURT & EX-OFFICIO SHERIFF of
the REGIONAL TRIAL COURT of CEBU CITY; HON. CLERK OF COURT & EX-OFFICIO
SHERIFF of the REGIONAL TRIAL COURT of BULACAN and his DEPUTIES; and EMILY ROSE
GO KO LIM CHAO, doing business under the name and style KD SURPLUS, Respondents.
DECISION
PERALTA, J.:
This is a Joint Petition1 under Rule 45 of the Rules of Court brought by the Municipality of Hagonoy,
Bulacan and its former chief executive, Mayor Felix V. Ople in his official and personal capacity, from
the January 31, 2005 Decision2 and the May 23, 2005 Resolution3 of the Court of Appeals in CAG.R. SP No. 81888. The assailed decision affirmed the October 20, 2003 Order 4 issued by the
Regional Trial Court of Cebu City, Branch 7 in Civil Case No. CEB-28587 denying petitioners motion
to dismiss and motion to discharge/dissolve the writ of preliminary attachment previously issued in
the case. The assailed resolution denied reconsideration.
The case stems from a Complaint5 filed by herein private respondent Emily Rose Go Ko Lim Chao
against herein petitioners, the Municipality of Hagonoy, Bulacan and its chief executive, Felix V. Ople
(Ople) for collection of a sum of money and damages. It was alleged that sometime in the middle of
the year 2000, respondent, doing business as KD Surplus and as such engaged in buying and
selling surplus trucks, heavy equipment, machinery, spare parts and related supplies, was contacted
by petitioner Ople. Respondent had entered into an agreement with petitioner municipality through
Ople for the delivery of motor vehicles, which supposedly were needed to carry out certain
developmental undertakings in the municipality. Respondent claimed that because of Oples earnest
representation that funds had already been allocated for the project, she agreed to deliver from her
principal place of business in Cebu City twenty-one motor vehicles whose value
totaled P5,820,000.00. To prove this, she attached to the complaint copies of the bills of lading
showing that the items were consigned, delivered to and received by petitioner municipality on
different dates.6 However, despite having made several deliveries, Ople allegedly did not heed
respondents claim for payment. As of the filing of the complaint, the total obligation of petitioner had

already totaled P10,026,060.13 exclusive of penalties and damages. Thus, respondent prayed for
full payment of the said amount, with interest at not less than 2% per month, plus P500,000.00 as
damages for business losses, P500,000.00 as exemplary damages, attorneys fees of P100,000.00
and the costs of the suit.
On February 13, 2003, the trial court issued an Order 7 granting respondents prayer for a writ of
preliminary attachment conditioned upon the posting of a bond equivalent to the amount of the claim.
On March 20, 2003, the trial court issued the Writ of Preliminary Attachment8 directing the sheriff "to
attach the estate, real and personal properties" of petitioners.
Instead of addressing private respondents allegations, petitioners filed a Motion to Dismiss 9 on the
ground that the claim on which the action had been brought was unenforceable under the statute of
frauds, pointing out that there was no written contract or document that would evince the supposed
agreement they entered into with respondent. They averred that contracts of this nature, before
being undertaken by the municipality, would ordinarily be subject to several preconditions such as a
public bidding and prior approval of the municipal council which, in this case, did not obtain. From
this, petitioners impress upon us the notion that no contract was ever entered into by the local
government with respondent.10 To address the claim that respondent had made the deliveries under
the agreement, they advanced that the bills of lading attached to the complaint were hardly
probative, inasmuch as these documents had been accomplished and handled exclusively by
respondent herself as well as by her employees and agents.11
Petitioners also filed a Motion to Dissolve and/or Discharge the Writ of Preliminary Attachment
Already Issued,12invoking immunity of the state from suit, unenforceability of the contract, and failure
to substantiate the allegation of fraud.13
On October 20, 2003, the trial court issued an Order 14 denying the two motions. Petitioners moved
for reconsideration, but they were denied in an Order15 dated December 29, 2003.
Believing that the trial court had committed grave abuse of discretion in issuing the two orders,
petitioners elevated the matter to the Court of Appeals via a petition for certiorari under Rule 65. In it,
they faulted the trial court for not dismissing the complaint despite the fact that the alleged contract
was unenforceable under the statute of frauds, as well as for ordering the filing of an answer and in
effect allowing private respondent to prove that she did make several deliveries of the subject motor
vehicles. Additionally, it was likewise asserted that the trial court committed grave abuse of discretion
in not discharging/dissolving the writ of preliminary attachment, as prayed for in the motion, and in
effect disregarding the rule that the local government is immune from suit.
On January 31, 2005, following assessment of the parties arguments, the Court of Appeals, finding
no merit in the petition, upheld private respondents claim and affirmed the trial courts
order.16 Petitioners moved for reconsideration, but the same was likewise denied for lack of merit and
for being a mere scrap of paper for having been filed by an unauthorized counsel. 17 Hence, this
petition.
In their present recourse, which raises no matter different from those passed upon by the Court of
Appeals, petitioners ascribe error to the Court of Appeals for dismissing their challenge against the

trial courts October 20 and December 29, 2003 Orders. Again, they reason that the complaint
should have been dismissed at the first instance based on unenforceability and that the motion to
dissolve/discharge the preliminary attachment should have been granted. 18
Commenting on the petition, private respondent notes that with respect to the Court of Appeals
denial of thecertiorari petition, the same was rightly done, as the fact of delivery may be properly and
adequately addressed at the trial of the case on the merits; and that the dissolution of the writ of
preliminary attachment was not proper under the premises inasmuch as the application for the writ
sufficiently alleged fraud on the part of petitioners. In the same breath, respondent laments that the
denial of petitioners motion for reconsideration was rightly done by the Court of Appeals, because it
raised no new matter that had not yet been addressed.19
After the filing of the parties respective memoranda, the case was deemed submitted for decision.
We now rule on the petition.
To begin with, the Statute of Frauds found in paragraph (2), Article 1403 of the Civil Code, 20 requires
for enforceability certain contracts enumerated therein to be evidenced by some note or
memorandum. The term "Statute of Frauds" is descriptive of statutes that require certain classes of
contracts to be in writing; and that do not deprive the parties of the right to contract with respect to
the matters therein involved, but merely regulate the formalities of the contract necessary to render it
enforceable.21
In other words, the Statute of Frauds only lays down the method by which the enumerated contracts
may be proved. But it does not declare them invalid because they are not reduced to writing
inasmuch as, by law, contracts are obligatory in whatever form they may have been entered into,
provided all the essential requisites for their validity are present.22 The object is to prevent fraud and
perjury in the enforcement of obligations depending, for evidence thereof, on the unassisted memory
of witnesses by requiring certain enumerated contracts and transactions to be evidenced by a writing
signed by the party to be charged.23 The effect of noncompliance with this requirement is simply that
no action can be enforced under the given contracts.24 If an action is nevertheless filed in court, it
shall warrant a dismissal under Section 1(i),25 Rule 16 of the Rules of Court, unless there has been,
among others, total or partial performance of the obligation on the part of either party.26
It has been private respondents consistent stand, since the inception of the instant case that she
has entered into a contract with petitioners. As far as she is concerned, she has already performed
her part of the obligation under the agreement by undertaking the delivery of the 21 motor vehicles
contracted for by Ople in the name of petitioner municipality. This claim is well substantiated at
least for the initial purpose of setting out a valid cause of action against petitioners by copies of
the bills of lading attached to the complaint, naming petitioner municipality as consignee of the
shipment. Petitioners have not at any time expressly denied this allegation and, hence, the same is
binding on the trial court for the purpose of ruling on the motion to dismiss. In other words, since
there exists an indication by way of allegation that there has been performance of the obligation on
the part of respondent, the case is excluded from the coverage of the rule on dismissals based on
unenforceability under the statute of frauds, and either party may then enforce its claims against the
other.

No other principle in remedial law is more settled than that when a motion to dismiss is filed, the
material allegations of the complaint are deemed to be hypothetically admitted. 27 This hypothetical
admission, according to Viewmaster Construction Corporation v. Roxas 28 and Navoa v. Court of
Appeals,29 extends not only to the relevant and material facts well pleaded in the complaint, but also
to inferences that may be fairly deduced from them. Thus, where it appears that the allegations in
the complaint furnish sufficient basis on which the complaint can be maintained, the same should not
be dismissed regardless of the defenses that may be raised by the defendants. 30 Stated differently,
where the motion to dismiss is predicated on grounds that are not indubitable, the better policy is to
deny the motion without prejudice to taking such measures as may be proper to assure that the ends
of justice may be served.31
It is interesting to note at this point that in their bid to have the case dismissed, petitioners theorize
that there could not have been a contract by which the municipality agreed to be bound, because it
was not shown that there had been compliance with the required bidding or that the municipal
council had approved the contract. The argument is flawed. By invoking unenforceability under the
Statute of Frauds, petitioners are in effect acknowledging the existence of a contract between them
and private respondent only, the said contract cannot be enforced by action for being noncompliant with the legal requisite that it be reduced into writing. Suffice it to say that while this
assertion might be a viable defense against respondents claim, it is principally a matter of evidence
that may be properly ventilated at the trial of the case on the merits.
Verily, no grave abuse of discretion has been committed by the trial court in denying petitioners
motion to dismiss this case. The Court of Appeals is thus correct in affirming the same.
We now address the question of whether there is a valid reason to deny petitioners motion to
discharge the writ of preliminary attachment.
Petitioners, advocating a negative stance on this issue, posit that as a municipal corporation, the
Municipality of Hagonoy is immune from suit, and that its properties are by law exempt from
execution and garnishment. Hence, they submit that not only was there an error committed by the
trial court in denying their motion to dissolve the writ of preliminary attachment; they also advance
that it should not have been issued in the first place. Nevertheless, they believe that respondent has
not been able to substantiate her allegations of fraud necessary for the issuance of the writ. 32
Private respondent, for her part, counters that, contrary to petitioners claim, she has amply
discussed the basis for the issuance of the writ of preliminary attachment in her affidavit; and that
petitioners claim of immunity from suit is negated by Section 22 of the Local Government Code,
which vests municipal corporations with the power to sue and be sued. Further, she contends that
the arguments offered by petitioners against the writ of preliminary attachment clearly touch on
matters that when ruled upon in the hearing for the motion to discharge, would amount to a trial of
the case on the merits.33
The general rule spelled out in Section 3, Article XVI of the Constitution is that the state and its
political subdivisions may not be sued without their consent. Otherwise put, they are open to suit but
only when they consent to it. Consent is implied when the government enters into a business
contract, as it then descends to the level of the other contracting party; or it may be embodied in a

general or special law34 such as that found in Book I, Title I, Chapter 2, Section 22 of the Local
Government Code of 1991, which vests local government units with certain corporate powers one
of them is the power to sue and be sued.
Be that as it may, a difference lies between suability and liability. As held in City of Caloocan v.
Allarde,35 where the suability of the state is conceded and by which liability is ascertained judicially,
the state is at liberty to determine for itself whether to satisfy the judgment or not. Execution may not
issue upon such judgment, because statutes waiving non-suability do not authorize the seizure of
property to satisfy judgments recovered from the action. These statutes only convey an implication
that the legislature will recognize such judgment as final and make provisions for its full satisfaction.
Thus, where consent to be sued is given by general or special law, the implication thereof is limited
only to the resultant verdict on the action before execution of the judgment. 36
Traders Royal Bank v. Intermediate Appellate Court,37 citing Commissioner of Public Highways v.
San Diego,38 is instructive on this point. In that case which involved a suit on a contract entered into
by an entity supervised by the Office of the President, the Court held that while the said entity
opened itself to suit by entering into the subject contract with a private entity; still, the trial court was
in error in ordering the garnishment of its funds, which were public in nature and, hence, beyond the
reach of garnishment and attachment proceedings. Accordingly, the Court ordered that the writ of
preliminary attachment issued in that case be lifted, and that the parties be allowed to prove their
respective claims at the trial on the merits. There, the Court highlighted the reason for the rule, to wit:
The universal rule that where the State gives its consent to be sued by private parties either by
general or special law, it may limit claimants action "only up to the completion of proceedings
anterior to the stage of execution" and that the power of the Courts ends when the judgment is
rendered, since government funds and properties may not be seized under writs of execution or
garnishment to satisfy such judgments, is based on obvious considerations of public policy.
Disbursements of public funds must be covered by the corresponding appropriations as required by
law. The functions and public services rendered by the State cannot be allowed to be paralyzed or
disrupted by the diversion of public funds from their legitimate and specific objects. x x x 39
With this in mind, the Court holds that the writ of preliminary attachment must be dissolved and,
indeed, it must not have been issued in the very first place. While there is merit in private
respondents position that she, by affidavit, was able to substantiate the allegation of fraud in the
same way that the fraud attributable to petitioners was sufficiently alleged in the complaint and,
hence, the issuance of the writ would have been justified. Still, the writ of attachment in this case
would only prove to be useless and unnecessary under the premises, since the property of the
municipality may not, in the event that respondents claim is validated, be subjected to writs of
execution and garnishment unless, of course, there has been a corresponding appropriation
provided by law.40
1avvphi1

Anent the other issues raised by petitioners relative to the denial of their motion to dissolve the writ
of attachment, i.e., unenforceability of the contract and the veracity of private respondents allegation
of fraud, suffice it to say that these pertain to the merits of the main action. Hence, these issues are
not to be taken up in resolving the motion to discharge, lest we run the risk of deciding or prejudging
the main case and force a trial on the merits at this stage of the proceedings. 41

There is one final concern raised by petitioners relative to the denial of their motion for
reconsideration. They complain that it was an error for the Court of Appeals to have denied the
motion on the ground that the same was filed by an unauthorized counsel and, hence, must be
treated as a mere scrap of paper.42
It can be derived from the records that petitioner Ople, in his personal capacity, filed his Rule 65
petition with the Court of Appeals through the representation of the law firm Chan Robles &
Associates. Later on, municipal legal officer Joselito Reyes, counsel for petitioner Ople, in his official
capacity and for petitioner municipality, filed with the Court of Appeals a Manifestation with Entry of
Appearance43 to the effect that he, as counsel, was "adopting all the pleadings filed for and in behalf
of [Oples personal representation] relative to this case."44
It appears, however, that after the issuance of the Court of Appeals decision, only Oples personal
representation signed the motion for reconsideration. There is no showing that the municipal legal
officer made the same manifestation, as he previously did upon the filing of the petition. 45 From this,
the Court of Appeals concluded that it was as if petitioner municipality and petitioner Ople, in his
official capacity, had never moved for reconsideration of the assailed decision, and adverts to the
ruling in Ramos v. Court of Appeals46 and Municipality of Pililla, Rizal v. Court of Appeals47 that only
under well-defined exceptions may a private counsel be engaged in lawsuits involving a municipality,
none of which exceptions obtains in this case.48
The Court of Appeals is mistaken. As can be seen from the manner in which the Manifestation with
Entry of Appearance is worded, it is clear that petitioner municipalitys legal officer was intent on
adopting, for both the municipality and Mayor Ople, not only the certiorari petition filed with the Court
of Appeals, but also all other pleadings that may be filed thereafter by Oples personal
representation, including the motion for reconsideration subject of this case. In any event, however,
the said motion for reconsideration would warrant a denial, because there seems to be no matter
raised therein that has not yet been previously addressed in the assailed decision of the Court of
Appeals as well as in the proceedings below, and that would have otherwise warranted a different
treatment of the issues involved.
WHEREFORE, the Petition is GRANTED IN PART. The January 31, 2005 Decision of the Court of
Appeals in CA-G.R. SP No. 81888 is AFFIRMED insofar as it affirmed the October 20, 2003
Decision of the Regional Trial Court of Cebu City, Branch 7 denying petitioners motion to dismiss in
Civil Case No. CEB-28587. The assailed decision is REVERSED insofar as it affirmed the said trial
courts denial of petitioners motion to discharge the writ of preliminary attachment issued in that
case. Accordingly, the August 4, 2003 Writ of Preliminary Attachment issued in Civil Case No. CEB28587 is ordered lifted.
SO ORDERED.

Republic of the Philippines


SUPREME COURT
Manila
SECOND DIVISION
G.R. No. 131544

March 16, 2001

EPG CONSTRUCTION CO., CIPER ELECTRICAL & ENGINEERING, SEPTA CONSTRUCTION


CO., PHIL. PLUMBING CO., HOME CONSTRUCTION INC., WORLD BUILDERS CO., GLASS
WORLD INC., PERFORMANCE BUILDERS DEV'T. CO., DE LEON-ARANETA CONST. CO., J.D.
MACAPAGAL CONST. CO., All represented by their Atty. IN FACT, MARCELO D,
FORONDA, petitioners,
vs.
HON. GREGORIO R. VIGILAR, In His Capacity as Secretary of Public Works and
Highways, respondent.
BUENA, J.:
Sought to be reversed in the instant Petition for Certiorari is the Decision, dated 07 November 1997,
of the Regional Trial Court of Quezon City, Branch 226, in Civil Case No. Q-96-29243, 1 dismissing
the Petition for Mandamus filed by herein petitioners against herein respondent Hon. Gregorio
Vigilar, in his capacity as Secretary of the Department of Public Works and Highways (DPWH).
The tapestry of facts unfurls.
In 1983, the Ministry of Human Settlement, through the BLISS Development Corporation, initiated a
housing project on a government property along the east bank of the Manggahan Floodway in Pasig
City. For this purpose, the Ministry of Human Settlement entered into a Memorandum of Agreement
(MOA) with the Ministry of Public Works and Highways, 2 where the latter undertook to develop the
housing site and construct thereon 145 housing units.
By virtue of the MOA, the Ministry of Public Works and Highways forged individual contracts with
herein petitioners EPG Construction Co., Ciper Electrical and Engineering, Septa Construction Co.,
Phil. Plumbing Co., Home Construction Inc., World Builders Inc., Glass World Inc., Performance
Builders Development Co. and De Leon Araneta Construction Co., for the construction of the
housing units. Under the contracts, the scope of construction and funding therefor covered only
around "2/3 of each housing unit."3 After complying with the terms of said contracts, and by reason of
the verbal request and assurance of then DPWH Undersecretary Aber Canlas that additional funds
would be available and forthcoming, petitioners agreed to undertake and perform "additional
constructions"4 for the completion of the housing units, despite the absence of appropriations and
written contracts to cover subsequent expenses for the "additional constructions."
Petitioners then received payment for the construction work duly covered by the individual written
contracts, thereby leaving an unpaid balance of P5,918,315.63,5 which amount represents the
expenses for the "additional constructions" for the completion of the existing housing units. On 14

November 1988, petitioners sent a demand letter to the DPWH Secretary and submitted that their
claim for payment was favorably recommended by DPWH Assistant Secretary for Legal Services
Dominador Madamba, who recognized the existence of implied contractscovering the additional
constructions. Notwithstanding, DPWH Assistant Secretary Madamba opined that payment of
petitioners' money claims should be based on quantum meruit and should be forwarded to the
Commission on Audit (COA) for its due consideration and approval. The money claims were then
referred to COA which returned the same to the DPWH Auditor for auditorial action. On the basis of
the Inspection Report of the Auditor's Technical Staff, the DPWH Auditor interposed no objection to
the payment of the money claims subject to whatever action the COA may adopt.
In a Second Indorsement dated 27 July 1992, the COA returned the documents to the DPWH,
stating that funds should first be made available before COA could pass upon and act on the money
claims. In a Memorandum dated 30 July 1992, then DPWH Secretary Jose De Jesus requested the
Secretary of Budget and Management to release public funds for the payment of petitioners' money
claims, stating that the "amount is urgently needed in order to settle once and for all this (sic)
outstanding obligations of the government." In a Letter of the Undersecretary of Budget and
Management dated 20 December 1994, the amount of P5,819,316.00 was then released for the
payment of petitioners' money claims, under Advise of Allotment No. A4-1303-04-41-303.
In an Indorsement dated 27 December 1995, the COA referred anew the money claims to the
DPWH pursuant to COA Circular 95-006, thus:
"Respectfully returned thru the Auditor to the Honorable Secretary, Department of Public
Works and Highways, Port Area, Manila, the above-captioned subject (Re: Claim of Ten (10)
contractors for payment of Work accomplishments on the construction of the COGEO II
Housing Project, Pasig, Metro Manila) and reiterating the policy of this office as embodied in
COA Circular No. 95-006 dated May 18, 1995 totally lifting its pre-audit activities on all
financial transactions of the agencies of the government involving
implementation/prosecution of projects and/or payment of claims without exception so as to
vest on agency heads the prerogative to exercise fiscal responsibility thereon.
"The audit of the transaction shall be done after payment."
In a letter dated 26 August 1996, respondent DPWH Secretary Gregorio Vigilar denied the subject
money claims prompting herein petitioners to file before the Regional Trial Court of Quezon City,
Branch 226, a Petition for Mandamus praying that herein respondent be ordered:
"1) To pay petitioners the total of P5,819,316.00;
"2) To pay petitioners moral and exemplary damages in the amount to be fixed by the Court
and sum of P500,000.00 as attorney's fees.
On 18 February 1997, the lower court conducted a pre-trial conference where the parties appeared
and filed their respective pre-trial briefs. Further, respondent submitted a Memorandum to which
petitioners filed a Rejoinder.

On 07 November 1997, the lower court denied the Petition for Mandamus, in a Decision which
disposed as follows:
"WHEREFORE, in view of all the foregoing, the instant Petition for Mandamus is dismissed.
The order of September 24, 1997, submitting the Manifestation and Motion for Resolution, is
hereby withdrawn.
"SO ORDERED."
Hence, this petition where the core issue for resolution focuses on the right of petitioners-contractors
to compensation for a public works housing project.
In the case before us, respondent, citing among others Sections 46 6 and 47,7 Chapter 7, Sub-Title B,
Title I, Book V of the Administrative Code of 1987 (E.O 292), posits that the "existence of
appropriations and availability of funds as certified to and verified by the proper accounting officials
are conditions sine qua non for the execution of government contracts."8 Respondent harps on the
fact that "the additional work was pursued through the verbal request of then DPWH Undersecretary
Aber P. Canlas, despite the absence of the corresponding supplemental contracts and appropriate
funding."9 According to respondent, "sans showing of certificate of availability of funds, the implied
contracts are considered fatally defective and considered inexistent and void ab initio." Respondent
concludes that "inasmuch as the additional work done was pursued in violation of the mandatory
provisions of the laws concerning contracts involving expenditure of public funds and in excess of
the public official's contracting authority, the same is not binding on the government and impose no
liability therefor."10
Although this Court agrees with respondent's postulation that the "implied contracts", which covered
the additional constructions, are void, in view of violation of applicable laws, auditing rules and lack
of legal requirements,11 we nonetheless find the instant petition laden with merit and uphold, in the
interest of substantial justice, petitioners-contractors' right to be compensated for the "additional
constructions" on the public works housing project, applying the principle of quantum meruit.
Interestingly, this case is not of first impression. In Eslao vs. Commission on Audit,12 this Court
likewise allowed recovery by the contractor on the basis of quantum meruit, following our
pronouncement in Royal Trust Construction vs. Commission on Audit,13 thus:
"In Royal Trust Construction vs. COA, a case involving the widening and deepening of the
Betis River in Pampanga at the urgent request of the local officials and with the knowledge
and consent of the Ministry of Public Works, even without a written contract and the covering
appropriation, the project was undertaken to prevent the overflowing of the neighboring
areas and to irrigate the adjacent farmlands. The contractor sought compensation for the
completed portion in the sum of over P1 million. While the payment was favorably
recommended by the Ministry of Public Works, it was denied by the respondent COA on the
ground of violation of mandatory legal provisions as the existence of corresponding
appropriations covering the contract cost. Under COA Res. No. 36-58 dated November 15,
1986, its existing policy is to allow recovery from covering contracts on the basis of quantum

meruit if there is delay in the accomplishment of the required certificate of availability of


funds to support a contract." (Emphasis ours)
In the Royal Construction case, this Court, applying the principle of quantum meruit in allowing
recovery by the contractor, elucidated:
"The work done by it (the contractor) was impliedly authorized and later expressly
acknowledged by the Ministry of Public Works, which has twice recommended favorable
action on the petitioner's request for payment. Despite the admitted absence of a specific
covering appropriation as required under COA Resolution No. 36-58, the petitioner may
nevertheless be compensated for the services rendered by it, concededly for the public
benefit, from the general fund allotted by law to the Betis River project. Substantial
compliance with the said resolution, in view of the circumstances of this case, should
suffice.The Court also feels that the remedy suggested by the respondent, to wit, the filing of
a complaint in court for recovery of the compensation claimed, would entail additional
expense, inconvenience and delay which in fairness should be imposed on the petitioner.
"Accordingly, in the interest of substantial justice and equity, the respondent Commission on
Audit is DIRECTED to determine on a quantum meruit basis the total compensation due to
the petitioner for the services rendered by it in the channel improvement of the Betis River in
Pampanga and to allow the payment thereof immediately upon completion of the said
determination." (Emphasis ours)
Similarly, this Court applied the doctrine of quantum meruit in Melchor vs. Commission on Audit14 and
explained that where payment is based on quantum meruit, the amount of recovery would only be
the reasonable value of the thing or services rendered regardless of any agreement as to value. 15
Notably, the peculiar circumstances present in the instant case buttress petitioners' claim for
compensation for the additional constructions, despite the illegality and void nature of the "implied
contracts" forged between the DPWH and petitioners-contractors. On this matter, it bears stressing
that the illegality of the subject contracts proceeds from an express declaration or prohibition by
law,16 and not from any intrinsic illegality. Stated differently, the subject contracts are not illegal per
se.
Of equal significance are circumstances attendant and peculiar in this case which necessitate
allowance of petitioners' money claims on the basis of quantum meruit for work accomplished
on the government housing project.
To begin with, petitioners-contractors assented and agreed to undertake additional constructions for
the completion of the housing units, believing in good faith and in the interest of the government and,
in effect, the public in general, that appropriations to cover the additional constructions and
completion of the public works housing project would be available and forthcoming. On this particular
score, the records reveal that the verbal request and assurance of then DPWH Undersecretary
Canlas led petitioners-contractors to undertake thecompletion of the government housing project,
despite the absence of covering appropriations, written contracts, and certification of availability of
funds, as mandated by law and pertinent auditing rules and issuances. To put it differently, the

"implied contracts," declared void in this case, covered only the completion and final phase of
construction of the housing units, which structures, concededly, were already existing, albeit not yet
finished in their entirety at the time the "implied contracts" were entered into between the
government and the contractors.
Further, petitioners-contractors sent to the DPWH Secretary a demand letter pressing for their
money claims, on the strength of a favorable recommendation from the DPWH Assistant Secretary
for Legal Affairs to the effect that implied contracts existed and that the money claims had ample
basis applying the principle of quantum meruit. Moreover, as can be gleaned from the records, even
the DPWH Auditor interposed no objection to the payment of the money claims, subject to whatever
action the COA may adopt.
Beyond this, the sum of P5,819,316.00 representing the amount of petitioners' money claims, had
already been released by the Department of Budget and Management (DBM), under Advise of
Allotment No. A4-1303-04-41-303. Equally important is the glaring fact that the construction of the
housing units had already been completed by petitioners-contractors and the subject housing units
had been, since their completion, under the control and disposition of the government pursuant to its
public works housing project.
To our mind, it would be the apex of injustice and highly inequitable for us to defeat petitionerscontractors' right to be duly compensated for actual work performed and services rendered, where
both the government and the public have, for years, received and accepted benefits from said
housing project and reaped the fruits of petitioners-contractors' honest toil and labor.
Incidentally, respondent likewise argues that the State may not be sued in the instant case, invoking
the constitutional doctrine of Non-suability of the State,17 otherwise known as the Royal Prerogative
of Dishonesty.
Respondent's argument is misplaced inasmuch as the Principle of State Immunity finds no
application in the case before us.
Under these circumstances, respondent may not validly invoke the Royal Prerogative of
Dishonesty and conveniently hide under the State's cloak of invincibility against suit, considering that
this principle yields to certain settled exceptions. True enough, the rule, in any case, is not absolute
for it does not say that the state may not be sued under any circumstance. 18
Thus, in Amigable vs. Cuenca,19 this Court, in effect, shred the protective shroud which shields the
State from suit, reiterating our decree in the landmark case of Ministerio vs. CFI of Cebu20 that "the
doctrine of governmental immunity from suit cannot serve as an instrument for perpetrating an
injustice on a citizen." It is just as important, if not more so, that there be fidelity to legal norms on the
part of officialdom if the rule of law were to be maintained. 21
Although the Amigable and Ministerio cases generously tackled the issue of the State's immunity
from suit vis a vis the payment of just compensation for expropriated property, this Court nonetheless
finds the doctrine enunciated in the aforementioned cases applicable to the instant controversy,

considering that the ends of justice would be subverted if we were to uphold, in this particular
instance, the State's immunity from suit.
To be sure, this Court as the staunch guardian of the citizens' rights and welfare cannot
sanction an injustice so patent on its face, and allow itself to be an instrument in the perpetration
thereof. Justice and equity sternly demand that the State's cloak of invincibility against suit be shred
in this particular instance, and that petitioners-contractors be duly compensated on the basis
of quantum meruit for construction done on the public works housing project.
IN VIEW WHEREOF, the instant petition is GRANTED. The assailed decision of the Regional Trial
Court dated 07 November 1997 is REVERSED AND SET ASIDE.
ACCORDINGLY, the Commission on Audit is hereby directed to determine and ascertain with
dispatch, on aquantum meruit basis, the total compensation due to petitioners-contractors for the
additional constructions on the housing project and to allow payment thereof upon the completion of
said determination. No costs.
SO ORDERED.

Republic of the Philippines


SUPREME COURT
Manila
EN BANC
G.R. No. 176579

June 28, 2011

WILSON P. GAMBOA, Petitioner,


vs.
FINANCE SECRETARY MARGARITO B. TEVES, FINANCE UNDERSECRETARY JOHN P.
SEVILLA, AND COMMISSIONER RICARDO ABCEDE OF THE PRESIDENTIAL COMMISSION
ON GOOD GOVERNMENT (PCGG) IN THEIR CAPACITIES AS CHAIR AND MEMBERS,
RESPECTIVELY, OF THE PRIVATIZATION COUNCIL, CHAIRMAN ANTHONI SALIM OF FIRST
PACIFIC CO., LTD. IN HIS CAPACITY AS DIRECTOR OF METRO PACIFIC ASSET HOLDINGS
INC., CHAIRMAN MANUEL V. PANGILINAN OF PHILIPPINE LONG DISTANCE TELEPHONE
COMPANY (PLDT) IN HIS CAPACITY AS MANAGING DIRECTOR OF FIRST PACIFIC CO., LTD.,
PRESIDENT NAPOLEON L. NAZARENO OF PHILIPPINE LONG DISTANCE TELEPHONE
COMPANY, CHAIR FE BARIN OF THE SECURITIES EXCHANGE COMMISSION, and
PRESIDENT FRANCIS LIM OF THE PHILIPPINE STOCK EXCHANGE, Respondents.
PABLITO V. SANIDAD and ARNO V. SANIDAD, Petitioners-in-Intervention.
DECISION
CARPIO, J.:

The Case
This is an original petition for prohibition, injunction, declaratory relief and declaration of nullity of the
sale of shares of stock of Philippine Telecommunications Investment Corporation (PTIC) by the
government of the Republic of the Philippines to Metro Pacific Assets Holdings, Inc. (MPAH), an
affiliate of First Pacific Company Limited (First Pacific).
The Antecedents
The facts, according to petitioner Wilson P. Gamboa, a stockholder of Philippine Long Distance
Telephone Company (PLDT), are as follows:1
On 28 November 1928, the Philippine Legislature enacted Act No. 3436 which granted PLDT a
franchise and the right to engage in telecommunications business. In 1969, General Telephone and
Electronics Corporation (GTE), an American company and a major PLDT stockholder, sold 26
percent of the outstanding common shares of PLDT to PTIC. In 1977, Prime Holdings, Inc. (PHI)
was incorporated by several persons, including Roland Gapud and Jose Campos, Jr. Subsequently,
PHI became the owner of 111,415 shares of stock of PTIC by virtue of three Deeds of Assignment
executed by PTIC stockholders Ramon Cojuangco and Luis Tirso Rivilla. In 1986, the 111,415
shares of stock of PTIC held by PHI were sequestered by the Presidential Commission on Good
Government (PCGG). The 111,415 PTIC shares, which represent about 46.125 percent of the
outstanding capital stock of PTIC, were later declared by this Court to be owned by the Republic of
the Philippines.2
In 1999, First Pacific, a Bermuda-registered, Hong Kong-based investment firm, acquired the
remaining 54 percent of the outstanding capital stock of PTIC. On 20 November 2006, the InterAgency Privatization Council (IPC) of the Philippine Government announced that it would sell the
111,415 PTIC shares, or 46.125 percent of the outstanding capital stock of PTIC, through a public
bidding to be conducted on 4 December 2006. Subsequently, the public bidding was reset to 8
December 2006, and only two bidders, Parallax Venture Fund XXVII (Parallax) and Pan-Asia
Presidio Capital, submitted their bids. Parallax won with a bid of P25.6 billion or US$510 million.
Thereafter, First Pacific announced that it would exercise its right of first refusal as a PTIC
stockholder and buy the 111,415 PTIC shares by matching the bid price of Parallax. However, First
Pacific failed to do so by the 1 February 2007 deadline set by IPC and instead, yielded its right to
PTIC itself which was then given by IPC until 2 March 2007 to buy the PTIC shares. On 14 February
2007, First Pacific, through its subsidiary, MPAH, entered into a Conditional Sale and Purchase
Agreement of the 111,415 PTIC shares, or 46.125 percent of the outstanding capital stock of PTIC,
with the Philippine Government for the price of P25,217,556,000 or US$510,580,189. The sale was
completed on 28 February 2007.
Since PTIC is a stockholder of PLDT, the sale by the Philippine Government of 46.125 percent of
PTIC shares is actually an indirect sale of 12 million shares or about 6.3 percent of the outstanding
common shares of PLDT.With the sale, First Pacifics common shareholdings in PLDT
increased from 30.7 percent to 37 percent, thereby increasing the common shareholdings of
foreigners in PLDT to about 81.47 percent. This violates Section 11, Article XII of the 1987

Philippine Constitution which limits foreign ownership of the capital of a public utility to not more than
40 percent.3
On the other hand, public respondents Finance Secretary Margarito B. Teves, Undersecretary John
P. Sevilla, and PCGG Commissioner Ricardo Abcede allege the following relevant facts:
On 9 November 1967, PTIC was incorporated and had since engaged in the business of investment
holdings. PTIC held 26,034,263 PLDT common shares, or 13.847 percent of the total PLDT
outstanding common shares. PHI, on the other hand, was incorporated in 1977, and became the
owner of 111,415 PTIC shares or 46.125 percent of the outstanding capital stock of PTIC by virtue of
three Deeds of Assignment executed by Ramon Cojuangco and Luis Tirso Rivilla. In 1986, the
111,415 PTIC shares held by PHI were sequestered by the PCGG, and subsequently declared by
this Court as part of the ill-gotten wealth of former President Ferdinand Marcos. The sequestered
PTIC shares were reconveyed to the Republic of the Philippines in accordance with this Courts
decision4 which became final and executory on 8 August 2006.
The Philippine Government decided to sell the 111,415 PTIC shares, which represent 6.4 percent of
the outstanding common shares of stock of PLDT, and designated the Inter-Agency Privatization
Council (IPC), composed of the Department of Finance and the PCGG, as the disposing entity. An
invitation to bid was published in seven different newspapers from 13 to 24 November 2006. On 20
November 2006, a pre-bid conference was held, and the original deadline for bidding scheduled on 4
December 2006 was reset to 8 December 2006. The extension was published in nine different
newspapers.
During the 8 December 2006 bidding, Parallax Capital Management LP emerged as the highest
bidder with a bid of P25,217,556,000. The government notified First Pacific, the majority owner of
PTIC shares, of the bidding results and gave First Pacific until 1 February 2007 to exercise its right
of first refusal in accordance with PTICs Articles of Incorporation. First Pacific announced its
intention to match Parallaxs bid.
On 31 January 2007, the House of Representatives (HR) Committee on Good Government
conducted a public hearing on the particulars of the then impending sale of the 111,415 PTIC shares.
Respondents Teves and Sevilla were among those who attended the public hearing. The HR
Committee Report No. 2270 concluded that: (a) the auction of the governments 111,415 PTIC
shares bore due diligence, transparency and conformity with existing legal procedures; and (b) First
Pacifics intended acquisition of the governments 111,415 PTIC shares resulting in First
Pacifics 100% ownership of PTIC will not violate the 40 percent constitutional limit on foreign
ownership of a public utility since PTIC holds only 13.847 percent of the total outstanding
common shares of PLDT.5 On 28 February 2007, First Pacific completed the acquisition of the
111,415 shares of stock of PTIC.
Respondent Manuel V. Pangilinan admits the following facts: (a) the IPC conducted a public bidding
for the sale of 111,415 PTIC shares or 46 percent of the outstanding capital stock of PTIC (the
remaining 54 percent of PTIC shares was already owned by First Pacific and its affiliates); (b)
Parallax offered the highest bid amounting toP25,217,556,000; (c) pursuant to the right of first
refusal in favor of PTIC and its shareholders granted in PTICs Articles of Incorporation, MPAH, a

First Pacific affiliate, exercised its right of first refusal by matching the highest bid offered for PTIC
shares on 13 February 2007; and (d) on 28 February 2007, the sale was consummated when MPAH
paid IPC P25,217,556,000 and the government delivered the certificates for the 111,415 PTIC
shares. Respondent Pangilinan denies the other allegations of facts of petitioner.
On 28 February 2007, petitioner filed the instant petition for prohibition, injunction, declaratory relief,
and declaration of nullity of sale of the 111,415 PTIC shares. Petitioner claims, among others, that
the sale of the 111,415 PTIC shares would result in an increase in First Pacifics common
shareholdings in PLDT from 30.7 percent to 37 percent, and this, combined with Japanese NTT
DoCoMos common shareholdings in PLDT, would result to a total foreign common shareholdings in
PLDT of 51.56 percent which is over the 40 percent constitutional limit.6 Petitioner asserts:
If and when the sale is completed, First Pacifics equity in PLDT will go up from 30.7 percent to 37.0
percent of its common or voting- stockholdings, x x x. Hence, the consummation of the sale will put
the two largest foreign investors in PLDT First Pacific and Japans NTT DoCoMo, which is the
worlds largest wireless telecommunications firm, owning 51.56 percent of PLDT common equity. x x
x With the completion of the sale, data culled from the official website of the New York Stock
Exchange (www.nyse.com) showed that those foreign entities, which own at least five percent of
common equity, will collectively own 81.47 percent of PLDTs common equity. x x x
x x x as the annual disclosure reports, also referred to as Form 20-K reports x x x which PLDT
submitted to the New York Stock Exchange for the period 2003-2005, revealed that First Pacific and
several other foreign entities breached the constitutional limit of 40 percent ownership as early as
2003. x x x"7
Petitioner raises the following issues: (1) whether the consummation of the then impending sale of
111,415 PTIC shares to First Pacific violates the constitutional limit on foreign ownership of a public
utility; (2) whether public respondents committed grave abuse of discretion in allowing the sale of the
111,415 PTIC shares to First Pacific; and (3) whether the sale of common shares to foreigners in
excess of 40 percent of the entire subscribed common capital stock violates the constitutional limit
on foreign ownership of a public utility.8
On 13 August 2007, Pablito V. Sanidad and Arno V. Sanidad filed a Motion for Leave to Intervene
and Admit Attached Petition-in-Intervention. In the Resolution of 28 August 2007, the Court granted
the motion and noted the Petition-in-Intervention.
Petitioners-in-intervention "join petitioner Wilson Gamboa x x x in seeking, among others, to enjoin
and/or nullify the sale by respondents of the 111,415 PTIC shares to First Pacific or assignee."
Petitioners-in-intervention claim that, as PLDT subscribers, they have a "stake in the outcome of the
controversy x x x where the Philippine Government is completing the sale of government owned
assets in [PLDT], unquestionably a public utility, in violation of the nationality restrictions of the
Philippine Constitution."
The Issue

This Court is not a trier of facts. Factual questions such as those raised by petitioner,9 which
indisputably demand a thorough examination of the evidence of the parties, are generally beyond
this Courts jurisdiction. Adhering to this well-settled principle, the Court shall confine the resolution
of the instant controversy solely on the threshold and purely legal issue of whether the term
"capital" in Section 11, Article XII of the Constitution refers to the total common shares only or to the
total outstanding capital stock (combined total of common and non-voting preferred shares) of PLDT,
a public utility.
The Ruling of the Court
The petition is partly meritorious.
Petition for declaratory relief treated as petition for mandamus
At the outset, petitioner is faced with a procedural barrier. Among the remedies petitioner seeks, only
the petition for prohibition is within the original jurisdiction of this court, which however is not
exclusive but is concurrent with the Regional Trial Court and the Court of Appeals. The actions for
declaratory relief,10 injunction, and annulment of sale are not embraced within the original jurisdiction
of the Supreme Court. On this ground alone, the petition could have been dismissed outright.
While direct resort to this Court may be justified in a petition for prohibition, 11 the Court shall
nevertheless refrain from discussing the grounds in support of the petition for prohibition since on 28
February 2007, the questioned sale was consummated when MPAH paid IPC P25,217,556,000 and
the government delivered the certificates for the 111,415 PTIC shares.
However, since the threshold and purely legal issue on the definition of the term "capital" in Section
11, Article XII of the Constitution has far-reaching implications to the national economy, the Court
treats the petition for declaratory relief as one for mandamus.12
In Salvacion v. Central Bank of the Philippines,13 the Court treated the petition for declaratory relief
as one for mandamus considering the grave injustice that would result in the interpretation of a
banking law. In that case, which involved the crime of rape committed by a foreign tourist against a
Filipino minor and the execution of the final judgment in the civil case for damages on the tourists
dollar deposit with a local bank, the Court declared Section 113 of Central Bank Circular No. 960,
exempting foreign currency deposits from attachment, garnishment or any other order or process of
any court, inapplicable due to the peculiar circumstances of the case. The Court held that "injustice
would result especially to a citizen aggrieved by a foreign guest like accused x x x" that would
"negate Article 10 of the Civil Code which provides that in case of doubt in the interpretation or
application of laws, it is presumed that the lawmaking body intended right and justice to prevail."
The Court therefore required respondents Central Bank of the Philippines, the local bank, and the
accused to comply with the writ of execution issued in the civil case for damages and to release the
dollar deposit of the accused to satisfy the judgment.
In Alliance of Government Workers v. Minister of Labor,14 the Court similarly brushed aside the
procedural infirmity of the petition for declaratory relief and treated the same as one for mandamus.
In Alliance, the issue was whether the government unlawfully excluded petitioners, who were

government employees, from the enjoyment of rights to which they were entitled under the law.
Specifically, the question was: "Are the branches, agencies, subdivisions, and instrumentalities of
the Government, including government owned or controlled corporations included among the four
employers under Presidential Decree No. 851 which are required to pay their employees x x x a
thirteenth (13th) month pay x x x ?" The Constitutional principle involved therein affected all
government employees, clearly justifying a relaxation of the technical rules of procedure, and
certainly requiring the interpretation of the assailed presidential decree.
In short, it is well-settled that this Court may treat a petition for declaratory relief as one for
mandamus if the issue involved has far-reaching implications. As this Court held in Salvacion:
The Court has no original and exclusive jurisdiction over a petition for declaratory relief. However,
exceptions to this rule have been recognized. Thus, where the petition has far-reaching
implications and raises questions that should be resolved, it may be treated as one for
mandamus.15 (Emphasis supplied)
In the present case, petitioner seeks primarily the interpretation of the term "capital" in Section 11,
Article XII of the Constitution. He prays that this Court declare that the term "capital" refers to
common shares only, and that such shares constitute "the sole basis in determining foreign equity in
a public utility." Petitioner further asks this Court to declare any ruling inconsistent with such
interpretation unconstitutional.
The interpretation of the term "capital" in Section 11, Article XII of the Constitution has far-reaching
implications to the national economy. In fact, a resolution of this issue will determine whether
Filipinos are masters, or second class citizens, in their own country. What is at stake here is whether
Filipinos or foreigners will have effective control of the national economy. Indeed, if ever there is a
legal issue that has far-reaching implications to the entire nation, and to future generations of
Filipinos, it is the threshhold legal issue presented in this case.
The Court first encountered the issue on the definition of the term "capital" in Section 11, Article XII
of the Constitution in the case of Fernandez v. Cojuangco, docketed as G.R. No. 157360.16 That
case involved the same public utility (PLDT) and substantially the same private respondents. Despite
the importance and novelty of the constitutional issue raised therein and despite the fact that the
petition involved a purely legal question, the Court declined to resolve the case on the merits, and
instead denied the same for disregarding the hierarchy of courts.17 There, petitioner Fernandez
assailed on a pure question of law the Regional Trial Courts Decision of 21 February 2003 via a
petition for review under Rule 45. The Courts Resolution, denying the petition, became final on 21
December 2004.
The instant petition therefore presents the Court with another opportunity to finally settle this purely
legal issuewhich is of transcendental importance to the national economy and a fundamental
requirement to a faithful adherence to our Constitution. The Court must forthwith seize such
opportunity, not only for the benefit of the litigants, but more significantly for the benefit of the entire
Filipino people, to ensure, in the words of the Constitution, "a self-reliant and independent national
economy effectively controlled by Filipinos."18 Besides, in the light of vague and confusing
positions taken by government agencies on this purely legal issue, present and future foreign

investors in this country deserve, as a matter of basic fairness, a categorical ruling from this Court on
the extent of their participation in the capital of public utilities and other nationalized businesses.
Despite its far-reaching implications to the national economy, this purely legal issue has remained
unresolved for over 75 years since the 1935 Constitution. There is no reason for this Court to evade
this ever recurring fundamental issue and delay again defining the term "capital," which appears not
only in Section 11, Article XII of the Constitution, but also in Section 2, Article XII on co-production
and joint venture agreements for the development of our natural resources, 19 in Section 7, Article XII
on ownership of private lands,20 in Section 10, Article XII on the reservation of certain investments to
Filipino citizens,21 in Section 4(2), Article XIV on the ownership of educational institutions, 22 and in
Section 11(2), Article XVI on the ownership of advertising companies.23
Petitioner has locus standi
There is no dispute that petitioner is a stockholder of PLDT. As such, he has the right to question the
subject sale, which he claims to violate the nationality requirement prescribed in Section 11, Article
XII of the Constitution. If the sale indeed violates the Constitution, then there is a possibility that
PLDTs franchise could be revoked, a dire consequence directly affecting petitioners interest as a
stockholder.
More importantly, there is no question that the instant petition raises matters of transcendental
importance to the public. The fundamental and threshold legal issue in this case, involving the
national economy and the economic welfare of the Filipino people, far outweighs any perceived
impediment in the legal personality of the petitioner to bring this action.
In Chavez v. PCGG,24 the Court upheld the right of a citizen to bring a suit on matters of
transcendental importance to the public, thus:
In Taada v. Tuvera, the Court asserted that when the issue concerns a public right and the
object of mandamus is to obtain the enforcement of a public duty, the people are regarded as
the real parties in interest; and because it is sufficient that petitioner is a citizen and as such
is interested in the execution of the laws, he need not show that he has any legal or special
interest in the result of the action. In the aforesaid case, the petitioners sought to enforce their
right to be informed on matters of public concern, a right then recognized in Section 6, Article IV of
the 1973 Constitution, in connection with the rule that laws in order to be valid and enforceable must
be published in the Official Gazette or otherwise effectively promulgated. In ruling for the petitioners
legal standing, the Court declared that the right they sought to be enforced is a public right
recognized by no less than the fundamental law of the land.
Legaspi v. Civil Service Commission, while reiterating Taada, further declared that when a
mandamus proceeding involves the assertion of a public right, the requirement of personal
interest is satisfied by the mere fact that petitioner is a citizen and, therefore, part of the
general public which possesses the right.
Further, in Albano v. Reyes, we said that while expenditure of public funds may not have been
involved under the questioned contract for the development, management and operation of the

Manila International Container Terminal, public interest [was] definitely involved considering the
important role [of the subject contract] . . . in the economic development of the country and
the magnitude of the financial consideration involved. We concluded that, as a consequence,
the disclosure provision in the Constitution would constitute sufficient authority for upholding the
petitioners standing. (Emphasis supplied)
Clearly, since the instant petition, brought by a citizen, involves matters of transcendental public
importance, the petitioner has the requisite locus standi.
Definition of the Term "Capital" in
Section 11, Article XII of the 1987 Constitution
Section 11, Article XII (National Economy and Patrimony) of the 1987 Constitution mandates the
Filipinization of public utilities, to wit:
Section 11. No franchise, certificate, or any other form of authorization for the operation of a
public utility shall be granted except to citizens of the Philippines or to corporations or
associations organized under the laws of the Philippines, at least sixty per centum of whose
capital is owned by such citizens; nor shall such franchise, certificate, or authorization be
exclusive in character or for a longer period than fifty years. Neither shall any such franchise or right
be granted except under the condition that it shall be subject to amendment, alteration, or repeal by
the Congress when the common good so requires. The State shall encourage equity participation in
public utilities by the general public. The participation of foreign investors in the governing body of
any public utility enterprise shall be limited to their proportionate share in its capital, and all the
executive and managing officers of such corporation or association must be citizens of the
Philippines. (Emphasis supplied)
The above provision substantially reiterates Section 5, Article XIV of the 1973 Constitution, thus:
Section 5. No franchise, certificate, or any other form of authorization for the operation of a
public utility shall be granted except to citizens of the Philippines or to corporations or
associations organized under the laws of the Philippines at least sixty per centum of the
capital of which is owned by such citizens, nor shall such franchise, certificate, or authorization
be exclusive in character or for a longer period than fifty years. Neither shall any such franchise or
right be granted except under the condition that it shall be subject to amendment, alteration, or
repeal by the National Assembly when the public interest so requires. The State shall encourage
equity participation in public utilities by the general public. The participation of foreign investors in the
governing body of any public utility enterprise shall be limited to their proportionate share in the
capital thereof. (Emphasis supplied)
The foregoing provision in the 1973 Constitution reproduced Section 8, Article XIV of the 1935
Constitution, viz:
Section 8. No franchise, certificate, or any other form of authorization for the operation of a
public utility shall be granted except to citizens of the Philippines or to corporations or other
entities organized under the laws of the Philippines sixty per centum of the capital of which is

owned by citizens of the Philippines, nor shall such franchise, certificate, or authorization be
exclusive in character or for a longer period than fifty years. No franchise or right shall be granted to
any individual, firm, or corporation, except under the condition that it shall be subject to amendment,
alteration, or repeal by the Congress when the public interest so requires. (Emphasis supplied)
Father Joaquin G. Bernas, S.J., a leading member of the 1986 Constitutional Commission, reminds
us that the Filipinization provision in the 1987 Constitution is one of the products of the spirit of
nationalism which gripped the 1935 Constitutional Convention.25 The 1987 Constitution "provides for
the Filipinization of public utilities by requiring that any form of authorization for the operation of
public utilities should be granted only to citizens of the Philippines or to corporations or associations
organized under the laws of the Philippines at least sixty per centum of whose capital is owned by
such citizens. The provision is [an express] recognition of the sensitive and vital position of
public utilities both in the national economy and for national security."26 The evident purpose of
the citizenship requirement is to prevent aliens from assuming control of public utilities, which may
be inimical to the national interest.27 This specific provision explicitly reserves to Filipino citizens
control of public utilities, pursuant to an overriding economic goal of the 1987 Constitution: to
"conserve and develop our patrimony"28 and ensure "a self-reliant and independent national
economy effectively controlled by Filipinos."29
Any citizen or juridical entity desiring to operate a public utility must therefore meet the minimum
nationality requirement prescribed in Section 11, Article XII of the Constitution. Hence, for a
corporation to be granted authority to operate a public utility, at least 60 percent of its "capital" must
be owned by Filipino citizens.
The crux of the controversy is the definition of the term "capital." Does the term "capital" in Section
11, Article XII of the Constitution refer to common shares or to the total outstanding capital stock
(combined total of common and non-voting preferred shares)?
Petitioner submits that the 40 percent foreign equity limitation in domestic public utilities refers only
to common shares because such shares are entitled to vote and it is through voting that control over
a corporation is exercised. Petitioner posits that the term "capital" in Section 11, Article XII of the
Constitution refers to "the ownership of common capital stock subscribed and outstanding, which
class of shares alone, under the corporate set-up of PLDT, can vote and elect members of the board
of directors." It is undisputed that PLDTs non-voting preferred shares are held mostly by Filipino
citizens.30 This arose from Presidential Decree No. 217,31 issued on 16 June 1973 by then President
Ferdinand Marcos, requiring every applicant of a PLDT telephone line to subscribe to non-voting
preferred shares to pay for the investment cost of installing the telephone line. 32
Petitioners-in-intervention basically reiterate petitioners arguments and adopt petitioners definition
of the term "capital."33 Petitioners-in-intervention allege that "the approximate foreign ownership of
common capital stock of PLDT x x x already amounts to at least 63.54% of the total outstanding
common stock," which means that foreigners exercise significant control over PLDT, patently
violating the 40 percent foreign equity limitation in public utilities prescribed by the Constitution.

Respondents, on the other hand, do not offer any definition of the term "capital" in Section 11, Article
XII of the Constitution. More importantly, private respondents Nazareno and Pangilinan of PLDT do
not dispute that more than 40 percent of the common shares of PLDT are held by foreigners.
In particular, respondent Nazarenos Memorandum, consisting of 73 pages, harps mainly on the
procedural infirmities of the petition and the supposed violation of the due process rights of the
"affected foreign common shareholders." Respondent Nazareno does not deny petitioners
allegation of foreigners dominating the common shareholdings of PLDT. Nazareno stressed mainly
that the petition "seeks to divest foreign common shareholders purportedly exceeding 40% of
the total common shareholdings in PLDT of their ownership over their shares." Thus, "the
foreign natural and juridical PLDT shareholders must be impleaded in this suit so that they can be
heard."34 Essentially, Nazareno invokes denial of due process on behalf of the foreign common
shareholders.
While Nazareno does not introduce any definition of the term "capital," he states that "among the
factual assertions that need to be established to counter petitioners allegations is the
uniform interpretation by government agencies (such as the SEC), institutions and
corporations (such as the Philippine National Oil Company-Energy Development Corporation
or PNOC-EDC) of including both preferred shares and common shares in "controlling
interest" in view of testing compliance with the 40% constitutional limitation on foreign
ownership in public utilities."35
Similarly, respondent Manuel V. Pangilinan does not define the term "capital" in Section 11, Article
XII of the Constitution. Neither does he refute petitioners claim of foreigners holding more than 40
percent of PLDTs common shares. Instead, respondent Pangilinan focuses on the procedural flaws
of the petition and the alleged violation of the due process rights of foreigners. Respondent
Pangilinan emphasizes in his Memorandum (1) the absence of this Courts jurisdiction over the
petition; (2) petitioners lack of standing; (3) mootness of the petition; (4) non-availability of
declaratory relief; and (5) the denial of due process rights. Moreover, respondent Pangilinan alleges
that the issue should be whether "owners of shares in PLDT as well as owners of shares in
companies holding shares in PLDT may be required to relinquish their shares in PLDT and in those
companies without any law requiring them to surrender their shares and also without notice and
trial."
Respondent Pangilinan further asserts that "Section 11, [Article XII of the Constitution] imposes
no nationality requirement on the shareholders of the utility company as a condition for
keeping their shares in the utility company." According to him, "Section 11 does not authorize
taking one persons property (the shareholders stock in the utility company) on the basis of another
partys alleged failure to satisfy a requirement that is a condition only for that other partys retention
of another piece of property (the utility company being at least 60% Filipino-owned to keep its
franchise)."36
The OSG, representing public respondents Secretary Margarito Teves, Undersecretary John P.
Sevilla, Commissioner Ricardo Abcede, and Chairman Fe Barin, is likewise silent on the definition of
the term "capital." In its Memorandum37 dated 24 September 2007, the OSG also limits its discussion
on the supposed procedural defects of the petition, i.e. lack of standing, lack of jurisdiction, non-

inclusion of interested parties, and lack of basis for injunction. The OSG does not present any
definition or interpretation of the term "capital" in Section 11, Article XII of the Constitution. The OSG
contends that "the petition actually partakes of a collateral attack on PLDTs franchise as a public
utility," which in effect requires a "full-blown trial where all the parties in interest are given their day in
court."38
Respondent Francisco Ed Lim, impleaded as President and Chief Executive Officer of the Philippine
Stock Exchange (PSE), does not also define the term "capital" and seeks the dismissal of the
petition on the following grounds: (1) failure to state a cause of action against Lim; (2) the PSE
allegedly implemented its rules and required all listed companies, including PLDT, to make proper
and timely disclosures; and (3) the reliefs prayed for in the petition would adversely impact the stock
market.
In the earlier case of Fernandez v. Cojuangco, petitioner Fernandez who claimed to be a stockholder
of record of PLDT, contended that the term "capital" in the 1987 Constitution refers to shares entitled
to vote or the common shares. Fernandez explained thus:
The forty percent (40%) foreign equity limitation in public utilities prescribed by the Constitution
refers to ownership of shares of stock entitled to vote, i.e., common shares, considering that it is
through voting that control is being exercised. x x x
Obviously, the intent of the framers of the Constitution in imposing limitations and restrictions on fully
nationalized and partially nationalized activities is for Filipino nationals to be always in control of the
corporation undertaking said activities. Otherwise, if the Trial Courts ruling upholding respondents
arguments were to be given credence, it would be possible for the ownership structure of a public
utility corporation to be divided into one percent (1%) common stocks and ninety-nine percent (99%)
preferred stocks. Following the Trial Courts ruling adopting respondents arguments, the common
shares can be owned entirely by foreigners thus creating an absurd situation wherein foreigners,
who are supposed to be minority shareholders, control the public utility corporation.
xxxx
Thus, the 40% foreign ownership limitation should be interpreted to apply to both the beneficial
ownership and the controlling interest.
xxxx
Clearly, therefore, the forty percent (40%) foreign equity limitation in public utilities prescribed by the
Constitution refers to ownership of shares of stock entitled to vote, i.e., common shares.
Furthermore, ownership of record of shares will not suffice but it must be shown that the legal and
beneficial ownership rests in the hands of Filipino citizens. Consequently, in the case of petitioner
PLDT, since it is already admitted that the voting interests of foreigners which would gain entry to
petitioner PLDT by the acquisition of SMART shares through the Questioned Transactions is
equivalent to 82.99%, and the nominee arrangements between the foreign principals and the Filipino
owners is likewise admitted, there is, therefore, a violation of Section 11, Article XII of the
Constitution.

Parenthetically, the Opinions dated February 15, 1988 and April 14, 1987 cited by the Trial Court to
support the proposition that the meaning of the word "capital" as used in Section 11, Article XII of the
Constitution allegedly refers to the sum total of the shares subscribed and paid-in by the shareholder
and it allegedly is immaterial how the stock is classified, whether as common or preferred, cannot
stand in the face of a clear legislative policy as stated in the FIA which took effect in 1991 or way
after said opinions were rendered, and as clarified by the above-quoted Amendments. In this regard,
suffice it to state that as between the law and an opinion rendered by an administrative agency, the
law indubitably prevails. Moreover, said Opinions are merely advisory and cannot prevail over the
clear intent of the framers of the Constitution.
In the same vein, the SECs construction of Section 11, Article XII of the Constitution is at best
merely advisory for it is the courts that finally determine what a law means. 39
On the other hand, respondents therein, Antonio O. Cojuangco, Manuel V. Pangilinan, Carlos A.
Arellano, Helen Y. Dee, Magdangal B. Elma, Mariles Cacho-Romulo, Fr. Bienvenido F. Nebres, Ray
C. Espinosa, Napoleon L. Nazareno, Albert F. Del Rosario, and Orlando B. Vea, argued that the term
"capital" in Section 11, Article XII of the Constitution includes preferred shares since the Constitution
does not distinguish among classes of stock, thus:
16. The Constitution applies its foreign ownership limitation on the corporations "capital," without
distinction as to classes of shares. x x x
In this connection, the Corporation Code which was already in force at the time the present (1987)
Constitution was drafted defined outstanding capital stock as follows:
Section 137. Outstanding capital stock defined. The term "outstanding capital stock", as used in
this Code, means the total shares of stock issued under binding subscription agreements to
subscribers or stockholders, whether or not fully or partially paid, except treasury shares.
Section 137 of the Corporation Code also does not distinguish between common and preferred
shares, nor exclude either class of shares, in determining the outstanding capital stock (the "capital")
of a corporation. Consequently, petitioners suggestion to reckon PLDTs foreign equity only on the
basis of PLDTs outstanding common shares is without legal basis. The language of the Constitution
should be understood in the sense it has in common use.
xxxx
17. But even assuming that resort to the proceedings of the Constitutional Commission is necessary,
there is nothing in the Record of the Constitutional Commission (Vol. III) which petitioner
misleadingly cited in the Petition x x x which supports petitioners view that only common shares
should form the basis for computing a public utilitys foreign equity.
xxxx
18. In addition, the SEC the government agency primarily responsible for implementing the
Corporation Code, and which also has the responsibility of ensuring compliance with the

Constitutions foreign equity restrictions as regards nationalized activities x x x has categorically


ruled that both common and preferred shares are properly considered in determining outstanding
capital stock and the nationality composition thereof. 40
We agree with petitioner and petitioners-in-intervention. The term "capital" in Section 11, Article XII of
the Constitution refers only to shares of stock entitled to vote in the election of directors, and thus in
the present case only to common shares,41 and not to the total outstanding capital stock comprising
both common and non-voting preferred shares.
The Corporation Code of the Philippines42 classifies shares as common or preferred, thus:
Sec. 6. Classification of shares. - The shares of stock of stock corporations may be divided into
classes or series of shares, or both, any of which classes or series of shares may have such rights,
privileges or restrictions as may be stated in the articles of incorporation: Provided, That no share
may be deprived of voting rights except those classified and issued as "preferred" or
"redeemable" shares, unless otherwise provided in this Code: Provided, further, That there shall
always be a class or series of shares which have complete voting rights. Any or all of the shares or
series of shares may have a par value or have no par value as may be provided for in the articles of
incorporation: Provided, however, That banks, trust companies, insurance companies, public utilities,
and building and loan associations shall not be permitted to issue no-par value shares of stock.
Preferred shares of stock issued by any corporation may be given preference in the distribution of
the assets of the corporation in case of liquidation and in the distribution of dividends, or such other
preferences as may be stated in the articles of incorporation which are not violative of the provisions
of this Code: Provided, That preferred shares of stock may be issued only with a stated par value.
The Board of Directors, where authorized in the articles of incorporation, may fix the terms and
conditions of preferred shares of stock or any series thereof: Provided, That such terms and
conditions shall be effective upon the filing of a certificate thereof with the Securities and Exchange
Commission.
Shares of capital stock issued without par value shall be deemed fully paid and non-assessable and
the holder of such shares shall not be liable to the corporation or to its creditors in respect thereto:
Provided; That shares without par value may not be issued for a consideration less than the value of
five (P5.00) pesos per share: Provided, further, That the entire consideration received by the
corporation for its no-par value shares shall be treated as capital and shall not be available for
distribution as dividends.
A corporation may, furthermore, classify its shares for the purpose of insuring compliance with
constitutional or legal requirements.
Except as otherwise provided in the articles of incorporation and stated in the certificate of stock,
each share shall be equal in all respects to every other share.
Where the articles of incorporation provide for non-voting shares in the cases allowed by this Code,
the holders of such shares shall nevertheless be entitled to vote on the following matters:

1. Amendment of the articles of incorporation;


2. Adoption and amendment of by-laws;
3. Sale, lease, exchange, mortgage, pledge or other disposition of all or substantially all of
the corporate property;
4. Incurring, creating or increasing bonded indebtedness;
5. Increase or decrease of capital stock;
6. Merger or consolidation of the corporation with another corporation or other corporations;
7. Investment of corporate funds in another corporation or business in accordance with this
Code; and
8. Dissolution of the corporation.
Except as provided in the immediately preceding paragraph, the vote necessary to approve a
particular corporate act as provided in this Code shall be deemed to refer only to stocks with voting
rights.
Indisputably, one of the rights of a stockholder is the right to participate in the control or management
of the corporation.43 This is exercised through his vote in the election of directors because it is the
board of directors that controls or manages the corporation. 44 In the absence of provisions in the
articles of incorporation denying voting rights to preferred shares, preferred shares have the same
voting rights as common shares. However, preferred shareholders are often excluded from any
control, that is, deprived of the right to vote in the election of directors and on other matters, on the
theory that the preferred shareholders are merely investors in the corporation for income in the same
manner as bondholders.45 In fact, under the Corporation Code only preferred or redeemable shares
can be deprived of the right to vote.46 Common shares cannot be deprived of the right to vote in any
corporate meeting, and any provision in the articles of incorporation restricting the right of common
shareholders to vote is invalid.47
Considering that common shares have voting rights which translate to control, as opposed to
preferred shares which usually have no voting rights, the term "capital" in Section 11, Article XII of
the Constitution refers only to common shares. However, if the preferred shares also have the right
to vote in the election of directors, then the term "capital" shall include such preferred shares
because the right to participate in the control or management of the corporation is exercised through
the right to vote in the election of directors. In short, the term "capital" in Section 11, Article XII of
the Constitution refers only to shares of stock that can vote in the election of directors.
This interpretation is consistent with the intent of the framers of the Constitution to place in the hands
of Filipino citizens the control and management of public utilities. As revealed in the deliberations of
the Constitutional Commission, "capital" refers to the voting stock or controlling interest of a
corporation, to wit:

MR. NOLLEDO. In Sections 3, 9 and 15, the Committee stated local or Filipino equity and foreign
equity; namely, 60-40 in Section 3, 60-40 in Section 9 and 2/3-1/3 in Section 15.
MR. VILLEGAS. That is right.
MR. NOLLEDO. In teaching law, we are always faced with this question: "Where do we base the
equity requirement, is it on the authorized capital stock, on the subscribed capital stock, or on the
paid-up capital stock of a corporation"? Will the Committee please enlighten me on this?
MR. VILLEGAS. We have just had a long discussion with the members of the team from the UP Law
Center who provided us a draft. The phrase that is contained here which we adopted from the
UP draft is "60 percent of voting stock."
MR. NOLLEDO. That must be based on the subscribed capital stock, because unless declared
delinquent, unpaid capital stock shall be entitled to vote.
MR. VILLEGAS. That is right.
MR. NOLLEDO. Thank you.
With respect to an investment by one corporation in another corporation, say, a corporation with 6040 percent equity invests in another corporation which is permitted by the Corporation Code, does
the Committee adopt the grandfather rule?
MR. VILLEGAS. Yes, that is the understanding of the Committee.
MR. NOLLEDO. Therefore, we need additional Filipino capital?
MR. VILLEGAS. Yes.48
xxxx
MR. AZCUNA. May I be clarified as to that portion that was accepted by the Committee.
MR. VILLEGAS. The portion accepted by the Committee is the deletion of the phrase "voting stock
or controlling interest."
MR. AZCUNA. Hence, without the Davide amendment, the committee report would read:
"corporations or associations at least sixty percent of whose CAPITAL is owned by such citizens."
MR. VILLEGAS. Yes.
MR. AZCUNA. So if the Davide amendment is lost, we are stuck with 60 percent of the capital to be
owned by citizens.
MR. VILLEGAS. That is right.

MR. AZCUNA. But the control can be with the foreigners even if they are the minority. Let us
say 40 percent of the capital is owned by them, but it is the voting capital, whereas, the
Filipinos own the nonvoting shares. So we can have a situation where the corporation is
controlled by foreigners despite being the minority because they have the voting capital. That
is the anomaly that would result here.
MR. BENGZON. No, the reason we eliminated the word "stock" as stated in the 1973 and 1935
Constitutions is that according to Commissioner Rodrigo, there are associations that do not
have stocks. That is why we say "CAPITAL."
MR. AZCUNA. We should not eliminate the phrase "controlling interest."
MR. BENGZON. In the case of stock corporations, it is assumed.49 (Emphasis supplied)
Thus, 60 percent of the "capital" assumes, or should result in, "controlling interest" in the
corporation. Reinforcing this interpretation of the term "capital," as referring to controlling interest or
shares entitled to vote, is the definition of a "Philippine national" in the Foreign Investments Act of
1991,50 to wit:
SEC. 3. Definitions. - As used in this Act:
a. The term "Philippine national" shall mean a citizen of the Philippines; or a domestic partnership or
association wholly owned by citizens of the Philippines; or a corporation organized under the laws
of the Philippines of which at least sixty percent (60%) of the capital stock
outstanding and entitled to vote is owned and held by citizens of the Philippines; or a
corporation organized abroad and registered as doing business in the Philippines under the
Corporation Code of which one hundred percent (100%) of the capital stock outstanding and entitled
to vote is wholly owned by Filipinos or a trustee of funds for pension or other employee retirement or
separation benefits, where the trustee is a Philippine national and at least sixty percent (60%) of the
fund will accrue to the benefit of Philippine nationals: Provided, That where a corporation and its
non-Filipino stockholders own stocks in a Securities and Exchange Commission (SEC) registered
enterprise, at least sixty percent (60%) of the capital stock outstanding and entitled to vote of each of
both corporations must be owned and held by citizens of the Philippines and at least sixty percent
(60%) of the members of the Board of Directors of each of both corporations must be citizens of the
Philippines, in order that the corporation, shall be considered a "Philippine national." (Emphasis
supplied)
In explaining the definition of a "Philippine national," the Implementing Rules and Regulations of the
Foreign Investments Act of 1991 provide:
b. "Philippine national" shall mean a citizen of the Philippines or a domestic partnership or
association wholly owned by the citizens of the Philippines; or a corporation organized under the
laws of the Philippines of which at least sixty percent [60%] of the capital stock outstanding
and entitled to vote is owned and held by citizens of the Philippines; or a trustee of funds for
pension or other employee retirement or separation benefits, where the trustee is a Philippine
national and at least sixty percent [60%] of the fund will accrue to the benefit of the Philippine

nationals; Provided, that where a corporation its non-Filipino stockholders own stocks in a Securities
and Exchange Commission [SEC] registered enterprise, at least sixty percent [60%] of the capital
stock outstanding and entitled to vote of both corporations must be owned and held by citizens of the
Philippines and at least sixty percent [60%] of the members of the Board of Directors of each of both
corporation must be citizens of the Philippines, in order that the corporation shall be considered a
Philippine national. The control test shall be applied for this purpose.
Compliance with the required Filipino ownership of a corporation shall be determined on the
basis of outstanding capital stock whether fully paid or not, but only such stocks which are
generally entitled to vote are considered.
For stocks to be deemed owned and held by Philippine citizens or Philippine nationals, mere
legal title is not enough to meet the required Filipino equity. Full beneficial ownership of the
stocks, coupled with appropriate voting rights is essential. Thus, stocks, the voting rights of
which have been assigned or transferred to aliens cannot be considered held by Philippine
citizens or Philippine nationals.
Individuals or juridical entities not meeting the aforementioned qualifications are considered
as non-Philippine nationals. (Emphasis supplied)
Mere legal title is insufficient to meet the 60 percent Filipino-owned "capital" required in the
Constitution. Full beneficial ownership of 60 percent of the outstanding capital stock, coupled with 60
percent of the voting rights, is required. The legal and beneficial ownership of 60 percent of the
outstanding capital stock must rest in the hands of Filipino nationals in accordance with the
constitutional mandate. Otherwise, the corporation is "considered as non-Philippine national[s]."
Under Section 10, Article XII of the Constitution, Congress may "reserve to citizens of the Philippines
or to corporations or associations at least sixty per centum of whose capital is owned by such
citizens, or such higher percentage as Congress may prescribe, certain areas of investments." Thus,
in numerous laws Congress has reserved certain areas of investments to Filipino citizens or to
corporations at least sixty percent of the "capital" of which is owned by Filipino citizens. Some of
these laws are: (1) Regulation of Award of Government Contracts or R.A. No. 5183; (2) Philippine
Inventors Incentives Act or R.A. No. 3850; (3) Magna Carta for Micro, Small and Medium Enterprises
or R.A. No. 6977; (4) Philippine Overseas Shipping Development Act or R.A. No. 7471; (5) Domestic
Shipping Development Act of 2004 or R.A. No. 9295; (6) Philippine Technology Transfer Act of 2009
or R.A. No. 10055; and (7) Ship Mortgage Decree or P.D. No. 1521. Hence, the term "capital" in
Section 11, Article XII of the Constitution is also used in the same context in numerous
laws reserving certain areas of investments to Filipino citizens.
To construe broadly the term "capital" as the total outstanding capital stock, including both common
and non-voting preferred shares, grossly contravenes the intent and letter of the Constitution that the
"State shall develop a self-reliant and independent national economy effectively controlled by
Filipinos." A broad definition unjustifiably disregards who owns the all-important voting stock, which
necessarily equates to control of the public utility.

We shall illustrate the glaring anomaly in giving a broad definition to the term "capital." Let us
assume that a corporation has 100 common shares owned by foreigners and 1,000,000 non-voting
preferred shares owned by Filipinos, with both classes of share having a par value of one peso
(P1.00) per share. Under the broad definition of the term "capital," such corporation would be
considered compliant with the 40 percent constitutional limit on foreign equity of public utilities since
the overwhelming majority, or more than 99.999 percent, of the total outstanding capital stock is
Filipino owned. This is obviously absurd.
In the example given, only the foreigners holding the common shares have voting rights in the
election of directors, even if they hold only 100 shares. The foreigners, with a minuscule equity of
less than 0.001 percent, exercise control over the public utility. On the other hand, the Filipinos,
holding more than 99.999 percent of the equity, cannot vote in the election of directors and hence,
have no control over the public utility. This starkly circumvents the intent of the framers of the
Constitution, as well as the clear language of the Constitution, to place the control of public utilities in
the hands of Filipinos. It also renders illusory the State policy of an independent national
economy effectively controlled by Filipinos.
The example given is not theoretical but can be found in the real world, and in fact exists in the
present case.
Holders of PLDT preferred shares are explicitly denied of the right to vote in the election of directors.
PLDTs Articles of Incorporation expressly state that "the holders of Serial Preferred Stock shall
not be entitled to vote at any meeting of the stockholders for the election of directors or for
any other purpose or otherwise participate in any action taken by the corporation or its
stockholders, or to receive notice of any meeting of stockholders." 51
On the other hand, holders of common shares are granted the exclusive right to vote in the election
of directors. PLDTs Articles of Incorporation52 state that "each holder of Common Capital Stock shall
have one vote in respect of each share of such stock held by him on all matters voted upon by the
stockholders, and the holders of Common Capital Stock shall have the exclusive right to vote
for the election of directors and for all other purposes."53
In short, only holders of common shares can vote in the election of directors, meaning only common
shareholders exercise control over PLDT. Conversely, holders of preferred shares, who have no
voting rights in the election of directors, do not have any control over PLDT. In fact, under PLDTs
Articles of Incorporation, holders of common shares have voting rights for all purposes, while holders
of preferred shares have no voting right for any purpose whatsoever.
It must be stressed, and respondents do not dispute, that foreigners hold a majority of the
common shares of PLDT. In fact, based on PLDTs 2010 General Information Sheet (GIS), 54 which is
a document required to be submitted annually to the Securities and Exchange
Commission,55 foreigners hold 120,046,690 common shares of PLDT whereas Filipinos hold only
66,750,622 common shares.56 In other words, foreigners hold 64.27% of the total number of PLDTs
common shares, while Filipinos hold only 35.73%. Since holding a majority of the common shares
equates to control, it is clear that foreigners exercise control over PLDT. Such amount of control

unmistakably exceeds the allowable 40 percent limit on foreign ownership of public utilities expressly
mandated in Section 11, Article XII of the Constitution.
Moreover, the Dividend Declarations of PLDT for 2009,57 as submitted to the SEC, shows that per
share the SIP58 preferred shares earn a pittance in dividends compared to the common shares.
PLDT declared dividends for the common shares at P70.00 per share, while the declared dividends
for the preferred shares amounted to a measly P1.00 per share.59 So the preferred shares not only
cannot vote in the election of directors, they also have very little and obviously negligible dividend
earning capacity compared to common shares.
As shown in PLDTs 2010 GIS,60 as submitted to the SEC, the par value of PLDT common shares
is P5.00 per share, whereas the par value of preferred shares is P10.00 per share. In other words,
preferred shares have twice the par value of common shares but cannot elect directors and have
only 1/70 of the dividends of common shares. Moreover, 99.44% of the preferred shares are owned
by Filipinos while foreigners own only a minuscule 0.56% of the preferred shares. 61 Worse, preferred
shares constitute 77.85% of the authorized capital stock of PLDT while common shares constitute
only 22.15%.62 This undeniably shows that beneficial interest in PLDT is not with the non-voting
preferred shares but with the common shares, blatantly violating the constitutional requirement of 60
percent Filipino control and Filipino beneficial ownership in a public utility.
The legal and beneficial ownership of 60 percent of the outstanding capital stock must rest in the
hands of Filipinos in accordance with the constitutional mandate. Full beneficial ownership of 60
percent of the outstanding capital stock, coupled with 60 percent of the voting rights, is
constitutionally required for the States grant of authority to operate a public utility. The undisputed
fact that the PLDT preferred shares, 99.44% owned by Filipinos, are non-voting and earn only 1/70
of the dividends that PLDT common shares earn, grossly violates the constitutional requirement of
60 percent Filipino control and Filipino beneficial ownership of a public utility.
In short, Filipinos hold less than 60 percent of the voting stock, and earn less than 60 percent
of the dividends, of PLDT. This directly contravenes the express command in Section 11, Article XII
of the Constitution that "[n]o franchise, certificate, or any other form of authorization for the operation
of a public utility shall be granted except to x x x corporations x x x organized under the laws of the
Philippines, at least sixty per centum of whose capital is owned by such citizens x x x."
To repeat, (1) foreigners own 64.27% of the common shares of PLDT, which class of shares
exercises the soleright to vote in the election of directors, and thus exercise control over PLDT; (2)
Filipinos own only 35.73% of PLDTs common shares, constituting a minority of the voting stock, and
thus do not exercise control over PLDT; (3) preferred shares, 99.44% owned by Filipinos, have no
voting rights; (4) preferred shares earn only 1/70 of the dividends that common shares earn; 63 (5)
preferred shares have twice the par value of common shares; and (6) preferred shares constitute
77.85% of the authorized capital stock of PLDT and common shares only 22.15%. This kind of
ownership and control of a public utility is a mockery of the Constitution.
Incidentally, the fact that PLDT common shares with a par value of P5.00 have a current stock
market value ofP2,328.00 per share,64 while PLDT preferred shares with a par value of P10.00 per
share have a current stock market value ranging from only P10.92 to P11.06 per share,65 is a glaring

confirmation by the market that control and beneficial ownership of PLDT rest with the common
shares, not with the preferred shares.
Indisputably, construing the term "capital" in Section 11, Article XII of the Constitution to include both
voting and non-voting shares will result in the abject surrender of our telecommunications industry to
foreigners, amounting to a clear abdication of the States constitutional duty to limit control of public
utilities to Filipino citizens. Such an interpretation certainly runs counter to the constitutional
provision reserving certain areas of investment to Filipino citizens, such as the exploitation of natural
resources as well as the ownership of land, educational institutions and advertising businesses. The
Court should never open to foreign control what the Constitution has expressly reserved to Filipinos
for that would be a betrayal of the Constitution and of the national interest. The Court must perform
its solemn duty to defend and uphold the intent and letter of the Constitution to ensure, in the words
of the Constitution, "a self-reliant and independent national economy effectively controlled by
Filipinos."
Section 11, Article XII of the Constitution, like other provisions of the Constitution expressly reserving
to Filipinosspecific areas of investment, such as the development of natural resources and
ownership of land, educational institutions and advertising business, is self-executing. There is no
need for legislation to implement these self-executing provisions of the Constitution. The rationale
why these constitutional provisions are self-executing was explained in Manila Prince Hotel v.
GSIS,66 thus:
x x x Hence, unless it is expressly provided that a legislative act is necessary to enforce a
constitutional mandate, the presumption now is that all provisions of the constitution are selfexecuting. If the constitutional provisions are treated as requiring legislation instead of selfexecuting, the legislature would have the power to ignore and practically nullify the mandate of the
fundamental law. This can be cataclysmic. That is why the prevailing view is, as it has always been,
that
. . . in case of doubt, the Constitution should be considered self-executing rather than non-selfexecuting. . . .Unless the contrary is clearly intended, the provisions of the Constitution
should be considered self-executing, as a contrary rule would give the legislature discretion
to determine when, or whether, they shall be effective. These provisions would be subordinated
to the will of the lawmaking body, which could make them entirely meaningless by simply refusing to
pass the needed implementing statute. (Emphasis supplied)
In Manila Prince Hotel, even the Dissenting Opinion of then Associate Justice Reynato S. Puno, later
Chief Justice, agreed that constitutional provisions are presumed to be self-executing. Justice Puno
stated:
Courts as a rule consider the provisions of the Constitution as self-executing, rather than as
requiring future legislation for their enforcement. The reason is not difficult to discern. For if they are
not treated as self-executing, the mandate of the fundamental law ratified by the sovereign
people can be easily ignored and nullified by Congress. Suffused with wisdom of the ages is
the unyielding rule that legislative actions may give breath to constitutional rights but
congressional inaction should not suffocate them.

Thus, we have treated as self-executing the provisions in the Bill of Rights on arrests, searches and
seizures, the rights of a person under custodial investigation, the rights of an accused, and the
privilege against self-incrimination. It is recognized that legislation is unnecessary to enable courts to
effectuate constitutional provisions guaranteeing the fundamental rights of life, liberty and the
protection of property. The same treatment is accorded to constitutional provisions forbidding the
taking or damaging of property for public use without just compensation. (Emphasis supplied)
Thus, in numerous cases,67 this Court, even in the absence of implementing legislation, applied
directly the provisions of the 1935, 1973 and 1987 Constitutions limiting land ownership to Filipinos.
In Soriano v. Ong Hoo,68this Court ruled:
x x x As the Constitution is silent as to the effects or consequences of a sale by a citizen of his land
to an alien, and as both the citizen and the alien have violated the law, none of them should have a
recourse against the other, and it should only be the State that should be allowed to intervene and
determine what is to be done with the property subject of the violation. We have said that what the
State should do or could do in such matters is a matter of public policy, entirely beyond the scope of
judicial authority. (Dinglasan, et al. vs. Lee Bun Ting, et al., 6 G. R. No. L-5996, June 27,
1956.) While the legislature has not definitely decided what policy should be followed in
cases of violations against the constitutional prohibition, courts of justice cannot go beyond
by declaring the disposition to be null and void as violative of the Constitution. x x x
(Emphasis supplied)
To treat Section 11, Article XII of the Constitution as not self-executing would mean that since the
1935 Constitution, or over the last 75 years, not one of the constitutional provisions expressly
reserving specific areas of investments to corporations, at least 60 percent of the "capital" of which is
owned by Filipinos, was enforceable. In short, the framers of the 1935, 1973 and 1987 Constitutions
miserably failed to effectively reserve to Filipinos specific areas of investment, like the operation by
corporations of public utilities, the exploitation by corporations of mineral resources, the ownership
by corporations of real estate, and the ownership of educational institutions. All the legislatures that
convened since 1935 also miserably failed to enact legislations to implement these vital
constitutional provisions that determine who will effectively control the national economy, Filipinos or
foreigners. This Court cannot allow such an absurd interpretation of the Constitution.
This Court has held that the SEC "has both regulatory and adjudicative functions." 69 Under its
regulatory functions, the SEC can be compelled by mandamus to perform its statutory duty when it
unlawfully neglects to perform the same. Under its adjudicative or quasi-judicial functions, the SEC
can be also be compelled by mandamus to hear and decide a possible violation of any law it
administers or enforces when it is mandated by law to investigate such violation.
1awphi1

Under Section 17(4)70 of the Corporation Code, the SEC has the regulatory function to reject or
disapprove the Articles of Incorporation of any corporation where "the required percentage of
ownership of the capital stock to be owned by citizens of the Philippines has not been
complied with as required by existing laws or the Constitution." Thus, the SEC is the
government agency tasked with the statutory duty to enforce the nationality requirement prescribed
in Section 11, Article XII of the Constitution on the ownership of public utilities. This Court, in a
petition for declaratory relief that is treated as a petition for mandamus as in the present case, can

direct the SEC to perform its statutory duty under the law, a duty that the SEC has apparently
unlawfully neglected to do based on the 2010 GIS that respondent PLDT submitted to the SEC.
Under Section 5(m) of the Securities Regulation Code, 71 the SEC is vested with the "power and
function" to "suspend or revoke, after proper notice and hearing, the franchise or certificate of
registration of corporations, partnerships or associations, upon any of the grounds provided
by law." The SEC is mandated under Section 5(d) of the same Code with the "power and function"
to "investigate x x x the activities of persons to ensure compliance" with the laws and
regulations that SEC administers or enforces. The GIS that all corporations are required to submit to
SEC annually should put the SEC on guard against violations of the nationality requirement
prescribed in the Constitution and existing laws. This Court can compel the SEC, in a petition for
declaratory relief that is treated as a petition for mandamus as in the present case, to hear and
decide a possible violation of Section 11, Article XII of the Constitution in view of the ownership
structure of PLDTs voting shares, as admitted by respondents and as stated in PLDTs 2010 GIS
that PLDT submitted to SEC.
WHEREFORE, we PARTLY GRANT the petition and rule that the term "capital" in Section 11, Article
XII of the 1987 Constitution refers only to shares of stock entitled to vote in the election of directors,
and thus in the present case only to common shares, and not to the total outstanding capital stock
(common and non-voting preferred shares). Respondent Chairperson of the Securities and
Exchange Commission is DIRECTED to apply this definition of the term "capital" in determining the
extent of allowable foreign ownership in respondent Philippine Long Distance Telephone Company,
and if there is a violation of Section 11, Article XII of the Constitution, to impose the appropriate
sanctions under the law.

Republic of the Philippines


SUPREME COURT
Manila
EN BANC
G.R. No. 161872

April 13, 2004

REV. ELLY CHAVEZ PAMATONG, ESQUIRE, petitioner,


vs.
COMMISSION ON ELECTIONS, respondent.
RESOLUTION
TINGA, J.:
Petitioner Rev. Elly Velez Pamatong filed his Certificate of Candidacy for President on December 17,
2003. Respondent Commission on Elections (COMELEC) refused to give due course to
petitioners Certificate of Candidacy in its Resolution No. 6558 dated January 17, 2004. The
decision, however, was not unanimous since Commissioners Luzviminda G. Tancangco and Mehol

K. Sadain voted to include petitioner as they believed he had parties or movements to back up his
candidacy.
On January 15, 2004, petitioner moved for reconsideration of Resolution No.
6558. Petitioners Motion for Reconsideration was docketed as SPP (MP) No. 04-001. The
COMELEC, acting on petitioners Motion for Reconsideration and on similar motions filed by other
aspirants for national elective positions, denied the same under the aegis of Omnibus Resolution
No. 6604 dated February 11, 2004. The COMELEC declared petitioner and thirty-five (35) others
nuisance candidates who could not wage a nationwide campaign and/or are not nominated by a
political party or are not supported by a registered political party with a national constituency.
Commissioner Sadain maintained his vote for petitioner. By then, Commissioner Tancangco had
retired.
In this Petition For Writ of Certiorari, petitioner seeks to reverse the resolutions which were allegedly
rendered in violation of his right to "equal access to opportunities for public service" under Section
26, Article II of the 1987
Constitution,1 by limiting the number of qualified candidates only to those who can afford to wage a
nationwide campaign and/or are nominated by political parties. In so doing, petitioner argues that the
COMELEC indirectly amended the constitutional provisions on the electoral process and limited the
power of the sovereign people to choose their leaders. The COMELEC supposedly erred in
disqualifying him since he is the most qualified among all the presidential candidates, i.e., he
possesses all the constitutional and legal qualifications for the office of the president, he is capable
of waging a national campaign since he has numerous national organizations under his leadership,
he also has the capacity to wage an international campaign since he has practiced law in other
countries, and he has a platform of government. Petitioner likewise attacks the validity of the form for
theCertificate of Candidacy prepared by the COMELEC. Petitioner claims that the form does not
provide clear and reasonable guidelines for determining the qualifications of candidates since it does
not ask for the candidates bio-data and his program of government.
First, the constitutional and legal dimensions involved.
Implicit in the petitioners invocation of the constitutional provision ensuring "equal access to
opportunities for public office" is the claim that there is a constitutional right to run for or hold public
office and, particularly in his case, to seek the presidency. There is none. What is recognized is
merely a privilege subject to limitations imposed by law. Section 26, Article II of the Constitution
neither bestows such a right nor elevates the privilege to the level of an enforceable right. There is
nothing in the plain language of the provision which suggests such a thrust or justifies an
interpretation of the sort.
The "equal access" provision is a subsumed part of Article II of the Constitution, entitled "Declaration
of Principles and State Policies." The provisions under the Article are generally considered not selfexecuting,2 and there is no plausible reason for according a different treatment to the "equal access"
provision. Like the rest of the policies enumerated in Article II, the provision does not contain any
judicially enforceable constitutional right but merely specifies a guideline for legislative or executive
action.3 The disregard of the provision does not give rise to any cause of action before the courts. 4

An inquiry into the intent of the framers5 produces the same determination that the provision is not
self-executory. The original wording of the present Section 26, Article II had read, "The State shall
broaden opportunities to public office and prohibit public dynasties." 6 Commissioner (now Chief
Justice) Hilario Davide, Jr. successfully brought forth an amendment that changed the word
"broaden" to the phrase "ensure equal access," and the substitution of the word "office" to "service."
He explained his proposal in this wise:
I changed the word "broaden" to "ENSURE EQUAL ACCESS TO" because what is important
would be equal access to the opportunity. If you broaden, it would necessarily mean that
the government would be mandated to create as many offices as are possible to
accommodate as many people as are also possible. That is the meaning of broadening
opportunities to public service. So, in order that we should not mandate the State to
make the government the number one employer and to limit offices only to what may
be necessary and expedient yet offering equal opportunities to access to it, I change
the word "broaden."7 (emphasis supplied)
Obviously, the provision is not intended to compel the State to enact positive measures that would
accommodate as many people as possible into public office. The approval of the "Davide
amendment" indicates the design of the framers to cast the provision as simply enunciatory of a
desired policy objective and not reflective of the imposition of a clear State burden.
Moreover, the provision as written leaves much to be desired if it is to be regarded as the source of
positive rights. It is difficult to interpret the clause as operative in the absence of legislation since its
effective means and reach are not properly defined. Broadly written, the myriad of claims that can be
subsumed under this rubric appear to be entirely open-ended.8 Words and phrases such as "equal
access," "opportunities," and "public service" are susceptible to countless interpretations owing to
their inherent impreciseness. Certainly, it was not the intention of the framers to inflict on the people
an operative but amorphous foundation from which innately unenforceable rights may be sourced.
As earlier noted, the privilege of equal access to opportunities to public office may be subjected to
limitations. Some valid limitations specifically on the privilege to seek elective office are found in the
provisions9 of the Omnibus Election Code on "Nuisance Candidates" and COMELEC Resolution No.
645210 dated December 10, 2002 outlining the instances wherein the COMELEC may motu
proprio refuse to give due course to or cancel aCertificate of Candidacy.
As long as the limitations apply to everybody equally without discrimination, however, the equal
access clause is not violated. Equality is not sacrificed as long as the burdens engendered by the
limitations are meant to be borne by any one who is minded to file a certificate of candidacy. In the
case at bar, there is no showing that any person is exempt from the limitations or the burdens which
they create.
Significantly, petitioner does not challenge the constitutionality or validity of Section 69 of the
Omnibus Election Code and COMELEC Resolution No. 6452 dated 10 December 2003. Thus, their
presumed validity stands and has to be accorded due weight.

Clearly, therefore, petitioners reliance on the equal access clause in Section 26, Article II of the
Constitution is misplaced.
The rationale behind the prohibition against nuisance candidates and the disqualification of
candidates who have not evinced a bona fide intention to run for office is easy to divine. The State
has a compelling interest to ensure that its electoral exercises are rational, objective, and orderly.
Towards this end, the State takes into account the practical considerations in conducting elections.
Inevitably, the greater the number of candidates, the greater the opportunities for logistical
confusion, not to mention the increased allocation of time and resources in preparation for the
election. These practical difficulties should, of course, never exempt the State from the conduct of a
mandated electoral exercise. At the same time, remedial actions should be available to alleviate
these logistical hardships, whenever necessary and proper. Ultimately, a disorderly election is not
merely a textbook example of inefficiency, but a rot that erodes faith in our democratic institutions. As
the United States Supreme Court held:
[T]here is surely an important state interest in requiring some preliminary showing of a
significant modicum of support before printing the name of a political organization and its
candidates on the ballot the interest, if no other, in avoiding confusion, deception and even
frustration of the democratic [process].11
The COMELEC itself recognized these practical considerations when it promulgated Resolution No.
6558 on 17 January 2004, adopting the study Memorandum of its Law Department dated 11 January
2004. As observed in the COMELECs Comment:
There is a need to limit the number of candidates especially in the case of candidates for
national positions because the election process becomes a mockery even if those who
cannot clearly wage a national campaign are allowed to run. Their names would have to be
printed in the Certified List of Candidates, Voters Information Sheet and the Official Ballots.
These would entail additional costs to the government. For the official ballots in automated
counting and canvassing of votes, an additional page would amount to more or less FOUR
HUNDRED FIFTY MILLION PESOS (P450,000,000.00).
xxx[I]t serves no practical purpose to allow those candidates to continue if they cannot wage
a decent campaign enough to project the prospect of winning, no matter how slim. 12
The preparation of ballots is but one aspect that would be affected by allowance of "nuisance
candidates" to run in the elections. Our election laws provide various entitlements for candidates for
public office, such as watchers in every polling place, 13 watchers in the board of canvassers,14 or
even the receipt of electoral contributions.15Moreover, there are election rules and regulations the
formulations of which are dependent on the number of candidates in a given election.
Given these considerations, the ignominious nature of a nuisance candidacy becomes even more
galling. The organization of an election with bona fide candidates standing is onerous enough. To
add into the mix candidates with no serious intentions or capabilities to run a viable campaign would
actually impair the electoral process. This is not to mention the candidacies which are palpably
ridiculous so as to constitute a one-note joke. The poll body would be bogged by irrelevant minutiae

covering every step of the electoral process, most probably posed at the instance of these nuisance
candidates. It would be a senseless sacrifice on the part of the State.
Owing to the superior interest in ensuring a credible and orderly election, the State could exclude
nuisance candidates and need not indulge in, as the song goes, "their trips to the moon on
gossamer wings."
The Omnibus Election Code and COMELEC Resolution No. 6452 are cognizant of the compelling
State interest to ensure orderly and credible elections by excising impediments thereto, such as
nuisance candidacies that distract and detract from the larger purpose. The COMELEC is mandated
by the Constitution with the administration of elections16 and endowed with considerable latitude in
adopting means and methods that will ensure the promotion of free, orderly and honest
elections.17 Moreover, the Constitution guarantees that only bona fidecandidates for public office
shall be free from any form of harassment and discrimination.18 The determination ofbona
fide candidates is governed by the statutes, and the concept, to our mind is, satisfactorily defined in
the Omnibus Election Code.
Now, the needed factual premises.
However valid the law and the COMELEC issuance involved are, their proper application in the case
of the petitioner cannot be tested and reviewed by this Court on the basis of what is now before it.
The assailed resolutions of the COMELEC do not direct the Court to the evidence which it
considered in determining that petitioner was a nuisance candidate. This precludes the Court from
reviewing at this instance whether the COMELEC committed grave abuse of discretion in
disqualifying petitioner, since such a review would necessarily take into account the matters which
the COMELEC considered in arriving at its decisions.
Petitioner has submitted to this Court mere photocopies of various documents purportedly evincing
his credentials as an eligible candidate for the presidency. Yet this Court, not being a trier of facts,
can not properly pass upon the reproductions as evidence at this level. Neither the COMELEC nor
the Solicitor General appended any document to their respective Comments.
The question of whether a candidate is a nuisance candidate or not is both legal and factual. The
basis of the factual determination is not before this Court. Thus, the remand of this case for the
reception of further evidence is in order.
A word of caution is in order. What is at stake is petitioners aspiration and offer to serve in the
government. It deserves not a cursory treatment but a hearing which conforms to the requirements
of due process.
As to petitioners attacks on the validity of the form for the certificate of candidacy, suffice it to say
that the form strictly complies with Section 74 of the Omnibus Election Code. This provision
specifically enumerates what a certificate of candidacy should contain, with the required information
tending to show that the candidate possesses the minimum qualifications for the position aspired for
as established by the Constitution and other election laws.

IN VIEW OF THE FOREGOING, COMELEC Case No. SPP (MP) No. 04-001 is hereby remanded to
the COMELEC for the reception of further evidence, to determine the question on whether petitioner
Elly Velez Lao Pamatong is a nuisance candidate as contemplated in Section 69 of the Omnibus
Election Code.
The COMELEC is directed to hold and complete the reception of evidence and report its findings to
this Court with deliberate dispatch.
SO ORDERED.

Republic of the Philippines


SUPREME COURT
Manila
EN BANC

G.R. No. 110526 February 10, 1998


ASSOCIATION OF PHILIPPINE COCONUT DESICCATORS, petitioner,
vs.
PHILIPPINE COCONUT AUTHORITY, respondent.

MENDOZA, J.:
At issue in this case is the validity of a resolution, dated March 24, 1993, of the Philippine Coconut
Authority in which it declares that it will no longer require those wishing to engage in coconut
processing to apply to it for a license or permit as a condition for engaging in such business.
Petitioner Association of Philippine Coconut Desiccators (hereafter referred to as APCD) brought this
suit forcertiorari and mandamus against respondent Philippine Coconut Authority (PCA) to invalidate
the latter's Board Resolution No. 018-93 and the certificates of registration issued under it on the
ground that the resolution in question is beyond the power of the PCA to adopt, and to compel said
administrative agency to comply instead with the mandatory provisions of statutes regulating the
desiccated coconut industry, in particular, and the coconut industry, in general.
As disclosed by the parties' pleadings, the facts are as follows:
On November 5, 1992, seven desiccated coconut processing companies belonging to the APCD
brought suit in the Regional Trial Court, National Capital Judicial Region in Makati, Metro Manila, to
enjoin the PCA from issuing permits to certain applicants for the establishment of new desiccated
coconut processing plants. Petitioner alleged that the issuance of licenses to the applicants would
violate PCA's Administrative Order No. 02, series of 1991, as the applicants were seeking permits to
operate in areas considered "congested" under the administrative order. 1

On November 6, 1992, the trial court issued a temporary restraining order and, on November 25,
1992, a writ of preliminary injunction, enjoining the PCA from processing and issuing licenses to
Primex Products, Inc., Coco Manila, Superstar (Candelaria) and Superstar (Davao) upon the posting
of a bond in the amount of P100,000.00.2
Subsequently and while the case was pending in the Regional Trial Court, the Governing Board of
the PCA issued on March 24, 1993 Resolution No. 018-93, providing for the withdrawal of the
Philippine Coconut Authority from all regulation of the coconut product processing industry. While it
continues the registration of coconut product processors, the registration would be limited to the
"monitoring" of their volumes of production and administration of quality standards. The full text of
the resolution reads:
RESOLUTION NO. 018-93
POLICY DECLARATION DEREGULATING
THE ESTABLISHMENT OF NEW COCONUT
PROCESSING PLANTS
WHEREAS, it is the policy of the State to promote free enterprise unhampered by protective
regulations and unnecessary bureaucratic red tapes;
WHEREAS, the deregulation of certain sectors of the coconut industry, such as marketing of
coconut oils pursuant to Presidential Decree No. 1960, the lifting of export and commodity
clearances under Executive Order No. 1016, and relaxation of regulated capacity for the
desiccated coconut sector pursuant to Presidential Memorandum of February 11, 1988, has
become a centerpiece of the present dispensation;
WHEREAS, the issuance of permits or licenses prior to business operation is a form of
regulation which is not provided in the charter of nor included among the powers of the PCA;
WHEREAS, the Governing Board of PCA has determined to follow and further support the
deregulation policy and effort of the government to promote free enterprise;
NOW THEREFORE, BE IT RESOLVED AS IT IS HEREBY RESOLVED, that, henceforth,
PCA shall no longer require any coconut oil mill, coconut oil refinery, coconut desiccator,
coconut product processor/factory, coconut fiber plant or any similar coconut processing
plant to apply with PCA and the latter shall no longer issue any form of license or permit as
condition prior to establishment or operation of such mills or plants;
RESOLVED, FURTHER, that the PCA shall limit itself only to simply registering the
aforementioned coconut product processors for the purpose of monitoring their volumes of
production, administration of quality standards with the corresponding service fees/charges.
ADOPTED this 24th day of March 1993, at Quezon City. 3
The PCA then proceeded to issue "certificates of registration" to those wishing to operate desiccated
coconut processing plants, prompting petitioner to appeal to the Office of the President of the
Philippines on April 26, 1993 not to approve the resolution in question. Despite follow-up letters sent
on May 25 and June 2, 1993, petitioner received no reply from the Office of the President. The
"certificates of registration" issued in the meantime by the PCA has enabled a number of new
coconut mills to operate. Hence this petition.

Petitioner alleges:
I
RESPONDENT PCA'S BOARD RESOLUTION NO. 018-93 IS NULL AND VOID FOR BEING
AN UNDUE EXERCISE OF LEGISLATIVE POWER BY AN ADMINISTRATIVE BODY.
II
ASIDE FROM BEING ULTRA-VIRES, BOARD RESOLUTION NO. 018-93 IS WITHOUT
ANY BASIS, ARBITRARY, UNREASONABLE AND THEREFORE IN VIOLATION OF
SUBSTANTIVE DUE PROCESS OF LAW.
III
IN PASSING BOARD RESOLUTION NO. 018-93, RESPONDENT PCA VIOLATED THE
PROCEDURAL DUE PROCESS REQUIREMENT OF CONSULTATION PROVIDED IN
PRESIDENTIAL DECREE NO. 1644, EXECUTIVE ORDER NO. 826 AND PCA
ADMINISTRATIVE ORDER NO. 002, SERIES OF 1991.
On the other hand, in addition to answering petitioner's arguments, respondent PCA alleges that this
petition should be denied on the ground that petitioner has a pending appeal before the Office of the
President. Respondent accuses petitioner of forum-shopping in filing this petition and of failing to
exhaust available administrative remedies before coming to this Court. Respondent anchors its
argument on the general rule that one who brings an action under Rule 65 must show that one has
no appeal nor any plain, speedy, and adequate remedy in the ordinary course of law.
I.
The rule of requiring exhaustion of administrative remedies before a party may seek judicial review,
so strenuously urged by the Solicitor General on behalf of respondent, has obviously no application
here. The resolution in question was issued by the PCA in the exercise of its rule-making or
legislative power. However, only judicial review of decisions of administrative agencies made in the
exercise of their quasi-judicial function is subject to the exhaustion doctrine. The exhaustion doctrine
stands as a bar to an action which is not yet complete4 and it is clear, in the case at bar, that after its
promulgation the resolution of the PCA abandoning regulation of the desiccated coconut industry became
effective. To be sure, the PCA is under the direct supervision of the President of the Philippines but there
is nothing in P.D. No. 232, P.D. No. 961, P.D. No. 1468 and P.D. No. 1644 defining the powers and
functions of the PCA which requires rules and regulations issued by it to be approved by the President
before they become effective.
In any event, although the APCD has appealed the resolution in question to the Office of the
President, considering the fact that two months after they had sent their first letter on April 26, 1993
they still had to hear from the President's office, meanwhile respondent PCA was issuing certificates
of registration indiscriminately to new coconut millers, we hold that petitioner was justified in filing
this case on June 25, 1993. 5 Indeed, after writing the Office of the President on April 26, 1993 6 petitioner
sent inquiries to that office not once, but twice, on May 26, 1993 7and on June 2, 1993, 8 but petitioner did
not receive any reply.
II.

We now turn to the merit of the present petition. The Philippine Coconut Authority was originally
created by P.D. 232 on June 30, 1973, to take over the powers and functions of the Coconut
Coordinating Council, the Philippine Coconut Administration and the Philippine Coconut Research
Institute. On June 11, 1978, by P.D. No. 1468, it was made "an independent public corporation . . .
directly reporting to, and supervised by, the President of the Philippines," 9 and charged with carrying
out the State's policy "to promote the rapid integrated development and growth of the coconut and other
palm oil industry in all its aspects and to ensure that the coconut farmers become direct participants in,
and beneficiaries of, such development and growth." 10 through a regulatory scheme set up by law. 11
Through this scheme, the government, on August 28, 1982, temporarily prohibited the opening of
new coconut processing plants and, four months later, phased out some of the existing ones in view
of overproduction in the coconut industry which resulted in cut-throat competition, underselling and
smuggling of poor quality products and ultimately in the decline of the export performance of
coconut-based commodities. The establishment of new plants could be authorized only upon
determination by the PCA of the existence of certain economic conditions and the approval of the
President of the Philippines. Thus, Executive Order No. 826, dated August 28, 1982, provided:
Sec. 1. Prohibition. Except as herein provided, no government agency or instrumentality
shall hereafter authorize, approve or grant any permit or license for the establishment or
operation of new desiccated coconut processing plants, including the importation of
machinery or equipment for the purpose. In the event of a need to establish a new plant, or
expand the capacity, relocate or upgrade the efficiencies of any existing desiccated plant, the
Philippine Coconut Authority may, upon proper determination of such need and evaluation of
the condition relating to:
a. the existing market demand;
b. the production capacity prevailing in the country or locality;
c. the level and flow of raw materials; and
d. other circumstances which may affect the growth or viability of the industry concerned,
authorize or grant the application for, the establishment or expansion of capacity, relocation
or upgrading of efficiencies of such desiccated coconut processing plant, subject to the
approval of the President.
On December 6, 1982, a phase-out of some of the existing plants was ordered by the government
after finding that "a mere freeze in the present capacity of existing plants will not afford a viable
solution to the problem considering that the total available limited market is not adequate to support
all the existing processing plants, making it imperative to reduce the number of existing processing
plants." 12 Accordingly, it was ordered: 13
Sec. 1. The Philippine Coconut Authority is hereby ordered to take such action as may be
necessary to reduce the number of existing desiccated coconut processing plants to a level which
will insure the survival of the remaining plants. The Authority is hereby directed to determine
which of the existing processing plants should be phased out and to enter into appropriate
contracts with such plants for the above purpose.

It was only on October 23, 1987 when the PCA adopted Resolution No. 058-87, authorizing the
establishment and operation of additional DCN plants, in view of the increased demand for
desiccated coconut products in the world's markets, particularly in Germany, the Netherlands and

Australia. Even then, the opening of new plants was made subject to "such implementing guidelines
to be set forth by the Authority" and "subject to the final approval of the President."
The guidelines promulgated by the PCA, as embodied in Administrative Order No. 002, series of
1991, inter aliaauthorized the opening of new plants in "non-congested areas only as declared by the
PCA" and subject to compliance by applicants with "all procedures and requirements for registration
under Administrative Order No. 003, series of 1981 and this Order." In addition, as the opening of
new plants was premised on the increased global demand for desiccated coconut products, the new
entrants were required to submit sworn statements of the names and addresses of prospective
foreign buyers.
This form of "deregulation" was approved by President Aquino in her memorandum, dated February
11, 1988, to the PCA. Affirming the regulatory scheme, the President stated in her memorandum:
It appears that pursuant to Executive Order No. 826 providing measures for the protection of
the Desiccated Coconut Industry, the Philippine Coconut Authority evaluated the conditions
relating to: (a) the existing market demands; (b) the production capacity prevailing in the
country or locality; (c) the level and flow of raw materials; and (d) other circumstances which
may affect the growth or viability of the industry concerned and that the result of such
evaluation favored the expansion of production and market of desiccated coconut products.
In view hereof and the favorable recommendation of the Secretary of Agriculture, the
deregulation of the Desiccated Coconut Industry as recommended in Resolution No. 058-87
adopted by the PCA Governing Board on October 28, 1987 (sic) is hereby approved. 14
These measures the restriction in 1982 on entry into the field, the reduction the same year of the
number of the existing coconut mills and then the lifting of the restrictions in 1987 were adopted
within the framework of regulation as established by law "to promote the rapid integrated
development and growth of the coconut and other palm oil industry in all its aspects and to ensure
that the coconut farmers become direct participants in, and beneficiaries of, such development and
growth." 15 Contrary to the assertion in the dissent, the power given to the Philippine Coconut Authority
and before it to the Philippine Coconut Administration "to formulate and adopt a general program of
development for the coconut and other palm oils industry" 16 is not a roving commission to adopt any
program deemed necessary to promote the development of the coconut and other palm oils industry, but
one to be exercised in the context of this regulatory structure.
In plain disregard of this legislative purpose, the PCA adopted on March 24, 1993 the questioned
resolution which allows not only the indiscriminate opening of new coconut processing plants but the
virtual dismantling of the regulatory infrastructure whereby, forsaking controls theretofore placed in
its keeping, the PCA limits its function to the innocuous one of "monitoring" compliance by coconut
millers with quality standards and volumes of production. In effect, the PCA would simply be
compiling statistical data on these matters, but in case of violations of standards there would be
nothing much it would do. The field would be left without an umpire who would retire to the bleachers
to become a mere spectator. As the PCA provided in its Resolution No. 018-93:
NOW, THEREFORE, BE IT RESOLVED AS IT IS HEREBY RESOLVED, that, henceforth,
PCA shall no longer require any coconut oil mill, coconut oil refinery, coconut desiccator,
coconut product processor/factory, coconut fiber plant or any similar coconut processing
plant to apply with PCA and the latter shall no longer issue any form of license or permit as
condition prior to establishment or operation of such mills or plants;

RESOLVED, FURTHER, that the PCA shall limit itself only to simply registering the
aforementioned coconut product processors for the purpose of monitoring their volumes of
production, administration of quality standards with the corresponding service fees/charges.
The issue is not whether the PCA has the power to adopt this resolution to carry out its mandate
under the law "to promote the accelerated growth and development of the coconut and other palm oil
industry." 17 The issue rather is whether it can renounce the power to regulate implicit in the law creating it
for that is what the resolution in question actually is.
Under Art. II, 3(a) of the Revised Coconut Code (P.D. No. 1468), the role of the PCA is "To
formulate and adopt a general program of development for the coconut and other palm oil industry in
all its aspects." By limiting the purpose of registration to merely "monitoring volumes of production
[and] administration of quality standards" of coconut processing plants, the PCA in effect abdicates
its role and leaves it almost completely to market forces how the coconut industry will develop.
Art. II, 3 of P.D. No. 1468 further requires the PCA:
(h) To regulate the marketing and the exportation of copra and its by-products by establishing
standards for domestic trade and export and, thereafter, to conduct an inspection of all copra
and its by-products proposed for export to determine if they conform to the standards
established;
Instead of determining the qualifications of market players and preventing the entry into the field of
those who are unfit, the PCA now relies entirely on competition with all its wastefulness and
inefficiency to do the weeding out, in its naive belief in survival of the fittest. The result can very
well be a repeat of 1982 when free enterprise degenerated into a "free-for-all," resulting in cut-throat
competition, underselling, the production of inferior products and the like, which badly affected the
foreign trade performance of the coconut industry.
Indeed, by repudiating its role in the regulatory scheme, the PCA has put at risk other statutory
provisions, particularly those of P.D. No. 1644, to wit:
Sec. 1. The Philippine Coconut Authority shall have full power and authority to regulate the
marketing and export of copra, coconut oil and their by-products, in furtherance of the steps
being taken to rationalize the coconut oil milling industry.
Sec. 2. In the exercise of its powers under Section 1 hereof, the Philippine Coconut Authority
may initiate and implement such measures as may be necessary to attain the rationalization
of the coconut oil milling industry, including, but not limited to, the following measures:
(a) Imposition of floor and/or ceiling prices for all exports of copra, coconut oil and their byproducts;
(b) Prescription of quality standards;
(c) Establishment of maximum quantities for particular periods and particular markets;
(d) Inspection and survey of export shipments through an independent international
superintendent or surveyor.

In the exercise of its powers hereunder, the Philippine Coconut Authority shall consult with,
and be guided by, the recommendation of the coconut farmers, through corporations owned
or controlled by them through the Coconut Industry Investment Fund and the private
corporation authorized to be organized under Letter of Instructions No. 926.
and the Revised Coconut Code (P.D. No. 1468), Art. II, 3, to wit:
(m) Except in respect of entities owned or controlled by the Government or by the coconut
farmers under Sections 9 and 10, Article III hereof, the Authority shall have full power and
authority to regulate the production, distribution and utilization of all subsidized coconutbased products, and to require the submission of such reports or documents as may be
deemed necessary by the Authority to ascertain whether the levy payments and/or subsidy
claims are due and correct and whether the subsidized products are distributed among, and
utilized by, the consumers authorized by the Authority.
The dissent seems to be saying that in the same way that restrictions on entry into the field were
imposed in 1982 and then relaxed in 1987, they can be totally lifted now without prejudice to
reimposing them in the future should it become necessary to do so. There is really no renunciation of
the power to regulate, it is claimed. Trimming down of PCA's function to registration is not an
abdication of the power to regulate but is regulation itself. But how can this be done when, under
Resolution No. 018-93, the PCA no longer requires a license as condition for the establishment or
operation of a plant? If a number of processing firms go to areas which are already congested, the
PCA cannot stop them from doing so. If there is overproduction, the PCA cannot order a cut back in
their production. This is because the licensing system is the mechanism for regulation. Without it the
PCA will not be able to regulate coconut plants or mills.
In the first "whereas" clause of the questioned resolution as set out above, the PCA invokes a policy
of free enterprise that is "unhampered by protective regulations and unnecessary bureaucratic red
tape" as justification for abolishing the licensing system. There can be no quarrel with the elimination
of "unnecessary red tape." That is within the power of the PCA to do and indeed it should eliminate
red tape. Its success in doing so will be applauded. But free enterprise does not call for removal of
"protective regulations."
Our Constitutions, beginning with the 1935 document, have repudiated laissez-faire as an economic
principle. 18Although the present Constitution enshrines free enterprise as a policy, 19 it nonetheless
reserves to the government the power to intervene whenever necessary to promote the general welfare.
This is clear from the following provisions of Art. XII of the Constitution which, so far as pertinent, state:
Sec. 6. . . . Individuals and private groups, including corporations, cooperatives, and similar
collective organizations, shall have the right to own, establish, and operate economic
enterprises, subject to the duty of the State to promote distributive justice and to intervene
when the common good so demands.
Sec. 19. The State shall regulate or prohibit monopolies when the public interest so requires.
No combinations in restraint of trade or unfair competition shall be allowed. (Emphasis
added).
At all events, any change in policy must be made by the legislative department of the government.
The regulatory system has been set up by law. It is beyond the power of an administrative agency to
dismantle it. Indeed, petitioner charges the PCA of seeking to render moot a case filed by some of its
members questioning the grant of licenses to certain parties by adopting the resolution in question. It
is alleged that members of petitioner complained to the court that the PCA had authorized the

establishment and operation of new plants in areas which were already crowded, in violation of its
Administrative Order No. 002, series of 1991. In response, the Regional Trial Court issued a writ of
preliminary injunction, enjoining the PCA from issuing licenses to the private respondent in that case.
These allegations of petitioner have not been denied here. It would thus seem that instead of
defending its decision to allow new entrants into the field against petitioner's claim that the PCA
decision violated the guidelines in Administrative Order No. 002, series of 1991, the PCA adopted
the resolution in question to render the case moot. In so doing, the PCA abdicated its function of
regulation and left the field to untrammeled competition that is likely to resurrect the evils of cutthroat competition, underselling and overproduction which in 1982 required the temporary closing of
the field to new players in order to save the industry.
The PCA cannot rely on the memorandum of then President Aquino for authority to adopt the
resolution in question. As already stated, what President Aquino approved in 1988 was the
establishment and operation of new DCN plants subject to the guidelines to be drawn by the
PCA. 20 In the first place, she could not have intended to amend the several laws already mentioned,
which set up the regulatory system, by a mere memoranda to the PCA. In the second place, even if that
had been her intention, her act would be without effect considering that, when she issued the
memorandum in question on February 11, 1988, she was no longer vested with legislative authority. 21
WHEREFORE, the petition is GRANTED. PCA Resolution No. 018-93 and all certificates of
registration issued under it are hereby declared NULL and VOID for having been issued in excess of
the power of the Philippine Coconut Authority to adopt or issue.
SO ORDERED.

Republic of the Philippines


SUPREME COURT
Manila
G.R. No. 193459

February 15, 2011

MA. MERCEDITAS N. GUTIERREZ Petitioner,


vs.
THE HOUSE OF REPRESENTATIVES COMMITTEE ON JUSTICE, RISA HONTIVEROSBARAQUEL, DANILO D. LIM, FELIPE PESTAO, EVELYN PESTAO, RENATO M. REYES, JR.,
SECRETARY GENERAL OF BAGONG ALYANSANG MAKABAYAN (BAYAN); MOTHER MARY
JOHN MANANZAN, CO-CHAIRPERSON OF PAGBABAGO; DANILO RAMOS, SECRETARYGENERAL OF KILUSANG MAGBUBUKID NG PILIPINAS (KMP); ATTY. EDRE OLALIA, ACTING
SECRETARY GENERAL OF THE NATIONAL UNION OF PEOPLE'S LAWYERS (NUPL);
FERDINAND R. GAITE, CHAIRPERSON, CONFEDERATION FOR UNITY, RECOGNITION AND
ADVANCEMENT OF GOVERNMENT EMPLOYEES (COURAGE); and JAMES TERRY RIDON OF
THE LEAGUE OF FILIPINO STUDENTS (LFS), Respondents.
FELICIANO BELMONTE, JR., Respondent-Intervenor.
DECISION
CARPIO MORALES, J.:

The Ombudsman, Ma. Merceditas Gutierrez (petitioner), challenges via petition for certiorari and
prohibition the Resolutions of September 1 and 7, 2010 of the House of Representatives Committee
on Justice (public respondent).
Before the 15th Congress opened its first session on July 26, 2010 (the fourth Monday of July, in
accordance with Section 15, Article VI of the Constitution) or on July 22, 2010, private respondents
Risa Hontiveros-Baraquel, Danilo Lim, and spouses Felipe and Evelyn Pestao (Baraquel group)
filed an impeachment complaint1 against petitioner, upon the endorsement of Party-List
Representatives Arlene Bag-ao and Walden Bello.2
A day after the opening of the 15th Congress or on July 27, 2010, Atty. Marilyn Barua-Yap, Secretary
General of the House of Representatives, transmitted the impeachment complaint to House Speaker
Feliciano Belmonte, Jr.3who, by Memorandum of August 2, 2010, directed the Committee on Rules
to include it in the Order of Business.4
On August 3, 2010, private respondents Renato Reyes, Jr., Mother Mary John Mananzan, Danilo
Ramos, Edre Olalia, Ferdinand Gaite and James Terry Ridon (Reyes group) filed another
impeachment complaint5 against petitioner with a resolution of endorsement by Party-List
Representatives Neri Javier Colmenares, Teodoro Casio, Rafael Mariano, Luzviminda Ilagan,
Antonio Tinio and Emerenciana de Jesus.6 On even date, the House of Representatives provisionally
adopted the Rules of Procedure in Impeachment Proceedings of the 14th Congress. By letter still of
even date,7 the Secretary General transmitted the Reyes groups complaint to Speaker Belmonte
who, by Memorandum of August 9, 2010,8 also directed the Committee on Rules to include it in the
Order of Business.
On August 10, 2010, House Majority Leader Neptali Gonzales II, as chairperson of the Committee
on Rules,9instructed Atty. Artemio Adasa, Jr., Deputy Secretary General for Operations, through Atty.
Cesar Pareja, Executive Director of the Plenary Affairs Department, to include the two complaints in
the Order of Business,10which was complied with by their inclusion in the Order of Business for the
following day, August 11, 2010.
On August 11, 2010 at 4:47 p.m., during its plenary session, the House of
Representatives simultaneouslyreferred both complaints to public respondent.11
After hearing, public respondent, by Resolution of September 1, 2010, found both complaints
sufficient in form, which complaints it considered to have been referred to it at exactly the same time.
Meanwhile, the Rules of Procedure in Impeachment Proceedings of the 15th Congress was
published on September 2, 2010.
On September 6, 2010, petitioner tried to file a motion to reconsider the September 1, 2010
Resolution of public respondent. Public respondent refused to accept the motion, however, for
prematurity; instead, it advised petitioner to await the notice for her to file an answer to the
complaints, drawing petitioner to furnish copies of her motion to each of the 55 members of public
respondent.
After hearing, public respondent, by Resolution of September 7, 2010, found the two complaints,
which both allege culpable violation of the Constitution and betrayal of public trust,12 sufficient in
substance. The determination of the sufficiency of substance of the complaints by public respondent,
which assumed hypothetically the truth of their allegations, hinged on the issue of whether valid
judgment to impeach could be rendered thereon. Petitioner was served also on September 7, 2010 a
notice directing her to file an answer to the complaints within 10 days. 13

Six days following her receipt of the notice to file answer or on September 13, 2010, petitioner filed
with this Court the present petition with application for injunctive reliefs. The following day or on
September 14, 2010, the Court En Banc RESOLVED to direct the issuance of a status quo
ante order14 and to require respondents to comment on the petition in 10 days. The Court
subsequently, by Resolution of September 21, 2010, directed the Office of the Solicitor General
(OSG) to file in 10 days its Comment on the petition
The Baraquel group which filed the first complaint, the Reyes group which filed the second
complaint, and public respondent (through the OSG and private counsel) filed their respective
Comments on September 27, 29 and 30, 2010.
Speaker Belmonte filed a Motion for Leave to Intervene dated October 4, 2010 which the Court
granted by Resolution of October 5, 2010.
Under an Advisory15 issued by the Court, oral arguments were conducted on October 5 and 12,
2010, followed by petitioners filing of a Consolidated Reply of October 15, 2010 and the filing by the
parties of Memoranda within the given 15-day period.
The petition is harangued by procedural objections which the Court shall first resolve.
Respondents raise the impropriety of the remedies of certiorari and prohibition. They argue that
public respondent was not exercising any judicial, quasi-judicial or ministerial function in taking
cognizance of the two impeachment complaints as it was exercising a political act that is
discretionary in nature,16 and that its function is inquisitorial that is akin to a preliminary
investigation.17
These same arguments were raised in Francisco, Jr. v. House of Representatives. 18 The argument
that impeachment proceedings are beyond the reach of judicial review was debunked in this wise:
The major difference between the judicial power of the Philippine Supreme Court and that of the
U.S. Supreme Court is that while the power of judicial review is only impliedly granted to the U.S.
Supreme Court and is discretionary in nature, that granted to the Philippine Supreme Court and
lower courts, as expressly provided for in the Constitution, is not just a power but also a duty, and it
was given an expanded definition to include the power to correct any grave abuse of discretion on
the part of any government branch or instrumentality.
There are also glaring distinctions between the U.S. Constitution and the Philippine Constitution with
respect to the power of the House of Representatives over impeachment proceedings. While the
U.S. Constitution bestows sole power of impeachment to the House of Representatives without
limitation, our Constitution, though vesting in the House of Representatives the exclusive power to
initiate impeachment cases, provides for several limitations to the exercise of such power as
embodied in Section 3(2), (3), (4) and (5), Article XI thereof. These limitations include the manner of
filing, required vote to impeach, and the one year bar on the impeachment of one and the same
official.
Respondents are also of the view that judicial review of impeachments undermines their finality and
may also lead to conflicts between Congress and the judiciary. Thus, they call upon this Court to
exercise judicial statesmanship on the principle that "whenever possible, the Court should defer to
the judgment of the people expressed legislatively, recognizing full well the perils of judicial
willfulness and pride."

But did not the people also express their will when they instituted the above-mentioned safeguards
in the Constitution? This shows that the Constitution did not intend to leave the matter of
impeachment to the sole discretion of Congress. Instead, it provided for certain well-defined limits, or
in the language of Baker v. Carr,"judicially discoverable standards" for determining the validity of the
exercise of such discretion, through the power of judicial review.
xxxx
There is indeed a plethora of cases in which this Court exercised the power of judicial review over
congressional action. Thus, in Santiago v. Guingona, Jr., this Court ruled that it is well within the
power and jurisdiction of the Court to inquire whether the Senate or its officials committed a violation
of the Constitution or grave abuse of discretion in the exercise of their functions and prerogatives.
In Taada v. Angara, in seeking to nullify an act of the Philippine Senate on the ground that it
contravened the Constitution, it held that the petition raises a justiciable controversy and that when
an action of the legislative branch is seriously alleged to have infringed the Constitution, it becomes
not only the right but in fact the duty of the judiciary to settle the dispute. In Bondoc v. Pineda, this
Court declared null and void a resolution of the House of Representatives withdrawing the
nomination, and rescinding the election, of a congressman as a member of the House Electoral
Tribunal for being violative of Section 17, Article VI of the Constitution. In Coseteng v. Mitra, it held
that the resolution of whether the House representation in the Commission on Appointments was
based on proportional representation of the political parties as provided in Section 18, Article VI of
the Constitution is subject to judicial review. In Daza v. Singson, it held that the act of the House of
Representatives in removing the petitioner from the Commission on Appointments is subject to
judicial review. In Taada v. Cuenco, it held that although under the Constitution, the legislative
power is vested exclusively in Congress, this does not detract from the power of the courts to pass
upon the constitutionality of acts of Congress. In Angara v. Electoral Commission, it ruled that
confirmation by the National Assembly of the election of any member, irrespective of whether his
election is contested, is not essential before such member-elect may discharge the duties and enjoy
the privileges of a member of the National Assembly.
Finally, there exists no constitutional basis for the contention that the exercise of judicial review over
impeachment proceedings would upset the system of checks and balances. Verily, the Constitution
is to be interpreted as a whole and "one section is not to be allowed to defeat another." Both are
integral components of the calibrated system of independence and interdependence that insures
that no branch of government act beyond the powers assigned to it by the Constitution. 19 (citations
omitted; italics in the original; underscoring supplied)
Francisco characterizes the power of judicial review as a duty which, as the expanded certiorari
jurisdiction20 of this Court reflects, includes the power to "determine whether or not there has been a
grave abuse of discretion amounting to lack or excess of jurisdiction on the part of any branch or
instrumentality of the Government."21
In the present case, petitioner invokes the Courts expanded certiorari jurisdiction, using the special
civil actions of certiorari and prohibition as procedural vehicles. The Court finds it well-within its
power to determine whether public respondent committed a violation of the Constitution or gravely
abused its discretion in the exercise of its functions and prerogatives that could translate as lack or
excess of jurisdiction, which would require corrective measures from the Court.
Indubitably, the Court is not asserting its ascendancy over the Legislature in this instance, but simply
upholding the supremacy of the Constitution as the repository of the sovereign will. 22

Respondents do not seriously contest all the essential requisites for the exercise of judicial review,
as they only assert that the petition is premature and not yet ripe for adjudication since petitioner has
at her disposal a plain, speedy and adequate remedy in the course of the proceedings before public
respondent. Public respondent argues that when petitioner filed the present petition 23 on September
13, 2010, it had not gone beyond the determination of the sufficiency of form and substance of the
two complaints.
An aspect of the "case-or-controversy" requirement is the requisite of ripeness. 24 The question of
ripeness is especially relevant in light of the direct, adverse effect on an individual by the challenged
conduct.25 In the present petition, there is no doubt that questions on, inter alia, the validity of the
simultaneous referral of the two complaints and on the need to publish as a mode of promulgating
the Rules of Procedure in Impeachment Proceedings of the House (Impeachment Rules) present
constitutional vagaries which call for immediate interpretation.
The unusual act of simultaneously referring to public respondent two impeachment complaints
presents a novel situation to invoke judicial power. Petitioner cannot thus be considered to have
acted prematurely when she took the cue from the constitutional limitation that only one
impeachment proceeding should be initiated against an impeachable officer within a period of one
year.
And so the Court proceeds to resolve the substantive issue whether public respondent
committed grave abuse of discretion amounting to lack or excess of jurisdiction in issuing its two
assailed Resolutions. Petitioner basically anchors her claim on alleged violation of the due process
clause (Art. III, Sec. 1) and of the one-year bar provision (Art. XI, Sec 3, par. 5) of the Constitution.
Due process of law
Petitioner alleges that public respondents chairperson, Representative Niel Tupas, Jr. (Rep. Tupas),
is the subject of an investigation she is conducting, while his father, former Iloilo Governor Niel
Tupas, Sr., had been charged by her with violation of the Anti-Graft and Corrupt Practices Act before
the Sandiganbayan. To petitioner, the actions taken by her office against Rep. Tupas and his father
influenced the proceedings taken by public respondent in such a way that bias and vindictiveness
played a big part in arriving at the finding of sufficiency of form and substance of the complaints
against her.
The Court finds petitioners allegations of bias and vindictiveness bereft of merit, there being hardly
any indication thereof. Mere suspicion of partiality does not suffice.26
The act of the head of a collegial body cannot be considered as that of the entire body itself. So
GMCR, Inc. v. Bell Telecommunications Phils.27 teaches:
First. We hereby declare that the NTC is a collegial body requiring a majority vote out of the three
members of the commission in order to validly decide a case or any incident therein. Corollarily, the
vote alone of the chairman of the commission, as in this case, the vote of Commissioner Kintanar,
absent the required concurring vote coming from the rest of the membership of the commission to at
least arrive at a majority decision, is not sufficient to legally render an NTC order, resolution or
decision.
Simply put, Commissioner Kintanar is not the National Telecommunications Commission. He alone
does not speak and in behalf of the NTC. The NTC acts through a three-man body x x x. 28

In the present case, Rep. Tupas, public respondent informs, did not, in fact, vote and merely
presided over the proceedings when it decided on the sufficiency of form and substance of the
complaints.29
Even petitioners counsel conceded during the oral arguments that there are no grounds to compel
the inhibition of Rep. Tupas.
JUSTICE CUEVAS:
Well, the Committee is headed by a gentleman who happened to be a respondent in the charges
that the Ombudsman filed. In addition to that[,] his father was likewise a respondent in another case.
How can he be expected to act with impartiality, in fairness and in accordance with law under that
matter, he is only human we grant him that benefit.
JUSTICE MORALES:
Is he a one-man committee?
JUSTICE CUEVAS:
He is not a one-man committee, Your Honor, but he decides.
JUSTICE MORALES:
Do we presume good faith or we presume bad faith?
JUSTICE CUEVAS:
We presume that he is acting in good faith, Your Honor, but then (interrupted)
JUSTICE MORALES:
So, that he was found liable for violation of the Anti Graft and Corrupt Practices Act, does that mean
that your client will be deprived of due process of law?
JUSTICE CUEVAS:
No, what we are stating, Your Honor, is that expectation of a client goes with the Ombudsman, which
goes with the element of due process is the lack of impartiality that may be expected of him.
JUSTICE MORALES:
But as you admitted the Committee is not a one-man committee?
JUSTICE CUEVAS:
That is correct, Your Honor.
JUSTICE MORALES:

So, why do you say then that there is a lack of impartiality?


JUSTICE CUEVAS:
Because if anything before anything goes (sic) he is the presiding officer of the committee as in this
case there were objections relative to the existence of the implementing rules not heard, there was
objection made by Congressman Golez to the effect that this may give rise to a constitutional crisis.
JUSTICE MORALES:
That called for a voluntary inhibition. Is there any law or rule you can cite which makes it mandatory
for the chair of the committee to inhibit given that he had previously been found liable for violation of
a law[?]
JUSTICE CUEVAS:
There is nothing, Your Honor. In our jurisprudence which deals with the situation whereby with that
background as the material or pertinent antecedent that there could be no violation of the right of the
petitioner to due process. What is the effect of notice, hearing if the judgment cannot come from an
impartial adjudicator.30 (emphasis and underscoring supplied)
Petitioner contends that the "indecent and precipitate haste" of public respondent in finding the two
complaints sufficient in form and substance is a clear indication of bias, she pointing out that it only
took public respondent five minutes to arrive thereat.
lawphi1

An abbreviated pace in the conduct of proceedings is not per se an indication of bias, however. So
Santos-Concio v. Department of Justice31 holds:
Speed in the conduct of proceedings by a judicial or quasi-judicial officer cannot per se be instantly
attributed to an injudicious performance of functions. For ones prompt dispatch may be anothers
undue haste. The orderly administration of justice remains as the paramount and constant
consideration, with particular regard of the circumstances peculiar to each case.
The presumption of regularity includes the public officers official actuations in all phases of work.
Consistent with such presumption, it was incumbent upon petitioners to present contradictory
evidence other than a mere tallying of days or numerical calculation. This, petitioners failed to
discharge. The swift completion of the Investigating Panels initial task cannot be relegated as
shoddy or shady without discounting the presumably regular performance of not just one but five
state prosecutors.32 (italics in the original; emphasis and underscoring supplied)
Petitioner goes on to contend that her participation in the determination of sufficiency of form and
substance was indispensable. As mandated by the Impeachment Rules, however, and as, in fact,
conceded by petitioners counsel, the participation of the impeachable officer starts with the filing of
an answer.
JUSTICE MORALES:
Is it not that the Committee should first determine that there is sufficiency in form and
substance before she is asked to file her answer (interrupted)
JUSTICE CUEVAS:

That is correct, Your Honor.


JUSTICE MORALES:
During which she can raise any defenses she can assail the regularity of the proceedings and
related irregularities?
JUSTICE CUEVAS:
Yes. We are in total conformity and in full accord with that statement, Your Honor, because it is only
after a determination that the complaint is sufficient in form and substance that a complaint may be
filed, Your Honor, without that but it may be asked, how is not your action premature, Your Honor, our
answer is- no, because of the other violations involved and that is (interrupted). 33 (emphasis and
underscoring supplied)
Rule III(A) of the Impeachment Rules of the 15th Congress reflects the impeachment procedure at
the Committee-level, particularly Section 534 which denotes that petitioners initial participation in the
impeachment proceedings the opportunity to file an Answer starts after the Committee on Justice
finds the complaint sufficient in form and substance. That the Committee refused to accept
petitioners motion for reconsideration from its finding of sufficiency of form of the impeachment
complaints is apposite, conformably with the Impeachment Rules.
Petitioner further claims that public respondent failed to ascertain the sufficiency of form and
substance of the complaints on the basis of the standards set by the Constitution and its own
Impeachment Rules.35
The claim fails.
The determination of sufficiency of form and substance of an impeachment complaint is an exponent
of the express constitutional grant of rule-making powers of the House of Representatives which
committed such determinative function to public respondent. In the discharge of that power and in
the exercise of its discretion, the House has formulated determinable standards as to the form and
substance of an impeachment complaint. Prudential considerations behoove the Court to respect the
compliance by the House of its duty to effectively carry out the constitutional purpose, absent any
contravention of the minimum constitutional guidelines.
Contrary to petitioners position that the Impeachment Rules do not provide for comprehensible
standards in determining the sufficiency of form and substance, the Impeachment Rules are clear in
echoing the constitutional requirements and providing that there must be a "verified complaint or
resolution,"36 and that the substance requirement is met if there is "a recital of facts constituting the
offense charged and determinative of the jurisdiction of the committee."37
Notatu dignum is the fact that it is only in the Impeachment Rules where a determination of
sufficiency of form and substance of an impeachment complaint is made necessary. This
requirement is not explicitly found in the organic law, as Section 3(2), Article XI of the Constitution
basically merely requires a "hearing."38 In the discharge of its constitutional duty, the House deemed
that a finding of sufficiency of form and substance in an impeachment complaint is vital "to effectively
carry out" the impeachment process, hence, such additional requirement in the Impeachment Rules.
Petitioner urges the Court to look into the narration of facts constitutive of the offenses vis--vis her
submissions disclaiming the allegations in the complaints.

This the Court cannot do.


Francisco instructs that this issue would "require the Court to make a determination of what
constitutes an impeachable offense. Such a determination is a purely political question which the
Constitution has left to the sound discretion of the legislature. Such an intent is clear from the
deliberations of the Constitutional Commission. x x x x Clearly, the issue calls upon this court to
decide a non-justiciable political question which is beyond the scope of its judicial power[.]" 39 Worse,
petitioner urges the Court to make a preliminary assessment of certain grounds raised, upon a
hypothetical admission of the facts alleged in the complaints, which involve matters of defense.
In another vein, petitioner, pursuing her claim of denial of due process, questions the lack of or, more
accurately, delay in the publication of the Impeachment Rules.
To recall, days after the 15th Congress opened on July 26, 2010 or on August 3, 2010, public
respondent provisionally adopted the Impeachment Rules of the 14th Congress and thereafter
published on September 2, 2010 its Impeachment Rules, admittedly substantially identical with that
of the 14th Congress, in two newspapers of general circulation. 40
Citing Taada v. Tuvera,41 petitioner contends that she was deprived of due process since the
Impeachment Rules was published only on September 2, 2010 a day after public respondent ruled
on the sufficiency of form of the complaints. She likewise tacks her contention on Section 3(8),
Article XI of the Constitution which directs that "Congress shall promulgate its rules on impeachment
to effectively carry out the purpose of this section."
Public respondent counters that "promulgation" in this case refers to "the publication of rules in any
medium of information, not necessarily in the Official Gazette or newspaper of general circulation." 42
Differentiating Neri v. Senate Committee on Accountability of Public Officers and
Investigations43 which held that the Constitution categorically requires publication of the rules of
procedure in legislative inquiries, public respondent explains that the Impeachment Rules is intended
to merely enable Congress to effectively carry outthe purpose of Section 3(8), Art. XI of Constitution.
Blacks Law Dictionary broadly defines promulgate as
To publish; to announce officially; to make public as important or obligatory. The formal act of
announcing a statute or rule of court. An administrative order that is given to cause an agency law or
regulation to become known or obligatory.44 (emphasis supplied)
While "promulgation" would seem synonymous to "publication," there is a statutory difference in their
usage.
The Constitution notably uses the word "promulgate" 12 times. 45 A number of those instances
involves the promulgation of various rules, reports and issuances emanating from Congress, this
Court, the Office of the Ombudsman as well as other constitutional offices.
To appreciate the statutory difference in the usage of the terms "promulgate" and "publish," the case
of the Judiciary is in point. In promulgating rules concerning the protection and enforcement of
constitutional rights, pleading, practice and procedure in all courts, the Court has invariably required
the publication of these rules for their effectivity. As far as promulgation of judgments is concerned,
however, promulgation means "the delivery of the decision to the clerk of court for filing and
publication."46

Section 4, Article VII of the Constitution contains a similar provision directing Congress to
"promulgate its rules for the canvassing of the certificates" in the presidential and vice presidential
elections. Notably, when Congress approved its canvassing rules for the May 14, 2010 national
elections on May 25, 2010,47 it did not require the publication thereof for its effectivity. Rather,
Congress made the canvassing rules effective upon its adoption.
In the case of administrative agencies, "promulgation" and "publication" likewise take on different
meanings as they are part of a multi-stage procedure in quasi-legislation. As detailed in one
case,48 the publication of implementing rules occurs after their promulgation or adoption.
Promulgation must thus be used in the context in which it is generally understoodthat is, to make
known. Generalia verba sunt generaliter inteligencia. What is generally spoken shall be generally
understood. Between the restricted sense and the general meaning of a word, the general must
prevail unless it was clearly intended that the restricted sense was to be used. 49
Since the Constitutional Commission did not restrict "promulgation" to "publication," the former
should be understood to have been used in its general sense. It is within the discretion of Congress
to determine on how to promulgate its Impeachment Rules, in much the same way that the Judiciary
is permitted to determine that to promulgate a decision means to deliver the decision to the clerk of
court for filing and publication.
It is not for this Court to tell a co-equal branch of government how to promulgate when the
Constitution itself has not prescribed a specific method of promulgation. The Court is in no position
to dictate a mode of promulgation beyond the dictates of the Constitution.
Publication in the Official Gazette or a newspaper of general circulation is but one avenue for
Congress to make known its rules. Jurisprudence emphatically teaches that
x x x in the absence of constitutional or statutory guidelines or specific rules, this Court is devoid of
any basis upon which to determine the legality of the acts of the Senate relative thereto. On grounds
of respect for the basic concept of separation of powers, courts may not intervene in
the internal affairs of the legislature; it is not within the province of courts to direct Congress how to
do its work. In the words of Justice Florentino P. Feliciano, this Court is of the opinion that where no
specific, operable norms and standards are shown to exist, then the legislature must be given a real
and effective opportunity to fashion and promulgate as well as to implement them, before the courts
may intervene.50 (italics in the original; emphasis and underscoring supplied; citations omitted)
Had the Constitution intended to have the Impeachment Rules published, it could have stated so as
categorically as it did in the case of the rules of procedure in legislative inquiries, per Neri. Other
than "promulgate," there is no other single formal term in the English language to appropriately refer
to an issuance without need of it being published.
IN FINE, petitioner cannot take refuge in Neri since inquiries in aid of legislation under Section 21,
Article VI of the Constitution is the sole instance in the Constitution where there is a categorical
directive to duly publish a set of rules of procedure. Significantly notable in Neri is that with respect
to the issue of publication, the Court anchored its ruling on the 1987 Constitutions directive, without
any reliance on or reference to the 1986 case of Taada v. Tuvera.51 Taada naturally could neither
have interpreted a forthcoming 1987 Constitution nor had kept a tight rein on the Constitutions
intentions as expressed through the allowance of either a categorical term or a general sense of
making known the issuances.

From the deliberations of the Constitutional Commission, then Commissioner, now retired Associate
Justice Florenz Regalado intended Section 3(8), Article XI to be the vehicle for the House to fill the
gaps in the impeachment process.
MR. REGALADO. Mr. Presiding Officer, I have decided to put in an additional section because, for
instance, under Section 3 (2), there is mention of indorsing a verified complaint for impeachment by
any citizen alleging ultimate facts constituting a ground or grounds for impeachment. In other words,
it is just like a provision in the rules of court. Instead, I propose that this procedural requirement, like
indorsement of a complaint by a citizen to avoid harassment or crank complaints, could very well be
taken up in a new section 4 which shall read as follows: THE CONGRESS SHALL PROMULGATE
ITS RULES ON IMPEACHMENT TO EFFECTIVELY CARRY OUT THE PURPOSES THEREOF. I
think all these other procedural requirements could be taken care of by the Rules of
Congress.52 (emphasis and underscoring supplied)
The discussion clearly rejects the notion that the impeachment provisions are not self-executing.
Section 3(8) does not, in any circumstance, operate to suspend the entire impeachment mechanism
which the Constitutional Commission took pains in designing even its details.
As against constitutions of the past, modern constitutions have been generally drafted upon a
different principle and have often become in effect extensive codes of laws intended to operate
directly upon the people in a manner similar to that of statutory enactments, and the function of
constitutional conventions has evolved into one more like that of a legislative body. Hence, unless it
is expressly provided that a legislative act is necessary to enforce a constitutional
mandate, the presumption now is that all provisions of the constitution are self-executing. If
the constitutional provisions are treated as requiring legislation instead of self-executing, the
legislature would have the power to ignore and practically nullify the mandate of the fundamental
law. This can be cataclysmic. That is why the prevailing view is, as it has always been, that
. . . in case of doubt, the Constitution should be considered self-executing rather than nonself-executing . . . . Unless the contrary is clearly intended, the provisions of the Constitution should
be considered self-executing, as a contrary rule would give the legislature discretion to determine
when, or whether, they shall be effective. These provisions would be subordinated to the will of the
lawmaking body, which could make them entirely meaningless by simply refusing to pass the
needed implementing statute.53 (emphasis and underscoring supplied)
Even assuming arguendo that publication is required, lack of it does not nullify the proceedings
taken prior to the effectivity of the Impeachment Rules which faithfully comply with the relevant selfexecuting provisions of the Constitution. Otherwise, in cases where impeachment complaints are
filed at the start of each Congress, the mandated periods under Section 3, Article XI of the
Constitution would already run or even lapse while awaiting the expiration of the 15-day period of
publication prior to the effectivity of the Impeachment Rules. In effect, the House would already
violate the Constitution for its inaction on the impeachment complaints pending the completion of the
publication requirement.
Given that the Constitution itself states that any promulgation of the rules on impeachment is aimed
at "effectively carry[ing] out the purpose" of impeachment proceedings, the Court finds no grave
abuse of discretion when the House deemed it proper to provisionally adopt the Rules on
Impeachment of the 14th Congress, to meet the exigency in such situation of early filing and in
keeping with the "effective" implementation of the "purpose" of the impeachment provisions. In other
words, the provisional adoption of the previous Congress Impeachment Rules is within the power of
the House to promulgate its rules on impeachment to effectively carry out the avowed purpose.

Moreover, the rules on impeachment, as contemplated by the framers of the Constitution, merely aid
or supplement the procedural aspects of impeachment. Being procedural in nature, they may be
given retroactive application to pending actions. "It is axiomatic that the retroactive application of
procedural laws does not violate any right of a person who may feel that he is adversely affected,
nor is it constitutionally objectionable. The reason for this is that, as a general rule, no vested right
may attach to, nor arise from, procedural laws."54 In the present case, petitioner fails to allege any
impairment of vested rights.
It bears stressing that, unlike the process of inquiry in aid of legislation where the rights of witnesses
are involved, impeachment is primarily for the protection of the people as a body politic, and not for
the punishment of the offender.55
Even Neri concedes that the unpublished rules of legislative inquiries were not considered null and
void in its entirety. Rather,
x x x [o]nly those that result in violation of the rights of witnesses should be considered null and void,
considering that the rationale for the publication is to protect the rights of witnesses as expressed in
Section 21, Article VI of the Constitution. Sans such violation, orders and proceedings are
considered valid and effective.56 (emphasis and underscoring supplied)
Petitioner in fact does not deny that she was fully apprised of the proper procedure. She even
availed of and invoked certain provisions57 of the Impeachment Rules when she, on September 7,
2010, filed the motion for reconsideration and later filed the present petition. The Court thus finds no
violation of the due process clause.
The one-year bar rule
Article XI, Section 3, paragraph (5) of the Constitution reads: "No impeachment proceedings shall
be initiatedagainst the same official more than once within a period of one year."
Petitioner reckons the start of the one-year bar from the filing of the first impeachment complaint
against her on July 22, 2010 or four days before the opening on July 26, 2010 of the 15th Congress.
She posits that within one year from July 22, 2010, no second impeachment complaint may be
accepted and referred to public respondent.
On the other hand, public respondent, respondent Reyes group and respondent-intervenor submit
that the initiation starts with the filing of the impeachment complaint and ends with the referral to the
Committee, followingFrancisco, but venture to alternatively proffer that the initiation ends somewhere
between the conclusion of the Committee Report and the transmittal of the Articles of Impeachment
to the Senate. Respondent Baraquel group, meanwhile, essentially maintains that under either the
prevailing doctrine or the parties interpretation, its impeachment complaint could withstand
constitutional scrutiny.
Contrary to petitioners asseveration, Francisco58 states that the term "initiate" means to file the
complaint andtake initial action on it.59 The initiation starts with the filing of the complaint which must
be accompanied with an action to set the complaint moving. It refers to the filing of the impeachment
complaint coupled with Congress taking initial action of said complaint. The initial action taken by
the House on the complaint is the referral of the complaint to the Committee on Justice.
Petitioner misreads the remark of Commissioner Joaquin Bernas, S.J. that "no second verified
impeachment may be accepted and referred to the Committee on Justice for action" 60 which

contemplates a situation where a first impeachment complaint had already been referred. Bernas
and Regalado, who both acted as amici curiae inFrancisco, affirmed that the act of
initiating includes the act of taking initial action on the complaint.
From the records of the Constitutional Commission, to the amicus curiae briefs of two former
Constitutional Commissioners, it is without a doubt that the term "to initiate" refers to the filing of the
impeachment complaint coupled with Congress' taking initial action of said complaint.
Having concluded that the initiation takes place by the act of filing and referral or endorsement of the
impeachment complaint to the House Committee on Justice or, by the filing by at least one-third61 of
the members of the House of Representatives with the Secretary General of the House, the meaning
of Section 3 (5) of Article XI becomes clear. Once an impeachment complaint has been initiated,
another impeachment complaint may not be filed against the same official within a one year
period.62 (emphasis and underscoring supplied)
The Court, in Francisco, thus found that the assailed provisions of the 12th Congress Rules of
Procedure in Impeachment Proceedings Sections 1663 and 1764 of Rule V thereof "clearly
contravene Section 3(5) of Article XI since they g[a]ve the term initiate a meaning different
from filing and referral."65
Petitioner highlights certain portions of Francisco which delve on the relevant records of the
Constitutional Commission, particularly Commissioner Maambongs statements 66 that the initiation
starts with the filing of the complaint.
Petitioner fails to consider the verb "starts" as the operative word. Commissioner Maambong was all
too keen to stress that the filing of the complaint indeed starts the initiation and that the
Houses action on the committee report/resolution is not part of that initiation phase.
Commissioner Maambong saw the need "to be very technical about this," 67 for certain exchanges in
the Constitutional Commission deliberations loosely used the term, as shown in the following
exchanges.
MR. DAVIDE. That is for conviction, but not for initiation. Initiation of impeachment proceedings still
requires a vote of one-fifth of the membership of the House under the 1935 Constitution.
MR. MONSOD. A two-thirds vote of the membership of the House is required to initiate proceedings.
MR. DAVIDE. No. for initiation of impeachment proceedings, only one-fifth vote of the membership
of the House is required; for conviction, a two-thirds vote of the membership is required.
xxxx
MR. DAVIDE. However, if we allow one-fifth of the membership of the legislature to overturn a report
of the committee, we have here Section 3 (4) which reads:
No impeachment proceedings shall be initiated against the same official more than once within a
period of one year.
So, necessarily, under this particular subsection, we will, in effect, disallow one-fifth of the members
of the National Assembly to revive an impeachment move by an individual or an ordinary Member.

MR. ROMULO. Yes. May I say that Section 3 (4) is there to look towards the possibility of a very
liberal impeachment proceeding. Second, we were ourselves struggling with that problem where we
are faced with just a verified complaint rather than the signatures of one-fifth, or whatever it is we
decide, of the Members of the House. So whether to put a period for the Committee to report,
whether we should not allow the Committee to overrule a mere verified complaint, are some of the
questions we would like to be discussed.
MR. DAVIDE. We can probably overrule a rejection by the Committee by providing that it can be
overturned by, say, one-half or a majority, or one-fifth of the members of the legislature, and
that such overturning will not amount to a refiling which is prohibited under Section 3 (4).
Another point, Madam President. x x x68 (emphasis and underscoring supplied)
An apparent effort to clarify the term "initiate" was made by Commissioner Teodulo Natividad:
MR. NATIVIDAD. How many votes are needed to initiate?
MR. BENGZON. One-third.
MR. NATIVIDAD. To initiate is different from to impeach; to impeach is different from to convict.
To impeach means to file the case before the Senate.
MR. REGALADO. When we speak of "initiative," we refer here to the Articles of Impeachment.
MR. NATIVIDAD. So, that is the impeachment itself, because when we impeach, we are charging
him with the Articles of Impeachment. That is my understanding.69 (emphasis and underscoring
supplied)
Capping these above-quoted discussions was the explanation of Commissioner Maambong
delivered on at least two occasions:
[I]
MR. MAAMBONG. Mr. Presiding Officer, I am not moving for a reconsideration of the approval of the
amendment submitted by Commissioner Regalado, but I will just make of record my thinking that we
do not really initiate the filing of the Articles of Impeachment on the floor. The procedure, as I have
pointed out earlier, was that the initiation starts with the filing of the complaint. And what is actually
done on the floor is that the committee resolution containing the Articles of Impeachment is the one
approved by the body.
As the phraseology now runs, which may be corrected by the Committee on Style, it appears that
the initiation starts on the floor. If we only have time, I could cite examples in the case of the
impeachment proceedings of President Richard Nixon wherein the Committee on the Judiciary
submitted the recommendation, the resolution, and the Articles of Impeachment to the body, and it
was the body who approved the resolution. It is not the body which initiates it. It only approves or
disapproves the resolution. So, on that score, probably the Committee on Style could help in
rearranging the words because we have to be very technical about this. I have been bringing with
me The Rules of the House of Representatives of the U.S. Congress. The Senate Rules are with
me. The proceedings on the case of Richard Nixon are with me. I have submitted my proposal, but
the Committee has already decided. Nevertheless, I just want to indicate this on record.

Thank you, Mr. Presiding Officer.70 (italics in the original; emphasis and underscoring supplied)
[II]
MR. MAAMBONG. I would just like to move for a reconsideration of the approval of Section 3 (3). My
reconsideration will not at all affect the substance, but it is only with keeping with the exact
formulation of the Rules of the House of Representatives of the United States regarding
impeachment.
I am proposing, Madam President, without doing damage to any of its provision, that on page 2,
Section 3 (3), from lines 17 to 18, we delete the words which read: "to initiate impeachment
proceedings" and the comma (,) and insert on line 19 after the word "resolution" the phrase WITH
THE ARTICLES, and then capitalize the letter "i" in "impeachment" and replace the word "by" with
OF, so that the whole section will now read: "A vote of at least one-third of all the Members of the
House shall be necessary either to affirm a resolution WITH THE ARTICLES of impeachment OF the
committee or to override its contrary resolution. The vote of each Member shall be recorded."
I already mentioned earlier yesterday that the initiation, as far as the House of Representatives of
the United States is concerned, really starts from the filing of the verified complaint and every
resolution to impeach always carries with it the Articles of Impeachment. As a matter of fact, the
words "Articles of Impeachment" are mentioned on line 25 in the case of the direct filing of a verified
complaint of one-third of all the Members of the House. I will mention again, Madam President, that
my amendment will not vary the substance in any way. It is only in keeping with the uniform
procedure of the House of Representatives of the United States Congress.
Thank you, Madam President.71 (emphasis and underscoring supplied)
To the next logical question of what ends or completes the initiation, Commissioners Bernas and
Regalado lucidly explained that the filing of the complaint must be accompanied by the referral to the
Committee on Justice, which is the action that sets the complaint moving. Francisco cannot be any
clearer in pointing out the material dates.
Having concluded that the initiation takes place by the act of filing of the impeachment complaint and
referral to the House Committee on Justice, the initial action taken thereon, the meaning of Section 3
(5) of Article XI becomes clear. Once an impeachment complaint has been initiated in the foregoing
manner, another may not be filed against the same official within a one year period following Article
XI, Section 3(5) of the Constitution.
In fine, considering that the first impeachment complaint was filed by former President Estrada
against Chief Justice Hilario G. Davide, Jr., along with seven associate justices of this Court,
on June 2, 2003 and referred to the House Committee on Justice on August 5, 2003,
the second impeachment complaint filed by Representatives Gilberto C. Teodoro, Jr. and Felix
William Fuentebella against the Chief Justice on October 23, 2003 violates the constitutional
prohibition against the initiation of impeachment proceedings against the same impeachable officer
within a one-year period.72 (emphasis, italics and underscoring supplied)
These clear pronouncements notwithstanding, petitioner posits that the date of referral was
considered irrelevant in Francisco. She submits that referral could not be the reckoning point of
initiation because "something prior to that had already been done," 73 apparently citing Bernas
discussion.

The Court cannot countenance any attempt at obscurantism.


What the cited discussion was rejecting was the view that the Houses action on the committee
report initiates the impeachment proceedings. It did not state that to determine the initiating step,
absolutely nothing prior to it must be done. Following petitioners line of reasoning, the verification of
the complaint or the endorsement by a member of the House steps done prior to the filing would
already initiate the impeachment proceedings.
Contrary to petitioners emphasis on impeachment complaint, what the Constitution mentions is
impeachment "proceedings." Her reliance on the singular tense of the word "complaint" 74 to denote
the limit prescribed by the Constitution goes against the basic rule of statutory construction that a
word covers its enlarged and plural sense.75
The Court, of course, does not downplay the importance of an impeachment complaint, for it is the
matchstick that kindles the candle of impeachment proceedings. The filing of an impeachment
complaint is like the lighting of a matchstick. Lighting the matchstick alone, however, cannot light up
the candle, unless the lighted matchstick reaches or torches the candle wick. Referring the complaint
to the proper committee ignites the impeachment proceeding. With a simultaneous referral of
multiple complaints filed, more than one lighted matchsticks light the candle at the same time. What
is important is that there should only be ONE CANDLE that is kindled in a year, such that once the
candle starts burning, subsequent matchsticks can no longer rekindle the candle.
A restrictive interpretation renders the impeachment mechanism both illusive and illusory.
For one, it puts premium on senseless haste. Petitioners stance suggests that whoever files the first
impeachment complaint exclusively gets the attention of Congress which sets in motion an
exceptional once-a-year mechanism wherein government resources are devoted. A prospective
complainant, regardless of ill motives or best intentions, can wittingly or unwittingly desecrate the
entire process by the expediency of submitting a haphazard complaint out of sheer hope to be the
first in line. It also puts to naught the effort of other prospective complainants who, after diligently
gathering evidence first to buttress the case, would be barred days or even hours later from filing an
impeachment complaint.
Placing an exceedingly narrow gateway to the avenue of impeachment proceedings turns its
laudable purpose into a laughable matter. One needs only to be an early bird even without seriously
intending to catch the worm, when the process is precisely intended to effectively weed out "worms"
in high offices which could otherwise be ably caught by other prompt birds within the ultra-limited
season.
Moreover, the first-to-file scheme places undue strain on the part of the actual complainants, injured
party or principal witnesses who, by mere happenstance of an almost always unforeseeable filing of
a first impeachment complaint, would be brushed aside and restricted from directly participating in
the impeachment process.
Further, prospective complainants, along with their counsel and members of the House of
Representatives who sign, endorse and file subsequent impeachment complaints against the same
impeachable officer run the risk of violating the Constitution since they would have already initiated a
second impeachment proceeding within the same year. Virtually anybody can initiate a second or
third impeachment proceeding by the mere filing of endorsed impeachment complaints. Without any
public notice that could charge them with knowledge, even members of the House of
Representatives could not readily ascertain whether no other impeachment complaint has been filed
at the time of committing their endorsement.

The question as to who should administer or pronounce that an impeachment proceeding has been
initiated rests also on the body that administers the proceedings prior to the impeachment trial. As
gathered from Commissioner Bernas disquisition76 in Francisco, a proceeding which "takes place not
in the Senate but in the House"77precedes the bringing of an impeachment case to the Senate. In
fact, petitioner concedes that the initiation of impeachment proceedings is within the sole and
absolute control of the House of Representatives. 78 Conscious of the legal import of each step, the
House, in taking charge of its own proceedings, must deliberately decide to initiate an impeachment
proceeding, subject to the time frame and other limitations imposed by the Constitution. This
chamber of Congress alone, not its officers or members or any private individual, should own up to
its processes.
The Constitution did not place the power of the "final say" on the lips of the House Secretary General
who would otherwise be calling the shots in forwarding or freezing any impeachment complaint.
Referral of the complaint to the proper committee is not done by the House Speaker alone either,
which explains why there is a need to include it in the Order of Business of the House. It is the
House of Representatives, in public plenary session, which has the power to set its own chamber
into special operation by referring the complaint or to otherwise guard against the initiation of a
second impeachment proceeding by rejecting a patently unconstitutional complaint.
Under the Rules of the House, a motion to refer is not among those motions that shall be decided
without debate, but any debate thereon is only made subject to the five-minute rule. 79 Moreover, it is
common parliamentary practice that a motion to refer a matter or question to a committee may be
debated upon, not as to the merits thereof, but only as to the propriety of the referral. 80 With respect
to complaints for impeachment, the House has the discretion not to refer a subsequent impeachment
complaint to the Committee on Justice where official records and further debate show that an
impeachment complaint filed against the same impeachable officer has already been referred to the
said committee and the one year period has not yet expired, lest it becomes instrumental in
perpetrating a constitutionally prohibited second impeachment proceeding. Far from being
mechanical, before the referral stage, a period of deliberation is afforded the House, as the
Constitution, in fact, grants a maximum of three session days within which to make the proper
referral.
As mentioned, one limitation imposed on the House in initiating an impeachment proceeding deals
with deadlines. The Constitution states that "[a] verified complaint for impeachment may be filed by
any Member of the House of Representatives or by any citizen upon a resolution or endorsement by
any Member thereof, which shall be included in the Order of Business within ten session days, and
referred to the proper Committee within three session days thereafter."
In the present case, petitioner failed to establish grave abuse of discretion on the allegedly "belated"
referral of the first impeachment complaint filed by the Baraquel group. For while the said complaint
was filed on July 22, 2010, there was yet then no session in Congress. It was only four days later or
on July 26, 2010 that the 15th Congress opened from which date the 10-day session period started
to run. When, by Memorandum of August 2, 2010, Speaker Belmonte directed the Committee on
Rules to include the complaint in its Order of Business, it was well within the said 10-day session
period.81
There is no evident point in rushing at closing the door the moment an impeachment complaint is
filed. Depriving the people (recall that impeachment is primarily for the protection of the people as a
body politic) of reasonable access to the limited political vent simply prolongs the agony and
frustrates the collective rage of an entire citizenry whose trust has been betrayed by an impeachable
officer. It shortchanges the promise of reasonable opportunity to remove an impeachable officer
through the mechanism enshrined in the Constitution.

But neither does the Court find merit in respondents alternative contention that the initiation of the
impeachment proceedings, which sets into motion the one-year bar, should include or await, at the
earliest, the Committee on Justice report. To public respondent, the reckoning point of initiation
should refer to the disposition of the complaint by the vote of at least one-third (1/3) of all the
members of the House.82 To the Reyes group, initiation means the act of transmitting the Articles of
Impeachment to the Senate.83 To respondent-intervenor, it should last until the Committee on
Justices recommendation to the House plenary.84
The Court, in Francisco, rejected a parallel thesis in which a related proposition was inputed in the
therein assailed provisions of the Impeachment Rules of the 12th Congress. The present case
involving an impeachment proceeding against the Ombudsman offers no cogent reason for the
Court to deviate from what was settled inFrancisco that dealt with the impeachment proceeding
against the then Chief Justice. To change the reckoning point of initiation on no other basis but to
accommodate the socio-political considerations of respondents does not sit well in a court of law.
x x x We ought to be guided by the doctrine of stare decisis et non quieta movere. This doctrine,
which is really "adherence to precedents," mandates that once a case has been decided one way,
then another case involving exactly the same point at issue should be decided in the same manner.
This doctrine is one of policy grounded on the necessity for securing certainty and stability of judicial
decisions. As the renowned jurist Benjamin Cardozo stated in his treatise The Nature of the Judicial
Process:
It will not do to decide the same question one way between one set of litigants and the opposite way
between another. "If a group of cases involves the same point, the parties expect the same decision.
It would be a gross injustice to decide alternate cases on opposite principles. If a case was decided
against me yesterday when I was a defendant, I shall look for the same judgment today if I am
plaintiff. To decide differently would raise a feeling of resentment and wrong in my breast; it would be
an infringement, material and moral, of my rights." Adherence to precedent must then be the rule
rather than the exception if litigants are to have faith in the even-handed administration of justice in
the courts.85
As pointed out in Francisco, the impeachment proceeding is not initiated "when the House
deliberates on the resolution passed on to it by the Committee, because something prior to that has
already been done. The action of the House is already a further step in the proceeding, not its
initiation or beginning. Rather, the proceeding is initiated or begins, when a verified complaint is filed
and referred to the Committee on Justice for action. This is the initiating step which triggers the
series of steps that follow."86
Allowing an expansive construction of the term "initiate" beyond the act of referral allows the
unmitigated influx of successive complaints, each having their own respective 60-session-day period
of disposition from referral. Worse, the Committee shall conduct overlapping hearings until and
unless the disposition of one of the complaints ends with the affirmance of a resolution for
impeachment or the overriding87 of a contrary resolution (as espoused by public respondent), or the
House transmits the Articles of Impeachment (as advocated by the Reyes group), 88 or the Committee
on Justice concludes its first report to the House plenary regardless of the recommendation (as
posited by respondent-intervenor). Each of these scenarios runs roughshod the very purpose behind
the constitutionally imposed one-year bar. Opening the floodgates too loosely would disrupt the
series of steps operating in unison under one proceeding.
The Court does not lose sight of the salutary reason of confining only one impeachment proceeding
in a year. Petitioner concededly cites Justice Adolfo Azcunas separate opinion that concurred with
the Francisco ruling.89Justice Azcuna stated that the purpose of the one-year bar is two-fold: "to

prevent undue or too frequentharassment; and 2) to allow the legislature to do its principal task [of]
legislation," with main reference to the records of the Constitutional Commission, that reads:
MR. ROMULO. Yes, the intention here really is to limit. This is not only to protect public officials who,
in this case, are of the highest category from harassment but also to allow the legislative body to do
its work which is lawmaking. Impeachment proceedings take a lot of time. And if we allow multiple
impeachment charges on the same individual to take place, the legislature will do nothing else but
that.90 (underscoring supplied)
It becomes clear that the consideration behind the intended limitation refers to the element of time,
and not the number of complaints. The impeachable officer should defend himself in only one
impeachment proceeding, so that he will not be precluded from performing his official functions and
duties. Similarly, Congress should run only one impeachment proceeding so as not to leave it with
little time to attend to its main work of law-making. The doctrine laid down in Francisco that initiation
means filing and referral remains congruent to the rationale of the constitutional provision.
Petitioner complains that an impeachable officer may be subjected to harassment by the filing of
multiple impeachment complaints during the intervening period of a maximum of 13 session days
between the date of the filing of the first impeachment complaint to the date of referral.
As pointed out during the oral arguments91 by the counsel for respondent-intervenor, the framework
of privilege and layers of protection for an impeachable officer abound. The requirements or
restrictions of a one-year bar, a single proceeding, verification of complaint, endorsement by a
House member, and a finding of sufficiency of form and substance all these must be met before
bothering a respondent to answer already weigh heavily in favor of an impeachable officer.
Aside from the probability of an early referral and the improbability of inclusion in the agenda of a
complaint filed on the 11th hour (owing to pre-agenda standard operating procedure), the number of
complaints may still be filtered or reduced to nil after the Committee decides once and for all on the
sufficiency of form and substance. Besides, if only to douse petitioners fear, a complaint will not last
the primary stage if it does not have the stated preliminary requisites.
To petitioner, disturbance of her performance of official duties and the deleterious effects of bad
publicity are enough oppression.
Petitioners claim is based on the premise that the exertion of time, energy and other resources runs
directly proportional to the number of complaints filed. This is non sequitur. What the Constitution
assures an impeachable officer is not freedom from arduous effort to defend oneself, which depends
on the qualitative assessment of the charges and evidence and not on the quantitative aspect of
complaints or offenses. In considering the side of the impeachable officers, the Constitution does not
promise an absolutely smooth ride for them, especially if the charges entail genuine and grave
issues. The framers of the Constitution did not concern themselves with the media tolerance level or
internal disposition of an impeachable officer when they deliberated on the impairment of
performance of official functions. The measure of protection afforded by the Constitution is that if the
impeachable officer is made to undergo such ride, he or she should be made to traverse it just once.
Similarly, if Congress is called upon to operate itself as a vehicle, it should do so just once. There is
no repeat ride for one full year. This is the whole import of the constitutional safeguard of one-year
bar rule.
Applicability of the Rules on Criminal Procedure

On another plane, petitioner posits that public respondent gravely abused its discretion when it
disregarded its own Impeachment Rules, the same rules she earlier chastised.
In the exercise of the power to promulgate rules "to effectively carry out" the provisions of Section 3,
Article XI of the Constitution, the House promulgated the Impeachment Rules, Section 16 of which
provides that "the Rules ofCriminal Procedure under the Rules of Court shall, as far as practicable,
apply to impeachment proceedings before the House."
Finding that the Constitution, by express grant, permits the application of additional adjective rules
that Congress may consider in effectively carrying out its mandate, petitioner either asserts or rejects
two procedural devices.
First is on the "one offense, one complaint" rule. By way of reference to Section 16 of the
Impeachment Rules, petitioner invokes the application of Section 13, Rule 110 of the Rules on
Criminal Procedure which states that "[a] complaint or information must charge only one offense,
except when the law prescribes a single punishment for various offenses." To petitioner, the two
impeachment complaints are insufficient in form and substance since each charges her with both
culpable violation of the Constitution and betrayal of public trust. She concludes that public
respondent gravely abused its discretion when it disregarded its own rules.
Petitioner adds that heaping two or more charges in one complaint will confuse her in preparing her
defense; expose her to the grave dangers of the highly political nature of the impeachment process;
constitute a whimsical disregard of certain rules; impair her performance of official functions as well
as that of the House; and prevent public respondent from completing its report within the deadline.
Public respondent counters that there is no requirement in the Constitution that an impeachment
complaint must charge only one offense, and the nature of impeachable offenses precludes the
application of the above-said Rule on Criminal Procedure since the broad terms cannot be defined
with the same precision required in defining crimes. It adds that the determination of the grounds for
impeachment is an exercise of political judgment, which issue respondent-intervenor also considers
as non-justiciable, and to which the Baraquel group adds that impeachment is a political process and
not a criminal prosecution, during which criminal prosecution stage the complaint or information
referred thereto and cited by petitioner, unlike an impeachment complaint, must already be in the
name of the People of the Philippines.
The Baraquel group deems that there are provisions92 outside the Rules on Criminal Procedure that
are more relevant to the issue. Both the Baraquel and Reyes groups point out that even if Sec. 13 of
Rule 110 is made to apply, petitioners case falls under the exception since impeachment prescribes
a single punishment removal from office and disqualification to hold any public office even for
various offenses. Both groups also observe that petitioner concededly and admittedly was not keen
on pursuing this issue during the oral arguments.
Petitioners claim deserves scant consideration.
Without going into the effectiveness of the suppletory application of the Rules on Criminal
Procedure in carrying out the relevant constitutional provisions, which prerogative the Constitution
vests on Congress, and without delving into the practicability of the application of the one offense
per complaint rule, the initial determination of which must be made by the House 93 which has yet to
pass upon the question, the Court finds that petitioners invocation of that particular rule of Criminal
Procedure does not lie. Suffice it to state that the Constitution allows the indictment for multiple
impeachment offenses, with each charge representing an article of impeachment, assembled in one

set known as the "Articles of Impeachment."94 It, therefore, follows that an impeachment complaint
need not allege only one impeachable offense.
The second procedural matter deals with the rule on consolidation. In rejecting a consolidation,
petitioner maintains that the Constitution allows only one impeachment complaint against her within
one year.
Records show that public respondent disavowed any immediate need to consolidate. Its chairperson
Rep. Tupas stated that "[c]onsolidation depends on the Committee whether to consolidate[;
c]onsolidation may come today or may come later on after determination of the sufficiency in form
and substance," and that "for purposes of consolidation, the Committee will decide when is the time
to consolidate[, a]nd if, indeed, we need to consolidate." 95 Petitioners petition, in fact, initially
describes the consolidation as merely "contemplated."96
Since public respondent, whether motu proprio or upon motion, did not yet order a consolidation, the
Court will not venture to make a determination on this matter, as it would be premature, conjectural
or anticipatory.97
Even if the Court assumes petitioners change of stance that the two impeachment complaints
were deemedconsolidated,98 her claim that consolidation is a legal anomaly fails. Petitioners theory
obviously springs from her "proceeding = complaint" equation which the Court already brushed
aside.
WHEREFORE, the petition is DISMISSED. The assailed Resolutions of September 1, 2010 and
September 7, 2010 of public respondent, the House of Representatives Committee on Justice,
are NOT UNCONSTITUTIONAL. The Status Quo Ante Order issued by the Court on September 14,
2010 is LIFTED.
SO ORDERED.
Republic of the Philippines
SUPREME COURT
Manila
EN BANC
G.R. No. 173034

October 9, 2007

PHARMACEUTICAL AND HEALTH CARE ASSOCIATION OF THE PHILIPPINES, petitioner,


vs.
HEALTH SECRETARY FRANCISCO T. DUQUE III; HEALTH UNDER SECRETARIES DR.
ETHELYN P. NIETO, DR. MARGARITA M. GALON, ATTY. ALEXANDER A. PADILLA, & DR. JADE
F. DEL MUNDO; and ASSISTANT SECRETARIES DR. MARIO C. VILLAVERDE, DR. DAVID J.
LOZADA, AND DR. NEMESIO T. GAKO, respondents.
DECISION
AUSTRIA-MARTINEZ, J.:
The Court and all parties involved are in agreement that the best nourishment for an infant is
mother's milk. There is nothing greater than for a mother to nurture her beloved child straight from

her bosom. The ideal is, of course, for each and every Filipino child to enjoy the unequaled benefits
of breastmilk. But how should this end be attained?
Before the Court is a petition for certiorari under Rule 65 of the Rules of Court, seeking to nullify
Administrative Order (A.O.) No. 2006-0012 entitled, Revised Implementing Rules and
Regulations of Executive Order No. 51, Otherwise Known as The "Milk Code," Relevant
International Agreements, Penalizing Violations Thereof, and for Other Purposes (RIRR).
Petitioner posits that the RIRR is not valid as it contains provisions that are not constitutional and go
beyond the law it is supposed to implement.
Named as respondents are the Health Secretary, Undersecretaries, and Assistant Secretaries of the
Department of Health (DOH). For purposes of herein petition, the DOH is deemed impleaded as a
co-respondent since respondents issued the questioned RIRR in their capacity as officials of said
executive agency.1
Executive Order No. 51 (Milk Code) was issued by President Corazon Aquino on October 28, 1986
by virtue of the legislative powers granted to the president under the Freedom Constitution. One of
the preambular clauses of the Milk Code states that the law seeks to give effect to Article 112 of the
International Code of Marketing of Breastmilk Substitutes (ICMBS), a code adopted by the World
Health Assembly (WHA) in 1981. From 1982 to 2006, the WHA adopted several Resolutions to the
effect that breastfeeding should be supported, promoted and protected, hence, it should be ensured
that nutrition and health claims are not permitted for breastmilk substitutes.
In 1990, the Philippines ratified the International Convention on the Rights of the Child. Article 24 of
said instrument provides that State Parties should take appropriate measures to diminish infant and
child mortality, and ensure that all segments of society, specially parents and children, are informed
of the advantages of breastfeeding.
On May 15, 2006, the DOH issued herein assailed RIRR which was to take effect on July 7, 2006.
However, on June 28, 2006, petitioner, representing its members that are manufacturers of
breastmilk substitutes, filed the present Petition for Certiorari and Prohibition with Prayer for the
Issuance of a Temporary Restraining Order (TRO) or Writ of Preliminary Injunction.
The main issue raised in the petition is whether respondents officers of the DOH acted without or in
excess of jurisdiction, or with grave abuse of discretion amounting to lack or excess of jurisdiction,
and in violation of the provisions of the Constitution in promulgating the RIRR. 3
On August 15, 2006, the Court issued a Resolution granting a TRO enjoining respondents from
implementing the questioned RIRR.
After the Comment and Reply had been filed, the Court set the case for oral arguments on June 19,
2007. The Court issued an Advisory (Guidance for Oral Arguments) dated June 5, 2007, to wit:
The Court hereby sets the following issues:
1. Whether or not petitioner is a real party-in-interest;
2. Whether Administrative Order No. 2006-0012 or the Revised Implementing Rules and
Regulations (RIRR) issued by the Department of Health (DOH) is not constitutional;

2.1 Whether the RIRR is in accord with the provisions of Executive Order No. 51 (Milk Code);
2.2 Whether pertinent international agreements1 entered into by the Philippines are part of
the law of the land and may be implemented by the DOH through the RIRR; If in the
affirmative, whether the RIRR is in accord with the international agreements;
2.3 Whether Sections 4, 5(w), 22, 32, 47, and 52 of the RIRR violate the due process clause
and are in restraint of trade; and
2.4 Whether Section 13 of the RIRR on Total Effect provides sufficient standards.
_____________
1 (1) United Nations Convention on the Rights of the Child; (2) the WHO and Unicef "2002
Global Strategy on Infant and Young Child Feeding;" and (3) various World Health Assembly
(WHA) Resolutions.
The parties filed their respective memoranda.
The petition is partly imbued with merit.
On the issue of petitioner's standing
With regard to the issue of whether petitioner may prosecute this case as the real party-in-interest,
the Court adopts the view enunciated in Executive Secretary v. Court of Appeals, 4 to wit:
The modern view is that an association has standing to complain of injuries to its members.
This view fuses the legal identity of an association with that of its members. An association
has standing to file suit for its workers despite its lack of direct interest if its members
are affected by the action. An organization has standing to assert the concerns of its
constituents.
xxxx
x x x We note that, under its Articles of Incorporation, the respondent was organized x x x to
act as the representative of any individual, company, entity or association on matters related
to the manpower recruitment industry, and to perform other acts and activities necessary to
accomplish the purposes embodied therein. The respondent is, thus, the appropriate
party to assert the rights of its members, because it and its members are in every
practical sense identical. x x x The respondent [association] is but the medium
through which its individual members seek to make more effective the expression of
their voices and the redress of their grievances. 5 (Emphasis supplied)
which was reasserted in Purok Bagong Silang Association, Inc. v. Yuipco,6 where the Court ruled that
an association has the legal personality to represent its members because the results of the case
will affect their vital interests.7
Herein petitioner's Amended Articles of Incorporation contains a similar provision just like in
Executive Secretary, that the association is formed "to represent directly or through approved
representatives the pharmaceutical and health care industry before the Philippine Government and
any of its agencies, the medical professions and the general public."8 Thus, as an organization,

petitioner definitely has an interest in fulfilling its avowed purpose of representing members who are
part of the pharmaceutical and health care industry. Petitioner is duly authorized 9to take the
appropriate course of action to bring to the attention of government agencies and the courts any
grievance suffered by its members which are directly affected by the RIRR. Petitioner, which is
mandated by its Amended Articles of Incorporation to represent the entire industry, would be remiss
in its duties if it fails to act on governmental action that would affect any of its industry members, no
matter how few or numerous they are. Hence, petitioner, whose legal identity is deemed fused with
its members, should be considered as a real party-in-interest which stands to be benefited or injured
by any judgment in the present action.
On the constitutionality of the provisions of the RIRR
First, the Court will determine if pertinent international instruments adverted to by respondents are
part of the law of the land.
Petitioner assails the RIRR for allegedly going beyond the provisions of the Milk Code, thereby
amending and expanding the coverage of said law. The defense of the DOH is that the RIRR
implements not only the Milk Code but also various international instruments 10 regarding infant and
young child nutrition. It is respondents' position that said international instruments are deemed part
of the law of the land and therefore the DOH may implement them through the RIRR.
The Court notes that the following international instruments invoked by respondents, namely: (1) The
United Nations Convention on the Rights of the Child; (2) The International Covenant on Economic,
Social and Cultural Rights; and (3) the Convention on the Elimination of All Forms of Discrimination
Against Women, only provide in general terms that steps must be taken by State Parties to diminish
infant and child mortality and inform society of the advantages of breastfeeding, ensure the health
and well-being of families, and ensure that women are provided with services and nutrition in
connection with pregnancy and lactation. Said instruments do not contain specific provisions
regarding the use or marketing of breastmilk substitutes.
The international instruments that do have specific provisions regarding breastmilk substitutes are
the ICMBS and various WHA Resolutions.
Under the 1987 Constitution, international law can become part of the sphere of domestic law either
bytransformation or incorporation.11 The transformation method requires that an international law
be transformed into a domestic law through a constitutional mechanism such as local legislation.
The incorporation method applies when, by mere constitutional declaration, international law is
deemed to have the force of domestic law.12
Treaties become part of the law of the land through transformation pursuant to Article VII, Section
21 of the Constitution which provides that "[n]o treaty or international agreement shall be valid and
effective unless concurred in by at least two-thirds of all the members of the Senate." Thus, treaties
or conventional international law must go through a process prescribed by the Constitution for it to
be transformed into municipal law that can be applied to domestic conflicts. 13
The ICMBS and WHA Resolutions are not treaties as they have not been concurred in by at least
two-thirds of all members of the Senate as required under Section 21, Article VII of the 1987
Constitution.
However, the ICMBS which was adopted by the WHA in 1981 had been transformed into domestic
law through local legislation, the Milk Code. Consequently, it is the Milk Code that has the force and
effect of law in this jurisdiction and not the ICMBS per se.

The Milk Code is almost a verbatim reproduction of the ICMBS, but it is well to emphasize at this
point that the Code did not adopt the provision in the ICMBS absolutely prohibiting advertising or
other forms of promotion to the general public of products within the scope of the ICMBS.
Instead, the Milk Code expressly provides that advertising, promotion, or other marketing
materials may be allowed if such materials are duly authorized and approved by the InterAgency Committee (IAC).
On the other hand, Section 2, Article II of the 1987 Constitution, to wit:
SECTION 2. The Philippines renounces war as an instrument of national policy, adopts the
generally accepted principles of international law as part of the law of the land and
adheres to the policy of peace, equality, justice, freedom, cooperation and amity with all
nations. (Emphasis supplied)
embodies the incorporation method.14
In Mijares v. Ranada,15 the Court held thus:
[G]enerally accepted principles of international law, by virtue of the incorporation clause of
the Constitution, form part of the laws of the land even if they do not derive from treaty
obligations. The classical formulation in international law sees those customary rules
accepted as binding result from the combination [of] two elements: the established,
widespread, and consistent practice on the part of States; and a psychological element
known as the opinion juris sive necessitates (opinion as to law or necessity). Implicit in the
latter element is a belief that the practice in question is rendered obligatory by the existence
of a rule of law requiring it.16 (Emphasis supplied)
"Generally accepted principles of international law" refers to norms of general or customary
international law which are binding on all states, 17 i.e., renunciation of war as an instrument of
national policy, the principle of sovereign immunity,18 a person's right to life, liberty and due
process,19 and pacta sunt servanda,20 among others. The concept of "generally accepted principles
of law" has also been depicted in this wise:
Some legal scholars and judges look upon certain "general principles of law" as a primary source of
international law because they have the "character of jus rationale" and are "valid through all
kinds of human societies." (Judge Tanaka in his dissenting opinion in the 1966 South West Africa
Case, 1966 I.C.J. 296). O'Connell holds that certain priniciples are part of international law
because they are "basic to legal systems generally" and hence part of the jus gentium. These
principles, he believes, are established by a process of reasoning based on the common identity of
all legal systems. If there should be doubt or disagreement, one must look to state practice and
determine whether the municipal law principle provides a just and acceptable solution. x x
x 21 (Emphasis supplied)
Fr. Joaquin G. Bernas defines customary international law as follows:
Custom or customary international law means "a general and consistent practice of states
followed by them from a sense of legal obligation [opinio juris]." (Restatement) This
statement contains the two basic elements of custom: the material factor, that is, how
states behave, and the psychological orsubjective factor, that is, why they behave the
way they do.

xxxx
The initial factor for determining the existence of custom is the actual behavior of states. This
includes several elements: duration, consistency, and generality of the practice of states.
The required duration can be either short or long. x x x
xxxx
Duration therefore is not the most important element. More important is the consistency and
the generality of the practice. x x x
xxxx
Once the existence of state practice has been established, it becomes necessary to
determine why states behave the way they do. Do states behave the way they do
because they consider it obligatory to behave thus or do they do it only as a matter of
courtesy? Opinio juris, or the belief that a certain form of behavior is obligatory, is
what makes practice an international rule. Without it, practice is not law.22 (Underscoring
and Emphasis supplied)
Clearly, customary international law is deemed incorporated into our domestic system. 23
WHA Resolutions have not been embodied in any local legislation. Have they attained the status of
customary law and should they then be deemed incorporated as part of the law of the land?
The World Health Organization (WHO) is one of the international specialized agencies allied with the
United Nations (UN) by virtue of Article 57,24 in relation to Article 6325 of the UN Charter. Under the
1946 WHO Constitution, it is the WHA which determines the policies of the WHO,26 and has the
power to adopt regulations concerning "advertising and labeling of biological, pharmaceutical and
similar products moving in international commerce,"27 and to "make recommendations to members
with respect to any matter within the competence of the Organization."28 The legal effect of its
regulations, as opposed to recommendations, is quite different.
Regulations, along with conventions and agreements, duly adopted by the WHA bind member
states thus:
Article 19. The Health Assembly shall have authority to adopt conventions or agreements
with respect to any matter within the competence of the Organization. A two-thirds vote of the
Health Assembly shall be required for the adoption of such conventions or agreements,
which shall come into force for each Member when accepted by it in accordance with
its constitutional processes.
Article 20. Each Member undertakes that it will, within eighteen months after the adoption
by the Health Assembly of a convention or agreement, take action relative to the
acceptance of such convention or agreement. Each Member shall notify the DirectorGeneral of the action taken, and if it does not accept such convention or agreement within
the time limit, it will furnish a statement of the reasons for non-acceptance. In case of
acceptance, each Member agrees to make an annual report to the Director-General in
accordance with Chapter XIV.

Article 21. The Health Assembly shall have authority to adopt regulations concerning: (a)
sanitary and quarantine requirements and other procedures designed to prevent the
international spread of disease; (b) nomenclatures with respect to diseases, causes of death
and public health practices; (c) standards with respect to diagnostic procedures for
international use; (d) standards with respect to the safety, purity and potency of biological,
pharmaceutical and similar products moving in international commerce; (e) advertising and
labeling of biological, pharmaceutical and similar products moving in international commerce.
Article 22. Regulations adopted pursuant to Article 21 shall come into force for all
Members after due notice has been given of their adoption by the Health Assembly except
for such Members as may notify the Director-General of rejection or reservations within the
period stated in the notice. (Emphasis supplied)
On the other hand, under Article 23, recommendations of the WHA do not come into force for
members, in the same way that conventions or agreements under Article 19 and regulations under
Article 21 come into force. Article 23 of the WHO Constitution reads:
Article 23. The Health Assembly shall have authority to make recommendations to
Members with respect to any matter within the competence of the Organization. (Emphasis
supplied)
The absence of a provision in Article 23 of any mechanism by which the recommendation would
come into force for member states is conspicuous.
The former Senior Legal Officer of WHO, Sami Shubber, stated that WHA recommendations are
generally not binding, but they "carry moral and political weight, as they constitute the judgment on a
health issue of the collective membership of the highest international body in the field of
health."29 Even the ICMBS itself was adopted as a mere recommendation, as WHA Resolution No.
34.22 states:
"The Thirty-Fourth World Health Assembly x x x adopts, in the sense of Article 23 of the
Constitution, the International Code of Marketing of Breastmilk Substitutes annexed to the
present resolution." (Emphasis supplied)
The Introduction to the ICMBS also reads as follows:
In January 1981, the Executive Board of the World Health Organization at its sixty-seventh
session, considered the fourth draft of the code, endorsed it, and unanimously
recommended to the Thirty-fourth World Health Assembly the text of a resolution by which it
would adopt the code in the form of a recommendation rather than a regulation. x x x
(Emphasis supplied)
The legal value of WHA Resolutions as recommendations is summarized in Article 62 of the WHO
Constitution, to wit:
Art. 62. Each member shall report annually on the action taken with respect to
recommendations made to it by the Organization, and with respect to conventions,
agreements and regulations.
Apparently, the WHA Resolution adopting the ICMBS and subsequent WHA Resolutions urging
member states to implement the ICMBS are merely recommendatory and legally non-binding. Thus,

unlike what has been done with the ICMBS whereby the legislature enacted most of the
provisions into law which is the Milk Code, the subsequent WHA Resolutions,30 specifically
providing for exclusive breastfeeding from 0-6 months, continued breastfeeding up to 24
months, and absolutely prohibiting advertisements and promotions of breastmilk substitutes,
have not been adopted as a domestic law.
It is propounded that WHA Resolutions may constitute "soft law" or non-binding norms, principles
and practices that influence state behavior.31
"Soft law" does not fall into any of the categories of international law set forth in Article 38, Chapter
III of the 1946 Statute of the International Court of Justice. 32 It is, however, an expression of nonbinding norms, principles, and practices that influence state behavior.33 Certain declarations and
resolutions of the UN General Assembly fall under this category.34 The most notable is the UN
Declaration of Human Rights, which this Court has enforced in various cases,
specifically, Government of Hongkong Special Administrative Region v. Olalia,35 Mejoff v. Director of
Prisons,36 Mijares v. Raada37 and Shangri-la International Hotel Management, Ltd. v. Developers
Group of Companies, Inc..38
The World Intellectual Property Organization (WIPO), a specialized agency attached to the UN with
the mandate to promote and protect intellectual property worldwide, has resorted to soft law as a
rapid means of norm creation, in order "to reflect and respond to the changing needs and demands
of its constituents."39 Other international organizations which have resorted to soft law include the
International Labor Organization and the Food and Agriculture Organization (in the form of
the Codex Alimentarius).40
WHO has resorted to soft law. This was most evident at the time of the Severe Acute Respiratory
Syndrome (SARS) and Avian flu outbreaks.
Although the IHR Resolution does not create new international law binding on WHO
member states, it provides an excellent example of the power of "soft law" in
international relations. International lawyers typically distinguish binding rules of
international law-"hard law"-from non-binding norms, principles, and practices that
influence state behavior-"soft law." WHO has during its existence generated many
soft law norms, creating a "soft law regime" in international governance for public
health.
The "soft law" SARS and IHR Resolutions represent significant steps in laying the political
groundwork for improved international cooperation on infectious diseases. These resolutions
clearly define WHO member states' normative duty to cooperate fully with other countries
and with WHO in connection with infectious disease surveillance and response to outbreaks.
This duty is neither binding nor enforceable, but, in the wake of the SARS epidemic,
the duty is powerful politically for two reasons. First, the SARS outbreak has taught the
lesson that participating in, and enhancing, international cooperation on infectious disease
controls is in a country's self-interest x x x if this warning is heeded, the "soft law" in the
SARS and IHR Resolution could inform the development of general and consistent state
practice on infectious disease surveillance and outbreak response, perhaps crystallizing
eventually into customary international law on infectious disease prevention and control. 41
In the Philippines, the executive department implemented certain measures recommended by WHO
to address the outbreaks of SARS and Avian flu by issuing Executive Order (E.O.) No. 201 on April
26, 2003 and E.O. No. 280 on February 2, 2004, delegating to various departments broad powers to

close down schools/establishments, conduct health surveillance and monitoring, and ban importation
of poultry and agricultural products.
It must be emphasized that even under such an international emergency, the duty of a state to
implement the IHR Resolution was still considered not binding or enforceable, although said
resolutions had great political influence.
As previously discussed, for an international rule to be considered as customary law, it must be
established that such rule is being followed by states because they consider it obligatory to comply
with such rules (opinio juris). Respondents have not presented any evidence to prove that the WHA
Resolutions, although signed by most of the member states, were in fact enforced or practiced by at
least a majority of the member states; neither have respondents proven that any compliance by
member states with said WHA Resolutions was obligatory in nature.
Respondents failed to establish that the provisions of pertinent WHA Resolutions are customary
international law that may be deemed part of the law of the land.
Consequently, legislation is necessary to transform the provisions of the WHA Resolutions into
domestic law. The provisions of the WHA Resolutions cannot be considered as part of the law
of the land that can be implemented by executive agencies without the need of a law enacted
by the legislature.
Second, the Court will determine whether the DOH may implement the provisions of the WHA
Resolutions by virtue of its powers and functions under the Revised Administrative Code even in the
absence of a domestic law.
Section 3, Chapter 1, Title IX of the Revised Administrative Code of 1987 provides that the DOH
shall define the national health policy and implement a national health plan within the framework
of the government's general policies and plans, and issue orders and regulations concerning the
implementation of established health policies.
It is crucial to ascertain whether the absolute prohibition on advertising and other forms of promotion
of breastmilk substitutes provided in some WHA Resolutions has been adopted as part of the
national health policy.
Respondents submit that the national policy on infant and young child feeding is embodied in A.O.
No. 2005-0014, dated May 23, 2005. Basically, the Administrative Order declared the following policy
guidelines: (1) ideal breastfeeding practices, such as early initiation of breastfeeding, exclusive
breastfeeding for the first six months, extended breastfeeding up to two years and beyond; (2)
appropriate complementary feeding, which is to start at age six months; (3) micronutrient
supplementation; (4) universal salt iodization; (5) the exercise of other feeding options; and (6)
feeding in exceptionally difficult circumstances. Indeed, the primacy of breastfeeding for children is
emphasized as a national health policy. However, nowhere in A.O. No. 2005-0014 is it declared
that as part of such health policy, the advertisement or promotion of breastmilk substitutes
should be absolutely prohibited.
The national policy of protection, promotion and support of breastfeeding cannot automatically be
equated with a total ban on advertising for breastmilk substitutes.
In view of the enactment of the Milk Code which does not contain a total ban on the advertising and
promotion of breastmilk substitutes, but instead, specifically creates an IAC which will regulate said

advertising and promotion, it follows that a total ban policy could be implemented only pursuant to a
law amending the Milk Code passed by the constitutionally authorized branch of government, the
legislature.
Thus, only the provisions of the Milk Code, but not those of subsequent WHA Resolutions, can
be validly implemented by the DOH through the subject RIRR.
Third, the Court will now determine whether the provisions of the RIRR are in accordance with those
of the Milk Code.
In support of its claim that the RIRR is inconsistent with the Milk Code, petitioner alleges the
following:
1. The Milk Code limits its coverage to children 0-12 months old, but the RIRR extended its
coverage to "young children" or those from ages two years old and beyond:

MILK CODE

RIRR

WHEREAS, in order to ensure that safe and


Section 2. Purpose These Revised Rules
adequate nutrition for infants is provided, there and Regulations are hereby promulgated to
is a need to protect and promote breastfeeding ensure the provision of safe and adequate
and to inform the public about the proper use of nutrition for infants and young children by the
breastmilk substitutes and supplements and
promotion, protection and support of
related products through adequate, consistent breastfeeding and by ensuring the proper use
and objective information and appropriate
of breastmilk substitutes, breastmilk
regulation of the marketing and distribution of supplements and related products when these
the said substitutes, supplements and related are medically indicated and only when
products;
necessary, on the basis of adequate
information and through appropriate marketing
SECTION 4(e). "Infant" means a person falling and distribution.
within the age bracket of 0-12 months.
Section 5(ff). "Young Child" means a person
from the age of more than twelve (12) months
up to the age of three (3) years (36 months).

2. The Milk Code recognizes that infant formula may be a proper and possible substitute for
breastmilk in certain instances; but the RIRR provides "exclusive breastfeeding for infants
from 0-6 months" and declares that "there is no substitute nor replacement for breastmilk":

MILK CODE

RIRR

WHEREAS, in order to ensure that safe and


Section 4. Declaration of Principles The
adequate nutrition for infants is provided, there following are the underlying principles from
is a need to protect and promote breastfeeding which the revised rules and regulations are
and to inform the public about the proper use of

breastmilk substitutes and supplements and


related products through adequate, consistent
and objective information and appropriate
regulation of the marketing and distribution of
the said substitutes, supplements and related
products;

premised upon:
a. Exclusive breastfeeding is for infants from 0
to six (6) months.
b. There is no substitute or replacement for
breastmilk.

3. The Milk Code only regulates and does not impose unreasonable requirements for
advertising and promotion; RIRR imposes an absolute ban on such activities for breastmilk
substitutes intended for infants from 0-24 months old or beyond, and forbids the use of
health and nutritional claims. Section 13 of the RIRR, which provides for a "total effect" in the
promotion of products within the scope of the Code, is vague:

MILK CODE

RIRR

SECTION 6. The General Public and


Mothers.

Section 4. Declaration of Principles The


following are the underlying principles from
which the revised rules and regulations are
premised upon:

(a) No advertising, promotion or other


marketing materials, whether written, audio or
visual, for products within the scope of this
Code shall be printed, published, distributed,
exhibited and broadcast unless such materials
are duly authorized and approved by an interagency committee created herein pursuant to
the applicable standards provided for in this
Code.

xxxx
f. Advertising, promotions, or sponsor-shipsof
infant formula, breastmilk substitutes and other
related products are prohibited.
Section 11. Prohibition No advertising,
promotions, sponsorships, or marketing
materials and activities for breastmilk
substitutes intended for infants and young
children up to twenty-four (24) months, shall be
allowed, because they tend to convey or give
subliminal messages or impressions that
undermine breastmilk and breastfeeding or
otherwise exaggerate breastmilk substitutes
and/or replacements, as well as related
products covered within the scope of this Code.
Section 13. "Total Effect" - Promotion of
products within the scope of this Code must be
objective and should not equate or make the
product appear to be as good or equal to
breastmilk or breastfeeding in the advertising
concept. It must not in any case undermine
breastmilk or breastfeeding. The "total effect"
should not directly or indirectly suggest that

buying their product would produce better


individuals, or resulting in greater love,
intelligence, ability, harmony or in any manner
bring better health to the baby or other such
exaggerated and unsubstantiated claim.
Section 15. Content of Materials. - The
following shall not be included in advertising,
promotional and marketing materials:
a. Texts, pictures, illustrations or information
which discourage or tend to undermine the
benefits or superiority of breastfeeding or which
idealize the use of breastmilk substitutes and
milk supplements. In this connection, no
pictures of babies and children together with
their mothers, fathers, siblings, grandparents,
other relatives or caregivers (or yayas) shall be
used in any advertisements for infant formula
and breastmilk supplements;
b. The term "humanized," "maternalized,"
"close to mother's milk" or similar words in
describing breastmilk substitutes or milk
supplements;
c. Pictures or texts that idealize the use of
infant and milk formula.
Section 16. All health and nutrition claims for
products within the scope of the Code are
absolutely prohibited. For this purpose, any
phrase or words that connotes to increase
emotional, intellectual abilities of the infant and
young child and other like phrases shall not be
allowed.

4. The RIRR imposes additional labeling requirements not found in the Milk Code:

MILK CODE

SECTION 10. Containers/Label.

RIRR

Section 26. Content Each container/label


shall contain such message, in both Filipino
(a) Containers and/or labels shall be designed and English languages, and which message
to provide the necessary information about the cannot be readily separated therefrom, relative

appropriate use of the products, and in such a the following points:


way as not to discourage breastfeeding.
(a) The words or phrase "Important Notice" or
(b) Each container shall have a clear,
"Government Warning" or their equivalent;
conspicuous and easily readable and
understandable message in Pilipino or English (b) A statement of the superiority of
printed on it, or on a label, which message can breastfeeding;
not readily become separated from it, and
which shall include the following points:
(c) A statement that there is no substitute for
(i) the words "Important Notice" or their
equivalent;
(ii) a statement of the superiority of
breastfeeding;

breastmilk;
(d) A statement that the product shall be used
only on the advice of a health worker as to the
need for its use and the proper methods of use;

(e) Instructions for appropriate prepara-tion,


(iii) a statement that the product shall be used and a warning against the health hazards of
only on the advice of a health worker as to the inappropriate preparation; and
need for its use and the proper methods of use;
and
(f) The health hazards of unnecessary or
(iv) instructions for appropriate preparation,
and a warning against the health hazards of
inappropriate preparation.

improper use of infant formula and other


related products including information that
powdered infant formula may contain
pathogenic microorganisms and must be
prepared and used appropriately.

5. The Milk Code allows dissemination of information on infant formula to health


professionals; the RIRR totally prohibits such activity:

MILK CODE

RIRR

SECTION 7. Health Care System.

Section 22. No manufacturer, distributor, or


representatives of products covered by the
Code shall be allowed to conduct or be
involved in any activity on breastfeeding
promotion, education and production of
Information, Education and Communication
(IEC) materials on breastfeeding, holding of or
participating as speakers in classes or
seminars for women and children activities and
to avoid the use of these venues to market
their brands or company names.

(b) No facility of the health care system shall


be used for the purpose of promoting infant
formula or other products within the scope of
this Code. This Code does not, however,
preclude the dissemination of information to
health professionals as provided in Section
8(b).
SECTION 8. Health Workers. -

(b) Information provided by manufacturers and SECTION 16. All health and nutrition claims for
distributors to health professionals regarding products within the scope of the Code are
products within the scope of this Code shall be absolutely prohibited. For this purpose, any

restricted to scientific and factual matters and


such information shall not imply or create a
belief that bottle-feeding is equivalent or
superior to breastfeeding. It shall also include
the information specified in Section 5(b).

phrase or words that connotes to increase


emotional, intellectual abilities of the infant and
young child and other like phrases shall not be
allowed.

6. The Milk Code permits milk manufacturers and distributors to extend assistance in
research and continuing education of health professionals; RIRR absolutely forbids the
same.

MILK CODE

RIRR

SECTION 8. Health Workers

Section 4. Declaration of Principles

(e) Manufacturers and distributors of products


within the scope of this Code may assist in the
research, scholarships and continuing
education, of health professionals,in
accordance with the rules and regulations
promulgated by the Ministry of Health.

The following are the underlying principles from


which the revised rules and regulations are
premised upon:
i. Milk companies, and their
representatives,should not form part of any
policymaking body or entity in relation to the
advancement of breasfeeding.
SECTION 22. No manufacturer, distributor, or
representatives of products covered by the
Code shall be allowed to conduct or be
involved in any activity on breastfeeding
promotion, education and production of
Information, Education and Communication
(IEC) materials on breastfeeding, holding of or
participating as speakers in classes or
seminars for women and children activitiesand
to avoid the use of these venues to market
their brands or company names.
SECTION 32. Primary Responsibility of
Health Workers - It is the primary
responsibility of the health workers to promote,
protect and support breastfeeding and
appropriate infant and young child feeding.
Part of this responsibility is to continuously
update their knowledge and skills on
breastfeeding. No assistance, support, logistics
or training from milk companies shall be
permitted.

7. The Milk Code regulates the giving of donations; RIRR absolutely prohibits it.

MILK CODE

RIRR

SECTION 6. The General Public and


Mothers.

Section 51. Donations Within the Scope of


This Code - Donations of products, materials,
defined and covered under the Milk Code and
these implementing rules and regulations, shall
(f) Nothing herein contained shall prevent
donations from manufacturers and distributors be strictly prohibited.
of products within the scope of this Code upon
request by or with the approval of the Ministry Section 52. Other Donations By Milk
of Health.
Companies Not Covered by this Code. Donations of products, equipments, and the
like, not otherwise falling within the scope of
this Code or these Rules, given by milk
companies and their agents, representatives,
whether in kind or in cash, may only be
coursed through the Inter Agency Committee
(IAC), which shall determine whether such
donation be accepted or otherwise.

8. The RIRR provides for administrative sanctions not imposed by the Milk Code.

MILK CODE

RIRR

Section 46. Administrative Sanctions. The


following administrative sanctions shall be
imposed upon any person, juridical or natural,
found to have violated the provisions of the
Code and its implementing Rules and
Regulations:
a) 1st violation Warning;
b) 2nd violation Administrative fine of a
minimum of Ten Thousand (P10,000.00) to
Fifty Thousand (P50,000.00) Pesos, depending
on the gravity and extent of the violation,
including the recall of the offending product;
c) 3rd violation Administrative Fine of a
minimum of Sixty Thousand (P60,000.00) to
One Hundred Fifty Thousand (P150,000.00)
Pesos, depending on the gravity and extent of
the violation, and in addition thereto, the recall

of the offending product, and suspension of the


Certificate of Product Registration (CPR);
d) 4th violation Administrative Fine of a
minimum of Two Hundred Thousand
(P200,000.00) to Five Hundred (P500,000.00)
Thousand Pesos, depending on the gravity and
extent of the violation; and in addition thereto,
the recall of the product, revocation of the
CPR, suspension of the License to Operate
(LTO) for one year;
e) 5th and succeeding repeated violations
Administrative Fine of One Million
(P1,000,000.00) Pesos, the recall of the
offending product, cancellation of the CPR,
revocation of the License to Operate (LTO) of
the company concerned, including the
blacklisting of the company to be furnished the
Department of Budget and Management
(DBM) and the Department of Trade and
Industry (DTI);
f) An additional penalty of Two Thou-sand Five
Hundred (P2,500.00) Pesos per day shall be
made for every day the violation continues
after having received the order from the IAC or
other such appropriate body, notifying and
penalizing the company for the infraction.
For purposes of determining whether or not
there is "repeated" violation, each product
violation belonging or owned by a company,
including those of their subsidiaries, are
deemed to be violations of the concerned milk
company and shall not be based on the
specific violating product alone.

9. The RIRR provides for repeal of existing laws to the contrary.


The Court shall resolve the merits of the allegations of petitioner seriatim.
1. Petitioner is mistaken in its claim that the Milk Code's coverage is limited only to children 0-12
months old. Section 3 of the Milk Code states:
SECTION 3. Scope of the Code The Code applies to the marketing, and practices related
thereto, of the following products: breastmilk substitutes, including infant formula; other milk
products, foods and beverages, including bottle-fed complementary foods, when marketed or

otherwise represented to be suitable, with or without modification, for use as a partial or total
replacement of breastmilk; feeding bottles and teats. It also applies to their quality and
availability, and to information concerning their use.
Clearly, the coverage of the Milk Code is not dependent on the age of the child but on the kind of
product being marketed to the public. The law treats infant formula, bottle-fed complementary food,
and breastmilk substitute as separate and distinct product categories.
Section 4(h) of the Milk Code defines infant formula as "a breastmilk substitute x x x to satisfy the
normal nutritional requirements of infants up to between four to six months of age, and adapted to
their physiological characteristics"; while under Section 4(b), bottle-fed complementary food refers to
"any food, whether manufactured or locally prepared, suitable as a complement to breastmilk or
infant formula, when either becomes insufficient to satisfy the nutritional requirements of the infant."
An infant under Section 4(e) is a person falling within the age bracket 0-12 months. It is the
nourishment of this group of infants or children aged 0-12 months that is sought to be promoted and
protected by the Milk Code.
But there is another target group. Breastmilk substitute is defined under Section 4(a) as "any food
being marketed or otherwise presented as a partial or total replacement for breastmilk, whether or
not suitable for that purpose."This section conspicuously lacks reference to any particular agegroup of children. Hence, the provision of the Milk Code cannot be considered exclusive for
children aged 0-12 months. In other words, breastmilk substitutes may also be intended for young
children more than 12 months of age. Therefore, by regulating breastmilk substitutes, the Milk Code
also intends to protect and promote the nourishment of children more than 12 months old.
Evidently, as long as what is being marketed falls within the scope of the Milk Code as provided in
Section 3, then it can be subject to regulation pursuant to said law, even if the product is to be used
by children aged over 12 months.
There is, therefore, nothing objectionable with Sections 242 and 5(ff)43 of the RIRR.
2. It is also incorrect for petitioner to say that the RIRR, unlike the Milk Code, does not recognize that
breastmilk substitutes may be a proper and possible substitute for breastmilk.
The entirety of the RIRR, not merely truncated portions thereof, must be considered and construed
together. As held in De Luna v. Pascual,44 "[t]he particular words, clauses and phrases in the Rule
should not be studied as detached and isolated expressions, but the whole and every part thereof
must be considered in fixing the meaning of any of its parts and in order to produce a harmonious
whole."
Section 7 of the RIRR provides that "when medically indicated and only when necessary, the use of
breastmilk substitutes is proper if based on complete and updated information." Section 8 of the
RIRR also states that information and educational materials should include information on the proper
use of infant formula when the use thereof is needed.
Hence, the RIRR, just like the Milk Code, also recognizes that in certain cases, the use of
breastmilk substitutes may be proper.
3. The Court shall ascertain the merits of allegations 345 and 446 together as they are interlinked with
each other.

To resolve the question of whether the labeling requirements and advertising regulations under the
RIRR are valid, it is important to deal first with the nature, purpose, and depth of the regulatory
powers of the DOH, as defined in general under the 1987 Administrative Code, 47 and as delegated in
particular under the Milk Code.
Health is a legitimate subject matter for regulation by the DOH (and certain other administrative
agencies) in exercise of police powers delegated to it. The sheer span of jurisprudence on that
matter precludes the need to further discuss it..48 However, health information, particularly advertising
materials on apparently non-toxic products like breastmilk substitutes and supplements, is a
relatively new area for regulation by the DOH.49
As early as the 1917 Revised Administrative Code of the Philippine Islands, 50 health information was
already within the ambit of the regulatory powers of the predecessor of DOH. 51 Section 938 thereof
charged it with the duty to protect the health of the people, and vested it with such powers as "(g) the
dissemination of hygienic information among the people and especially the inculcation of
knowledge as to the proper care of infantsand the methods of preventing and combating
dangerous communicable diseases."
Seventy years later, the 1987 Administrative Code tasked respondent DOH to carry out the state
policy pronounced under Section 15, Article II of the 1987 Constitution, which is "to protect and
promote the right to health of the people and instill health consciousness among them."52 To that
end, it was granted under Section 3 of the Administrative Code the power to "(6) propagate health
information and educate the populationon important health, medical and environmental matters
which have health implications."53
When it comes to information regarding nutrition of infants and young children, however, the Milk
Code specifically delegated to the Ministry of Health (hereinafter referred to as DOH) the power to
ensure that there is adequate, consistent and objective information on breastfeeding and use of
breastmilk substitutes, supplements and related products; and the power to control such
information. These are expressly provided for in Sections 12 and 5(a), to wit:
SECTION 12. Implementation and Monitoring
xxxx
(b) The Ministry of Health shall be principally responsible for the implementation and
enforcement of the provisions of this Code. For this purpose, the Ministry of Health shall
have the following powers and functions:
(1) To promulgate such rules and regulations as are necessary or proper for the
implementation of this Code and the accomplishment of its purposes and objectives.
xxxx
(4) To exercise such other powers and functions as may be necessary for or
incidental to the attainment of the purposes and objectives of this Code.
SECTION 5. Information and Education
(a) The government shall ensure that objective and consistent information is provided on
infant feeding, for use by families and those involved in the field of infant nutrition. This

responsibility shall cover the planning, provision, design and dissemination of information,
and the control thereof, on infant nutrition. (Emphasis supplied)
Further, DOH is authorized by the Milk Code to control the content of any information on
breastmilk vis--visbreastmilk substitutes, supplement and related products, in the following manner:
SECTION 5. x x x
(b) Informational and educational materials, whether written, audio, or visual, dealing with the
feeding of infants and intended to reach pregnant women and mothers of infants, shall
include clear information on all the following points: (1) the benefits and superiority of
breastfeeding; (2) maternal nutrition, and the preparation for and maintenance of
breastfeeding; (3) the negative effect on breastfeeding of introducing partial bottlefeeding; (4)
the difficulty of reversing the decision not to breastfeed; and (5) where needed, the proper
use of infant formula, whether manufactured industrially or home-prepared. When such
materials contain information about the use of infant formula, they shall include the
social and financial implications of its use; the health hazards of inappropriate foods
or feeding methods; and, in particular, the health hazards of unnecessary or improper
use of infant formula and other breastmilk substitutes. Such materials shall not use
any picture or text which may idealize the use of breastmilk substitutes.
SECTION 8. Health Workers
xxxx
(b) Information provided by manufacturers and distributors to health professionals regarding
products within the scope of this Code shall be restricted to scientific and factual
matters, and such information shall not imply or create a belief that bottlefeeding is
equivalent or superior to breastfeeding. It shall also include the information specified
in Section 5(b).
SECTION 10. Containers/Label
(a) Containers and/or labels shall be designed to provide the necessary information about
the appropriate use of the products, and in such a way as not to discourage
breastfeeding.
xxxx
(d) The term "humanized," "maternalized" or similar terms shall not be used. (Emphasis
supplied)
The DOH is also authorized to control the purpose of the information and to whom such information
may be disseminated under Sections 6 through 9 of the Milk Code54 to ensure that the information
that would reach pregnant women, mothers of infants, and health professionals and workers in the
health care system is restricted to scientific and factual matters and shall not imply or create a belief
that bottlefeeding is equivalent or superior to breastfeeding.
It bears emphasis, however, that the DOH's power under the Milk Code to control information
regarding breastmilk vis-a-vis breastmilk substitutes is not absolute as the power to control does

not encompass the power to absolutely prohibit the advertising, marketing, and promotion of
breastmilk substitutes.
The following are the provisions of the Milk Code that unequivocally indicate that the control over
information given to the DOH is not absolute and that absolute prohibition is not contemplated by the
Code:
a) Section 2 which requires adequate information and appropriate marketing and distribution
of breastmilk substitutes, to wit:
SECTION 2. Aim of the Code The aim of the Code is to contribute to the provision
of safe and adequate nutrition for infants by the protection and promotion of
breastfeeding and by ensuring the proper use of breastmilk substitutes and
breastmilk supplements when these are necessary, on the basis of adequate
information and through appropriate marketing and distribution.
b) Section 3 which specifically states that the Code applies to the marketing of and practices
related to breastmilk substitutes, including infant formula, and to information concerning their
use;
c) Section 5(a) which provides that the government shall ensure that objective and
consistent information is provided on infant feeding;
d) Section 5(b) which provides that written, audio or visual informational and educational
materials shall not use any picture or text which may idealize the use of breastmilk
substitutes and should include information on the health hazards of unnecessary or improper
use of said product;
e) Section 6(a) in relation to Section 12(a) which creates and empowers the IAC to review
and examine advertising, promotion, and other marketing materials;
f) Section 8(b) which states that milk companies may provide information to health
professionals but such information should be restricted to factual and scientific matters and
shall not imply or create a belief that bottlefeeding is equivalent or superior to breastfeeding;
and
g) Section 10 which provides that containers or labels should not contain information that
would discourage breastfeeding and idealize the use of infant formula.
It is in this context that the Court now examines the assailed provisions of the RIRR regarding
labeling and advertising.
Sections 1355 on "total effect" and 2656 of Rule VII of the RIRR contain some labeling requirements,
specifically: a) that there be a statement that there is no substitute to breastmilk; and b) that there be
a statement that powdered infant formula may contain pathogenic microorganisms and must be
prepared and used appropriately. Section 1657 of the RIRR prohibits all health and nutrition claims for
products within the scope of the Milk Code, such as claims of increased emotional and intellectual
abilities of the infant and young child.
These requirements and limitations are consistent with the provisions of Section 8 of the Milk Code,
to wit:

SECTION 8. Health workers xxxx


(b) Information provided by manufacturers and distributors to health professionals regarding
products within the scope of this Code shall be restricted to scientific and factual matters,
and such informationshall not imply or create a belief that bottlefeeding
is equivalent or superior to breastfeeding. It shall also include the information specified in
Section 5.58 (Emphasis supplied)
and Section 10(d)59 which bars the use on containers and labels of the terms "humanized,"
"maternalized," or similar terms.
These provisions of the Milk Code expressly forbid information that would imply or create a belief
that there is any milk product equivalent to breastmilk or which is humanized or maternalized, as
such information would be inconsistent with the superiority of breastfeeding.
It may be argued that Section 8 of the Milk Code refers only to information given to health workers
regarding breastmilk substitutes, not to containers and labels thereof. However, such restrictive
application of Section 8(b) will result in the absurd situation in which milk companies and distributors
are forbidden to claim to health workers that their products are substitutes or equivalents of
breastmilk, and yet be allowed to display on the containers and labels of their products the exact
opposite message. That askewed interpretation of the Milk Code is precisely what Section 5(a)
thereof seeks to avoid by mandating that all information regarding breastmilk vis-a-visbreastmilk
substitutes be consistent, at the same time giving the government control over planning, provision,
design, and dissemination of information on infant feeding.
Thus, Section 26(c) of the RIRR which requires containers and labels to state that the product
offered is not a substitute for breastmilk, is a reasonable means of enforcing Section 8(b) of the Milk
Code and deterring circumvention of the protection and promotion of breastfeeding as embodied in
Section 260 of the Milk Code.
Section 26(f)61 of the RIRR is an equally reasonable labeling requirement. It implements Section 5(b)
of the Milk Code which reads:
SECTION 5. x x x
xxxx
(b) Informational and educational materials, whether written, audio, or visual, dealing with the
feeding of infants and intended to reach pregnant women and mothers of infants, shall
include clear information on all the following points: x x x (5) where needed, the proper use of
infant formula, whether manufactured industrially or home-prepared. When such materials
contain information about the use of infant formula, they shall include the social and financial
implications of its use; the health hazards of inappropriate foods or feeding methods;
and, in particular, the health hazards of unnecessary or improper use of infant formula
and other breastmilk substitutes. Such materials shall not use any picture or text which
may idealize the use of breastmilk substitutes. (Emphasis supplied)
The label of a product contains information about said product intended for the buyers thereof. The
buyers of breastmilk substitutes are mothers of infants, and Section 26 of the RIRR merely adds a

fair warning about the likelihood of pathogenic microorganisms being present in infant formula and
other related products when these are prepared and used inappropriately.
Petitioners counsel has admitted during the hearing on June 19, 2007 that formula milk is prone to
contaminations and there is as yet no technology that allows production of powdered infant formula
that eliminates all forms of contamination.62
Ineluctably, the requirement under Section 26(f) of the RIRR for the label to contain the message
regarding health hazards including the possibility of contamination with pathogenic microorganisms
is in accordance with Section 5(b) of the Milk Code.
The authority of DOH to control information regarding breastmilk vis-a-vis breastmilk substitutes and
supplements and related products cannot be questioned. It is its intervention into the area of
advertising, promotion, and marketing that is being assailed by petitioner.
In furtherance of Section 6(a) of the Milk Code, to wit:
SECTION 6. The General Public and Mothers.
(a) No advertising, promotion or other marketing materials, whether written, audio or visual,
for products within the scope of this Code shall be printed, published, distributed, exhibited
and broadcast unless such materials are duly authorized and approved by an inter-agency
committee created herein pursuant to the applicable standards provided for in this Code.
the Milk Code invested regulatory authority over advertising, promotional and marketing materials to
an IAC, thus:
SECTION 12. Implementation and Monitoring (a) For purposes of Section 6(a) of this Code, an inter-agency committee composed of the
following members is hereby created:

Minister of Health

-------------------

Chairman

Minister of Trade and Industry

-------------------

Member

Minister of Justice

-------------------

Member

Minister of Social Services and Development

-------------------

Member

The members may designate their duly authorized representative to every meeting of the
Committee.

The Committee shall have the following powers and functions:


(1) To review and examine all advertising. promotion or other marketing materials,
whether written, audio or visual, on products within the scope of this Code;
(2) To approve or disapprove, delete objectionable portions from and prohibit the
printing, publication, distribution, exhibition and broadcast of, all advertising
promotion or other marketing materials, whether written, audio or visual, on products
within the scope of this Code;
(3) To prescribe the internal and operational procedure for the exercise of its powers
and functions as well as the performance of its duties and responsibilities; and
(4) To promulgate such rules and regulations as are necessary or proper for
the implementation of Section 6(a) of this Code. x x x (Emphasis supplied)
However, Section 11 of the RIRR, to wit:
SECTION 11. Prohibition No advertising, promotions, sponsorships, or marketing materials
and activities for breastmilk substitutes intended for infants and young children up to twentyfour (24) months, shall be allowed, because they tend to convey or give subliminal
messages or impressions that undermine breastmilk and breastfeeding or otherwise
exaggerate breastmilk substitutes and/or replacements, as well as related products covered
within the scope of this Code.
prohibits advertising, promotions, sponsorships or marketing materials and activities for breastmilk
substitutes in line with the RIRRs declaration of principle under Section 4(f), to wit:
SECTION 4. Declaration of Principles
xxxx
(f) Advertising, promotions, or sponsorships of infant formula, breastmilk substitutes and
other related products are prohibited.
The DOH, through its co-respondents, evidently arrogated to itself not only the regulatory authority
given to the IAC but also imposed absolute prohibition on advertising, promotion, and marketing.
Yet, oddly enough, Section 12 of the RIRR reiterated the requirement of the Milk Code in Section 6
thereof for prior approval by IAC of all advertising, marketing and promotional materials prior to
dissemination.
Even respondents, through the OSG, acknowledged the authority of IAC, and repeatedly insisted,
during the oral arguments on June 19, 2007, that the prohibition under Section 11 is not actually
operational, viz:
SOLICITOR GENERAL DEVANADERA:
xxxx

x x x Now, the crux of the matter that is being questioned by Petitioner is whether or not
there is an absolute prohibition on advertising making AO 2006-12 unconstitutional. We
maintained that what AO 2006-12 provides is not an absolute prohibition because Section 11
while it states and it is entitled prohibition it states that no advertising, promotion,
sponsorship or marketing materials and activities for breast milk substitutes intended for
infants and young children up to 24 months shall be allowed because this is the standard
they tend to convey or give subliminal messages or impression undermine that breastmilk or
breastfeeding x x x.
We have to read Section 11 together with the other Sections because the other Section,
Section 12, provides for the inter agency committee that is empowered to process and
evaluate all the advertising and promotion materials.
xxxx
What AO 2006-12, what it does, it does not prohibit the sale and manufacture, it simply
regulates the advertisement and the promotions of breastfeeding milk substitutes.
xxxx
Now, the prohibition on advertising, Your Honor, must be taken together with the provision on
the Inter-Agency Committee that processes and evaluates because there may be some
information dissemination that are straight forward information dissemination. What the AO
2006 is trying to prevent is any material that will undermine the practice of breastfeeding,
Your Honor.
xxxx
ASSOCIATE JUSTICE SANTIAGO:
Madam Solicitor General, under the Milk Code, which body has authority or power to
promulgate Rules and Regulations regarding the Advertising, Promotion and Marketing of
Breastmilk Substitutes?
SOLICITOR GENERAL DEVANADERA:
Your Honor, please, it is provided that the Inter-Agency Committee, Your Honor.
xxxx
ASSOCIATE JUSTICE SANTIAGO:
x x x Don't you think that the Department of Health overstepped its rule making authority
when it totally banned advertising and promotion under Section 11 prescribed the total effect
rule as well as the content of materials under Section 13 and 15 of the rules and regulations?
SOLICITOR GENERAL DEVANADERA:
Your Honor, please, first we would like to stress that there is no total absolute ban. Second,
the Inter-Agency Committee is under the Department of Health, Your Honor.

xxxx
ASSOCIATE JUSTICE NAZARIO:
x x x Did I hear you correctly, Madam Solicitor, that there is no absolute ban on advertising of
breastmilk substitutes in the Revised Rules?
SOLICITOR GENERAL DEVANADERA:
Yes, your Honor.
ASSOCIATE JUSTICE NAZARIO:
But, would you nevertheless agree that there is an absolute ban on advertising of breastmilk
substitutes intended for children two (2) years old and younger?
SOLICITOR GENERAL DEVANADERA:
It's not an absolute ban, Your Honor, because we have the Inter-Agency Committee that can
evaluate some advertising and promotional materials, subject to the standards that we have
stated earlier, which are- they should not undermine breastfeeding, Your Honor.
xxxx
x x x Section 11, while it is titled Prohibition, it must be taken in relation with the other
Sections, particularly 12 and 13 and 15, Your Honor, because it is recognized that the InterAgency Committee has that power to evaluate promotional materials, Your Honor.
ASSOCIATE JUSTICE NAZARIO:
So in short, will you please clarify there's no absolute ban on advertisement regarding milk
substitute regarding infants two (2) years below?
SOLICITOR GENERAL DEVANADERA:
We can proudly say that the general rule is that there is a prohibition, however, we take
exceptions and standards have been set. One of which is that, the Inter-Agency Committee
can allow if the advertising and promotions will not undermine breastmilk and breastfeeding,
Your Honor.63
Sections 11 and 4(f) of the RIRR are clearly violative of the Milk Code.
However, although it is the IAC which is authorized to promulgate rules and regulations for the
approval or rejection of advertising, promotional, or other marketing materials under Section 12(a) of
the Milk Code, said provision must be related to Section 6 thereof which in turn provides that the
rules and regulations must be "pursuant to the applicable standards provided for in this Code." Said
standards are set forth in Sections 5(b), 8(b), and 10 of the Code, which, at the risk of being
repetitious, and for easy reference, are quoted hereunder:
SECTION 5. Information and Education

xxxx
(b) Informational and educational materials, whether written, audio, or visual, dealing with the
feeding of infants and intended to reach pregnant women and mothers of infants, shall
include clear information on all the following points: (1) the benefits and superiority of
breastfeeding; (2) maternal nutrition, and the preparation for and maintenance of
breastfeeding; (3) the negative effect on breastfeeding of introducing partial bottlefeeding; (4)
the difficulty of reversing the decision not to breastfeed; and (5) where needed, the proper
use of infant formula, whether manufactured industrially or home-prepared. When such
materials contain information about the use of infant formula, they shall include the social
and financial implications of its use; the health hazards of inappropriate foods of feeding
methods; and, in particular, the health hazards of unnecessary or improper use of infant
formula and other breastmilk substitutes. Such materials shall not use any picture or text
which may idealize the use of breastmilk substitutes.
xxxx
SECTION 8. Health Workers.
xxxx
(b) Information provided by manufacturers and distributors to health professionals regarding
products within the scope of this Code shall be restricted to scientific and factual matters and
such information shall not imply or create a belief that bottle feeding is equivalent or superior
to breastfeeding. It shall also include the information specified in Section 5(b).
xxxx
SECTION 10. Containers/Label
(a) Containers and/or labels shall be designed to provide the necessary information about
the appropriate use of the products, and in such a way as not to discourage breastfeeding.
(b) Each container shall have a clear, conspicuous and easily readable and understandable
message in Pilipino or English printed on it, or on a label, which message can not readily
become separated from it, and which shall include the following points:
(i) the words "Important Notice" or their equivalent;
(ii) a statement of the superiority of breastfeeding;
(iii) a statement that the product shall be used only on the advice of a health worker
as to the need for its use and the proper methods of use; and
(iv) instructions for appropriate preparation, and a warning against the health hazards
of inappropriate preparation.
Section 12(b) of the Milk Code designates the DOH as the principal implementing agency for the
enforcement of the provisions of the Code. In relation to such responsibility of the DOH, Section 5(a)
of the Milk Code states that:

SECTION 5. Information and Education


(a) The government shall ensure that objective and consistent information is provided on
infant feeding, for use by families and those involved in the field of infant nutrition. This
responsibility shall cover the planning, provision, design and dissemination of information,
and the control thereof, on infant nutrition. (Emphasis supplied)
Thus, the DOH has the significant responsibility to translate into operational terms the
standards set forth in Sections 5, 8, and 10 of the Milk Code, by which the IAC shall screen
advertising, promotional, or other marketing materials.
It is pursuant to such responsibility that the DOH correctly provided for Section 13 in the RIRR which
reads as follows:
SECTION 13. "Total Effect" - Promotion of products within the scope of this Code must be
objective and should not equate or make the product appear to be as good or equal to
breastmilk or breastfeeding in the advertising concept. It must not in any case undermine
breastmilk or breastfeeding. The "total effect" should not directly or indirectly suggest that
buying their product would produce better individuals, or resulting in greater love,
intelligence, ability, harmony or in any manner bring better health to the baby or other such
exaggerated and unsubstantiated claim.
Such standards bind the IAC in formulating its rules and regulations on advertising, promotion, and
marketing. Through that single provision, the DOH exercises control over the information content of
advertising, promotional and marketing materials on breastmilk vis-a-vis breastmilk substitutes,
supplements and other related products. It also sets a viable standard against which the IAC may
screen such materials before they are made public.
In Equi-Asia Placement, Inc. vs. Department of Foreign Affairs,64 the Court held:
x x x [T]his Court had, in the past, accepted as sufficient standards the following: "public
interest," "justice and equity," "public convenience and welfare," and "simplicity, economy
and welfare."65
In this case, correct information as to infant feeding and nutrition is infused with public interest and
welfare.
4. With regard to activities for dissemination of information to health professionals, the Court also
finds that there is no inconsistency between the provisions of the Milk Code and the RIRR. Section
7(b)66 of the Milk Code, in relation to Section 8(b)67 of the same Code, allows dissemination of
information to health professionals but suchinformation is restricted to scientific and factual
matters.
Contrary to petitioner's claim, Section 22 of the RIRR does not prohibit the giving of information to
health professionals on scientific and factual matters. What it prohibits is the involvement of the
manufacturer and distributor of the products covered by the Code in activities for the promotion,
education and production of Information, Education and Communication (IEC) materials regarding
breastfeeding that are intended forwomen and children. Said provision cannot be construed to
encompass even the dissemination of information to health professionals, as restricted by the
Milk Code.

5. Next, petitioner alleges that Section 8(e)68 of the Milk Code permits milk manufacturers and
distributors to extend assistance in research and in the continuing education of health professionals,
while Sections 22 and 32 of the RIRR absolutely forbid the same. Petitioner also assails Section
4(i)69 of the RIRR prohibiting milk manufacturers' and distributors' participation in any policymaking
body in relation to the advancement of breastfeeding.
Section 4(i) of the RIRR provides that milk companies and their representatives should not form part
of any policymaking body or entity in relation to the advancement of breastfeeding. The Court finds
nothing in said provisions which contravenes the Milk Code. Note that under Section 12(b) of the
Milk Code, it is the DOH which shall be principally responsible for the implementation and
enforcement of the provisions of said Code. It is entirely up to the DOH to decide which entities to
call upon or allow to be part of policymaking bodies on breastfeeding. Therefore, the RIRR's
prohibition on milk companies participation in any policymaking body in relation to the advancement
of breastfeeding is in accord with the Milk Code.
Petitioner is also mistaken in arguing that Section 22 of the RIRR prohibits milk companies from
giving reasearch assistance and continuing education to health professionals. Section 2270 of the
RIRR does not pertain to research assistance to or the continuing education of health
professionals; rather, it deals with breastfeeding promotion and education for women and
children. Nothing in Section 22 of the RIRR prohibits milk companies from giving assistance for
research or continuing education to health professionals; hence, petitioner's argument against this
particular provision must be struck down.
It is Sections 971 and 1072 of the RIRR which govern research assistance. Said sections of the RIRR
provide thatresearch assistance for health workers and researchers may be allowed upon
approval of an ethics committee, and with certain disclosure requirements imposed on the
milk company and on the recipient of the research award.
The Milk Code endows the DOH with the power to determine how such research or educational
assistance may be given by milk companies or under what conditions health workers may accept the
assistance. Thus, Sections 9 and 10 of the RIRR imposing limitations on the kind of research done
or extent of assistance given by milk companies are completely in accord with the Milk Code.
Petitioner complains that Section 3273 of the RIRR prohibits milk companies from giving assistance,
support, logistics or training to health workers. This provision is within the prerogative given to the
DOH under Section 8(e)74 of the Milk Code, which provides that manufacturers and distributors of
breastmilk substitutes may assist in researches, scholarships and the continuing education, of health
professionals in accordance with the rules and regulations promulgated by the Ministry of Health,
now DOH.
6. As to the RIRR's prohibition on donations, said provisions are also consistent with the Milk Code.
Section 6(f) of the Milk Code provides that donations may be made by manufacturers and
distributors of breastmilk substitutesupon the request or with the approval of the DOH. The law
does not proscribe the refusal of donations. The Milk Code leaves it purely to the discretion of the
DOH whether to request or accept such donations. The DOH then appropriately exercised its
discretion through Section 5175 of the RIRR which sets forth its policy not to request or approve
donations from manufacturers and distributors of breastmilk substitutes.
It was within the discretion of the DOH when it provided in Section 52 of the RIRR that any donation
from milk companies not covered by the Code should be coursed through the IAC which shall
determine whether such donation should be accepted or refused. As reasoned out by respondents,
the DOH is not mandated by the Milk Code to accept donations. For that matter, no person or entity

can be forced to accept a donation. There is, therefore, no real inconsistency between the RIRR and
the law because the Milk Code does not prohibit the DOH from refusing donations.
7. With regard to Section 46 of the RIRR providing for administrative sanctions that are not found in
the Milk Code, the Court upholds petitioner's objection thereto.
Respondent's reliance on Civil Aeronautics Board v. Philippine Air Lines, Inc.76 is misplaced. The
glaring difference in said case and the present case before the Court is that, in the Civil Aeronautics
Board, the Civil Aeronautics Administration (CAA) was expressly granted by the law (R.A. No.
776) the power to impose fines and civil penalties, while the Civil Aeronautics Board (CAB) was
granted by the same law the power to review on appeal the order or decision of the CAA and to
determine whether to impose, remit, mitigate, increase or compromise such fine and civil penalties.
Thus, the Court upheld the CAB's Resolution imposing administrative fines.
In a more recent case, Perez v. LPG Refillers Association of the Philippines, Inc.,77 the Court upheld
the Department of Energy (DOE) Circular No. 2000-06-10
implementing Batas Pambansa (B.P.) Blg. 33. The circular provided for fines for the commission of
prohibited acts. The Court found that nothing in the circular contravened the law because the DOE
was expressly authorized by B.P. Blg. 33 and R.A. No. 7638 to impose fines or penalties.
In the present case, neither the Milk Code nor the Revised Administrative Code grants the DOH the
authority to fix or impose administrative fines. Thus, without any express grant of power to fix or
impose such fines, the DOH cannot provide for those fines in the RIRR. In this regard, the DOH
again exceeded its authority by providing for such fines or sanctions in Section 46 of the RIRR. Said
provision is, therefore, null and void.
The DOH is not left without any means to enforce its rules and regulations. Section 12(b) (3) of the
Milk Code authorizes the DOH to "cause the prosecution of the violators of this Code and other
pertinent laws on products covered by this Code." Section 13 of the Milk Code provides for the
penalties to be imposed on violators of the provision of the Milk Code or the rules and regulations
issued pursuant to it, to wit:
SECTION 13. Sanctions
(a) Any person who violates the provisions of this Code or the rules and regulations
issued pursuant to this Code shall, upon conviction, be punished by a penalty of two (2)
months to one (1) year imprisonment or a fine of not less than One Thousand Pesos
(P1,000.00) nor more than Thirty Thousand Pesos (P30,000.00) or both. Should the offense
be committed by a juridical person, the chairman of the Board of Directors, the president,
general manager, or the partners and/or the persons directly responsible therefor, shall be
penalized.
(b) Any license, permit or authority issued by any government agency to any health worker,
distributor, manufacturer, or marketing firm or personnel for the practice of their profession or
occupation, or for the pursuit of their business, may, upon recommendation of the Ministry of
Health, be suspended or revoked in the event of repeated violations of this Code, or of the
rules and regulations issued pursuant to this Code. (Emphasis supplied)
8. Petitioners claim that Section 57 of the RIRR repeals existing laws that are contrary to the RIRR
is frivolous.

Section 57 reads:
SECTION 57. Repealing Clause - All orders, issuances, and rules and regulations or parts
thereof inconsistent with these revised rules and implementing regulations are hereby
repealed or modified accordingly.
Section 57 of the RIRR does not provide for the repeal of laws but only orders, issuances and rules
and regulations. Thus, said provision is valid as it is within the DOH's rule-making power.
An administrative agency like respondent possesses quasi-legislative or rule-making power or the
power to make rules and regulations which results in delegated legislation that is within the confines
of the granting statute and the Constitution, and subject to the doctrine of non-delegability and
separability of powers.78 Such express grant of rule-making power necessarily includes the power to
amend, revise, alter, or repeal the same.79 This is to allow administrative agencies flexibility in
formulating and adjusting the details and manner by which they are to implement the provisions of a
law,80 in order to make it more responsive to the times. Hence, it is a standard provision in
administrative rules that prior issuances of administrative agencies that are inconsistent therewith
are declared repealed or modified.
In fine, only Sections 4(f), 11 and 46 are ultra vires, beyond the authority of the DOH to promulgate
and in contravention of the Milk Code and, therefore, null and void. The rest of the provisions of the
RIRR are in consonance with the Milk Code.
Lastly, petitioner makes a "catch-all" allegation that:
x x x [T]he questioned RIRR sought to be implemented by the Respondents is unnecessary
and oppressive, and is offensive to the due process clause of the Constitution, insofar
as the same is in restraint of trade and because a provision therein is inadequate to
provide the public with a comprehensible basis to determine whether or not they have
committed a violation.81 (Emphasis supplied)
Petitioner refers to Sections 4(f),82 4(i),83 5(w),84 11,85 22,86 32,87 46,88 and 5289 as the provisions that
suppress the trade of milk and, thus, violate the due process clause of the Constitution.
The framers of the constitution were well aware that trade must be subjected to some form of
regulation for the public good. Public interest must be upheld over business interests. 90 In Pest
Management Association of the Philippines v. Fertilizer and Pesticide Authority,91 it was held thus:
x x x Furthermore, as held in Association of Philippine Coconut Desiccators v. Philippine
Coconut Authority,despite the fact that "our present Constitution enshrines free
enterprise as a policy, it nonetheless reserves to the government the power to
intervene whenever necessary to promote the general welfare." There can be no
question that the unregulated use or proliferation of pesticides would be hazardous to our
environment. Thus, in the aforecited case, the Court declared that "free enterprise does not
call for removal of protective regulations." x x x It must be clearly explained and
proven by competent evidence just exactly how such protective regulation would
result in the restraint of trade. [Emphasis and underscoring supplied]
In this case, petitioner failed to show that the proscription of milk manufacturers participation in any
policymaking body (Section 4(i)), classes and seminars for women and children (Section 22); the
giving of assistance, support and logistics or training (Section 32); and the giving of donations

(Section 52) would unreasonably hamper the trade of breastmilk substitutes. Petitioner has not
established that the proscribed activities are indispensable to the trade of breastmilk substitutes.
Petitioner failed to demonstrate that the aforementioned provisions of the RIRR are unreasonable
and oppressive for being in restraint of trade.
Petitioner also failed to convince the Court that Section 5(w) of the RIRR is unreasonable and
oppressive. Said section provides for the definition of the term "milk company," to wit:
SECTION 5 x x x. (w) "Milk Company" shall refer to the owner, manufacturer, distributor of
infant formula, follow-up milk, milk formula, milk supplement, breastmilk substitute or
replacement, or by any other description of such nature, including their representatives who
promote or otherwise advance their commercial interests in marketing those products;
On the other hand, Section 4 of the Milk Code provides:
(d) "Distributor" means a person, corporation or any other entity in the public or private sector
engaged in the business (whether directly or indirectly) of marketing at the wholesale or retail
level a product within the scope of this Code. A "primary distributor" is a manufacturer's sales
agent, representative, national distributor or broker.
xxxx
(j) "Manufacturer" means a corporation or other entity in the public or private sector engaged
in the business or function (whether directly or indirectly or through an agent or and entity
controlled by or under contract with it) of manufacturing a products within the scope of this
Code.
Notably, the definition in the RIRR merely merged together under the term "milk company" the
entities defined separately under the Milk Code as "distributor" and "manufacturer." The RIRR also
enumerated in Section 5(w) the products manufactured or distributed by an entity that would qualify
it as a "milk company," whereas in the Milk Code, what is used is the phrase "products within the
scope of this Code." Those are the only differences between the definitions given in the Milk Code
and the definition as re-stated in the RIRR.
Since all the regulatory provisions under the Milk Code apply equally to both manufacturers and
distributors, the Court sees no harm in the RIRR providing for just one term to encompass both
entities. The definition of "milk company" in the RIRR and the definitions of "distributor" and
"manufacturer" provided for under the Milk Code are practically the same.
The Court is not convinced that the definition of "milk company" provided in the RIRR would bring
about any change in the treatment or regulation of "distributors" and "manufacturers" of breastmilk
substitutes, as defined under the Milk Code.
Except Sections 4(f), 11 and 46, the rest of the provisions of the RIRR are in consonance with the
objective, purpose and intent of the Milk Code, constituting reasonable regulation of an industry
which affects public health and welfare and, as such, the rest of the RIRR do not constitute illegal
restraint of trade nor are they violative of the due process clause of the Constitution.
WHEREFORE, the petition is PARTIALLY GRANTED. Sections 4(f), 11 and 46 of Administrative
Order No. 2006-0012 dated May 12, 2006 are declared NULL and VOID for being ultra vires. The
Department of Health and respondents are PROHIBITED from implementing said provisions.

The Temporary Restraining Order issued on August 15, 2006 is LIFTED insofar as the rest of the
provisions of Administrative Order No. 2006-0012 is concerned.

Republic of the Philippines


SUPREME COURT
Manila
SECOND DIVISION
G.R. No. 144681

June 21, 2004

PROFESSIONAL REGULATION COMMISSION (PRC), CHAIRMAN HERMOGENES P. POBRE,


ASSOCIATE COMMISSIONER ARMANDO PASCUAL, BOARD OF MEDICINE, CHAIRMAN
RODOLFO P. DE GUZMAN, JOSE S. RAMIREZ, JUANITO B. BILLOTE, RUBEN R.
POLICARPIO, EDGARDO T. FERNANDO and RICARDO D. FULGENCIO II, petitioners,
vs.
ARLENE V. DE GUZMAN, VIOLETA V. MENESES, CELERINA S. NAVARRO, JOSE RAMONCITO
P. NAVARRO, ARNEL V. HERRERA and GERALDINE ELIZABETH M. PAGILAGAN, ELNORA R.
RAQUENO, MARISSA A. REGODON, LAURA M. SANTOS, KARANGALAN D. SERRANO,
DANILO A. VILLAVER, MARIA ROSARIO L. LEONOR, ALICIA S. LIZANO, MARITEL M.
ECHIVERRI, BERNADETTE T. MENDOZA, FERNANDO F. MANDAPAT, ALELI A. GOLLAYAN,
ELCIN C. ARRIOLA, HERMINIGILDA E. CONEJOS, SALLY B. BUNAGAN, ROGELIO B.
ANCHETA, OSCAR H. PADUA, JR., EVELYN D. GRAJO, EVELYN S. ACOSTA, MARGARITA
BELINDA L. VICENCIO, VALENTINO P. ARBOLEDA, EVELYN O. RAMOS, ACHILLES J.
PERALTA, CORAZON M. CRUZ, LEUVINA P. CHICO, JOSEPH A. JAO, MA. LUISA S.
GUTIERREZ, LYDIA C. CHAN, OPHELIA C. HIDALGO, FERNANDO T. CRUZ, MELVIN M. USITA,
RAFAEL I. TOLENTINO, GRACE E. UY, CHERYL R. TRIGUERO, MICHAEL L. SERRANO,
FEDERICO L. CASTILLO, MELITA J. CAEDO, SAMUEL B. BANGOY, BERNARDITA B. SY,
GLORIA T. JULARBAL, FREDERICK D. FRANCISCO, CARLOS M. BERNARDO, JR., HUBERT
S. NAZARENO, CLARISSA B. BACLIG, DAYMINDA G. BONTUYAN, BERNADETTE H.
CABUHAT, NANCY J. CHAVEZ, MARIO D. CUARESMA, ERNESTO L. CUE, EVELYN C.
CUNDANGAN, RHONEIL R. DEVERATURDA, DERILEEN D. DORADO, SAIBZUR N. EDDING,
VIOLETA C. FELIPE, HERMINIO V. FERNANDEZ, JR., MARIA VICTORIA M. LACSAMANA,
NORMA G. LAFAVILLA, RUBY B. LANTIN, MA. ELOISA Q. MALLARI, CLARISA SJ. NICOLAS,
PERCIVAL H. PANGILINAN, ARNULFO A. SALVADOR, ROBERT B. SANCHEZ, MERLY D. STA.
ANA and YOLANDA P. UNICA, respondents.
DECISION
TINGA, J.:
This petition for review under Rule 45 of the 1997 Rules of Civil Procedure seeks to nullify the D E C
I S I O N,1dated May 16, 2000, of the Court of Appeals in CA-G.R. SP No. 37283. The appellate
court affirmed the judgment2 dated December 19, 1994, of the Regional Trial Court (RTC) of Manila,
Branch 52, in Civil Case No. 93-66530. The trial court allowed the respondents to take their

physicians oath and to register as duly licensed physicians. Equally challenged is the R E S O L U T
I O N3 promulgated on August 25, 2000 of the Court of Appeals, denying petitioners Motion for
Reconsideration.
The facts of this case are as follows:
The respondents are all graduates of the Fatima College of Medicine, Valenzuela City, Metro
Manila. They passed the Physician Licensure Examination conducted in February 1993 by
the Board of Medicine (Board). Petitioner Professional Regulation Commission (PRC) then
released their names as successful examinees in the medical licensure examination.
Shortly thereafter, the Board observed that the grades of the seventy-nine successful
examinees from Fatima College in the two most difficult subjects in the medical licensure
exam, Biochemistry (Bio-Chem) and Obstetrics and Gynecology (OB-Gyne), were unusually
and exceptionally high. Eleven Fatima examinees scored 100% in Bio-Chem and ten got
100% in OB-Gyne, another eleven got 99% in Bio-Chem, and twenty-one scored 99% in OBGyne. The Board also observed that many of those who passed from Fatima got marks of
95% or better in both subjects, and no one got a mark lower than 90%. A comparison of the
performances of the candidates from other schools was made. The Board observed that
strangely, the unusually high ratings were true only for Fatima College examinees. It was a
record-breaking phenomenon in the history of the Physician Licensure Examination.
On June 7, 1993, the Board issued Resolution No. 19, withholding the registration as physicians of
all the examinees from the Fatima College of Medicine. 4 The PRC asked the National Bureau of
Investigation (NBI) to investigate whether any anomaly or irregularity marred the February 1993
Physician Licensure Examination.
Prior to the NBI investigation, the Board requested Fr. Bienvenido F. Nebres, S.J., an expert
mathematician and authority in statistics, and later president of the Ateneo de Manila University, to
conduct a statistical analysis of the results in Bio-Chem and Ob-Gyne of the said examination.
On June 10, 1993, Fr. Nebres submitted his report. He reported that a comparison of the scores in
Bio-Chem and Ob-Gyne, of the Fatima College examinees with those of examinees from De La
Salle University and Perpetual Help College of Medicine showed that the scores of Fatima College
examinees were not only incredibly high but unusually clustered close to each other. He concluded
that there must be some unusual reason creating the clustering of scores in the two subjects. It must
be a cause "strong enough to eliminate the normal variations that one should expect from the
examinees [of Fatima College] in terms of talent, effort, energy, etc." 5
For its part, the NBI found that "the questionable passing rate of Fatima examinees in the [1993]
Physician Examination leads to the conclusion that the Fatima examinees gained early access to the
test questions."6
On July 5, 1993, respondents Arlene V. De Guzman, Violeta V. Meneses, Celerina S. Navarro, Jose
Ramoncito P. Navarro, Arnel V. Herrera, and Geraldine Elizabeth M. Pagilagan (Arlene V. De
Guzman et al., for brevity) filed a special civil action for mandamus, with prayer for preliminary

mandatory injunction docketed as Civil Case No. 93-66530 with the Regional Trial Court (RTC) of
Manila, Branch 52. Their petition was adopted by the other respondents as intervenors.
Meanwhile, the Board issued Resolution No. 26, dated July 21, 1993, charging respondents with
"immorality, dishonest conduct, fraud, and deceit" in connection with the Bio-Chem and Ob-Gyne
examinations. It recommended that the test results of the Fatima examinees be nullified. The case
was docketed as Adm. Case No. 1687 by the PRC.
On July 28, 1993, the RTC issued an Order in Civil Case No. 93-66530 granting the preliminary
mandatory injunction sought by the respondents. It ordered the petitioners to administer the
physicians oath to Arlene V. De Guzman et al., and enter their names in the rolls of the PRC.
The petitioners then filed a special civil action for certiorari with the Court of Appeals to set aside the
mandatory injunctive writ, docketed as CA-G.R. SP No. 31701.
On October 21, 1993, the appellate court decided CA-G.R. SP No. 31701, with the dispositive
portion of theDecision ordaining as follows:
WHEREFORE, this petition is GRANTED. Accordingly, the writ of preliminary mandatory
injunction issued by the lower court against petitioners is hereby nullified and set aside.
SO ORDERED.7
Arlene V. de Guzman, et al., then elevated the foregoing Decision to this Court in G.R. No. 112315.
In ourResolution dated May 23, 1994, we denied the petition for failure to show reversible error on
the part of the appellate court.
Meanwhile, on November 22, 1993, during the pendency of the instant petition, the pre-trial
conference in Civil Case No. 93-66530 was held. Then, the parties, agreed to reduce the testimonies
of their respective witnesses to sworn questions-and-answers. This was without prejudice to crossexamination by the opposing counsel.
On December 13, 1993, petitioners counsel failed to appear at the trial in the mistaken belief that
the trial was set for December 15. The trial court then ruled that petitioners waived their right to
cross-examine the witnesses.
On January 27, 1994, counsel for petitioners filed a Manifestation and Motion stating the reasons for
her non-appearance and praying that the cross-examination of the witnesses for the opposing
parties be reset. The trial court denied the motion for lack of notice to adverse counsel. It also denied
the Motion for Reconsideration that followed on the ground that adverse counsel was notified less
than three (3) days prior to the hearing.
Meanwhile, to prevent the PRC and the Board from proceeding with Adm. Case No. 1687, the
respondents herein moved for the issuance of a restraining order, which the lower court granted in
its Order dated April 4, 1994.

The petitioners then filed with this Court a petition for certiorari docketed as G.R. No. 115704, to
annul the Ordersof the trial court dated November 13, 1993, February 28, 1994, and April 4, 1994.
We referred the petition to the Court of Appeals where it was docketed as CA-G.R. SP No. 34506.
On August 31, 1994, the appellate court decided CA-G.R. SP No. 34506 as follows:
WHEREFORE, the present petition for certiorari with prayer for temporary restraining
order/preliminary injunction is GRANTED and the Orders of December 13, 1993, February 7,
1994, February 28, 1994, and April 4, 1994 of the RTC-Manila, Branch 52, and all further
proceedings taken by it in Special Civil Action No. 93-66530 are hereby DECLARED NULL
and VOID. The said RTC-Manila is ordered to allow petitioners counsel to cross-examine the
respondents witnesses, to allow petitioners to present their evidence in due course of trial,
and thereafter to decide the case on the merits on the basis of the evidence of the parties.
Costs against respondents.
IT IS SO ORDERED.8
The trial was then set and notices were sent to the parties.
A day before the first hearing, on September 22, 1994, the petitioners filed an Urgent Ex-Parte
Manifestation and Motion praying for the partial reconsideration of the appellate courts decision in
CA-G.R. SP No. 34506, and for the outright dismissal of Civil Case No. 93-66530. The petitioners
asked for the suspension of the proceedings.
In its Order dated September 23, 1994, the trial court granted the aforesaid motion, cancelled the
scheduled hearing dates, and reset the proceedings to October 21 and 28, 1994.
Meanwhile, on October 25, 1994, the Court of Appeals denied the partial motion for reconsideration
in CA-G.R. SP No. 34506. Thus, petitioners filed with the Supreme Court a petition for review
docketed as G.R. No. 117817, entitled Professional Regulation Commission, et al. v. Court of
Appeals, et al.
On November 11, 1994, counsel for the petitioners failed to appear at the trial of Civil Case No. 9366530. Upon motion of the respondents herein, the trial court ruled that herein petitioners waived
their right to cross-examine the herein respondents. Trial was reset to November 28, 1994.
On November 25, 1994, petitioners counsel moved for the inhibition of the trial court judge for
alleged partiality. On November 28, 1994, the day the Motion to Inhibit was to be heard, petitioners
failed to appear. Thus, the trial court denied the Motion to Inhibit and declared Civil Case No. 9366530 deemed submitted for decision.
On December 19, 1994, the trial court handed down its judgment in Civil Case No. 93-66530, the
fallo of which reads:

WHEREFORE, judgment is rendered ordering the respondents to allow the petitioners and
intervenors (except those with asterisks and footnotes in pages 1 & 2 of this decision)
[sic],9 to take the physicians oath and to register them as physicians.
It should be made clear that this decision is without prejudice to any administrative
disciplinary action which may be taken against any of the petitioners for such causes and in
the manner provided by law and consistent with the requirements of the Constitution as any
other professionals.
No costs.
SO ORDERED.10
As a result of these developments, petitioners filed with this Court a petition for review on certiorari
docketed as G.R. No. 118437, entitled Professional Regulation Commission v. Hon. David G.
Nitafan, praying inter alia, that (1) G.R. No. 118437 be consolidated with G.R. No. 117817; (2) the
decision of the Court of Appeals dated August 31, 1994 in CA-G.R. SP No. 34506 be nullified for its
failure to decree the dismissal of Civil Case No. 93-66530, and in the alternative, to set aside the
decision of the trial court in Civil Case No. 93-66530, order the trial court judge to inhibit himself, and
Civil Case No. 93-66530 be re-raffled to another branch.
On December 26, 1994, the petitioners herein filed their Notice of Appeal11 in Civil Case No. 9366530, thereby elevating the case to the Court of Appeals, where it was docketed as CA-G.R. SP
No. 37283.
In our Resolution of June 7, 1995, G.R. No. 118437 was consolidated with G.R. No. 117817.
On July 9, 1998, we disposed of G.R. Nos. 117817 and 118437 in this wise:
WHEREFORE, the petition in G.R. No. 117817 is DISMISSED for being moot. The petition in
G.R. No. 118437 is likewise DISMISSED on the ground that there is a pending appeal before
the Court of Appeals. Assistant Solicitor General Amparo M. Cabotaje-Tang is advised to be
more circumspect in her dealings with the courts as a repetition of the same or similar acts
will be dealt with accordingly.
SO ORDERED.12
While CA-G.R. SP No. 37283 was awaiting disposition by the appellate court, Arnel V. Herrera, one
of the original petitioners in Civil Case No. 93-66530, joined by twenty-seven intervenors, to wit:
Fernando F. Mandapat, Ophelia C. Hidalgo, Bernadette T. Mendoza, Ruby B. Lantin-Tan, Fernando
T. Cruz, Marissa A. Regodon, Ma. Eloisa Q. Mallari-Largoza, Cheryl R. Triguero, Joseph A. Jao,
Bernadette H. Cabuhat, Evelyn S. Acosta-Cabanes, Laura M. Santos, Maritel M. Echiverri,
Bernadette C. Escusa, Carlosito C. Domingo, Alicia S. Lizano, Elnora R. Raqueno-Rabaino, Saibzur
N. Edding, Derileen D. Dorado-Edding, Robert B. Sanchez, Maria Rosario L. Leonor-Lacandula,
Geraldine Elizabeth M. Pagilagan-Palma, Margarita Belinda L. Vicencio-Gamilla, Herminigilda E.
Conejos, Leuvina P. Chico-Paguio, Elcin C. Arriola-Ocampo, and Jose Ramoncito P. Navarro,

manifested that they were no longer interested in proceeding with the case and moved for its
dismissal. A similar manifestation and motion was later filed by intervenors Mary Jean I. YebanMerlan, Michael L. Serrano, Norma G. Lafavilla, Arnulfo A. Salvador, Belinda C. Rabara, Yolanda P.
Unica, Dayminda G. Bontuyan, Clarissa B. Baclig, Ma. Luisa S. Gutierrez, Rhoneil R. Deveraturda,
Aleli A. Gollayan, Evelyn C. Cundangan, Frederick D. Francisco, Violeta V. Meneses, Melita J.
Caedo, Clarisa SJ. Nicolas, Federico L. Castillo, Karangalan D. Serrano, Danilo A. Villaver, Grace
E. Uy, Lydia C. Chan, and Melvin M. Usita. The Court of Appeals ruled that its decision in CA-G.R.
SP No. 37283 would not apply to them.
On May 16, 2000, the Court of Appeals decided CA-G.R. SP No. 37283, with the following fallo, to
wit:
WHEREFORE, finding no reversible error in the decision appealed from, We hereby AFFIRM
the same and DISMISS the instant appeal.
No pronouncement as to costs.
SO ORDERED.13
In sustaining the trial courts decision, the appellate court ratiocinated that the respondents complied
with all the statutory requirements for admission into the licensure examination for physicians in
February 1993. They all passed the said examination. Having fulfilled the requirements of Republic
Act No. 2382,14 they should be allowed to take their oaths as physicians and be registered in the rolls
of the PRC.
Hence, this petition raising the following issues:
I
WHETHER OR NOT RESPONDENTS HAVE A VALID CAUSE OF ACTION FOR
MANDAMUS AGAINST PETITIONERS IN THE LIGHT OF THE RESOLUTION OF THIS
HONORABLE COURT IN G.R. NO. 112315 AFFIRMING THE COURT OF APPEALS
DECISION DECLARING THAT IF EVER THERE IS SOME DOUBT AS TO THE MORAL
FITNESS OF EXAMINEES, THE ISSUANCE OF LICENSE TO PRACTICE MEDICINE IS
NOT AUTOMATICALLY GRANTED TO THE SUCCESSFUL EXAMINEES.
II
WHETHER OR NOT THE PETITION FOR MANDAMUS COULD PROCEED DESPITE THE
PENDENCY OF ADMINISTRATIVE CASE NO. 1687, WHICH WAS PRECISELY LODGED
TO DETERMINE THE MORAL FITNESS OF RESPONDENTS TO BECOME DOCTORS. 15
To our mind, the only issue is: Did the Court of Appeals commit a reversible error of law in sustaining
the judgment of the trial court that respondents are entitled to a writ of mandamus?

The petitioners submit that a writ of mandamus will not lie in this case. They point out that for a writ
of mandamus to issue, the applicant must have a well-defined, clear and certain legal right to the
thing demanded and it is the duty of the respondent to perform the act required. Thus, mandamus
may be availed of only when the duty sought to be performed is a ministerial and not a discretionary
one. The petitioners argue that the appellate courts decision in CA-G.R. SP No. 37283 upholding
the decision of the trial court in Civil Case No. 93-66530 overlooked its own pronouncement in CAG.R. SP No. 31701. The Court of Appeals held in CA-G.R. SP No. 31701 that the issuance of a
license to engage in the practice of medicine becomes discretionary on the PRC if there exists some
doubt that the successful examinee has not fully met the requirements of the law. The petitioners
stress that this Courts Resolution dated May 24, 1994 in G.R. No. 112315 held that there was no
showing "that the Court of Appeals had committed any reversible error in rendering the questioned
judgment" in CA-G.R. SP No. 31701. The petitioners point out that our Resolution in G.R. No.
112315 has long become final and executory.
Respondents counter that having passed the 1993 licensure examinations for physicians, the
petitioners have the obligation to administer to them the oath as physicians and to issue their
certificates of registration as physicians pursuant to Section 2016 of Rep. Act No. 2382. The Court of
Appeals in CA-G.R. SP No. 37283, found that respondents complied with all the requirements of
Rep. Act No. 2382. Furthermore, respondents were admitted by the Medical Board to the licensure
examinations and had passed the same. Hence, pursuant to Section 20 of Rep. Act No. 2382, the
petitioners had the obligation to administer their oaths as physicians and register them.
Mandamus is a command issuing from a court of competent jurisdiction, in the name of the state or
the sovereign, directed to some inferior court, tribunal, or board, or to some corporation or person
requiring the performance of a particular duty therein specified, which duty results from the official
station of the party to whom the writ is directed, or from operation of law.17 Section 3 of Rule 6518 of
the 1997 Rules of Civil Procedure outlines two situations when a writ of mandamus may issue, when
any tribunal, corporation, board, officer or person unlawfully (1) neglects the performance of an act
which the law specifically enjoins as a duty resulting from an office, trust, or station; or (2) excludes
another from the use and enjoyment of a right or office to which the other is entitled.
We shall discuss the issues successively.
1. On The Existence of a Duty of the Board of Medicine To Issue Certificates of Registration as
Physicians under Rep. Act No. 2382.
For mandamus to prosper, there must be a showing that the officer, board, or official concerned, has
a clear legal duty, not involving discretion.19 Moreover, there must be statutory authority for the
performance of the act,20 and the performance of the duty has been refused.21 Thus, it must be
pertinently asked now: Did petitioners have the duty to administer the Hippocratic Oath and register
respondents as physicians under the Medical Act of 1959?
As found by the Court of Appeals, on which we agree on the basis of the records:
It bears emphasizing herein that petitioner-appellees and intervenor-appellees have fully
complied with all the statutory requirements for admission into the licensure examinations for

physicians conducted and administered by the respondent-appellants on February 12, 14, 20


and 21, 1993. Stress, too, must be made of the fact that all of them successfully passed the
same examinations.22
The crucial query now is whether the Court of Appeals erred in concluding that petitioners should
allow the respondents to take their oaths as physicians and register them, steps which would enable
respondents to practice the medical profession23 pursuant to Section 20 of the Medical Act of 1959?
The appellate court relied on a single provision, Section 20 of Rep. Act No. 2382, in concluding that
the petitioners had the ministerial obligation to administer the Hippocratic Oath to respondents and
register them as physicians. But it is a basic rule in statutory construction that each part of a statute
should be construed in connection with every other part to produce a harmonious whole, not
confining construction to only one section.24 The intent or meaning of the statute should be
ascertained from the statute taken as a whole, not from an isolated part of the provision. Accordingly,
Section 20, of Rep. Act No. 2382, as amended should be read in conjunction with the other
provisions of the Act. Thus, to determine whether the petitioners had the ministerial obligation to
administer the Hippocratic Oath to respondents and register them as physicians, recourse must be
had to the entirety of the Medical Act of 1959.
A careful reading of Section 20 of the Medical Act of 1959 discloses that the law uses the word
"shall" with respect to the issuance of certificates of registration. Thus, the petitioners "shall sign and
issue certificates of registration to those who have satisfactorily complied with the requirements of
the Board." In statutory construction the term "shall" is a word of command. It is given imperative
meaning. Thus, when an examinee satisfies the requirements for the grant of his physicians license,
the Board is obliged to administer to him his oath and register him as a physician, pursuant to
Section 20 and par. (1) of Section 2225 of the Medical Act of 1959.
However, the surrounding circumstances in this case call for serious inquiry concerning the
satisfactory compliance with the Board requirements by the respondents. The unusually high scores
in the two most difficult subjects was phenomenal, according to Fr. Nebres, the consultant of PRC on
the matter, and raised grave doubts about the integrity, if not validity, of the tests. These doubts have
to be appropriately resolved.
Under the second paragraph of Section 22, the Board is vested with the power to conduct
administrative investigations and "disapprove applications for examination or registration," pursuant
to the objectives of Rep. Act No. 2382 as outlined in Section 1 26 thereof. In this case, after the
investigation, the Board filed before the PRC, Adm. Case No. 1687 against the respondents to
ascertain their moral and mental fitness to practice medicine, as required by Section 9 27 of Rep. Act
No. 2382. In its Decision dated July 1, 1997, the Board ruled:
WHEREFORE, the BOARD hereby CANCELS the respondents[] examination papers in the
Physician Licensure Examinations given in February 1993 and further DEBARS them from
taking any licensure examination for a period of ONE (1) YEAR from the date of the
promulgation of this DECISION. They may, if they so desire, apply for the scheduled
examinations for physicians after the lapse of the period imposed by the BOARD.

SO ORDERED.28
Until the moral and mental fitness of the respondents could be ascertained, according to petitioners,
the Board has discretion to hold in abeyance the administration of the Hippocratic Oath and the
issuance of the certificates to them. The writ of mandamus does not lie to compel performance of an
act which is not duly authorized.
The respondents nevertheless argue that under Section 20, the Board shall not issue a certificate of
registration only in the following instances: (1) to any candidate who has been convicted by a court
of competent jurisdiction of any criminal offense involving moral turpitude; (2) or has been found
guilty of immoral or dishonorable conduct after the investigation by the Board; or (3) has been
declared to be of unsound mind. They aver that none of these circumstances are present in their
case.
Petitioners reject respondents argument. We are informed that in Board Resolution No. 26, 29 dated
July 21, 1993, the Board resolved to file charges against the examinees from Fatima College of
Medicine for "immorality, dishonesty, fraud, and deceit in the Obstetrics-Gynecology and
Biochemistry examinations." It likewise sought to cancel the examination results obtained by the
examinees from the Fatima College.
Section 830 of Rep. Act No. 2382 prescribes, among others, that a person who aspires to practice
medicine in the Philippines, must have "satisfactorily passed the corresponding Board Examination."
Section 22, in turn, provides that the oath may only be administered "to physicians who qualified in
the examinations." The operative word here is "satisfactorily," defined as "sufficient to meet a
condition or obligation" or "capable of dispelling doubt or ignorance." 31 Gleaned from Board
Resolution No. 26, the licensing authority apparently did not find that the respondents "satisfactorily
passed" the licensure examinations. The Board instead sought to nullify the examination results
obtained by the respondents.
2. On the Right Of The Respondents To Be Registered As Physicians
The function of mandamus is not to establish a right but to enforce one that has been established by
law. If no legal right has been violated, there can be no application of a legal remedy, and the writ of
mandamus is a legal remedy for a legal right.32 There must be a well-defined, clear and certain legal
right to the thing demanded.33 It is long established rule that a license to practice medicine is a
privilege or franchise granted by the government.34
It is true that this Court has upheld the constitutional right 35 of every citizen to select a profession or
course of study subject to a fair, reasonable, and equitable admission and academic
requirements.36 But like all rights and freedoms guaranteed by the Charter, their exercise may be so
regulated pursuant to the police power of the State to safeguard health, morals, peace, education,
order, safety, and general welfare of the people.37 Thus, persons who desire to engage in the learned
professions requiring scientific or technical knowledge may be required to take an examination as a
prerequisite to engaging in their chosen careers. This regulation takes particular pertinence in the
field of medicine, to protect the public from the potentially deadly effects of incompetence and
ignorance among those who would practice medicine. In a previous case, it may be recalled, this

Court has ordered the Board of Medical Examiners to annul both its resolution and certificate
authorizing a Spanish subject, with the degree of Licentiate in Medicine and Surgery from the
University of Barcelona, Spain, to practice medicine in the Philippines, without first passing the
examination required by the Philippine Medical Act.38 In another case worth noting, we upheld the
power of the State to upgrade the selection of applicants into medical schools through admission
tests.39
It must be stressed, nevertheless, that the power to regulate the exercise of a profession or pursuit
of an occupation cannot be exercised by the State or its agents in an arbitrary, despotic, or
oppressive manner. A political body that regulates the exercise of a particular privilege has the
authority to both forbid and grant such privilege in accordance with certain conditions. Such
conditions may not, however, require giving up ones constitutional rights as a condition to acquiring
the license.40 Under the view that the legislature cannot validly bestow an arbitrary power to grant or
refuse a license on a public agency or officer, courts will generally strike down license legislation that
vests in public officials discretion to grant or refuse a license to carry on some ordinarily lawful
business, profession, or activity without prescribing definite rules and conditions for the guidance of
said officials in the exercise of their power.41
In the present case, the aforementioned guidelines are provided for in Rep. Act No. 2382, as
amended, which prescribes the requirements for admission to the practice of medicine, the
qualifications of candidates for the board examinations, the scope and conduct of the examinations,
the grounds for denying the issuance of a physicians license, or revoking a license that has been
issued. Verily, to be granted the privilege to practice medicine, the applicant must show that he
possesses all the qualifications and none of the disqualifications. Furthermore, it must appear that
he has fully complied with all the conditions and requirements imposed by the law and the licensing
authority. Should doubt taint or mar the compliance as being less than satisfactory, then the privilege
will not issue. For said privilege is distinguishable from a matter of right, which may be demanded if
denied. Thus, without a definite showing that the aforesaid requirements and conditions have been
satisfactorily met, the courts may not grant the writ of mandamus to secure said privilege without
thwarting the legislative will.
3. On the Ripeness of the Petition for Mandamus
Lastly, the petitioners herein contend that the Court of Appeals should have dismissed the petition
for mandamus below for being premature. They argue that the administrative remedies had not been
exhausted. The records show that this is not the first time that petitioners have sought the dismissal
of Civil Case No. 93-66530. This issue was raised in G.R. No. 115704, which petition we referred to
the Court of Appeals, where it was docketed as CA-G.R. SP No. 34506. On motion for
reconsideration in CA-G.R. SP No. 34506, the appellate court denied the motion to dismiss on the
ground that the prayers for the nullification of the order of the trial court and the dismissal of Civil
Case No. 93-66530 were inconsistent reliefs. In G.R. No. 118437, the petitioners sought to nullify the
decision of the Court of Appeals in CA-G.R. SP No. 34506 insofar as it did not order the dismissal of
Civil Case No. 93-66530. In our consolidated decision, dated July 9, 1998, in G.R. Nos. 117817 &
118437, this Court speaking through Justice Bellosillo opined that:

Indeed, the issue as to whether the Court of Appeals erred in not ordering the dismissal of
Civil Case No. 93-66530 sought to be resolved in the instant petition has been rendered
meaningless by an event taking place prior to the filing of this petition and denial thereof
should follow as a logical consequence.42 There is no longer any justiciable controversy so
that any declaration thereon would be of no practical use or value. 43 It should be recalled that
in its decision of 19 December 1994 the trial court granted the writ of mandamus prayed for
by private respondents, which decision was received by petitioners on 20 December 1994.
Three (3) days after, or on 23 December 1994, petitioners filed the instant petition. By then,
the remedy available to them was to appeal the decision to the Court of Appeals, which they
in fact did, by filing a notice of appeal on 26 December 1994.44
The petitioners have shown no cogent reason for us to reverse the aforecited ruling. Nor will their
reliance upon the doctrine of the exhaustion of administrative remedies in the instant case advance
their cause any.
Section 2645 of the Medical Act of 1959 provides for the administrative and judicial remedies that
respondents herein can avail to question Resolution No. 26 of the Board of Medicine, namely: (a)
appeal the unfavorable judgment to the PRC; (b) should the PRC ruling still be unfavorable, to
elevate the matter on appeal to the Office of the President; and (c) should they still be unsatisfied, to
ask for a review of the case or to bring the case to court via a special civil action of certiorari. Thus,
as a rule, mandamus will not lie when administrative remedies are still available. 46 However, the
doctrine of exhaustion of administrative remedies does not apply where, as in this case, a pure
question of law is raised.47 On this issue, no reversible error may, thus, be laid at the door of the
appellate court in CA-G.R. SP No. 37283, when it refused to dismiss Civil Case No. 93-66530.
As we earlier pointed out, herein respondents Arnel V. Herrera, Fernando F. Mandapat, Ophelia C.
Hidalgo, Bernadette T. Mendoza, Ruby B. Lantin-Tan, Fernando T. Cruz, Marissa A. Regodon, Ma.
Eloisa Q. Mallari-Largoza, Cheryl R. Triguero, Joseph A. Jao, Bernadette H. Cabuhat, Evelyn S.
Acosta-Cabanes, Laura M. Santos, Maritel M. Echiverri, Bernadette C. Escusa, Carlosito C.
Domingo, Alicia S. Lizano, Elnora R. Raqueno-Rabaino, Saibzur N. Edding, Derileen D. DoradoEdding, Robert B. Sanchez, Maria Rosario Leonor-Lacandula, Geraldine Elizabeth M. PagilaganPalma, Margarita Belinda L. Vicencio-Gamilla, Herminigilda E. Conejos, Leuvina P. Chico-Paguio,
Elcin C. Arriola-Ocampo, and Jose Ramoncito P. Navarro manifested to the Court of Appeals during
the pendency of CA-G.R. SP No. 37283, that they were no longer interested in proceeding with the
case and moved for its dismissal insofar as they were concerned. A similar manifestation and motion
were later filed by intervenors Mary Jean I. Yeban-Merlan, Michael L. Serrano, Norma G. Lafavilla,
Arnulfo A. Salvador, Belinda C. Rabarra, Yolanda P. Unica, Dayminda G. Bontuyan, Clarissa B.
Baclig, Ma. Luisa S. Gutierrez, Rhoneil R. Deveraturda, Aleli A. Gollayan, Evelyn C. Cundangan,
Frederick D. Francisco, Violeta V. Meneses, Melita J. Caedo, Clarisa SJ. Nicolas, Federico L.
Castillo, Karangalan D. Serrano, Danilo A. Villaver, Grace E. Uy, Lydia C. Chan, and Melvin M. Usita.
Following these manifestations and motions, the appellate court in CA-G.R. SP No. 37283 decreed
that its ruling would not apply to them. Thus, inasmuch as the instant case is a petition for review of
the appellate courts ruling in CA-G.R. SP No. 37283, a decision which is inapplicable to the
aforementioned respondents will similarly not apply to them.

As to Achilles J. Peralta, Evelyn O. Ramos, Sally B. Bunagan, Rogelio B. Ancheta, Oscar H. Padua,
Jr., Evelyn D. Grajo, Valentino P. Arboleda, Carlos M. Bernardo, Jr., Mario D. Cuaresma, Violeta C.
Felipe, Percival H. Pangilinan, Corazon M. Cruz and Samuel B. Bangoy, herein decision shall not
apply pursuant to the Orders of the trial court in Civil Case No. 93-66530, dropping their names from
the suit.
Consequently, this Decision is binding only on the remaining respondents, namely: Arlene V. de
Guzman, Celerina S. Navarro, Rafael I. Tolentino, Bernardita B. Sy, Gloria T. Jularbal, Hubert S.
Nazareno, Nancy J. Chavez, Ernesto L. Cue, Herminio V. Fernandez, Jr., Maria Victoria M.
Lacsamana and Merly D. Sta. Ana, as well as the petitioners.
WHEREFORE, the instant petition is GRANTED. Accordingly, (1) the assailed decision dated May
16, 2000, of the Court of Appeals, in CA-G.R. SP No. 37283, which affirmed the judgment dated
December 19, 1994, of the Regional Trial Court of Manila, Branch 52, in Civil Case No. 93-66530,
ordering petitioners to administer the physicians oath to herein respondents as well as the resolution
dated August 25, 2000, of the appellate court, denying the petitioners motion for reconsideration, are
REVERSED and SET ASIDE; and (2) the writ of mandamus, issued in Civil Case No. 93-66530, and
affirmed by the appellate court in CA-G.R. SP No. 37283 is NULLIFIED AND SET ASIDE.
Republic of the Philippines
SUPREME COURT
Manila
EN BANC

G.R. No. 101083 July 30, 1993


JUAN ANTONIO, ANNA ROSARIO and JOSE ALFONSO, all surnamed OPOSA, minors, and
represented by their parents ANTONIO and RIZALINA OPOSA, ROBERTA NICOLE SADIUA,
minor, represented by her parents CALVIN and ROBERTA SADIUA, CARLO, AMANDA SALUD
and PATRISHA, all surnamed FLORES, minors and represented by their parents ENRICO and
NIDA FLORES, GIANINA DITA R. FORTUN, minor, represented by her parents SIGRID and
DOLORES FORTUN, GEORGE II and MA. CONCEPCION, all surnamed MISA, minors and
represented by their parents GEORGE and MYRA MISA, BENJAMIN ALAN V. PESIGAN, minor,
represented by his parents ANTONIO and ALICE PESIGAN, JOVIE MARIE ALFARO, minor,
represented by her parents JOSE and MARIA VIOLETA ALFARO, MARIA CONCEPCION T.
CASTRO, minor, represented by her parents FREDENIL and JANE CASTRO, JOHANNA
DESAMPARADO,
minor, represented by her parents JOSE and ANGELA DESAMPRADO, CARLO JOAQUIN T.
NARVASA, minor, represented by his parents GREGORIO II and CRISTINE CHARITY
NARVASA, MA. MARGARITA, JESUS IGNACIO, MA. ANGELA and MARIE GABRIELLE, all
surnamed SAENZ, minors, represented by their parents ROBERTO and AURORA SAENZ,
KRISTINE, MARY ELLEN, MAY, GOLDA MARTHE and DAVID IAN, all surnamed KING, minors,
represented by their parents MARIO and HAYDEE KING, DAVID, FRANCISCO and THERESE

VICTORIA, all surnamed ENDRIGA, minors, represented by their parents BALTAZAR and
TERESITA ENDRIGA, JOSE MA. and REGINA MA., all surnamed ABAYA, minors, represented
by their parents ANTONIO and MARICA ABAYA, MARILIN, MARIO, JR. and MARIETTE, all
surnamed CARDAMA, minors, represented by their parents MARIO and LINA CARDAMA,
CLARISSA, ANN MARIE, NAGEL, and IMEE LYN, all surnamed OPOSA, minors and
represented by their parents RICARDO and MARISSA OPOSA, PHILIP JOSEPH, STEPHEN
JOHN and ISAIAH JAMES, all surnamed QUIPIT, minors, represented by their parents JOSE
MAX and VILMI QUIPIT, BUGHAW CIELO, CRISANTO, ANNA, DANIEL and FRANCISCO, all
surnamed BIBAL, minors, represented by their parents FRANCISCO, JR. and MILAGROS
BIBAL, and THE PHILIPPINE ECOLOGICAL NETWORK, INC., petitioners,
vs.
THE HONORABLE FULGENCIO S. FACTORAN, JR., in his capacity as the Secretary of the
Department of Environment and Natural Resources, and THE HONORABLE ERIBERTO U.
ROSARIO, Presiding Judge of the RTC, Makati, Branch 66, respondents.
Oposa Law Office for petitioners.
The Solicitor General for respondents.

DAVIDE, JR., J.:


In a broader sense, this petition bears upon the right of Filipinos to a balanced and healthful ecology
which the petitioners dramatically associate with the twin concepts of "inter-generational
responsibility" and "inter-generational justice." Specifically, it touches on the issue of whether the
said petitioners have a cause of action to "prevent the misappropriation or impairment" of Philippine
rainforests and "arrest the unabated hemorrhage of the country's vital life support systems and
continued rape of Mother Earth."
The controversy has its genesis in Civil Case No. 90-77 which was filed before Branch 66 (Makati,
Metro Manila) of the Regional Trial Court (RTC), National Capital Judicial Region. The principal
plaintiffs therein, now the principal petitioners, are all minors duly represented and joined by their
respective parents. Impleaded as an additional plaintiff is the Philippine Ecological Network, Inc.
(PENI), a domestic, non-stock and non-profit corporation organized for the purpose of, inter alia,
engaging in concerted action geared for the protection of our environment and natural resources.
The original defendant was the Honorable Fulgencio S. Factoran, Jr., then Secretary of the
Department of Environment and Natural Resources (DENR). His substitution in this petition by the
new Secretary, the Honorable Angel C. Alcala, was subsequently ordered upon proper motion by the
petitioners. 1 The complaint 2 was instituted as a taxpayers' class suit 3 and alleges that the plaintiffs "are
all citizens of the Republic of the Philippines, taxpayers, and entitled to the full benefit, use and enjoyment
of the natural resource treasure that is the country's virgin tropical forests." The same was filed for
themselves and others who are equally concerned about the preservation of said resource but are "so
numerous that it is impracticable to bring them all before the Court." The minors further asseverate that
they "represent their generation as well as generations yet unborn." 4 Consequently, it is prayed for that
judgment be rendered:

. . . ordering defendant, his agents, representatives and other persons acting in his
behalf to
(1) Cancel all existing timber license agreements in the country;
(2) Cease and desist from receiving, accepting, processing, renewing or approving
new timber license agreements.
and granting the plaintiffs ". . . such other reliefs just and equitable under the premises." 5
The complaint starts off with the general averments that the Philippine archipelago of 7,100 islands
has a land area of thirty million (30,000,000) hectares and is endowed with rich, lush and verdant
rainforests in which varied, rare and unique species of flora and fauna may be found; these
rainforests contain a genetic, biological and chemical pool which is irreplaceable; they are also the
habitat of indigenous Philippine cultures which have existed, endured and flourished since time
immemorial; scientific evidence reveals that in order to maintain a balanced and healthful ecology,
the country's land area should be utilized on the basis of a ratio of fifty-four per cent (54%) for forest
cover and forty-six per cent (46%) for agricultural, residential, industrial, commercial and other uses;
the distortion and disturbance of this balance as a consequence of deforestation have resulted in a
host of environmental tragedies, such as (a) water shortages resulting from drying up of the water
table, otherwise known as the "aquifer," as well as of rivers, brooks and streams, (b) salinization of
the water table as a result of the intrusion therein of salt water, incontrovertible examples of which
may be found in the island of Cebu and the Municipality of Bacoor, Cavite, (c) massive erosion and
the consequential loss of soil fertility and agricultural productivity, with the volume of soil eroded
estimated at one billion (1,000,000,000) cubic meters per annum approximately the size of the
entire island of Catanduanes, (d) the endangering and extinction of the country's unique, rare and
varied flora and fauna, (e) the disturbance and dislocation of cultural communities, including the
disappearance of the Filipino's indigenous cultures, (f) the siltation of rivers and seabeds and
consequential destruction of corals and other aquatic life leading to a critical reduction in marine
resource productivity, (g) recurrent spells of drought as is presently experienced by the entire
country, (h) increasing velocity of typhoon winds which result from the absence of windbreakers, (i)
the floodings of lowlands and agricultural plains arising from the absence of the absorbent
mechanism of forests, (j) the siltation and shortening of the lifespan of multi-billion peso dams
constructed and operated for the purpose of supplying water for domestic uses, irrigation and the
generation of electric power, and (k) the reduction of the earth's capacity to process carbon dioxide
gases which has led to perplexing and catastrophic climatic changes such as the phenomenon of
global warming, otherwise known as the "greenhouse effect."
Plaintiffs further assert that the adverse and detrimental consequences of continued and
deforestation are so capable of unquestionable demonstration that the same may be submitted as a
matter of judicial notice. This notwithstanding, they expressed their intention to present expert
witnesses as well as documentary, photographic and film evidence in the course of the trial.
As their cause of action, they specifically allege that:
CAUSE OF ACTION

7. Plaintiffs replead by reference the foregoing allegations.


8. Twenty-five (25) years ago, the Philippines had some sixteen (16) million hectares
of rainforests constituting roughly 53% of the country's land mass.
9. Satellite images taken in 1987 reveal that there remained no more than 1.2 million
hectares of said rainforests or four per cent (4.0%) of the country's land area.
10. More recent surveys reveal that a mere 850,000 hectares of virgin old-growth
rainforests are left, barely 2.8% of the entire land mass of the Philippine archipelago
and about 3.0 million hectares of immature and uneconomical secondary growth
forests.
11. Public records reveal that the defendant's, predecessors have granted timber
license agreements ('TLA's') to various corporations to cut the aggregate area of
3.89 million hectares for commercial logging purposes.
A copy of the TLA holders and the corresponding areas covered is hereto attached
as Annex "A".
12. At the present rate of deforestation, i.e. about 200,000 hectares per annum or 25
hectares per hour nighttime, Saturdays, Sundays and holidays included the
Philippines will be bereft of forest resources after the end of this ensuing decade, if
not earlier.
13. The adverse effects, disastrous consequences, serious injury and irreparable
damage of this continued trend of deforestation to the plaintiff minor's generation and
to generations yet unborn are evident and incontrovertible. As a matter of fact, the
environmental damages enumerated in paragraph 6 hereof are already being felt,
experienced and suffered by the generation of plaintiff adults.
14. The continued allowance by defendant of TLA holders to cut and deforest the
remaining forest stands will work great damage and irreparable injury to plaintiffs
especially plaintiff minors and their successors who may never see, use, benefit
from and enjoy this rare and unique natural resource treasure.
This act of defendant constitutes a misappropriation and/or impairment of the natural
resource property he holds in trust for the benefit of plaintiff minors and succeeding
generations.
15. Plaintiffs have a clear and constitutional right to a balanced and healthful ecology
and are entitled to protection by the State in its capacity as the parens patriae.
16. Plaintiff have exhausted all administrative remedies with the defendant's office.
On March 2, 1990, plaintiffs served upon defendant a final demand to cancel all
logging permits in the country.

A copy of the plaintiffs' letter dated March 1, 1990 is hereto attached as Annex "B".
17. Defendant, however, fails and refuses to cancel the existing TLA's to the
continuing serious damage and extreme prejudice of plaintiffs.
18. The continued failure and refusal by defendant to cancel the TLA's is an act
violative of the rights of plaintiffs, especially plaintiff minors who may be left with a
country that is desertified (sic), bare, barren and devoid of the wonderful flora, fauna
and indigenous cultures which the Philippines had been abundantly blessed with.
19. Defendant's refusal to cancel the aforementioned TLA's is manifestly contrary to
the public policy enunciated in the Philippine Environmental Policy which, in pertinent
part, states that it is the policy of the State
(a) to create, develop, maintain and improve conditions under which man and nature
can thrive in productive and enjoyable harmony with each other;
(b) to fulfill the social, economic and other requirements of present and future
generations of Filipinos and;
(c) to ensure the attainment of an environmental quality that is conductive to a life of
dignity and well-being. (P.D. 1151, 6 June 1977)
20. Furthermore, defendant's continued refusal to cancel the aforementioned TLA's
is contradictory to the Constitutional policy of the State to
a. effect "a more equitable distribution of opportunities, income and wealth" and
"make full and efficient use of natural resources (sic)." (Section 1, Article XII of the
Constitution);
b. "protect the nation's marine wealth." (Section 2, ibid);
c. "conserve and promote the nation's cultural heritage and resources (sic)" (Section
14, Article XIV,id.);
d. "protect and advance the right of the people to a balanced and healthful ecology in
accord with the rhythm and harmony of nature." (Section 16, Article II, id.)
21. Finally, defendant's act is contrary to the highest law of humankind the natural
law and violative of plaintiffs' right to self-preservation and perpetuation.
22. There is no other plain, speedy and adequate remedy in law other than the
instant action to arrest the unabated hemorrhage of the country's vital life support
systems and continued rape of Mother Earth. 6

On 22 June 1990, the original defendant, Secretary Factoran, Jr., filed a Motion to Dismiss the
complaint based on two (2) grounds, namely: (1) the plaintiffs have no cause of action against him
and (2) the issue raised by the plaintiffs is a political question which properly pertains to the
legislative or executive branches of Government. In their 12 July 1990 Opposition to the Motion, the
petitioners maintain that (1) the complaint shows a clear and unmistakable cause of action, (2) the
motion is dilatory and (3) the action presents a justiciable question as it involves the defendant's
abuse of discretion.
On 18 July 1991, respondent Judge issued an order granting the aforementioned motion to
dismiss. 7 In the said order, not only was the defendant's claim that the complaint states no cause of
action against him and that it raises a political question sustained, the respondent Judge further ruled
that the granting of the relief prayed for would result in the impairment of contracts which is prohibited by
the fundamental law of the land.
Plaintiffs thus filed the instant special civil action for certiorari under Rule 65 of the Revised Rules of
Court and ask this Court to rescind and set aside the dismissal order on the ground that the
respondent Judge gravely abused his discretion in dismissing the action. Again, the parents of the
plaintiffs-minors not only represent their children, but have also joined the latter in this case. 8
On 14 May 1992, We resolved to give due course to the petition and required the parties to submit
their respective Memoranda after the Office of the Solicitor General (OSG) filed a Comment in behalf
of the respondents and the petitioners filed a reply thereto.
Petitioners contend that the complaint clearly and unmistakably states a cause of action as it
contains sufficient allegations concerning their right to a sound environment based on Articles 19, 20
and 21 of the Civil Code (Human Relations), Section 4 of Executive Order (E.O.) No. 192 creating
the DENR, Section 3 of Presidential Decree (P.D.) No. 1151 (Philippine Environmental Policy),
Section 16, Article II of the 1987 Constitution recognizing the right of the people to a balanced and
healthful ecology, the concept of generational genocide in Criminal Law and the concept of man's
inalienable right to self-preservation and self-perpetuation embodied in natural law. Petitioners
likewise rely on the respondent's correlative obligation per Section 4 of E.O. No. 192, to safeguard
the people's right to a healthful environment.
It is further claimed that the issue of the respondent Secretary's alleged grave abuse of discretion in
granting Timber License Agreements (TLAs) to cover more areas for logging than what is available
involves a judicial question.
Anent the invocation by the respondent Judge of the Constitution's non-impairment clause,
petitioners maintain that the same does not apply in this case because TLAs are not contracts. They
likewise submit that even if TLAs may be considered protected by the said clause, it is well settled
that they may still be revoked by the State when the public interest so requires.
On the other hand, the respondents aver that the petitioners failed to allege in their complaint a
specific legal right violated by the respondent Secretary for which any relief is provided by law. They
see nothing in the complaint but vague and nebulous allegations concerning an "environmental right"
which supposedly entitles the petitioners to the "protection by the state in its capacity as parens

patriae." Such allegations, according to them, do not reveal a valid cause of action. They then
reiterate the theory that the question of whether logging should be permitted in the country is a
political question which should be properly addressed to the executive or legislative branches of
Government. They therefore assert that the petitioners' resources is not to file an action to court, but
to lobby before Congress for the passage of a bill that would ban logging totally.
As to the matter of the cancellation of the TLAs, respondents submit that the same cannot be done
by the State without due process of law. Once issued, a TLA remains effective for a certain period of
time usually for twenty-five (25) years. During its effectivity, the same can neither be revised nor
cancelled unless the holder has been found, after due notice and hearing, to have violated the terms
of the agreement or other forestry laws and regulations. Petitioners' proposition to have all the TLAs
indiscriminately cancelled without the requisite hearing would be violative of the requirements of due
process.
Before going any further, We must first focus on some procedural matters. Petitioners instituted Civil
Case No. 90-777 as a class suit. The original defendant and the present respondents did not take
issue with this matter. Nevertheless, We hereby rule that the said civil case is indeed a class suit.
The subject matter of the complaint is of common and general interest not just to several, but to all
citizens of the Philippines. Consequently, since the parties are so numerous, it, becomes
impracticable, if not totally impossible, to bring all of them before the court. We likewise declare that
the plaintiffs therein are numerous and representative enough to ensure the full protection of all
concerned interests. Hence, all the requisites for the filing of a valid class suit under Section 12, Rule
3 of the Revised Rules of Court are present both in the said civil case and in the instant petition, the
latter being but an incident to the former.
This case, however, has a special and novel element. Petitioners minors assert that they represent
their generation as well as generations yet unborn. We find no difficulty in ruling that they can, for
themselves, for others of their generation and for the succeeding generations, file a class suit. Their
personality to sue in behalf of the succeeding generations can only be based on the concept of
intergenerational responsibility insofar as the right to a balanced and healthful ecology is concerned.
Such a right, as hereinafter expounded, considers
the "rhythm and harmony of nature." Nature means the created world in its entirety. 9 Such rhythm and
harmony indispensably include, inter alia, the judicious disposition, utilization, management, renewal and
conservation of the country's forest, mineral, land, waters, fisheries, wildlife, off-shore areas and other
natural resources to the end that their exploration, development and utilization be equitably accessible to
the present as well as future generations. 10 Needless to say, every generation has a responsibility to the
next to preserve that rhythm and harmony for the full enjoyment of a balanced and healthful ecology. Put
a little differently, the minors' assertion of their right to a sound environment constitutes, at the same time,
the performance of their obligation to ensure the protection of that right for the generations to come.
The locus standi of the petitioners having thus been addressed, We shall now proceed to the merits
of the petition.
After a careful perusal of the complaint in question and a meticulous consideration and evaluation of
the issues raised and arguments adduced by the parties, We do not hesitate to find for the
petitioners and rule against the respondent Judge's challenged order for having been issued with

grave abuse of discretion amounting to lack of jurisdiction. The pertinent portions of the said order
reads as follows:
xxx xxx xxx
After a careful and circumspect evaluation of the Complaint, the Court cannot help
but agree with the defendant. For although we believe that plaintiffs have but the
noblest of all intentions, it (sic) fell short of alleging, with sufficient definiteness, a
specific legal right they are seeking to enforce and protect, or a specific legal wrong
they are seeking to prevent and redress (Sec. 1, Rule 2, RRC). Furthermore, the
Court notes that the Complaint is replete with vague assumptions and vague
conclusions based on unverified data. In fine, plaintiffs fail to state a cause of action
in its Complaint against the herein defendant.
Furthermore, the Court firmly believes that the matter before it, being impressed with
political color and involving a matter of public policy, may not be taken cognizance of
by this Court without doing violence to the sacred principle of "Separation of Powers"
of the three (3) co-equal branches of the Government.
The Court is likewise of the impression that it cannot, no matter how we stretch our
jurisdiction, grant the reliefs prayed for by the plaintiffs, i.e., to cancel all existing
timber license agreements in the country and to cease and desist from receiving,
accepting, processing, renewing or approving new timber license agreements. For to
do otherwise would amount to "impairment of contracts" abhored (sic) by the
fundamental law. 11
We do not agree with the trial court's conclusions that the plaintiffs failed to allege with sufficient
definiteness a specific legal right involved or a specific legal wrong committed, and that the
complaint is replete with vague assumptions and conclusions based on unverified data. A reading of
the complaint itself belies these conclusions.
The complaint focuses on one specific fundamental legal right the right to a balanced and
healthful ecology which, for the first time in our nation's constitutional history, is solemnly
incorporated in the fundamental law. Section 16, Article II of the 1987 Constitution explicitly provides:
Sec. 16. The State shall protect and advance the right of the people to a balanced
and healthful ecology in accord with the rhythm and harmony of nature.
This right unites with the right to health which is provided for in the preceding section
of the same article:
Sec. 15. The State shall protect and promote the right to health of the people and
instill health consciousness among them.
While the right to a balanced and healthful ecology is to be found under the Declaration of Principles
and State Policies and not under the Bill of Rights, it does not follow that it is less important than any

of the civil and political rights enumerated in the latter. Such a right belongs to a different category of
rights altogether for it concerns nothing less than self-preservation and self-perpetuation aptly and
fittingly stressed by the petitioners the advancement of which may even be said to predate all
governments and constitutions. As a matter of fact, these basic rights need not even be written in the
Constitution for they are assumed to exist from the inception of humankind. If they are now explicitly
mentioned in the fundamental charter, it is because of the well-founded fear of its framers that unless
the rights to a balanced and healthful ecology and to health are mandated as state policies by the
Constitution itself, thereby highlighting their continuing importance and imposing upon the state a
solemn obligation to preserve the first and protect and advance the second, the day would not be too
far when all else would be lost not only for the present generation, but also for those to come
generations which stand to inherit nothing but parched earth incapable of sustaining life.
The right to a balanced and healthful ecology carries with it the correlative duty to refrain from
impairing the environment. During the debates on this right in one of the plenary sessions of the
1986 Constitutional Commission, the following exchange transpired between Commissioner Wilfrido
Villacorta and Commissioner Adolfo Azcuna who sponsored the section in question:
MR. VILLACORTA:
Does this section mandate the State to provide sanctions against all
forms of pollution air, water and noise pollution?
MR. AZCUNA:
Yes, Madam President. The right to healthful (sic) environment
necessarily carries with it the correlative duty of not impairing the
same and, therefore, sanctions may be provided for impairment of
environmental balance. 12
The said right implies, among many other things, the judicious management and conservation of the
country's forests.
Without such forests, the ecological or environmental balance would be irreversiby disrupted.
Conformably with the enunciated right to a balanced and healthful ecology and the right to health, as
well as the other related provisions of the Constitution concerning the conservation, development
and utilization of the country's natural resources, 13 then President Corazon C. Aquino promulgated on
10 June 1987 E.O. No. 192, 14 Section 4 of which expressly mandates that the Department of
Environment and Natural Resources "shall be the primary government agency responsible for the
conservation, management, development and proper use of the country's environment and natural
resources, specifically forest and grazing lands, mineral, resources, including those in reservation and
watershed areas, and lands of the public domain, as well as the licensing and regulation of all natural
resources as may be provided for by law in order to ensure equitable sharing of the benefits derived
therefrom for the welfare of the present and future generations of Filipinos." Section 3 thereof makes the
following statement of policy:

Sec. 3. Declaration of Policy. It is hereby declared the policy of the State to ensure
the sustainable use, development, management, renewal, and conservation of the
country's forest, mineral, land, off-shore areas and other natural resources, including
the protection and enhancement of the quality of the environment, and equitable
access of the different segments of the population to the development and the use of
the country's natural resources, not only for the present generation but for future
generations as well. It is also the policy of the state to recognize and apply a true
value system including social and environmental cost implications relative to their
utilization, development and conservation of our natural resources.
This policy declaration is substantially re-stated it Title XIV, Book IV of the Administrative Code of
1987, 15specifically in Section 1 thereof which reads:
Sec. 1. Declaration of Policy. (1) The State shall ensure, for the benefit of the
Filipino people, the full exploration and development as well as the judicious
disposition, utilization, management, renewal and conservation of the country's
forest, mineral, land, waters, fisheries, wildlife, off-shore areas and other natural
resources, consistent with the necessity of maintaining a sound ecological balance
and protecting and enhancing the quality of the environment and the objective of
making the exploration, development and utilization of such natural resources
equitably accessible to the different segments of the present as well as future
generations.
(2) The State shall likewise recognize and apply a true value system that takes into
account social and environmental cost implications relative to the utilization,
development and conservation of our natural resources.
The above provision stresses "the necessity of maintaining a sound ecological balance and
protecting and enhancing the quality of the environment." Section 2 of the same Title, on the other
hand, specifically speaks of the mandate of the DENR; however, it makes particular reference to the
fact of the agency's being subject to law and higher authority. Said section provides:
Sec. 2. Mandate. (1) The Department of Environment and Natural Resources shall
be primarily responsible for the implementation of the foregoing policy.
(2) It shall, subject to law and higher authority, be in charge of carrying out the State's
constitutional mandate to control and supervise the exploration, development,
utilization, and conservation of the country's natural resources.
Both E.O. NO. 192 and the Administrative Code of 1987 have set the objectives which will serve as
the bases for policy formulation, and have defined the powers and functions of the DENR.
It may, however, be recalled that even before the ratification of the 1987 Constitution, specific
statutes already paid special attention to the "environmental right" of the present and future
generations. On 6 June 1977, P.D. No. 1151 (Philippine Environmental Policy) and P.D. No. 1152
(Philippine Environment Code) were issued. The former "declared a continuing policy of the State (a)

to create, develop, maintain and improve conditions under which man and nature can thrive in
productive and enjoyable harmony with each other, (b) to fulfill the social, economic and other
requirements of present and future generations of Filipinos, and (c) to insure the attainment of an
environmental quality that is conducive to a life of dignity and well-being." 16 As its goal, it speaks of
the "responsibilities of each generation as trustee and guardian of the environment for succeeding
generations." 17 The latter statute, on the other hand, gave flesh to the said policy.
Thus, the right of the petitioners (and all those they represent) to a balanced and healthful ecology is
as clear as the DENR's duty under its mandate and by virtue of its powers and functions under
E.O. No. 192 and the Administrative Code of 1987 to protect and advance the said right.
A denial or violation of that right by the other who has the corelative duty or obligation to respect or
protect the same gives rise to a cause of action. Petitioners maintain that the granting of the TLAs,
which they claim was done with grave abuse of discretion, violated their right to a balanced and
healthful ecology; hence, the full protection thereof requires that no further TLAs should be renewed
or granted.
A cause of action is defined as:
. . . an act or omission of one party in violation of the legal right or rights of the other;
and its essential elements are legal right of the plaintiff, correlative obligation of the
defendant, and act or omission of the defendant in violation of said legal right. 18
It is settled in this jurisdiction that in a motion to dismiss based on the ground that the complaint fails
to state a cause of action, 19 the question submitted to the court for resolution involves the sufficiency of
the facts alleged in the complaint itself. No other matter should be considered; furthermore, the truth of
falsity of the said allegations is beside the point for the truth thereof is deemed hypothetically admitted.
The only issue to be resolved in such a case is: admitting such alleged facts to be true, may the court
render a valid judgment in accordance with the prayer in the complaint? 20 InMilitante vs.
Edrosolano, 21 this Court laid down the rule that the judiciary should "exercise the utmost care and
circumspection in passing upon a motion to dismiss on the ground of the absence thereof [cause of
action] lest, by its failure to manifest a correct appreciation of the facts alleged and deemed hypothetically
admitted, what the law grants or recognizes is effectively nullified. If that happens, there is a blot on the
legal order. The law itself stands in disrepute."
After careful examination of the petitioners' complaint, We find the statements under the introductory
affirmative allegations, as well as the specific averments under the sub-heading CAUSE OF
ACTION, to be adequate enough to show, prima facie, the claimed violation of their rights. On the
basis thereof, they may thus be granted, wholly or partly, the reliefs prayed for. It bears stressing,
however, that insofar as the cancellation of the TLAs is concerned, there is the need to implead, as
party defendants, the grantees thereof for they are indispensable parties.
The foregoing considered, Civil Case No. 90-777 be said to raise a political question. Policy
formulation or determination by the executive or legislative branches of Government is not squarely
put in issue. What is principally involved is the enforcement of a right vis-a-vis policies already
formulated and expressed in legislation. It must, nonetheless, be emphasized that the political
question doctrine is no longer, the insurmountable obstacle to the exercise of judicial power or the

impenetrable shield that protects executive and legislative actions from judicial inquiry or review. The
second paragraph of section 1, Article VIII of the Constitution states that:
Judicial power includes the duty of the courts of justice to settle actual controversies
involving rights which are legally demandable and enforceable, and to determine
whether or not there has been a grave abuse of discretion amounting to lack or
excess of jurisdiction on the part of any branch or instrumentality of the Government.
Commenting on this provision in his book, Philippine Political Law, 22 Mr. Justice Isagani A. Cruz, a
distinguished member of this Court, says:
The first part of the authority represents the traditional concept of judicial power,
involving the settlement of conflicting rights as conferred as law. The second part of
the authority represents a broadening of judicial power to enable the courts of justice
to review what was before forbidden territory, to wit, the discretion of the political
departments of the government.
As worded, the new provision vests in the judiciary, and particularly the Supreme
Court, the power to rule upon even the wisdom of the decisions of the executive and
the legislature and to declare their acts invalid for lack or excess of jurisdiction
because tainted with grave abuse of discretion. The catch, of course, is the meaning
of "grave abuse of discretion," which is a very elastic phrase that can expand or
contract according to the disposition of the judiciary.
In Daza vs. Singson, 23 Mr. Justice Cruz, now speaking for this Court, noted:
In the case now before us, the jurisdictional objection becomes even less tenable
and decisive. The reason is that, even if we were to assume that the issue presented
before us was political in nature, we would still not be precluded from revolving it
under the expanded jurisdiction conferred upon us that now covers, in proper cases,
even the political question. Article VII, Section 1, of the Constitution clearly provides: .
..
The last ground invoked by the trial court in dismissing the complaint is the non-impairment of
contracts clause found in the Constitution. The court a quo declared that:
The Court is likewise of the impression that it cannot, no matter how we stretch our
jurisdiction, grant the reliefs prayed for by the plaintiffs, i.e., to cancel all existing
timber license agreements in the country and to cease and desist from receiving,
accepting, processing, renewing or approving new timber license agreements. For to
do otherwise would amount to "impairment of contracts" abhored (sic) by the
fundamental law. 24
We are not persuaded at all; on the contrary, We are amazed, if not shocked, by such a sweeping
pronouncement. In the first place, the respondent Secretary did not, for obvious reasons, even
invoke in his motion to dismiss the non-impairment clause. If he had done so, he would have acted

with utmost infidelity to the Government by providing undue and unwarranted benefits and
advantages to the timber license holders because he would have forever bound the Government to
strictly respect the said licenses according to their terms and conditions regardless of changes in
policy and the demands of public interest and welfare. He was aware that as correctly pointed out by
the petitioners, into every timber license must be read Section 20 of the Forestry Reform Code (P.D.
No. 705) which provides:
. . . Provided, That when the national interest so requires, the President may amend,
modify, replace or rescind any contract, concession, permit, licenses or any other
form of privilege granted herein . . .
Needless to say, all licenses may thus be revoked or rescinded by executive action. It is not
a contract, property or a property right protested by the due process clause of the
Constitution. In Tan vs. Director of Forestry, 25 this Court held:
. . . A timber license is an instrument by which the State regulates the utilization and
disposition of forest resources to the end that public welfare is promoted. A timber
license is not a contract within the purview of the due process clause; it is only a
license or privilege, which can be validly withdrawn whenever dictated by public
interest or public welfare as in this case.
A license is merely a permit or privilege to do what otherwise would be unlawful, and
is not a contract between the authority, federal, state, or municipal, granting it and the
person to whom it is granted; neither is it property or a property right, nor does it
create a vested right; nor is it taxation (37 C.J. 168). Thus, this Court held that the
granting of license does not create irrevocable rights, neither is it property or property
rights (People vs. Ong Tin, 54 O.G. 7576).
We reiterated this pronouncement in Felipe Ysmael, Jr. & Co., Inc. vs. Deputy Executive Secretary: 26
. . . Timber licenses, permits and license agreements are the principal instruments by
which the State regulates the utilization and disposition of forest resources to the end
that public welfare is promoted. And it can hardly be gainsaid that they merely
evidence a privilege granted by the State to qualified entities, and do not vest in the
latter a permanent or irrevocable right to the particular concession area and the
forest products therein. They may be validly amended, modified, replaced or
rescinded by the Chief Executive when national interests so require. Thus, they are
not deemed contracts within the purview of the due process of law clause
[See Sections 3(ee) and 20 of Pres. Decree No. 705, as amended. Also, Tan v.
Director of Forestry, G.R. No. L-24548, October 27, 1983, 125 SCRA 302].
Since timber licenses are not contracts, the non-impairment clause, which reads:
Sec. 10. No law impairing, the obligation of contracts shall be passed.
cannot be invoked.

27

In the second place, even if it is to be assumed that the same are contracts, the instant case does
not involve a law or even an executive issuance declaring the cancellation or modification of existing
timber licenses. Hence, the non-impairment clause cannot as yet be invoked. Nevertheless, granting
further that a law has actually been passed mandating cancellations or modifications, the same
cannot still be stigmatized as a violation of the non-impairment clause. This is because by its very
nature and purpose, such as law could have only been passed in the exercise of the police power of
the state for the purpose of advancing the right of the people to a balanced and healthful ecology,
promoting their health and enhancing the general welfare. In Abe vs. Foster Wheeler
Corp. 28 this Court stated:
The freedom of contract, under our system of government, is not meant to be
absolute. The same is understood to be subject to reasonable legislative regulation
aimed at the promotion of public health, moral, safety and welfare. In other words,
the constitutional guaranty of non-impairment of obligations of contract is limited by
the exercise of the police power of the State, in the interest of public health, safety,
moral and general welfare.
The reason for this is emphatically set forth in Nebia vs. New York, 29 quoted in Philippine American
Life Insurance Co. vs. Auditor General, 30 to wit:
Under our form of government the use of property and the making of contracts are
normally matters of private and not of public concern. The general rule is that both
shall be free of governmental interference. But neither property rights nor contract
rights are absolute; for government cannot exist if the citizen may at will use his
property to the detriment of his fellows, or exercise his freedom of contract to work
them harm. Equally fundamental with the private right is that of the public to regulate
it in the common interest.
In short, the non-impairment clause must yield to the police power of the state.

31

Finally, it is difficult to imagine, as the trial court did, how the non-impairment clause could apply with
respect to the prayer to enjoin the respondent Secretary from receiving, accepting, processing,
renewing or approving new timber licenses for, save in cases of renewal, no contract would have as
of yet existed in the other instances. Moreover, with respect to renewal, the holder is not entitled to it
as a matter of right.
WHEREFORE, being impressed with merit, the instant Petition is hereby GRANTED, and the
challenged Order of respondent Judge of 18 July 1991 dismissing Civil Case No. 90-777 is hereby
set aside. The petitioners may therefore amend their complaint to implead as defendants the holders
or grantees of the questioned timber license agreements.
No pronouncement as to costs. SO ORDERED.
Republic of the Philippines
SUPREME COURT
Manila

EN BANC
G.R. No. 127882

December 1, 2004

LA BUGAL-B'LAAN TRIBAL ASSOCIATION, INC., Represented by its Chairman F'LONG


MIGUEL M. LUMAYONG; WIGBERTO E. TAADA; PONCIANO BENNAGEN; JAIME TADEO;
RENATO R. CONSTANTINO JR.; F'LONG AGUSTIN M. DABIE; ROBERTO P. AMLOY; RAQIM L.
DABIE; SIMEON H. DOLOJO; IMELDA M. GANDON; LENY B. GUSANAN; MARCELO L.
GUSANAN; QUINTOL A. LABUAYAN; LOMINGGES D. LAWAY; BENITA P. TACUAYAN; Minors
JOLY L. BUGOY, Represented by His Father UNDERO D. BUGOY and ROGER M. DADING;
Represented by His Father ANTONIO L. DADING; ROMY M. LAGARO, Represented by His
Father TOTING A. LAGARO; MIKENY JONG B. LUMAYONG, Represented by His Father
MIGUEL M. LUMAYONG; RENE T. MIGUEL, Represented by His Mother EDITHA T. MIGUEL;
ALDEMAR L. SAL, Represented by His Father DANNY M. SAL; DAISY RECARSE,
Represented by Her Mother LYDIA S. SANTOS; EDWARD M. EMUY; ALAN P. MAMPARAIR;
MARIO L. MANGCAL; ALDEN S. TUSAN; AMPARO S. YAP; VIRGILIO CULAR; MARVIC M.V.F.
LEONEN; JULIA REGINA CULAR, GIAN CARLO CULAR, VIRGILIO CULAR JR., Represented
by Their Father VIRGILIO CULAR; PAUL ANTONIO P. VILLAMOR, Represented by His Parents
JOSE VILLAMOR and ELIZABETH PUA-VILLAMOR; ANA GININA R. TALJA, Represented by
Her Father MARIO JOSE B. TALJA; SHARMAINE R. CUNANAN, Represented by Her Father
ALFREDO M. CUNANAN; ANTONIO JOSE A. VITUG III, Represented by His Mother ANNALIZA
A. VITUG, LEAN D. NARVADEZ, Represented by His Father MANUEL E. NARVADEZ JR.;
ROSERIO MARALAG LINGATING, Represented by Her Father RIO OLIMPIO A. LINGATING;
MARIO JOSE B. TALJA; DAVID E. DE VERA; MARIA MILAGROS L. SAN JOSE; Sr. SUSAN O.
BOLANIO, OND; LOLITA G. DEMONTEVERDE; BENJIE L. NEQUINTO;1 ROSE LILIA S.
ROMANO; ROBERTO S. VERZOLA; EDUARDO AURELIO C. REYES; LEAN LOUEL A. PERIA,
Represented by His Father ELPIDIO V. PERIA;2 GREEN FORUM PHILIPPINES; GREEN FORUM
WESTERN VISAYAS (GF-WV); ENVIRONMENTAL LEGAL ASSISTANCE CENTER (ELAC);
KAISAHAN TUNGO SA KAUNLARAN NG KANAYUNAN AT REPORMANG PANSAKAHAN
(KAISAHAN);3PARTNERSHIP FOR AGRARIAN REFORM and RURAL DEVELOPMENT
SERVICES, INC. (PARRDS); PHILIPPINE PARTNERSHIP FOR THE DEVELOPMENT OF HUMAN
RESOURCES IN THE RURAL AREAS, INC. (PHILDHRRA); WOMEN'S LEGAL BUREAU (WLB);
CENTER FOR ALTERNATIVE DEVELOPMENT INITIATIVES, INC. (CADI); UPLAND
DEVELOPMENT INSTITUTE (UDI); KINAIYAHAN FOUNDATION, INC.; SENTRO NG
ALTERNATIBONG LINGAP PANLIGAL (SALIGAN); and LEGAL RIGHTS AND NATURAL
RESOURCES CENTER, INC. (LRC), petitioners,
vs.
VICTOR O. RAMOS, Secretary, Department of Environment and Natural Resources (DENR);
HORACIO RAMOS, Director, Mines and Geosciences Bureau (MGB-DENR); RUBEN TORRES,
Executive Secretary; and WMC (PHILIPPINES), INC.,4 respondents.

RESOLUTION

PANGANIBAN, J.:

All mineral resources are owned by the State. Their exploration, development and utilization (EDU)
must always be subject to the full control and supervision of the State. More specifically, given the
inadequacy of Filipino capital and technology in large-scale EDU activities, the State may secure the
help of foreign companies in all relevant matters -- especially financial and technical assistance -provided that, at all times, the State maintains its right of full control. The foreign assistor or
contractor assumes all financial, technical and entrepreneurial risks in the EDU activities; hence, it
may be given reasonable management, operational, marketing, audit and other prerogatives to
protect its investments and to enable the business to succeed.
Full control is not anathematic to day-to-day management by the contractor, provided that the State
retains the power to direct overall strategy; and to set aside, reverse or modify plans and actions of
the contractor. The idea of full control is similar to that which is exercised by the board of directors of
a private corporation: the performance of managerial, operational, financial, marketing and other
functions may be delegated to subordinate officers or given to contractual entities, but the board
retains full residual control of the business.
Who or what organ of government actually exercises this power of control on behalf of the State?
The Constitution is crystal clear: the President. Indeed, the Chief Executive is the official
constitutionally mandated to "enter into agreements with foreign owned corporations." On the other
hand, Congress may review the action of the President once it is notified of "every contract entered
into in accordance with this [constitutional] provision within thirty days from its execution." In contrast
to this express mandate of the President and Congress in the EDU of natural resources, Article XII of
the Constitution is silent on the role of the judiciary. However, should the President and/or Congress
gravely abuse their discretion in this regard, the courts may -- in a proper case -- exercise their
residual duty under Article VIII. Clearly then, the judiciary should not inordinately interfere in the
exercise of this presidential power of control over the EDU of our natural resources.
The Constitution should be read in broad, life-giving strokes. It should not be used to strangulate
economic growth or to serve narrow, parochial interests. Rather, it should be construed to grant the
President and Congress sufficient discretion and reasonable leeway to enable them to attract foreign
investments and expertise, as well as to secure for our people and our posterity the blessings of
prosperity and peace.
On the basis of this control standard, this Court upholds the constitutionality of the Philippine Mining
Law, its Implementing Rules and Regulations -- insofar as they relate to financial and technical
agreements -- as well as the subject Financial and Technical Assistance Agreement (FTAA). 5
Background
The Petition for Prohibition and Mandamus before the Court challenges the constitutionality of (1)
Republic Act No. [RA] 7942 (The Philippine Mining Act of 1995); (2) its Implementing Rules and
Regulations (DENR Administrative Order No. [DAO] 96-40); and (3) the FTAA dated March 30,
1995,6 executed by the government with Western Mining Corporation (Philippines), Inc. (WMCP). 7
On January 27, 2004, the Court en banc promulgated its Decision8 granting the Petition and
declaring the unconstitutionality of certain provisions of RA 7942, DAO 96-40, as well as of the entire
FTAA executed between the government and WMCP, mainly on the finding that FTAAs are service
contracts prohibited by the 1987 Constitution.
The Decision struck down the subject FTAA for being similar to service contracts,9 which, though
permitted under the 1973 Constitution,10 were subsequently denounced for being antithetical to the

principle of sovereignty over our natural resources, because they allowed foreign control over the
exploitation of our natural resources, to the prejudice of the Filipino nation.
The Decision quoted several legal scholars and authors who had criticized service contracts
for, inter alia, vesting in the foreign contractor exclusive management and control of the enterprise,
including operation of the field in the event petroleum was discovered; control of production,
expansion and development; nearly unfettered control over the disposition and sale of the products
discovered/extracted; effective ownership of the natural resource at the point of extraction; and
beneficial ownership of our economic resources. According to the Decision, the 1987 Constitution
(Section 2 of Article XII) effectively banned such service contracts.
Subsequently, respondents filed separate Motions for Reconsideration. In a Resolution dated March
9, 2004, the Court required petitioners to comment thereon. In the Resolution of June 8, 2004, it set
the case for Oral Argument on June 29, 2004.
After hearing the opposing sides, the Court required the parties to submit their respective
Memoranda in amplification of their arguments. In a Resolution issued later the same day, June 29,
2004, the Court noted, inter alia, the Manifestation and Motion (in lieu of comment) filed by the Office
of the Solicitor General (OSG) on behalf of public respondents. The OSG said that it was not
interposing any objection to the Motion for Intervention filed by the Chamber of Mines of the
Philippines, Inc. (CMP) and was in fact joining and adopting the latter's Motion for Reconsideration.

Memoranda were accordingly filed by the intervenor as well as by petitioners, public respondents,
and private respondent, dwelling at length on the three issues discussed below. Later, WMCP
submitted its Reply Memorandum, while the OSG -- in obedience to an Order of this Court -- filed a
Compliance submitting copies of more FTAAs entered into by the government.
Three Issues Identified by the Court
During the Oral Argument, the Court identified the three issues to be resolved in the present
controversy, as follows:
1. Has the case been rendered moot by the sale of WMC shares in WMCP to Sagittarius (60 percent
of Sagittarius' equity is owned by Filipinos and/or Filipino-owned corporations while 40 percent is
owned by Indophil Resources NL, an Australian company) and by the subsequent transfer and
registration of the FTAA from WMCP to Sagittarius?
2. Assuming that the case has been rendered moot, would it still be proper to resolve the
constitutionality of the assailed provisions of the Mining Law, DAO 96-40 and the WMCP FTAA?
3. What is the proper interpretation of the phrase Agreements Involving Either Technical or Financial
Assistancecontained in paragraph 4 of Section 2 of Article XII of the Constitution?
Should the Motion for Reconsideration Be Granted?
Respondents' and intervenor's Motions for Reconsideration should be granted, for the reasons
discussed below. The foregoing three issues identified by the Court shall now be taken up seriatim.
First Issue:

Mootness
In declaring unconstitutional certain provisions of RA 7942, DAO 96-40, and the WMCP FTAA, the
majority Decision agreed with petitioners' contention that the subject FTAA had been executed in
violation of Section 2 of Article XII of the 1987 Constitution. According to petitioners, the FTAAs
entered into by the government with foreign-owned corporations are limited by the fourth paragraph
of the said provision to agreements involving only technical or financial assistance for large-scale
exploration, development and utilization of minerals, petroleum and other mineral oils. Furthermore,
the foreign contractor is allegedly permitted by the FTAA in question to fully manage and control the
mining operations and, therefore, to acquire "beneficial ownership" of our mineral resources.
The Decision merely shrugged off the Manifestation by WMPC informing the Court (1) that on
January 23, 2001, WMC had sold all its shares in WMCP to Sagittarius Mines, Inc., 60 percent of
whose equity was held by Filipinos; and (2) that the assailed FTAA had likewise been transferred
from WMCP to Sagittarius.11 The ponenciadeclared that the instant case had not been rendered
moot by the transfer and registration of the FTAA to a Filipino-owned corporation, and that the
validity of the said transfer remained in dispute and awaited final judicial determination. 12 Patently
therefore, the Decision is anchored on the assumption that WMCP had remained
aforeign corporation.
The crux of this issue of mootness is the fact that WMCP, at the time it entered into the
FTAA, happened to be wholly owned by WMC Resources International Pty., Ltd. (WMC), which in
turn was a wholly owned subsidiary of Western Mining Corporation Holdings Ltd., a publicly listed
major Australian mining and exploration company.
The nullity of the FTAA was obviously premised upon the contractor being a foreign corporation.
Had the FTAA been originally issued to a Filipino-owned corporation, there would have been no
constitutionality issue to speak of. Upon the other hand, the conveyance of the WMCP FTAA to a
Filipino corporation can be likened to the sale of land to a foreigner who subsequently acquires
Filipino citizenship, or who later resells the same land to a Filipino citizen. The conveyance would be
validated, as the property in question would no longer be owned by a disqualified vendee.
And, inasmuch as the FTAA is to be implemented now by a Filipino corporation, it is no longer
possible for the Court to declare it unconstitutional. The case pending in the Court of Appeals is a
dispute between two Filipino companies (Sagittarius and Lepanto), both claiming the right to
purchase the foreign shares in WMCP. So, regardless of which side eventually wins, the FTAA would
still be in the hands of a qualified Filipino company. Considering that there is no longer any
justiciable controversy, the plea to nullify the Mining Law has become a virtual petition for declaratory
relief, over which this Court has no original jurisdiction.
In their Final Memorandum, however, petitioners argue that the case has not become moot,
considering the invalidity of the alleged sale of the shares in WMCP from WMC to Sagittarius, and of
the transfer of the FTAA from WMCP to Sagittarius, resulting in the change of contractor in the FTAA
in question. And even assuming that the said transfers were valid, there still exists an actual case
predicated on the invalidity of RA 7942 and its Implementing Rules and Regulations (DAO 96-40).
Presently, we shall discuss petitioners' objections to the transfer of both the shares and the
FTAA. We shall take up the alleged invalidity of RA 7942 and DAO 96-40 later on in the discussion
of the third issue.
No Transgression of the Constitution
by the Transfer of the WMCP Shares

Petitioners claim, first, that the alleged invalidity of the transfer of the WMCP shares to Sagittarius
violates the fourth paragraph of Section 2 of Article XII of the Constitution; second, that it is contrary
to the provisions of the WMCP FTAA itself; and third, that the sale of the shares is suspect and
should therefore be the subject of a case in which its validity may properly be litigated.
On the first ground, petitioners assert that paragraph 4 of Section 2 of Article XII permits the
government to enter into FTAAs only with foreign-owned corporations. Petitioners insist that the first
paragraph of this constitutional provision limits the participation of Filipino corporations in the
exploration, development and utilization of natural resources to only three species of contracts -production sharing, co-production and joint venture -- to the exclusion of all other arrangements or
variations thereof, and the WMCP FTAA may therefore not be validly assumed and implemented by
Sagittarius. In short, petitioners claim that a Filipino corporation is not allowed by the Constitution to
enter into an FTAA with the government.
However, a textual analysis of the first paragraph of Section 2 of Article XII does not support
petitioners' argument. The pertinent part of the said provision states: "Sec. 2. x x x The exploration,
development and utilization of natural resources shall be under the full control and supervision of the
State. The State may directly undertake such activities, or it may enter into co-production, joint
venture, or production-sharing agreements with Filipino citizens, or corporations or associations at
least sixty per centum of whose capital is owned by such citizens. x x x." Nowhere in the provision is
there any express limitation or restriction insofar as arrangements other than the three
aforementioned contractual schemes are concerned.
Neither can one reasonably discern any implied stricture to that effect. Besides, there is no basis to
believe that the framers of the Constitution, a majority of whom were obviously concerned with
furthering the development and utilization of the country's natural resources, could have wanted to
restrict Filipino participation in that area. This point is clear, especially in the light of the overarching
constitutional principle of giving preference and priority to Filipinos and Filipino corporations in the
development of our natural resources.
Besides, even assuming (purely for argument's sake) that a constitutional limitation barring Filipino
corporations from holding and implementing an FTAA actually exists, nevertheless, such provision
would apply only to the transfer of the FTAA to Sagittarius, but definitely not to the sale of WMC's
equity stake in WMCP to Sagittarius. Otherwise, an unreasonable curtailment of property rights
without due process of law would ensue. Petitioners' argument must therefore fail.
FTAA Not Intended
Solely for Foreign Corporation
Equally barren of merit is the second ground cited by petitioners -- that the FTAA was intended to
apply solely to a foreign corporation, as can allegedly be seen from the provisions therein. They
manage to cite only one WMCP FTAA provision that can be regarded as clearly intended to apply
only to a foreign contractor: Section 12, which provides for international commercial arbitration under
the auspices of the International Chamber of Commerce, after local remedies are exhausted. This
provision, however, does not necessarily imply that the WMCP FTAA cannot be transferred to and
assumed by a Filipino corporation like Sagittarius, in which event the said provision should simply be
disregarded as a superfluity.
No Need for a Separate
Litigation of the Sale of Shares

Petitioners claim as third ground the "suspicious" sale of shares from WMC to Sagittarius; hence, the
need to litigate it in a separate case. Section 40 of RA 7942 (the Mining Law) allegedly requires the
President's prior approval of a transfer.
A re-reading of the said provision, however, leads to a different conclusion. "Sec.
40. Assignment/Transfer -- A financial or technical assistance agreement may be assigned or
transferred, in whole or in part, to a qualified person subject to the prior approval of the President:
Provided, That the President shall notify Congress of every financial or technical assistance
agreement assigned or converted in accordance with this provision within thirty (30) days from the
date of the approval thereof."
Section 40 expressly applies to the assignment or transfer of the FTAA, not to the sale and transfer
of shares of stock in WMCP. Moreover, when the transferee of an FTAA is
another foreign corporation, there is a logical application of the requirement of prior approval by the
President of the Republic and notification to Congress in the event of assignment or transfer of an
FTAA. In this situation, such approval and notification are appropriate safeguards, considering that
the new contractor is the subject of a foreign government.
On the other hand, when the transferee of the FTAA happens to be a Filipino corporation, the need
for such safeguard is not critical; hence, the lack of prior approval and notification may not be
deemed fatal as to render the transfer invalid. Besides, it is not as if approval by the President is
entirely absent in this instance. As pointed out by private respondent in its Memorandum,13 the issue
of approval is the subject of one of the cases brought by Lepanto against Sagittarius in GR No.
162331. That case involved the review of the Decision of the Court of Appeals dated November 21,
2003 in CA-GR SP No. 74161, which affirmed the DENR Order dated December 31, 2001 and the
Decision of the Office of the President dated July 23, 2002, both approving the assignment of the
WMCP FTAA to Sagittarius.
Petitioners also question the sale price and the financial capacity of the transferee. According to the
Deed of Absolute Sale dated January 23, 2001, executed between WMC and Sagittarius, the price
of the WMCP shares was fixed at US$9,875,000, equivalent to P553 million at an exchange rate of
56:1. Sagittarius had an authorized capital stock of P250 million and a paid up capital of P60 million.
Therefore, at the time of approval of the sale by the DENR, the debt-to-equity ratio of the transferee
was over 9:1 -- hardly ideal for an FTAA contractor, according to petitioners.
However, private respondents counter that the Deed of Sale specifically provides that the payment of
the purchase price would take place only after Sagittarius' commencement of commercial production
from mining operations, if at all. Consequently, under the circumstances, we believe it would not be
reasonable to conclude, as petitioners did, that the transferee's high debt-to-equity ratio per se
necessarily carried negative implications for the enterprise; and it would certainly be improper to
invalidate the sale on that basis, as petitioners propose.
FTAA Not Void,
Thus Transferrable
To bolster further their claim that the case is not moot, petitioners insist that the FTAA is void and,
hence cannot be transferred; and that its transfer does not operate to cure the constitutional infirmity
that is inherent in it; neither will a change in the circumstances of one of the parties serve to ratify the
void contract.
While the discussion in their Final Memorandum was skimpy, petitioners in their Comment (on the
MR) did ratiocinate that this Court had declared the FTAA to be void because, at the time it was

executed with WMCP, the latter was a fully foreign-owned corporation, in which the former vested full
control and management with respect to the exploration, development and utilization of mineral
resources, contrary to the provisions of paragraph 4 of Section 2 of Article XII of the Constitution.
And since the FTAA was per se void, no valid right could be transferred; neither could it be ratified,
so petitioners conclude.
Petitioners have assumed as fact that which has yet to be established. First and foremost, the
Decision of this Court declaring the FTAA void has not yet become final. That was precisely the
reason the Court still heard Oral Argument in this case. Second, the FTAA does not vest in the
foreign corporation full control and supervision over the exploration, development and utilization of
mineral resources, to the exclusion of the government. This point will be dealt with in greater detail
below; but for now, suffice it to say that a perusal of the FTAA provisions will prove that the
government has effective overall direction and control of the mining operations, including marketing
and product pricing, and that the contractor's work programs and budgets are subject to its review
and approval or disapproval.
As will be detailed later on, the government does not have to micro-manage the mining operations
and dip its hands into the day-to-day management of the enterprise in order to be considered as
having overall control and direction. Besides, for practical and pragmatic reasons, there is a need for
government agencies to delegate certain aspects of the management work to the contractor. Thus
the basis for declaring the FTAA void still has to be revisited, reexamined and reconsidered.
Petitioners sniff at the citation of Chavez v. Public Estates Authority,14 and Halili v. CA,15 claiming that
the doctrines in these cases are wholly inapplicable to the instant case.
Chavez clearly teaches: "Thus, the Court has ruled consistently that where a Filipino citizen sells
land to an alien who later sells the land to a Filipino, the invalidity of the first transfer is corrected by
the subsequent sale to a citizen. Similarly, where the alien who buys the land subsequently acquires
Philippine citizenship, the sale is validated since the purpose of the constitutional ban to limit land
ownership to Filipinos has been achieved. In short, the law disregards the constitutional
disqualification of the buyer to hold land if the land is subsequently transferred to a qualified party, or
the buyer himself becomes a qualified party."16
In their Comment, petitioners contend that in Chavez and Halili, the object of the transfer (the land)
was not what was assailed for alleged unconstitutionality. Rather, it was the transaction that was
assailed; hence subsequent compliance with constitutional provisions would cure its infirmity. In
contrast, in the instant case it is the FTAA itself, the object of the transfer, that is being assailed as
invalid and unconstitutional. So, petitioners claim that the subsequent transfer of a void FTAA to a
Filipino corporation would not cure the defect.
Petitioners are confusing themselves. The present Petition has been filed, precisely because the
grantee of the FTAA was a wholly owned subsidiary of a foreign corporation. It cannot be gainsaid
that anyone would have asserted that the same FTAA was void if it had at the outset been issued to
a Filipino corporation. The FTAA, therefore, is not per se defective or unconstitutional. It was
questioned only because it had been issued to an allegedly non-qualified, foreign-owned
corporation.
We believe that this case is clearly analogous to Halili, in which the land acquired by a non-Filipino
was re-conveyed to a qualified vendee and the original transaction was thereby cured.
Paraphrasing Halili, the same rationale applies to the instant case: assuming arguendo the invalidity
of its prior grant to a foreign corporation, the disputed FTAA -- being now held by a Filipino
corporation -- can no longer be assailed; the objective of the constitutional provision -- to keep the

exploration, development and utilization of our natural resources in Filipino hands -- has been
served.
More accurately speaking, the present situation is one degree better than that obtaining in Halili, in
which the original sale to a non-Filipino was clearly and indisputably violative of the constitutional
prohibition and thus voidab initio. In the present case, the issuance/grant of the subject FTAA to the
then foreign-owned WMCP was notillegal, void or unconstitutional at the time. The matter had to be
brought to court, precisely for adjudication as to whether the FTAA and the Mining Law had indeed
violated the Constitution. Since, up to this point, the decision of this Court declaring the FTAA void
has yet to become final, to all intents and purposes, the FTAA must be deemed valid and
constitutional.17
At bottom, we find completely outlandish petitioners' contention that an FTAA could be entered into
by the government only with a foreign corporation, never with a Filipino enterprise. Indeed, the
nationalistic provisions of the Constitution are all anchored on the protection of Filipino interests.
How petitioners can now argue that foreigners have the exclusive right to FTAAs totally overturns the
entire basis of the Petition -- preference for the Filipino in the exploration, development and
utilization of our natural resources. It does not take deep knowledge of law and logic to understand
that what the Constitution grants to foreigners should be equally available to Filipinos.
Second Issue:
Whether the Court Can Still Decide the Case,
Even Assuming It Is Moot
All the protagonists are in agreement that the Court has jurisdiction to decide this controversy, even
assuming it to be moot.
Petitioners stress the following points. First, while a case becomes moot and academic when "there
is no more actual controversy between the parties or no useful purpose can be served in passing
upon the merits,"18 what is at issue in the instant case is not only the validity of the WMCP FTAA, but
also the constitutionality of RA 7942 and its Implementing Rules and Regulations. Second, the acts
of private respondent cannot operate to cure the law of its alleged unconstitutionality or to divest this
Court of its jurisdiction to decide. Third, the Constitution imposes upon the Supreme Court the duty
to declare invalid any law that offends the Constitution.
Petitioners also argue that no amendatory laws have been passed to make the Mining Act of 1995
conform to constitutional strictures (assuming that, at present, it does not); that public respondents
will continue to implement and enforce the statute until this Court rules otherwise; and that the said
law continues to be the source of legal authority in accepting, processing and approving numerous
applications for mining rights.
Indeed, it appears that as of June 30, 2002, some 43 FTAA applications had been filed with the
Mines and Geosciences Bureau (MGB), with an aggregate area of 2,064,908.65 hectares -- spread
over Luzon, the Visayas and Mindanao19 -- applied for. It may be a bit far-fetched to assert, as
petitioners do, that each and every FTAA that was entered into under the provisions of the Mining
Act "invites potential litigation" for as long as the constitutional issues are not resolved with finality.
Nevertheless, we must concede that there exists the distinct possibility that one or more of the future
FTAAs will be the subject of yet another suit grounded on constitutional issues.
But of equal if not greater significance is the cloud of uncertainty hanging over the mining industry,
which is even now scaring away foreign investments. Attesting to this climate of anxiety is the fact

that the Chamber of Mines of the Philippines saw the urgent need to intervene in the case and to
present its position during the Oral Argument; and that Secretary General Romulo Neri of the
National Economic Development Authority (NEDA) requested this Court to allow him to speak,
during that Oral Argument, on the economic consequences of the Decision of January 27, 2004. 20
We are convinced. We now agree that the Court must recognize the exceptional character of the
situation and the paramount public interest involved, as well as the necessity for a ruling to put an
end to the uncertainties plaguing the mining industry and the affected communities as a result of
doubts cast upon the constitutionality and validity of the Mining Act, the subject FTAA and future
FTAAs, and the need to avert a multiplicity of suits. ParaphrasingGonzales v. Commission on
Elections,21 it is evident that strong reasons of public policy demand that the constitutionality issue be
resolved now.22
In further support of the immediate resolution of the constitutionality issue, public respondents
cite Acop v. Guingona,23 to the effect that the courts will decide a question -- otherwise moot and
academic -- if it is "capable of repetition, yet evading review."24 Public respondents ask the Court to
avoid a situation in which the constitutionality issue may again arise with respect to another FTAA,
the resolution of which may not be achieved until after it has become too late for our mining industry
to grow out of its infancy. They also recall Salonga v. Cruz Pao,25 in which this Court declared
that "(t)he Court also has the duty to formulate guiding and controlling constitutional principles,
precepts, doctrines or rules. It has the symbolic function of educating the bench and bar on the
extent of protection given by constitutional guarantees. x x x."
The mootness of the case in relation to the WMCP FTAA led the undersigned ponente to state in his
dissent to the Decision that there was no more justiciable controversy and the plea to nullify the
Mining Law has become a virtual petition for declaratory relief.26 The entry of the Chamber of Mines
of the Philippines, Inc., however, has put into focus the seriousness of the allegations of
unconstitutionality of RA 7942 and DAO 96-40 which converts the case to one for prohibition 27 in the
enforcement of the said law and regulations.
Indeed, this CMP entry brings to fore that the real issue in this case is whether paragraph 4 of
Section 2 of Article XII of the Constitution is contravened by RA 7942 and DAO 96-40, not whether it
was violated by specific acts implementing RA 7942 and DAO 96-40. "[W]hen an act of the
legislative department is seriously alleged to have infringed the Constitution, settling the controversy
becomes the duty of this Court. By the mere enactment of the questioned law or the approval of the
challenged action, the dispute is said to have ripened into a judicial controversy even without any
other overt act."28 This ruling can be traced from Taada v. Angara,29 in which the Court said:
"In seeking to nullify an act of the Philippine Senate on the ground that it contravenes the
Constitution, the petition no doubt raises a justiciable controversy. Where an action of the
legislative branch is seriously alleged to have infringed the Constitution, it becomes not only
the right but in fact the duty of the judiciary to settle the dispute.
xxxxxxxxx
"As this Court has repeatedly and firmly emphasized in many cases, it will not shirk, digress
from or abandon its sacred duty and authority to uphold the Constitution in matters that
involve grave abuse of discretion brought before it in appropriate cases, committed by any
officer, agency, instrumentality or department of the government." 30
Additionally, the entry of CMP into this case has also effectively forestalled any possible objections
arising from the standing or legal interest of the original parties.

For all the foregoing reasons, we believe that the Court should proceed to a resolution of the
constitutional issues in this case.
Third Issue:
The Proper Interpretation of the Constitutional Phrase
"Agreements Involving Either Technical or Financial Assistance"
The constitutional provision at the nucleus of the controversy is paragraph 4 of Section 2 of Article
XII of the 1987 Constitution. In order to appreciate its context, Section 2 is reproduced in full:
"Sec. 2. All lands of the public domain, waters, minerals, coal, petroleum, and other mineral
oils, all forces of potential energy, fisheries, forests or timber, wildlife, flora and fauna, and
other natural resources are owned by the State. With the exception of agricultural lands, all
other natural resources shall not be alienated. The exploration, development and utilization
of natural resources shall be under the full control and supervision of the State. The State
may directly undertake such activities, or it may enter into co-production, joint venture or
production-sharing agreements with Filipino citizens or corporations or associations at least
sixty per centum of whose capital is owned by such citizens. Such agreements may be for a
period not exceeding twenty-five years, renewable for not more than twenty-five years, and
under such terms and conditions as may be provided by law. In cases of water rights for
irrigation, water supply, fisheries, or industrial uses other than the development of water
power, beneficial use may be the measure and limit of the grant.
"The State shall protect the nation's marine wealth in its archipelagic waters, territorial sea,
and exclusive economic zone, and reserve its use and enjoyment exclusively to Filipino
citizens.
"The Congress may, by law, allow small-scale utilization of natural resources by Filipino
citizens, as well as cooperative fish farming, with priority to subsistence fishermen and fishworkers in rivers, lakes, bays and lagoons.
"The President may enter into agreements with foreign-owned corporations involving
either technical or financial assistance for large-scale exploration, development, and
utilization of minerals, petroleum, and other mineral oils according to the general terms
and conditions provided by law, based on real contributions to the economic growth and
general welfare of the country. In such agreements, the State shall promote the development
and use of local scientific and technical resources.
"The President shall notify the Congress of every contract entered into in accordance with
this provision, within thirty days from its execution." 31
No Restriction of Meaning by
a Verba Legis Interpretation
To interpret the foregoing provision, petitioners adamantly assert that the language of the
Constitution should prevail; that the primary method of interpreting it is to seek the ordinary meaning
of the words used in its provisions. They rely on rulings of this Court, such as the following:
"The fundamental principle in constitutional construction however is that the primary source
from which to ascertain constitutional intent or purpose is the language of the provision itself.

The presumption is that the words in which the constitutional provisions are couched
express the objective sought to be attained. In other words, verba legis prevails. Only when
the meaning of the words used is unclear and equivocal should resort be made to
extraneous aids of construction and interpretation, such as the proceedings of the
Constitutional Commission or Convention to shed light on and ascertain the true intent or
purpose of the provision being construed."32
Very recently, in Francisco v. The House of Representatives,33 this Court indeed had the occasion to
reiterate the well-settled principles of constitutional construction:
"First, verba legis, that is, wherever possible, the words used in the Constitution must be
given theirordinary meaning except where technical terms are employed. x x x.
xxxxxxxxx
"Second, where there is ambiguity, ratio legis est anima. The words of the Constitution
should be interpreted in accordance with the intent of its framers. x x x.
xxxxxxxxx
"Finally, ut magis valeat quam pereat. The Constitution is to be interpreted as a whole."34
For ease of reference and in consonance with verba legis, we reconstruct and stratify the
aforequoted Section 2 as follows:
1. All natural resources are owned by the State. Except for agricultural lands, natural
resources cannot be alienated by the State.
2. The exploration, development and utilization (EDU) of natural resources shall be under the
full control and supervision of the State.
3. The State may undertake these EDU activities through either of the following:
(a) By itself directly and solely
(b) By (i) co-production; (ii) joint venture; or (iii) production sharing agreements with
Filipino citizens or corporations, at least 60 percent of the capital of which is owned
by such citizens
4. Small-scale utilization of natural resources may be allowed by law in favor of Filipino
citizens.
5. For large-scale EDU of minerals, petroleum and other mineral oils, the President may
enter into "agreements with foreign-owned corporations involving either technical or financial
assistance according to the general terms and conditions provided by law x x x."
Note that in all the three foregoing mining activities -- exploration, development and utilization -- the
State may undertake such EDU activities by itself or in tandem with Filipinos or Filipino corporations,
except in two instances:first, in small-scale utilization of natural resources, which Filipinos may be
allowed by law to undertake; andsecond, in large-scale EDU of minerals, petroleum and mineral oils,

which may be undertaken by the State via "agreements with foreign-owned corporations involving
either technical or financial assistance" as provided by law.
Petitioners claim that the phrase "agreements x x x involving either technical or financial
assistance" simply means technical assistance or financial assistance agreements, nothing more
and nothing else. They insist that there is no ambiguity in the phrase, and that a plain reading of
paragraph 4 quoted above leads to the inescapable conclusion that what a foreign-owned
corporation may enter into with the government is merely an agreement
for either financial or technical assistance only, for the large-scale exploration, development and
utilization of minerals, petroleum and other mineral oils; such a limitation, they argue, excludes
foreign management and operation of a mining enterprise. 35
This restrictive interpretation, petitioners believe, is in line with the general policy enunciated by the
Constitution reserving to Filipino citizens and corporations the use and enjoyment of the country's
natural resources. They maintain that this Court's Decision36 of January 27, 2004 correctly declared
the WMCP FTAA, along with pertinent provisions of RA 7942, void for allowing a foreign contractor to
have direct and exclusive management of a mining enterprise. Allowing such a privilege not only
runs counter to the "full control and supervision" that the State is constitutionally mandated to
exercise over the exploration, development and utilization of the country's natural resources; doing
so also vests in the foreign company "beneficial ownership" of our mineral resources. It will be
recalled that the Decision of January 27, 2004 zeroed in on "management or other forms of
assistance" or other activities associated with the "service contracts" of the martial law regime,
since "the management or operation of mining activities by foreign contractors, which is the primary
feature of service contracts, was precisely the evil that the drafters of the 1987 Constitution sought
to eradicate."
On the other hand, the intervenor37 and public respondents argue that the FTAA allowed by
paragraph 4 is not merely an agreement for supplying limited and specific financial or technical
services to the State. Rather, such FTAA is a comprehensive agreement for the foreign-owned
corporation's integrated exploration, development and utilization of mineral, petroleum or other
mineral oils on a large-scale basis. The agreement, therefore, authorizes the foreign contractor's
rendition of a whole range of integrated and comprehensive services, ranging from the discovery to
the development, utilization and production of minerals or petroleum products.
We do not see how applying a strictly literal or verba legis interpretation of paragraph 4 could
inexorably lead to the conclusions arrived at in the ponencia. First, the drafters' choice of words -their use of the phraseagreements x x x involving either technical or financial assistance -- does not
indicate the intent to exclude other modes of assistance. The drafters opted to use involving when
they could have simply said agreements forfinancial or technical assistance, if that was their
intention to begin with. In this case, the limitation would be very clear and no further debate would
ensue.
In contrast, the use of the word "involving" signifies the possibility of the inclusion of other forms
of assistance or activities having to do with, otherwise related to or compatible with financial or
technical assistance. The word "involving" as used in this context has three connotations that can be
differentiated thus:one, the sense of "concerning," "having to do with," or "affecting"; two, "entailing,"
"requiring," "implying" or "necessitating"; and three, "including," "containing" or "comprising."38
Plainly, none of the three connotations convey a sense of exclusivity. Moreover, the word "involving,"
when understood in the sense of "including," as in including technical or financial
assistance, necessarily implies that there are activities other than those that are being included. In

other words, if an agreement includes technical or financial assistance, there is apart from such
assistance -- something else already in, and covered or may be covered by, the said agreement.
In short, it allows for the possibility that matters, other than those explicitly mentioned, could be
made part of the agreement. Thus, we are now led to the conclusion that the use of the word
"involving" implies that these agreements with foreign corporations are not limited to mere financial
or technical assistance. The difference in sense becomes very apparent when we juxtapose
"agreements for technical or financial assistance" against "agreements including technical or
financial assistance." This much is unalterably clear in a verba legisapproach.
Second, if the real intention of the drafters was to confine foreign corporations to financial or
technical assistance and nothing more, their language would have certainly been so unmistakably
restrictive and stringent as to leave no doubt in anyone's mind about their true intent. For example,
they would have used the sentence foreign corporations are absolutely prohibited from
involvement in the management or operation of mining or similar ventures or words of similar import.
A search for such stringent wording yields negative results. Thus, we come to the inevitable
conclusion that there was a conscious and deliberate decision to avoid the use of restrictive
wording that bespeaks an intent not to use the expression "agreements x x x involving either
technical or financial assistance" in an exclusionary and limiting manner.
Deletion of "Service Contracts" to
Avoid Pitfalls of Previous Constitutions,
Not to Ban Service Contracts Per Se
Third, we do not see how a verba legis approach leads to the conclusion that "the management or
operation of mining activities by foreign contractors, which is the primary feature of service
contracts, was precisely the evil that the drafters of the 1987 Constitution sought to
eradicate." Nowhere in the above-quoted Section can be discerned the objective to keep out of
foreign hands the management or operation of mining activities or the plan to eradicate service
contracts as these were understood in the 1973 Constitution. Still, petitioners maintain that the
deletion or omission from the 1987 Constitution of the term "service contracts" found in the 1973
Constitution sufficiently proves the drafters' intent to exclude foreigners from the management of the
affected enterprises.
To our mind, however, such intent cannot be definitively and conclusively established from the mere
failure to carry the same expression or term over to the new Constitution, absent a more specific,
explicit and unequivocal statement to that effect. What petitioners seek (a complete ban on foreign
participation in the management of mining operations, as previously allowed by the earlier
Constitutions) is nothing short of bringing about a momentous sea change in the economic and
developmental policies; and the fundamentally capitalist, free-enterprise philosophy of our
government. We cannot imagine such a radical shift being undertaken by our government, to the
great prejudice of the mining sector in particular and our economy in general, merely on the basis of
the omission of the terms service contract from or the failure to carry them over to the new
Constitution. There has to be a much more definite and even unarguable basis for such a drastic
reversal of policies.
Fourth, a literal and restrictive interpretation of paragraph 4, such as that proposed by petitioners,
suffers from certain internal logical inconsistencies that generate ambiguities in the understanding of
the provision. As the intervenor pointed out, there has never been any constitutional or statutory
provision that reserved to Filipino citizens or corporations, at least 60 percent of which is Filipinoowned, the rendition of financial or technical assistance to companies engaged in mining or the
development of any other natural resource. The taking out of foreign-currency or peso-denominated

loans or any other kind of financial assistance, as well as the rendition of technical assistance -whether to the State or to any other entity in the Philippines -- has never been restricted in favor of
Filipino citizens or corporations having a certain minimum percentage of Filipino equity. Such a
restriction would certainly be preposterous and unnecessary. As a matter of fact, financial, and even
technical assistance,regardless of the nationality of its source, would be welcomed in the mining
industry anytime with open arms, on account of the dearth of local capital and the need to continually
update technological know-how and improve technical skills.
There was therefore no need for a constitutional provision specifically allowing foreign-owned
corporations to render financial or technical assistance, whether in respect of mining or some other
resource development or commercial activity in the Philippines. The last point needs to be
emphasized: if merely financial or technical assistance agreements are allowed, there would
be no need to limit them to large-scale mining operations, as there would be far greater need
for them in the smaller-scale mining activities (and even in non-mining areas). Obviously, the
provision in question was intended to refer to agreements other than those for mere financial
or technical assistance.
In like manner, there would be no need to require the President of the Republic to report to
Congress, if only financial or technical assistance agreements are involved. Such agreements are in
the nature of foreign loans that -- pursuant to Section 20 of Article VII 39 of the 1987 Constitution -- the
President may contract or guarantee, merely with the prior concurrence of the Monetary Board. In
turn, the Board is required to report to Congresswithin thirty days from the end of every quarter of
the calendar year, not thirty days after the agreement is entered into.
And if paragraph 4 permits only agreements for loans and other forms of financial, or technical
assistance, what is the point of requiring that they be based on real contributions to the economic
growth and general welfare of the country? For instance, how is one to measure and assess the
"real contributions" to the "economic growth" and "general welfare" of the country that may ensue
from a foreign-currency loan agreement or a technical-assistance agreement for, say, the
refurbishing of an existing power generating plant for a mining operation somewhere in Mindanao?
Such a criterion would make more sense when applied to a major business investment in a principal
sector of the industry.
The conclusion is clear and inescapable -- a verba legis construction shows that paragraph 4 is not
to be understood as one limited only to foreign loans (or other forms of financial support) and to
technical assistance. There is definitely more to it than that. These are provisions permitting
participation by foreign companies; requiring the President's report to Congress; and using,
as yardstick, contributions based on economic growth and general welfare. These were
neither accidentally inserted into the Constitution nor carelessly cobbled together by the
drafters in lip service to shallow nationalism. The provisions patently have significance and
usefulness in a context that allows agreements with foreign companies to include more than mere
financial or technical assistance.
Fifth, it is argued that Section 2 of Article XII authorizes nothing more than a rendition of specific and
limited financial service or technical assistance by a foreign company. This argument begs the
question "To whom or for whom would it be rendered"? or Who is being assisted? If the answer is
"The State," then it necessarily implies that the State itself is the one directly and solely undertaking
the large-scale exploration, development and utilization of a mineral resource, so it follows that the
State must itself bear the liability and cost of repaying the financing sourced from the foreign lender
and/or of paying compensation to the foreign entity rendering technical assistance.

However, it is of common knowledge, and of judicial notice as well, that the government is and has
for many many years been financially strapped, to the point that even the most essential services
have suffered serious curtailments -- education and health care, for instance, not to mention judicial
services -- have had to make do with inadequate budgetary allocations. Thus, government has had
to resort to build-operate-transfer and similar arrangements with the private sector, in order to get
vital infrastructure projects built without any governmental outlay.
The very recent brouhaha over the gargantuan "fiscal crisis" or "budget deficit" merely confirms what
the ordinary citizen has suspected all along. After the reality check, one will have to admit the
implausibility of a direct undertaking -- by the State itself -- of large-scale exploration, development
and utilization of minerals, petroleum and other mineral oils. Such an undertaking entails not only
humongous capital requirements, but also the attendant risk of never finding and developing
economically viable quantities of minerals, petroleum and other mineral oils. 40
It is equally difficult to imagine that such a provision restricting foreign companies to the rendition of
only financial or technical assistance to the government was deliberately crafted by the drafters of
the Constitution, who were all well aware of the capital-intensive and technology-oriented nature of
large-scale mineral or petroleum extraction and the country's deficiency in precisely those areas. 41 To
say so would be tantamount to asserting that the provision was purposely designed to ladle the
large-scale development and utilization of mineral, petroleum and related resources with impossible
conditions; and to remain forever and permanently "reserved" for future generations of Filipinos.
A More Reasonable Look
at the Charter's Plain Language
Sixth, we shall now look closer at the plain language of the Charter and examining the logical
inferences. The drafters chose to emphasize and highlight agreements x x x involving either
technical or financial assistance in relation to foreign corporations' participation in large-scale EDU.
The inclusion of this clause on "technical or financial assistance" recognizes the fact that foreign
business entities and multinational corporations are the ones with the resources and know-how to
provide technical and/or financial assistance of the magnitude and type required for large-scale
exploration, development and utilization of these resources.
The drafters -- whose ranks included many academicians, economists, businessmen, lawyers,
politicians and government officials -- were not unfamiliar with the practices of foreign corporations
and multinationals.
Neither were they so nave as to believe that these entities would provide "assistance" without
conditionalities or some quid pro quo. Definitely, as business persons well know and as a matter of
judicial notice, this matter is not just a question of signing a promissory note or executing a
technology transfer agreement. Foreign corporations usually require that they be given a say in the
management, for instance, of day-to-day operations of the joint venture. They would demand the
appointment of their own men as, for example, operations managers, technical experts, quality
control heads, internal auditors or comptrollers. Furthermore, they would probably require seats on
the Board of Directors -- all these to ensure the success of the enterprise and the repayment of the
loans and other financial assistance and to make certain that the funding and the technology they
supply would not go to waste. Ultimately, they would also want to protect their business reputation
and bottom lines.42
In short, the drafters will have to be credited with enough pragmatism and savvy to know that these
foreign entities will not enter into such "agreements involving assistance" without requiring
arrangements for the protection of their investments, gains and benefits.

Thus, by specifying such "agreements involving assistance," the drafters necessarily gave implied
assent to everything that these agreements necessarily entailed; or that could reasonably be
deemed necessary to make them tenable and effective, including management authority with
respect to the day-to-day operations of the enterprise and measures for the protection of the
interests of the foreign corporation, PROVIDED THAT Philippine sovereignty over natural resources
and full control over the enterprise undertaking the EDU activities remain firmly in the State.
Petitioners' Theory Deflated by the
Absence of Closing-Out Rules or Guidelines
Seventh and final point regarding the plain-language approach, one of the practical difficulties that
results from it is the fact that there is nothing by way of transitory provisions that would serve to
confirm the theory that the omission of the term "service contract" from the 1987 Constitution
signaled the demise of service contracts.
The framers knew at the time they were deliberating that there were various service contracts extant
and in force and effect, including those in the petroleum industry. Many of these service contracts
were long-term (25 years) and had several more years to run. If they had meant to ban service
contracts altogether, they would have had to provide for the termination or pretermination of the
existing contracts. Accordingly, they would have supplied the specifics and the when and how of
effecting the extinguishment of these existing contracts (or at least the mechanics for determining
them); and of putting in place the means to address the just claims of the contractors for
compensation for their investments, lost opportunities, and so on, if not for the recovery thereof.
If the framers had intended to put an end to service contracts, they would have at least left specific
instructions to Congress to deal with these closing-out issues, perhaps by way of general guidelines
and a timeline within which to carry them out. The following are some extant examples of such
transitory guidelines set forth in Article XVIII of our Constitution:
"Section 23. Advertising entities affected by paragraph (2), Section 11 of Article XVI of this
Constitution shall have five years from its ratification to comply on a graduated and
proportionate basis with the minimum Filipino ownership requirement therein.
xxxxxxxxx
"Section 25. After the expiration in 1991 of the Agreement between the Republic of the
Philippines and the United States of America concerning military bases, foreign military
bases, troops, or facilities shall not be allowed in the Philippines except under a treaty duly
concurred in by the Senate and, when the Congress so requires, ratified by a majority of the
votes cast by the people in a national referendum held for that purpose, and recognized as a
treaty by the other contracting State.
"Section 26. The authority to issue sequestration or freeze orders under Proclamation No. 3
dated March 25, 1986 in relation to the recovery of ill-gotten wealth shall remain operative
for not more than eighteen months after the ratification of this Constitution. However, in the
national interest, as certified by the President, the Congress may extend such period.
A sequestration or freeze order shall be issued only upon showing of a prima facie case. The
order and the list of the sequestered or frozen properties shall forthwith be registered with
the proper court. For orders issued before the ratification of this Constitution, the
corresponding judicial action or proceeding shall be filed within six months from its

ratification. For those issued after such ratification, the judicial action or proceeding shall be
commenced within six months from the issuance thereof.
The sequestration or freeze order is deemed automatically lifted if no judicial action or
proceeding is commenced as herein provided." 43]
It is inconceivable that the drafters of the Constitution would leave such an important matter -- an
expression of sovereignty as it were -- indefinitely hanging in the air in a formless and ineffective
state. Indeed, the complete absence of even a general framework only serves to further deflate
petitioners' theory, like a child's balloon losing its air.
Under the circumstances, the logical inconsistencies resulting from petitioners' literal and
purely verba legisapproach to paragraph 4 of Section 2 of Article XII compel a resort to other aids to
interpretation.
Petitioners' Posture Also Negated
by Ratio Legis Et Anima
Thus, in order to resolve the inconsistencies, incongruities and ambiguities encountered and to
supply the deficiencies of the plain-language approach, there is a need for recourse to the
proceedings of the 1986 Constitutional Commission. There is a need for ratio legis et anima.
Service Contracts Not
"Deconstitutionalized"
Pertinent portions of the deliberations of the members of the Constitutional Commission (ConCom)
conclusively show that they discussed agreements involving either technical or financial
assistance in the same breadth asservice contracts and used the terms interchangeably. The
following exchange between Commissioner Jamir (sponsor of the provision) and Commissioner
Suarez irrefutably proves that the "agreements involving technical or financial assistance" were none
other than service contracts.
THE PRESIDENT. Commissioner Jamir is recognized. We are still on Section 3.
MR. JAMIR. Yes, Madam President. With respect to the second paragraph of Section 3, my
amendment by substitution reads: THE PRESIDENT MAY ENTER INTO AGREEMENTS
WITH FOREIGN-OWNED CORPORATIONS INVOLVING EITHER TECHNICAL OR
FINANCIAL ASSISTANCE FOR LARGE-SCALE EXPLORATION, DEVELOPMENT AND
UTILIZATION OF NATURAL RESOURCES ACCORDING TO THE TERMS AND
CONDITIONS PROVIDED BY LAW.
MR. VILLEGAS. The Committee accepts the amendment. Commissioner Suarez will give the
background.
MR. JAMIR. Thank you.
THE PRESIDENT. Commissioner Suarez is recognized.
MR. SUAREZ. Thank you, Madam President.
Will Commissioner Jamir answer a few clarificatory questions?

MR. JAMIR. Yes, Madam President.


MR. SUAREZ. This particular portion of the section has reference to what was popularly
known before as service contracts, among other things, is that correct?
MR. JAMIR. Yes, Madam President.
MR. SUAREZ. As it is formulated, the President may enter into service contracts but
subject to the guidelines that may be promulgated by Congress?
MR. JAMIR. That is correct.
MR. SUAREZ. Therefore, that aspect of negotiation and consummation will fall on the
President, not upon Congress?
MR. JAMIR. That is also correct, Madam President.
MR. SUAREZ. Except that all of these contracts, service or otherwise, must be made
strictly in accordance with guidelines prescribed by Congress?
MR. JAMIR. That is also correct.
MR. SUAREZ. And the Gentleman is thinking in terms of a law that uniformly covers
situations of the same nature?
MR. JAMIR. That is 100 percent correct.
MR. SUAREZ. I thank the Commissioner.
MR. JAMIR. Thank you very much.44
The following exchange leaves no doubt that the commissioners knew exactly what they were
dealing with: service contracts.
THE PRESIDENT. Commissioner Gascon is recognized.
MR. GASCON. Commissioner Jamir had proposed an amendment with regard to
special service contracts which was accepted by the Committee. Since the Committee has
accepted it, I would like to ask some questions.
THE PRESIDENT. Commissioner Gascon may proceed.
MR. GASCON. As it is proposed now, such service contracts will be entered into by the
President with the guidelines of a general law on service contract to be enacted by
Congress. Is that correct?
MR. VILLEGAS. The Commissioner is right, Madam President.
MR. GASCON. According to the original proposal, if the President were to enter into a
particular agreement, he would need the concurrence of Congress. Now that it has been

changed by the proposal of Commissioner Jamir in that Congress will set the general law to
which the President shall comply, the President will, therefore, not need the concurrence of
Congress every time he enters into service contracts. Is that correct?
MR. VILLEGAS. That is right.
MR. GASCON. The proposed amendment of Commissioner Jamir is in indirect contrast to
my proposed amendment, so I would like to object and present my proposed amendment to
the body.
xxxxxxxxx
MR. GASCON. Yes, it will be up to the body.
I feel that the general law to be set by Congress as regard service contract
agreements which the President will enter into might be too general or since we do not know
the content yet of such a law, it might be that certain agreements will be detrimental to the
interest of the Filipinos. This is in direct contrast to my proposal which provides that there be
effective constraints in the implementation of service contracts.
So instead of a general law to be passed by Congress to serve as a guideline to the
President when entering into service contract agreements, I propose that every service
contract entered into by the President would need the concurrence of Congress, so as to
assure the Filipinos of their interests with regard to the issue in Section 3 on all lands of the
public domain. My alternative amendment, which we will discuss later, reads: THAT THE
PRESIDENT SHALL ENTER INTO SUCH AGREEMENTS ONLY WITH THE
CONCURRENCE OF TWO-THIRDS VOTE OF ALL THE MEMBERS OF CONGRESS
SITTING SEPARATELY.
xxxxxxxxx
MR. BENGZON. The reason we made that shift is that we realized the original proposal
could breed corruption. By the way, this is not just confined to service contracts but also
to financial assistance. If we are going to make every single contract subject to the
concurrence of Congress which, according to the Commissioner's amendment is the
concurrence of two-thirds of Congress voting separately then (1) there is a very great
chance that each contract will be different from another; and (2) there is a great temptation
that it would breed corruption because of the great lobbying that is going to happen. And we
do not want to subject our legislature to that.
Now, to answer the Commissioner's apprehension, by "general law," we do not mean
statements of motherhood. Congress can build all the restrictions that it wishes into that
general law so that every contract entered into by the President under that specific area will
have to be uniform. The President has no choice but to follow all the guidelines that will be
provided by law.
MR. GASCON. But my basic problem is that we do not know as of yet the contents of such a
general law as to how much constraints there will be in it. And to my mind, although the
Committee's contention that the regular concurrence from Congress would subject Congress
to extensive lobbying, I think that is a risk we will have to take since Congress is a body of
representatives of the people whose membership will be changing regularly as there will be

changing circumstances every time certain agreements are made. It would be best then to
keep in tab and attuned to the interest of the Filipino people, whenever the President enters
into any agreement with regard to such an important matter as technical or financial
assistance for large-scale exploration, development and utilization of natural
resources or service contracts, the people's elected representatives should be on top of it.
xxxxxxxxx
MR. OPLE. Madam President, we do not need to suspend the session. If Commissioner
Gascon needs a few minutes, I can fill up the remaining time while he completes his
proposed amendment. I just wanted to ask Commissioner Jamir whether he would entertain
a minor amendment to his amendment, and it reads as follows: THE PRESIDENT SHALL
SUBSEQUENTLY NOTIFY CONGRESS OF EVERY SERVICE CONTRACT ENTERED
INTO IN ACCORDANCE WITH THE GENERAL LAW. I think the reason is, if I may state it
briefly, as Commissioner Bengzon said, Congress can always change the general law later
on to conform to new perceptions of standards that should be built into service contracts.
But the only way Congress can do this is if there were a notification requirement from the
Office of the President that suchservice contracts had been entered into, subject then to
the scrutiny of the Members of Congress. This pertains to a situation where the service
contracts are already entered into, and all that this amendment seeks is the reporting
requirement from the Office of the President. Will Commissioner Jamir entertain that?
MR. JAMIR. I will gladly do so, if it is still within my power.
MR. VILLEGAS. Yes, the Committee accepts the amendment.
xxxxxxxxx
SR. TAN. Madam President, may I ask a question?
THE PRESIDENT. Commissioner Tan is recognized.
SR. TAN. Am I correct in thinking that the only difference between these future service
contracts and the past service contracts under Mr. Marcos is the general law to be
enacted by the legislature and the notification of Congress by the President? That is the only
difference, is it not?
MR. VILLEGAS. That is right.
SR. TAN. So those are the safeguards.
MR. VILLEGAS. Yes. There was no law at all governing service contracts before.
SR. TAN. Thank you, Madam President.45
More Than Mere Financial
and Technical Assistance
Entailed by the Agreements
The clear words of Commissioner Jose N. Nolledo quoted below explicitly and eloquently
demonstrate that the drafters knew that the agreements with foreign corporations were going to

entail not mere technical or financial assistance but, rather, foreign investment in and management
of an enterprise involved in large-scale exploration,development and utilization of minerals,
petroleum, and other mineral oils.
THE PRESIDENT. Commissioner Nolledo is recognized.
MR. NOLLEDO. Madam President, I have the permission of the Acting Floor Leader to speak
for only two minutes in favor of the amendment of Commissioner Gascon.
THE PRESIDENT. Commissioner Nolledo may proceed.
MR. NOLLEDO. With due respect to the members of the Committee and Commissioner
Jamir, I am in favor of the objection of Commissioner Gascon.
Madam President, I was one of those who refused to sign the 1973 Constitution, and
one of the reasons is that there were many provisions in the Transitory Provisions
therein that favored aliens. I was shocked when I read a provision
authorizing service contracts while we, in this Constitutional Commission, provided
for Filipino control of the economy. We are, therefore, providing for exceptional
instances where aliens may circumvent Filipino control of our economy. And one way
of circumventing the rule in favor of Filipino control of the economy is to
recognize service contracts.
As far as I am concerned, if I should have my own way, I am for the complete
deletion of this provision. However, we are presenting a compromise in the sense
that we are requiring a two-thirds vote of all the Members of Congress as a
safeguard. I think we should not mistrust the future Members of Congress by saying
that the purpose of this provision is to avoid corruption. We cannot claim that they
are less patriotic than we are. I think the Members of this Commission should know
that entering into service contracts is an exception to the rule on protection of
natural resources for the interest of the nation, and therefore, being an exception it
should be subject, whenever possible, to stringent rules. It seems to me that we are
liberalizing the rules in favor of aliens.
I say these things with a heavy heart, Madam President. I do not claim to be a
nationalist, but I love my country. Although we need investments, we must adopt
safeguards that are truly reflective of the sentiments of the people and not mere
cosmetic safeguards as they now appear in the Jamir amendment. (Applause)
Thank you, Madam President.46
Another excerpt, featuring then Commissioner (now Chief Justice) Hilario G. Davide Jr., indicates the
limitations of the scope of such service contracts -- they are valid only in regard to minerals,
petroleum and other mineral oils, not to all natural resources.
THE PRESIDENT. Commissioner Davide is recognized.
MR. DAVIDE. Thank you, Madam President. This is an amendment to the Jamir amendment
and also to the Ople amendment. I propose to delete "NATURAL RESOURCES" and
substitute it with the following: MINERALS, PETROLEUM AND OTHER MINERAL OILS. On

the Ople amendment, I propose to add: THE NOTIFICATION TO CONGRESS SHALL BE


WITHIN THIRTY DAYS FROM THE EXECUTION OF THE SERVICE CONTRACT.
THE PRESIDENT. What does the Committee say with respect to the first amendment in lieu
of "NATURAL RESOURCES"?
MR. VILLEGAS. Could Commissioner Davide explain that?
MR. DAVIDE. Madam President, with the use of "NATURAL RESOURCES" here, it would
necessarily include all lands of the public domain, our marine resources, forests, parks and
so on. So we would like to limit the scope of these service contracts to those areas really
where these may be needed, the exploitation, development and exploration of minerals,
petroleum and other mineral oils. And so, we believe that we should really, if we want to
grant service contracts at all, limit the same to only those particular areas where Filipino
capital may not be sufficient, and not to all natural resources.
MR. SUAREZ. Just a point of clarification again, Madam President. When the Commissioner
made those enumerations and specifications, I suppose he deliberately did not include
"agricultural land"?
MR. DAVIDE. That is precisely the reason we have to enumerate what these resources are
into whichservice contracts may enter. So, beyond the reach of any service contract will
be lands of the public domain, timberlands, forests, marine resources, fauna and flora,
wildlife and national parks.47
After the Jamir amendment was voted upon and approved by a vote of 21 to 10 with 2 abstentions,
Commissioner Davide made the following statement, which is very relevant to our quest:
THE PRESIDENT. Commissioner Davide is recognized.
MR. DAVIDE. I am very glad that Commissioner Padilla emphasized minerals, petroleum
and mineral oils. The Commission has just approved the possible foreign entry into the
development, exploration and utilization of these minerals, petroleum and other mineral oils
by virtue of the Jamir amendment. I voted in favor of the Jamir amendment because it will
eventually give way to vesting in exclusively Filipino citizens and corporations wholly owned
by Filipino citizens the right to utilize the other natural resources. This means that as a matter
of policy, natural resources should be utilized and exploited only by Filipino citizens or
corporations wholly owned by such citizens. But by virtue of the Jamir amendment, since we
feel that Filipino capital may not be enough for the development and utilization of minerals,
petroleum and other mineral oils, the President can enter into service contracts with foreign
corporations precisely for the development and utilization of such resources. And so, there is
nothing to fear that we will stagnate in the development of minerals, petroleum and mineral
oils because we now allow service contracts. x x x."48
The foregoing are mere fragments of the framers' lengthy discussions of the provision dealing
with agreements x x x involving either technical or financial assistance, which ultimately became
paragraph 4 of Section 2 of Article XII of the Constitution. Beyond any doubt, the members of the
ConCom were actually debating about the martial-law-era service contracts for which they were
crafting appropriate safeguards.

In the voting that led to the approval of Article XII by the ConCom, the explanations given by
Commissioners Gascon, Garcia and Tadeo indicated that they had voted to reject this provision on
account of their objections to the "constitutionalization" of the "service contract" concept.
Mr. Gascon said, "I felt that if we would constitutionalize any provision on service contracts, this
should always be with the concurrence of Congress and not guided only by a general law to be
promulgated by Congress."49Mr. Garcia explained, "Service contracts are given constitutional
legitimization in Sec. 3, even when they have been proven to be inimical to the interests of the
nation, providing, as they do, the legal loophole for the exploitation of our natural resources for the
benefit of foreign interests."50 Likewise, Mr. Tadeo cited inter alia the fact that service contracts
continued to subsist, enabling foreign interests to benefit from our natural resources. 51It was hardly
likely that these gentlemen would have objected so strenuously, had the provision called for
mere technical or financial assistance and nothing more.
The deliberations of the ConCom and some commissioners' explanation of their votes leave no room
for doubt that the service contract concept precisely underpinned the commissioners' understanding
of the "agreements involving either technical or financial assistance."
Summation of the
Concom Deliberations
At this point, we sum up the matters established, based on a careful reading of the ConCom
deliberations, as follows:
In their deliberations on what was to become paragraph 4, the framers used the
term service contracts in referring to agreements x x x involving either technical or financial
assistance.
They spoke of service contracts as the concept was understood in the 1973 Constitution.
It was obvious from their discussions that they were not about to ban or eradicate service
contracts.
Instead, they were plainly crafting provisions to put in place safeguards that would eliminate
or minimize the abuses prevalent during the marital law regime. In brief, they were going to
permit service contracts with foreign corporations as contractors, but with safety measures to
prevent abuses, as an exception to the general norm established in the first paragraph of
Section 2 of Article XII. This provision reserves or limits to Filipino citizens -- and
corporations at least 60 percent of which is owned by such citizens -- the exploration,
development and utilization of natural resources.
This provision was prompted by the perceived insufficiency of Filipino capital and the felt
need for foreign investments in the EDU of minerals and petroleum resources.
The framers for the most part debated about the sort of safeguards that would be
considered adequate and reasonable. But some of them, having more "radical" leanings,
wanted to ban service contracts altogether; for them, the provision would permit aliens to
exploit and benefit from the nation's natural resources, which they felt should be reserved
only for Filipinos.

In the explanation of their votes, the individual commissioners were heard by the entire
body. They sounded off their individual opinions, openly enunciated their philosophies, and
supported or attacked the provisions with fervor. Everyone's viewpoint was heard.
In the final voting, the Article on the National Economy and Patrimony -- including
paragraph 4 allowing service contracts with foreign corporations as an exception to the
general norm in paragraph 1 of Section 2 of the same article -- was resoundingly approved
by a vote of 32 to 7, with 2 abstentions.
Agreements Involving Technical
or Financial Assistance Are
Service Contracts With Safeguards
From the foregoing, we are impelled to conclude that the phrase agreements involving either
technical or financial assistance, referred to in paragraph 4, are in fact service contracts. But unlike
those of the 1973 variety, the new ones are between foreign corporations acting as contractors on
the one hand; and on the other, the government as principal or "owner" of the works. In the new
service contracts, the foreign contractors provide capital, technology and technical know-how, and
managerial expertise in the creation and operation of large-scale mining/extractive enterprises; and
the government, through its agencies (DENR, MGB), actively exercises control and supervision over
the entire operation.
Such service contracts may be entered into only with respect to minerals, petroleum and other
mineral oils. The grant thereof is subject to several safeguards, among which are these
requirements:
(1) The service contract shall be crafted in accordance with a general law that will set
standard or uniform terms, conditions and requirements, presumably to attain a certain
uniformity in provisions and avoid the possible insertion of terms disadvantageous to the
country.
(2) The President shall be the signatory for the government because, supposedly before an
agreement is presented to the President for signature, it will have been vetted several times
over at different levels to ensure that it conforms to law and can withstand public scrutiny.
(3) Within thirty days of the executed agreement, the President shall report it to Congress to
give that branch of government an opportunity to look over the agreement and interpose
timely objections, if any.
Use of the Record of the
ConCom to Ascertain Intent
At this juncture, we shall address, rather than gloss over, the use of the "framers' intent" approach,
and the criticism hurled by petitioners who quote a ruling of this Court:
"While it is permissible in this jurisdiction to consult the debates and proceedings of the
constitutional convention in order to arrive at the reason and purpose of the resulting
Constitution, resort thereto may be had only when other guides fail as said proceedings are

powerless to vary the terms of the Constitution when the meaning is clear. Debates in the
constitutional convention 'are of value as showing the views of the individual members, and
as indicating the reason for their votes, but they give us no light as to the views of the large
majority who did not talk, much less the mass of our fellow citizens whose votes at the polls
gave that instrument the force of fundamental law. We think it safer to construe the
constitution from what appears upon its face.' The proper interpretation therefore depends
more on how it was understood by the people adopting it than in the framers' understanding
thereof."52
The notion that the deliberations reflect only the views of those members who spoke out and not the
views of the majority who remained silent should be clarified. We must never forget that those who
spoke out were heard by those who remained silent and did not react. If the latter were silent
because they happened not to be present at the time, they are presumed to have read the minutes
and kept abreast of the deliberations. By remaining silent, they are deemed to have signified their
assent to and/or conformity with at least some of the views propounded or their lack of objections
thereto. It was incumbent upon them, as representatives of the entire Filipino people, to follow the
deliberations closely and to speak their minds on the matter if they did not see eye to eye with the
proponents of the draft provisions.
In any event, each and every one of the commissioners had the opportunity to speak out and to vote
on the matter. Moreover, the individual explanations of votes are on record, and they show where
each delegate stood on the issues. In sum, we cannot completely denigrate the value or
usefulness of the record of the ConCom, simply because certain members chose not to
speak out.
It is contended that the deliberations therein did not necessarily reflect the thinking of the voting
population that participated in the referendum and ratified the Constitution. Verily, whether we like it
or not, it is a bit too much to assume that every one of those who voted to ratify the proposed
Charter did so only after carefully reading and mulling over it, provision by provision.
Likewise, it appears rather extravagant to assume that every one of those who did in fact bother to
read the draft Charter actually understood the import of its provisions, much less analyzed it vis--vis
the previous Constitutions. We believe that in reality, a good percentage of those who voted in favor
of it did so more out of faith and trust. For them, it was the product of the hard work and careful
deliberation of a group of intelligent, dedicated and trustworthy men and women of integrity and
conviction, whose love of country and fidelity to duty could not be questioned.
In short, a large proportion of the voters voted "yes" because the drafters, or a majority of them,
endorsed the proposed Constitution. What this fact translates to is the inescapable conclusion that
many of the voters in the referendum did not form their own isolated judgment about the draft
Charter, much less about particular provisions therein. They only relied or fell back and acted upon
the favorable endorsement or recommendation of the framers as a group. In other words, by
voting yes, they may be deemed to have signified their voluntary adoption of the understanding and
interpretation of the delegates with respect to the proposed Charter and its particular provisions. "If
it's good enough for them, it's good enough for me;" or, in many instances, "If it's good enough for
President Cory Aquino, it's good enough for me."
And even for those who voted based on their own individual assessment of the proposed Charter,
there is no evidence available to indicate that their assessment or understanding of its provisions
was in fact different from that of the drafters. This unwritten assumption seems to be petitioners' as
well. For all we know, this segment of voters must have read and understood the provisions of the

Constitution in the same way the framers had, an assumption that would account for the favorable
votes.
Fundamentally speaking, in the process of rewriting the Charter, the members of the ConCom as a
group were supposed to represent the entire Filipino people. Thus, we cannot but regard their views
as being very much indicative of the thinking of the people with respect to the matters deliberated
upon and to the Charter as a whole.
It is therefore reasonable and unavoidable to make the following conclusion, based on the
above arguments. As written by the framers and ratified and adopted by the people, the
Constitution allows the continued use of service contracts with foreign corporations -- as
contractors who would invest in and operate and manage extractive enterprises, subject to
the full control and supervision of the State -- sans the abuses of the past regime. The
purpose is clear: to develop and utilize our mineral, petroleum and other resources on a large
scale for the immediate and tangible benefit of the Filipino people.
In view of the foregoing discussion, we should reverse the Decision of January 27, 2004, and in fact
now hold a view different from that of the Decision, which had these findings: (a) paragraph 4 of
Section 2 of Article XII limits foreign involvement in the local mining industry to agreements strictly
for either financial or technical assistance only; (b) the same paragraph precludes agreements that
grant to foreign corporations the management of local mining operations, as such agreements are
purportedly in the nature of service contracts as these were understood under the 1973 Constitution;
(c) these service contracts were supposedly "de-constitutionalized" and proscribed by the omission
of the term service contracts from the 1987 Constitution; (d) since the WMCP FTAA contains
provisions permitting the foreign contractor to manage the concern, the said FTAA is invalid for being
a prohibited service contract; and (e) provisions of RA 7942 and DAO 96-40, which likewise grant
managerial authority to the foreign contractor, are also invalid and unconstitutional.
Ultimate Test: State's "Control"
Determinative of Constitutionality
But we are not yet at the end of our quest. Far from it. It seems that we are confronted with a
possible collision of constitutional provisions. On the one hand, paragraph 1 of Section 2 of Article
XII explicitly mandates the State to exercise "full control and supervision" over the exploration,
development and utilization of natural resources. On the other hand, paragraph 4 permits
safeguarded service contracts with foreign contractors. Normally, pursuant thereto, the contractors
exercise management prerogatives over the mining operations and the enterprise as a whole. There
is thus a legitimate ground to be concerned that either the State's full control and supervision may
rule out any exercise of management authority by the foreign contractor; or, the other way around,
allowing the foreign contractor full management prerogatives may ultimately negate the State's full
control and supervision.
Ut Magis Valeat
Quam Pereat
Under the third principle of constitutional construction laid down in Francisco -- ut magis valeat quam
pereat --every part of the Constitution is to be given effect, and the Constitution is to be read and
understood as a harmonious whole. Thus, "full control and supervision" by the State must be
understood as one that does not preclude the legitimate exercise of management prerogatives by
the foreign contractor. Before any further discussion, we must stress the primacy and supremacy of
the principle of sovereignty and State control and supervision over all aspects of exploration,

development and utilization of the country's natural resources, as mandated in the first paragraph of
Section 2 of Article XII.
But in the next breadth we have to point out that "full control and supervision" cannot be taken
literally to mean that the State controls and supervises everything involved, down to the minutest
details, and makes all decisionsrequired in the mining operations. This strained concept of control
and supervision over the mining enterprise would render impossible the legitimate exercise by the
contractors of a reasonable degree of management prerogative and authority necessary and
indispensable to their proper functioning.
For one thing, such an interpretation would discourage foreign entry into large-scale exploration,
development and utilization activities; and result in the unmitigated stagnation of this sector, to the
detriment of our nation's development. This scenario renders paragraph 4 inoperative and useless.
And as respondents have correctly pointed out, the government does not have to micro-manage the
mining operations and dip its hands into the day-to-day affairs of the enterprise in order for it to be
considered as having full control and supervision.
The concept of control53 adopted in Section 2 of Article XII must be taken to mean less than
dictatorial, all-encompassing control; but nevertheless sufficient to give the State the power to direct,
restrain, regulate and govern the affairs of the extractive enterprises. Control by the State may be on
a macro level, through the establishment of policies, guidelines, regulations, industry standards and
similar measures that would enable the government to control the conduct of affairs in various
enterprises and restrain activities deemed not desirable or beneficial.
The end in view is ensuring that these enterprises contribute to the economic development and
general welfare of the country, conserve the environment, and uplift the well-being of the affected
local communities. Such a concept of control would be compatible with permitting the foreign
contractor sufficient and reasonable management authority over the enterprise it invested in, in order
to ensure that it is operating efficiently and profitably, to protect its investments and to enable it to
succeed.
The question to be answered, then, is whether RA 7942 and its Implementing Rules enable
the government to exercise that degree of control sufficient to direct and regulate the
conduct of affairs of individual enterprises and restrain undesirable activities.
On the resolution of these questions will depend the validity and constitutionality of certain provisions
of the Philippine Mining Act of 1995 (RA 7942) and its Implementing Rules and Regulations (DAO
96-40), as well as the WMCP FTAA.
Indeed, petitioners charge54 that RA 7942, as well as its Implementing Rules and Regulations, makes
it possible for FTAA contracts to cede full control and management of mining enterprises over to fully
foreign-owned corporations, with the result that the State is allegedly reduced to a passive regulator
dependent on submitted plans and reports, with weak review and audit powers. The State does not
supposedly act as the owner of the natural resources for and on behalf of the Filipino people; it
practically has little effective say in the decisions made by the enterprise. Petitioners then conclude
that the law, the implementing regulations, and the WMCP FTAA cede "beneficial ownership" of the
mineral resources to the foreign contractor.
A careful scrutiny of the provisions of RA 7942 and its Implementing Rules belies petitioners' claims.
Paraphrasing the Constitution, Section 4 of the statute clearly affirms the State's control thus:

"Sec. 4. Ownership of Mineral Resources. Mineral resources are owned by the State and
the exploration, development, utilization and processing thereof shall be under its full control
and supervision. The State may directly undertake such activities or it may enter into mineral
agreements with contractors.
"The State shall recognize and protect the rights of the indigenous cultural communities to
their ancestral lands as provided for by the Constitution."
The aforequoted provision is substantively reiterated in Section 2 of DAO 96-40 as follows:
"Sec. 2. Declaration of Policy. All mineral resources in public and private lands within the
territory and exclusive economic zone of the Republic of the Philippines are owned by the
State. It shall be the responsibility of the State to promote their rational exploration,
development, utilization and conservation through the combined efforts of the Government
and private sector in order to enhance national growth in a way that effectively safeguards
the environment and protects the rights of affected communities."
Sufficient Control Over Mining
Operations Vested in the State
by RA 7942 and DAO 96-40
RA 7942 provides for the State's control and supervision over mining operations. The following
provisions thereof establish the mechanism of inspection and visitorial rights over mining operations
and institute reportorial requirements in this manner:
1. Sec. 8 which provides for the DENR's power of over-all supervision and periodic review for
"the conservation, management, development and proper use of the State's mineral
resources";
2. Sec. 9 which authorizes the Mines and Geosciences Bureau (MGB) under the DENR to
exercise "direct charge in the administration and disposition of mineral resources", and
empowers the MGB to "monitor the compliance by the contractor of the terms and conditions
of the mineral agreements", "confiscate surety and performance bonds", and deputize
whenever necessary any member or unit of the Phil. National Police, barangay, duly
registered non-governmental organization (NGO) or any qualified person to police mining
activities;
3. Sec. 66 which vests in the Regional Director "exclusive jurisdiction over safety inspections
of all installations, whether surface or underground", utilized in mining operations.
4. Sec. 35, which incorporates into all FTAAs the following terms, conditions and warranties:
"(g) Mining operations shall be conducted in accordance with the provisions of the
Act and its IRR.
"(h) Work programs and minimum expenditures commitments.
xxxxxxxxx
"(k) Requiring proponent to effectively use appropriate anti-pollution technology and
facilities to protect the environment and restore or rehabilitate mined-out areas.

"(l) The contractors shall furnish the Government records of geologic, accounting and
other relevant data for its mining operation, and that books of accounts and records
shall be open for inspection by the government. x x x.
"(m) Requiring the proponent to dispose of the minerals at the highest price and
more advantageous terms and conditions.
"(n) x x x x x x x x x
"(o) Such other terms and conditions consistent with the Constitution and with this
Act as the Secretary may deem to be for the best interest of the State and the
welfare of the Filipino people."
The foregoing provisions of Section 35 of RA 7942 are also reflected and
implemented in Section 56 (g), (h), (l), (m) and (n) of the Implementing Rules, DAO
96-40.
Moreover, RA 7942 and DAO 96-40 also provide various stipulations confirming the government's
control over mining enterprises:
The contractor is to relinquish to the government those portions of the contract area not
needed for mining operations and not covered by any declaration of mining feasibility
(Section 35-e, RA 7942; Section 60, DAO 96-40).
The contractor must comply with the provisions pertaining to mine safety, health and
environmental protection (Chapter XI, RA 7942; Chapters XV and XVI, DAO 96-40).
For violation of any of its terms and conditions, government may cancel an FTAA. (Chapter
XVII, RA 7942; Chapter XXIV, DAO 96-40).
An FTAA contractor is obliged to open its books of accounts and records for inspection by
the government (Section 56-m, DAO 96-40).
An FTAA contractor has to dispose of the minerals and by-products at the highest market
price and register with the MGB a copy of the sales agreement (Section 56-n, DAO 96-40).
MGB is mandated to monitor the contractor's compliance with the terms and conditions of
the FTAA; and to deputize, when necessary, any member or unit of the Philippine National
Police, the barangay or a DENR-accredited nongovernmental organization to police mining
activities (Section 7-d and -f, DAO 96-40).
An FTAA cannot be transferred or assigned without prior approval by the President (Section
40, RA 7942; Section 66, DAO 96-40).
A mining project under an FTAA cannot proceed to the construction/development/utilization
stage, unless its Declaration of Mining Project Feasibility has been approved by government
(Section 24, RA 7942).
The Declaration of Mining Project Feasibility filed by the contractor cannot be approved
without submission of the following documents:

1. Approved mining project feasibility study (Section 53-d, DAO 96-40)


2. Approved three-year work program (Section 53-a-4, DAO 96-40)
3. Environmental compliance certificate (Section 70, RA 7942)
4. Approved environmental protection and enhancement program (Section 69, RA
7942)
5. Approval by the Sangguniang Panlalawigan/Bayan/Barangay (Section 70, RA
7942; Section 27, RA 7160)
6. Free and prior informed consent by the indigenous peoples concerned, including
payment of royalties through a Memorandum of Agreement (Section 16, RA 7942;
Section 59, RA 8371)
The FTAA contractor is obliged to assist in the development of its mining community,
promotion of the general welfare of its inhabitants, and development of science and mining
technology (Section 57, RA 7942).
The FTAA contractor is obliged to submit reports (on quarterly, semi-annual or annual basis
as the case may be; per Section 270, DAO 96-40), pertaining to the following:
1. Exploration
2. Drilling
3. Mineral resources and reserves
4. Energy consumption
5. Production
6. Sales and marketing
7. Employment
8. Payment of taxes, royalties, fees and other Government Shares
9. Mine safety, health and environment
10. Land use
11. Social development
12. Explosives consumption
An FTAA pertaining to areas within government reservations cannot be granted without a
written clearance from the government agencies concerned (Section 19, RA 7942; Section
54, DAO 96-40).

An FTAA contractor is required to post a financial guarantee bond in favor of the


government in an amount equivalent to its expenditures obligations for any particular year.
This requirement is apart from the representations and warranties of the contractor that it
has access to all the financing, managerial and technical expertise and technology
necessary to carry out the objectives of the FTAA (Section 35-b, -e, and -f, RA 7942).
Other reports to be submitted by the contractor, as required under DAO 96-40, are as
follows: an environmental report on the rehabilitation of the mined-out area and/or mine
waste/tailing covered area, and anti-pollution measures undertaken (Section 35-a-2); annual
reports of the mining operations and records of geologic accounting (Section 56-m); annual
progress reports and final report of exploration activities (Section 56-2).
Other programs required to be submitted by the contractor, pursuant to DAO 96-40, are the
following: a safety and health program (Section 144); an environmental work program
(Section 168); an annual environmental protection and enhancement program (Section 171).
The foregoing gamut of requirements, regulations, restrictions and limitations imposed upon the
FTAA contractor by the statute and regulations easily overturns petitioners' contention. The setup
under RA 7942 and DAO 96-40 hardly relegates the State to the role of a "passive regulator"
dependent on submitted plans and reports. On the contrary, the government agencies concerned
are empowered to approve or disapprove -- hence, to influence, direct and change -- the various
work programs and the corresponding minimum expenditure commitments for each of the
exploration, development and utilization phases of the mining enterprise.
Once these plans and reports are approved, the contractor is bound to comply with its commitments
therein. Figures for mineral production and sales are regularly monitored and subjected to
government review, in order to ensure that the products and by-products are disposed of at the best
prices possible; even copies of sales agreements have to be submitted to and registered with MGB.
And the contractor is mandated to open its books of accounts and records for scrutiny, so as to
enable the State to determine if the government share has been fully paid.
The State may likewise compel the contractor's compliance with mandatory requirements on mine
safety, health and environmental protection, and the use of anti-pollution technology and facilities.
Moreover, the contractor is also obligated to assist in the development of the mining community and
to pay royalties to the indigenous peoples concerned.
Cancellation of the FTAA may be the penalty for violation of any of its terms and conditions and/or
noncompliance with statutes or regulations. This general, all-around, multipurpose sanction is no
trifling matter, especially to a contractor who may have yet to recover the tens or hundreds of
millions of dollars sunk into a mining project.
Overall, considering the provisions of the statute and the regulations just discussed, we believe that
the State definitely possesses the means by which it can have the ultimate word in the operation of
the enterprise, set directions and objectives, and detect deviations and noncompliance by the
contractor; likewise, it has the capability to enforce compliance and to impose sanctions, should the
occasion therefor arise.
In other words, the FTAA contractor is not free to do whatever it pleases and get away with it;
on the contrary, it will have to follow the government line if it wants to stay in the enterprise.
Ineluctably then, RA 7942 and DAO 96-40 vest in the government more than a sufficient
degree of control and supervision over the conduct of mining operations.

Section 3(aq) of RA 7942


Not Unconstitutional
An objection has been expressed that Section 3(aq)55 of RA 7942 -- which allows a foreign contractor
to apply for and hold an exploration permit -- is unconstitutional. The reasoning is that Section 2 of
Article XII of the Constitution does not allow foreign-owned corporations to undertake mining
operations directly. They may act only as contractors of the State under an FTAA; and the State, as
the party directly undertaking exploitation of its natural resources, must hold through the government
all exploration permits and similar authorizations. Hence, Section 3(aq), in permitting foreign-owned
corporations to hold exploration permits, is unconstitutional.
The objection, however, is not well-founded. While the Constitution mandates the State to exercise
full control and supervision over the exploitation of mineral resources, nowhere does it require the
government to hold all exploration permits and similar authorizations. In fact, there is no prohibition
at all against foreign or local corporations or contractors holding exploration permits. The reason is
not hard to see.
Pursuant to Section 20 of RA 7942, an exploration permit merely grants to a qualified person the
right to conduct exploration for all minerals in specified areas. Such a permit does not amount to an
authorization to extract and carry off the mineral resources that may be discovered. This phase
involves nothing but expenditures for exploring the contract area and locating the mineral bodies. As
no extraction is involved, there are no revenues or incomes to speak of. In short, the exploration
permit is an authorization for the grantee to spend its own funds on exploration programs that are
pre-approved by the government, without any right to recover anything should no minerals in
commercial quantities be discovered. The State risks nothing and loses nothing by granting these
permits to local or foreign firms; in fact, it stands to gain in the form of data generated by the
exploration activities.
Pursuant to Section 24 of RA 7942, an exploration permit grantee who determines the commercial
viability of a mining area may, within the term of the permit, file with the MGB a declaration of mining
project feasibility accompanied by a work program for development. The approval of the mining
project feasibility and compliance with other requirements of RA 7942 vests in the grantee the
exclusive right to an MPSA or any other mineral agreement, or to an FTAA.
Thus, the permit grantee may apply for an MPSA, a joint venture agreement, a co-production
agreement, or an FTAA over the permit area, and the application shall be approved if the permit
grantee meets the necessary qualifications and the terms and conditions of any such agreement.
Therefore, the contractor will be in a position to extract minerals and earn revenues only when the
MPSA or another mineral agreement, or an FTAA, is granted. At that point, the contractor's rights
and obligations will be covered by an FTAA or a mineral agreement.
But prior to the issuance of such FTAA or mineral agreement, the exploration permit grantee (or
prospective contractor) cannot yet be deemed to have entered into any contract or agreement with
the State, and the grantee would definitely need to have some document or instrument as evidence
of its right to conduct exploration works within the specified area. This need is met by the exploration
permit issued pursuant to Sections 3(aq), 20 and 23 of RA 7942.
In brief, the exploration permit serves a practical and legitimate purpose in that it protects the
interests and preserves the rights of the exploration permit grantee (the would-be contractor)
-- foreign or local -- during the period of time that it is spending heavily on exploration works,
without yet being able to earn revenues to recoup any of its investments and
expenditures. Minus this permit and the protection it affords, the exploration works and

expenditures may end up benefiting only claim-jumpers. Such a possibility tends to discourage
investors and contractors. Thus, Section 3(aq) of RA 7942 may not be deemed unconstitutional.
The Terms of the WMCP FTAA
A Deference to State Control
A perusal of the WMCP FTAA also reveals a slew of stipulations providing for State control and
supervision:
1. The contractor is obligated to account for the value of production and sale of minerals
(Clause 1.4).
2. The contractor's work program, activities and budgets must be approved by/on behalf of
the State (Clause 2.1).
3. The DENR secretary has the power to extend the exploration period (Clause 3.2-a).
4. Approval by the State is necessary for incorporating lands into the FTAA contract area
(Clause 4.3-c).
5. The Bureau of Forest Development is vested with discretion in regard to approving the
inclusion of forest reserves as part of the FTAA contract area (Clause 4.5).
6. The contractor is obliged to relinquish periodically parts of the contract area not needed for
exploration and development (Clause 4.6).
7. A Declaration of Mining Feasibility must be submitted for approval by the State (Clause
4.6-b).
8. The contractor is obligated to report to the State its exploration activities (Clause 4.9).
9. The contractor is required to obtain State approval of its work programs for the succeeding
two-year periods, containing the proposed work activities and expenditures budget related to
exploration (Clause 5.1).
10. The contractor is required to obtain State approval for its proposed expenditures for
exploration activities (Clause 5.2).
11. The contractor is required to submit an annual report on geological, geophysical,
geochemical and other information relating to its explorations within the FTAA area (Clause
5.3-a).
12. The contractor is to submit within six months after expiration of exploration period a final
report on all its findings in the contract area (Clause 5.3-b).
13. The contractor, after conducting feasibility studies, shall submit a declaration of mining
feasibility, along with a description of the area to be developed and mined, a description of
the proposed mining operations and the technology to be employed, and a proposed work
program for the development phase, for approval by the DENR secretary (Clause 5.4).

14. The contractor is obliged to complete the development of the mine, including construction
of the production facilities, within the period stated in the approved work program (Clause
6.1).
15. The contractor is obligated to submit for approval of the DENR secretary a work program
covering each period of three fiscal years (Clause 6.2).
16. The contractor is to submit reports to the DENR secretary on the production, ore
reserves, work accomplished and work in progress, profile of its work force and management
staff, and other technical information (Clause 6.3).
17. Any expansions, modifications, improvements and replacements of mining facilities shall
be subject to the approval of the secretary (Clause 6.4).
18. The State has control with respect to the amount of funds that the contractor may borrow
within the Philippines (Clause 7.2).
19. The State has supervisory power with respect to technical, financial and marketing
issues (Clause 10.1-a).
20. The contractor is required to ensure 60 percent Filipino equity in the contractor, within ten
years of recovering specified expenditures, unless not so required by subsequent legislation
(Clause 10.1).
21. The State has the right to terminate the FTAA for the contractor's unremedied substantial
breach thereof (Clause 13.2);
22. The State's approval is needed for any assignment of the FTAA by the contractor to an
entity other than an affiliate (Clause 14.1).
We should elaborate a little on the work programs and budgets, and what they mean with respect to
the State's ability to exercise full control and effective supervision over the enterprise. For instance,
throughout the initial five-year exploration and feasibility phase of the project, the contractor is
mandated by Clause 5.1 of the WMCP FTAA to submit a series of work programs (copy furnished
the director of MGB) to the DENR secretary for approval.The programs will detail the contractor's
proposed exploration activities and budget covering each subsequent period of two fiscal years.
In other words, the concerned government officials will be informed beforehand of the proposed
exploration activities and expenditures of the contractor for each succeeding two-year period, with
the right to approve/disapprove them or require changes or adjustments therein if deemed
necessary.
Likewise, under Clause 5.2(a), the amount that the contractor was supposed to spend for exploration
activities during the first contract year of the exploration period was fixed at not less than P24 million;
and then for the succeeding years, the amount shall be as agreed between the DENR secretary and
the contractor prior to the commencement of each subsequent fiscal year. If no such agreement is
arrived upon, the previous year's expenditure commitment shall apply.
This provision alone grants the government through the DENR secretary a very big say in the
exploration phase of the project. This fact is not something to be taken lightly, considering that
the government has absolutely no contribution to the exploration expenditures or work activities and

yet is given veto power over such a critical aspect of the project. We cannot but construe as very
significant such a degree of control over the project and, resultantly, over the mining enterprise itself.
Following its exploration activities or feasibility studies, if the contractor believes that any part of the
contract area is likely to contain an economic mineral resource, it shall submit to the DENR secretary
a declaration of mining feasibility (per Clause 5.4 of the FTAA), together with a technical description
of the area delineated for development and production, a description of the proposed mining
operations including the technology to be used, a work program for development, an environmental
impact statement, and a description of the contributions to the economic and general welfare of the
country to be generated by the mining operations (pursuant to Clause 5.5).
The work program for development is subject to the approval of the DENR secretary. Upon its
approval, the contractor must comply with it and complete the development of the mine, including
the construction of production facilities and installation of machinery and equipment, within the
period provided in the approved work program for development (per Clause 6.1).
Thus, notably, the development phase of the project is likewise subject to the control and supervision
of the government. It cannot be emphasized enough that the proper and timely construction and
deployment of the production facilities and the development of the mine are of pivotal significance to
the success of the mining venture. Any missteps here will potentially be very costly to remedy.
Hence, the submission of the work program for development to the DENR secretary for approval is
particularly noteworthy, considering that so many millions of dollars worth of investments -- courtesy
of the contractor -- are made to depend on the State's consideration and action.
Throughout the operating period, the contractor is required to submit to the DENR secretary for
approval, copy furnished the director of MGB, work programs covering each period of three fiscal
years (per Clause 6.2). During the same period (per Clause 6.3), the contractor is mandated to
submit various quarterly and annual reports to the DENR secretary, copy furnished the director of
MGB, on the tonnages of production in terms of ores and concentrates, with corresponding grades,
values and destinations; reports of sales; total ore reserves, total tonnage of ores, work
accomplished and work in progress (installations and facilities related to mining operations),
investments made or committed, and so on and so forth.
Under Section VIII, during the period of mining operations, the contractor is also required to submit
to the DENR secretary (copy furnished the director of MGB) the work program and corresponding
budget for the contract area, describing the mining operations that are proposed to be carried out
during the period covered. The secretary is, of course, entitled to grant or deny approval of any work
program or budget and/or propose revisions thereto. Once the program/budget has been approved,
the contractor shall comply therewith.
In sum, the above provisions of the WMCP FTAA taken together, far from constituting a surrender of
control and a grant of beneficial ownership of mineral resources to the contractor in
question, bestow upon the State more than adequate control and supervision over the
activities of the contractor and the enterprise.
No Surrender of Control
Under the WMCP FTAA
Petitioners, however, take aim at Clause 8.2, 8.3, and 8.5 of the WMCP FTAA which, they say,
amount to a relinquishment of control by the State, since it "cannot truly impose its own discretion" in
respect of the submitted work programs.

"8.2. The Secretary shall be deemed to have approved any Work Programme or Budget or
variation thereofsubmitted by the Contractor unless within sixty (60) days after submission
by the Contractor the Secretary gives notice declining such approval or proposing a revision
of certain features and specifying its reasons therefor ('the Rejection Notice').
8.3. If the Secretary gives a Rejection Notice, the Parties shall promptly meet and endeavor
to agree on amendments to the Work Programme or Budget. If the Secretary and the
Contractor fail to agree on the proposed revision within 30 days from delivery of the
Rejection Notice then the Work Programme or Budget or variation thereof proposed by the
Contractor shall be deemed approved, so as not to unnecessarily delay the performance of
the Agreement.
8.4. x x x x x x x x x
8.5. So far as is practicable, the Contractor shall comply with any approved Work
Programme and Budget. It is recognized by the Secretary and the Contractor that the details
of any Work Programmes or Budgets may require changes in the light of changing
circumstances. The Contractor may make such changes without approval of the Secretary
provided they do not change the general objective of any Work Programme, nor entail a
downward variance of more than twenty per centum (20percent) of the relevant Budget. All
other variations to an approved Work Programme or Budget shall be submitted for approval
of the Secretary."
From the provisions quoted above, petitioners generalize by asserting that the government does not
participate in making critical decisions regarding the operations of the mining firm. Furthermore,
while the State can require the submission of work programs and budgets, the decision of the
contractor will still prevail, if the parties have a difference of opinion with regard to matters affecting
operations and management.
We hold, however, that the foregoing provisions do not manifest a relinquishment of control. For
instance, Clause 8.2 merely provides a mechanism for preventing the business or mining operations
from grinding to a complete halt as a result of possibly over-long and unjustified delays in the
government's handling, processing and approval of submitted work programs and budgets. Anyway,
the provision does give the DENR secretary more than sufficient time (60 days) to react to submitted
work programs and budgets. It cannot be supposed that proper grounds for objecting thereto, if any
exist, cannot be discovered within a period of two months.
On the other hand, Clause 8.3 seeks to provide a temporary, stop-gap solution in the event a
disagreement over the submitted work program or budget arises between the State and the
contractor and results in a stalemate or impasse, in order that there will be no unreasonably long
delays in the performance of the works.
These temporary or stop-gap solutions are not necessarily evil or wrong. Neither does it follow that
the government will inexorably be aggrieved if and when these temporary remedies come into
play. First, avoidance of long delays in these situations will undoubtedly redound to the benefit of the
State as well as the contractor.Second, who is to say that the work program or budget proposed by
the contractor and deemed approved under Clause 8.3 would not be the better or more reasonable
or more effective alternative? The contractor, being the "insider," as it were, may be said to be in a
better position than the State -- an outsider looking in -- to determine what work program or budget
would be appropriate, more effective, or more suitable under the circumstances.

All things considered, we take exception to the characterization of the DENR secretary as a
subservient nonentity whom the contractor can overrule at will, on account of Clause 8.3. And
neither is it true that under the same clause, the DENR secretary has no authority whatsoever to
disapprove the work program. As Respondent WMCP reasoned in its Reply-Memorandum, the State
-- despite Clause 8.3 -- still has control over the contract area and it may, as sovereign authority,
prohibit work thereon until the dispute is resolved. And ultimately, the State may terminate the
agreement, pursuant to Clause 13.2 of the same FTAA, citing substantial breach thereof. Hence, it
clearly retains full and effective control of the exploitation of the mineral resources.
On the other hand, Clause 8.5 is merely an acknowledgment of the parties' need for flexibility, given
that no one can accurately forecast under all circumstances, or predict how situations may change.
Hence, while approved work programs and budgets are to be followed and complied with as far as
practicable, there may be instances in which changes will have to be effected, and effected rapidly,
since events may take shape and unfold with suddenness and urgency. Thus, Clause 8.5 allows the
contractor to move ahead and make changes without the express or implicit approval of the DENR
secretary. Such changes are, however, subject to certain conditions that will serve to limit or restrict
the variance and prevent the contractor from straying very far from what has been approved.
Clause 8.5 provides the contractor a certain amount of flexibility to meet unexpected situations, while
still guaranteeing that the approved work programs and budgets are not abandoned altogether.
Clause 8.5 does not constitute proof that the State has relinquished control. And ultimately, should
there be disagreement with the actions taken by the contractor in this instance as well as under
Clause 8.3 discussed above, the DENR secretary may resort to cancellation/termination of the FTAA
as the ultimate sanction.
Discretion to Select Contract
Area Not an Abdication of Control
Next, petitioners complain that the contractor has full discretion to select -- and the government has
no say whatsoever as to -- the parts of the contract area to be relinquished pursuant to Clause 4.6 of
the WMCP FTAA.56 This clause, however, does not constitute abdication of control. Rather, it is a
mere acknowledgment of the fact that the contractor will have determined, after appropriate
exploration works, which portions of the contract area do not contain minerals in commercial
quantities sufficient to justify developing the same and ought therefore to be relinquished. The State
cannot just substitute its judgment for that of the contractor and dictate upon the latter which areas to
give up.
Moreover, we can be certain that the contractor's self-interest will propel proper and efficient
relinquishment. According to private respondent,57 a mining company tries to relinquish as much nonmineral areas as soon as possible, because the annual occupation fees paid to the government are
based on the total hectarage of the contract area, net of the areas relinquished. Thus, the larger the
remaining area, the heftier the amount of occupation fees to be paid by the contractor. Accordingly,
relinquishment is not an issue, given that the contractor will not want to pay the annual occupation
fees on the non-mineral parts of its contract area. Neither will it want to relinquish promising sites,
which other contractors may subsequently pick up.
Government Not a Subcontractor
Petitioners further maintain that the contractor can compel the government to exercise its power of
eminent domain to acquire surface areas within the contract area for the contractor's use. Clause
10.2 (e) of the WMCP FTAA provides that the government agrees that the contractor shall "(e) have
the right to require the Government at the Contractor's own cost, to purchase or acquire surface

areas for and on behalf of the Contractor at such price and terms as may be acceptable to the
contractor. At the termination of this Agreement such areas shall be sold by public auction or tender
and the Contractor shall be entitled to reimbursement of the costs of acquisition and maintenance,
adjusted for inflation, from the proceeds of sale."
According to petitioners, "government becomes a subcontractor to the contractor" and may, on
account of this provision, be compelled "to make use of its power of eminent domain, not for public
purposes but on behalf of a private party, i.e., the contractor." Moreover, the power of the courts to
determine the amount corresponding to the constitutional requirement of just compensation has
allegedly also been contracted away by the government, on account of the latter's commitment that
the acquisition shall be at such terms as may be acceptable to the contractor.
However, private respondent has proffered a logical explanation for the provision. 58 Section 10.2(e)
contemplates a situation applicable to foreign-owned corporations. WMCP, at the time of the
execution of the FTAA, was a foreign-owned corporation and therefore not qualified to own land. As
contractor, it has at some future date to construct the infrastructure -- the mine processing plant, the
camp site, the tailings dam, and other infrastructure -- needed for the large-scale mining operations.
It will then have to identify and pinpoint, within the FTAA contract area, the particular surface areas
with favorable topography deemed ideal for such infrastructure and will need to acquire the surface
rights. The State owns the mineral deposits in the earth, and is also qualified to own land.
Section 10.2(e) sets forth the mechanism whereby the foreign-owned contractor, disqualified to own
land, identifies to the government the specific surface areas within the FTAA contract area to be
acquired for the mine infrastructure. The government then acquires ownership of the surface land
areas on behalf of the contractor, in order to enable the latter to proceed to fully implement the
FTAA.
The contractor, of course, shoulders the purchase price of the land. Hence, the provision allows it,
after termination of the FTAA, to be reimbursed from proceeds of the sale of the surface areas,
which the government will dispose of through public bidding. It should be noted that this provision
will not be applicable to Sagittarius as the present FTAA contractor, since it is a Filipino corporation
qualified to own and hold land. As such, it may therefore freely negotiate with the surface rights
owners and acquire the surface property in its own right.
Clearly, petitioners have needlessly jumped to unwarranted conclusions, without being aware of the
rationale for the said provision. That provision does not call for the exercise of the power of eminent
domain -- and determination of just compensation is not an issue -- as much as it calls for a qualified
party to acquire the surface rights on behalf of a foreign-owned contractor.
Rather than having the foreign contractor act through a dummy corporation, having the State do the
purchasing is a better alternative. This will at least cause the government to be aware of such
transaction/s and foster transparency in the contractor's dealings with the local property owners. The
government, then, will not act as a subcontractor of the contractor; rather, it will facilitate the
transaction and enable the parties to avoid a technical violation of the Anti-Dummy Law.
Absence of Provision
Requiring Sale at Posted
Prices Not Problematic
The supposed absence of any provision in the WMCP FTAA directly and explicitly requiring the
contractor to sell the mineral products at posted or market prices is not a problem. Apart from Clause
1.4 of the FTAA obligating the contractor to account for the total value of mineral production and the

sale of minerals, we can also look to Section 35 of RA 7942, which incorporates into all FTAAs
certain terms, conditions and warranties, including the following:
"(l) The contractors shall furnish the Government records of geologic, accounting and other
relevant data for its mining operation, and that books of accounts and records shall be open
for inspection by the government. x x x
(m) Requiring the proponent to dispose of the minerals at the highest price and more
advantageous terms and conditions."
For that matter, Section 56(n) of DAO 99-56 specifically obligates an FTAA contractor to dispose of
the minerals and by-products at the highest market price and to register with the MGB a copy of the
sales agreement. After all, the provisions of prevailing statutes as well as rules and regulations are
deemed written into contracts.
Contractor's Right to Mortgage
Not Objectionable Per Se
Petitioners also question the absolute right of the contractor under Clause 10.2 (l) to mortgage and
encumber not only its rights and interests in the FTAA and the infrastructure and improvements
introduced, but also the mineral products extracted. Private respondents do not touch on this matter,
but we believe that this provision may have to do with the conditions imposed by the creditor-banks
of the then foreign contractor WMCP to secure the lendings made or to be made to the latter.
Ordinarily, banks lend not only on the security of mortgages on fixed assets, but also on
encumbrances of goods produced that can easily be sold and converted into cash that can be
applied to the repayment of loans. Banks even lend on the security of accounts receivable that are
collectible within 90 days.59
It is not uncommon to find that a debtor corporation has executed deeds of assignment "by way of
security" over the production for the next twelve months and/or the proceeds of the sale thereof -- or
the corresponding accounts receivable, if sold on terms -- in favor of its creditor-banks. Such deeds
may include authorizing the creditors to sell the products themselves and to collect the sales
proceeds and/or the accounts receivable.
Seen in this context, Clause 10.2(l) is not something out of the ordinary or objectionable. In any
case, as will be explained below, even if it is allowed to mortgage or encumber the mineral endproducts themselves, the contractor is not freed of its obligation to pay the government its basic and
additional shares in the net mining revenue, which is the essential thing to consider.
In brief, the alarum raised over the contractor's right to mortgage the minerals is simply unwarranted.
Just the same, the contractor must account for the value of mineral production and the sales
proceeds therefrom. Likewise, under the WMCP FTAA, the government remains entitled to its sixty
percent share in the net mining revenues of the contractor. The latter's right to mortgage the
minerals does not negate the State's right to receive its share of net mining revenues.
Shareholders Free to Sell Their Stocks
Petitioners likewise criticize Clause 10.2(k), which gives the contractor authority "to change its equity
structure at any time." This provision may seem somewhat unusual, but considering that WMCP then
was 100 percent foreign-owned, any change would mean that such percentage would either stay

unaltered or be decreased in favor of Filipino ownership. Moreover, the foreign-held shares may
change hands freely. Such eventuality is as it should be.
We believe it is not necessary for government to attempt to limit or restrict the freedom of the
shareholders in the contractor to freely transfer, dispose of or encumber their shareholdings,
consonant with the unfettered exercise of their business judgment and discretion. Rather, what is
critical is that, regardless of the identity, nationality and percentage ownership of the various
shareholders of the contractor -- and regardless of whether these shareholders decide to take the
company public, float bonds and other fixed-income instruments, or allow the creditor-banks to take
an equity position in the company -- the foreign-owned contractor is always in a position to render
the services required under the FTAA, under the direction and control of the government.
Contractor's Right to Ask
For Amendment Not Absolute
With respect to Clauses 10.4(e) and (i), petitioners complain that these provisions bind government
to allow amendments to the FTAA if required by banks and other financial institutions as part of the
conditions for new lendings. However, we do not find anything wrong with Clause 10.4(e), which only
states that "if the Contractor seeks to obtain financing contemplated herein from banks or other
financial institutions, (the Government shall) cooperate with the Contractor in such efforts provided
that such financing arrangements will in no event reduce the Contractor's obligations or the
Government's rights hereunder." The colatilla obviously safeguards the State's interests; if breached,
it will give the government cause to object to the proposed amendments.
On the other hand, Clause 10.4(i) provides that "the Government shall favourably consider any
request from [the] Contractor for amendments of this Agreement which are necessary in order for
the Contractor to successfully obtain the financing." Petitioners see in this provision a complete
renunciation of control. We disagree.
The proviso does not say that the government shall grant any request for amendment. Clause 10.4(i)
only obliges the State to favorably consider any such request, which is not at all unreasonable, as it
is not equivalent to saying that the government must automatically consent to it. This provision
should be read together with the rest of the FTAA provisions instituting government control and
supervision over the mining enterprise. The clause should not be given an interpretation that enables
the contractor to wiggle out of the restrictions imposed upon it by merely suggesting that certain
amendments are requested by the lenders.
Rather, it is up to the contractor to prove to the government that the requested changes to the FTAA
are indispensable, as they enable the contractor to obtain the needed financing; that without such
contract changes, the funders would absolutely refuse to extend the loan; that there are no other
sources of financing available to the contractor (a very unlikely scenario); and that without the
needed financing, the execution of the work programs will not proceed. But the bottom line is, in the
exercise of its power of control, the government has thefinal say on whether to approve or
disapprove such requested amendments to the FTAA. In short, approval thereof is not mandatory on
the part of the government.
In fine, the foregoing evaluation and analysis of the aforementioned FTAA provisions
sufficiently overturns petitioners' litany of objections to and criticisms of the State's alleged
lack of control.
Financial Benefits Not
Surrendered to the Contractor

One of the main reasons certain provisions of RA 7942 were struck down was the finding mentioned
in the Decision that beneficial ownership of the mineral resources had been conveyed to the
contractor. This finding was based on the underlying assumption, common to the said provisions,
that the foreign contractor manages the mineral resources in the same way that foreign contractors
in service contracts used to. "By allowing foreign contractors to manage or operate all the aspects of
the mining operation, the above-cited provisions of R.A. No. 7942 have in effect conveyed
beneficial ownership over the nation's mineral resources to these contractors, leaving the State
with nothing but bare title thereto."60 As the WMCP FTAA contained similar provisions deemed by
the ponente to be abhorrent to the Constitution, the Decision struck down the Contract as well.
Beneficial ownership has been defined as ownership recognized by law and capable of being
enforced in the courts at the suit of the beneficial owner.61 Black's Law Dictionary indicates that the
term is used in two senses:first, to indicate the interest of a beneficiary in trust property (also called
"equitable ownership"); and second, to refer to the power of a corporate shareholder to buy or sell
the shares, though the shareholder is not registered in the corporation's books as the
owner.62 Usually, beneficial ownership is distinguished from naked ownership, which is the enjoyment
of all the benefits and privileges of ownership, as against possession of the bare title to property.
An assiduous examination of the WMCP FTAA uncovers no indication that it confers upon WMCP
ownership, beneficial or otherwise, of the mining property it is to develop, the minerals to be
produced, or the proceeds of their sale, which can be legally asserted and enforced as against the
State.
As public respondents correctly point out, any interest the contractor may have in the proceeds of
the mining operation is merely the equivalent of the consideration the government has undertaken to
pay for its services. All lawful contracts require such mutual prestations, and the WMCP FTAA is no
different. The contractor commits to perform certain services for the government in respect of the
mining operation, and in turn it is to be compensated out of the net mining revenues generated from
the sale of mineral products. What would be objectionable is a contractual provision that unduly
benefits the contractor far in excess of the service rendered or value delivered, if any, in exchange
therefor.
A careful perusal of the statute itself and its implementing rules reveals that neither RA 7942 nor
DAO 99-56 can be said to convey beneficial ownership of any mineral resource or product to any
foreign FTAA contractor.
Equitable Sharing
of Financial Benefits
On the contrary, DAO 99-56, entitled "Guidelines Establishing the Fiscal Regime of Financial or
Technical Assistance Agreements" aims to ensure an equitable sharing of the benefits derived from
mineral resources. These benefits are to be equitably shared among the government (national and
local), the FTAA contractor, and the affected communities. The purpose is to ensure sustainable
mineral resources development; and a fair, equitable, competitive and stable investment regime for
the large-scale exploration, development and commercial utilization of minerals. The general
framework or concept followed in crafting the fiscal regime of the FTAA is based on the principle that
the government expects real contributions to the economic growth and general welfare of the
country, while the contractor expects a reasonable return on its investments in the project. 63
Specifically, under the fiscal regime, the government's expectation is, inter alia, the receipt of its
share from the taxes and fees normally paid by a mining enterprise. On the other hand, the FTAA
contractor is granted by the government certain fiscal and non-fiscal incentives 64 to help support the

former's cash flow during the most critical phase (cost recovery) and to make the Philippines
competitive with other mineral-producing countries. After the contractor has recovered its initial
investment, it will pay all the normal taxes and fees comprising the basic share of the government,
plus an additional share for the government based on the options and formulae set forth in DAO 9956.
The said DAO spells out the financial benefits the government will receive from an FTAA, referred to
as "the Government Share," composed of a basic government share and an additional
government share.
The basic government share is comprised of all direct taxes, fees and royalties, as well as other
payments made by the contractor during the term of the FTAA. These are amounts paid directly to (i)
the national government (through the Bureau of Internal Revenue, Bureau of Customs, Mines &
Geosciences Bureau and other national government agencies imposing taxes or fees), (ii) the local
government units where the mining activity is conducted, and (iii) persons and communities directly
affected by the mining project. The major taxes and other payments constituting the basic
government share are enumerated below:65
Payments to the National Government:
Excise tax on minerals - 2 percent of the gross output of mining operations
Contractor' income tax - maximum of 32 percent of taxable income for corporations
Customs duties and fees on imported capital equipment -the rate is set by the Tariff
and Customs Code (3-7 percent for chemicals; 3-10 percent for explosives; 3-15
percent for mechanical and electrical equipment; and 3-10 percent for vehicles,
aircraft and vessels
VAT on imported equipment, goods and services 10 percent of value
Royalties due the government on minerals extracted from mineral reservations, if
applicable 5 percent of the actual market value of the minerals produced
Documentary stamp tax - the rate depends on the type of transaction
Capital gains tax on traded stocks - 5 to 10 percent of the value of the shares
Withholding tax on interest payments on foreign loans -15 percent of the amount of
interest
Withholding tax on dividend payments to foreign stockholders 15 percent of the
dividend
Wharfage and port fees
Licensing fees (for example, radio permit, firearms permit, professional fees)
Other national taxes and fees.
Payments to Local Governments:

Local business tax - a maximum of 2 percent of gross sales or receipts (the rate
varies among local government units)
Real property tax - 2 percent of the fair market value of the property, based on an
assessment level set by the local government
Special education levy - 1 percent of the basis used for the real property tax
Occupation fees - PhP50 per hectare per year; PhP100 per hectare per year if
located in a mineral reservation
Community tax - maximum of PhP10,500 per year
All other local government taxes, fees and imposts as of the effective date of the
FTAA - the rate and the type depend on the local government
Other Payments:
Royalty to indigenous cultural communities, if any 1 percent of gross output from
mining operations
Special allowance - payment to claim owners and surface rights holders
Apart from the basic share, an additional government share is also collected from the FTAA
contractor in accordance with the second paragraph of Section 81 of RA 7942, which provides that
the government share shall be comprised of, among other things, certain taxes, duties and fees. The
subject proviso reads:
"The Government share in a financial or technical assistance agreement shall consist of, among
other things, the contractor's corporate income tax, excise tax, special allowance, withholding tax
due from the contractor's foreign stockholders arising from dividend or interest payments to the said
foreign stockholder in case of a foreign national, and all such other taxes, duties and fees as
provided for under existing laws." (Bold types supplied.)
The government, through the DENR and the MGB, has interpreted the insertion of the
phrase among other thingsas signifying that the government is entitled to an "additional government
share" to be paid by the contractor apart from the "basic share," in order to attain a fifty-fifty sharing
of net benefits from mining.
The additional government share is computed by using one of three options or schemes
presented in DAO 99-56: (1) a fifty-fifty sharing in the cumulative present value of cash flows; (2) the
share based on excess profits; and (3) the sharing based on the cumulative net mining revenue. The
particular formula to be applied will be selected by the contractor, with a written notice to the
government prior to the commencement of the development and construction phase of the mining
project.66
Proceeds from the government shares arising from an FTAA contract are distributed to and received
by the different levels of government in the following proportions:

National Government

50 percent

Provincial
Government

10 percent

Municipal
Government

20 percent

Affected Barangays

20 percent

The portion of revenues remaining after the deduction of the basic and additional government shares
is what goes to the contractor.
Government's Share in an
FTAA Not Consisting Solely
of Taxes, Duties and Fees
In connection with the foregoing discussion on the basic and additional government shares, it is
pertinent at this juncture to mention the criticism leveled at the second paragraph of Section 81 of
RA 7942, quoted earlier. The said proviso has been denounced, because, allegedly, the State's
share in FTAAs with foreign contractors has been limited to taxes, fees and duties only; in effect, the
State has been deprived of a share in the after-tax income of the enterprise. In the face of this
allegation, one has to consider that the law does not define the termamong other things; and the
Office of the Solicitor General, in its Motion for Reconsideration, appears to have erroneously
claimed that the phrase refers to indirect taxes.
The law provides no definition of the term among other things, for the reason that Congress
deliberately avoided setting unnecessary limitations as to what may constitute compensation to the
State for the exploitation and use of mineral resources. But the inclusion of that phrase clearly and
unmistakably reveals the legislative intent to have the State collect more than just the usual taxes,
duties and fees. Certainly, there is nothing in that phrase -- or in the second paragraph of Section 81
-- that would suggest that such phrase should be interpreted as referring only to taxes, duties, fees
and the like.
Precisely for that reason, to fulfill the legislative intent behind the inclusion of the phrase among
other things in the second paragraph of Section 81,67 the DENR structured and formulated in DAO
99-56 the said additional government share. Such a share was to consist not of taxes, but of a
share in the earnings or cash flows of the mining enterprise. The additional government share
was to be paid by the contractor on top of the basic share, so as to achieve a fifty-fifty sharing
-- between the government and the contractor -- of net benefits from mining. In the Ramos-DeVera
paper, the explanation of the three options or formulas68 -- presented in DAO 99-56 for the
computation of the additional government share -- serves to debunk the claim that the government's
take from an FTAA consists solely of taxes, fees and duties.

Unfortunately, the Office of the Solicitor General -- although in possession of the relevant data -failed to fully replicate or echo the pertinent elucidation in the Ramos-DeVera paper regarding the
three schemes or options for computing the additional government share presented in DAO 99-56.
Had due care been taken by the OSG, the Court would have been duly apprised of the real nature
and particulars of the additional share.
But, perhaps, on account of the esoteric discussion in the Ramos-DeVera paper, and the even more
abstruse mathematical jargon employed in DAO 99-56, the OSG omitted any mention of the three
options. Instead, the OSG skipped to a side discussion of the effect of indirect taxes, which
had nothing at all to do with the additional government share, to begin with. Unfortunately, this move
created the wrong impression, pointed out in Justice Antonio T. Carpio's Opinion, that the OSG had
taken the position that the additional government share consisted of indirect taxes.
In any event, what is quite evident is the fact that the additional government share, as formulated,
has nothing to do with taxes -- direct or indirect -- or with duties, fees or charges. To repeat, it is over
and above the basic government share composed of taxes and duties. Simply put, the additional
share may be (a) an amount that will result in a 50-50 sharing of the cumulative present value of
the cash flows69 of the enterprise; (b) an amount equivalent to 25 percent of the additional or excess
profits of the enterprise, reckoned against a benchmark return on investments; or (c) an amount that
will result in a fifty-fifty sharing of the cumulative net mining revenue from the end of the recovery
period up to the taxable year in question. The contractor is required to select one of the three options
or formulae for computing the additional share, an option it will apply to all of its mining operations.
As used above, "net mining revenue" is defined as the gross output from mining operations for a
calendar year, less deductible expenses (inclusive of taxes, duties and fees). Such revenue would
roughly be equivalent to "taxable income" or income before income tax. Definitely, as compared with,
say, calculating the additional government share on the basis of net income (after income tax), the
net mining revenue is a better and much more reasonable basis for such computation, as it gives a
truer picture of the profitability of the company.
To demonstrate that the three options or formulations will operate as intended, Messrs. Ramos and
de Vera also performed some quantifications of the government share via a financial modeling of
each of the three options discussed above. They found that the government would get the highest
share from the option that is based on the net mining revenue, as compared with the other two
options, considering only the basic and the additional shares; and that, even though production rate
decreases, the government share will actually increase when the net mining revenue and the
additional profit-based options are used.
Furthermore, it should be noted that the three options or formulae do not yet take into account the
indirect taxes70 and other financial contributions71 of mining projects. These indirect taxes and other
contributions are real and actual benefits enjoyed by the Filipino people and/or government. Now, if
some of the quantifiable items are taken into account in the computations, the financial modeling
would show that the total government share increases to 60 percent or higher -- in one instance, as
much as 77 percent and even 89 percent -- of the net present value of total benefits from the project.
As noted in the Ramos-DeVera paper, these results are not at all shabby, considering that the
contractor puts in all the capital requirements and assumes all the risks, without the government
having to contribute or risk anything.
Despite the foregoing explanation, Justice Carpio still insisted during the Court's deliberations that
the phraseamong other things refers only to taxes, duties and fees. We are bewildered by his
position. On the one hand, he condemns the Mining Law for allegedly limiting the government's
benefits only to taxes, duties and fees; and on the other, he refuses to allow the State to benefit from

the correct and proper interpretation of the DENR/MGB. To remove all doubts then, we hold that the
State's share is not limited to taxes, duties and fees only and that the DENR/MGB interpretation of
the phrase among other things is correct. Definitely, this DENR/MGB interpretation is not only legally
sound, but also greatly advantageous to the government.
One last point on the subject. The legislature acted judiciously in not defining the terms among other
things and, instead, leaving it to the agencies concerned to devise and develop the various modes of
arriving at a reasonable and fair amount for the additional government share. As can be seen from
DAO 99-56, the agencies concerned did an admirable job of conceiving and developing not just one
formula, but three different formulae for arriving at the additional government share. Each of these
options is quite fair and reasonable; and, as Messrs. Ramos and De Vera stated, other alternatives
or schemes for a possible improvement of the fiscal regime for FTAAs are also being studied by the
government.
Besides, not locking into a fixed definition of the term among other things will ultimately be more
beneficial to the government, as it will have that innate flexibility to adjust to and cope with rapidly
changing circumstances, particularly those in the international markets. Such flexibility is especially
significant for the government in terms of helping our mining enterprises remain competitive in world
markets despite challenging and shifting economic scenarios.
In conclusion, we stress that we do not share the view that in FTAAs with foreign contractors
under RA 7942, the government's share is limited to taxes, fees and duties. Consequently, we
find the attacks on the second paragraph of Section 81 of RA 7942 totally unwarranted.
Collections Not Made Uncertain
by the Third Paragraph of Section 81
The third or last paragraph of Section 8172 provides that the government share in FTAAs shall be
collected when the contractor shall have recovered its pre-operating expenses and exploration and
development expenditures. The objection has been advanced that, on account of the proviso, the
collection of the State's share is not even certain, as there is no time limit in RA 7942 for this grace
period or recovery period.
We believe that Congress did not set any time limit for the grace period, preferring to leave it to the
concerned agencies, which are, on account of their technical expertise and training, in a better
position to determine the appropriate durations for such recovery periods. After all, these recovery
periods are determined, to a great extent, by technical and technological factors peculiar to the
mining industry. Besides, with developments and advances in technology and in the geosciences,
we cannot discount the possibility of shorter recovery periods. At any rate, the concerned agencies
have not been remiss in this area. The 1995 and 1996 Implementing Rules and Regulations of RA
7942 specify that the period of recovery, reckoned from the date of commercial operation, shall be
for a period not exceeding five years, or until the date of actual recovery, whichever comes earlier.
Approval of Pre-Operating
Expenses Required by RA 7942
Still, RA 7942 is criticized for allegedly not requiring government approval of pre-operating,
exploration and development expenses of the foreign contractors, who are in effect given unfettered
discretion to determine the amounts of such expenses. Supposedly, nothing prevents the contractors
from recording such expenses in amounts equal to the mining revenues anticipated for the first 10 or
15 years of commercial production, with the result that the share of the State will be zero for the first

10 or 15 years. Moreover, under the circumstances, the government would be unable to say when it
would start to receive its share under the FTAA.
We believe that the argument is based on incorrect information as well as speculation. Obviously,
certain crucial provisions in the Mining Law were overlooked. Section 23, dealing with the rights and
obligations of the exploration permit grantee, states: "The permittee shall undertake exploration work
on the area as specified by its permit based on an approved work program." The next proviso
reads: "Any expenditure in excess of the yearly budget of the approved work program may be
carried forward and credited to the succeeding years covering the duration of the permit. x x
x." (underscoring supplied)
Clearly, even at the stage of application for an exploration permit, the applicant is required to submit
-- for approval by the government -- a proposed work program for exploration, containing a yearly
budget of proposed expenditures. The State has the opportunity to pass upon (and approve or
reject) such proposed expenditures, with the foreknowledge that -- if approved -- these will
subsequently be recorded as pre-operating expenses that the contractor will have to recoup over the
grace period. That is not all.
Under Section 24, an exploration permit holder who determines the commercial viability of a project
covering a mining area may, within the term of the permit, file with the Mines and Geosciences
Bureau a declaration of mining project feasibility. This declaration is to be accompanied by a work
program for development for the Bureau's approval, the necessary prelude for entering into an
FTAA, a mineral production sharing agreement (MPSA), or some other mineral agreement. At this
stage, too, the government obviously has the opportunity to approve or reject the proposed work
program and budgeted expenditures for development works on the project. Such expenditures will
ultimately become the pre-operating and development costs that will have to be recovered by the
contractor.
Naturally, with the submission of approved work programs and budgets for the exploration and the
development/construction phases, the government will be able to scrutinize and approve or
reject such expenditures. It will be well-informed as to the amounts of pre-operating and other
expenses that the contractor may legitimately recover and the approximate period of time needed to
effect such a recovery. There is therefore no way the contractor can just randomly post any amount
of pre-operating expenses and expect to recover the same.
The aforecited provisions on approved work programs and budgets have counterparts in Section 35,
which deals with the terms and conditions exclusively applicable to FTAAs. The said provision
requires certain terms and conditions to be incorporated into FTAAs; among them, "a firm
commitment x x x of an amount corresponding to the expenditure obligation that will be invested in
the contract area" and "representations and warranties x x x to timely deploy these [financing,
managerial and technical expertise and technological] resources under its supervision pursuant to
the periodic work programs and related budgets x x x," as well as "work programs andminimum
expenditures commitments." (underscoring supplied)
Unarguably, given the provisions of Section 35, the State has every opportunity to pass upon the
proposed expenditures under an FTAA and approve or reject them. It has access to all the
information it may need in order to determine in advance the amounts of pre-operating and
developmental expenses that will have to be recovered by the contractor and the amount of time
needed for such recovery.
In summary, we cannot agree that the third or last paragraph of Section 81 of RA 7942 is in
any manner unconstitutional.

No Deprivation of Beneficial Rights


It is also claimed that aside from the second and the third paragraphs of Section 81 (discussed
above), Sections 80, 84 and 112 of RA 7942 also operate to deprive the State of beneficial rights of
ownership over mineral resources; and give them away for free to private business enterprises
(including foreign owned corporations). Likewise, the said provisions have been construed as
constituting, together with Section 81, an ingenious attempt to resurrect the old and discredited
system of "license, concession or lease."
Specifically, Section 80 is condemned for limiting the State's share in a mineral production-sharing
agreement (MPSA) to just the excise tax on the mineral product. Under Section 151(A) of the Tax
Code, such tax is only 2 percent of the market value of the gross output of the minerals.
The colatilla in Section 84, the portion considered offensive to the Constitution, reiterates the same
limitation made in Section 80.73
It should be pointed out that Section 80 and the colatilla in Section 84 pertain only to MPSAs and
have no application to FTAAs. These particular statutory provisions do not come within the issues
that were defined and delineated by this Court during the Oral Argument -- particularly the third
issue, which pertained exclusively to FTAAs. Neither did the parties argue upon them in their
pleadings. Hence, this Court cannot make any pronouncement in this case regarding the
constitutionality of Sections 80 and 84 without violating the fundamental rules of due process.
Indeed, the two provisos will have to await another case specifically placing them in issue.
On the other hand, Section 11274 is disparaged for allegedly reverting FTAAs and all mineral
agreements to the old and discredited "license, concession or lease" system. This Section states in
relevant part that "the provisions of Chapter XIV [which includes Sections 80 to 82] on government
share in mineral production-sharing agreement x x x shall immediately govern and apply to a mining
lessee or contractor." (underscoring supplied) This provision is construed as signifying that the 2
percent excise tax which, pursuant to Section 80, comprises the government share in MPSAs shall
now also constitute the government share in FTAAs -- as well as in co-production agreements and
joint venture agreements -- to the exclusion of revenues of any other nature or from any other
source.
Apart from the fact that Section 112 likewise does not come within the issues delineated by this
Court during the Oral Argument, and was never touched upon by the parties in their pleadings, it
must also be noted that the criticism hurled against this Section is rooted in unwarranted conclusions
made without considering other relevant provisions in the statute. Whether Section 112 may properly
apply to co-production or joint venture agreements, the fact of the matter is that it cannot be made to
apply to FTAAs.
First, Section 112 does not specifically mention or refer to FTAAs; the only reason it is being applied
to them at all is the fact that it happens to use the word "contractor." Hence, it is a bit of a stretch to
insist that it covers FTAAs as well. Second, mineral agreements, of which there are three types -MPSAs, co-production agreements, and joint venture agreements -- are covered by Chapter V of RA
7942. On the other hand, FTAAs are covered by and in fact are the subject of Chapter VI, an entirely
different chapter altogether. The law obviously intends to treat them as a breed apart from mineral
agreements, since Section 35 (found in Chapter VI) creates a long list of specific terms, conditions,
commitments, representations and warranties -- which have not been made applicable to mineral
agreements -- to be incorporated into FTAAs.
Third, under Section 39, the FTAA contractor is given the option to "downgrade" -- to convert the
FTAA into a mineral agreement at any time during the term if the economic viability of the contract

area is inadequate to sustain large-scale mining operations. Thus, there is no reason to think that
the law through Section 112 intends to exact from FTAA contractors merely the same government
share (a 2 percent excise tax) that it apparently demands from contractors under the three forms of
mineral agreements. In brief, Section 112 does not apply to FTAAs.
Notwithstanding the foregoing explanation, Justices Carpio and Morales maintain that the Court
must rule now on the constitutionality of Sections 80, 84 and 112, allegedly because the WMCP
FTAA contains a provision which grants the contractor unbridled and "automatic" authority to convert
the FTAA into an MPSA; and should such conversion happen, the State would be prejudiced since
its share would be limited to the 2 percent excise tax. Justice Carpio adds that there are five MPSAs
already signed just awaiting the judgment of this Court on respondents' and intervenor's Motions for
Reconsideration. We hold however that, at this point, this argument is based on pure speculation.
The Court cannot rule on mere surmises and hypothetical assumptions, without firm factual anchor.
We repeat: basic due process requires that we hear the parties who have a real legal interest in the
MPSAs (i.e. the parties who executed them) before these MPSAs can be reviewed, or worse, struck
down by the Court. Anything less than that requirement would be arbitrary and capricious.
In any event, the conversion of the present FTAA into an MPSA is problematic. First, the contractor
must comply with the law, particularly Section 39 of RA 7942; inter alia, it must convincingly show
that the "economic viability of the contract is found to be inadequate to justify large-scale mining
operations;" second, it must contend with the President's exercise of the power of State control over
the EDU of natural resources; and third, it will have to risk a possible declaration of the
unconstitutionality (in a proper case) of Sections 80, 84 and 112.
The first requirement is not as simple as it looks. Section 39 contemplates a situation in which an
FTAA has already been executed and entered into, and is presumably being implemented, when the
contractor "discovers" that the mineral ore reserves in the contract area are not sufficient to justify
large-scale mining, and thus the contractor requests the conversion of the FTAA into an MPSA. The
contractor in effect needs to explain why, despite its exploration activities, including the conduct of
various geologic and other scientific tests and procedures in the contract area, it was unable to
determine correctly the mineral ore reserves and the economic viability of the area. The contractor
must explain why, after conducting such exploration activities, it decided to file a declaration of
mining feasibility, and to apply for an FTAA, thereby leading the State to believe that the area could
sustain large-scale mining. The contractor must justify fully why its earlier findings, based on
scientific procedures, tests and data, turned out to be wrong, or were way off. It must likewise prove
that its new findings, also based on scientific tests and procedures, are correct. Right away, this puts
the contractor's technical capabilities and expertise into serious doubt. We wonder if anyone would
relish being in this situation. The State could even question and challenge the contractor's
qualification and competence to continue the activity under an MPSA.
All in all, while there may be cogent grounds to assail the aforecited Sections, this Court -- on
considerations of due process -- cannot rule upon them here. Anyway, if later on these
Sections are declared unconstitutional, such declaration will not affect the other portions
since they are clearly separable from the rest.
Our Mineral Resources Not
Given Away for Free by RA 7942
Nevertheless, if only to disabuse our minds, we should address the contention that our mineral
resources are effectively given away for free by the law (RA 7942) in general and by Sections 80, 81,
84 and 112 in particular.

Foreign contractors do not just waltz into town one day and leave the next, taking away mineral
resources without paying anything. In order to get at the minerals, they have to invest huge sums of
money (tens or hundreds of millions of dollars) in exploration works first. If the exploration proves
unsuccessful, all the cash spent thereon will not be returned to the foreign investors; rather, those
funds will have been infused into the local economy, to remain there permanently. The benefits
therefrom cannot be simply ignored. And assuming that the foreign contractors are successful in
finding ore bodies that are viable for commercial exploitation, they do not just pluck out the minerals
and cart them off. They have first to build camp sites and roadways; dig mine shafts and connecting
tunnels; prepare tailing ponds, storage areas and vehicle depots; install their machinery and
equipment, generator sets, pumps, water tanks and sewer systems, and so on.
In short, they need to expend a great deal more of their funds for facilities, equipment and supplies,
fuel, salaries of local labor and technical staff, and other operating expenses. In the meantime, they
also have to pay taxes,75duties, fees, and royalties. All told, the exploration, pre-feasibility, feasibility,
development and construction phases together add up to as many as eleven years. 76 The
contractors have to continually shell out funds for the duration of over a decade, before they can
commence commercial production from which they would eventually derive revenues. All that money
translates into a lot of "pump-priming" for the local economy.
Granted that the contractors are allowed subsequently to recover their pre-operating expenses, still,
that eventuality will happen only after they shall have first put out the cash and fueled the economy.
Moreover, in the process of recouping their investments and costs, the foreign contractors do not
actually pull out the money from the economy. Rather, they recover or recoup their investments out
of actual commercial production by not paying a portion of the basic government share
corresponding to national taxes, along with the additional government share, for a period of not more
than five years77 counted from the commencement of commercial production.
It must be noted that there can be no recovery without commencing actual commercial production. In
the meantime that the contractors are recouping costs, they need to continue operating; in order to
do so, they have to disburse money to meet their various needs. In short, money is continually
infused into the economy.
The foregoing discussion should serve to rid us of the mistaken belief that, since the foreign
contractors are allowed to recover their investments and costs, the end result is that they practically
get the minerals for free, which leaves the Filipino people none the better for it.
All Businesses Entitled
to Cost Recovery
Let it be put on record that not only foreign contractors, but all businessmen and all business entities
in general, have to recoup their investments and costs. That is one of the first things a student learns
in business school. Regardless of its nationality, and whether or not a business entity has a five-year
cost recovery period, it will -- must -- have to recoup its investments, one way or another. This is just
common business sense. Recovery of investments is absolutely indispensable for business survival;
and business survival ensures soundness of the economy, which is critical and contributory to the
general welfare of the people. Even government corporations must recoup their investments in order
to survive and continue in operation. And, as the preceding discussion has shown, there is no
business that gets ahead or earns profits without any cost to it.
It must also be stressed that, though the State owns vast mineral wealth, such wealth is not readily
accessible or transformable into usable and negotiable currency without the intervention of the
credible mining companies. Those untapped mineral resources, hidden beneath tons of earth and

rock, may as well not be there for all the good they do us right now. They have first to be extracted
and converted into marketable form, and the country needs the foreign contractor's funds,
technology and know-how for that.
After about eleven years of pre-operation and another five years for cost recovery, the foreign
contractors will have just broken even. Is it likely that they would at that point stop their operations
and leave? Certainly not. They have yet to make profits. Thus, for the remainder of the contract term,
they must strive to maintain profitability. During this period, they pay the whole of the basic
government share and the additional government share which, taken together with indirect taxes
and other contributions, amount to approximately 60 percent or more of the entire financial benefits
generated by the mining venture.
In sum, we can hardly talk about foreign contractors taking our mineral resources for free. It takes a
lot of hard cash to even begin to do what they do. And what they do in this country ultimately
benefits the local economy, grows businesses, generates employment, and creates infrastructure, as
discussed above. Hence, we definitely disagree with the sweeping claim that no FTAA under Section
81 will ever make any real contribution to the growth of the economy or to the general welfare of the
country. This is not a plea for foreign contractors. Rather, this is a question of focusing the judicial
spotlight squarely on all the pertinent facts as they bear upon the issue at hand, in order to avoid
leaping precipitately to ill-conceived conclusions not solidly grounded upon fact.
Repatriation of After-Tax Income
Another objection points to the alleged failure of the Mining Law to ensure real contributions to the
economic growth and general welfare of the country, as mandated by Section 2 of Article XII of the
Constitution. Pursuant to Section 81 of the law, the entire after-tax income arising from the
exploitation of mineral resources owned by the State supposedly belongs to the foreign contractors,
which will naturally repatriate the said after-tax income to their home countries, thereby resulting in
no real contribution to the economic growth of this country. Clearly, this contention is premised on
erroneous assumptions.
First, as already discussed in detail hereinabove, the concerned agencies have correctly interpreted
the second paragraph of Section 81 of RA 7942 to mean that the government is entitled to an
additional share, to be computed based on any one of the following factors: net mining revenues, the
present value of the cash flows, or excess profits reckoned against a benchmark rate of return on
investments. So it is not correct to say that all of the after-tax income will accrue to the foreign FTAA
contractor, as the government effectively receives a significant portion thereof.
Second, the foreign contractors can hardly "repatriate the entire after-tax income to their home
countries." Even a bit of knowledge of corporate finance will show that it will be impossible to
maintain a business as a "going concern" if the entire "net profit" earned in any particular year will be
taken out and repatriated. The "net income" figure reflected in the bottom line is a mere accounting
figure not necessarily corresponding to cash in the bank, or other quick assets. In order to produce
and set aside cash in an amount equivalent to the bottom line figure, one may need to sell off assets
or immediately collect receivables or liquidate short-term investments; but doing so may very likely
disrupt normal business operations.
In terms of cash flows, the funds corresponding to the net income as of a particular point in time
are actually in use in the normal course of business operations. Pulling out such net income disrupts
the cash flows and cash position of the enterprise and, depending on the amount being taken out,
could seriously cripple or endanger the normal operations and financial health of the business
enterprise. In short, no sane business person, concerned with maintaining the mining

enterprise as a going concern and keeping a foothold in its market, can afford to repatriate
the entire after-tax income to the home country.
The State's Receipt of Sixty
Percent of an FTAA Contractor's
After-Tax Income Not Mandatory
We now come to the next objection which runs this way: In FTAAs with a foreign contractor, the
State must receive at least 60 percent of the after-tax income from the exploitation of its mineral
resources. This share is the equivalent of the constitutional requirement that at least 60 percent of
the capital, and hence 60 percent of the income, of mining companies should remain in Filipino
hands.
First, we fail to see how we can properly conclude that the Constitution mandates the State to
extract at least 60 percent of the after-tax income from a mining company run by a foreign
contractor. The argument is that the Charter requires the State's partner in a co-production
agreement, joint venture agreement or MPSA to be a Filipino corporation (at least 60 percent owned
by Filipino citizens).
We question the logic of this reasoning, premised on a supposedly parallel or analogous situation.
We are, after all, dealing with an essentially different equation, one that involves different
elements. The Charter did not intend to fix an iron-clad rule on the 60 percent share,
applicable to all situations at all times and in all circumstances. If ever such was the intention of
the framers, they would have spelt it out in black and white.Verba legis will serve to dispel
unwarranted and untenable conclusions.
Second, if we would bother to do the math, we might better appreciate the impact (and
reasonableness) of what we are demanding of the foreign contractor. Let us use
a simplified illustration. Let us base it on gross revenues of, say, P500. After deducting operating
expenses, but prior to income tax, suppose a mining firm makes ataxable income of P100. A
corporate income tax of 32 percent results in P32 of taxable income going to the government,
leaving the mining firm with P68. Government then takes 60 percent thereof, equivalent to P40.80,
leaving only P27.20 for the mining firm.
At this point the government has pocketed P32.00 plus P40.80, or a total of P72.80 for every P100
of taxable income, leaving the mining firm with only P27.20. But that is not all. The government has
also taken 2 percent excise tax "off the top," equivalent to another P10. Under the minimum 60
percent proposal, the government nets around P82.80 (not counting other taxes, duties, fees and
charges) from a taxable income of P100 (assuming gross revenues of P500, for purposes of
illustration). On the other hand, the foreign contractor, which provided all the capital, equipment and
labor, and took all the entrepreneurial risks -- receives P27.20. One cannot but wonder whether such
a distribution is even remotely equitable and reasonable, considering the nature of the mining
business. The amount of P82.80 out of P100.00 is really a lot it does not matter that we call part of
it excise taxor income tax, and another portion thereof income from exploitation of mineral
resources. Some might think it wonderful to be able to take the lion's share of the benefits. But we
have to ask ourselves if we are really serious in attracting the investments that are the indispensable
and key element in generating the monetary benefits of which we wish to take the lion's
share. Fairness is a credo not only in law, but also in business.
Third, the 60 percent rule in the petroleum industry cannot be insisted upon at all times in the mining
business. The reason happens to be the fact that in petroleum operations, the bulk of expenditures
is in exploration, but once the contractor has found and tapped into the deposit, subsequent

investments and expenditures are relatively minimal. The crude (or gas) keeps gushing out, and the
work entailed is just a matter of piping, transporting and storing. Not so in mineral mining. The ore
body does not pop out on its own. Even after it has been located, the contractor must continually
invest in machineries and expend funds to dig and build tunnels in order to access and extract the
minerals from underneath hundreds of tons of earth and rock.
As already stated, the numerous intrinsic differences involved in their respective operations and
requirements, cost structures and investment needs render it highly inappropriate to use petroleum
operations FTAAs as benchmarks for mining FTAAs. Verily, we cannot just ignore the realities of
the distinctly different situations and stubbornly insist on the "minimum 60 percent."
The Mining and the Oil Industries
Different From Each Other
To stress, there is no independent showing that the taking of at least a 60 percent share in the aftertax income ofa mining company operated by a foreign contractor is fair and reasonable under most if
not all circumstances. The fact that some petroleum companies like Shell acceded to such
percentage of sharing does not ipso facto mean that it is per se reasonable and applicable to nonpetroleum situations (that is, mining companies) as well. We can take judicial notice of the fact that
there are, after all, numerous intrinsic differences involved in their respective operations and
equipment or technological requirements, costs structures and capital investment needs, and
product pricing and markets.
There is no showing, for instance, that mining companies can readily cope with a 60 percent
government share in the same way petroleum companies apparently can. What we have is a
suggestion to enforce the 60 percent quota on the basis of a disjointed analogy. The only factor
common to the two disparate situations is the extraction of natural resources.
Indeed, we should take note of the fact that Congress made a distinction between mining firms and
petroleum companies. In Republic Act No. 7729 -- "An Act Reducing the Excise Tax Rates on
Metallic and Non-Metallic Minerals and Quarry Resources, Amending for the Purpose Section
151(a) of the National Internal Revenue Code, as amended" -- the lawmakers fixed the excise tax
rate on metallic and non-metallic minerals at two percent of the actual market value of the annual
gross output at the time of removal. However, in the case of petroleum, the lawmakers set the excise
tax rate for the first taxable sale at fifteen percent of the fair international market price thereof.
There must have been a very sound reason that impelled Congress to impose two very dissimilar
excise tax rate. We cannot assume, without proof, that our honorable legislators acted arbitrarily,
capriciously and whimsically in this instance. We cannot just ignore the reality of two distinctly
different situations and stubbornly insist on going "minimum 60 percent."
To repeat, the mere fact that gas and oil exploration contracts grant the State 60 percent of the net
revenues does not necessarily imply that mining contracts should likewise yield a minimum of 60
percent for the State.Jumping to that erroneous conclusion is like comparing apples with oranges.
The exploration, development and utilization of gas and oil are simply different from those of mineral
resources.
To stress again, the main risk in gas and oil is in the exploration. But once oil in commercial
quantities is struck and the wells are put in place, the risk is relatively over and black gold simply
flows out continuously withcomparatively less need for fresh investments and technology.

On the other hand, even if minerals are found in viable quantities, there is still need for continuous
fresh capital and expertise to dig the mineral ores from the mines. Just because deposits of mineral
ores are found in one area is no guarantee that an equal amount can be found in the adjacent areas.
There are simply continuing risks and need for more capital, expertise and industry all the time.
Note, however, that the indirect benefits -- apart from the cash revenues -- are much more in the
mineral industry. As mines are explored and extracted, vast employment is created, roads and other
infrastructure are built, and other multiplier effects arise. On the other hand, once oil wells start
producing, there is less need for employment. Roads and other public works need not be
constructed continuously. In fine, there is no basis for saying that government revenues from the oil
industry and from the mineral industries are to be identical all the time.
Fourth, to our mind, the proffered "minimum 60 percent" suggestion tends to limit the flexibility and
tie the hands of government, ultimately hampering the country's competitiveness in the international
market, to the detriment of the Filipino people. This "you-have-to-give-us-60-percent-of-after-taxincome-or-we-don't-do- business-with-you" approach is quite perilous. True, this situation may not
seem too unpalatable to the foreign contractor during good years, when international market prices
are up and the mining firm manages to keep its costs in check. However, under unfavorable
economic and business conditions, with costs spiraling skywards and minerals prices plummeting, a
mining firm may consider itself lucky to make just minimal profits.
The inflexible, carved-in-granite demand for a 60 percent government share may spell the end of the
mining venture, scare away potential investors, and thereby further worsen the already dismal
economic scenario. Moreover, such an unbending or unyielding policy prevents the government from
responding appropriately to changing economic conditions and shifting market forces. This
inflexibility further renders our country less attractive as an investment option compared with other
countries.
And fifth, for this Court to decree imperiously that the government's share should be not less than 60
percent of the after-tax income of FTAA contractors at all times is nothing short of dictating upon the
government. The result, ironically, is that the State ends up losing control. To avoid compromising
the State's full control and supervision over the exploitation of mineral resources, this Court must
back off from insisting upon a "minimum 60 percent" rule. It is sufficient that the State has the power
and means, should it so decide, to get a 60 percent share (or more) in the contractor's net mining
revenues or after-tax income, or whatever other basis the government may decide to use in
reckoning its share. It is not necessary for it to do so in every case, regardless of circumstances.
In fact, the government must be trusted, must be accorded the liberty and the utmost flexibility to
deal, negotiate and transact with contractors and third parties as it sees fit; and upon terms that it
ascertains to be most favorable or most acceptable under the circumstances, even if it means
agreeing to less than 60 percent. Nothing must prevent the State from agreeing to a share less than
that, should it be deemed fit; otherwise the State will be deprived of full control over mineral
exploitation that the Charter has vested in it.
To stress again, there is simply no constitutional or legal provision fixing the minimum share of the
government in an FTAA at 60 percent of the net profit. For this Court to decree such minimum is to
wade into judicial legislation, and thereby inordinately impinge on the control power of the State. Let
it be clear: the Court is not against the grant of more benefits to the State; in fact, the more the
better. If during the FTAA negotiations, the President can secure 60 percent,78 or even 90 percent,
then all the better for our people. But, if under the peculiar circumstances of a specific contract, the
President could secure only 50 percent or 55 percent, so be it. Needless to say, the President will
have to report (and be responsible for) the specific FTAA to Congress, and eventually to the people.

Finally, if it should later be found that the share agreed to is grossly disadvantageous to the
government, the officials responsible for entering into such a contract on its behalf will have to
answer to the courts for their malfeasance. And the contract provision voided. But this Court would
abuse its own authority should it force the government's hand to adopt the 60 percent demand of
some of our esteemed colleagues.
Capital and Expertise Provided,
Yet All Risks Assumed by Contractor
Here, we will repeat what has not been emphasized and appreciated enough: the fact that the
contractor in an FTAA provides all the needed capital, technical and managerial expertise, and
technology required to undertake the project.
In regard to the WMCP FTAA, the then foreign-owned WMCP as contractor committed, at the very
outset, to make capital investments of up to US$50 million in that single mining project. WMCP
claims to have already poured in well over P800 million into the country as of February 1998, with
more in the pipeline. These resources, valued in the tens or hundreds of millions of dollars, are
invested in a mining project that provides no assurance whatsoever that any part of the investment
will be ultimately recouped.
At the same time, the contractor must comply with legally imposed environmental standards and the
social obligations, for which it also commits to make significant expenditures of funds. Throughout,
the contractor assumes all the risks79 of the business, as mentioned earlier. These risks are indeed
very high, considering that the rate of success in exploration is extremely low. The probability of
finding any mineral or petroleum in commercially viable quantities is estimated to be about 1:1,000
only. On that slim chance rides the contractor's hope of recouping investments and generating
profits. And when the contractor has recouped its initial investments in the project, the government
share increases to sixty percent of net benefits -- without the State ever being in peril of incurring
costs, expenses and losses.
And even in the worst possible scenario -- an absence of commercial quantities of minerals to justify
development -- the contractor would already have spent several million pesos for exploration works,
before arriving at the point in which it can make that determination and decide to cut its losses. In
fact, during the first year alone of the exploration period, the contractor was already committed to
spend not less than P24 million. The FTAA therefore clearly ensures benefits for the local economy,
courtesy of the contractor.
All in all, this setup cannot be regarded as disadvantageous to the State or the Filipino
people; it certainly cannot be said to convey beneficial ownership of our mineral resources to
foreign contractors.
Deductions Allowed by the
WMCP FTAA Reasonable
Petitioners question whether the State's weak control might render the sharing arrangements
ineffective. They cite the so-called "suspicious" deductions allowed by the WMCP FTAA in arriving at
the net mining revenue, which is the basis for computing the government share. The WMCP FTAA,
for instance, allows expenditures for "development within and outside the Contract Area relating to
the Mining Operations,"80 "consulting fees incurred both inside and outside the Philippines for work
related directly to the Mining Operations,"81 and "the establishment and administration of field offices
including administrative overheads incurred within and outside the Philippines which are properly

allocatable to the Mining Operations and reasonably related to the performance of the Contractor's
obligations and exercise of its rights under this Agreement." 82
It is quite well known, however, that mining companies do perform some marketing activities abroad
in respect of selling their mineral products and by-products. Hence, it would not be improper to allow
the deduction ofreasonable consulting fees incurred abroad, as well as administrative expenses and
overheads related to marketing offices also located abroad -- provided that these deductions are
directly related or properly allocatable to the mining operations and reasonably related to the
performance of the contractor's obligations and exercise of its rights. In any event, more facts are
needed. Until we see how these provisions actually operate, mere "suspicions" will not suffice to
propel this Court into taking action.
Section 7.9 of the WMCP FTAA
Invalid and Disadvantageous
Having defended the WMCP FTAA, we shall now turn to two defective provisos. Let us start with
Section 7.9 of the WMCP FTAA. While Section 7.7 gives the government a 60 percent share in the
net mining revenues of WMCP from the commencement of commercial production, Section 7.9
deprives the government of part or all of the said 60 percent. Under the latter provision, should
WMCP's foreign shareholders -- who originally owned 100 percent of the equity -- sell 60 percent or
more of its outstanding capital stock to a Filipino citizen or corporation, the State loses its right to
receive its 60 percent share in net mining revenues under Section 7.7.
Section 7.9 provides:
The percentage of Net Mining Revenues payable to the Government pursuant to Clause 7.7
shall be reduced by 1percent of Net Mining Revenues for every 1percent ownership interest
in the Contractor (i.e., WMCP) held by a Qualified Entity.83
Evidently, what Section 7.7 grants to the State is taken away in the next breath by Section
7.9 without any offsetting compensation to the State. Thus, in reality, the State has no vested right to
receive any income from the FTAA for the exploitation of its mineral resources. Worse, it would seem
that what is given to the State in Section 7.7 is by mere tolerance of WMCP's foreign
stockholders, who can at any time cut off the government's entire 60 percent share. They can do so
by simply selling 60 percent of WMCP's outstanding capital stock to a Philippine citizen or
corporation. Moreover, the proceeds of such sale will of course accrue to the foreign stockholders of
WMCP, not to the State.
The sale of 60 percent of WMCP's outstanding equity to a corporation that is 60 percent Filipinoowned and 40 percent foreign-owned will still trigger the operation of Section 7.9. Effectively, the
State will lose its right to receive all 60 percent of the net mining revenues of WMCP; and foreign
stockholders will own beneficially up to 64 percent of WMCP, consisting of the remaining 40 percent
foreign equity therein, plus the 24 percent pro-rata share in the buyer-corporation. 84
In fact, the January 23, 2001 sale by WMCP's foreign stockholder of the entire outstanding equity in
WMCP to Sagittarius Mines, Inc. -- a domestic corporation at least 60 percent Filipino owned -- may
be deemed to have automatically triggered the operation of Section 7.9, without need of further
action by any party, and removed the State's right to receive the 60 percent share in net mining
revenues.
At bottom, Section 7.9 has the effect of depriving the State of its 60 percent share in the net mining
revenues of WMCP without any offset or compensation whatsoever. It is possible that the inclusion

of the offending provision was initially prompted by the desire to provide some form of incentive for
the principal foreign stockholder in WMCP to eventually reduce its equity position and ultimately
divest in favor of Filipino citizens and corporations. However, as finally structured, Section 7.9 has
the deleterious effect of depriving government of the entire 60 percent share in WMCP's net mining
revenues, without any form of compensation whatsoever. Such an outcome is completely
unacceptable.
The whole point of developing the nation's natural resources is to benefit the Filipino people, future
generations included. And the State as sovereign and custodian of the nation's natural wealth is
mandated to protect, conserve, preserve and develop that part of the national patrimony for their
benefit. Hence, the Charter lays great emphasis on "real contributions to the economic growth and
general welfare of the country"85 as essential guiding principles to be kept in mind when negotiating
the terms and conditions of FTAAs.
Earlier, we held (1) that the State must be accorded the liberty and the utmost flexibility to deal,
negotiate and transact with contractors and third parties as it sees fit, and upon terms that it
ascertains to be most favorable or most acceptable under the circumstances, even if that should
mean agreeing to less than 60 percent; (2) that it is not necessary for the State to extract a 60
percent share in every case and regardless of circumstances; and (3) that should the State be
prevented from agreeing to a share less than 60 percent as it deems fit, it will be deprived of the full
control over mineral exploitation that the Charter has vested in it.
That full control is obviously not an end in itself; it exists and subsists precisely because of the need
to serve and protect the national interest. In this instance, national interest finds particular application
in the protection of the national patrimony and the development and exploitation of the country's
mineral resources for the benefit of the Filipino people and the enhancement of economic growth
and the general welfare of the country. Undoubtedly, such full control can be misused and
abused, as we now witness.
Section 7.9 of the WMCP FTAA effectively gives away the State's share of net mining revenues
(provided for in Section 7.7) without anything in exchange. Moreover, this outcome constitutes unjust
enrichment on the part of the local and foreign stockholders of WMCP. By their mere divestment of
up to 60 percent equity in WMCP in favor of Filipino citizens and/or corporations, the local and
foreign stockholders get a windfall. Their share in the net mining revenues of WMCP is automatically
increased, without their having to pay the government anything for it. In short, the provision in
question is without a doubt grossly disadvantageous to the government, detrimental to the interests
of the Filipino people, and violative of public policy.
Moreover, it has been reiterated in numerous decisions86 that the parties to a contract may establish
any agreements, terms and conditions that they deem convenient; but these should not be contrary
to law, morals, good customs, public order or public policy.87 Being precisely violative of anti-graft
provisions and contrary to public policy, Section 7.9 must therefore be stricken off as invalid.
Whether the government officials concerned acceded to that provision by sheer mistake or with full
awareness of the ill consequences, is of no moment. It is hornbook doctrine that the principle of
estoppel does not operate against the government for the act of its agents, 88 and that it is never
estopped by any mistake or error on their part.89 It is therefore possible and proper to rectify the
situation at this time. Moreover, we may also say that the FTAA in question does not involve mere
contractual rights; being impressed as it is with public interest, the contractual provisions and
stipulations must yield to the common good and the national interest.

Since the offending provision is very much separable90 from Section 7.7 and the rest of the FTAA, the
deletion of Section 7.9 can be done without affecting or requiring the invalidation of the WMCP FTAA
itself. Such a deletion will preserve for the government its due share of the benefits. This way, the
mandates of the Constitution are complied with and the interests of the government fully protected,
while the business operations of the contractor are not needlessly disrupted.
Section 7.8(e) of the WMCP FTAA
Also Invalid and Disadvantageous
Section 7.8(e) of the WMCP FTAA is likewise invalid. It provides thus:
"7.8 The Government Share shall be deemed to include all of the following sums:
"(a) all Government taxes, fees, levies, costs, imposts, duties and royalties including
excise tax, corporate income tax, customs duty, sales tax, value added tax,
occupation and regulatory fees, Government controlled price stabilization schemes,
any other form of Government backed schemes, any tax on dividend payments by
the Contractor or its Affiliates in respect of revenues from the Mining Operations and
any tax on interest on domestic and foreign loans or other financial arrangements or
accommodations, including loans extended to the Contractor by its stockholders;
"(b) any payments to local and regional government, including taxes, fees, levies,
costs, imposts, duties, royalties, occupation and regulatory fees and infrastructure
contributions;
"(c) any payments to landowners, surface rights holders, occupiers, indigenous
people or Claimowners;
"(d) costs and expenses of fulfilling the Contractor's obligations to contribute to
national development in accordance with Clause 10.1(i) (1) and 10.1(i) (2);
"(e) an amount equivalent to whatever benefits that may be extended in the future by
the Government to the Contractor or to financial or technical assistance agreement
contractors in general;
"(f) all of the foregoing items which have not previously been offset against the
Government Share in an earlier Fiscal Year, adjusted for inflation." (underscoring
supplied)
Section 7.8(e) is out of place in the FTAA. It makes no sense why, for instance, money spent by the
government for the benefit of the contractor in building roads leading to the mine site should still be
deductible from the State's share in net mining revenues. Allowing this deduction results in benefiting
the contractor twice over. It constitutesunjust enrichment on the part of the contractor at the expense
of the government, since the latter is effectively being made to pay twice for the same item. 91 For
being grossly disadvantageous and prejudicial to the government and contrary to public policy,
Section 7.8(e) is undoubtedly invalid and must be declared to be without effect. Fortunately, this
provision can also easily be stricken off without affecting the rest of the FTAA.
Nothing Left Over
After Deductions?

In connection with Section 7.8, an objection has been raised: Specified in Section 7.8 are numerous
items of deduction from the State's 60 percent share. After taking these into account, will the State
ever receive anything for its ownership of the mineral resources?
We are confident that under normal circumstances, the answer will be yes. If we examine the
various items of "deduction" listed in Section 7.8 of the WMCP FTAA, we will find that they
correspond closely to the components or elements of the basic government share established in
DAO 99-56, as discussed in the earlier part of this Opinion.
Likewise, the balance of the government's 60 percent share -- after netting out the items of
deduction listed in Section 7.8 --corresponds closely to the additional government share provided
for in DAO 99-56 which, we once again stress, has nothing at all to do with indirect taxes. The
Ramos-DeVera paper92 concisely presents the fiscal contribution of an FTAA under DAO 99-56 in
this equation:
Receipts from an FTAA = basic gov't share + add'l gov't share
Transposed into a similar equation, the fiscal payments system from the WMCP FTAA assumes the
following formulation:
Government's 60 percent share in net mining revenues of WMCP = items listed in Sec. 7.8 of
the FTAA + balance of Gov't share, payable 4 months from the end of the fiscal year
It should become apparent that the fiscal arrangement under the WMCP FTAA is very similar to that
under DAO 99-56, with the "balance of government share payable 4 months from end of fiscal year"
being the equivalent of the additional government share computed in accordance with the "netmining-revenue-based option" under DAO 99-56, as discussed above. As we have emphasized
earlier, we find each of the three options for computing the additional government share -- as
presented in DAO 99-56 -- to be sound and reasonable.
We therefore conclude that there is nothing inherently wrong in the fiscal regime of the
WMCP FTAA, and certainly nothing to warrant the invalidation of the FTAA in its entirety.
Section 3.3 of the WMCP
FTAA Constitutional
Section 3.3 of the WMCP FTAA is assailed for violating supposed constitutional restrictions on the
term of FTAAs. The provision in question reads:
"3.3 This Agreement shall be renewed by the Government for a further period of twenty-five
(25) years under the same terms and conditions provided that the Contractor lodges a
request for renewal with the Government not less than sixty (60) days prior to the expiry of
the initial term of this Agreement and provided that the Contractor is not in breach of any of
the requirements of this Agreement."
Allegedly, the above provision runs afoul of Section 2 of Article XII of the 1987 Constitution, which
states:
"Sec. 2. All lands of the public domain, waters, minerals, coal, petroleum, and other mineral
oils, all forces of potential energy, fisheries, forests or timber, wildlife, flora and fauna, and
other natural resources are owned by the State. With the exception of agricultural lands, all

other natural resources shall not be alienated. The exploration, development and utilization
of natural resources shall be under the full control and supervision of the State. The State
may directly undertake such activities, or it may enter into co-production, joint venture or
production-sharing agreements with Filipino citizens or corporations or associations at least
sixty per centum of whose capital is owned by such citizens. Such agreements may be for
a period not exceeding twenty-five years, renewable for not more than twenty-five
years, and under such terms and conditions as may be provided by law. In cases of
water rights for irrigation, water supply, fisheries, or industrial uses other than the
development of water power, beneficial use may be the measure and limit of the grant.
"The State shall protect the nation's marine wealth in its archipelagic waters, territorial sea,
and exclusive economic zone, and reserve its use and enjoyment exclusively to Filipino
citizens.
"The Congress may, by law, allow small-scale utilization of natural resources by Filipino
citizens, as well as cooperative fish farming, with priority to subsistence fishermen and fishworkers in rivers, lakes, bays and lagoons.
"The President may enter into agreements with foreign-owned corporations involving either
technical or financial assistance for large-scale exploration, development, and utilization of
minerals, petroleum, and other mineral oils according to the general terms and conditions
provided by law, based on real contributions to the economic growth and general welfare of
the country. In such agreements, the State shall promote the development and use of local
scientific and technical resources.
"The President shall notify the Congress of every contract entered into in accordance with
this provision, within thirty days from its execution." 93
We hold that the term limitation of twenty-five years does not apply to FTAAs. The reason is that the
above provision is found within paragraph 1 of Section 2 of Article XII, which refers to mineral
agreements -- co-production agreements, joint venture agreements and mineral production-sharing
agreements -- which the government may enter into with Filipino citizens and corporations, at least
60 percent owned by Filipino citizens. The word "such" clearly refers to these three mineral
agreements -- CPAs, JVAs and MPSAs -- not to FTAAs.
Specifically, FTAAs are covered by paragraphs 4 and 5 of Section 2 of Article XII of the Constitution.
It will be noted that there are no term limitations provided for in the said paragraphs dealing with
FTAAs. This shows that FTAAs are sui generis, in a class of their own. This omission was obviously
a deliberate move on the part of the framers. They probably realized that FTAAs would be different in
many ways from MPSAs, JVAs and CPAs. The reason the framers did not fix term limitations
applicable to FTAAs is that they preferred to leave the matter to the discretion of the legislature
and/or the agencies involved in implementing the laws pertaining to FTAAs, in order to give the latter
enough flexibility and elbow room to meet changing circumstances.
Note also that, as previously stated, the exploratory phrases of an FTAA lasts up to eleven years.
Thereafter, a few more years would be gobbled up in start-up operations. It may take fifteen years
before an FTAA contractor can start earning profits. And thus, the period of 25 years may really be
short for an FTAA. Consider too that in this kind of agreement, the contractor assumes all
entrepreneurial risks. If no commercial quantities of minerals are found, the contractor bears all
financial losses. To compensate for this long gestation period and extra business risks, it would not
be totally unreasonable to allow it to continue EDU activities for another twenty five years.

In any event, the complaint is that, in essence, Section 3.3 gives the contractor the power to compel
the government to renew the WMCP FTAA for another 25 years and deprives the State of any say
on whether to renew the contract.
While we agree that Section 3.3 could have been worded so as to prevent it from favoring the
contractor, this provision does not violate any constitutional limits, since the said term limitation does
not apply at all to FTAAs. Neither can the provision be deemed in any manner to be illegal, as no law
is being violated thereby. It is certainly not illegal for the government to waive its option to refuse the
renewal of a commercial contract.
Verily, the government did not have to agree to Section 3.3. It could have said "No" to the stipulation,
but it did not. It appears that, in the process of negotiations, the other contracting party was able to
convince the government to agree to the renewal terms. Under the circumstances, it does not seem
proper for this Court to intervene and step in to undo what might have perhaps been a possible
miscalculation on the part of the State. If government believes that it is or will be aggrieved by the
effects of Section 3.3, the remedy is the renegotiation of the provision in order to provide the State
the option to not renew the FTAA.
Financial Benefits for Foreigners
Not Forbidden by the Constitution
Before leaving this subject matter, we find it necessary for us to rid ourselves of the false belief that
the Constitution somehow forbids foreign-owned corporations from deriving financial benefits from
the development of our natural or mineral resources.
The Constitution has never prohibited foreign corporations from acquiring and enjoying "beneficial
interest" in the development of Philippine natural resources. The State itself need not directly
undertake exploration, development, and utilization activities. Alternatively, the Constitution
authorizes the government to enter into joint venture agreements (JVAs), co-production agreements
(CPAs) and mineral production sharing agreements (MPSAs) with contractors who are Filipino
citizens or corporations that are at least 60 percent Filipino-owned. They may do the actual "dirty
work" -- the mining operations.
In the case of a 60 percent Filipino-owned corporation, the 40 percent individual and/or
corporate non-Filipino stakeholders obviously participate in the beneficial interest derived from the
development and utilization of our natural resources. They may receive by way of dividends, up to
40 percent of the contractor's earnings from the mining project. Likewise, they may have a say in the
decisions of the board of directors, since they are entitled to representation therein to the extent of
their equity participation, which the Constitution permits to be up to 40 percent of the contractor's
equity. Hence, the non-Filipino stakeholders may in that manner also participate in the management
of the contractor's natural resource development work. All of this is permitted by our Constitution, for
any natural resource, and without limitation even in regard to the magnitude of the mining project or
operations (see paragraph 1 of Section 2 of Article XII).
It is clear, then, that there is nothing inherently wrong with or constitutionally objectionable about the
idea of foreign individuals and entities having or enjoying "beneficial interest" in -- and participating
in the management of operations relative to -- the exploration, development and utilization of our
natural resources.
FTAA More Advantageous
Than Other Schemes
Like CPA, JVA and MPSA

A final point on the subject of beneficial interest. We believe the FTAA is a more advantageous
proposition for the government as compared with other agreements permitted by the Constitution. In
a CPA that the government enters into with one or more contractors, the government shall provide
inputs to the mining operations other than the mineral resource itself. 94
In a JVA, a JV company is organized by the government and the contractor, with both parties having
equity shares (investments); and the contractor is granted the exclusive right to conduct mining
operations and to extract minerals found in the area. 95 On the other hand, in an MPSA, the
government grants the contractor the exclusive right to conduct mining operations within the contract
area and shares in the gross output; and the contractor provides the necessary financing,
technology, management and manpower.
The point being made here is that, in two of the three types of agreements under consideration,
the government has to ante up some risk capital for the enterprise. In other words, government
funds (public moneys) are withdrawn from other possible uses, put to work in the venture and placed
at risk in case the venture fails. This notwithstanding, management and control of the operations of
the enterprise are -- in all three arrangements -- in the hands of the contractor, with the government
being mainly a silent partner. The three types of agreement mentioned above apply to any natural
resource, without limitation and regardless of the size or magnitude of the project or operations.
In contrast to the foregoing arrangements, and pursuant to paragraph 4 of Section 2 of Article XII,
the FTAA is limited to large-scale projects and only for minerals, petroleum and other mineral oils.
Here, the Constitution removes the 40 percent cap on foreign ownership and allows the foreign
corporation to own up to 100 percent of the equity. Filipino capital may not be sufficient on account of
the size of the project, so the foreign entity may have to ante up all the risk capital.
Correlatively, the foreign stakeholder bears up to 100 percent of the risk of loss if the project fails. In
respect of the particular FTAA granted to it, WMCP (then 100 percent foreign owned) was
responsible, as contractor, for providing the entire equity, including all the inputs for the project. It
was to bear 100 percent of the risk of loss if the project failed, but its maximum potential "beneficial
interest" consisted only of 40 percent of the net beneficial interest, because the other 60 percent is
the share of the government, which will never be exposed to any risk of loss whatsoever.
In consonance with the degree of risk assumed, the FTAA vested in WMCP the day-to-day
management of the mining operations. Still such management is subject to the overall control and
supervision of the State in terms of regular reporting, approvals of work programs and budgets, and
so on.
So, one needs to consider in relative terms, the costs of inputs for, degree of risk attendant to, and
benefits derived or to be derived from a CPA, a JVA or an MPSA vis--vis those pertaining to an
FTAA. It may not be realistically asserted that the foreign grantee of an FTAA is being unduly favored
or benefited as compared with a foreign stakeholder in a corporation holding a CPA, a JVA or an
MPSA. Seen the other way around, the government is definitely better off with an FTAA than a CPA,
a JVA or an MPSA.
Developmental Policy on the Mining Industry
During the Oral Argument and in their Final Memorandum, petitioners repeatedly urged the Court to
consider whether mining as an industry and economic activity deserved to be accorded priority,
preference and government support as against, say, agriculture and other activities in which Filipinos
and the Philippines may have an "economic advantage." For instance, a recent US study 96 reportedly

examined the economic performance of all local US counties that were dependent on mining and 20
percent of whose labor earnings between 1970 and 2000 came from mining enterprises.
The study -- covering 100 US counties in 25 states dependent on mining -- showed that per capita
income grew about 30 percent less in mining-dependent communities in the 1980s and 25 percent
less for the entire period 1980 to 2000; the level of per capita income was also lower. Therefore,
given the slower rate of growth, the gap between these and other local counties increased.
Petitioners invite attention to the OXFAM America Report's warning to developing nations that
mining brings with it serious economic problems, including increased regional inequality,
unemployment and poverty. They also cite the final report 97 of the Extractive Industries Review
project commissioned by the World Bank (the WB-EIR Report), which warns of environmental
degradation, social disruption, conflict, and uneven sharing of benefits with local communities that
bear the negative social and environmental impact. The Report suggests that countries need to
decide on the best way to exploit their natural resources, in order to maximize the value added from
the development of their resources and ensure that they are on the path to sustainable development
once the resources run out.
Whatever priority or preference may be given to mining vis--vis other economic or non-economic
activities is a question of policy that the President and Congress will have to address; it is not for this
Court to decide. This Court declares what the Constitution and the laws say, interprets only when
necessary, and refrains from delving into matters of policy.
Suffice it to say that the State control accorded by the Constitution over mining activities assures a
proper balancing of interests. More pointedly, such control will enable the President to demand the
best mining practices and the use of the best available technologies to protect the environment and
to rehabilitate mined-out areas. Indeed, under the Mining Law, the government can ensure the
protection of the environment during and after mining. It can likewise provide for the mechanisms to
protect the rights of indigenous communities, and thereby mold a more socially-responsive,
culturally-sensitive and sustainable mining industry.
Early on during the launching of the Presidential Mineral Industry Environmental Awards on February
6, 1997, then President Fidel V. Ramos captured the essence of balanced and sustainable mining in
these words:
"Long term, high profit mining translates into higher revenues for government, more decent
jobs for the population, more raw materials to feed the engines of downstream and allied
industries, and improved chances of human resource and countryside development by
creating self-reliant communities away from urban centers.
xxxxxxxxx
"Against a fragile and finite environment, it is sustainability that holds the key. In sustainable
mining, we take a middle ground where both production and protection goals are balanced,
and where parties-in-interest come to terms."
Neither has the present leadership been remiss in addressing the concerns of sustainable mining
operations. Recently, on January 16, 2004 and April 20, 2004, President Gloria Macapagal Arroyo
issued Executive Orders Nos. 270 and 270-A, respectively, "to promote responsible mineral
resources exploration, development and utilization, in order to enhance economic growth, in a
manner that adheres to the principles of sustainable development and with due regard for justice
and equity, sensitivity to the culture of the Filipino people and respect for Philippine sovereignty." 98

REFUTATION OF DISSENTS
The Court will now take up a number of other specific points raised in the dissents of Justices Carpio
and Morales.
1. Justice Morales introduced us to Hugh Morgan, former president and chief executive officer of
Western Mining Corporation (WMC) and former president of the Australian Mining Industry Council,
who spearheaded the vociferous opposition to the filing by aboriginal peoples of native title claims
against mining companies in Australia in the aftermath of the landmark Mabo decision by the
Australian High Court. According to sources quoted by our esteemed colleague, Morgan was also
a racist and a bigot. In the course of protesting Mabo, Morgan allegedly uttered derogatory remarks
belittling the aboriginal culture and race.
An unwritten caveat of this introduction is that this Court should be careful not to permit the entry
of the likes of Hugh Morgan and his hordes of alleged racist-bigots at WMC. With all due respect,
such scare tactics should have no place in the discussion of this case. We are deliberating on the
constitutionality of RA 7942, DAO 96-40 and the FTAA originally granted to WMCP, which had been
transferred to Sagittarius Mining, a Filipino corporation. We are not discussing the apparition of white
Anglo-Saxon racists/bigots massing at our gates.
2. On the proper interpretation of the phrase agreements involving either technical or financial
assistance, Justice Morales points out that at times we "conveniently omitted" the use of the
disjunctive eitheror, which according to her denotes restriction; hence the phrase must be deemed
to connote restriction and limitation.
But, as Justice Carpio himself pointed out during the Oral Argument, the disjunctive phrase either
technical or financial assistance would, strictly speaking, literally mean that a foreign contractor
may provide only one or the other, but not both. And if both technical and financial assistance were
required for a project, the State would have to deal with at least two different foreign contractors -one for financial and the other for technical assistance. And following on that, a foreign contractor,
though very much qualified to provide both kinds of assistance, would nevertheless be prohibited
from providing one kind as soon as it shall have agreed to provide the other.
But if the Court should follow this restrictive and literal construction, can we really find two (or more)
contractors who are willing to participate in one single project -- one to provide the "financial
assistance" only and the other the "technical assistance" exclusively; it would be excellent if these
two or more contractors happen to be willing and are able to cooperate and work closely together on
the same project (even if they are otherwise competitors). And it would be superb if no conflicts
would arise between or among them in the entire course of the contract. But what are the chances
things will turn out this way in the real world? To think that the framers deliberately imposed this kind
of restriction is to say that they were either exceedingly optimistic, or incredibly nave. This begs the
question -- What laudable objective or purpose could possibly be served by such strict and restrictive
literal interpretation?
3. Citing Oposa v. Factoran Jr., Justice Morales claims that a service contract is not a contract or
property right which merits protection by the due process clause of the Constitution, but merely a
license or privilege which may be validly revoked, rescinded or withdrawn by executive action
whenever dictated by public interest or public welfare.
Oposa cites Tan v. Director of Forestry and Ysmael v. Deputy Executive Secretary as authority. The
latter cases dealt specifically with timber licenses only. Oposa allegedly reiterated that a license is
merely a permit or privilege to do what otherwise would be unlawful, and is not a contract between

the authority, federal, state or municipal, granting it and the person to whom it is granted; neither is it
property or a property right, nor does it create a vested right; nor is it taxation. Thus this Court held
that the granting of license does not create irrevocable rights, neither is it property or property rights.
Should Oposa be deemed applicable to the case at bar, on the argument that natural resources are
also involved in this situation? We do not think so. A grantee of a timber license, permit or license
agreement gets to cut the timber already growing on the surface; it need not dig up tons of earth to
get at the logs. In a logging concession, the investment of the licensee is not as substantial as the
investment of a large-scale mining contractor. If a timber license were revoked, the licensee packs
up its gear and moves to a new area applied for, and starts over; what it leaves behind are mainly
the trails leading to the logging site.
In contrast, the mining contractor will have sunk a great deal of money (tens of millions of dollars)
into the ground, so to speak, for exploration activities, for development of the mine site and
infrastructure, and for the actual excavation and extraction of minerals, including the extensive
tunneling work to reach the ore body. The cancellation of the mining contract will utterly deprive the
contractor of its investments (i.e., prevent recovery of investments), most of which cannot be pulled
out.
To say that an FTAA is just like a mere timber license or permit and does not involve contract or
property rights which merit protection by the due process clause of the Constitution, and may
therefore be revoked or cancelled in the blink of an eye, is to adopt a well-nigh confiscatory stance;
at the very least, it is downright dismissive of the property rights of businesspersons and corporate
entities that have investments in the mining industry, whose investments, operations and
expenditures do contribute to the general welfare of the people, the coffers of government, and the
strength of the economy. Such a pronouncement will surely discourage investments (local and
foreign) which are critically needed to fuel the engine of economic growth and move this country out
of the rut of poverty. In sum, Oposa is not applicable.
4. Justice Morales adverts to the supposedly "clear intention" of the framers of the Constitution to
reserve our natural resources exclusively for the Filipino people. She then quoted from the records
of the ConCom deliberations a passage in which then Commissioner Davide explained his vote,
arguing in the process that aliens ought not be allowed to participate in the enjoyment of our natural
resources. One passage does not suffice to capture the tenor or substance of the entire extensive
deliberations of the commissioners, or to reveal the clear intention of the framers as a group. A rereading of the entire deliberations (quoted here earlier) is necessary if we are to understand the true
intent of the framers.
5. Since 1935, the Filipino people, through their Constitution, have decided that the retardation or
delay in the exploration, development or utilization of the nation's natural resources is merely
secondary to the protection and preservation of their ownership of the natural resources, so says
Justice Morales, citing Aruego. If it is true that the framers of the 1987 Constitution did not care much
about alleviating the retardation or delay in the development and utilization of our natural
resources, why did they bother to write paragraph 4 at all? Were they merely paying lip service to
large-scale exploration, development and utilization? They could have just completely ignored the
subject matter and left it to be dealt with through a future constitutional amendment. But we have to
harmonize every part of the Constitution and to interpret each provision in a manner that would give
life and meaning to it and to the rest of the provisions. It is obvious that a literal interpretation of
paragraph 4 will render it utterly inutile and inoperative.
6. According to Justice Morales, the deliberations of the Constitutional Commission do not support
our contention that the framers, by specifying such agreements involving financial or technical

assistance, necessarily gave implied assent to everything that these agreements implicitly entailed,
or that could reasonably be deemed necessary to make them tenable and effective, including
management authority in the day-to-day operations. As proof thereof, she quotes one single
passage from the ConCom deliberations, consisting of an exchange among Commissioners
Tingson, Garcia and Monsod.
However, the quoted exchange does not serve to contradict our argument; it even bolsters it. Comm.
Christian Monsod was quoted as saying: "xxx I think we have to make a distinction that it is not
really realistic to say that we will borrow on our own terms. Maybe we can say that we inherited
unjust loans, and we would like to repay these on terms that are not prejudicial to our own growth.
But the general statement that we should only borrow on our own terms is a bit unrealistic." Comm.
Monsod is one who knew whereof he spoke.
7. Justice Morales also declares that the optimal time for the conversion of an FTAA into an MPSA is
after completion of the exploration phase and just before undertaking the development and
construction phase, on account of the fact that the requirement for a minimum investment of $50
million is applicable only during the development, construction and utilization phase, but not during
the exploration phase, when the foreign contractor need merely comply with minimum ground
expenditures. Thus by converting, the foreign contractor maximizes its profits by avoiding its
obligation to make the minimum investment of $50 million.
This argument forgets that the foreign contractor is in the game precisely to make money. In order to
come anywhere near profitability, the contractor must first extract and sell the mineral ore. In order to
do that, it must also develop and construct the mining facilities, set up its machineries and
equipment and dig the tunnels to get to the deposit. The contractor is thus compelled to expend
funds in order to make profits. If it decides to cut back on investments and expenditures, it will
necessarily sacrifice the pace of development and utilization; it will necessarily sacrifice the amount
of profits it can make from the mining operations. In fact, at certain less-than-optimal levels of
operation, the stream of revenues generated may not even be enough to cover variable expenses,
let alone overhead expenses; this is a dismal situation anyone would want to avoid. In order to make
money, one has to spend money. This truism applies to the mining industry as well.
8. Mortgaging the minerals to secure a foreign FTAA contractor's obligations is anomalous,
according to Justice Morales since the contractor was from the beginning obliged to provide all
financing needed for the mining operations. However, the mortgaging of minerals by the contractor
does not necessarily signify that the contractor is unable to provide all financing required for the
project, or that it does not have the financial capability to undertake large-scale operations.
Mortgaging of mineral products, just like the assignment (by way of security) of manufactured goods
and goods in inventory, and the assignment of receivables, is an ordinary requirement of banks,
even in the case of clients with more than sufficient financial resources. And nowadays, even the
richest and best managed corporations make use of bank credit facilities -- it does not necessarily
signify that they do not have the financial resources or are unable to provide the financing on their
own; it is just a manner of maximizing the use of their funds.
9. Does the contractor in reality acquire the surface rights "for free," by virtue of the fact that it is
entitled to reimbursement for the costs of acquisition and maintenance, adjusted for inflation? We
think not. The "reimbursement" is possible only at the end of the term of the contract, when the
surface rights will no longer be needed, and the land previously acquired will have to be disposed of,
in which case the contractor gets reimbursement from the sales proceeds. The contractor has to pay
out the acquisition price for the land. That money will belong to the seller of the land. Only if and
when the land is finally sold off will the contractor get any reimbursement. In other words, the
contractor will have been cash-out for the entire duration of the term of the contract -- 25 or 50 years,

depending. If we calculate the cost of money at say 12 percent per annum, that is the cost or
opportunity loss to the contractor, in addition to the amount of the acquisition price. 12 percent per
annum for 50 years is 600 percent; this, without any compounding yet. The cost of money is
therefore at least 600 percent of the original acquisition cost; it is in addition to the acquisition
cost. "For free"? Not by a long shot.
10. The contractor will acquire and hold up to 5,000 hectares? We doubt it. The acquisition by the
State of land for the contractor is just to enable the contractor to establish its mine site, build its
facilities, establish a tailings pond, set up its machinery and equipment, and dig mine shafts and
tunnels, etc. It is impossible that the surface requirement will aggregate 5,000 hectares. Much of the
operations will consist of the tunneling and digging underground, which will not require possessing or
using any land surface. 5,000 hectares is way too much for the needs of a mining operator. It simply
will not spend its cash to acquire property that it will not need; the cash may be better employed for
the actual mining operations, to yield a profit.
11. Justice Carpio claims that the phrase among other things (found in the second paragraph of
Section 81 of the Mining Act) is being incorrectly treated as a delegation of legislative power to the
DENR secretary to issue DAO 99-56 and prescribe the formulae therein on the State's share from
mining operations. He adds that the phraseamong other things was not intended as a delegation of
legislative power to the DENR secretary, much less could it be deemed a valid delegation of
legislative power, since there is nothing in the second paragraph of Section 81 which can be said to
grant any delegated legislative power to the DENR secretary. And even if there were, such
delegation would be void, for lack of any standards by which the delegated power shall be exercised.
While there is nothing in the second paragraph of Section 81 which can directly be construed as a
delegation of legislative power to the DENR secretary, it does not mean that DAO 99-56 is invalid
per se, or that the secretary acted without any authority or jurisdiction in issuing DAO 99-56. As we
stated earlier in our Prologue, "Who or what organ of government actually exercises this power of
control on behalf of the State? The Constitution is crystal clear: the President. Indeed, the Chief
Executive is the official constitutionally mandated to 'enter into agreements with foreign owned
corporations.' On the other hand, Congress may review the action of the President once it is notified
of 'every contract entered into in accordance with this [constitutional] provision within thirty days from
its execution.'" It is the President who is constitutionally mandated to enter into FTAAs with foreign
corporations, and in doing so, it is within the President's prerogative to specify certain terms and
conditions of the FTAAs, for example, the fiscal regime of FTAAs -- i.e., the sharing of the net
mining revenues between the contractor and the State.
Being the President's alter ego with respect to the control and supervision of the mining industry, the
DENR secretary, acting for the President, is necessarily clothed with the requisite authority and
power to draw up guidelines delineating certain terms and conditions, and specifying therein the
terms of sharing of benefits from mining, to be applicable to FTAAs in general. It is important to
remember that DAO 99-56 has been in existence for almost six years, and has not been amended or
revoked by the President.
The issuance of DAO 99-56 did not involve the exercise of delegated legislative power. The
legislature did not delegate the power to determine the nature, extent and composition of the items
that would come under the phrase among other things. The legislature's power pertains to the
imposition of taxes, duties and fees. This power was not delegated to the DENR secretary. But the
power to negotiate and enter into FTAAs was withheld from Congress, and reserved for the
President. In determining the sharing of mining benefits, i.e., in specifying what the phrase among
other things include, the President (through the secretary acting in his/her behalf) was not
determining the amount or rate of taxes, duties and fees, but rather the amount of INCOME to be

derived from minerals to be extracted and sold, income which belongs to the State as owner of the
mineral resources. We may say that, in the second paragraph of Section 81, the legislature in a
sense intruded partially into the President's sphere of authority when the former provided that
"The Government share in financial or technical assistance agreement shall consist of,
among other things, the contractor's corporate income tax, excise tax, special allowance,
withholding tax due from the contractor's foreign stockholders arising from dividend or
interest payments to the said foreign stockholder in case of a foreign national and all such
other taxes, duties and fees as provided for under existing laws."(Italics supplied)
But it did not usurp the President's authority since the provision merely included the enumerated
items as part of the government share, without foreclosing or in any way preventing (as in fact
Congress could not validly prevent) the President from determining what constitutes the State's
compensation derived from FTAAs. In this case, the President in effect directed the inclusion or
addition of "other things," viz., INCOME for the owner of the resources, in the government's share,
while adopting the items enumerated by Congress as part of the government share also.
12. Justice Carpio's insistence on applying the ejusdem generis rule of statutory construction to the
phraseamong other things is therefore useless, and must fall by the wayside. There is no point trying
to construe that phrase in relation to the enumeration of taxes, duties and fees found in paragraph 2
of Section 81, precisely because "the constitutional power to prescribe the sharing of mining
income between the State and mining companies," to quote Justice Carpio pursuant to an FTAA
is constitutionally lodged with the President, not with Congress. It thus makes no sense to
persist in giving the phrase among other things a restricted meaning referring only to taxes, duties
and fees.
13. Strangely, Justice Carpio claims that the DENR secretary can change the formulae in DAO 99-56
any time even without the approval of the President, and the secretary is the sole authority to
determine the amount of consideration that the State shall receive in an FTAA, because Section 5 of
the DAO states that "xxx any amendment of an FTAA other than the provision on fiscal regime shall
require the negotiation with the Negotiation Panel and the recommendation of the Secretary for
approval of the President xxx". Allegedly, because of that provision, if an amendment in the FTAA
involves non-fiscal matters, the amendment requires approval of the President, but if the amendment
involves a change in the fiscal regime, the DENR secretary has the final authority, and approval of
the President may be dispensed with; hence the secretary is more powerful than the President.
We believe there is some distortion resulting from the quoted provision being taken out of context.
Section 5 of DAO 99-56 reads as follows:
"Section 5. Status of Existing FTAAs. All FTAAs approved prior to the effectivity of this
Administrative Order shall remain valid and be recognized by the Government: Provided,
That should a Contractor desire to amend its FTAA, it shall do so by filing a Letter of Intent
(LOI) to the Secretary thru the Director. Provided, further, That if the Contractor desires to
amend the fiscal regime of its FTAA, it may do so by seeking for the amendment of its
FTAA's whole fiscal regime by adopting the fiscal regime provided hereof: Provided, finally,
That any amendment of an FTAA other than the provision on fiscal regime shall require the
negotiation with the Negotiating Panel and the recommendation of the Secretary for approval
of the President of the Republic of the Philippines." (underscoring supplied)
It looks like another case of misapprehension. The proviso being objected to by Justice Carpio is
actually preceded by a phrase that requires a contractor desiring to amend the fiscal regime of its
FTAA, to amend thesame by adopting the fiscal regime prescribed in DAO 99-56 -- i.e., solely in that

manner, and in no other.Obviously, since DAO 99-56 was issued by the secretary under the
authority and with the presumed approval of the President, the amendment of an FTAA by
merely adopting the fiscal regime prescribed in said DAO 99-56 (and nothing more) need not
have the express clearance of the President anymore. It is as if the same had been preapproved. We cannot fathom the complaint that that makes the secretary more powerful than the
President, or that the former is trying to hide things from the President or Congress.
14. Based on the first sentence of Section 5 of DAO 99-56, which states "[A]ll FTAAs approved prior
to the effectivity of this Administrative Order shall remain valid and be recognized by the
Government", Justice Carpio concludes that said Administrative Order allegedly exempts FTAAs
approved prior to its effectivity -- like the WMCP FTAA -- from having to pay the State any share from
their mining income, apart from taxes, duties and fees.
We disagree. What we see in black and white is the statement that the FTAAs approved before the
DAO came into effect are to continue to be valid and will be recognized by the State. Nothing is said
about their fiscal regimes. Certainly, there is no basis to claim that the contractors under said FTAAs
were being exempted from paying the government a share in their mining incomes.
For the record, the WMCP FTAA is NOT and has never been exempt from paying the government
share. The WMCP FTAA has its own fiscal regime -- Section 7.7 -- which gives the government
a 60 percent share in the net mining revenues of WMCP from the commencement of
commercial production.
For that very reason, we have never said that DAO 99-56 is the basis for claiming that the WMCP
FTAA has a consideration. Hence, we find quite out of place Justice Carpio's statement
that ironically, DAO 99-56, the very authority cited to support the claim that the WMCP FTAA has a
consideration, does not apply to the WMCP FTAA. By its own express terms, DAO 99-56 does not
apply to FTAAs executed before the issuance of DAO 99-56, like the WMCP FTAA. The majority's
position has allegedly no leg to stand on since even DAO 99-56, assuming it is valid, cannot save
the WMCP FTAA from want of consideration. Even assuming arguendo that DAO 99-56 does not
apply to the WMCP FTAA, nevertheless, the WMCP FTAA has its own fiscal regime, found in Section
7.7 thereof. Hence, there is no such thing as "want of consideration" here.
Still more startling is this claim: The majority supposedly agrees that the provisions of the WMCP
FTAA, which grant a sham consideration to the State, are void. Since the majority agrees that the
WMCP FTAA has a sham consideration, the WMCP FTAA thus lacks the third element of a valid
contract. The Decision should declare the WMCP FTAA void for want of consideration unless it
treats the contract as an MPSA under Section 80. Indeed the only recourse of WMCP to save the
validity of its contract is to convert it into an MPSA.
To clarify, we said that Sections 7.9 and 7.8(e) of the WMCP FTAA are provisions grossly
disadvantageous to government and detrimental to the interests of the Filipino people, as well as
violative of public policy, and must therefore be stricken off as invalid. Since the offending provisions
are very much separable from Section 7.7 and the rest of the FTAA, the deletion of Sections 7.9 and
7.8(e) can be done without affecting or requiring the invalidation of the WMCP FTAA itself, and such
deletion will preserve for government its due share of the 60 percent benefits. Therefore, the WMCP
FTAA is NOT bereft of a valid consideration (assuming for the nonce that indeed this is the
"consideration" of the FTAA).
SUMMATION
To conclude, a summary of the key points discussed above is now in order.

The Meaning of "Agreements Involving


Either Technical or Financial Assistance"
Applying familiar principles of constitutional construction to the phrase agreements involving either
technical or financial assistance, the framers' choice of words does not indicate the intent to exclude
other modes of assistance, but rather implies that there are other things being included or possibly
being made part of the agreement, apart from financial or technical assistance. The drafters avoided
the use of restrictive and stringent phraseology; a verba legis scrutiny of Section 2 of Article XII of
the Constitution discloses not even a hint of a desire to prohibit foreign involvement in the
management or operation of mining activities, or to eradicate service contracts. Such moves would
necessarily imply an underlying drastic shift in fundamental economic and developmental policies of
the State. That change requires a much more definite and irrefutable basis than mere omission of
the words "service contract" from the new Constitution.
Furthermore, a literal and restrictive interpretation of this paragraph leads to logical
inconsistencies. A constitutional provision specifically allowing foreign-owned corporations to render
financial or technical assistancein respect of mining or any other commercial activity was clearly
unnecessary; the provision was meant to refer to more than mere financial or technical assistance.
Also, if paragraph 4 permits only agreements for financial or technical assistance, there would be no
point in requiring that they be "based on real contributions to the economic growth and general
welfare of the country."And considering that there were various long-term service contracts still in
force and effect at the time the new Charter was being drafted, the absence of any transitory
provisions to govern the termination and closing-out of the then existing service contracts strongly
militates against the theory that the mere omission of "service contracts" signaled their prohibition by
the new Constitution.
Resort to the deliberations of the Constitutional Commission is therefore unavoidable, and a careful
scrutiny thereof conclusively shows that the ConCom members discussed agreements involving
either technical or financial assistance in the same sense as service contracts and used the terms
interchangeably. The drafters in fact knew that the agreements with foreign corporations were going
to entail not mere technical or financial assistance but, rather, foreign investment in and
management of an enterprise for large-scale exploration, development and utilization of minerals.
The framers spoke about service contracts as the concept was understood in the 1973 Constitution.
It is obvious from their discussions that they did not intend to ban or eradicate service contracts.
Instead, they were intent on crafting provisions to put in place safeguards that would eliminate or
minimize the abuses prevalent during the martial law regime. In brief, they were going to permit
service contracts with foreign corporations as contractors, but with safety measures to
prevent abuses, as an exception to the general norm established in the first paragraph of
Section 2 of Article XII, which reserves or limits to Filipino citizens and corporations at least
60 percent owned by such citizens the exploration, development and utilization of mineral or
petroleum resources. This was prompted by the perceived insufficiency of Filipino capital and the
felt need for foreign expertise in the EDU of mineral resources.
Despite strong opposition from some ConCom members during the final voting, the Article on the
National Economy and Patrimony -- including paragraph 4 allowing service contracts with foreign
corporations as an exception to the general norm in paragraph 1 of Section 2 of the same Article -was resoundingly and overwhelmingly approved.
The drafters, many of whom were economists, academicians, lawyers, businesspersons and
politicians knew that foreign entities will not enter into agreements involving assistance without

requiring measures of protection to ensure the success of the venture and repayment of their
investments, loans and other financial assistance, and ultimately to protect the business reputation
of the foreign corporations. The drafters, by specifying such agreements involving assistance,
necessarily gave implied assent to everything that these agreements entailed or that could
reasonably be deemed necessary to make them tenable and effective -- including management
authority with respect to the day-to-day operations of the enterprise, and measures for the protection
of the interests of the foreign corporation, at least to the extent that they are consistent with
Philippine sovereignty over natural resources, the constitutional requirement of State control, and
beneficial ownership of natural resources remaining vested in the State.
From the foregoing, it is clear that agreements involving either technical or financial
assistance referred to in paragraph 4 are in fact service contracts, but such new service contracts
are between foreign corporations acting as contractors on the one hand, and on the other hand
government as principal or "owner" (of the works), whereby the foreign contractor provides the
capital, technology and technical know-how, and managerial expertise in the creation and operation
of the large-scale mining/extractive enterprise, and government through its agencies (DENR, MGB)
actively exercises full control and supervision over the entire enterprise.
Such service contracts may be entered into only with respect to minerals, petroleum and other
mineral oils. The grant of such service contracts is subject to several safeguards, among them: (1)
that the service contract be crafted in accordance with a general law setting standard or uniform
terms, conditions and requirements; (2) the President be the signatory for the government; and (3)
the President report the executed agreement to Congress within thirty days.
Ultimate Test: Full State Control
To repeat, the primacy of the principle of the State's sovereign ownership of all mineral resources,
and its full control and supervision over all aspects of exploration, development and utilization of
natural resources must be upheld. But "full control and supervision" cannot be taken literally to mean
that the State controls and superviseseverything down to the minutest details and makes all required
actions, as this would render impossible the legitimate exercise by the contractor of a reasonable
degree of management prerogative and authority, indispensable to the proper functioning of the
mining enterprise. Also, government need not micro-manage mining operations and day-to-day
affairs of the enterprise in order to be considered as exercising full control and supervision.
Control, as utilized in Section 2 of Article XII, must be taken to mean a degree of control sufficient to
enable the State to direct, restrain, regulate and govern the affairs of the extractive enterprises.
Control by the State may be on a macro level, through the establishment of policies, guidelines,
regulations, industry standards and similar measures that would enable government to regulate the
conduct of affairs in various enterprises, and restrain activities deemed not desirable or beneficial,
with the end in view of ensuring that these enterprises contribute to the economic development and
general welfare of the country, conserve the environment, and uplift the well-being of the local
affected communities. Such a degree of control would be compatible with permitting the foreign
contractor sufficient and reasonable management authority over the enterprise it has invested in, to
ensure efficient and profitable operation.
Government Granted Full Control
by RA 7942 and DAO 96-40
Baseless are petitioners' sweeping claims that RA 7942 and its Implementing Rules and Regulations
make it possible for FTAA contracts to cede full control and management of mining enterprises over
to fully foreign owned corporations. Equally wobbly is the assertion that the State is reduced to a

passive regulator dependent on submitted plans and reports, with weak review and audit powers
and little say in the decision-making of the enterprise, for which reasons "beneficial ownership" of the
mineral resources is allegedly ceded to the foreign contractor.
As discussed hereinabove, the State's full control and supervision over mining operations are
ensured through the following provisions in RA 7942: Sections 8, 9, 16, 19, 24, 35[(b), (e), (f), (g),
(h), (k), (l), (m) and (o)], 40, 57, 66, 69, 70, and Chapters XI and XVII; as well as the following
provisions of DAO 96-40: Sections7[(d) and (f)], 35(a-2), 53[(a-4) and (d)], 54, 56[(g), (h), (l), (m) and
(n)], 56(2), 60, 66, 144, 168, 171 and 270, and also Chapters XV, XVI and XXIV.
Through the foregoing provisions, the government agencies concerned are empowered to approve
or disapprove -- hence, in a position to influence, direct, and change -- the various work programs
and the corresponding minimum expenditure commitments for each of the exploration, development
and utilization phases of the enterprise. Once they have been approved, the contractor's compliance
with its commitments therein will be monitored. Figures for mineral production and sales are
regularly monitored and subjected to government review, to ensure that the products and byproducts are disposed of at the best prices; copies of sales agreements have to be submitted to and
registered with MGB.
The contractor is mandated to open its books of accounts and records for scrutiny, to enable the
State to determine that the government share has been fully paid. The State may likewise compel
compliance by the contractor with mandatory requirements on mine safety, health and environmental
protection, and the use of anti-pollution technology and facilities. The contractor is also obligated to
assist the development of the mining community, and pay royalties to the indigenous peoples
concerned. And violation of any of the FTAA's terms and conditions, and/or non-compliance with
statutes or regulations, may be penalized by cancellation of the FTAA. Such sanction is significant to
a contractor who may have yet to recover the tens or hundreds of millions of dollars sunk into a
mining project.
Overall, the State definitely has a pivotal say in the operation of the individual enterprises, and can
set directions and objectives, detect deviations and non-compliances by the contractor, and enforce
compliance and impose sanctions should the occasion arise. Hence, RA 7942 and DAO 96-40 vest
in government more than a sufficient degree of control and supervision over the conduct of mining
operations.
Section 3(aq) of RA 7942 was objected to as being unconstitutional for allowing a foreign contractor
to apply for and hold an exploration permit. During the exploration phase, the permit grantee (and
prospective contractor) is spending and investing heavily in exploration activities without yet being
able to extract minerals and generate revenues. The exploration permit issued under Sections 3(aq),
20 and 23 of RA 7942, which allows exploration but not extraction, serves to protect the interests
and rights of the exploration permit grantee (and would-be contractor), foreign or local. Otherwise,
the exploration works already conducted, and expenditures already made, may end up only
benefiting claim-jumpers. Thus, Section 3(aq) of RA 7942 is not unconstitutional.
WMCP FTAA Likewise Gives the
State Full Control and Supervision
The WMCP FTAA obligates the contractor to account for the value of production and sale of minerals
(Clause 1.4); requires that the contractor's work program, activities and budgets be approved by the
State (Clause 2.1); gives the DENR secretary power to extend the exploration period (Clause 3.2-a);
requires approval by the State for incorporation of lands into the contract area (Clause 4.3-c);
requires Bureau of Forest Development approval for inclusion of forest reserves as part of the FTAA

contract area (Clause 4.5); obligates the contractor to periodically relinquish parts of the contract
area not needed for exploration and development (Clause 4.6); requires submission of a declaration
of mining feasibility for approval by the State (Clause 4.6-b); obligates the contractor to report to the
State the results of its exploration activities (Clause 4.9); requires the contractor to obtain State
approval for its work programs for the succeeding two year periods, containing the proposed work
activities and expenditures budget related to exploration (Clause 5.1); requires the contractor to
obtain State approval for its proposed expenditures for exploration activities (Clause 5.2); requires
the contractor to submit an annual report on geological, geophysical, geochemical and other
information relating to its explorations within the FTAA area (Clause 5.3-a); requires the contractor to
submit within six months after expiration of exploration period a final report on all its findings in the
contract area (Clause 5.3-b); requires the contractor after conducting feasibility studies to submit a
declaration of mining feasibility, along with a description of the area to be developed and mined, a
description of the proposed mining operations and the technology to be employed, and the proposed
work program for the development phase, for approval by the DENR secretary (Clause 5.4);
obligates the contractor to complete the development of the mine, including construction of the
production facilities, within the period stated in the approved work program (Clause 6.1); requires the
contractor to submit for approval a work program covering each period of three fiscal years (Clause
6.2); requires the contractor to submit reports to the secretary on the production, ore reserves, work
accomplished and work in progress, profile of its work force and management staff, and other
technical information (Clause 6.3); subjects any expansions, modifications, improvements and
replacements of mining facilities to the approval of the secretary (Clause 6.4); subjects to State
control the amount of funds that the contractor may borrow within the Philippines (Clause 7.2);
subjects to State supervisory power any technical, financial and marketing issues (Clause 10.1-a);
obligates the contractor to ensure 60 percent Filipino equity in the contractor within ten years of
recovering specified expenditures unless not so required by subsequent legislation (Clause 10.1);
gives the State the right to terminate the FTAA for unremedied substantial breach thereof by the
contractor (Clause 13.2); requires State approval for any assignment of the FTAA by the contractor
to an entity other than an affiliate (Clause 14.1).
In short, the aforementioned provisions of the WMCP FTAA, far from constituting a surrender of
control and a grant of beneficial ownership of mineral resources to the contractor in question, vest
the State with control and supervision over practically all aspects of the operations of the FTAA
contractor, including the charging of pre-operating and operating expenses, and the disposition of
mineral products.
There is likewise no relinquishment of control on account of specific provisions of the WMCP FTAA.
Clause 8.2 provides a mechanism to prevent the mining operations from grinding to a complete halt
as a result of possible delays of more than 60 days in the government's processing and approval of
submitted work programs and budgets. Clause 8.3 seeks to provide a temporary, stop-gap solution
in case a disagreement between the State and the contractor (over the proposed work program or
budget submitted by the contractor) should result in a deadlock or impasse, to avoid unreasonably
long delays in the performance of the works.
The State, despite Clause 8.3, still has control over the contract area, and it may, as sovereign
authority, prohibit work thereon until the dispute is resolved, or it may terminate the FTAA, citing
substantial breach thereof. Hence, the State clearly retains full and effective control.
Clause 8.5, which allows the contractor to make changes to approved work programs and budgets
without the prior approval of the DENR secretary, subject to certain limitations with respect to the
variance/s, merely provides the contractor a certain amount of flexibility to meet unexpected
situations, while still guaranteeing that the approved work programs and budgets are not abandoned

altogether. And if the secretary disagrees with the actions taken by the contractor in this instance, he
may also resort to cancellation/termination of the FTAA as the ultimate sanction.
Clause 4.6 of the WMCP FTAA gives the contractor discretion to select parts of the contract area to
be relinquished. The State is not in a position to substitute its judgment for that of the contractor, who
knows exactly which portions of the contract area do not contain minerals in commercial quantities
and should be relinquished. Also, since the annual occupation fees paid to government are based on
the total hectarage of the contract area, net of the areas relinquished, the contractor's self-interest
will assure proper and efficient relinquishment.
Clause 10.2(e) of the WMCP FTAA does not mean that the contractor can compel government to
use its power of eminent domain. It contemplates a situation in which the contractor is a foreignowned corporation, hence, not qualified to own land. The contractor identifies the surface areas
needed for it to construct the infrastructure for mining operations, and the State then acquires the
surface rights on behalf of the former. The provision does not call for the exercise of the power of
eminent domain (or determination of just compensation); it seeks to avoid a violation of the antidummy law.
Clause 10.2(l) of the WMCP FTAA giving the contractor the right to mortgage and encumber the
mineral products extracted may have been a result of conditions imposed by creditor-banks to
secure the loan obligations of WMCP. Banks lend also upon the security of encumbrances on goods
produced, which can be easily sold and converted into cash and applied to the repayment of loans.
Thus, Clause 10.2(l) is not something out of the ordinary. Neither is it objectionable, because even
though the contractor is allowed to mortgage or encumber the mineral end-products themselves, the
contractor is not thereby relieved of its obligation to pay the government its basic and additional
shares in the net mining revenue. The contractor's ability to mortgage the minerals does not negate
the State's right to receive its share of net mining revenues.
Clause 10.2(k) which gives the contractor authority "to change its equity structure at any time,"
means that WMCP, which was then 100 percent foreign owned, could permit Filipino equity
ownership. Moreover, what is important is that the contractor, regardless of its ownership, is always
in a position to render the services required under the FTAA, under the direction and control of the
government.
Clauses 10.4(e) and (i) bind government to allow amendments to the FTAA if required by banks and
other financial institutions as part of the conditions of new lendings. There is nothing objectionable
here, since Clause 10.4(e) also provides that such financing arrangements should in no event
reduce the contractor's obligations or the government's rights under the FTAA. Clause 10.4(i)
provides that government shall "favourably consider" any request for amendments of this agreement
necessary for the contractor to successfully obtain financing. There is no renunciation of control, as
the proviso does not say that government shall automatically grant any such request. Also, it is up to
the contractor to prove the need for the requested changes. The government always has the final
say on whether to approve or disapprove such requests.
In fine, the FTAA provisions do not reduce or abdicate State control.
No Surrender of Financial Benefits
The second paragraph of Section 81 of RA 7942 has been denounced for allegedly limiting the
State's share in FTAAs with foreign contractors to just taxes, fees and duties, and depriving the State
of a share in the after-tax income of the enterprise. However, the inclusion of the phrase "among

other things" in the second paragraph of Section 81 clearly and unmistakably reveals the legislative
intent to have the State collect more than just the usual taxes, duties and fees.
Thus, DAO 99-56, the "Guidelines Establishing the Fiscal Regime of Financial or Technical
Assistance Agreements," spells out the financial benefits government will receive from an FTAA, as
consisting of not only abasic government share, comprised of all direct taxes, fees and royalties,
as well as other payments made by the contractor during the term of the FTAA, but also
an additional government share, being a share in the earnings or cash flows of the mining
enterprise, so as to achieve a fifty-fifty sharing of net benefits from mining between the government
and the contractor.
The additional government share is computed using one of three (3) options or schemes detailed
in DAO 99-56, viz., (1) the fifty-fifty sharing of cumulative present value of cash flows; (2) the excess
profit-related additional government share; and (3) the additional sharing based on the cumulative
net mining revenue. Whichever option or computation is used, the additional government share has
nothing to do with taxes, duties, fees or charges. The portion of revenues remaining after the
deduction of the basic and additional government shares is what goes to the contractor.
The basic government share and the additional government share do not yet take into account the
indirect taxes and other financial contributions of mining projects, which are real and actual benefits
enjoyed by the Filipino people; if these are taken into account, total government share increases to
60 percent or higher (as much as 77 percent, and 89 percent in one instance) of the net present
value of total benefits from the project.
The third or last paragraph of Section 81 of RA 7942 is slammed for deferring the payment of the
government share in FTAAs until after the contractor shall have recovered its pre-operating
expenses, exploration and development expenditures. Allegedly, the collection of the State's share is
rendered uncertain, as there is no time limit in RA 7942 for this grace period or recovery period. But
although RA 7942 did not limit the grace period, the concerned agencies (DENR and MGB) in
formulating the 1995 and 1996 Implementing Rules and Regulations provided that the period of
recovery, reckoned from the date of commercial operation, shall be for a period not exceeding five
years, or until the date of actual recovery, whichever comes earlier.
And since RA 7942 allegedly does not require government approval for the pre-operating,
exploration and development expenses of the foreign contractors, it is feared that such expenses
could be bloated to wipe out mining revenues anticipated for 10 years, with the result that the State's
share is zero for the first 10 years. However, the argument is based on incorrect information.
Under Section 23 of RA 7942, the applicant for exploration permit is required to submit a proposed
work program for exploration, containing a yearly budget of proposed expenditures, which the State
passes upon and either approves or rejects; if approved, the same will subsequently be recorded as
pre-operating expenses that the contractor will have to recoup over the grace period.
Under Section 24, when an exploration permittee files with the MGB a declaration of mining project
feasibility, it must submit a work program for development, with corresponding budget, for approval
by the Bureau, before government may grant an FTAA or MPSA or other mineral agreements; again,
government has the opportunity to approve or reject the proposed work program and budgeted
expenditures for development works, which will become the pre-operating and development costs
that will have to be recovered. Government is able to know ahead of time the amounts of preoperating and other expenses to be recovered, and the approximate period of time needed therefor.
The aforecited provisions have counterparts in Section 35, which deals with the terms and conditions

exclusively applicable to FTAAs. In sum, the third or last paragraph of Section 81 of RA 7942 cannot
be deemed defective.
Section 80 of RA 7942 allegedly limits the State's share in a mineral production-sharing agreement
(MPSA) to just the excise tax on the mineral product, i.e., only 2 percent of market value of the
minerals. The colatilla in Section 84 reiterates the same limitation in Section 80. However, these
two provisions pertain only to MPSAs, and have no application to FTAAs. These particular
provisions do not come within the issues defined by this Court. Hence, on due process
grounds, no pronouncement can be made in this case in respect of the constitutionality of
Sections 80 and 84.
Section 112 is disparaged for reverting FTAAs and all mineral agreements to the old "license,
concession or lease" system, because it allegedly effectively reduces the government share in
FTAAs to just the 2 percent excise tax which pursuant to Section 80 comprises the government
share in MPSAs. However, Section 112 likewise does not come within the issues delineated by this
Court, and was never touched upon by the parties in their pleadings. Moreover, Section 112 may not
properly apply to FTAAs. The mining law obviously meant to treat FTAAs as a breed apart from
mineral agreements. There is absolutely no basis to believe that the law intends to exact from FTAA
contractors merely the same government share (i.e., the 2 percent excise tax) that it apparently
demands from contractors under the three forms of mineral agreements.
While there is ground to believe that Sections 80, 84 and 112 are indeed unconstitutional, they
cannot be ruled upon here. In any event, they are separable; thus, a later finding of nullity will not
affect the rest of RA 7942.
In fine, the challenged provisions of RA 7942 cannot be said to surrender financial benefits
from an FTAA to the foreign contractors.
Moreover, there is no concrete basis for the view that, in FTAAs with a foreign contractor, the State
must receive at least 60 percent of the after-tax income from the exploitation of its mineral
resources, and that such share is the equivalent of the constitutional requirement that at least 60
percent of the capital, and hence 60 percent of the income, of mining companies should remain in
Filipino hands. Even if the State is entitled to a 60 percent share from other mineral agreements
(CPA, JVA and MPSA), that would not create a parallel or analogous situation for FTAAs. We are
dealing with an essentially different equation. Here we have the old apples and oranges syndrome.
The Charter did not intend to fix an iron-clad rule of 60 percent share, applicable to all situations,
regardless of circumstances. There is no indication of such an intention on the part of the framers.
Moreover, the terms and conditions of petroleum FTAAs cannot serve as standards for mineral
mining FTAAs, because the technical and operational requirements, cost structures and
investment needs of off-shore petroleum exploration and drilling companies do not have the
remotest resemblance to those of on-shore mining companies.
To take the position that government's share must be not less than 60 percent of after-tax income of
FTAA contractors is nothing short of this Court dictating upon the government. The State resultantly
ends up losing control. To avoid compromising the State's full control and supervision over the
exploitation of mineral resources, there must be no attempt to impose a "minimum 60 percent" rule.
It is sufficient that the State has the power and means, should it so decide, to get a 60 percent share
(or greater); and it is not necessary that the State does so in every case.
Invalid Provisions of the WMCP FTAA

Section 7.9 of the WMCP FTAA clearly renders illusory the State's 60 percent share of WMCP's
revenues. Under Section 7.9, should WMCP's foreign stockholders (who originally owned 100
percent of the equity) sell 60 percent or more of their equity to a Filipino citizen or corporation, the
State loses its right to receive its share in net mining revenues under Section 7.7, without any
offsetting compensation to the State. And what is given to the State in Section 7.7 is by mere
tolerance of WMCP's foreign stockholders, who can at any time cut off the government's entire share
by simply selling 60 percent of WMCP's equity to a Philippine citizen or corporation.
In fact, the sale by WMCP's foreign stockholder on January 23, 2001 of the entire outstanding equity
in WMCP to Sagittarius Mines, Inc., a domestic corporation at least 60 percent Filipino owned, can
be deemed to have automatically triggered the operation of Section 7.9 and removed the State's
right to receive its 60 percent share. Section 7.9 of the WMCP FTAA has effectively given away the
State's share without anything in exchange.
Moreover, it constitutes unjust enrichment on the part of the local and foreign stockholders in WMCP,
because by the mere act of divestment, the local and foreign stockholders get a windfall, as their
share in the net mining revenues of WMCP is automatically increased, without having to pay
anything for it.
Being grossly disadvantageous to government and detrimental to the Filipino people, as well as
violative of public policy, Section 7.9 must therefore be stricken off as invalid. The FTAA in question
does not involve mere contractual rights but, being impressed as it is with public interest, the
contractual provisions and stipulations must yield to the common good and the national interest.
Since the offending provision is very much separable from the rest of the FTAA, the deletion of
Section 7.9 can be done without affecting or requiring the invalidation of the entire WMCP FTAA
itself.
Section 7.8(e) of the WMCP FTAA likewise is invalid, since by allowing the sums spent by
government for the benefit of the contractor to be deductible from the State's share in net mining
revenues, it results in benefiting the contractor twice over. This constitutes unjust enrichment on the
part of the contractor, at the expense of government. For being grossly disadvantageous and
prejudicial to government and contrary to public policy, Section 7.8(e) must also be declared without
effect. It may likewise be stricken off without affecting the rest of the FTAA.
EPILOGUE
AFTER ALL IS SAID AND DONE, it is clear that there is unanimous agreement in the Court upon the
key principle that the State must exercise full control and supervision over the exploration,
development and utilization of mineral resources.
The crux of the controversy is the amount of discretion to be accorded the Executive Department,
particularly the President of the Republic, in respect of negotiations over the terms of FTAAs,
particularly when it comes to the government share of financial benefits from FTAAs. The Court
believes that it is not unconstitutional to allow a wide degree of discretion to the Chief Executive,
given the nature and complexity of such agreements, the humongous amounts of capital and
financing required for large-scale mining operations, the complicated technology needed, and the
intricacies of international trade, coupled with the State's need to maintain flexibility in its dealings, in
order to preserve and enhance our country's competitiveness in world markets.
We are all, in one way or another, sorely affected by the recently reported scandals involving
corruption in high places, duplicity in the negotiation of multi-billion peso government contracts, huge
payoffs to government officials, and other malfeasances; and perhaps, there is the desire to see

some measures put in place to prevent further abuse. However, dictating upon the President
what minimum share to get from an FTAA is not the solution. It sets a bad precedent since such
a move institutionalizes the very reduction if not deprivation of the State's control. The remedy may
be worse than the problem it was meant to address. In any event, provisions in such future
agreements which may be suspected to be grossly disadvantageous or detrimental to government
may be challenged in court, and the culprits haled before the bar of justice.
Verily, under the doctrine of separation of powers and due respect for co-equal and coordinate
branches of government, this Court must restrain itself from intruding into policy matters and must
allow the President and Congress maximum discretion in using the resources of our country and in
securing the assistance of foreign groups to eradicate the grinding poverty of our people and answer
their cry for viable employment opportunities in the country.
"The judiciary is loath to interfere with the due exercise by coequal branches of government of their
official functions."99 As aptly spelled out seven decades ago by Justice George Malcolm, "Just as the
Supreme Court, as the guardian of constitutional rights, should not sanction usurpations by any
other department of government, so should it as strictly confine its own sphere of influence to the
powers expressly or by implication conferred on it by the Organic Act."100 Let the development of the
mining industry be the responsibility of the political branches of government. And let not this Court
interfere inordinately and unnecessarily.
The Constitution of the Philippines is the supreme law of the land. It is the repository of all the
aspirations and hopes of all the people. We fully sympathize with the plight of Petitioner La Bugal
B'laan and other tribal groups, and commend their efforts to uplift their communities. However, we
cannot justify the invalidation of an otherwise constitutional statute along with its implementing rules,
or the nullification of an otherwise legal and binding FTAA contract.
We must never forget that it is not only our less privileged brethren in tribal and cultural communities
who deserve the attention of this Court; rather, all parties concerned -- including the State itself, the
contractor (whether Filipino or foreign), and the vast majority of our citizens -- equally deserve the
protection of the law and of this Court. To stress, the benefits to be derived by the State from mining
activities must ultimately serve the great majority of our fellow citizens. They have as much right and
interest in the proper and well-ordered development and utilization of the country's mineral resources
as the petitioners.
Whether we consider the near term or take the longer view, we cannot overemphasize the need for
anappropriate balancing of interests and needs -- the need to develop our stagnating mining
industry and extract what NEDA Secretary Romulo Neri estimates is some US$840 billion (approx.
PhP47.04 trillion) worth of mineral wealth lying hidden in the ground, in order to jumpstart our
floundering economy on the one hand, and on the other, the need to enhance our nationalistic
aspirations, protect our indigenous communities, and prevent irreversible ecological damage.
This Court cannot but be mindful that any decision rendered in this case will ultimately impact not
only the cultural communities which lodged the instant Petition, and not only the larger community of
the Filipino people now struggling to survive amidst a fiscal/budgetary deficit, ever increasing prices
of fuel, food, and essential commodities and services, the shrinking value of the local currency, and
a government hamstrung in its delivery of basic services by a severe lack of resources, but also
countless future generations of Filipinos.
For this latter group of Filipinos yet to be born, their eventual access to education, health care and
basic services, their overall level of well-being, the very shape of their lives are even now being

determined and affected partly by the policies and directions being adopted and implemented by
government today. And in part by the this Resolution rendered by this Court today.
Verily, the mineral wealth and natural resources of this country are meant to benefit not merely a
select group of people living in the areas locally affected by mining activities, but the entire Filipino
nation, present and future, to whom the mineral wealth really belong. This Court has therefore
weighed carefully the rights and interests of all concerned, and decided for the greater good of the
greatest number. JUSTICE FOR ALL, not just for some; JUSTICE FOR THE PRESENT AND THE
FUTURE, not just for the here and now.
WHEREFORE, the Court RESOLVES to GRANT the respondents' and the intervenors' Motions for
Reconsideration; to REVERSE and SET ASIDE this Court's January 27, 2004 Decision;
to DISMISS the Petition; and to issue this new judgment declaring CONSTITUTIONAL (1) Republic
Act No. 7942 (the Philippine Mining Law), (2) its Implementing Rules and Regulations contained in
DENR Administrative Order (DAO) No. 9640 -- insofar as they relate to financial and technical
assistance agreements referred to in paragraph 4 of Section 2 of Article XII of the Constitution; and
(3) the Financial and Technical Assistance Agreement (FTAA) dated March 30, 1995 executed by the
government and Western Mining Corporation Philippines Inc. (WMCP), except Sections 7.8 and 7.9
of the subject FTAA which are hereby INVALIDATED for being contrary to public policy and for being
grossly disadvantageous to the government.
SO ORDERED.

G.R. No. 134577. November 18, 1998]

SEN. MIRIAM DEFENSOR SANTIAGO and SEN. FRANCISCO S.


TATAD, petitioners, vs. SEN. TEOFISTO T. GUINGONA, JR. and SEN.
MARCELO B. FERNAN, respondents.
DECISION
PANGANIBAN, J.:

The principle of separation of powers ordains that each of the three great branches of
government has exclusive cognizance of and is supreme in matters falling within its own
constitutionally allocated sphere.
Constitutional respect and a becoming regard for the sovereign acts of a coequal branch
prevents this Court from prying into the internal workings of the Senate. Where no provision of
the Constitution or the laws or even the Rules of the Senate is clearly shown to have been
violated, disregarded or overlooked, grave abuse of discretion cannot be imputed to Senate
officials for acts done within their competence and authority. This Court will be neither a tyrant
nor a wimp; rather, it will remain steadfast and judicious in upholding the rule and majesty of the
law.

The Case

On July 31, 1998, Senators Miriam Defensor Santiago and Francisco S. Tatad instituted an
original petition for quo warranto under Rule 66, Section 5, Rules of Court, seeking the ouster of
Senator Teofisto T. Guingona Jr. as minority leader of the Senate and the declaration of Senator
Tatad as the rightful minority leader.
On August 4, 1998, the Court, upon receipt of the Petition, required the respondents and the
solicitor general to file COMMENT thereon within a non-extendible period of fifteen (15) days
from notice.On August 25, 1998, both respondents and the solicitor general submitted their
respective Comments. In compliance with a Resolution of the Court dated September 1, 1998,
petitioners filed their Consolidated Reply on September 23, 1998. Noting said pleading, this
Court gave due course to the petition and deemed the controversy submitted for decision,
without need of memoranda, on September 29, 1998.
In the regular course, the regional trial courts and this Court have concurrent jurisdiction [1] to
hear and decide petitions for quo warranto (as well as certiorari, prohibition and mandamus),
and a basic deference to the hierarchy of courts impels a filing of such petitions in the lower
tribunals.[2] However, for special and important reasons or for exceptional and compelling
circumstances, as in the present case, this Court has allowed exceptions to this doctrine. [3] In fact,
original petitions for certiorari, prohibition, mandamus and quo warranto assailing acts of
legislative officers like the Senate President[4] and the Speaker of the House[5] have been
recognized as exceptions to this rule.
The Facts

The Senate of the Philippines, with Sen. John Henry R. Osmea as presiding officer,
convened on July 27, 1998 for the first regular session of the eleventh Congress. At
the time, in terms of party affiliation, the composition of the Senate was as follows: [6]
10 members -Laban ng Masang Pilipino (LAMP)
7 members - Lakas-National Union of Christian Democrats-United Muslim
Democrats of the Philippines (Lakas-NUCD-UMDP)
1 member - Liberal Party (LP)
1 member - Aksyon Demokrasya
1 member - Peoples Reform Party (PRP)
1 member - Gabay Bayan
2 members - Independent

---------23 - total number of senators[7] (The last six members are all classified by petitioners as
independent.)
On the agenda for the day was the election of officers. Nominated by Sen. Blas F. Ople to
the position of Senate President was Sen. Marcelo B. Fernan. Sen. Francisco S. Tatad was also
nominated to the same position by Sen. Miriam Defensor Santiago. By a vote of 20 to 2,
[8]
Senator Fernan was declared the duly elected President of the Senate.
The following were likewise elected: Senator Ople as president pro tempore, and Sen.
Franklin M. Drilon as majority leader.
Senator Tatad thereafter manifested that, with the agreement of Senator Santiago, allegedly
the only other member of the minority, he was assuming the position of minority leader. He
explained that those who had voted for Senator Fernan comprised the majority, while only those
who had voted for him, the losing nominee, belonged to the minority.
During the discussion on who should constitute the Senate minority, Sen. Juan M. Flavier
manifested that the senators belonging to the Lakas-NUCD-UMDP Party -- numbering seven (7)
and, thus, also a minority -- had chosen Senator Guingona as the minority leader. No consensus
on the matter was arrived at. The following session day, the debate on the
question continued, with Senators Santiago and Tatad delivering privilege speeches. On the third
session day, the Senate met in caucus, but still failed to resolve the issue.
On July 30, 1998, the majority leader informed the body that he was in receipt of a letter
signed by the seven Lakas-NUCD-UMDP senators,[9] stating that they had elected Senator
Guingona as the minority leader. By virtue thereof, the Senate President formally recognized
Senator Guingona as the minority leader of the Senate.
The following day, Senators Santiago and Tatad filed before this Court the subject petition
for quo warranto, alleging in the main that Senator Guingona had been usurping, unlawfully
holding and exercising the position of Senate minority leader, a position that, according to them,
rightfully belonged to Senator Tatad.
Issues

From the parties pleadings, the Court formulated the following issues for resolution:

1. Does the Court have jurisdiction over the petition?


2. Was there an actual violation of the Constitution?
3. Was Respondent Guingona usurping, unlawfully holding and exercising the
position of Senate minority leader?

4. Did Respondent Fernan act with grave abuse of discretion in recognizing


Respondent Guingona as the minority leader?
The Courts Ruling

After a close perusal of the pleadings[10] and a careful deliberation on the


arguments, pro and con, the Court finds that no constitutional or legal infirmity or grave abuse of
discretion attended the recognition of and the assumption into office by Respondent Guingona as
the Senate minority leader.
First Issue: The Courts Jurisdiction

Petitioners principally invoke Avelino v. Cuenco[11] in arguing that this Court has jurisdiction
to settle the issue of who is the lawful Senate minority leader. They submit that the definitions of
majority and minority involve an interpretation of the Constitution, specifically Section 16 (1),
Article VI thereof, stating that [t]he Senate shall elect its President and the House of
Representatives its Speaker, by a majority vote of all its respective Members.
Respondents and the solicitor general, in their separate Comments, contend in common that
the issue of who is the lawful Senate minority leader is an internal matter pertaining exclusively
to the domain of the legislature, over which the Court cannot exercise jurisdiction without
transgressing the principle of separation of powers. Allegedly, no constitutional issue is involved,
as the fundamental law does not provide for the office of a minority leader in the Senate. The
legislature alone has the full discretion to provide for such office and, in that event, to determine
the procedure of selecting its occupant.
Respondents also maintain that Avelino cannot apply, because there exists no question
involving an interpretation or application of the Constitution, the laws or even the Rules of the
Senate; neither are there peculiar circumstances impelling the Court to assume jurisdiction over
the petition. The solicitor general adds that there is not even any legislative practice to support
the petitioners theory that a senator who votes for the winning Senate President is precluded
from becoming the minority leader.
To resolve the issue of jurisdiction, this Court carefully reviewed and deliberated on the
various important cases involving this very important and basic question, which it has ruled upon
in the past.
The early case Avelino v. Cuenco cautiously tackled the scope of the Courts power of
judicial review; that is, questions involving an interpretation or application of a provision of the
Constitution or the law, including the rules of either house of Congress. Within this scope falls
the jurisdiction of the Court over questions on the validity of legislative or executive acts that are
political in nature, whenever the tribunal finds constitutionally imposed limits on powers or
functions conferred upon political bodies.[12]

In the aforementioned case, the Court initially declined to resolve the question of who was
the rightful Senate President, since it was deemed a political controversy falling exclusively
within the domain of the Senate. Upon a motion for reconsideration, however, the Court
ultimately assumed jurisdiction (1) in the light of subsequent events which justify its
intervention; and (2) because the resolution of the issue hinged on the interpretation of the
constitutional provision on the presence of a quorum to hold a session[13] and therein elect a
Senate President.
Justice
Feria
elucidated
in
his
Concurring
Opinion: [I]
concur
with
the majority that this Court has jurisdiction over cases like the present x x x so as to establish in
this country the judicial supremacy, with the Supreme Court as the final arbiter, to see that no
one branch or agency of the government transcends the Constitution, not only in justiceable but
political questions as well.[14]
Justice Perfecto, also concurring, said in part:

Indeed there is no denying that the situation, as obtaining in the upper chamber of
Congress, is highly explosive. It had echoed in the House of Representatives. It has
already involved the President of the Philippines. The situation has created a veritable
national crisis, and it is apparent that solution cannot be expected from any quarter
other than this Supreme Court, upon which the hopes of the people for an effective
settlement are pinned.[15]
x x x This case raises vital constitutional questions which no one can settle or decide
if this Court should refuse to decide them.[16]
x x x The constitutional question of quorum should not be left unanswered. [17]
In Taada v. Cuenco,[18] this Court endeavored to define political question. And we said that it
refers to those questions which, under the Constitution, are to be decided by the people in their
sovereign capacity, or in regard to which full discretionary authority has been delegated to the
legislative or executive branch of the government. It is concerned with issues dependent upon
the wisdom, not [the] legality, of a particular measure.[19]
The Court ruled that the validity of the selection of members of the Senate Electoral
Tribunal by the senators was not a political question. The choice of these members did not
depend on the Senates full discretionary authority, but was subject to mandatory constitutional
limitations.[20] Thus, the Court held that not only was it clearly within its jurisdiction to pass upon
the validity of the selection proceedings, but it was also its duty to consider and determine the
issue.
In another landmark case, Lansang v. Garcia,[21] Chief Justice Roberto Concepcion wrote
that the Court had authority to and should inquire into the existence of the factual bases required
by the Constitution for the suspension of the privilege of the writ [of habeas corpus]. This ruling
was made in spite of the previous pronouncements in Barcelon v. Baker[22] and Montenegro v.
Castaeda[23] that the authority to decide whether the exigency has arisen requiring suspension (of
the privilege x x x) belongs to the President and his decision is final and conclusive upon the

courts and upon all other persons. But the Chief Justice cautioned: the function of the Court is
merely to check -- not to supplant --- the Executive, or to ascertain merely whether he
has gone beyond the constitutional limits of his jurisdiction, notto exercise the power
vested in him or to determine the wisdom of his act.
The eminent Chief Justice aptly explained later in Javellana v. Executive Secretary:[24]

The reason why the issue under consideration and other issues of similar character are
justiciable, not political, is plain and simple. One of the principal bases of the nonjusticiability of so-called political questions is the principle of separation of powers -characteristic of the presidential system of government -- the functions of which are
classified or divided, by reason of their nature, into three (3) categories, namely, 1)
those involving the making of laws, which are allocated to the legislative department;
2) those concerning mainly with the enforcement of such laws and of judicial
decisions applying and/or interpreting the same, which belong to the executive
department; and 3) those dealing with the settlement of disputes, controversies or
conflicts involving rights, duties or prerogatives that are legally demandable and
enforceable, which are apportioned to courts of justice. Within its own sphere -- but
only within such sphere each department is supreme and independent of the others,
and each is devoid of authority not only to encroach upon the powers or field of action
assigned to any of the other departments, but also to inquire into or pass upon
the advisability or wisdom of the acts performed, measures taken or decisions made by
the other departments -- provided that such acts, measures or decision are within the
area allocated thereto by the Constitution."
Accordingly, when the grant of power is qualified, conditional or subject to
limitations, the issue of whether or not the prescribed qualifications or conditions have
been met, or the limitations respected is justiciable or non-political, the crux of the
problem being one of legality or validity of the contested act, not its
wisdom. Otherwise, said qualifications, conditions or limitations -- particularly those
prescribed by the Constitution -- would be set at naught. What is more, the judicial
inquiry into such issue and the settlement thereof are the main functions of the courts
of justice under the presidential form of government adopted in our 1935 Constitution,
and the system of checks and balances, one of its basic predicates. As a consequence,
we have neither the authority nor the discretion to decline passing upon said issue, but
are under the ineluctable obligation -- made particularly more exacting and
peremptory by our oath, as members of the highest Court of the land, to support and
defend the Constitution -- to settle it. This explains why, in Miller v. Johnson [92 Ky.
589, 18 SW 522, 523], it was held that courts have a duty, rather than a power, to
determine whether another branch of the government has kept within constitutional
limits.

Unlike our previous constitutions, the 1987 Constitution is explicit in defining the scope of
judicial power. The present Constitution now fortifies the authority of the courts to determine in
an appropriate action the validity of the acts of the political departments. It speaks of judicial
prerogative in terms of duty, viz.:

Judicial power includes the duty of the courts of justice to settle actual controversies
involving rights which are legally demandable and enforceable, and to determine
whether or not there has been a grave abuse of discretion amounting to lack or excess
of jurisdiction on the part of any branch or instrumentality of the Government. [25]
This express definition has resulted in clearer and more resolute pronouncements of the
Court. Daza v. Singson,[26] Coseteng v. Mitra Jr.[27] and Guingona Jr. v. Gonzales[28] similarly
resolved issues assailing the acts of the leaders of both houses of Congress in apportioning
among political parties the seats to which each chamber was entitled in the Commission on
Appointments. The Court held that the issue was justiciable, even if the question were political in
nature, since it involved the legality, not the wisdom, of the manner of filling the Commission on
Appointments as prescribed by [Section 18, Article VI of] the Constitution.
The same question of jurisdiction was raised in Taada v. Angara,[29] wherein the petitioners
sought to nullify the Senates concurrence in the ratification of the World Trade Organization
(WTO) Agreement. The Court ruled: Where an action of the legislative branch is seriously
alleged to have infringed the Constitution, it becomes not only the right but in fact the duty of the
judiciary to settle the dispute. The Court en banc unanimously stressed that in taking jurisdiction
over petitions questioning an act of the political departments of government, it will not review
the wisdom, merits or propriety of such action, and will strike it down only on either of two
grounds: (1) unconstitutionality or illegality and (2) grave abuse of discretion.
Earlier in Co v. Electoral Tribunal of the House of Representatives [30] (HRET), the Court
refused to reverse a decision of the HRET, in the absence of a showing that said tribunal had
committed grave abuse of discretion amounting to lack of jurisdiction. The Court ruled that full
authority had been conferred upon the electoral tribunals of the House of Representatives and of
the
Senate
as sole
judges of
all
contests
relating
to
the election, the returns, and the qualifications of their respective members. Such jurisdiction is
original and exclusive.[31] The Court may inquire into a decision or resolution of said tribunals
only if such decision or resolution was rendered without or in excess of jurisdiction, or with
grave abuse of discretion.[32]
Recently, the Court, in Arroyo v. De Venecia,[33] was asked to reexamine the enrolled bill
doctrine and to look beyond the certification of the Speaker of the House of Representatives that
the bill, which was later enacted as Republic Act 8240, was properly approved by the legislative
body. Petitioners claimed that certain procedural rules of the House had been breached in the
passage of the bill. They averred further that a violation of the constitutionally mandated House
rules was a violation of the Constitution itself.
The Court, however, dismissed the petition, because the matter complained of concerned the
internal procedures of the House, with which the Court had no concern. It enucleated:[34]

It would be an unwarranted invasion of the prerogative of a coequal department for


this Court either to set aside a legislative action as void because the Court thinks the
House has disregarded its own rules of procedure, or to allow those defeated in the
political arena to seek a rematch in the judicial forum when petitioners can find their
remedy in that department itself. The Court has not been invested with a roving
commission to inquire into complaints, real or imagined, of legislative skullduggery. It
would be acting in excess of its power and would itself be guilty of grave abuse of
discretion were it to do so. x x x In the absence of anything to the contrary, the Court
must assume that Congress or any House thereof acted in the good faith belief that its
conduct was permitted by its rules, and deference rather than disrespect is due the
judgment of that body.
In the instant controversy, the petitioners -- one of whom is Senator Santiago, a well-known
constitutionalist -- try to hew closely to these jurisprudential parameters. They claim that Section
16 (1), Article VI of the Constitution, has not been observed in the selection of the Senate
minority leader. They also invoke the Courts expanded judicial power to determine whether or
not there has been a grave abuse of discretion amounting to lack or excess of jurisdiction on the
part of respondents.
Dissenting in part, Mr. Justice Vicente V. Mendoza submits that the Court has no jurisdiction
over the petition. Well-settled is the doctrine, however, that jurisdiction over the subject matter of
a case is determined by the allegations of the complaint or petition, regardless of whether the
plaintiff or petitioner is entitled to the relief asserted. [35] In light of the aforesaid allegations of
petitioners, it is clear that this Court has jurisdiction over the petition. It is well within the power
and jurisdiction of the Court to inquire whether indeed the Senate or its officials committed a
violation of the Constitution or gravely abused their discretion in the exercise of their functions
and prerogatives.
Second Issue: Violation of the Constitution

Having assumed jurisdiction over the petition, we now go to the next crucial question: In
recognizing Respondent Guingona as the Senate minority leader, did the Senate or its officials,
particularly Senate President Fernan, violate the Constitution or the laws?
Petitioners answer the above question in the affirmative. They contend that the constitutional
provision requiring the election of the Senate President by majority vote of all its members
carries with it a judicial duty to determine the concepts of majority and minority, as well as who
may elect a minority leader. They argue that majority in the aforequoted constitutional provision
refers to that group of senators who (1) voted for the winning Senate President and (2) accepted
committee chairmanships. Accordingly, those who voted for the losing nominee and accepted no
such chairmanships comprise the minority, to whom the right to determine the minority leader
belongs. As a result, petitioners assert, Respondent Guingona cannot be the legitimate minority
leader, since he voted for Respondent Fernan as Senate President. Furthermore,

the members of the Lakas-NUCD-UMDP cannot choose the minority leader, because they did
not belong to the minority, having voted for Fernan and accepted committee chairmanships.
We believe, however, that the interpretation proposed by petitioners finds no clear support
from the Constitution, the laws, the Rules of the Senate or even from practices of the Upper
House.
The term majority has been judicially defined a number of times. When referring to a certain
number out of a total or aggregate, it simply means the number greater than half or more than
half of any total.[36] The plain and unambiguous words of the subject constitutional clause simply
mean that the Senate President must obtain the votes of more than one half of all the
senators. Not by any construal does it thereby delineate who comprise the majority, much less the
minority, in the said body. And there is no showing that the framers of our Constitution had in
mind other than the usual meanings of these terms.
In effect, while the Constitution mandates that the President of the Senate must be elected by
a number constituting more than one half of all the members thereof, it does not provide that the
members who will not vote for him shall ipso facto constitute the minority, who could thereby
elect the minority leader. Verily, no law or regulation states that the defeated candidate shall
automatically become the minority leader.
The Comment[37] of Respondent Guingona furnishes some relevant precedents, which were
not contested in petitioners Reply. During the eighth Congress, which was the first to convene
after the ratification of the 1987 Constitution, the nomination of Sen. Jovito R. Salonga as Senate
President was seconded by a member of the minority, then Sen. Joseph E. Estrada. [38] During the
ninth regular session, when Sen. Edgardo J. Angara assumed the Senate presidency in 1993, a
consensus was reached to assign committee chairmanships to all senators, including those
belonging to the minority.[39] This practice continued during the tenth Congress, where even the
minority leader was allowed to chair a committee. [40] History would also show that the majority in
either house of Congress has referred to the political party to which the most number of
lawmakers belonged, while the minority normally referred to a party with a lesser number of
members.
Let us go back to the definitions of the terms majority and minority. Majority may also refer
to the group, party, or faction with the larger number of votes, [41] not necessarily more than one
half. This is sometimes referred to as plurality. In contrast, minority is a group, party, or faction
with a smaller number of votes or adherents than the majority.[42] Between two unequal parts or
numbers comprising a whole or totality, the greater number would obviously be the majority,
while the lesser would be the minority. But where there are more than two unequal groupings, it
is not as easy to say which is theminority entitled to select the leader representing all the
minorities. In a government with a multi-party system such as in the Philippines (as pointed out by
petitioners themselves), there could be several minority parties, one of which has to be identified
by the Comelec as the dominant minority party for purposes of the general elections. In the
prevailing composition of the present Senate, members either belong to different political parties
or are independent. No constitutional or statutory provision prescribe which of the many
minority groups or the independents or a combination thereof has the right to select the minority
leader.

While the Constitution is explicit on the manner of electing a Senate President and a House
Speaker, it is, however, dead silent on the manner of selecting the other officers in both chambers
of Congress.All that the Charter says is that [e]ach House shall choose such other officers as it
may deem necessary.[43] To our mind, the method of choosing who will be such other officers is
merely a derivative of the exercise of the prerogative conferred by the aforequoted
constitutional provision. Therefore, such method must be prescribed by the Senate itself,
not by this Court.
In this regard, the Constitution vests in each house of Congress the power to determine the
rules of its proceedings.[44] Pursuant thereto, the Senate formulated and adopted a set of rules to
govern its internal affairs.[45] Pertinent to the instant case are Rules I and II thereof, which
provide:

Rule I
ELECTIVE OFFICERS
SECTION 1. The Senate shall elect, in the manner hereinafter provided, a President, a
President Pro Tempore, a Secretary, and a Sergeant-at-Arms.
These officers shall take their oath of office before entering into the discharge of their
duties.
Rule II
ELECTION OF OFFICERS
SEC. 2. The officers of the Senate shall be elected by the majority vote of all its
Members. Should there be more than one candidate for the same office, a nominal
vote shall be taken; otherwise, the elections shall be by viva voce or by resolution.
Notably, the Rules of the Senate do not provide for the positions of majority and minority
leaders. Neither is there an open clause providing specifically for such offices and prescribing the
manner of creating them or of choosing the holders thereof. At any rate, such offices, by tradition
and long practice, are actually extant. But, in the absence of constitutional or statutory guidelines
or specific rules, this Court is devoid of any basis upon which to determine the legality of the
acts of the Senate relative thereto. On grounds of respect for the basic concept of separation of
powers, courts may not intervene in the internal affairs of the legislature; it is not within the
province of courts to direct Congress how to do its work. [46] Paraphrasing the words of Justice
Florentino P. Feliciano, this Court is of the opinion that where no specific, operable norms and
standards are shown to exist, then the legislature must be given a real and effective opportunity
to fashion and promulgate as well as to implement them, before the courts may intervene.[47]
Needless to state, legislative rules, unlike statutory laws, do not have the imprints of
permanence and obligatoriness during their effectivity. In fact, they are subject to revocation,
modification or waiver at the pleasure of the body adopting them.[48] Being merely matters of

procedure, their observance are of no concern to the courts, for said rules may be waived or
disregarded by the legislative body[49] at will, upon the concurrence of a majority.
In view of the foregoing, Congress verily has the power and prerogative to provide for such
officers as it may deem. And it is certainly within its own jurisdiction and discretion to prescribe
the parameters for the exercise of this prerogative. This Court has no authority to interfere
and unilaterally intrude into that exclusive realm, without running afoul of constitutional
principles that it is bound to protect and uphold -- the very duty that justifies the Courts
being. Constitutional respect and a becoming regard for the sovereign acts of a coequal
branch prevents this Court from prying into the internal workings of the Senate. To repeat,
this Court will be neither a tyrant nor a wimp; rather, it will remain steadfast and judicious
in upholding the rule and majesty of the law.
To accede, then, to the interpretation of petitioners would practically amount to
judicial legislation, a clear breach of the constitutional doctrine of separation of powers. If
for this argument alone, the petition would easily fail.
While no provision of the Constitution or the laws or the rules and even the practice of the
Senate was violated, and while the judiciary is without power to decide matters over which full
discretionary authority has been lodged in the legislative department, this Court may still inquire
whether an act of Congress or its officials has been made with grave abuse of discretion.[50] This
is the plain implication of Section 1, Article VIII of the Constitution, which expressly confers
upon the judiciary the power and the duty not only to settle actual controversies involving rights
which are legally demandable and enforceable, but likewise to determine whether or not there
has been a grave abuse of discretion amounting to lack or excess of jurisdiction on the part of
any branch or instrumentality of the Government.
Explaining the above-quoted clause, former Chief Justice Concepcion, who was a member
of the 1986 Constitutional Commission, said in part:[51]

xxx the powers of government are generally considered divided into three
branches: the Legislative, the Executive and the Judiciary. Each one is supreme within
its own sphere and independent of the others.Because of that supremacy[, the] power
to determine whether a given law is valid or not is vested in courts of justice.
Briefly stated, courts of justice determine the limits of power of the agencies and
offices of the government as well as those of its officers. In other words, the judiciary
is the final arbiter on the question whether or not a branch of government or any of its
officials has acted without jurisdiction or in excess of jurisdiction, or so capriciously
as to constitute an abuse of discretion amounting to excess of jurisdiction or lack of
jurisdiction. This is not only a judicial power but a duty to pass judgment on matters
of this nature.
This is the background of paragraph 2 of Section 1, which means that the courts
cannot hereafter evade the duty to settle matters of this nature, by claiming that such
matters constitute a political question.

With this paradigm, we now examine the two other issues challenging the actions, first, of
Respondent Guingona and, second, of Respondent Fernan.
Third Issue: Usurpation of Office

Usurpation generally refers to unauthorized arbitrary assumption and exercise of power [52] by
one without color of title or who is not entitled by law thereto. [53] A quo warranto proceeding is
the proper legal remedy to determine the right or title to the contested public office and to oust
the holder from its enjoyment.[54] The action may be brought by the solicitor general or a public
prosecutor[55] or any person claiming to be entitled to the public office or position usurped or
unlawfully held or exercised by another.[56] The action shall be brought against the person who
allegedly usurped, intruded into or is unlawfully holding or exercising such office.[57]
In order for a quo warranto proceeding to be successful, the person suing must show that he
or she has a clear right to the contested office or to use or exercise the functions of the office
allegedly usurped or unlawfully held by the respondent. [58] In this case, petitioners present no
sufficient proof of a clear and indubitable franchise to the office of the Senate minority leader.
As discussed earlier, the specific norms or standards that may be used in determining who
may lawfully occupy the disputed position has not been laid down by the Constitution, the
statutes, or the Senate itself in which the power has been vested. Absent any clear-cut guideline,
in no way can it be said that illegality or irregularity tainted Respondent Guingonas assumption
and exercise of the powers of the office of Senate minority leader. Furthermore, no grave abuse
of discretion has been shown to characterize any of his specific acts as minority leader.
Fourth Issue: Fernans Recognition of Guingona

The all-embracing and plenary power and duty of the Court to determine whether or not
there has been a grave abuse of discretion amounting to lack or excess of jurisdiction on the part
of any branch or instrumentality of the Government is restricted only by the definition and
confines of the term grave abuse of discretion.

By grave abuse of discretion is meant such capricious or whimsical exercise of


judgment as is equivalent to lack of jurisdiction. The abuse of discretion must be
patent and gross as to amount to an evasion of positive duty or a virtual refusal to
perform a duty enjoined by law, or to act at all in contemplation of law as where
the power is exercised in an arbitrary and despotic manner by reason of passion
and hostility.[59]
By the above standard, we hold that Respondent Fernan did not gravely abuse his discretion
as Senate President in recognizing Respondent Guingona as the minority leader. Let us recall that
the latter belongs to one of the minority parties in the Senate, the Lakas-NUCD-UMDP. By
unanimous resolution of the members of this party that he be the minority leader, he was

recognized as such by the Senate President. Such formal recognition by Respondent Fernan came
only after at least two Senate sessions and a caucus, wherein both sides were liberally allowed to
articulate their standpoints.
Under these circumstances, we believe that the Senate President cannot be accused of
capricious or whimsical exercise of judgment or of an arbitrary and despotic manner by reason of
passion or hostility. Where no provision of the Constitution, the laws or even the rules of the
Senate has been clearly shown to have been violated, disregarded or overlooked, grave
abuse of discretion cannot be imputed to Senate officials for acts done within their
competence and authority.
WHEREFORE, for the above reasons, the petition is hereby DISMISSED.
Republic of the Philippines
SUPREME COURT
Manila
EN BANC
G.R. No. 159796

July 17, 2007

ROMEO P. GEROCHI, KATULONG NG BAYAN (KB) and ENVIRONMENTALIST CONSUMERS


NETWORK, INC. (ECN), Petitioners,
vs.
DEPARTMENT OF ENERGY (DOE), ENERGY REGULATORY COMMISSION (ERC), NATIONAL
POWER CORPORATION (NPC), POWER SECTOR ASSETS AND LIABILITIES MANAGEMENT
GROUP (PSALM Corp.), STRATEGIC POWER UTILITIES GROUP (SPUG), and PANAY
ELECTRIC COMPANY INC. (PECO),Respondents.
DECISION
NACHURA, J.:
Petitioners Romeo P. Gerochi, Katulong Ng Bayan (KB), and Environmentalist Consumers Network,
Inc. (ECN) (petitioners), come before this Court in this original action praying that Section 34 of
Republic Act (RA) 9136, otherwise known as the "Electric Power Industry Reform Act of 2001"
(EPIRA), imposing the Universal Charge, 1and Rule 18 of the Rules and Regulations (IRR) 2 which
seeks to implement the said imposition, be declared unconstitutional. Petitioners also pray that the
Universal Charge imposed upon the consumers be refunded and that a preliminary injunction and/or
temporary restraining order (TRO) be issued directing the respondents to refrain from implementing,
charging, and collecting the said charge.3 The assailed provision of law reads:
SECTION 34. Universal Charge. Within one (1) year from the effectivity of this Act, a universal
charge to be determined, fixed and approved by the ERC, shall be imposed on all electricity endusers for the following purposes:
(a) Payment for the stranded debts4 in excess of the amount assumed by the National
Government and stranded contract costs of NPC5 and as well as qualified stranded contract
costs of distribution utilities resulting from the restructuring of the industry;
(b) Missionary electrification;6

(c) The equalization of the taxes and royalties applied to indigenous or renewable sources of
energy vis--vis imported energy fuels;
(d) An environmental charge equivalent to one-fourth of one centavo per kilowatt-hour
(P0.0025/kWh), which shall accrue to an environmental fund to be used solely for watershed
rehabilitation and management. Said fund shall be managed by NPC under existing
arrangements; and
(e) A charge to account for all forms of cross-subsidies for a period not exceeding three (3)
years.
The universal charge shall be a non-bypassable charge which shall be passed on and collected from
all end-users on a monthly basis by the distribution utilities. Collections by the distribution utilities
and the TRANSCO in any given month shall be remitted to the PSALM Corp. on or before the
fifteenth (15th) of the succeeding month, net of any amount due to the distribution utility. Any enduser or self-generating entity not connected to a distribution utility shall remit its corresponding
universal charge directly to the TRANSCO. The PSALM Corp., as administrator of the fund, shall
create a Special Trust Fund which shall be disbursed only for the purposes specified herein in an
open and transparent manner. All amount collected for the universal charge shall be distributed to
the respective beneficiaries within a reasonable period to be provided by the ERC.
The Facts
Congress enacted the EPIRA on June 8, 2001; on June 26, 2001, it took effect. 7
On April 5, 2002, respondent National Power Corporation-Strategic Power Utilities Group 8 (NPCSPUG) filed with respondent Energy Regulatory Commission (ERC) a petition for the availment from
the Universal Charge of its share for Missionary Electrification, docketed as ERC Case No. 2002165.9
On May 7, 2002, NPC filed another petition with ERC, docketed as ERC Case No. 2002-194,
praying that the proposed share from the Universal Charge for the Environmental charge of P0.0025
per kilowatt-hour (/kWh), or a total of P119,488,847.59, be approved for withdrawal from the Special
Trust Fund (STF) managed by respondent Power Sector Assets and
Liabilities Management Group (PSALM)10 for the rehabilitation and management of watershed
areas.11
On December 20, 2002, the ERC issued an Order 12 in ERC Case No. 2002-165 provisionally
approving the computed amount of P0.0168/kWh as the share of the NPC-SPUG from the Universal
Charge for Missionary Electrification and authorizing the National Transmission Corporation
(TRANSCO) and Distribution Utilities to collect the same from its end-users on a monthly basis.
On June 26, 2003, the ERC rendered its Decision13 (for ERC Case No. 2002-165) modifying its
Order of December 20, 2002, thus:
WHEREFORE, the foregoing premises considered, the provisional authority granted to petitioner
National Power Corporation-Strategic Power Utilities Group (NPC-SPUG) in the Order dated
December 20, 2002 is hereby modified to the effect that an additional amount of P0.0205 per
kilowatt-hour should be added to the P0.0168 per kilowatt-hour provisionally authorized by the
Commission in the said Order. Accordingly, a total amount ofP0.0373 per kilowatt-hour is hereby

APPROVED for withdrawal from the Special Trust Fund managed by PSALM as its share from the
Universal Charge for Missionary Electrification (UC-ME) effective on the following billing cycles:
(a) June 26-July 25, 2003 for National Transmission Corporation (TRANSCO); and
(b) July 2003 for Distribution Utilities (Dus).
Relative thereto, TRANSCO and Dus are directed to collect the UC-ME in the amount of P0.0373
per kilowatt-hour and remit the same to PSALM on or before the 15th day of the succeeding month.
In the meantime, NPC-SPUG is directed to submit, not later than April 30, 2004, a detailed report to
include Audited Financial Statements and physical status (percentage of completion) of the projects
using the prescribed format.
1avvphi1

Let copies of this Order be furnished petitioner NPC-SPUG and all distribution utilities (Dus).
SO ORDERED.
On August 13, 2003, NPC-SPUG filed a Motion for Reconsideration asking the ERC, among
others,14 to set aside the above-mentioned Decision, which the ERC granted in its Order dated
October 7, 2003, disposing:
WHEREFORE, the foregoing premises considered, the "Motion for Reconsideration" filed by
petitioner National Power Corporation-Small Power Utilities Group (NPC-SPUG) is hereby
GRANTED. Accordingly, the Decision dated June 26, 2003 is hereby modified accordingly.
Relative thereto, NPC-SPUG is directed to submit a quarterly report on the following:
1. Projects for CY 2002 undertaken;
2. Location
3. Actual amount utilized to complete the project;
4. Period of completion;
5. Start of Operation; and
6. Explanation of the reallocation of UC-ME funds, if any.
SO ORDERED.15
Meanwhile, on April 2, 2003, ERC decided ERC Case No. 2002-194, authorizing the NPC to draw up
toP70,000,000.00 from PSALM for its 2003 Watershed Rehabilitation Budget subject to the
availability of funds for the Environmental Fund component of the Universal Charge. 16
On the basis of the said ERC decisions, respondent Panay Electric Company, Inc. (PECO) charged
petitioner Romeo P. Gerochi and all other end-users with the Universal Charge as reflected in their
respective electric bills starting from the month of July 2003. 17

Hence, this original action.


Petitioners submit that the assailed provision of law and its IRR which sought to implement the same
are unconstitutional on the following grounds:
1) The universal charge provided for under Sec. 34 of the EPIRA and sought to be
implemented under Sec. 2, Rule 18 of the IRR of the said law is a tax which is to be collected
from all electric end-users and self-generating entities. The power to tax is strictly a
legislative function and as such, the delegation of said power to any executive or
administrative agency like the ERC is unconstitutional, giving the same unlimited authority.
The assailed provision clearly provides that the Universal Charge is to be determined, fixed
and approved by the ERC, hence leaving to the latter complete discretionary legislative
authority.
2) The ERC is also empowered to approve and determine where the funds collected should
be used.
3) The imposition of the Universal Charge on all end-users is oppressive and confiscatory
and amounts to taxation without representation as the consumers were not given a chance
to be heard and represented.18
Petitioners contend that the Universal Charge has the characteristics of a tax and is collected to fund
the operations of the NPC. They argue that the cases19 invoked by the respondents clearly show the
regulatory purpose of the charges imposed therein, which is not so in the case at bench. In said
cases, the respective funds20 were created in order to balance and stabilize the prices of oil and
sugar, and to act as buffer to counteract the changes and adjustments in prices, peso devaluation,
and other variables which cannot be adequately and timely monitored by the legislature. Thus, there
was a need to delegate powers to administrative bodies.21 Petitioners posit that the Universal Charge
is imposed not for a similar purpose.
On the other hand, respondent PSALM through the Office of the Government Corporate Counsel
(OGCC) contends that unlike a tax which is imposed to provide income for public purposes, such as
support of the government, administration of the law, or payment of public expenses, the assailed
Universal Charge is levied for a specific regulatory purpose, which is to ensure the viability of the
country's electric power industry. Thus, it is exacted by the State in the exercise of its inherent police
power. On this premise, PSALM submits that there is no undue delegation of legislative power to the
ERC since the latter merely exercises a limited authority or discretion as to the execution and
implementation of the provisions of the EPIRA.22
Respondents Department of Energy (DOE), ERC, and NPC, through the Office of the Solicitor
General (OSG), share the same view that the Universal Charge is not a tax because it is levied for a
specific regulatory purpose, which is to ensure the viability of the country's electric power industry,
and is, therefore, an exaction in the exercise of the State's police power. Respondents further
contend that said Universal Charge does not possess the essential characteristics of a tax, that its
imposition would redound to the benefit of the electric power industry and not to the public, and that
its rate is uniformly levied on electricity end-users, unlike a tax which is imposed based on the
individual taxpayer's ability to pay. Moreover, respondents deny that there is undue delegation of
legislative power to the ERC since the EPIRA sets forth sufficient determinable standards which
would guide the ERC in the exercise of the powers granted to it. Lastly, respondents argue that the
imposition of the Universal Charge is not oppressive and confiscatory since it is an exercise of the
police power of the State and it complies with the requirements of due process. 23

On its part, respondent PECO argues that it is duty-bound to collect and remit the amount pertaining
to the Missionary Electrification and Environmental Fund components of the Universal Charge,
pursuant to Sec. 34 of the EPIRA and the Decisions in ERC Case Nos. 2002-194 and 2002-165.
Otherwise, PECO could be held liable under Sec. 4624 of the EPIRA, which imposes fines and
penalties for any violation of its provisions or its IRR. 25
The Issues
The ultimate issues in the case at bar are:
1) Whether or not, the Universal Charge imposed under Sec. 34 of the EPIRA is a tax; and
2) Whether or not there is undue delegation of legislative power to tax on the part of the
ERC.26
Before we discuss the issues, the Court shall first deal with an obvious procedural lapse.
Petitioners filed before us an original action particularly denominated as a Complaint assailing the
constitutionality of Sec. 34 of the EPIRA imposing the Universal Charge and Rule 18 of the EPIRA's
IRR. No doubt, petitioners have locus standi. They impugn the constitutionality of Sec. 34 of the
EPIRA because they sustained a direct injury as a result of the imposition of the Universal Charge as
reflected in their electric bills.
However, petitioners violated the doctrine of hierarchy of courts when they filed this "Complaint"
directly with us. Furthermore, the Complaint is bereft of any allegation of grave abuse of discretion
on the part of the ERC or any of the public respondents, in order for the Court to consider it as a
petition for certiorari or prohibition.
Article VIII, Section 5(1) and (2) of the 1987 Constitution 27 categorically provides that:
SECTION 5. The Supreme Court shall have the following powers:
1. Exercise original jurisdiction over cases affecting ambassadors, other public ministers and
consuls, and over petitions for certiorari, prohibition, mandamus, quo warranto, and habeas
corpus.
2. Review, revise, reverse, modify, or affirm on appeal or certiorari, as the law or the rules of
court may provide, final judgments and orders of lower courts in:
(a) All cases in which the constitutionality or validity of any treaty, international or executive
agreement, law, presidential decree, proclamation, order, instruction, ordinance, or regulation is in
question.
But this Court's jurisdiction to issue writs of certiorari, prohibition, mandamus, quo warranto,
and habeas corpus, while concurrent with that of the regional trial courts and the Court of Appeals,
does not give litigants unrestrained freedom of choice of forum from which to seek such relief. 28 It
has long been established that this Court will not entertain direct resort to it unless the redress
desired cannot be obtained in the appropriate courts, or where exceptional and compelling
circumstances justify availment of a remedy within and call for the exercise of our primary
jurisdiction.29 This circumstance alone warrants the outright dismissal of the present action.

This procedural infirmity notwithstanding, we opt to resolve the constitutional issue raised herein. We
are aware that if the constitutionality of Sec. 34 of the EPIRA is not resolved now, the issue will
certainly resurface in the near future, resulting in a repeat of this litigation, and probably involving the
same parties. In the public interest and to avoid unnecessary delay, this Court renders its ruling now.
The instant complaint is bereft of merit.
The First Issue
To resolve the first issue, it is necessary to distinguish the States power of taxation from the police
power.
The power to tax is an incident of sovereignty and is unlimited in its range, acknowledging in its very
nature no limits, so that security against its abuse is to be found only in the responsibility of the
legislature which imposes the tax on the constituency that is to pay it. 30 It is based on the principle
that taxes are the lifeblood of the government, and their prompt and certain availability is an
imperious need.31 Thus, the theory behind the exercise of the power to tax emanates from necessity;
without taxes, government cannot fulfill its mandate of promoting the general welfare and well-being
of the people.32
On the other hand, police power is the power of the state to promote public welfare by restraining
and regulating the use of liberty and property.33 It is the most pervasive, the least limitable, and the
most demanding of the three fundamental powers of the State. The justification is found in the Latin
maxims salus populi est suprema lex (the welfare of the people is the supreme law) and sic utere
tuo ut alienum non laedas (so use your property as not to injure the property of others). As an
inherent attribute of sovereignty which virtually extends to all public needs, police power grants a
wide panoply of instruments through which the State, as parens patriae, gives effect to a host of its
regulatory powers.34 We have held that the power to "regulate" means the power to protect, foster,
promote, preserve, and control, with due regard for the interests, first and foremost, of the public,
then of the utility and of its patrons.35
The conservative and pivotal distinction between these two powers rests in the purpose for which
the charge is made. If generation of revenue is the primary purpose and regulation is merely
incidental, the imposition is a tax; but if regulation is the primary purpose, the fact that revenue is
incidentally raised does not make the imposition a tax.36
In exacting the assailed Universal Charge through Sec. 34 of the EPIRA, the State's police power,
particularly its regulatory dimension, is invoked. Such can be deduced from Sec. 34 which
enumerates the purposes for which the Universal Charge is imposed 37 and which can be amply
discerned as regulatory in character. The EPIRA resonates such regulatory purposes, thus:
SECTION 2. Declaration of Policy. It is hereby declared the policy of the State:
(a) To ensure and accelerate the total electrification of the country;
(b) To ensure the quality, reliability, security and affordability of the supply of electric power;
(c) To ensure transparent and reasonable prices of electricity in a regime of free and fair
competition and full public accountability to achieve greater operational and economic
efficiency and enhance the competitiveness of Philippine products in the global market;

(d) To enhance the inflow of private capital and broaden the ownership base of the power
generation, transmission and distribution sectors;
(e) To ensure fair and non-discriminatory treatment of public and private sector entities in the
process of restructuring the electric power industry;
(f) To protect the public interest as it is affected by the rates and services of electric utilities
and other providers of electric power;
(g) To assure socially and environmentally compatible energy sources and infrastructure;
(h) To promote the utilization of indigenous and new and renewable energy resources in
power generation in order to reduce dependence on imported energy;
(i) To provide for an orderly and transparent privatization of the assets and liabilities of the
National Power Corporation (NPC);
(j) To establish a strong and purely independent regulatory body and system to ensure
consumer protection and enhance the competitive operation of the electricity market; and
(k) To encourage the efficient use of energy and other modalities of demand side
management.
From the aforementioned purposes, it can be gleaned that the assailed Universal Charge is not a
tax, but an exaction in the exercise of the State's police power. Public welfare is surely promoted.
Moreover, it is a well-established doctrine that the taxing power may be used as an implement of
police power.38In Valmonte v. Energy Regulatory Board, et al.39 and in Gaston v. Republic Planters
Bank,40 this Court held that the Oil Price Stabilization Fund (OPSF) and the Sugar Stabilization Fund
(SSF) were exactions made in the exercise of the police power. The doctrine was reiterated
in Osmea v. Orbos41 with respect to the OPSF. Thus, we disagree with petitioners that the instant
case is different from the aforementioned cases. With the Universal Charge, a Special Trust Fund
(STF) is also created under the administration of PSALM. 42 The STF has some notable
characteristics similar to the OPSF and the SSF, viz.:
1) In the implementation of stranded cost recovery, the ERC shall conduct a review to
determine whether there is under-recovery or over recovery and adjust (true-up) the level of
the stranded cost recovery charge. In case of an over-recovery, the ERC shall ensure that
any excess amount shall be remitted to the STF. A separate account shall be created for
these amounts which shall be held in trust for any future claims of distribution utilities for
stranded cost recovery. At the end of the stranded cost recovery period, any remaining
amount in this account shall be used to reduce the electricity rates to the end-users. 43
2) With respect to the assailed Universal Charge, if the total amount collected for the same is
greater than the actual availments against it, the PSALM shall retain the balance within the
STF to pay for periods where a shortfall occurs.44
3) Upon expiration of the term of PSALM, the administration of the STF shall be transferred
to the DOF or any of the DOF attached agencies as designated by the DOF Secretary.45
The OSG is in point when it asseverates:

Evidently, the establishment and maintenance of the Special Trust Fund, under the last paragraph of
Section 34, R.A. No. 9136, is well within the pervasive and non-waivable power and responsibility of
the government to secure the physical and economic survival and well-being of the community, that
comprehensive sovereign authority we designate as the police power of the State. 46
This feature of the Universal Charge further boosts the position that the same is an exaction
imposed primarily in pursuit of the State's police objectives. The STF reasonably serves and assures
the attainment and perpetuity of the purposes for which the Universal Charge is imposed, i.e., to
ensure the viability of the country's electric power industry.
The Second Issue
The principle of separation of powers ordains that each of the three branches of government has
exclusive cognizance of and is supreme in matters falling within its own constitutionally allocated
sphere. A logical corollary to the doctrine of separation of powers is the principle of non-delegation of
powers, as expressed in the Latin maxim potestas delegata non delegari potest (what has been
delegated cannot be delegated). This is based on the ethical principle that such delegated power
constitutes not only a right but a duty to be performed by the delegate through the instrumentality of
his own judgment and not through the intervening mind of another. 47
In the face of the increasing complexity of modern life, delegation of legislative power to various
specialized administrative agencies is allowed as an exception to this principle. 48 Given the volume
and variety of interactions in today's society, it is doubtful if the legislature can promulgate laws that
will deal adequately with and respond promptly to the minutiae of everyday life. Hence, the need to
delegate to administrative bodies - the principal agencies tasked to execute laws in their specialized
fields - the authority to promulgate rules and regulations to implement a given statute and effectuate
its policies. All that is required for the valid exercise of this power of subordinate legislation is that the
regulation be germane to the objects and purposes of the law and that the regulation be not in
contradiction to, but in conformity with, the standards prescribed by the law. These requirements are
denominated as the completeness test and the sufficient standard test.
Under the first test, the law must be complete in all its terms and conditions when it leaves the
legislature such that when it reaches the delegate, the only thing he will have to do is to enforce it.
The second test mandates adequate guidelines or limitations in the law to determine the boundaries
of the delegate's authority and prevent the delegation from running riot. 49
The Court finds that the EPIRA, read and appreciated in its entirety, in relation to Sec. 34 thereof, is
complete in all its essential terms and conditions, and that it contains sufficient standards.
Although Sec. 34 of the EPIRA merely provides that "within one (1) year from the effectivity thereof,
a Universal Charge to be determined, fixed and approved by the ERC, shall be imposed on all
electricity end-users," and therefore, does not state the specific amount to be paid as Universal
Charge, the amount nevertheless is made certain by the legislative parameters provided in the law
itself. For one, Sec. 43(b)(ii) of the EPIRA provides:
SECTION 43. Functions of the ERC. The ERC shall promote competition, encourage market
development, ensure customer choice and penalize abuse of market power in the restructured
electricity industry. In appropriate cases, the ERC is authorized to issue cease and desist order after
due notice and hearing. Towards this end, it shall be responsible for the following key functions in the
restructured industry:
xxxx

(b) Within six (6) months from the effectivity of this Act, promulgate and enforce, in accordance with
law, a National Grid Code and a Distribution Code which shall include, but not limited to the
following:
xxxx
(ii) Financial capability standards for the generating companies, the TRANSCO, distribution utilities
and suppliers: Provided, That in the formulation of the financial capability standards, the nature and
function of the entity shall be considered: Provided, further, That such standards are set to ensure
that the electric power industry participants meet the minimum financial standards to protect the
public interest. Determine, fix, and approve, after due notice and public hearings the universal
charge, to be imposed on all electricity end-users pursuant to Section 34 hereof;
Moreover, contrary to the petitioners contention, the ERC does not enjoy a wide latitude of discretion
in the determination of the Universal Charge. Sec. 51(d) and (e) of the EPIRA 50 clearly provides:
SECTION 51. Powers. The PSALM Corp. shall, in the performance of its functions and for the
attainment of its objective, have the following powers:
xxxx
(d) To calculate the amount of the stranded debts and stranded contract costs of NPC
which shall form the basis for ERC in the determination of the universal charge;
(e) To liquidate the NPC stranded contract costs, utilizing the proceeds from sales and other
property contributed to it, including the proceeds from the universal charge.
Thus, the law is complete and passes the first test for valid delegation of legislative power.
As to the second test, this Court had, in the past, accepted as sufficient standards the following:
"interest of law and order;"51 "adequate and efficient instruction;"52 "public interest;"53 "justice and
equity;"54 "public convenience and welfare;"55 "simplicity, economy and efficiency;"56 "standardization
and regulation of medical education;"57and "fair and equitable employment practices."58 Provisions of
the EPIRA such as, among others, "to ensure the total electrification of the country and the quality,
reliability, security and affordability of the supply of electric power" 59 and "watershed rehabilitation
and management"60 meet the requirements for valid delegation, as they provide the limitations on the
ERCs power to formulate the IRR. These are sufficient standards.
It may be noted that this is not the first time that the ERC's conferred powers were challenged.
In Freedom from Debt Coalition v. Energy Regulatory Commission,61 the Court had occasion to say:
In determining the extent of powers possessed by the ERC, the provisions of the EPIRA must not be
read in separate parts. Rather, the law must be read in its entirety, because a statute is passed as a
whole, and is animated by one general purpose and intent. Its meaning cannot to be extracted from
any single part thereof but from a general consideration of the statute as a whole. Considering the
intent of Congress in enacting the EPIRA and reading the statute in its entirety, it is plain to see that
the law has expanded the jurisdiction of the regulatory body, the ERC in this case, to enable the
latter to implement the reforms sought to be accomplished by the EPIRA. When the legislators
decided to broaden the jurisdiction of the ERC, they did not intend to abolish or reduce the powers
already conferred upon ERC's predecessors. To sustain the view that the ERC possesses only the
powers and functions listed under Section 43 of the EPIRA is to frustrate the objectives of the law.

In his Concurring and Dissenting Opinion62 in the same case, then Associate Justice, now Chief
Justice, Reynato S. Puno described the immensity of police power in relation to the delegation of
powers to the ERC and its regulatory functions over electric power as a vital public utility, to wit:
Over the years, however, the range of police power was no longer limited to the preservation of
public health, safety and morals, which used to be the primary social interests in earlier times. Police
power now requires the State to "assume an affirmative duty to eliminate the excesses and
injustices that are the concomitants of an unrestrained industrial economy." Police power is now
exerted "to further the public welfare a concept as vast as the good of society itself." Hence,
"police power is but another name for the governmental authority to further the welfare of society
that is the basic end of all government." When police power is delegated to administrative bodies
with regulatory functions, its exercise should be given a wide latitude. Police power takes on an even
broader dimension in developing countries such as ours, where the State must take a more active
role in balancing the many conflicting interests in society. The Questioned Order was issued by the
ERC, acting as an agent of the State in the exercise of police power. We should have exceptionally
good grounds to curtail its exercise. This approach is more compelling in the field of rate-regulation
of electric power rates. Electric power generation and distribution is a traditional instrument of
economic growth that affects not only a few but the entire nation. It is an important factor in
encouraging investment and promoting business. The engines of progress may come to a
screeching halt if the delivery of electric power is impaired. Billions of pesos would be lost as a result
of power outages or unreliable electric power services. The State thru the ERC should be able to
exercise its police power with great flexibility, when the need arises.
This was reiterated in National Association of Electricity Consumers for Reforms v. Energy
Regulatory Commission63 where the Court held that the ERC, as regulator, should have sufficient
power to respond in real time to changes wrought by multifarious factors affecting public utilities.
From the foregoing disquisitions, we therefore hold that there is no undue delegation of legislative
power to the ERC.
Petitioners failed to pursue in their Memorandum the contention in the Complaint that the imposition
of the Universal Charge on all end-users is oppressive and confiscatory, and amounts to taxation
without representation. Hence, such contention is deemed waived or abandoned per Resolution 64 of
August 3, 2004.65Moreover, the determination of whether or not a tax is excessive, oppressive or
confiscatory is an issue which essentially involves questions of fact, and thus, this Court is precluded
from reviewing the same.66
As a penultimate statement, it may be well to recall what this Court said of EPIRA:
One of the landmark pieces of legislation enacted by Congress in recent years is the EPIRA. It
established a new policy, legal structure and regulatory framework for the electric power industry.
The new thrust is to tap private capital for the expansion and improvement of the industry as the
large government debt and the highly capital-intensive character of the industry itself have long been
acknowledged as the critical constraints to the program. To attract private investment, largely
foreign, the jaded structure of the industry had to be addressed. While the generation and
transmission sectors were centralized and monopolistic, the distribution side was fragmented with
over 130 utilities, mostly small and uneconomic. The pervasive flaws have caused a low utilization of
existing generation capacity; extremely high and uncompetitive power rates; poor quality of service
to consumers; dismal to forgettable performance of the government power sector; high system
losses; and an inability to develop a clear strategy for overcoming these shortcomings.

Thus, the EPIRA provides a framework for the restructuring of the industry, including the privatization
of the assets of the National Power Corporation (NPC), the transition to a competitive structure, and
the delineation of the roles of various government agencies and the private entities. The law ordains
the division of the industry into four (4) distinct sectors, namely: generation, transmission, distribution
and supply.
Corollarily, the NPC generating plants have to privatized and its transmission business spun off and
privatized thereafter.67
Finally, every law has in its favor the presumption of constitutionality, and to justify its nullification,
there must be a clear and unequivocal breach of the Constitution and not one that is doubtful,
speculative, or argumentative.68Indubitably, petitioners failed to overcome this presumption in favor
of the EPIRA. We find no clear violation of the Constitution which would warrant a pronouncement
that Sec. 34 of the EPIRA and Rule 18 of its IRR are unconstitutional and void.
WHEREFORE, the instant case is hereby DISMISSED for lack of merit.
SO ORDERED.

Republic of the Philippines


SUPREME COURT
Manila
EN BANC
G.R. No. 166715

August 14, 2008

ABAKADA GURO PARTY LIST (formerly AASJS)1 OFFICERS/MEMBERS


SAMSON S. ALCANTARA, ED VINCENT S. ALBANO, ROMEO R. ROBISO,
RENE B. GOROSPE and EDWIN R. SANDOVAL, petitioners,
vs.
HON. CESAR V. PURISIMA, in his capacity as Secretary of Finance, HON.
GUILLERMO L. PARAYNO, JR., in his capacity as Commissioner of the
Bureau of Internal Revenue, and HON. ALBERTO D. LINA, in his
Capacity as Commissioner of Bureau of Customs, respondents.
DECISION
CORONA, J.:
This petition for prohibition1 seeks to prevent respondents from implementing
and enforcing Republic Act (RA) 93352 (Attrition Act of 2005).

RA 9335 was enacted to optimize the revenue-generation capability and


collection of the Bureau of Internal Revenue (BIR) and the Bureau of Customs
(BOC). The law intends to encourage BIR and BOC officials and employees to
exceed their revenue targets by providing a system of rewards and sanctions
through the creation of a Rewards and Incentives Fund (Fund) and a Revenue
Performance Evaluation Board (Board).3 It covers all officials and employees
of the BIR and the BOC with at least six months of service, regardless of
employment status.4
The Fund is sourced from the collection of the BIR and the BOC in excess of
their revenue targets for the year, as determined by the Development Budget
and Coordinating Committee (DBCC). Any incentive or reward is taken from
the fund and allocated to the BIR and the BOC in proportion to their
contribution in the excess collection of the targeted amount of tax revenue.5
The Boards in the BIR and the BOC are composed of the Secretary of the
Department of Finance (DOF) or his/her Undersecretary, the Secretary of the
Department of Budget and Management (DBM) or his/her Undersecretary, the
Director General of the National Economic Development Authority (NEDA) or
his/her Deputy Director General, the Commissioners of the BIR and the BOC
or their Deputy Commissioners, two representatives from the rank-and-file
employees and a representative from the officials nominated by their
recognized organization.6
Each Board has the duty to (1) prescribe the rules and guidelines for the
allocation, distribution and release of the Fund; (2) set criteria and procedures
for removing from the service officials and employees whose revenue
collection falls short of the target; (3) terminate personnel in accordance with
the criteria adopted by the Board; (4) prescribe a system for performance
evaluation; (5) perform other functions, including the issuance of rules and
regulations and (6) submit an annual report to Congress.7
The DOF, DBM, NEDA, BIR, BOC and the Civil Service Commission (CSC)
were tasked to promulgate and issue the implementing rules and regulations
of RA 9335,8 to be approved by a Joint Congressional Oversight Committee
created for such purpose.9
Petitioners, invoking their right as taxpayers filed this petition challenging the
constitutionality of RA 9335, a tax reform legislation. They contend that, by
establishing a system of rewards and incentives, the law "transform[s] the
officials and employees of the BIR and the BOC into mercenaries and bounty
hunters" as they will do their best only in consideration of such rewards. Thus,

the system of rewards and incentives invites corruption and undermines the
constitutionally mandated duty of these officials and employees to serve the
people with utmost responsibility, integrity, loyalty and efficiency.
Petitioners also claim that limiting the scope of the system of rewards and
incentives only to officials and employees of the BIR and the BOC violates the
constitutional guarantee of equal protection. There is no valid basis for
classification or distinction as to why such a system should not apply to
officials and employees of all other government agencies.
In addition, petitioners assert that the law unduly delegates the power to fix
revenue targets to the President as it lacks a sufficient standard on that
matter. While Section 7(b) and (c) of RA 9335 provides that BIR and BOC
officials may be dismissed from the service if their revenue collections fall
short of the target by at least 7.5%, the law does not, however, fix the revenue
targets to be achieved. Instead, the fixing of revenue targets has been
delegated to the President without sufficient standards. It will therefore be
easy for the President to fix an unrealistic and unattainable target in order to
dismiss BIR or BOC personnel.
Finally, petitioners assail the creation of a congressional oversight committee
on the ground that it violates the doctrine of separation of powers. While the
legislative function is deemed accomplished and completed upon the
enactment and approval of the law, the creation of the congressional oversight
committee permits legislative participation in the implementation and
enforcement of the law.
In their comment, respondents, through the Office of the Solicitor General,
question the petition for being premature as there is no actual case or
controversy yet. Petitioners have not asserted any right or claim that will
necessitate the exercise of this Courts jurisdiction. Nevertheless, respondents
acknowledge that public policy requires the resolution of the constitutional
issues involved in this case. They assert that the allegation that the reward
system will breed mercenaries is mere speculation and does not suffice to
invalidate the law. Seen in conjunction with the declared objective of RA 9335,
the law validly classifies the BIR and the BOC because the functions they
perform are distinct from those of the other government agencies and
instrumentalities. Moreover, the law provides a sufficient standard that will
guide the executive in the implementation of its provisions. Lastly, the creation
of the congressional oversight committee under the law enhances, rather than
violates, separation of powers. It ensures the fulfillment of the legislative policy

and serves as a check to any over-accumulation of power on the part of the


executive and the implementing agencies.
After a careful consideration of the conflicting contentions of the parties, the
Court finds that petitioners have failed to overcome the presumption of
constitutionality in favor of RA 9335, except as shall hereafter be discussed.
Actual Case And Ripeness
An actual case or controversy involves a conflict of legal rights, an assertion of
opposite legal claims susceptible of judicial adjudication.10 A closely related
requirement is ripeness, that is, the question must be ripe for adjudication.
And a constitutional question is ripe for adjudication when the governmental
act being challenged has a direct adverse effect on the individual challenging
it.11Thus, to be ripe for judicial adjudication, the petitioner must show a
personal stake in the outcome of the case or an injury to himself that can be
redressed by a favorable decision of the Court.12
In this case, aside from the general claim that the dispute has ripened into a
judicial controversy by the mere enactment of the law even without any further
overt act,13 petitioners fail either to assert any specific and concrete legal claim
or to demonstrate any direct adverse effect of the law on them. They are
unable to show a personal stake in the outcome of this case or an injury to
themselves. On this account, their petition is procedurally infirm.
This notwithstanding, public interest requires the resolution of the
constitutional issues raised by petitioners. The grave nature of their
allegations tends to cast a cloud on the presumption of constitutionality in
favor of the law. And where an action of the legislative branch is alleged to
have infringed the Constitution, it becomes not only the right but in fact the
duty of the judiciary to settle the dispute.14
Accountability of
Public Officers
Section 1, Article 11 of the Constitution states:
Sec. 1. Public office is a public trust. Public officers and employees must
at all times be accountable to the people, serve them with utmost
responsibility, integrity, loyalty, and efficiency, act with patriotism, and
justice, and lead modest lives.

Public office is a public trust. It must be discharged by its holder not for his
own personal gain but for the benefit of the public for whom he holds it in trust.
By demanding accountability and service with responsibility, integrity, loyalty,
efficiency, patriotism and justice, all government officials and employees have
the duty to be responsive to the needs of the people they are called upon to
serve.
Public officers enjoy the presumption of regularity in the performance of their
duties. This presumption necessarily obtains in favor of BIR and BOC officials
and employees. RA 9335 operates on the basis thereof and reinforces it by
providing a system of rewards and sanctions for the purpose of encouraging
the officials and employees of the BIR and the BOC to exceed their revenue
targets and optimize their revenue-generation capability and collection.15
The presumption is disputable but proof to the contrary is required to rebut it.
It cannot be overturned by mere conjecture or denied in advance (as
petitioners would have the Court do) specially in this case where it is an
underlying principle to advance a declared public policy.
Petitioners claim that the implementation of RA 9335 will turn BIR and BOC
officials and employees into "bounty hunters and mercenaries" is not only
without any factual and legal basis; it is also purely speculative.
A law enacted by Congress enjoys the strong presumption of constitutionality.
To justify its nullification, there must be a clear and unequivocal breach of the
Constitution, not a doubtful and equivocal one.16 To invalidate RA 9335 based
on petitioners baseless supposition is an affront to the wisdom not only of the
legislature that passed it but also of the executive which approved it.
Public service is its own reward. Nevertheless, public officers may by law be
rewarded for exemplary and exceptional performance. A system of incentives
for exceeding the set expectations of a public office is not anathema to the
concept of public accountability. In fact, it recognizes and reinforces
dedication to duty, industry, efficiency and loyalty to public service of
deserving government personnel.
In United States v. Matthews,17 the U.S. Supreme Court validated a law which
awards to officers of the customs as well as other parties an amount not
exceeding one-half of the net proceeds of forfeitures in violation of the laws
against smuggling. Citing Dorsheimer v. United States,18 the U.S. Supreme
Court said:

The offer of a portion of such penalties to the collectors is to stimulate


and reward their zeal and industry in detecting fraudulent attempts to
evade payment of duties and taxes.
In the same vein, employees of the BIR and the BOC may by law be entitled
to a reward when, as a consequence of their zeal in the enforcement of tax
and customs laws, they exceed their revenue targets. In addition, RA 9335
establishes safeguards to ensure that the reward will not be claimed if it will
be either the fruit of "bounty hunting or mercenary activity" or the product of
the irregular performance of official duties. One of these precautionary
measures is embodied in Section 8 of the law:
SEC. 8. Liability of Officials, Examiners and Employees of the BIR and
the BOC. The officials, examiners, and employees of the [BIR] and the
[BOC] who violate this Act or who are guilty of negligence, abuses or
acts of malfeasance or misfeasance or fail to exercise extraordinary
diligence in the performance of their duties shall be held liable for any
loss or injury suffered by any business establishment or taxpayer as a
result of such violation, negligence, abuse, malfeasance, misfeasance
or failure to exercise extraordinary diligence.
Equal Protection
Equality guaranteed under the equal protection clause is equality under the
same conditions and among persons similarly situated; it is equality among
equals, not similarity of treatment of persons who are classified based on
substantial differences in relation to the object to be accomplished.19When
things or persons are different in fact or circumstance, they may be treated in
law differently. InVictoriano v. Elizalde Rope Workers Union,20 this Court
declared:
The guaranty of equal protection of the laws is not a guaranty of equality
in the application of the laws upon all citizens of the [S]tate. It is not,
therefore, a requirement, in order to avoid the constitutional prohibition
against inequality, that every man, woman and child should be affected
alike by a statute. Equality of operation of statutes does not mean
indiscriminate operation on persons merely as such, but on persons
according to the circumstances surrounding them. It guarantees
equality, not identity of rights. The Constitution does not require that
things which are different in fact be treated in law as though they
were the same. The equal protection clause does not forbid
discrimination as to things that are different. It does not prohibit

legislation which is limited either in the object to which it is


directed or by the territory within which it is to operate.
The equal protection of the laws clause of the Constitution allows
classification. Classification in law, as in the other departments of
knowledge or practice, is the grouping of things in speculation or
practice because they agree with one another in certain particulars. A
law is not invalid because of simple inequality. The very idea of
classification is that of inequality, so that it goes without saying that the
mere fact of inequality in no manner determines the matter of
constitutionality. All that is required of a valid classification is that it
be reasonable, which means that the classification should be
based on substantial distinctions which make for real differences,
that it must be germane to the purpose of the law; that it must not
be limited to existing conditions only; and that it must apply
equally to each member of the class. This Court has held that the
standard is satisfied if the classification or distinction is based on
a reasonable foundation or rational basis and is not palpably
arbitrary.
In the exercise of its power to make classifications for the purpose of
enacting laws over matters within its jurisdiction, the state is recognized
as enjoying a wide range of discretion. It is not necessary that the
classification be based on scientific or marked differences of things or in
their relation. Neither is it necessary that the classification be made with
mathematical nicety. Hence, legislative classification may in many cases
properly rest on narrow distinctions, for the equal protection guaranty
does not preclude the legislature from recognizing degrees of evil or
harm, and legislation is addressed to evils as they may
appear.21 (emphasis supplied)
The equal protection clause recognizes a valid classification, that is, a
classification that has a reasonable foundation or rational basis and not
arbitrary.22 With respect to RA 9335, its expressed public policy is the
optimization of the revenue-generation capability and collection of the BIR and
the BOC.23 Since the subject of the law is the revenue- generation capability
and collection of the BIR and the BOC, the incentives and/or sanctions
provided in the law should logically pertain to the said agencies. Moreover, the
law concerns only the BIR and the BOC because they have the common
distinct primary function of generating revenues for the national government
through the collection of taxes, customs duties, fees and charges.

The BIR performs the following functions:


Sec. 18. The Bureau of Internal Revenue. The Bureau of Internal
Revenue, which shall be headed by and subject to the supervision and
control of the Commissioner of Internal Revenue, who shall be
appointed by the President upon the recommendation of the Secretary
[of the DOF], shall have the following functions:
(1) Assess and collect all taxes, fees and charges and account for
all revenues collected;
(2) Exercise duly delegated police powers for the proper performance of
its functions and duties;
(3) Prevent and prosecute tax evasions and all other illegal economic
activities;
(4) Exercise supervision and control over its constituent and subordinate
units; and
(5) Perform such other functions as may be provided by law.24
xxx

xxx

xxx (emphasis supplied)

On the other hand, the BOC has the following functions:


Sec. 23. The Bureau of Customs. The Bureau of Customs which shall
be headed and subject to the management and control of the
Commissioner of Customs, who shall be appointed by the President
upon the recommendation of the Secretary[of the DOF] and hereinafter
referred to as Commissioner, shall have the following functions:
(1) Collect custom duties, taxes and the corresponding fees,
charges and penalties;
(2) Account for all customs revenues collected;
(3) Exercise police authority for the enforcement of tariff and customs
laws;
(4) Prevent and suppress smuggling, pilferage and all other economic
frauds within all ports of entry;

(5) Supervise and control exports, imports, foreign mails and the
clearance of vessels and aircrafts in all ports of entry;
(6) Administer all legal requirements that are appropriate;
(7) Prevent and prosecute smuggling and other illegal activities in all
ports under its jurisdiction;
(8) Exercise supervision and control over its constituent units;
(9) Perform such other functions as may be provided by law.25
xxx

xxx

xxx (emphasis supplied)

Both the BIR and the BOC are bureaus under the DOF. They principally
perform the special function of being the instrumentalities through which the
State exercises one of its great inherent functions taxation. Indubitably, such
substantial distinction is germane and intimately related to the purpose of the
law. Hence, the classification and treatment accorded to the BIR and the BOC
under RA 9335 fully satisfy the demands of equal protection.
Undue Delegation
Two tests determine the validity of delegation of legislative power: (1) the
completeness test and (2) the sufficient standard test. A law is complete when
it sets forth therein the policy to be executed, carried out or implemented by
the delegate.26 It lays down a sufficient standard when it provides adequate
guidelines or limitations in the law to map out the boundaries of the delegates
authority and prevent the delegation from running riot.27 To be sufficient, the
standard must specify the limits of the delegates authority, announce the
legislative policy and identify the conditions under which it is to be
implemented.28
RA 9335 adequately states the policy and standards to guide the President in
fixing revenue targets and the implementing agencies in carrying out the
provisions of the law. Section 2 spells out the policy of the law:
SEC. 2. Declaration of Policy. It is the policy of the State to optimize
the revenue-generation capability and collection of the Bureau of
Internal Revenue (BIR) and the Bureau of Customs (BOC) by providing
for a system of rewards and sanctions through the creation of a
Rewards and Incentives Fund and a Revenue Performance Evaluation

Board in the above agencies for the purpose of encouraging their


officials and employees to exceed their revenue targets.
Section 4 "canalized within banks that keep it from overflowing"29 the
delegated power to the President to fix revenue targets:
SEC. 4. Rewards and Incentives Fund. A Rewards and Incentives
Fund, hereinafter referred to as the Fund, is hereby created, to be
sourced from the collection of the BIR and the BOC in excess of their
respective revenue targets of the year, as determined by the
Development Budget and Coordinating Committee (DBCC), in the
following percentages:
Excess of Collection of the
Excess the Revenue Targets

Percent (%) of the Excess Collection to


Accrue to the Fund

30% or below

15%

More than 30%

15% of the first 30% plus 20% of the


remaining excess

The Fund shall be deemed automatically appropriated the year


immediately following the year when the revenue collection target was
exceeded and shall be released on the same fiscal year.
Revenue targets shall refer to the original estimated revenue
collection expected of the BIR and the BOC for a given fiscal year
as stated in the Budget of Expenditures and Sources of Financing
(BESF) submitted by the President to Congress. The BIR and the
BOC shall submit to the DBCC the distribution of the agencies revenue
targets as allocated among its revenue districts in the case of the BIR,
and the collection districts in the case of the BOC.
xxx

xxx

xxx (emphasis supplied)

Revenue targets are based on the original estimated revenue collection


expected respectively of the BIR and the BOC for a given fiscal year as
approved by the DBCC and stated in the BESF submitted by the President to

Congress.30 Thus, the determination of revenue targets does not rest solely on
the President as it also undergoes the scrutiny of the DBCC.
On the other hand, Section 7 specifies the limits of the Boards authority and
identifies the conditions under which officials and employees whose revenue
collection falls short of the target by at least 7.5% may be removed from the
service:
SEC. 7. Powers and Functions of the Board. The Board in the agency
shall have the following powers and functions:
xxx

xxx

xxx

(b) To set the criteria and procedures for removing from service
officials and employees whose revenue collection falls short of the
target by at least seven and a half percent (7.5%), with due
consideration of all relevant factors affecting the level of
collection as provided in the rules and regulations promulgated under
this Act, subject to civil service laws, rules and regulations and
compliance with substantive and procedural due process:
Provided, That the following exemptions shall apply:
1. Where the district or area of responsibility is newly-created, not
exceeding two years in operation, as has no historical record of
collection performance that can be used as basis for evaluation;
and
2. Where the revenue or customs official or employee is a recent
transferee in the middle of the period under consideration unless
the transfer was due to nonperformance of revenue targets or
potential nonperformance of revenue targets: Provided, however,
That when the district or area of responsibility covered by revenue
or customs officials or employees has suffered from economic
difficulties brought about by natural calamities orforce majeure or
economic causes as may be determined by the Board,
termination shall be considered only after careful and proper
review by the Board.
(c) To terminate personnel in accordance with the criteria adopted in the
preceding paragraph: Provided, That such decision shall be immediately
executory: Provided, further, That the application of the criteria for
the separation of an official or employee from service under this

Act shall be without prejudice to the application of other relevant


laws on accountability of public officers and employees, such as
the Code of Conduct and Ethical Standards of Public Officers and
Employees and the Anti-Graft and Corrupt Practices Act;
xxx

xxx

xxx (emphasis supplied)

Clearly, RA 9335 in no way violates the security of tenure of officials and


employees of the BIR and the BOC. The guarantee of security of tenure only
means that an employee cannot be dismissed from the service for causes
other than those provided by law and only after due process is accorded the
employee.31 In the case of RA 9335, it lays down a reasonable yardstick for
removal (when the revenue collection falls short of the target by at least 7.5%)
with due consideration of all relevant factors affecting the level of collection.
This standard is analogous to inefficiency and incompetence in the
performance of official duties, a ground for disciplinary action under civil
service laws.32 The action for removal is also subject to civil service laws, rules
and regulations and compliance with substantive and procedural due process.
At any rate, this Court has recognized the following as sufficient standards:
"public interest," "justice and equity," "public convenience and welfare" and
"simplicity, economy and welfare."33 In this case, the declared policy of
optimization of the revenue-generation capability and collection of the BIR and
the BOC is infused with public interest.
Separation Of Powers
Section 12 of RA 9335 provides:
SEC. 12. Joint Congressional Oversight Committee. There is hereby
created a Joint Congressional Oversight Committee composed of seven
Members from the Senate and seven Members from the House of
Representatives. The Members from the Senate shall be appointed by
the Senate President, with at least two senators representing the
minority. The Members from the House of Representatives shall be
appointed by the Speaker with at least two members representing the
minority. After the Oversight Committee will have approved the
implementing rules and regulations (IRR) it shall thereafter
become functus officio and therefore cease to exist.
The Joint Congressional Oversight Committee in RA 9335 was created for the
purpose of approving the implementing rules and regulations (IRR) formulated

by the DOF, DBM, NEDA, BIR, BOC and CSC. On May 22, 2006, it approved
the said IRR. From then on, it became functus officio and ceased to exist.
Hence, the issue of its alleged encroachment on the executive function of
implementing and enforcing the law may be considered moot and academic.
This notwithstanding, this might be as good a time as any for the Court to
confront the issue of the constitutionality of the Joint Congressional Oversight
Committee created under RA 9335 (or other similar laws for that matter).
The scholarly discourse of Mr. Justice (now Chief Justice) Puno on the
concept of congressional oversight in Macalintal v. Commission on
Elections34 is illuminating:
Concept and bases of congressional oversight
Broadly defined, the power of oversight embraces all activities
undertaken by Congress to enhance its understanding of and
influence over the implementation of legislation it has enacted.
Clearly, oversight concerns post-enactment measures undertaken
by Congress: (a) to monitor bureaucratic compliance with program
objectives, (b) to determine whether agencies are properly
administered, (c) to eliminate executive waste and dishonesty, (d)
to prevent executive usurpation of legislative authority, and (d) to
assess executive conformity with the congressional perception of
public interest.
The power of oversight has been held to be intrinsic in the grant of
legislative power itself and integral to the checks and balances inherent
in a democratic system of government. x x x x x x x x x
Over the years, Congress has invoked its oversight power with
increased frequency to check the perceived "exponential accumulation
of power" by the executive branch. By the beginning of the 20th century,
Congress has delegated an enormous amount of legislative authority to
the executive branch and the administrative agencies. Congress, thus,
uses its oversight power to make sure that the administrative agencies
perform their functions within the authority delegated to them. x x x x x x
xxx
Categories of congressional oversight functions

The acts done by Congress purportedly in the exercise of its oversight


powers may be divided into three categories,
namely: scrutiny, investigation and supervision.
a. Scrutiny
Congressional scrutiny implies a lesser intensity and continuity of
attention to administrative operations. Its primary purpose is to
determine economy and efficiency of the operation of government
activities. In the exercise of legislative scrutiny, Congress may
request information and report from the other branches of
government. It can give recommendations or pass resolutions for
consideration of the agency involved.
xxx

xxx

xxx

b. Congressional investigation
While congressional scrutiny is regarded as a passive process of
looking at the facts that are readily available, congressional
investigation involves a more intense digging of facts. The power
of Congress to conduct investigation is recognized by the 1987
Constitution under section 21, Article VI, xxx
xxx
xxx
c. Legislative supervision
The third and most encompassing form by which Congress exercises its
oversight power is thru legislative supervision. "Supervision" connotes a
continuing and informed awareness on the part of a congressional
committee regarding executive operations in a given administrative
area. While both congressional scrutiny and investigation involve inquiry
into past executive branch actions in order to influence future executive
branch performance, congressional supervision allows Congress to
scrutinize the exercise of delegated law-making authority, and permits
Congress to retain part of that delegated authority.
Congress exercises supervision over the executive agencies through its
veto power. It typically utilizes veto provisions when granting the
President or an executive agency the power to promulgate regulations
with the force of law. These provisions require the President or an
agency to present the proposed regulations to Congress, which retains
a "right" to approve or disapprove any regulation before it takes

effect. Such legislative veto provisions usually provide that a proposed


regulation will become a law after the expiration of a certain period of
time, only if Congress does not affirmatively disapprove of the regulation
in the meantime. Less frequently, the statute provides that a proposed
regulation will become law if Congress affirmatively approves it.
Supporters of legislative veto stress that it is necessary to maintain the
balance of power between the legislative and the executive branches of
government as it offers lawmakers a way to delegate vast power to the
executive branch or to independent agencies while retaining the option
to cancel particular exercise of such power without having to pass new
legislation or to repeal existing law. They contend that this arrangement
promotes democratic accountability as it provides legislative check on
the activities of unelected administrative agencies. One proponent thus
explains:
It is too late to debate the merits of this delegation policy: the
policy is too deeply embedded in our law and practice. It suffices
to say that the complexities of modern government have often led
Congress-whether by actual or perceived necessity- to legislate
by declaring broad policy goals and general statutory standards,
leaving the choice of policy options to the discretion of an
executive officer. Congress articulates legislative aims, but leaves
their implementation to the judgment of parties who may or may
not have participated in or agreed with the development of those
aims. Consequently, absent safeguards, in many instances the
reverse of our constitutional scheme could be effected: Congress
proposes, the Executive disposes. One safeguard, of course, is
the legislative power to enact new legislation or to change existing
law. But without some means of overseeing post enactment
activities of the executive branch, Congress would be unable to
determine whether its policies have been implemented in
accordance with legislative intent and thus whether legislative
intervention is appropriate.
Its opponents, however, criticize the legislative veto as undue
encroachment upon the executive prerogatives. They urge that any
post-enactment measures undertaken by the legislative branch
should be limited to scrutiny and investigation; any measure
beyond that would undermine the separation of powers
guaranteed by the Constitution. They contend that legislative veto

constitutes an impermissible evasion of the Presidents veto authority


and intrusion into the powers vested in the executive or judicial
branches of government. Proponents counter that legislative veto
enhances separation of powers as it prevents the executive branch and
independent agencies from accumulating too much power. They submit
that reporting requirements and congressional committee investigations
allow Congress to scrutinize only the exercise of delegated law-making
authority. They do not allow Congress to review executive proposals
before they take effect and they do not afford the opportunity for
ongoing and binding expressions of congressional intent. In contrast,
legislative veto permits Congress to participate prospectively in the
approval or disapproval of "subordinate law" or those enacted by the
executive branch pursuant to a delegation of authority by Congress.
They further argue that legislative veto "is a necessary response by
Congress to the accretion of policy control by forces outside its
chambers." In an era of delegated authority, they point out that
legislative veto "is the most efficient means Congress has yet devised to
retain control over the evolution and implementation of its policy as
declared by statute."
In Immigration and Naturalization Service v. Chadha, the U.S. Supreme
Court resolved the validity of legislative veto provisions. The case
arose from the order of the immigration judge suspending the
deportation of Chadha pursuant to 244(c)(1) of the Immigration and
Nationality Act. The United States House of Representatives passed a
resolution vetoing the suspension pursuant to 244(c)(2) authorizing
either House of Congress, by resolution, to invalidate the decision of the
executive branch to allow a particular deportable alien to remain in the
United States. The immigration judge reopened the deportation
proceedings to implement the House order and the alien was ordered
deported. The Board of Immigration Appeals dismissed the aliens
appeal, holding that it had no power to declare unconstitutional an act of
Congress. The United States Court of Appeals for Ninth Circuit held that
the House was without constitutional authority to order the aliens
deportation and that 244(c)(2) violated the constitutional doctrine on
separation of powers.
On appeal, the U.S. Supreme Court declared 244(c)(2)
unconstitutional. But the Court shied away from the issue of
separation of powers and instead held that the provision violates the
presentment clause and bicameralism. It held that the one-house veto

was essentially legislative in purpose and effect. As such, it is subject to


the procedures set out in Article I of the Constitution requiring the
passage by a majority of both Houses and presentment to the
President. x x x x x x x x x
Two weeks after the Chadha decision, the Court upheld, in
memorandum decision, two lower court decisions invalidating the
legislative veto provisions in the Natural Gas Policy Act of 1978 and the
Federal Trade Commission Improvement Act of 1980. Following this
precedence, lower courts invalidated statutes containing legislative veto
provisions although some of these provisions required the approval of
both Houses of Congress and thus met the bicameralism requirement of
Article I. Indeed, some of these veto provisions were not even
exercised.35(emphasis supplied)
In Macalintal, given the concept and configuration of the power of
congressional oversight and considering the nature and powers of a
constitutional body like the Commission on Elections, the Court struck down
the provision in RA 9189 (The Overseas Absentee Voting Act of 2003)
creating a Joint Congressional Committee. The committee was tasked not
only to monitor and evaluate the implementation of the said law but also to
review, revise, amend and approve the IRR promulgated by the Commission
on Elections. The Court held that these functions infringed on the
constitutional independence of the Commission on Elections.36
With this backdrop, it is clear that congressional oversight is not
unconstitutional per se, meaning, it neither necessarily constitutes an
encroachment on the executive power to implement laws nor undermines the
constitutional separation of powers. Rather, it is integral to the checks and
balances inherent in a democratic system of government. It may in fact even
enhance the separation of powers as it prevents the over-accumulation of
power in the executive branch.
However, to forestall the danger of congressional encroachment "beyond the
legislative sphere," the Constitution imposes two basic and related constraints
on Congress.37 It may not vest itself, any of its committees or its members with
either executive or judicial power.38 And, when it exercises its legislative
power, it must follow the "single, finely wrought and exhaustively considered,
procedures" specified under the Constitution,39 including the procedure for
enactment of laws and presentment.

Thus, any post-enactment congressional measure such as this should be


limited to scrutiny and investigation. In particular, congressional oversight
must be confined to the following:
(1) scrutiny based primarily on Congress power of appropriation and the
budget hearings conducted in connection with it, its power to ask heads
of departments to appear before and be heard by either of its Houses
on any matter pertaining to their departments and its power of
confirmation40 and
(2) investigation and monitoring41 of the implementation of laws pursuant
to the power of Congress to conduct inquiries in aid of legislation.42
Any action or step beyond that will undermine the separation of powers
guaranteed by the Constitution. Legislative vetoes fall in this class.
Legislative veto is a statutory provision requiring the President or an
administrative agency to present the proposed implementing rules and
regulations of a law to Congress which, by itself or through a committee
formed by it, retains a "right" or "power" to approve or disapprove such
regulations before they take effect. As such, a legislative veto in the form of a
congressional oversight committee is in the form of an inward-turning
delegation designed to attach a congressional leash (other than through
scrutiny and investigation) to an agency to which Congress has by law initially
delegated broad powers.43 It radically changes the design or structure of the
Constitutions diagram of power as it entrusts to Congress a direct role in
enforcing, applying or implementing its own laws.44
Congress has two options when enacting legislation to define national policy
within the broad horizons of its legislative competence.45 It can itself formulate
the details or it can assign to the executive branch the responsibility for
making necessary managerial decisions in conformity with those
standards.46 In the latter case, the law must be complete in all its essential
terms and conditions when it leaves the hands of the legislature.47 Thus, what
is left for the executive branch or the concerned administrative agency when it
formulates rules and regulations implementing the law is to fill up details
(supplementary rule-making) or ascertain facts necessary to bring the law into
actual operation (contingent rule-making).48
Administrative regulations enacted by administrative agencies to implement
and interpret the law which they are entrusted to enforce have the force of law
and are entitled to respect.49 Such rules and regulations partake of the nature

of a statute50 and are just as binding as if they have been written in the statute
itself. As such, they have the force and effect of law and enjoy the
presumption of constitutionality and legality until they are set aside with finality
in an appropriate case by a competent court.51 Congress, in the guise of
assuming the role of an overseer, may not pass upon their legality by
subjecting them to its stamp of approval without disturbing the calculated
balance of powers established by the Constitution. In exercising discretion to
approve or disapprove the IRR based on a determination of whether or not
they conformed with the provisions of RA 9335, Congress arrogated judicial
power unto itself, a power exclusively vested in this Court by the Constitution.
Considered Opinion of
Mr. Justice Dante O. Tinga
Moreover, the requirement that the implementing rules of a law be subjected
to approval by Congress as a condition for their effectivity violates the cardinal
constitutional principles of bicameralism and the rule on presentment.52
Section 1, Article VI of the Constitution states:
Section 1. The legislative power shall be vested in the Congress of
the Philippines which shall consist of a Senate and a House of
Representatives, except to the extent reserved to the people by the
provision on initiative and referendum. (emphasis supplied)
Legislative power (or the power to propose, enact, amend and repeal
laws)53 is vested in Congress which consists of two chambers, the Senate and
the House of Representatives. A valid exercise of legislative power requires
the act of both chambers. Corrollarily, it can be exercised neither solely by one
of the two chambers nor by a committee of either or both chambers. Thus,
assuming the validity of a legislative veto, both a single-chamber legislative
veto and a congressional committee legislative veto are invalid.
Additionally, Section 27(1), Article VI of the Constitution provides:
Section 27. (1) Every bill passed by the Congress shall, before it
becomes a law, be presented to the President. If he approves the
same, he shall sign it, otherwise, he shall veto it and return the same
with his objections to the House where it originated, which shall enter
the objections at large in its Journal and proceed to reconsider it. If,
after such reconsideration, two-thirds of all the Members of such House
shall agree to pass the bill, it shall be sent, together with the objections,

to the other House by which it shall likewise be reconsidered, and if


approved by two-thirds of all the Members of that House, it shall
become a law. In all such cases, the votes of each House shall be
determined by yeas or nays, and the names of the members voting for
or against shall be entered in its Journal. The President shall
communicate his veto of any bill to the House where it originated within
thirty days after the date of receipt thereof; otherwise, it shall become a
law as if he had signed it. (emphasis supplied)
Every bill passed by Congress must be presented to the President for
approval or veto. In the absence of presentment to the President, no bill
passed by Congress can become a law. In this sense, law-making under the
Constitution is a joint act of the Legislature and of the Executive. Assuming
that legislative veto is a valid legislative act with the force of law, it cannot take
effect without such presentment even if approved by both chambers of
Congress.
In sum, two steps are required before a bill becomes a law. First, it must be
approved by both Houses of Congress.54 Second, it must be presented to and
approved by the President.55 As summarized by Justice Isagani Cruz56 and Fr.
Joaquin G. Bernas, S.J.57, the following is the procedure for the approval of
bills:
A bill is introduced by any member of the House of Representatives or
the Senate except for some measures that must originate only in the
former chamber.
The first reading involves only a reading of the number and title of the
measure and its referral by the Senate President or the Speaker to the
proper committee for study.
The bill may be "killed" in the committee or it may be recommended for
approval, with or without amendments, sometimes after public hearings
are first held thereon. If there are other bills of the same nature or
purpose, they may all be consolidated into one bill under common
authorship or as a committee bill.
Once reported out, the bill shall be calendared for second reading. It is
at this stage that the bill is read in its entirety, scrutinized, debated upon
and amended when desired. The second reading is the most important
stage in the passage of a bill.

The bill as approved on second reading is printed in its final form and
copies thereof are distributed at least three days before the third
reading. On the third reading, the members merely register their votes
and explain them if they are allowed by the rules. No further debate is
allowed.
Once the bill passes third reading, it is sent to the other chamber, where
it will also undergo the three readings. If there are differences between
the versions approved by the two chambers, a conference
committee58 representing both Houses will draft a compromise measure
that if ratified by the Senate and the House of Representatives will then
be submitted to the President for his consideration.
The bill is enrolled when printed as finally approved by the Congress,
thereafter authenticated with the signatures of the Senate President, the
Speaker, and the Secretaries of their respective chambers59
The Presidents role in law-making.
The final step is submission to the President for approval. Once
approved, it takes effect as law after the required publication.60
Where Congress delegates the formulation of rules to implement the law it
has enacted pursuant to sufficient standards established in the said law, the
law must be complete in all its essential terms and conditions when it leaves
the hands of the legislature. And it may be deemed to have left the hands of
the legislature when it becomes effective because it is only upon effectivity of
the statute that legal rights and obligations become available to those entitled
by the language of the statute. Subject to the indispensable requisite of
publication under the due process clause,61 the determination as to when a
law takes effect is wholly the prerogative of Congress.62 As such, it is only
upon its effectivity that a law may be executed and the executive branch
acquires the duties and powers to execute the said law. Before that point, the
role of the executive branch, particularly of the President, is limited to
approving or vetoing the law.63
From the moment the law becomes effective, any provision of law that
empowers Congress or any of its members to play any role in the
implementation or enforcement of the law violates the principle of separation
of powers and is thus unconstitutional. Under this principle, a provision that
requires Congress or its members to approve the implementing rules of a law
after it has already taken effect shall be unconstitutional, as is a provision that

allows Congress or its members to overturn any directive or ruling made by


the members of the executive branch charged with the implementation of the
law.
Following this rationale, Section 12 of RA 9335 should be struck down as
unconstitutional. While there may be similar provisions of other laws that may
be invalidated for failure to pass this standard, the Court refrains from
invalidating them wholesale but will do so at the proper time when an
appropriate case assailing those provisions is brought before us.64
The next question to be resolved is: what is the effect of the unconstitutionality
of Section 12 of RA 9335 on the other provisions of the law? Will it render the
entire law unconstitutional? No.
Section 13 of RA 9335 provides:
SEC. 13. Separability Clause. If any provision of this Act is declared
invalid by a competent court, the remainder of this Act or any provision
not affected by such declaration of invalidity shall remain in force and
effect.
In Tatad v. Secretary of the Department of Energy,65 the Court laid down the
following rules:
The general rule is that where part of a statute is void as repugnant to
the Constitution, while another part is valid, the valid portion, if
separable from the invalid, may stand and be enforced. The presence of
a separability clause in a statute creates the presumption that the
legislature intended separability, rather than complete nullity of the
statute. To justify this result, the valid portion must be so far independent
of the invalid portion that it is fair to presume that the legislature would
have enacted it by itself if it had supposed that it could not
constitutionally enact the other. Enough must remain to make a
complete, intelligible and valid statute, which carries out the legislative
intent. x x x
The exception to the general rule is that when the parts of a statute are
so mutually dependent and connected, as conditions, considerations,
inducements, or compensations for each other, as to warrant a belief
that the legislature intended them as a whole, the nullity of one part will
vitiate the rest. In making the parts of the statute dependent, conditional,
or connected with one another, the legislature intended the statute to be

carried out as a whole and would not have enacted it if one part is void,
in which case if some parts are unconstitutional, all the other provisions
thus dependent, conditional, or connected must fall with them.
The separability clause of RA 9335 reveals the intention of the legislature to
isolate and detach any invalid provision from the other provisions so that the
latter may continue in force and effect. The valid portions can stand
independently of the invalid section. Without Section 12, the remaining
provisions still constitute a complete, intelligible and valid law which carries
out the legislative intent to optimize the revenue-generation capability and
collection of the BIR and the BOC by providing for a system of rewards and
sanctions through the Rewards and Incentives Fund and a Revenue
Performance Evaluation Board.
To be effective, administrative rules and regulations must be published in full if
their purpose is to enforce or implement existing law pursuant to a valid
delegation. The IRR of RA 9335 were published on May 30, 2006 in two
newspapers of general circulation66 and became effective 15 days
thereafter.67 Until and unless the contrary is shown, the IRR are presumed
valid and effective even without the approval of the Joint Congressional
Oversight Committee.
WHEREFORE, the petition is hereby PARTIALLY GRANTED. Section 12 of
RA 9335 creating a Joint Congressional Oversight Committee to approve the
implementing rules and regulations of the law is
declared UNCONSTITUTIONAL and therefore NULL and VOID. The
constitutionality of the remaining provisions of RA 9335 is UPHELD. Pursuant
to Section 13 of RA 9335, the rest of the provisions remain in force and effect.
SO ORDERED.
Republic of the Philippines
SUPREME COURT
Manila
EN BANC
G.R. No. 164171

February 20, 2006

HON. EXECUTIVE SECRETARY, HON. SECRETARY OF THE DEPARTMENT OF


TRANSPORTATION AND COMMUNICATIONS (DOTC), COMMISSIONER OF CUSTOMS,
ASSISTANT SECRETARY, LAND TRANSPORTATION OFFICE (LTO), COLLECTOR OF
CUSTOMS, SUBIC BAY FREE PORT ZONE, AND CHIEF OF LTO, SUBIC BAY FREE PORT

ZONE, Petitioners,
vs.
SOUTHWING HEAVY INDUSTRIES, INC., represented by its President JOSE T. DIZON, UNITED
AUCTIONEERS, INC., represented by its President DOMINIC SYTIN, and MICROVAN, INC.,
represented by its President MARIANO C. SONON, Respondents.
x---------------x
G.R. No. 164172

February 20, 2006

HON. EXECUTIVE SECRETARY, SECRETARY OF THE DEPARTMENT OF TRANSPORTATION


AND COMMUNICATION (DOTC), COMMISSIONER OF CUSTOMS, ASSISTANT SECRETARY,
LAND TRANSPORTATION OFFICE (LTO), COLLECTOR OF CUSTOMS, SUBIC BAY FREE
PORT ZONE AND CHIEF OF LTO, SUBIC BAY FREE PORT ZONE, Petitioners,
vs.
SUBIC INTEGRATED MACRO VENTURES CORP., represented by its President YOLANDA
AMBAR,Respondent.
x---------------x
G.R. No. 168741

February 20, 2006

HON. EXECUTIVE SECRETARY, HON. SECRETARY OF FINANCE, THE CHIEF OF THE LAND
TRANSPORTATION OFFICE, THE COMMISSIONER OF CUSTOMS, and THE COLLECTOR OF
CUSTOMS, SUBIC SPECIAL ECONOMIC ZONE, Petitioners,
vs.
MOTOR VEHICLE IMPORTERS ASSOCIATION OF SUBIC BAY FREEPORT, INC., represented
by its President ALFREDO S. GALANG, Respondent.
DECISION
YNARES-SANTIAGO, J.:
The instant consolidated petitions seek to annul and set aside the Decisions of the Regional Trial
Court of Olongapo City, Branch 72, in Civil Case No. 20-0-04 and Civil Case No. 22-0-04, both dated
May 24, 2004; and the February 14, 2005 Decision of the Court of Appeals in CA-G.R. SP. No.
83284, which declared Article 2, Section 3.1 of Executive Order No. 156 (EO 156) unconstitutional.
Said executive issuance prohibits the importation into the country, inclusive of the Special Economic
and Freeport Zone or the Subic Bay Freeport (SBF or Freeport), of used motor vehicles, subject to a
few exceptions.
The undisputed facts show that on December 12, 2002, President Gloria Macapagal-Arroyo, through
Executive Secretary Alberto G. Romulo, issued EO 156, entitled "Providing for a comprehensive
industrial policy and directions for the motor vehicle development program and its implementing
guidelines." The challenged provision states:

3.1 The importation into the country, inclusive of the Freeport, of all types of used
motor vehicles is prohibited, except for the following:
3.1.1 A vehicle that is owned and for the personal use of a returning resident or
immigrant and covered by an authority to import issued under the No-dollar
Importation Program. Such vehicles cannot be resold for at least three (3) years;
3.1.2 A vehicle for the use of an official of the Diplomatic Corps and authorized to be
imported by the Department of Foreign Affairs;
3.1.3 Trucks excluding pickup trucks;
1. with GVW of 2.5-6.0 tons covered by an authority to import issued by the
DTI.
2. With GVW above 6.0 tons.
3.1.4 Buses:
1. with GVW of 6-12 tons covered by an authority to import issued by DTI;
2. with GVW above 12 tons.
3.1.5 Special purpose vehicles:
1. fire trucks
2. ambulances
3. funeral hearse/coaches
4. crane lorries
5. tractor heads and truck tractors
6. boom trucks
7. tanker trucks
8. tank lorries with high pressure spray gun
9. reefers or refrigerated trucks
10. mobile drilling derricks
11. transit/concrete mixers

12. mobile radiological units


13. wreckers or tow trucks
14. concrete pump trucks
15. aerial/bucket flat-form trucks
16. street sweepers
17. vacuum trucks
18. garbage compactors
19. self loader trucks
20. man lift trucks
21. lighting trucks
22. trucks mounted with special purpose equipment
23. all other types of vehicle designed for a specific use.
The issuance of EO 156 spawned three separate actions for declaratory relief before Branch 72 of
the Regional Trial Court of Olongapo City, all seeking the declaration of the unconstitutionality of
Article 2, Section 3.1 of said executive order. The cases were filed by herein respondent entities,
who or whose members, are classified as Subic Bay Freeport Enterprises and engaged in the
business of, among others, importing and/or trading used motor vehicles.
G.R. No. 164171:
On January 16, 2004, respondents Southwing Heavy Industries, Inc., (Southwing) United
Auctioneers, Inc. (United Auctioneers), and Microvan, Inc. (Microvan), instituted a declaratory relief
case docketed as Civil Case No. 20-0-04,1 against the Executive Secretary, Secretary of
Transportation and Communication, Commissioner of Customs, Assistant Secretary and Head of the
Land Transportation Office, Subic Bay Metropolitan Authority (SBMA), Collector of Customs for the
Port at Subic Bay Freeport Zone, and the Chief of the Land Transportation Office at Subic Bay
Freeport Zone.
Southwing, United Auctioneers and Microvan prayed that judgment be rendered (1) declaring Article
2, Section 3.1 of EO 156 unconstitutional and illegal; (2) directing the Secretary of Finance,
Commissioner of Customs, Collector of Customs and the Chairman of the SBMA to allow the
importation of used motor vehicles; (2) ordering the Land Transportation Office and its subordinates
inside the Subic Special Economic Zone to process the registration of the imported used motor

vehicles; and (3) in general, to allow the unimpeded entry and importation of used motor vehicles
subject only to the payment of the required customs duties.
Upon filing of petitioners answer/comment, respondents Southwing and Microvan filed a motion for
summary judgment which was granted by the trial court. On May 24, 2004, a summary judgment
was rendered declaring that Article 2, Section 3.1 of EO 156 constitutes an unlawful usurpation of
legislative power vested by the Constitution with Congress. The trial court further held that the
proviso is contrary to the mandate of Republic Act No. 7227 (RA 7227) or the Bases Conversion and
Development Act of 1992 which allows the free flow of goods and capital within the Freeport. The
dispositive portion of the said decision reads:
WHEREFORE, judgment is hereby rendered in favor of petitioner declaring Executive Order 156
[Article 2, Section] 3.1 for being unconstitutional and illegal; directing respondents Collector of
Customs based at SBMA to allow the importation and entry of used motor vehicles pursuant to the
mandate of RA 7227; directing respondent Chief of the Land Transportation Office and its
subordinates inside the Subic Special Economic Zone or SBMA to process the registration of
imported used motor vehicle; and in general, to allow unimpeded entry and importation of used
motor vehicles to the Philippines subject only to the payment of the required customs duties.
SO ORDERED.2
From the foregoing decision, petitioners sought relief before this Court via a petition for review on
certiorari, docketed as G.R. No. 164171.
G.R. No. 164172:
On January 20, 2004, respondent Subic Integrated Macro Ventures Corporation (Macro Ventures)
filed with the same trial court, a similar action for declaratory relief docketed as Civil Case No. 22-004,3 with the same prayer and against the same parties4 as those in Civil Case No. 20-0-04.
In this case, the trial court likewise rendered a summary judgment on May 24, 2004, holding that
Article 2, Section 3.1 of EO 156, is repugnant to the constitution.5 Elevated to this Court via a petition
for review on certiorari, Civil Case No. 22-0-04 was docketed as G.R. No. 164172.
G.R. No. 168741
On January 22, 2003, respondent Motor Vehicle Importers Association of Subic Bay Freeport, Inc.
(Association), filed another action for declaratory relief with essentially the same prayer as those in
Civil Case No. 22-0-04 and Civil Case No. 20-0-04, against the Executive Secretary, Secretary of
Finance, Chief of the Land Transportation Office, Commissioner of Customs, Collector of Customs at
SBMA and the Chairman of SBMA. This was docketed as Civil Case No. 30-0-2003, 6 before the
same trial court.
In a decision dated March 10, 2004, the court a quo granted the Associations prayer and declared
the assailed proviso as contrary to the Constitution, to wit:

WHEREFORE, judgment is hereby rendered in favor of petitioner declaring Executive Order 156
[Article 2, Section] 3.1 for being unconstitutional and illegal; directing respondents Collector of
Customs based at SBMA to allow the importation and entry of used motor vehicles pursuant to the
mandate of RA 7227; directing respondent Chief of the Land Transportation Office and its
subordinates inside the Subic Special Economic Zone or SBMA to process the registration of
imported used motor vehicles; directing the respondent Chairman of the SBMA to allow the entry into
the Subic Special Economic Zone or SBMA imported used motor vehicle; and in general, to allow
unimpeded entry and importation of used motor vehicles to the Philippines subject only to the
payment of the required customs duties.
SO ORDERED.7
Aggrieved, the petitioners in Civil Case No. 30-0-2003, filed a petition for certiorari 8 with the Court of
Appeals (CA-G.R. SP. No. 83284) which denied the petition on February 14, 2005 and sustained the
finding of the trial court that Article 2, Section 3.1 of EO 156, is void for being repugnant to the
constitution. The dispositive portion thereof, reads:
WHEREFORE, the instant petition for certiorari is hereby DENIED. The assailed decision of the
Regional Trial Court, Third Judicial Region, Branch 72, Olongapo City, in Civil Case No. 30-0-2003,
accordingly, STANDS.
SO ORDERED.9
The aforequoted decision of the Court of Appeals was elevated to this Court and docketed as G.R.
No. 168741. In a Resolution dated October 4, 2005, 10 said case was consolidated with G.R. No.
164171 and G.R. No. 164172.
Petitioners are now before this Court contending that Article 2, Section 3.1 of EO 156 is valid and
applicable to the entire country, including the Freeeport. In support of their arguments, they raise
procedural and substantive issues bearing on the constitutionality of the assailed proviso.
The procedural issues are: the lack of respondents locus standi to question the validity of EO 156,
the propriety of challenging EO 156 in a declaratory relief proceeding and the applicability of a
judgment on the pleadings in this case.
Petitioners argue that respondents will not be affected by the importation ban considering that their
certificate of registration and tax exemption do not authorize them to engage in the importation
and/or trading of used cars. They also aver that the actions filed by respondents do not qualify as
declaratory relief cases. Section 1, Rule 63 of the Rules of Court provides that a petition for
declaratory relief may be filed before there is a breach or violation of rights. Petitioners claim that
there was already a breach of respondents supposed right because the cases were filed more than
a year after the issuance of EO 156. In fact, in Civil Case No. 30-0-2003, numerous warrants of
seizure and detention were issued against imported used motor vehicles belonging to respondent
Associations members.
Petitioners arguments lack merit.

The established rule that the constitutionality of a law or administrative issuance can be challenged
by one who will sustain a direct injury as a result of its enforcement 11 has been satisfied in the instant
case. The broad subject of the prohibited importation is "all types of used motor vehicles."
Respondents would definitely suffer a direct injury from the implementation of EO 156 because their
certificate of registration and tax exemption authorize them to trade and/or import new and
used motor vehicles and spare parts, except "used cars."12 Other types of motor vehicles imported
and/or traded by respondents and not falling within the category of used cars would thus be
subjected to the ban to the prejudice of their business. Undoubtedly, respondents have the legal
standing to assail the validity of EO 156.
As to the propriety of declaratory relief as a vehicle for assailing the executive issuance, suffice it to
state that any breach of the rights of respondents will not affect the case. In Commission on Audit of
the Province of Cebu v. Province of Cebu,13 the Court entertained a suit for declaratory relief to finally
settle the doubt as to the proper interpretation of the conflicting laws involved, notwithstanding a
violation of the right of the party affected. We find no reason to deviate from said ruling mindful of the
significance of the present case to the national economy.
So also, summary judgments were properly rendered by the trial court because the issues involved
in the instant case were pure questions of law. A motion for summary judgment is premised on the
assumption that the issues presented need not be tried either because these are patently devoid of
substance or that there is no genuine issue as to any pertinent fact. It is a method sanctioned by the
Rules of Court for the prompt disposition of a civil action in which the pleadings raise only a legal
issue, not a genuine issue as to any material fact. 14
At any rate, even assuming the procedural flaws raised by petitioners truly exist, the Court is not
precluded from brushing aside these technicalities and taking cognizance of the action filed by
respondents considering its importance to the public and in keeping with the duty to determine
whether the other branches of the government have kept themselves within the limits of the
Constitution.15
We now come to the substantive issues, which are: (1) whether there is statutory basis for the
issuance of EO 156; and (2) if the answer is in the affirmative, whether the application of Article 2,
Section 3.1 of EO 156, reasonable and within the scope provided by law.
The main thrust of the petition is that EO 156 is constitutional because it was issued pursuant to EO
226, the Omnibus Investment Code of the Philippines and that its application should be extended to
the Freeport because the guarantee of RA 7227 on the free flow of goods into the said zone is
merely an exemption from customs duties and taxes on items brought into the Freeport and not an
open floodgate for all kinds of goods and materials without restriction.
In G.R. No. 168741, the Court of Appeals invalidated Article 2, Section 3.1 of EO 156, on the ground
of lack of any statutory basis for the President to issue the same. It held that the prohibition on the
importation of used motor vehicles is an exercise of police power vested on the legislature and
absent any enabling law, the exercise thereof by the President through an executive issuance, is
void.

Police power is inherent in a government to enact laws, within constitutional limits, to promote the
order, safety, health, morals, and general welfare of society. It is lodged primarily with the legislature.
By virtue of a valid delegation of legislative power, it may also be exercised by the President and
administrative boards, as well as the lawmaking bodies on all municipal levels, including the
barangay.16 Such delegation confers upon the President quasi-legislative power which may be
defined as the authority delegated by the law-making body to the administrative body to adopt rules
and regulations intended to carry out the provisions of the law and implement legislative policy.17 To
be valid, an administrative issuance, such as an executive order, must comply with the following
requisites:
(1) Its promulgation must be authorized by the legislature;
(2) It must be promulgated in accordance with the prescribed procedure;
(3) It must be within the scope of the authority given by the legislature; and
(4) It must be reasonable.18
Contrary to the conclusion of the Court of Appeals, EO 156 actually satisfied the first requisite of a
valid administrative order. It has both constitutional and statutory bases.
Delegation of legislative powers to the President is permitted in Section 28(2) of Article VI of the
Constitution. It provides:
(2) The Congress may, by law, authorize the President to fix within specified limits, and subject to
such limitations and restrictions as it may impose, tariff rates, import and export quotas, tonnage and
wharfage dues, and other duties or imposts within the framework of the national development
program of the Government.19 (Emphasis supplied)
The relevant statutes to execute this provision are:
1) The Tariff and Customs Code which authorizes the President, in the interest of national
economy, general welfare and/or national security, to, inter alia, prohibit the importation of any
commodity. Section 401 thereof, reads:
Sec. 401. Flexible Clause.
a. In the interest of national economy, general welfare and/or national security, and subject to
the limitations herein prescribed, the President, upon recommendation of the National
Economic and Development Authority (hereinafter referred to as NEDA), is hereby
empowered: x x x (2) to establish import quota or to ban imports of any commodity, as may
be necessary; x x x Provided, That upon periodic investigations by the Tariff Commission and
recommendation of the NEDA, the President may cause a gradual reduction of protection levels
granted in Section One hundred and four of this Code, including those subsequently granted
pursuant to this section. (Emphasis supplied)

2) Executive Order No. 226, the Omnibus Investment Code of the Philippines which was issued on
July 16, 1987, by then President Corazon C. Aquino, in the exercise of legislative power under the
Provisional Freedom Constitution,20 empowers the President to approve or reject the prohibition on
the importation of any equipment or raw materials or finished products. Pertinent provisions thereof,
read:
ART. 4. Composition of the board. The Board of Investments shall be composed of seven (7)
governors: The Secretary of Trade and Industry, three (3) Undersecretaries of Trade and Industry to
be chosen by the President; and three (3) representatives from the government agencies and the
private sector x x x.
ART. 7. Powers and duties of the Board.
xxxx
(12) Formulate and implement rationalization programs for certain industries whose operation may
result in dislocation, overcrowding or inefficient use of resources, thus impeding economic growth.
For this purpose, the Board may formulate guidelines for progressive manufacturing programs, local
content programs, mandatory sourcing requirements and dispersal of industries. In appropriate
cases and upon approval of the President, the Board may restrict, either totally or partially,
the importation of any equipment or raw materials or finished products involved in the
rationalization program; (Emphasis supplied)
3) Republic Act No. 8800, otherwise known as the "Safeguard Measures Act" (SMA), and entitled
"An Act Protecting Local Industries By Providing Safeguard Measures To Be Undertaken In
Response To Increased Imports And Providing Penalties For Violation Thereof," 21 designated the
Secretaries22 of the Department of Trade and Industry (DTI) and the Department of Agriculture, in
their capacity as alter egos of the President, as the implementing authorities of the safeguard
measures, which include, inter alia, modification or imposition of any quantitative restriction on the
importation of a product into the Philippines. The purpose of the SMA is stated in the declaration of
policy, thus:
SEC. 2. Declaration of Policy. The State shall promote competitiveness of domestic industries and
producers based on sound industrial and agricultural development policies, and efficient use of
human, natural and technical resources. In pursuit of this goal and in the public interest, the State
shall provide safeguard measures to protect domestic industries and producers from increased
imports which cause or threaten to cause serious injury to those domestic industries and producers.
There are thus explicit constitutional and statutory permission authorizing the President to ban or
regulate importation of articles and commodities into the country.
Anent the second requisite, that is, that the order must be issued or promulgated in accordance
with the prescribed procedure, it is necessary that the nature of the administrative issuance is
properly determined. As in the enactment of laws, the general rule is that, the promulgation of
administrative issuances requires previous notice and hearing, the only exception being where the
legislature itself requires it and mandates that the regulation shall be based on certain facts as

determined at an appropriate investigation.23 This exception pertains to the issuance of legislative


rules as distinguished from interpretative rules which give no real consequence more than what
the law itself has already prescribed;24 and are designed merely to provide guidelines to the law
which the administrative agency is in charge of enforcing. 25 A legislative rule, on the other hand, is
in the nature of subordinate legislation, crafted to implement a primary legislation.
In Commissioner of Internal Revenue v. Court of Appeals,26 and Commissioner of Internal Revenue
v. Michel J. Lhuillier Pawnshop, Inc.,27 the Court enunciated the doctrine that when an administrative
rule goes beyond merely providing for the means that can facilitate or render less cumbersome the
implementation of the law and substantially increases the burden of those governed, it behooves the
agency to accord at least to those directly affected a chance to be heard and, thereafter, to be duly
informed, before the issuance is given the force and effect of law.
In the instant case, EO 156 is obviously a legislative rule as it seeks to implement or execute primary
legislative enactments intended to protect the domestic industry by imposing a ban on the
importation of a specified product not previously subject to such prohibition. The due process
requirements in the issuance thereof are embodied in Section 401 28 of the Tariff and Customs Code
and Sections 5 and 9 of the SMA29 which essentially mandate the conduct of investigation and public
hearings before the regulatory measure or importation ban may be issued.
In the present case, respondents neither questioned before this Court nor with the courts below the
procedure that paved the way for the issuance of EO 156. What they challenged in their petitions
before the trial court was the absence of "substantive due process" in the issuance of the EO. 30 Their
main contention before the court a quo is that the importation ban is illogical and unfair because it
unreasonably drives them out of business to the prejudice of the national economy.
Considering the settled principle that in the absence of strong evidence to the contrary, acts of the
other branches of the government are presumed to be valid,31 and there being no objection from the
respondents as to the procedure in the promulgation of EO 156, the presumption is that said
executive issuance duly complied with the procedures and limitations imposed by law.
To determine whether EO 156 has complied with the third and fourth requisites of a valid
administrative issuance, to wit, that it was issued within the scope of authority given by the
legislature and that it is reasonable, an examination of the nature of a Freeport under RA 7227 and
the primordial purpose of the importation ban under the questioned EO is necessary.
RA 7227 was enacted providing for, among other things, the sound and balanced conversion of the
Clark and Subic military reservations and their extensions into alternative productive uses in the form
of Special Economic and Freeport Zone, or the Subic Bay Freeport, in order to promote the
economic and social development of Central Luzon in particular and the country in general.
The Rules and Regulations Implementing RA 7227 specifically defines the territory comprising the
Subic Bay Freeport, referred to as the Special Economic and Freeport Zone in Section 12 of RA
7227 as "a separate customs territory consisting of the City of Olongapo and the Municipality of
Subic, Province of Zambales, the lands occupied by the Subic Naval Base and its contiguous
extensions as embraced, covered and defined by the 1947 Philippine-U.S. Military Base Agreement

as amended and within the territorial jurisdiction of Morong and Hermosa, Province of Bataan, the
metes and bounds of which shall be delineated by the President of the Philippines; provided further
that pending establishment of secure perimeters around the entire SBF, the SBF shall refer to the
area demarcated by the SBMA pursuant to Section 1332 hereof."
Among the salient provisions of RA 7227 are as follows:
SECTION 12. Subic Special Economic Zone.
xxxx
The abovementioned zone shall be subject to the following policies:
xxxx
(a) Within the framework and subject to the mandate and limitations of the Constitution and
the pertinent provisions of the Local Government Code, the Subic Special Economic Zone
shall be developed into a self-sustaining, industrial, commercial, financial and investment
center to generate employment opportunities in and around the zone and to attract and
promote productive foreign investments;
(b) The Subic Special Economic Zone shall be operated and managed as a separate
customs territory ensuring free flow or movement of goods and capital within, into and
exported out of the Subic Special Economic Zone, as well as provide incentives such as tax
and duty-free importations of raw materials, capital and equipment. However, exportation or
removal of goods from the territory of the Subic Special Economic Zone to the other parts of
the Philippine territory shall be subject to customs duties and taxes under the Customs and
Tariff Code and other relevant tax laws of the Philippines;
The Freeport was designed to ensure free flow or movement of goods and capital within a portion of
the Philippine territory in order to attract investors to invest their capital in a business climate with the
least governmental intervention. The concept of this zone was explained by Senator Guingona in
this wise:
Senator Guingona. Mr. President, the special economic zone is successful in many places,
particularly Hong Kong, which is a free port. The difference between a special economic zone and
an industrial estate is simply expansive in the sense that the commercial activities, including the
establishment of banks, services, financial institutions, agro-industrial activities, maybe agriculture to
a certain extent.
This delineates the activities that would have the least of government intervention, and the
running of the affairs of the special economic zone would be run principally by the investors
themselves, similar to a housing subdivision, where the subdivision owners elect their
representatives to run the affairs of the subdivision, to set the policies, to set the guidelines.

We would like to see Subic area converted into a little Hong Kong, Mr. President, where there
is a hub of free port and free entry, free duties and activities to a maximum spur generation of
investment and jobs.
While the investor is reluctant to come in the Philippines, as a rule, because of red tape and
perceived delays, we envision this special economic zone to be an area where there will be
minimum government interference.
The initial outlay may not only come from the Government or the Authority as envisioned here, but
from them themselves, because they would be encouraged to invest not only for the land but also for
the buildings and factories. As long as they are convinced that in such an area they can do business
and reap reasonable profits, then many from other parts, both local and foreign, would invest, Mr.
President.33 (Emphasis, added)
With minimum interference from the government, investors can, in general, engage in any kind of
business as well as import and export any article into and out of the Freeport. These are among the
rights accorded to Subic Bay Freeport Enterprises under Section 39 of the Rules and Regulations
Implementing RA 7227, thus
SEC. 39. Rights and Obligations.- SBF Enterprises shall have the following rights and obligations:
a. To freely engage in any business, trade, manufacturing, financial or service activity, and to import
and export freely all types of goods into and out of the SBF, subject to the provisions of the Act,
these Rules and other regulations that may be promulgated by the SBMA;
Citing, inter alia, the interpellations of Senator Enrile, petitioners claim that the "free flow or
movement of goods and capital" only means that goods and material brought within the Freeport
shall not be subject to customs duties and other taxes and should not be construed as an open
floodgate for entry of all kinds of goods. They thus surmise that the importation ban on motor
vehicles is applicable within the Freeport. Pertinent interpellations of Senator Enrile on the concept
of Freeport is as follows:
Senator Enrile: Mr. President, I think we are talking here of sovereign concepts, not territorial
concepts. The concept that we are supposed to craft here is to carve out a portion of our terrestrial
domain as well as our adjacent waters and say to the world: "Well, you can set up your factories in
this area that we are circumscribing, and bringing your equipment and bringing your goods, you are
not subject to any taxes and duties because you are not within the customs jurisdiction of the
Republic of the Philippines, whether you store the goods or only for purposes of transshipment or
whether you make them into finished products again to be reexported to other lands."
xxxx
My understanding of a "free port" is, we are in effect carving out a part of our territory and
make it as if it were foreign territory for purposes of our customs laws, and that people can
come, bring their goods, store them there and bring them out again, as long as they do not
come into the domestic commerce of the Republic.

We do not really care whether these goods are stored here. The only thing that we care is for our
people to have an employment because of the entry of these goods that are being discharged,
warehoused and reloaded into the ships so that they can be exported. That will generate
employment for us. For as long as that is done, we are saying, in effect, that we have the least
contact with our tariff and customs laws and our tax laws. Therefore, we consider these goods as
outside of the customs jurisdiction of the Republic of the Philippines as yet, until we draw them from
this territory and bring them inside our domestic commerce. In which case, they have to pass
through our customs gate. I thought we are carving out this entire area and convert it into this kind of
concept.34
However, contrary to the claim of petitioners, there is nothing in the foregoing excerpts which
absolutely limits the incentive to Freeport investors only to exemption from customs duties and
taxes. Mindful of the legislative intent to attract investors, enhance investment and boost the
economy, the legislature could not have limited the enticement only to exemption from taxes. The
minimum interference policy of the government on the Freeport extends to the kind of business that
investors may embark on and the articles which they may import or export into and out of the zone. A
contrary interpretation would defeat the very purpose of the Freeport and drive away investors.
It does not mean, however, that the right of Freeport enterprises to import all types of goods and
article is absolute. Such right is of course subject to the limitation that articles absolutely prohibited
by law cannot be imported into the Freeport.35 Nevertheless, in determining whether the prohibition
would apply to the Freeport, resort to the purpose of the prohibition is necessary.
In issuing EO 156, particularly the prohibition on importation under Article 2, Section 3.1, the
President envisioned to rationalize the importation of used motor vehicles and to enhance the
capabilities of the Philippine motor manufacturing firms to be globally competitive producers of
completely build-up units and their parts and components for the local and export markets. 36 In
justifying the issuance of EO 156, petitioners alleged that there has been a decline in the sales of
new vehicles and a remarkable growth of the sales of imported used motor vehicles. To address the
same, the President issued the questioned EO to prevent further erosion of the already depressed
market base of the local motor vehicle industry and to curtail the harmful effects of the increase in
the importation of used motor vehicles.37
Taking our bearings from the foregoing discussions, we hold that the importation ban runs afoul
the third requisite for a valid administrative order. To be valid, an administrative issuance must not
be ultra vires or beyond the limits of the authority conferred. It must not supplant or modify the
Constitution, its enabling statute and other existing laws, for such is the sole function of the
legislature which the other branches of the government cannot usurp. As held in United BF
Homeowners Association v. BF Homes, Inc.:38
The rule-making power of a public administrative body is a delegated legislative power, which it may
not use either to abridge the authority given it by Congress or the Constitution or to enlarge its power
beyond the scope intended. Constitutional and statutory provisions control what rules and
regulations may be promulgated by such a body, as well as with respect to what fields are subject to
regulation by it. It may not make rules and regulations which are inconsistent with the provisions of

the Constitution or a statute, particularly the statute it is administering or which created it, or which
are in derogation of, or defeat, the purpose of a statute.
In the instant case, the subject matter of the laws authorizing the President to regulate or forbid
importation of used motor vehicles, is the domestic industry. EO 156, however, exceeded the
scope of its application by extending the prohibition on the importation of used cars to the Freeport,
which RA 7227, considers to some extent, a foreign territory. The domestic industry which the EO
seeks to protect is actually the "customs territory" which is defined under the Rules and
Regulations Implementing RA 7227, as follows:
"the portion of the Philippines outside the Subic Bay Freeport where the Tariff and Customs
Code of the Philippines and other national tariff and customs laws are in force and effect."39
The proscription in the importation of used motor vehicles should be operative only outside the
Freeport and the inclusion of said zone within the ambit of the prohibition is an invalid modification of
RA 7227. Indeed, when the application of an administrative issuance modifies existing laws or
exceeds the intended scope, as in the instant case, the issuance becomes void, not only for
being ultra vires, but also for being unreasonable.
This brings us to the fourth requisite. It is an axiom in administrative law that administrative
authorities should not act arbitrarily and capriciously in the issuance of rules and regulations. To be
valid, such rules and regulations must be reasonable and fairly adapted to secure the end in view. If
shown to bear no reasonable relation to the purposes for which they were authorized to be issued,
then they must be held to be invalid.40
There is no doubt that the issuance of the ban to protect the domestic industry is a reasonable
exercise of police power. The deterioration of the local motor manufacturing firms due to the influx of
imported used motor vehicles is an urgent national concern that needs to be swiftly addressed by
the President. In the exercise of delegated police power, the executive can therefore validly
proscribe the importation of these vehicles. Thus, inTaxicab Operators of Metro Manila, Inc. v. Board
of Transportation,41 the Court held that a regulation phasing out taxi cabs more than six years old is a
valid exercise of police power. The regulation was sustained as reasonable holding that the purpose
thereof was to promote the convenience and comfort and protect the safety of the passengers.
The problem, however, lies with respect to the application of the importation ban to the Freeport. The
Court finds no logic in the all encompassing application of the assailed provision to the Freeport
which is outside the customs territory. As long as the used motor vehicles do not enter the customs
territory, the injury or harm sought to be prevented or remedied will not arise. The application of the
law should be consistent with the purpose of and reason for the law. Ratione cessat lex, et cessat
lex. When the reason for the law ceases, the law ceases. It is not the letter alone but the spirit of the
law also that gives it life.42 To apply the proscription to the Freeport would not serve the purpose of
the EO. Instead of improving the general economy of the country, the application of the importation
ban in the Freeport would subvert the avowed purpose of RA 7227 which is to create a market that
would draw investors and ultimately boost the national economy.

In similar cases, we also declared void the administrative issuance or ordinances concerned for
being unreasonable. To illustrate, in De la Cruz v. Paras,43 the Court held as unreasonable and
unconstitutional an ordinance characterized by overbreadth. In that case, the Municipality of Bocaue,
Bulacan, prohibited the operation of all night clubs, cabarets and dance halls within its jurisdiction for
the protection of public morals. As explained by the Court:
x x x It cannot be said that such a sweeping exercise of a lawmaking power by Bocaue could qualify
under the term reasonable. The objective of fostering public morals, a worthy and desirable end can
be attained by a measure that does not encompass too wide a field. Certainly the ordinance on its
face is characterized by overbreadth. The purpose sought to be achieved could have been attained
by reasonable restrictions rather than by an absolute prohibition. The admonition in Salaveria should
be heeded: "The Judiciary should not lightly set aside legislative action when there is not a clear
invasion of personal or property rights under the guise of police regulation." It is clear that in the
guise of a police regulation, there was in this instance a clear invasion of personal or property rights,
personal in the case of those individuals desirous of patronizing those night clubs and property in
terms of the investments made and salaries to be earned by those therein employed.
Lupangco v. Court of Appeals,44 is a case involving a resolution issued by the Professional
Regulation Commission which prohibited examinees from attending review classes and receiving
handout materials, tips, and the like three days before the date of examination in order to preserve
the integrity and purity of the licensure examinations in accountancy. Besides being unreasonable on
its face and violative of academic freedom, the measure was found to be more sweeping than what
was necessary, viz:
Needless to say, the enforcement of Resolution No. 105 is not a guarantee that the alleged leakages
in the licensure examinations will be eradicated or at least minimized. Making the examinees suffer
by depriving them of legitimate means of review or preparation on those last three precious days
when they should be refreshing themselves with all that they have learned in the review classes and
preparing their mental and psychological make-up for the examination day itself would be like
uprooting the tree to get rid of a rotten branch. What is needed to be done by the respondent is to
find out the source of such leakages and stop it right there. If corrupt officials or personnel should be
terminated from their loss, then so be it. Fixers or swindlers should be flushed out. Strict guidelines
to be observed by examiners should be set up and if violations are committed, then licenses should
be suspended or revoked. x x x
In Lucena Grand Central Terminal, Inc. v. JAC Liner, Inc.,45 the Court likewise struck down as
unreasonable and overbreadth a city ordinance granting an exclusive franchise for 25 years,
renewable for another 25 years, to one entity for the construction and operation of one common bus
and jeepney terminal facility in Lucena City. While professedly aimed towards alleviating the traffic
congestion alleged to have been caused by the existence of various bus and jeepney terminals
within the city, the ordinance was held to be beyond what is reasonably necessary to solve the traffic
problem in the city.
By parity of reasoning, the importation ban in this case should also be declared void for its too
sweeping and unnecessary application to the Freeport which has no bearing on the objective of the
prohibition. If the aim of the EO is to prevent the entry of used motor vehicles from the Freeport to

the customs territory, the solution is not to forbid entry of these vehicles into the Freeport, but to
intensify governmental campaign and measures to thwart illegal ingress of used motor vehicles into
the customs territory.
At this juncture, it must be mentioned that on June 19, 1993, President Fidel V. Ramos issued
Executive Order No. 97-A, "Further Clarifying The Tax And Duty-Free Privilege Within The Subic
Special Economic And Free Port Zone," Section 1 of which provides:
SECTION 1. The following guidelines shall govern the tax and duty-free privilege within the Secured
Area of the Subic Special Economic and Free Port Zone:
1.1. The Secured Area consisting of the presently fenced-in former Subic Naval Base shall be the
only completely tax and duty-free area in the SSEFPZ. Business enterprises and individuals
(Filipinos and foreigners) residing within the Secured Area are free to import raw materials, capital
goods, equipment, and consumer items tax and dutry-free. Consumption items, however, must be
consumed within the Secured Area. Removal of raw materials, capital goods, equipment and
consumer items out of the Secured Area for sale to non-SSEFPZ registered enterprises shall be
subject to the usual taxes and duties, except as may be provided herein.
In Tiu v. Court of Appeals46 as reiterated in Coconut Oil Refiners Association, Inc. v. Torres,47 this
provision limiting the special privileges on tax and duty-free importation in the presently fenced-in
former Subic Naval Base has been declared valid and constitutional and in accordance with RA
7227. Consistent with these rulings and for easier management and monitoring of activities and to
prevent fraudulent importation of merchandise and smuggling, the free flow and importation of used
motor vehicles shall be operative only within the "secured area."
In sum, the Court finds that Article 2, Section 3.1 of EO 156 is void insofar as it is made applicable to
the presently secured fenced-in former Subic Naval Base area as stated in Section 1.1 of EO 97-A.
Pursuant to the separability clause48 of EO 156, Section 3.1 is declared valid insofar as it applies to
the customs territory or the Philippine territory outside the presently secured fenced-in former Subic
Naval Base area as stated in Section 1.1 of EO 97-A. Hence, used motor vehicles that come into the
Philippine territory via the secured fenced-in former Subic Naval Base area may be stored, used or
traded therein, or exported out of the Philippine territory, but they cannot be imported into the
Philippine territory outside of the secured fenced-in former Subic Naval Base area.
WHEREFORE, the petitions are PARTIALLY GRANTED and the May 24, 2004 Decisions of Branch
72, Regional Trial Court of Olongapo City, in Civil Case No. 20-0-04 and Civil Case No. 22-0-04; and
the February 14, 2005 Decision of the Court of Appeals in CA-G.R. SP No. 63284, are MODIFIED
insofar as they declared Article 2, Section 3.1 of Executive Order No. 156, void in its entirety.
Said provision is declared VALID insofar as it applies to the Philippine territory outside the presently
fenced-in former Subic Naval Base area and VOID with respect to its application to the secured
fenced-in former Subic Naval Base area.
SO ORDERED.

Republic of the Philippines


SUPREME COURT
Manila
EN BANC
G.R. No. 161434

March 3, 2004

MARIA JEANETTE C. TECSON and FELIX B. DESIDERIO, JR., petitioners,


vs.
The COMMISSION ON ELECTIONS, RONALD ALLAN KELLY POE (a.k.a. FERNANDO POE,
JR.) and VICTORINO X. FORNIER, respondents.
x-----------------------------x
G.R. No. 161634

March 3, 2004

ZOILO ANTONIO VELEZ, petitioner,


vs.
RONALD ALLAN KELLEY POE, a.k.a. FERNANDO POE, JR., respondent.
x-----------------------------x
G. R. No. 161824

March 3, 2004

VICTORINO X. FORNIER, petitioner,


vs.
HON. COMMISSION ON ELECTIONS and RONALD ALLAN KELLEY POE, ALSO KNOWN AS
FERNANDO POE JR., respondents.
DECISION
VITUG, J.:
Citizenship is a treasured right conferred on those whom the state believes are deserving of
the privilege. It is a "precious heritage, as well as an inestimable acquisition," 1 that cannot be
taken lightly by anyone - either by those who enjoy it or by those who dispute it.
Before the Court are three consolidated cases, all of which raise a single question of profound
importance to the nation. The issue of citizenship is brought up to challenge the qualifications of a
presidential candidate to hold the highest office of the land. Our people are waiting for the judgment
of the Court with bated breath. Is Fernando Poe, Jr., the hero of silver screen, and now one of the
main contenders for the presidency, a natural-born Filipino or is he not?
The moment of introspection takes us face to face with Spanish and American colonial roots and
reminds us of the rich heritage of civil law and common law traditions, the fusion resulting in a hybrid
of laws and jurisprudence that could be no less than distinctly Filipino.

Antecedent Case Settings


On 31 December 2003, respondent Ronald Allan Kelly Poe, also known as Fernando Poe, Jr.
(hereinafter "FPJ"), filed his certificate of candidacy for the position of President of the Republic of
the Philippines under the Koalisyon ng Nagkakaisang Pilipino (KNP) Party, in the forthcoming
national elections. In his certificate of candidacy, FPJ, representing himself to be a natural-born
citizen of the Philippines, stated his name to be "Fernando Jr.," or "Ronald Allan" Poe, his date of
birth to be 20 August 1939 and his place of birth to be Manila.
Victorino X. Fornier, petitioner in G.R. No. 161824, entitled "Victorino X. Fornier, Petitioner, versus
Hon. Commission on Elections and Ronald Allan Kelley Poe, also known as Fernando Poe, Jr.,
Respondents," initiated, on 09 January 2004, a petition docketed SPA No. 04-003 before the
Commission on Elections ("COMELEC") to disqualify FPJ and to deny due course or to cancel his
certificate of candidacy upon the thesis that FPJ made a material misrepresentation in his certificate
of candidacy by claiming to be a natural-born Filipino citizen when in truth, according to Fornier, his
parents were foreigners; his mother, Bessie Kelley Poe, was an American, and his father, Allan Poe,
was a Spanish national, being the son of Lorenzo Pou, a Spanish subject. Granting, petitioner
asseverated, that Allan F. Poe was a Filipino citizen, he could not have transmitted his Filipino
citizenship to FPJ, the latter being an illegitimate child of an alien mother. Petitioner based the
allegation of the illegitimate birth of respondent on two assertions - first, Allan F. Poe contracted a
prior marriage to a certain Paulita Gomez before his marriage to Bessie Kelley and, second, even if
no such prior marriage had existed, Allan F. Poe, married Bessie Kelly only a year after the birth of
respondent.
In the hearing before the Third Division of the COMELEC on 19 January 2004, petitioner, in support
of his claim, presented several documentary exhibits - 1) a copy of the certificate of birth of FPJ, 2) a
certified photocopy of an affidavit executed in Spanish by Paulita Poe y Gomez attesting to her
having filed a case for bigamy and concubinage against the father of respondent, Allan F. Poe, after
discovering his bigamous relationship with Bessie Kelley, 3) an English translation of the affidavit
aforesaid, 4) a certified photocopy of the certificate of birth of Allan F. Poe, 5) a certification issued by
the Director of the Records Management and Archives Office, attesting to the fact that there was no
record in the National Archives that a Lorenzo Poe or Lorenzo Pou resided or entered the
Philippines before 1907, and 6) a certification from the Officer-In-Charge of the Archives Division of
the National Archives to the effect that no available information could be found in the files of the
National Archives regarding the birth of Allan F. Poe.
On his part, respondent, presented twenty-two documentary pieces of evidence, the more significant
ones being - a) a certification issued by Estrella M. Domingo of the Archives Division of the National
Archives that there appeared to be no available information regarding the birth of Allan F. Poe in the
registry of births for San Carlos, Pangasinan, b) a certification issued by the Officer-In-Charge of the
Archives Division of the National Archives that no available information about the marriage of Allan F.
Poe and Paulita Gomez could be found, c) a certificate of birth of Ronald Allan Poe, d) Original
Certificate of Title No. P-2247 of the Registry of Deeds for the Province of Pangasinan, in the name
of Lorenzo Pou, e) copies of Tax Declaration No. 20844, No. 20643, No. 23477 and No. 23478 in the
name of Lorenzo Pou, f) a copy of the certificate of death of Lorenzo Pou, g) a copy of the purported
marriage contract between Fernando Pou and Bessie Kelley, and h) a certification issued by the City
Civil Registrar of San Carlos City, Pangasinan, stating that the records of birth in the said office
during the period of from 1900 until May 1946 were totally destroyed during World War II.
On 23 January 2004, the COMELEC dismissed SPA No. 04-003 for lack of merit. Three days later,
or on 26 January 2004, Fornier filed his motion for reconsideration. The motion was denied on 06
February 2004 by the COMELEC en banc. On 10 February 2004, petitioner assailed the decision of

the COMELEC before this Court conformably with Rule 64, in relation to Rule 65, of the Revised
Rules of Civil Procedure. The petition, docketed G. R. No. 161824, likewise prayed for a temporary
restraining order, a writ of preliminary injunction or any other resolution that would stay the finality
and/or execution of the COMELEC resolutions.
The other petitions, later consolidated with G. R. No. 161824, would include G. R. No. 161434,
entitled "Maria Jeanette C. Tecson, and Felix B. Desiderio, Jr., vs. The Commission on Elections,
Ronald Allan Kelley Poe (a.k.a. Fernando Poe, Jr.), and Victorino X. Fornier," and the other,
docketed G. R. No. 161634, entitled "Zoilo Antonio G. Velez, vs. Ronald Allan Kelley Poe, a.k.a.
Fernando Poe, Jr.," both challenging the jurisdiction of the COMELEC and asserting that, under
Article VII, Section 4, paragraph 7, of the 1987 Constitution, only the Supreme Court had original
and exclusive jurisdiction to resolve the basic issue on the case.
Jurisdiction of the Court
In G. R. No. 161824
In seeking the disqualification of the candidacy of FPJ and to have the COMELEC deny due course
to or cancel FPJs certificate of candidacy for alleged misrepresentation of a material fact (i.e., that
FPJ was a natural-born citizen) before the COMELEC, petitioner Fornier invoked Section 78 of the
Omnibus Election Code
"Section 78. Petition to deny due course to or cancel a certificate of candidacy. --- A verified
petition seeking to deny due course or to cancel a certificate of candidacy may be filed by
any person exclusively on the ground that any material representation contained therein as
required under Section 74 hereof is false"
in consonance with the general powers of COMELEC expressed in Section 52 of the Omnibus
Election Code "Section 52. Powers and functions of the Commission on Elections. In addition to the powers
and functions conferred upon it by the Constitution, the Commission shall have exclusive
charge of the enforcement and administration of all laws relative to the conduct of elections
for the purpose of ensuring free, orderly and honest elections" and in relation to Article 69 of the Omnibus Election Code which would authorize "any
interested party" to file a verified petition to deny or cancel the certificate of candidacy of any
nuisance candidate.
Decisions of the COMELEC on disqualification cases may be reviewed by the Supreme Court per
Rule 642 in an action for certiorari under Rule 653 of the Revised Rules of Civil Procedure. Section 7,
Article IX, of the 1987 Constitution also reads
"Each Commission shall decide by a majority vote of all its Members any case or matter
brought before it within sixty days from the date of its submission for decision or resolution. A
case or matter is deemed submitted for decision or resolution upon the filing of the last
pleading, brief, or memorandum, required by the rules of the Commission or by the
Commission itself. Unless otherwise provided by this Constitution or by law, any decision,
order, or ruling of each Commission may be brought to the Supreme Court on certiorari by
the aggrieved party within thirty days from receipt of a copy thereof."

Additionally, Section 1, Article VIII, of the same Constitution provides that judicial power is vested in
one Supreme Court and in such lower courts as may be established by law which power "includes
the duty of the courts of justice to settle actual controversies involving rights which are legally
demandable and enforceable, and to determine whether or not there has been a grave abuse of
discretion amounting to lack or excess of jurisdiction on the part of any branch or instrumentality of
the Government."
It is sufficiently clear that the petition brought up in G. R. No. 161824 was aptly elevated to, and
could well be taken cognizance of by, this Court. A contrary view could be a gross denial to our
people of their fundamental right to be fully informed, and to make a proper choice, on who could or
should be elected to occupy the highest government post in the land.
In G. R. No. 161434 and G. R. No. 161634
Petitioners Tecson, et al., in G. R. No. 161434, and Velez, in G. R. No. 161634, invoke the provisions
of Article VII, Section 4, paragraph 7, of the 1987 Constitution in assailing the jurisdiction of the
COMELEC when it took cognizance of SPA No. 04-003 and in urging the Supreme Court to instead
take on the petitions they directly instituted before it. The Constitutional provision cited reads:
"The Supreme Court, sitting en banc, shall be the sole judge of all contests relating to the
election, returns, and qualifications of the President or Vice-President, and may promulgate
its rules for the purpose."
The provision is an innovation of the 1987 Constitution. The omission in the 1935 and the 1973
Constitution to designate any tribunal to be the sole judge of presidential and vice-presidential
contests, has constrained this Court to declare, in Lopez vs. Roxas, 4 as "not (being) justiciable"
controversies or disputes involving contests on the elections, returns and qualifications of the
President or Vice-President. The constitutional lapse prompted Congress, on 21 June 1957, to enact
Republic Act No. 1793, "An Act Constituting an Independent Presidential Electoral Tribunal to Try,
Hear and Decide Protests Contesting the Election of the President-Elect and the Vice-PresidentElect of the Philippines and Providing for the Manner of Hearing the Same." Republic Act 1793
designated the Chief Justice and the Associate Justices of the Supreme Court to be the members of
the tribunal. Although the subsequent adoption of the parliamentary form of government under the
1973 Constitution might have implicitly affected Republic Act No. 1793, the statutory set-up,
nonetheless, would now be deemed revived under the present Section 4, paragraph 7, of the 1987
Constitution.
Ordinary usage would characterize a "contest" in reference to a post-election scenario. Election
contests consist of either an election protest or a quo warranto which, although two distinct
remedies, would have one objective in view, i.e., to dislodge the winning candidate from office. A
perusal of the phraseology in Rule 12, Rule 13, and Rule 14 of the "Rules of the Presidential
Electoral Tribunal," promulgated by the Supreme Court en banc on 18 April 1992, would support this
premise "Rule 12. Jurisdiction. - The Tribunal shall be the sole judge of all contests relating to the
election, returns, and qualifications of the President or Vice-President of the Philippines.
"Rule 13. How Initiated. - An election contest is initiated by the filing of an election protest or
a petition for quo warranto against the President or Vice-President. An election protest shall
not include a petition for quo warranto. A petition for quo warranto shall not include an
election protest.

"Rule 14. Election Protest. - Only the registered candidate for President or for Vice-President
of the Philippines who received the second or third highest number of votes may contest the
election of the President or the Vice-President, as the case may be, by filing a verified
petition with the Clerk of the Presidential Electoral Tribunal within thirty (30) days after the
proclamation of the winner."
The rules categorically speak of the jurisdiction of the tribunal over contests relating to the election,
returns and qualifications of the "President" or "Vice-President", of the Philippines, and not of
"candidates" for President or Vice-President. A quo warranto proceeding is generally defined as
being an action against a person who usurps, intrudes into, or unlawfully holds or exercises a public
office.5 In such context, the election contest can only contemplate a post-election scenario. In Rule
14, only a registered candidate who would have received either the second or third highest number
of votes could file an election protest. This rule again presupposes a post-election scenario.
It is fair to conclude that the jurisdiction of the Supreme Court, defined by Section 4, paragraph 7, of
the 1987 Constitution, would not include cases directly brought before it, questioning the
qualifications of a candidate for the presidency or vice-presidency before the elections are held.
Accordingly, G. R. No. 161434, entitled "Maria Jeanette C. Tecson, et al., vs. Commission on
Elections et al.," and G. R. No. 161634, entitled "Zoilo Antonio Velez vs. Ronald Allan Kelley Poe
a.k.a. Fernando Poe, Jr." would have to be dismissed for want of jurisdiction.
The Citizenship Issue
Now, to the basic issue; it should be helpful to first give a brief historical background on the concept
of citizenship.
Perhaps, the earliest understanding of citizenship was that given by Aristotle, who, sometime in 384
to 322 B.C., described the "citizen" to refer to a man who shared in the administration of justice and
in the holding of an office.6 Aristotle saw its significance if only to determine the constituency of the
"State," which he described as being composed of such persons who would be adequate in number
to achieve a self-sufficient existence.7 The concept grew to include one who would both govern and
be governed, for which qualifications like autonomy, judgment and loyalty could be expected.
Citizenship was seen to deal with rights and entitlements, on the one hand, and with concomitant
obligations, on the other.8 In its ideal setting, a citizen was active in public life and fundamentally
willing to submit his private interests to the general interest of society.
The concept of citizenship had undergone changes over the centuries. In the 18th century, the
concept was limited, by and large, to civil citizenship, which established the rights necessary for
individual freedom, such as rights to property, personal liberty and justice. 9 Its meaning expanded
during the 19th century to include political citizenship, which encompassed the right to participate in
the exercise of political power.10 The 20th century saw the next stage of the development of social
citizenship, which laid emphasis on the right of the citizen to economic well-being and social
security.11 The idea of citizenship has gained expression in the modern welfare state as it so
developed in Western Europe. An ongoing and final stage of development, in keeping with the
rapidly shrinking global village, might well be the internationalization of citizenship. 12
The Local Setting - from Spanish Times to the Present
There was no such term as "Philippine citizens" during the Spanish regime but "subjects of Spain" or
"Spanish subjects."13 In church records, the natives were called 'indios', denoting a low regard for the
inhabitants of the archipelago. Spanish laws on citizenship became highly codified during the 19th

century but their sheer number made it difficult to point to one comprehensive law. Not all of these
citizenship laws of Spain however, were made to apply to the Philippine Islands except for those
explicitly extended by Royal Decrees.14
Spanish laws on citizenship were traced back to the Novisima Recopilacion, promulgated in Spain
on 16 July 1805 but as to whether the law was extended to the Philippines remained to be the
subject of differing views among experts;15 however, three royal decrees were undisputably made
applicable to Spaniards in the Philippines - the Order de la Regencia of 14 August 1841, 16 the Royal
Decree of 23 August 1868 specifically defining the political status of children born in the Philippine
Islands,17 and finally, the Ley Extranjera de Ultramar of 04 July 1870, which was expressly made
applicable to the Philippines by the Royal Decree of 13 July 1870. 18
The Spanish Constitution of 1876 was never extended to the Philippine Islands because of the
express mandate of its Article 89, according to which the provisions of the Ultramar among which
this country was included, would be governed by special laws.19
It was only the Civil Code of Spain, made effective in this jurisdiction on 18 December 1889, which
came out with the first categorical enumeration of who were Spanish citizens. "(a) Persons born in Spanish territory,
"(b) Children of a Spanish father or mother, even if they were born outside of Spain,
"(c) Foreigners who have obtained naturalization papers,
"(d) Those who, without such papers, may have become domiciled inhabitants of any town of
the Monarchy."20
The year 1898 was another turning point in Philippine history. Already in the state of decline as a
superpower, Spain was forced to so cede her sole colony in the East to an upcoming world power,
the United States. An accepted principle of international law dictated that a change in sovereignty,
while resulting in an abrogation of all political laws then in force, would have no effect on civil laws,
which would remain virtually intact.
The Treaty of Paris was entered into on 10 December 1898 between Spain and the United
States.21 Under Article IX of the treaty, the civil rights and political status of the native inhabitants of
the territories ceded to the United States would be determined by its Congress "Spanish subjects, natives of the Peninsula, residing in the territory over which Spain by the
present treaty relinquishes or cedes her sovereignty may remain in such territory or may
remove therefrom, retaining in either event all their rights of property, including the right to
sell or dispose of such property or of its proceeds; and they shall also have the right to carry
on their industry, commerce, and professions, being subject in respect thereof to such laws
as are applicable to foreigners. In case they remain in the territory they may preserve their
allegiance to the Crown of Spain by making, before a court of record, within a year from the
date of the exchange of ratifications of this treaty, a declaration of their decision to preserve
such allegiance; in default of which declaration they shall be held to have renounced it and to
have adopted the nationality of the territory in which they reside.
Thus

"The civil rights and political status of the native inhabitants of the territories hereby ceded to
the United States shall be determined by the Congress."22
Upon the ratification of the treaty, and pending legislation by the United States Congress on the
subject, the native inhabitants of the Philippines ceased to be Spanish subjects. Although they did
not become American citizens, they, however, also ceased to be "aliens" under American laws and
were thus issued passports describing them to be citizens of the Philippines entitled to the protection
of the United States.
The term "citizens of the Philippine Islands" appeared for the first time in the Philippine Bill of 1902,
also commonly referred to as the Philippine Organic Act of 1902, the first comprehensive legislation
of the Congress of the United States on the Philippines ".... that all inhabitants of the Philippine Islands continuing to reside therein, who were
Spanish subjects on the 11th day of April, 1891, and then resided in said Islands, and their
children born subsequent thereto, shall be deemed and held to be citizens of the Philippine
Islands and as such entitled to the protection of the United States, except such as shall have
elected to preserve their allegiance to the Crown of Spain in accordance with the provisions
of the treaty of peace between the United States and Spain, signed at Paris, December tenth
eighteen hundred and ninety eight."23
Under the organic act, a "citizen of the Philippines" was one who was an inhabitant of the
Philippines, and a Spanish subject on the 11th day of April 1899. The term "inhabitant" was taken to
include 1) a native-born inhabitant, 2) an inhabitant who was a native of Peninsular Spain, and 3) an
inhabitant who obtained Spanish papers on or before 11 April 1899. 24
Controversy arose on to the status of children born in the Philippines from 11 April 1899 to 01 July
1902, during which period no citizenship law was extant in the Philippines. Weight was given to the
view, articulated in jurisprudential writing at the time, that the common law principle of jus soli,
otherwise also known as the principle of territoriality, operative in the United States and England,
governed those born in the Philippine Archipelago within that period. 25 More about this later.
In 23 March 1912, the Congress of the United States made the following amendment to the
Philippine Bill of 1902 "Provided, That the Philippine Legislature is hereby authorized to provide by law for the
acquisition of Philippine citizenship by those natives of the Philippine Islands who do not
come within the foregoing provisions, the natives of other insular possession of the United
States, and such other persons residing in the Philippine Islands who would become citizens
of the United States, under the laws of the United States, if residing therein." 26
With the adoption of the Philippine Bill of 1902, the concept of "Philippine citizens" had for the first
time crystallized. The word "Filipino" was used by William H. Taft, the first Civil Governor General in
the Philippines when he initially made mention of it in his slogan, "The Philippines for the Filipinos."
In 1916, the Philippine Autonomy Act, also known as the Jones Law restated virtually the provisions
of the Philippine Bill of 1902, as so amended by the Act of Congress in 1912 "That all inhabitants of the Philippine Islands who were Spanish subjects on the eleventh day
of April, eighteen hundred and ninety-nine, and then resided in said Islands, and their
children born subsequently thereto, shall be deemed and held to be citizens of the Philippine
Islands, except such as shall have elected to preserve their allegiance to the Crown of Spain
in accordance with the provisions of the treaty of peace between the United States and

Spain, signed at Paris December tenth, eighteen hundred and ninety-eight and except such
others as have since become citizens of some other country; Provided, That the Philippine
Legislature, herein provided for, is hereby authorized to provide for the acquisition of
Philippine citizenship by those natives of the Philippine Islands who do not come within the
foregoing provisions, the natives of the insular possessions of the United States, and such
other persons residing in the Philippine Islands who are citizens of the United States, or who
could become citizens of the United States under the laws of the United States, if residing
therein."
Under the Jones Law, a native-born inhabitant of the Philippines was deemed to be a citizen of the
Philippines as of 11 April 1899 if he was 1) a subject of Spain on 11 April 1899, 2) residing in the
Philippines on said date, and, 3) since that date, not a citizen of some other country.
While there was, at one brief time, divergent views on whether or not jus soli was a mode of
acquiring citizenship, the 1935 Constitution brought to an end to any such link with common law, by
adopting, once and for all, jus sanguinis or blood relationship as being the basis of Filipino
citizenship "Section 1, Article III, 1935 Constitution. The following are citizens of the Philippines "(1) Those who are citizens of the Philippine Islands at the time of the adoption of this
Constitution
"(2) Those born in the Philippines Islands of foreign parents who, before the adoption of this
Constitution, had been elected to public office in the Philippine Islands.
"(3) Those whose fathers are citizens of the Philippines.
"(4) Those whose mothers are citizens of the Philippines and upon reaching the age of
majority, elect Philippine citizenship.
"(5) Those who are naturalized in accordance with law."
Subsection (4), Article III, of the 1935 Constitution, taken together with existing civil law provisions at
the time, which provided that women would automatically lose their Filipino citizenship and acquire
that of their foreign husbands, resulted in discriminatory situations that effectively incapacitated the
women from transmitting their Filipino citizenship to their legitimate children and required illegitimate
children of Filipino mothers to still elect Filipino citizenship upon reaching the age of majority.
Seeking to correct this anomaly, as well as fully cognizant of the newly found status of Filipino
women as equals to men, the framers of the 1973 Constitution crafted the provisions of the new
Constitution on citizenship to reflect such concerns "Section 1, Article III, 1973 Constitution - The following are citizens of the Philippines:
"(1) Those who are citizens of the Philippines at the time of the adoption of this Constitution.
"(2) Those whose fathers or mothers are citizens of the Philippines.
"(3) Those who elect Philippine citizenship pursuant to the provisions of the Constitution of
nineteen hundred and thirty-five.

"(4) Those who are naturalized in accordance with law."


For good measure, Section 2 of the same article also further provided that
"A female citizen of the Philippines who marries an alien retains her Philippine citizenship,
unless by her act or omission she is deemed, under the law to have renounced her
citizenship."
The 1987 Constitution generally adopted the provisions of the 1973 Constitution, except for
subsection (3) thereof that aimed to correct the irregular situation generated by the questionable
proviso in the 1935 Constitution.
Section I, Article IV, 1987 Constitution now provides:
"The following are citizens of the Philippines:
"(1) Those who are citizens of the Philippines at the time of the adoption of this
Constitution.
"(2) Those whose fathers or mothers are citizens of the Philippines.
"(3) Those born before January 17, 1973 of Filipino mothers, who elect Philippine
citizenship upon reaching the age of majority; and
"(4) Those who are naturalized in accordance with law."
The Case Of FPJ
Section 2, Article VII, of the 1987 Constitution expresses:
"No person may be elected President unless he is a natural-born citizen of the Philippines, a
registered voter, able to read and write, at least forty years of age on the day of the election,
and a resident of the Philippines for at least ten years immediately preceding such election."
The term "natural-born citizens," is defined to include "those who are citizens of the Philippines from
birth without having to perform any act to acquire or perfect their Philippine citizenship." 27
The date, month and year of birth of FPJ appeared to be 20 August 1939 during the regime of the
1935 Constitution. Through its history, four modes of acquiring citizenship - naturalization, jus soli,
res judicata and jus sanguinis28 had been in vogue. Only two, i.e., jus soli and jus sanguinis, could
qualify a person to being a "natural-born" citizen of the Philippines. Jus soli, per Roa vs. Collector of
Customs29 (1912), did not last long. With the adoption of the 1935 Constitution and the reversal of
Roa in Tan Chong vs. Secretary of Labor30 (1947), jus sanguinis or blood relationship would now
become the primary basis of citizenship by birth.
Documentary evidence adduced by petitioner would tend to indicate that the earliest established
direct ascendant of FPJ was his paternal grandfather Lorenzo Pou, married to Marta Reyes, the
father of Allan F. Poe. While the record of birth of Lorenzo Pou had not been presented in evidence,
his death certificate, however, identified him to be a Filipino, a resident of San Carlos, Pangasinan,
and 84 years old at the time of his death on 11 September 1954. The certificate of birth of the father
of FPJ, Allan F. Poe, showed that he was born on 17 May 1915 to an Espaol father, Lorenzo Pou,

and a mestiza Espaol mother, Marta Reyes. Introduced by petitioner was an "uncertified" copy of a
supposed certificate of the alleged marriage of Allan F. Poe and Paulita Gomez on 05 July 1936. The
marriage certificate of Allan F. Poe and Bessie Kelley reflected the date of their marriage to be on 16
September 1940. In the same certificate, Allan F. Poe was stated to be twenty-five years old,
unmarried, and a Filipino citizen, and Bessie Kelley to be twenty-two years old, unmarried, and an
American citizen. The birth certificate of FPJ, would disclose that he was born on 20 August 1939 to
Allan F. Poe, a Filipino, twenty-four years old, married to Bessie Kelly, an American citizen, twentyone years old and married.
Considering the reservations made by the parties on the veracity of some of the entries on the birth
certificate of respondent and the marriage certificate of his parents, the only conclusions that could
be drawn with some degree of certainty from the documents would be that 1. The parents of FPJ were Allan F. Poe and Bessie Kelley;
2. FPJ was born to them on 20 August 1939;
3. Allan F. Poe and Bessie Kelley were married to each other on 16 September, 1940;
4. The father of Allan F. Poe was Lorenzo Poe; and
5. At the time of his death on 11 September 1954, Lorenzo Poe was 84 years old.
Would the above facts be sufficient or insufficient to establish the fact that FPJ is a natural-born
Filipino citizen? The marriage certificate of Allan F. Poe and Bessie Kelley, the birth certificate of
FPJ, and the death certificate of Lorenzo Pou are documents of public record in the custody of a
public officer. The documents have been submitted in evidence by both contending parties during
the proceedings before the COMELEC.
The birth certificate of FPJ was marked Exhibit "A" for petitioner and Exhibit "3" for respondent. The
marriage certificate of Allan F. Poe to Bessie Kelley was submitted as Exhibit "21" for respondent.
The death certificate of Lorenzo Pou was submitted by respondent as his Exhibit "5." While the last
two documents were submitted in evidence for respondent, the admissibility thereof, particularly in
reference to the facts which they purported to show, i.e., the marriage certificate in relation to the
date of marriage of Allan F. Poe to Bessie Kelley and the death certificate relative to the death of
Lorenzo Pou on 11 September 1954 in San Carlos, Pangasinan, were all admitted by petitioner, who
had utilized those material statements in his argument. All three documents were certified true
copies of the originals.
Section 3, Rule 130, Rules of Court states that "Original document must be produced; exceptions. - When the subject of inquiry is the
contents of a document, no evidence shall be admissible other than the original document
itself, except in the following cases:
"x x x

xxx

xxx

"(d) When the original is a public record in the custody of a public office or is recorded in a
public office."

Being public documents, the death certificate of Lorenzo Pou, the marriage certificate of Allan F. Poe
and Bessie Kelly, and the birth certificate of FPJ, constitute prima facie proof of their contents.
Section 44, Rule 130, of the Rules of Court provides:
"Entries in official records. Entries in official records made in the performance of his duty by a
public officer of the Philippines, or by a person in the performance of a duty specially
enjoined by law, are prima facie evidence of the facts therein stated."
The trustworthiness of public documents and the value given to the entries made therein could be
grounded on 1) the sense of official duty in the preparation of the statement made, 2) the penalty
which is usually affixed to a breach of that duty, 3) the routine and disinterested origin of most such
statements, and 4) the publicity of record which makes more likely the prior exposure of such errors
as might have occurred.31
The death certificate of Lorenzo Pou would indicate that he died on 11 September 1954, at the age
of 84 years, in San Carlos, Pangasinan. It could thus be assumed that Lorenzo Pou was born
sometime in the year 1870 when the Philippines was still a colony of Spain. Petitioner would argue
that Lorenzo Pou was not in the Philippines during the crucial period of from 1898 to 1902
considering that there was no existing record about such fact in the Records Management and
Archives Office. Petitioner, however, likewise failed to show that Lorenzo Pou was at any other place
during the same period. In his death certificate, the residence of Lorenzo Pou was stated to be San
Carlos, Pangasinan. In the absence of any evidence to the contrary, it should be sound to conclude,
or at least to presume, that the place of residence of a person at the time of his death was also his
residence before death. It would be extremely doubtful if the Records Management and Archives
Office would have had complete records of all residents of the Philippines from 1898 to 1902.
Proof of Paternity and Filiation
Under Civil Law.
Petitioner submits, in any case, that in establishing filiation (relationship or civil status of the child to
the father [or mother]) or paternity (relationship or civil status of the father to the child) of an
illegitimate child, FPJ evidently being an illegitimate son according to petitioner, the mandatory rules
under civil law must be used.
Under the Civil Code of Spain, which was in force in the Philippines from 08 December 1889 up until
the day prior to 30 August 1950 when the Civil Code of the Philippines took effect, acknowledgment
was required to establish filiation or paternity. Acknowledgment was either judicial (compulsory) or
voluntary. Judicial or compulsory acknowledgment was possible only if done during the lifetime of
the putative parent; voluntary acknowledgment could only be had in a record of birth, a will, or a
public document.32 Complementary to the new code was Act No. 3753 or the Civil Registry Law
expressing in Section 5 thereof, that "In case of an illegitimate child, the birth certificate shall be signed and sworn to jointly by the
parents of the infant or only by the mother if the father refuses. In the latter case, it shall not
be permissible to state or reveal in the document the name of the father who refuses to
acknowledge the child, or to give therein any information by which such father could be
identified."
In order that the birth certificate could then be utilized to prove voluntary acknowledgment of filiation
or paternity, the certificate was required to be signed or sworn to by the father. The failure of such

requirement rendered the same useless as being an authoritative document of recognition. 33 In


Mendoza vs. Mella,34 the Court ruled "Since Rodolfo was born in 1935, after the registry law was enacted, the question here really
is whether or not his birth certificate (Exhibit 1), which is merely a certified copy of the
registry record, may be relied upon as sufficient proof of his having been voluntarily
recognized. No such reliance, in our judgment, may be placed upon it. While it contains the
names of both parents, there is no showing that they signed the original, let alone swore to
its contents as required in Section 5 of Act No. 3753. For all that might have happened, it
was not even they or either of them who furnished the data to be entered in the civil register.
Petitioners say that in any event the birth certificate is in the nature of a public document
wherein voluntary recognition of a natural child may also be made, according to the same
Article 131. True enough, but in such a case, there must be a clear statement in the
document that the parent recognizes the child as his or her own."
In the birth certificate of respondent FPJ, presented by both parties, nowhere in the document was
the signature of Allan F. Poe found. There being no will apparently executed, or at least shown to
have been executed, by decedent Allan F. Poe, the only other proof of voluntary recognition
remained to be "some other public document." In Pareja vs. Pareja,35 this Court defined what could
constitute such a document as proof of voluntary acknowledgment:
"Under the Spanish Civil Code there are two classes of public documents, those executed by
private individuals which must be authenticated by notaries, and those issued by competent
public officials by reason of their office. The public document pointed out in Article 131 as
one of the means by which recognition may be made belongs to the first class."
Let us leave it at that for the moment.
The 1950 Civil Code categorized the acknowledgment or recognition of illegitimate children into
voluntary, legal or compulsory. Voluntary recognition was required to be expressedly made in a
record of birth, a will, a statement before a court of record or in any authentic writing. Legal
acknowledgment took place in favor of full blood brothers and sisters of an illegitimate child who was
recognized or judicially declared as natural. Compulsory acknowledgment could be demanded
generally in cases when the child had in his favor any evidence to prove filiation. Unlike an action to
claim legitimacy which would last during the lifetime of the child, and might pass exceptionally to the
heirs of the child, an action to claim acknowledgment, however, could only be brought during the
lifetime of the presumed parent.
Amicus Curiae Ruben F. Balane defined, during the oral argument, "authentic writing," so as to be an
authentic writing for purposes of voluntary recognition, simply as being a genuine or indubitable
writing of the father. The term would include a public instrument (one duly acknowledged before a
notary public or other competent official) or a private writing admitted by the father to be his.
The Family Code has further liberalized the rules; Article 172, Article 173, and Article 175 provide:
"Art. 172. The filiation of legitimate children is established by any of the following:
"(1) The record of birth appearing in the civil register or a final judgment; or
"(2) An admission of legitimate filiation in a public document or a private handwritten
instrument and signed by the parent concerned.

"In the absence of the foregoing evidence, the legitimate filiation shall be proved by:
"(1) The open and continuous possession of the status of a legitimate child; or
"(2) Any other means allowed by the Rules of Court and special laws.
"Art. 173. The action to claim legitimacy may be brought by the child during his or her lifetime
and shall be transmitted to the heirs should the child die during minority or in a state of
insanity. In these cases, the heirs shall have a period of five years within which to institute
the action.
"The action already commenced by the child shall survive notwithstanding the death of either
or both of the parties.
"x x x

xxx

x x x.

"Art. 175. Illegitimate children may establish their illegitimate filiation in the same way and on
the same, evidence as legitimate children.
"The action must be brought within the same period specified in Article 173, except when the
action is based on the second paragraph of Article 172, in which case the action may be
brought during the lifetime of the alleged parent."
The provisions of the Family Code are retroactively applied; Article 256 of the code reads:
"Art. 256. This Code shall have retroactive effect insofar as it does not prejudice or impair
vested or acquired rights in accordance with the Civil Code or other laws."
Thus, in Vda. de Sy-Quia vs. Court of Appeals,36 the Court has ruled:
"We hold that whether Jose was a voluntarily recognized natural child should be decided
under Article 278 of the Civil Code of the Philippines. Article 2260 of that Code provides that
'the voluntary recognition of a natural child shall take place according to this Code, even if
the child was born before the effectivity of this body of laws' or before August 30, 1950.
Hence, Article 278 may be given retroactive effect."
It should be apparent that the growing trend to liberalize the acknowledgment or recognition of
illegitimate children is an attempt to break away from the traditional idea of keeping well apart
legitimate and non-legitimate relationships within the family in favor of the greater interest and
welfare of the child. The provisions are intended to merely govern the private and personal affairs of
the family. There is little, if any, to indicate that the legitimate or illegitimate civil status of the
individual would also affect his political rights or, in general, his relationship to the State. While,
indeed, provisions on "citizenship" could be found in the Civil Code, such provisions must be taken
in the context of private relations, the domain of civil law; particularly "Civil Law is that branch of law which has for its double purpose the organization of the
family and the regulation of property. It has thus [been] defined as the mass of precepts
which determine and regulate the relations of assistance, authority and obedience among
members of a family, and those which exist among members of a society for the protection of
private interests."37

In Yaez de Barnuevo vs. Fuster,38 the Court has held:


"In accordance with Article 9 of the Civil Code of Spain, x x x the laws relating to family rights
and duties, or to the status, condition and legal capacity of persons, govern Spaniards
although they reside in a foreign country; that, in consequence, 'all questions of a civil
nature, such as those dealing with the validity or nullity of the matrimonial bond, the domicile
of the husband and wife, their support, as between them, the separation of their properties,
the rules governing property, marital authority, division of conjugal property, the classification
of their property, legal causes for divorce, the extent of the latter, the authority to decree it,
and, in general, the civil effects of marriage and divorce upon the persons and properties of
the spouses, are questions that are governed exclusively by the national law of the husband
and wife."
The relevance of "citizenship" or "nationality" to Civil Law is best exemplified in Article 15 of the Civil
Code, stating that "Laws relating to family rights and duties, or to the status, condition and legal capacity of
persons are binding upon citizens of the Philippines, even though living abroad" that explains the need to incorporate in the code a reiteration of the Constitutional provisions on
citizenship. Similarly, citizenship is significant in civil relationships found in different parts of the Civil
Code,39 such as on successional rights and family relations.40 In adoption, for instance, an adopted
child would be considered the child of his adoptive parents and accorded the same rights as their
legitimate child but such legal fiction extended only to define his rights under civil law 41 and not his
political status.
Civil law provisions point to an obvious bias against illegitimacy. This discriminatory attitude may be
traced to the Spanish family and property laws, which, while defining proprietary and successional
rights of members of the family, provided distinctions in the rights of legitimate and illegitimate
children. In the monarchial set-up of old Spain, the distribution and inheritance of titles and wealth
were strictly according to bloodlines and the concern to keep these bloodlines uncontaminated by
foreign blood was paramount.
These distinctions between legitimacy and illegitimacy were codified in the Spanish Civil Code, and
the invidious discrimination survived when the Spanish Civil Code became the primary source of our
own Civil Code. Such distinction, however, remains and should remain only in the sphere of civil law
and not unduly impede or impinge on the domain of political law.
The proof of filiation or paternity for purposes of determining his citizenship status should thus be
deemed independent from and not inextricably tied up with that prescribed for civil law purposes.
The Civil Code or Family Code provisions on proof of filiation or paternity, although good law, do not
have preclusive effects on matters alien to personal and family relations. The ordinary rules on
evidence could well and should govern. For instance, the matter about pedigree is not necessarily
precluded from being applicable by the Civil Code or Family Code provisions.
Section 39, Rule 130, of the Rules of Court provides "Act or Declaration about pedigree. The act or declaration of a person deceased, or unable
to testify, in respect to the pedigree of another person related to him by birth or marriage,
may be received in evidence where it occurred before the controversy, and the relationship
between the two persons is shown by evidence other than such act or declaration. The word
`pedigree includes relationship, family genealogy, birth, marriage, death, the dates when and

the places where these facts occurred, and the names of the relatives. It embraces also facts
of family history intimately connected with pedigree."
For the above rule to apply, it would be necessary that (a) the declarant is already dead or unable to
testify, (b) the pedigree of a person must be at issue, (c) the declarant must be a relative of the
person whose pedigree is in question, (d) declaration must be made before the controversy has
occurred, and (e) the relationship between the declarant and the person whose pedigree is in
question must be shown by evidence other than such act or declaration.
Thus, the duly notarized declaration made by Ruby Kelley Mangahas, sister of Bessie Kelley Poe
submitted as Exhibit 20 before the COMELEC, might be accepted to prove the acts of Allan F. Poe,
recognizing his own paternal relationship with FPJ, i.e, living together with Bessie Kelley and his
children (including respondent FPJ) in one house, and as one family "I, Ruby Kelley Mangahas, of legal age and sound mind, presently residing in Stockton,
California, U.S.A., after being sworn in accordance with law do hereby declare that:
"1. I am the sister of the late Bessie Kelley Poe.
"2. Bessie Kelley Poe was the wife of Fernando Poe, Sr.
"3. Fernando and Bessie Poe had a son by the name of Ronald Allan Poe, more popularly
known in the Philippines as `Fernando Poe, Jr., or `FPJ.
"4. Ronald Allan Poe `FPJ was born on August 20, 1939 at St. Luke's Hospital, Magdalena
Street, Manila.
"x x x

xxx

xxx

"7. Fernando Poe Sr., and my sister Bessie, met and became engaged while they were
students at the University of the Philippines in 1936. I was also introduced to Fernando Poe,
Sr., by my sister that same year.
"8. Fernando Poe, Sr., and my sister Bessie had their first child in 1938.
"9. Fernando Poe, Sr., my sister Bessie and their first three children, Elizabeth, Ronald, Allan
and Fernando II, and myself lived together with our mother at our family's house on Dakota
St. (now Jorge Bocobo St.), Malate until the liberation of Manila in 1945, except for some
months between 1943-1944.
"10. Fernando Poe, Sr., and my sister, Bessie, were blessed with four (4) more children after
Ronald Allan Poe.
"x x x

xxx

xxx

"18. I am executing this Declaration to attest to the fact that my nephew, Ronald Allan Poe is
a natural born Filipino, and that he is the legitimate child of Fernando Poe, Sr.
"Done in City of Stockton, California, U.S.A., this 12th day of January 2004.
Ruby Kelley Mangahas Declarant DNA Testing

In case proof of filiation or paternity would be unlikely to satisfactorily establish or would be difficult to
obtain, DNA testing, which examines genetic codes obtained from body cells of the illegitimate child
and any physical residue of the long dead parent could be resorted to. A positive match would clear
up filiation or paternity. In Tijing vs. Court of Appeals,42 this Court has acknowledged the strong weight of
DNA testing "Parentage will still be resolved using conventional methods unless we adopt the modern and
scientific ways available. Fortunately, we have now the facility and expertise in using DNA test for
identification and parentage testing. The University of the Philippines Natural Science Research
Institute (UP-NSRI) DNA Analysis Laboratory has now the capability to conduct DNA typing using
short tandem repeat (STR) analysis. The analysis is based on the fact that the DNA of a child/person
has two (2) copies, one copy from the mother and the other from the father. The DNA from the
mother, the alleged father and the child are analyzed to establish parentage. Of course, being a
novel scientific technique, the use of DNA test as evidence is still open to challenge. Eventually, as
the appropriate case comes, courts should not hesitate to rule on the admissibility of DNA evidence.
For it was said, that courts should apply the results of science when competently obtained in aid of
situations presented, since to reject said result is to deny progress."
Petitioners Argument For Jurisprudential Conclusiveness
Petitioner would have it that even if Allan F. Poe were a Filipino citizen, he could not have
transmitted his citizenship to respondent FPJ, the latter being an illegitimate child. According to
petitioner, prior to his marriage to Bessie Kelley, Allan F. Poe, on July 5, 1936, contracted marriage
with a certain Paulita Gomez, making his subsequent marriage to Bessie Kelley bigamous and
respondent FPJ an illegitimate child. The veracity of the supposed certificate of marriage between
Allan F. Poe and Paulita Gomez could be most doubtful at best. But the documentary evidence
introduced by no less than respondent himself, consisting of a birth certificate of respondent and a
marriage certificate of his parents showed that FPJ was born on 20 August 1939 to a Filipino father
and an American mother who were married to each other a year later, or on 16 September 1940.
Birth to unmarried parents would make FPJ an illegitimate child. Petitioner contended that as an
illegitimate child, FPJ so followed the citizenship of his mother, Bessie Kelley, an American citizen,
basing his stand on the ruling of this Court in Morano vs. Vivo,43 citing Chiongbian vs. de Leo44 and
Serra vs. Republic.45
On the above score, the disquisition made by amicus curiae Joaquin G. Bernas, SJ, is most
convincing; he states "We must analyze these cases and ask what the lis mota was in each of them. If the
pronouncement of the Court on jus sanguinis was on the lis mota, the pronouncement would
be a decision constituting doctrine under the rule of stare decisis. But if the pronouncement
was irrelevant to the lis mota, the pronouncement would not be a decision but a mere obiter
dictum which did not establish doctrine. I therefore invite the Court to look closely into these
cases.
"First, Morano vs. Vivo. The case was not about an illegitimate child of a Filipino father. It
was about a stepson of a Filipino, a stepson who was the child of a Chinese mother and a
Chinese father. The issue was whether the stepson followed the naturalization of the
stepfather. Nothing about jus sanguinis there. The stepson did not have the blood of the
naturalized stepfather.
"Second, Chiongbian vs. de Leon. This case was not about the illegitimate son of a Filipino
father. It was about a legitimate son of a father who had become Filipino by election to public

office before the 1935 Constitution pursuant to Article IV, Section 1(2) of the 1935
Constitution. No one was illegitimate here.
"Third, Serra vs. Republic. The case was not about the illegitimate son of a Filipino father.
Serra was an illegitimate child of a Chinese father and a Filipino mother. The issue was
whether one who was already a Filipino because of his mother who still needed to be
naturalized. There is nothing there about invidious jus sanguinis.
"Finally, Paa vs. Chan.46 This is a more complicated case. The case was about the
citizenship of Quintin Chan who was the son of Leoncio Chan. Quintin Chan claimed that his
father, Leoncio, was the illegitimate son of a Chinese father and a Filipino mother. Quintin
therefore argued that he got his citizenship from Leoncio, his father. But the Supreme Court
said that there was no valid proof that Leoncio was in fact the son of a Filipina mother. The
Court therefore concluded that Leoncio was not Filipino. If Leoncio was not Filipino, neither
was his son Quintin. Quintin therefore was not only not a natural-born Filipino but was not
even a Filipino.
"The Court should have stopped there. But instead it followed with an obiter dictum. The
Court said obiter that even if Leoncio, Quintin's father, were Filipino, Quintin would not be
Filipino because Quintin was illegitimate. This statement about Quintin, based on a contrary
to fact assumption, was absolutely unnecessary for the case. x x x It was obiter dictum, pure
and simple, simply repeating the obiter dictum in Morano vs. Vivo.
"x x x

xxx

xxx

"Aside from the fact that such a pronouncement would have no textual foundation in the
Constitution, it would also violate the equal protection clause of the Constitution not once but
twice. First, it would make an illegitimate distinction between a legitimate child and an
illegitimate child, and second, it would make an illegitimate distinction between the
illegitimate child of a Filipino father and the illegitimate child of a Filipino mother.
"The doctrine on constitutionally allowable distinctions was established long ago by People
vs. Cayat.47 I would grant that the distinction between legitimate children and illegitimate
children rests on real differences. x x x But real differences alone do not justify invidious
distinction. Real differences may justify distinction for one purpose but not for another
purpose.
"x x x What is the relevance of legitimacy or illegitimacy to elective public service? What
possible state interest can there be for disqualifying an illegitimate child from becoming a
public officer. It was not the fault of the child that his parents had illicit liaison. Why deprive
the child of the fullness of political rights for no fault of his own? To disqualify an illegitimate
child from holding an important public office is to punish him for the indiscretion of his
parents. There is neither justice nor rationality in that. And if there is neither justice nor
rationality in the distinction, then the distinction transgresses the equal protection clause and
must be reprobated."
The other amici curiae, Mr. Justice Vicente Mendoza (a former member of this Court), Professor
Ruben Balane and Dean Martin Magallona, at bottom, have expressed similar views. The thesis of
petitioner, unfortunately hinging solely on pure obiter dicta, should indeed fail.
Where jurisprudence regarded an illegitimate child as taking after the citizenship of its mother, it did
so for the benefit the child. It was to ensure a Filipino nationality for the illegitimate child of an alien

father in line with the assumption that the mother had custody, would exercise parental authority and
had the duty to support her illegitimate child. It was to help the child, not to prejudice or discriminate
against him.
The fact of the matter perhaps the most significant consideration is that the 1935 Constitution,
the fundamental law prevailing on the day, month and year of birth of respondent FPJ, can never be
more explicit than it is. Providing neither conditions nor distinctions, the Constitution states that
among the citizens of the Philippines are "those whose fathers are citizens of the Philippines." There
utterly is no cogent justification to prescribe conditions or distinctions where there clearly are none
provided.
In Sum
(1) The Court, in the exercise of its power of judicial review, possesses jurisdiction over the
petition in G. R. No. 161824, filed under Rule 64, in relation to Rule 65, of the Revised Rules
of Civil Procedure. G.R. No. 161824 assails the resolution of the COMELEC for alleged
grave abuse of discretion in dismissing, for lack of merit, the petition in SPA No. 04-003
which has prayed for the disqualification of respondent FPJ from running for the position of
President in the 10th May 2004 national elections on the contention that FPJ has committed
material representation in his certificate of candidacy by representing himself to be a naturalborn citizen of the Philippines.
(2) The Court must dismiss, for lack of jurisdiction and prematurity, the petitions in G. R. No.
161434 and No. 161634 both having been directly elevated to this Court in the latters
capacity as the only tribunal to resolve a presidential and vice-presidential election contest
under the Constitution. Evidently, the primary jurisdiction of the Court can directly be invoked
only after, not before, the elections are held.
(3) In ascertaining, in G.R. No. 161824, whether grave abuse of discretion has been
committed by the COMELEC, it is necessary to take on the matter of whether or not
respondent FPJ is a natural-born citizen, which, in turn, depended on whether or not the
father of respondent, Allan F. Poe, would have himself been a Filipino citizen and, in the
affirmative, whether or not the alleged illegitimacy of respondent prevents him from taking
after the Filipino citizenship of his putative father. Any conclusion on the Filipino citizenship of
Lorenzo Pou could only be drawn from the presumption that having died in 1954 at 84 years
old, Lorenzo would have been born sometime in the year 1870, when the Philippines was
under Spanish rule, and that San Carlos, Pangasinan, his place of residence upon his death
in 1954, in the absence of any other evidence, could have well been his place of residence
before death, such that Lorenzo Pou would have benefited from the "en masse Filipinization"
that the Philippine Bill had effected in 1902. That citizenship (of Lorenzo Pou), if acquired,
would thereby extend to his son, Allan F. Poe, father of respondent FPJ. The 1935
Constitution, during which regime respondent FPJ has seen first light, confers citizenship to
all persons whose fathers are Filipino citizens regardless of whether such children are
legitimate or illegitimate.
(4) But while the totality of the evidence may not establish conclusively that respondent FPJ
is a natural-born citizen of the Philippines, the evidence on hand still would preponderate in
his favor enough to hold that he cannot be held guilty of having made a material
misrepresentation in his certificate of candidacy in violation of Section 78, in relation to
Section 74, of the Omnibus Election Code. Petitioner has utterly failed to substantiate his
case before the Court, notwithstanding the ample opportunity given to the parties to present
their position and evidence, and to prove whether or not there has been material

misrepresentation, which, as so ruled in Romualdez-Marcos vs. COMELEC, 48 must not only


be material, but also deliberate and willful.
WHEREFORE, the Court RESOLVES to DISMISS
1. G. R. No. 161434, entitled "Maria Jeanette C. Tecson and Felix B. Desiderio, Jr.,
Petitioners, versus Commission on Elections, Ronald Allan Kelley Poe (a.k.a. "Fernando
Poe, Jr.,) and Victorino X. Fornier, Respondents," and G. R. No. 161634, entitled "Zoilo
Antonio Velez, Petitioner, versus Ronald Allan Kelley Poe, a.k.a. Fernando Poe, Jr.,
Respondent," for want of jurisdiction.
2. G. R. No. 161824, entitled "Victorino X. Fornier, Petitioner, versus Hon. Commission on
Elections and Ronald Allan Kelley Poe, also known as Fernando Poe, Jr.," for failure to show
grave abuse of discretion on the part of respondent Commission on Elections in dismissing
the petition in SPA No. 04-003.
No Costs.
SO ORDERED.

Republic of the Philippines


SUPREME COURT
Manila
EN BANC
G.R. No. 137000

August 9, 2000

CIRILO R. VALLES, petitioner,


vs.
COMMISSION ON ELECTIONS and ROSALIND YBASCO LOPEZ, respondents.
DECISION
PURISIMA, J.:
This is a petition for certiorari under Rule 65, pursuant to Section 2, Rule 64 of the 1997 Rules of
Civil Procedure, assailing Resolutions dated July 17, 1998 and January 15, 1999, respectively, of the
Commission on Elections in SPA No. 98-336, dismissing the petition for disqualification filed by the
herein petitioner, Cirilo R. Valles, against private respondent Rosalind Ybasco Lopez, in the May
1998 elections for governor of Davao Oriental.
Rosalind Ybasco Lopez was born on May 16, 1934 in Napier Terrace, Broome, Western Australia, to
the spouses, Telesforo Ybasco, a Filipino citizen and native of Daet, Camarines Norte, and Theresa
Marquez, an Australian. In 1949, at the age of fifteen, she left Australia and came to settle in the
Philippines.

On June 27, 1952, she was married to Leopoldo Lopez, a Filipino citizen, at the Malate Catholic
Church in Manila. Since then, she has continuously participated in the electoral process not only as
a voter but as a candidate, as well. She served as Provincial Board Member of the Sangguniang
Panlalawigan of Davao Oriental. In 1992, she ran for and was elected governor of Davao Oriental.
Her election was contested by her opponent, Gil Taojo, Jr., in a petition for quo warranto, docketed
as EPC No. 92-54, alleging as ground therefor her alleged Australian citizenship. However, finding
no sufficient proof that respondent had renounced her Philippine citizenship, the Commission on
Elections en banc dismissed the petition, ratiocinating thus:
"A cursory reading of the records of this case vis-a-vis the impugned resolution shows that
respondent was able to produce documentary proofs of the Filipino citizenship of her late father...
and consequently, prove her own citizenship and filiation by virtue of the Principle of Jus Sanguinis,
the perorations of the petitioner to the contrary notwithstanding.
On the other hand, except for the three (3) alleged important documents . . . no other evidence
substantial in nature surfaced to confirm the allegations of petitioner that respondent is an Australian
citizen and not a Filipino. Express renunciation of citizenship as a mode of losing citizenship under
Commonwealth Act No. 63 is an equivocal and deliberate act with full awareness of its significance
and consequence. The evidence adduced by petitioner are inadequate, nay meager, to prove that
respondent contemplated renunciation of her Filipino citizenship". 1
In the 1995 local elections, respondent Rosalind Ybasco Lopez ran for re-election as governor of
Davao Oriental. Her opponent, Francisco Rabat, filed a petition for disqualification, docketed as SPA
No. 95-066 before the COMELEC, First Division, contesting her Filipino citizenship but the said
petition was likewise dismissed by the COMELEC, reiterating substantially its decision in EPC 92-54.
The citizenship of private respondent was once again raised as an issue when she ran for reelection as governor of Davao Oriental in the May 11, 1998 elections. Her candidacy was questioned
by the herein petitioner, Cirilo Valles, in SPA No. 98-336.
On July 17, 1998, the COMELECs First Division came out with a Resolution dismissing the petition,
and disposing as follows:
"Assuming arguendo that res judicata does not apply and We are to dispose the instant case on the
merits trying it de novo, the above table definitely shows that petitioner herein has presented no new
evidence to disturb the Resolution of this Commission in SPA No. 95-066. The present petition
merely restates the same matters and incidents already passed upon by this Commission not just in
1995 Resolution but likewise in the Resolution of EPC No. 92-54. Not having put forth any new
evidence and matter substantial in nature, persuasive in character or sufficiently provocative to
compel reversal of such Resolutions, the dismissal of the present petition follows as a matter of
course.
xxx

xxx

xxx

"WHEREFORE, premises considered and there being no new matters and issues tendered, We find
no convincing reason or impressive explanation to disturb and reverse the Resolutions promulgated

by this Commission in EPC 92-54 and SPA. 95-066. This Commission RESOLVES as it hereby
RESOLVES to DISMISS the present petition.
SO ORDERED."2
Petitioner interposed a motion for reconsideration of the aforesaid Resolution but to no avail. The
same was denied by the COMELEC in its en banc Resolution of January 15, 1999.
Undaunted, petitioner found his way to this Court via the present petition; questioning the citizenship
of private respondent Rosalind Ybasco Lopez.
The Commission on Elections ruled that private respondent Rosalind Ybasco Lopez is a Filipino
citizen and therefore, qualified to run for a public office because (1) her father, Telesforo Ybasco, is a
Filipino citizen, and by virtue of the principle of jus sanguinis she was a Filipino citizen under the
1987 Philippine Constitution; (2) she was married to a Filipino, thereby making her also a Filipino
citizen ipso jure under Section 4 of Commonwealth Act 473; (3) and that, she renounced her
Australian citizenship on January 15, 1992 before the Department of Immigration and Ethnic Affairs
of Australia and her Australian passport was accordingly cancelled as certified to by the Australian
Embassy in Manila; and (4) furthermore, there are the COMELEC Resolutions in EPC No. 92-54 and
SPA Case No. 95-066, declaring her a Filipino citizen duly qualified to run for the elective position of
Davao Oriental governor.
Petitioner, on the other hand, maintains that the private respondent is an Australian citizen, placing
reliance on the admitted facts that:
a) In 1988, private respondent registered herself with the Bureau of Immigration as an
Australian national and was issued Alien Certificate of Registration No. 404695 dated
September 19, 1988;
b) On even date, she applied for the issuance of an Immigrant Certificate of Residence
(ICR), and
c) She was issued Australian Passport No. H700888 on March 3, 1988.
Petitioner theorizes that under the aforestated facts and circumstances, the private respondent had
renounced her Filipino citizenship. He contends that in her application for alien certificate of
registration and immigrant certificate of residence, private respondent expressly declared under oath
that she was a citizen or subject of Australia; and said declaration forfeited her Philippine citizenship,
and operated to disqualify her to run for elective office.
As regards the COMELECs finding that private respondent had renounced her Australian citizenship
on January 15, 1992 before the Department of Immigration and Ethnic Affairs of Australia and had
her Australian passport cancelled on February 11, 1992, as certified to by the Australian Embassy
here in Manila, petitioner argues that the said acts did not automatically restore the status of private
respondent as a Filipino citizen. According to petitioner, for the private respondent to reacquire
Philippine citizenship she must comply with the mandatory requirements for repatriation under

Republic Act 8171; and the election of private respondent to public office did not mean the
restoration of her Filipino citizenship since the private respondent was not legally repatriated.
Coupled with her alleged renunciation of Australian citizenship, private respondent has effectively
become a stateless person and as such, is disqualified to run for a public office in the Philippines;
petitioner concluded.
Petitioner theorizes further that the Commission on Elections erred in applying the principle of res
judicata to the case under consideration; citing the ruling in Moy Ya Lim Yao vs. Commissioner of
Immigration,3 that:
"xxx Everytime the citizenship of a person is material or indispensable in a judicial or administrative
case, whatever the corresponding court or administrative authority decides therein as to such
citizenship is generally not considered as res adjudicata, hence it has to be threshed out again and
again as the occasion may demand. xxx"
The petition is unmeritorious.
The Philippine law on citizenship adheres to the principle of jus sanguinis. Thereunder, a child
follows the nationality or citizenship of the parents regardless of the place of his/her birth, as
opposed to the doctrine of jus soli which determines nationality or citizenship on the basis of place of
birth.
Private respondent Rosalind Ybasco Lopez was born on May 16, 1934 in Napier Terrace, Broome,
Western Australia, to the spouses, Telesforo Ybasco, a Filipino citizen and native of Daet, Camarines
Norte, and Theresa Marquez, an Australian. Historically, this was a year before the 1935 Constitution
took into effect and at that time, what served as the Constitution of the Philippines were the principal
organic acts by which the United States governed the country. These were the Philippine Bill of July
1, 1902 and the Philippine Autonomy Act of August 29, 1916, also known as the Jones Law.
Among others, these laws defined who were deemed to be citizens of the Philippine islands. The
Philippine Bill of 1902 defined Philippine citizens as:
SEC. 4 xxx all inhabitants of the Philippine Islands continuing to reside therein who were Spanish
subjects on the eleventh day of April, eighteen hundred and ninety-nine, and then resided in the
Philippine Islands, and their children born subsequent thereto, shall be deemed and held to be
citizens of the Philippine Islands and as such entitled to the protection of the United States, except
such as shall have elected to preserve their allegiance to the Crown of Spain in accordance with the
provisions of the treaty of peace between the United States and Spain signed at Paris December
tenth, eighteen hundred and ninety-eight. (underscoring ours)
The Jones Law, on the other hand, provides:
SEC. 2 That all inhabitants of the Philippine Islands who were Spanish subjects on the eleventh day
of April, eighteen hundred and ninety-nine, and then resided in said Islands, and their children born
subsequent thereto, shall be deemed and held to be citizens of the Philippine Islands, except such
as shall have elected to preserve their allegiance to the Crown of Spain in accordance with the

provisions of the treaty of peace between the United States and Spain, signed at Paris December
tenth, eighteen hundred and ninety-eight, and except such others as have since become citizens of
some other country: Provided, That the Philippine Legislature, herein provided for, is hereby
authorized to provide by law for the acquisition of Philippine citizenship by those natives of the
Philippine Islands who cannot come within the foregoing provisions, the natives of the insular
possessions of the United States, and such other persons residing in the Philippine Islands who are
citizens of the United States, or who could become citizens of the United States under the laws of
the United States if residing therein. (underscoring ours)
Under both organic acts, all inhabitants of the Philippines who were Spanish subjects on April 11,
1899 and resided therein including their children are deemed to be Philippine citizens. Private
respondents father, Telesforo Ybasco, was born on January 5, 1879 in Daet, Camarines Norte, a
fact duly evidenced by a certified true copy of an entry in the Registry of Births. Thus, under the
Philippine Bill of 1902 and the Jones Law, Telesforo Ybasco was deemed to be a Philippine citizen.
By virtue of the same laws, which were the laws in force at the time of her birth, Telesforos daughter,
herein private respondent Rosalind Ybasco Lopez, is likewise a citizen of the Philippines.
The signing into law of the 1935 Philippine Constitution has established the principle of jus
sanguinis as basis for the acquisition of Philippine citizenship, to wit:
(1) Those who are citizens of the Philippine Islands at the time of the adoption of this
Constitution.
(2) Those born in the Philippine Islands of foreign parents who, before the adoption of this
Constitution had been elected to public office in the Philippine Islands.
(3) Those whose fathers are citizens of the Philippines.
(4) Those whose mothers are citizens of the Philippines and, upon reaching the age of
majority, elect Philippine citizenship.
(5) Those who are naturalized in accordance with law.
So also, the principle of jus sanguinis, which confers citizenship by virtue of blood relationship, was
subsequently retained under the 19734 and 19875 Constitutions. Thus, the herein private respondent,
Rosalind Ybasco Lopez, is a Filipino citizen, having been born to a Filipino father. The fact of her
being born in Australia is not tantamount to her losing her Philippine citizenship. If Australia follows
the principle of jus soli, then at most, private respondent can also claim Australian citizenship
resulting to her possession of dual citizenship.
Petitioner also contends that even on the assumption that the private respondent is a Filipino citizen,
she has nonetheless renounced her Philippine citizenship. To buttress this contention, petitioner
cited private respondents application for an Alien Certificate of Registration (ACR) and Immigrant
Certificate of Residence (ICR), on September 19, 1988, and the issuance to her of an Australian
passport on March 3, 1988.

Under Commonwealth Act No. 63, a Filipino citizen may lose his citizenship:
(1) By naturalization in a foreign country;
(2) By express renunciation of citizenship;
(3) By subscribing to an oath of allegiance to support the constitution or laws of a foreign
country upon attaining twenty-one years of age or more;
(4) By accepting commission in the military, naval or air service of a foreign country;
(5) By cancellation of the certificate of naturalization;
(6) By having been declared by competent authority, a deserter of the Philippine armed
forces in time of war, unless subsequently, a plenary pardon or amnesty has been granted:
and
(7) In case of a woman, upon her marriage, to a foreigner if, by virtue of the laws in force in
her husbands country, she acquires his nationality.
In order that citizenship may be lost by renunciation, such renunciation must be express. Petitioners
contention that the application of private respondent for an alien certificate of registration, and her
Australian passport, is bereft of merit. This issue was put to rest in the case of Aznar vs.
COMELEC6 and in the more recent case ofMercado vs. Manzano and COMELEC.7
In the case of Aznar, the Court ruled that the mere fact that respondent Osmena was a holder of a
certificate stating that he is an American did not mean that he is no longer a Filipino, and that an
application for an alien certificate of registration was not tantamount to renunciation of his Philippine
citizenship.
And, in Mercado vs. Manzano and COMELEC, it was held that the fact that respondent Manzano
was registered as an American citizen in the Bureau of Immigration and Deportation and was holding
an American passport on April 22, 1997, only a year before he filed a certificate of candidacy for
vice-mayor of Makati, were just assertions of his American nationality before the termination of his
American citizenship.
Thus, the mere fact that private respondent Rosalind Ybasco Lopez was a holder of an Australian
passport and had an alien certificate of registration are not acts constituting an effective renunciation
of citizenship and do not militate against her claim of Filipino citizenship. For renunciation to
effectively result in the loss of citizenship, the same must be express.8 As held by this court in the
aforecited case of Aznar, an application for an alien certificate of registration does not amount to an
express renunciation or repudiation of ones citizenship. The application of the herein private
respondent for an alien certificate of registration, and her holding of an Australian passport, as in the
case of Mercado vs. Manzano, were mere acts of assertion of her Australian citizenship before she
effectively renounced the same. Thus, at the most, private respondent had dual citizenship - she was
an Australian and a Filipino, as well.

Moreover, under Commonwealth Act 63, the fact that a child of Filipino parent/s was born in another
country has not been included as a ground for losing ones Philippine citizenship. Since private
respondent did not lose or renounce her Philippine citizenship, petitioners claim that respondent
must go through the process of repatriation does not hold water.
Petitioner also maintains that even on the assumption that the private respondent had dual
citizenship, still, she is disqualified to run for governor of Davao Oriental; citing Section 40 of
Republic Act 7160 otherwise known as the Local Government Code of 1991, which states:
"SEC. 40. Disqualifications. The following persons are disqualified from running for any elective local
position:
xxx

xxx

xxx

xxx

xxx

xxx

(d) Those with dual citizenship;

Again, petitioners contention is untenable.


In the aforecited case of Mercado vs. Manzano, the Court clarified "dual citizenship" as used in the
Local Government Code and reconciled the same with Article IV, Section 5 of the 1987 Constitution
on dual allegiance.9Recognizing situations in which a Filipino citizen may, without performing any
act, and as an involuntary consequence of the conflicting laws of different countries, be also a citizen
of another state, the Court explained that dual citizenship as a disqualification must refer to citizens
with dual allegiance. The Court succinctly pronounced:
"xxx the phrase dual citizenship in R.A. No. 7160, xxx 40 (d) and in R.A. No. 7854, xxx 20 must be
understood as referring to dual allegiance. Consequently, persons with mere dual citizenship do not
fall under this disqualification."
Thus, the fact that the private respondent had dual citizenship did not automatically disqualify her
from running for a public office. Furthermore, it was ruled that for candidates with dual citizenship, it
is enough that they elect Philippine citizenship upon the filing of their certificate of candidacy, to
terminate their status as persons with dual citizenship.10 The filing of a certificate of candidacy
sufficed to renounce foreign citizenship, effectively removing any disqualification as a dual
citizen.11 This is so because in the certificate of candidacy, one declares that he/she is a Filipino
citizen and that he/she will support and defend the Constitution of the Philippines and will maintain
true faith and allegiance thereto. Such declaration, which is under oath, operates as an effective
renunciation of foreign citizenship. Therefore, when the herein private respondent filed her certificate
of candidacy in 1992, such fact alone terminated her Australian citizenship.
Then, too, it is significant to note that on January 15 1992, private respondent executed a
Declaration of Renunciation of Australian Citizenship, duly registered in the Department of
Immigration and Ethnic Affairs of Australia on May 12, 1992. And, as a result, on February 11, 1992,
the Australian passport of private respondent was cancelled, as certified to by Second Secretary

Richard F. Munro of the Embassy of Australia in Manila. As aptly appreciated by the COMELEC, the
aforesaid acts were enough to settle the issue of the alleged dual citizenship of Rosalind Ybasco
Lopez. Since her renunciation was effective, petitioners claim that private respondent must go
through the whole process of repatriation holds no water.
Petitioner maintains further that when citizenship is raised as an issue in judicial or administrative
proceedings, the resolution or decision thereon is generally not considered res judicata in any
subsequent proceeding challenging the same; citing the case of Moy Ya Lim Yao vs. Commissioner
of Immigration.12 He insists that the same issue of citizenship may be threshed out anew.
Petitioner is correct insofar as the general rule is concerned, i.e. the principle of res
judicata generally does not apply in cases hinging on the issue of citizenship. However, in the case
of Burca vs. Republic,13 an exception to this general rule was recognized. The Court ruled in that
case that in order that the doctrine of res judicata may be applied in cases of citizenship, the
following must be present:
1) a persons citizenship be raised as a material issue in a controversy where said person is
a party;
2) the Solicitor General or his authorized representative took active part in the resolution
thereof, and
3) the finding on citizenship is affirmed by this Court.
Although the general rule was set forth in the case of Moy Ya Lim Yao, the case did not foreclose the
weight of prior rulings on citizenship. It elucidated that reliance may somehow be placed on these
antecedent official findings, though not really binding, to make the effort easier or simpler.14 Indeed,
there appears sufficient basis to rely on the prior rulings of the Commission on Elections in SPA. No.
95-066 and EPC 92-54 which resolved the issue of citizenship in favor of the herein private
respondent. The evidence adduced by petitioner is substantially the same evidence presented in
these two prior cases. Petitioner failed to show any new evidence or supervening event to warrant a
reversal of such prior resolutions. However, the procedural issue notwithstanding, considered on the
merits, the petition cannot prosper.
WHEREFORE, the petition is hereby DISMISSED and the COMELEC Resolutions, dated July 17,
1998 and January 15, 1999, respectively, in SPA No. 98-336 AFFIRMED.
Private respondent Rosalind Ybasco Lopez is hereby adjudged qualified to run for governor of
Davao Oriental. No pronouncement as to costs.
SO ORDERED.

Republic of the Philippines


SUPREME COURT
Manila

EN BANC
G.R. No. 160869

May 11, 2007

AASJS (ADVOCATES AND ADHERENTS OF SOCIAL JUSTICE FOR SCHOOL TEACHERS AND
ALLIED WORKERS) MEMBER - HECTOR GUMANGAN CALILUNG, Petitioner,
vs.
THE HONORABLE SIMEON DATUMANONG, in his official capacity as the Secretary of
Justice,Respondent.
DECISION
QUISUMBING, J.:
This is an original action for prohibition under Rule 65 of the 1997 Revised Rules of Civil Procedure.
Petitioner filed the instant petition against respondent, then Secretary of Justice Simeon
Datumanong, the official tasked to implement laws governing citizenship. 1 Petitioner prays that a writ
of prohibition be issued to stop respondent from implementing Republic Act No. 9225, entitled "An
Act Making the Citizenship of Philippine Citizens Who Acquire Foreign Citizenship Permanent,
Amending for the Purpose Commonwealth Act No. 63, As Amended, and for Other Purposes."
Petitioner avers that Rep. Act No. 9225 is unconstitutional as it violates Section 5, Article IV of the
1987 Constitution that states, "Dual allegiance of citizens is inimical to the national interest and shall
be dealt with by law."
Rep. Act No. 9225, signed into law by President Gloria M. Arroyo on August 29, 2003, reads:
SECTION 1. Short Title.-This Act shall be known as the "Citizenship Retention and Reacquisition Act
of 2003."
SEC. 2. Declaration of Policy.-It is hereby declared the policy of the State that all Philippine citizens
who become citizens of another country shall be deemed not to have lost their Philippine citizenship
under the conditions of this Act.
SEC. 3. Retention of Philippine Citizenship.-Any provision of law to the contrary notwithstanding,
natural-born citizens of the Philippines who have lost their Philippine citizenship by reason of their
naturalization as citizens of a foreign country are hereby deemed to have reacquired Philippine
citizenship upon taking the following oath of allegiance to the Republic:
"I ___________________________, solemnly swear (or affirm) that I will support and defend the
Constitution of the Republic of the Philippines and obey the laws and legal orders promulgated by
the duly constituted authorities of the Philippines; and I hereby declare that I recognize and accept
the supreme authority of the Philippines and will maintain true faith and allegiance thereto; and that I
impose this obligation upon myself voluntarily without mental reservation or purpose of evasion."

Natural-born citizens of the Philippines who, after the effectivity of this Act, become citizens of a
foreign country shall retain their Philippine citizenship upon taking the aforesaid oath.
SEC. 4. Derivative Citizenship. - The unmarried child, whether legitimate, illegitimate or adopted,
below eighteen (18) years of age, of those who reacquire Philippine citizenship upon effectivity of
this Act shall be deemed citizens of the Philippines.
SEC. 5. Civil and Political Rights and Liabilities. - Those who retain or reacquire Philippine
citizenship under this Act shall enjoy full civil and political rights and be subject to all attendant
liabilities and responsibilities under existing laws of the Philippines and the following conditions:
(1) Those intending to exercise their right of suffrage must meet the requirements under
Section 1, Article V of the Constitution, Republic Act No. 9189, otherwise known as "The
Overseas Absentee Voting Act of 2003" and other existing laws;
(2) Those seeking elective public office in the Philippines shall meet the qualifications for
holding such public office as required by the Constitution and existing laws and, at the time
of the filing of the certificate of candidacy, make a personal and sworn renunciation of any
and all foreign citizenship before any public officer authorized to administer an oath;
(3) Those appointed to any public office shall subscribe and swear to an oath of allegiance to
the Republic of the Philippines and its duly constituted authorities prior to their assumption of
office: Provided, That they renounce their oath of allegiance to the country where they took
that oath;
(4) Those intending to practice their profession in the Philippines shall apply with the proper
authority for a license or permit to engage in such practice; and
(5) That right to vote or be elected or appointed to any public office in the Philippines cannot
be exercised by, or extended to, those who:
(a) are candidates for or are occupying any public office in the country of which they are
naturalized citizens; and/or
(b) are in the active service as commissioned or noncommissioned officers in the armed
forces of the country which they are naturalized citizens.
SEC. 6. Separability Clause. - If any section or provision of this Act is held unconstitutional or invalid,
any other section or provision not affected thereby shall remain valid and effective.
SEC. 7. Repealing Clause. - All laws, decrees, orders, rules and regulations inconsistent with the
provisions of this Act are hereby repealed or modified accordingly.
SEC. 8. Effectivity Clause. - This Act shall take effect after fifteen (15) days following its publication
in the Official Gazette or two (2) newspapers of general circulation.

In this petition for prohibition, the following issues have been raised: (1) Is Rep. Act No. 9225
unconstitutional? (2) Does this Court have jurisdiction to pass upon the issue of dual allegiance?
We shall discuss these issues jointly.
Petitioner contends that Rep. Act No. 9225 cheapens Philippine citizenship. He avers that Sections 2
and 3 of Rep. Act No. 9225, together, allow dual allegiance and not dual citizenship. Petitioner
maintains that Section 2 allows all Filipinos, either natural-born or naturalized, who become foreign
citizens, to retain their Philippine citizenship without losing their foreign citizenship. Section 3 permits
dual allegiance because said law allows natural-born citizens of the Philippines to regain their
Philippine citizenship by simply taking an oath of allegiance without forfeiting their foreign
allegiance.2 The Constitution, however, is categorical that dual allegiance is inimical to the national
interest.
The Office of the Solicitor General (OSG) claims that Section 2 merely declares as a state policy that
"Philippine citizens who become citizens of another country shall be deemed not to have lost their
Philippine citizenship." The OSG further claims that the oath in Section 3 does not allow dual
allegiance since the oath taken by the former Filipino citizen is an effective renunciation and
repudiation of his foreign citizenship. The fact that the applicant taking the oath recognizes and
accepts the supreme authority of the Philippines is an unmistakable and categorical affirmation of his
undivided loyalty to the Republic.3
In resolving the aforecited issues in this case, resort to the deliberations of Congress is necessary to
determine the intent of the legislative branch in drafting the assailed law. During the deliberations,
the issue of whether Rep. Act No. 9225 would allow dual allegiance had in fact been the subject of
debate. The record of the legislative deliberations reveals the following:
xxxx
Pursuing his point, Rep. Dilangalen noted that under the measure, two situations exist - - the
retention of foreign citizenship, and the reacquisition of Philippine citizenship. In this case, he
observed that there are two citizenships and therefore, two allegiances. He pointed out that under
the Constitution, dual allegiance is inimical to public interest. He thereafter asked whether with the
creation of dual allegiance by reason of retention of foreign citizenship and the reacquisition of
Philippine citizenship, there will now be a violation of the Constitution
Rep. Locsin underscored that the measure does not seek to address the constitutional injunction on
dual allegiance as inimical to public interest. He said that the proposed law aims to facilitate the
reacquisition of Philippine citizenship by speedy means. However, he said that in one sense, it
addresses the problem of dual citizenship by requiring the taking of an oath. He explained that the
problem of dual citizenship is transferred from the Philippines to the foreign country because the
latest oath that will be taken by the former Filipino is one of allegiance to the Philippines and not to
the United States, as the case may be. He added that this is a matter which the Philippine
government will have no concern and competence over.

Rep. Dilangalen asked why this will no longer be the country's concern, when dual allegiance is
involved.
Rep. Locsin clarified that this was precisely his objection to the original version of the bill, which did
not require an oath of allegiance. Since the measure now requires this oath, the problem of dual
allegiance is transferred from the Philippines to the foreign country concerned, he explained.
xxxx
Rep. Dilangalen asked whether in the particular case, the person did not denounce his foreign
citizenship and therefore still owes allegiance to the foreign government, and at the same time, owes
his allegiance to the Philippine government, such that there is now a case of dual citizenship and
dual allegiance.
Rep. Locsin clarified that by swearing to the supreme authority of the Republic, the person implicitly
renounces his foreign citizenship. However, he said that this is not a matter that he wishes to
address in Congress because he is not a member of a foreign parliament but a Member of the
House.
xxxx
Rep. Locsin replied that it is imperative that those who have dual allegiance contrary to national
interest should be dealt with by law. However, he said that the dual allegiance problem is not
addressed in the bill. He then cited the Declaration of Policy in the bill which states that "It is hereby
declared the policy of the State that all citizens who become citizens of another country shall be
deemed not to have lost their Philippine citizenship under the conditions of this Act." He stressed that
what the bill does is recognize Philippine citizenship but says nothing about the other citizenship.
Rep. Locsin further pointed out that the problem of dual allegiance is created wherein a natural-born
citizen of the Philippines takes an oath of allegiance to another country and in that oath says that he
abjures and absolutely renounces all allegiance to his country of origin and swears allegiance to that
foreign country. The original Bill had left it at this stage, he explained. In the present measure, he
clarified, a person is required to take an oath and the last he utters is one of allegiance to the
country. He then said that the problem of dual allegiance is no longer the problem of the Philippines
but of the other foreign country.4 (Emphasis supplied.)
From the above excerpts of the legislative record, it is clear that the intent of the legislature in
drafting Rep. Act No. 9225 is to do away with the provision in Commonwealth Act No. 63 5 which
takes away Philippine citizenship from natural-born Filipinos who become naturalized citizens of
other countries. What Rep. Act No. 9225 does is allow dual citizenship to natural-born Filipino
citizens who have lost Philippine citizenship by reason of their naturalization as citizens of a foreign
country. On its face, it does not recognize dual allegiance. By swearing to the supreme authority of
the Republic, the person implicitly renounces his foreign citizenship. Plainly, from Section 3, Rep. Act
No. 9225 stayed clear out of the problem of dual allegiance and shifted the burden of confronting the
issue of whether or not there is dual allegiance to the concerned foreign country. What happens to
the other citizenship was not made a concern of Rep. Act No. 9225.

Petitioner likewise advances the proposition that although Congress has not yet passed any law on
the matter of dual allegiance, such absence of a law should not be justification why this Court could
not rule on the issue. He further contends that while it is true that there is no enabling law yet on dual
allegiance, the Supreme Court, through Mercado v. Manzano, 6 already had drawn up the guidelines
on how to distinguish dual allegiance from dual citizenship. 7
For its part, the OSG counters that pursuant to Section 5, Article IV of the 1987 Constitution, dual
allegiance shall be dealt with by law. Thus, until a law on dual allegiance is enacted by Congress,
the Supreme Court is without any jurisdiction to entertain issues regarding dual allegiance. 8
To begin with, Section 5, Article IV of the Constitution is a declaration of a policy and it is not a selfexecuting provision. The legislature still has to enact the law on dual allegiance. In Sections 2 and 3
of Rep. Act No. 9225, the framers were not concerned with dual citizenship per se, but with the
status of naturalized citizens who maintain their allegiance to their countries of origin even after their
naturalization.9 Congress was given a mandate to draft a law that would set specific parameters of
what really constitutes dual allegiance.10 Until this is done, it would be premature for the judicial
department, including this Court, to rule on issues pertaining to dual allegiance.
Neither can we subscribe to the proposition of petitioner that a law is not needed since the case of
Mercado had already set the guidelines for determining dual allegiance. Petitioner
misreads Mercado. That case did not set the parameters of what constitutes dual allegiance but
merely made a distinction between dual allegiance and dual citizenship.
Moreover, in Estrada v. Sandiganbayan,11 we said that the courts must assume that the legislature is
ever conscious of the borders and edges of its plenary powers, and passed laws with full knowledge
of the facts and for the purpose of promoting what is right and advancing the welfare of the majority.
Hence, in determining whether the acts of the legislature are in tune with the fundamental law, we
must proceed with judicial restraint and act with caution and forbearance. 12 The doctrine of
separation of powers demands no less. We cannot arrogate the duty of setting the parameters of
what constitutes dual allegiance when the Constitution itself has clearly delegated the duty of
determining what acts constitute dual allegiance for study and legislation by Congress.
WHEREFORE, the petition is hereby DISMISSED for lack of merit.
SO ORDERED.
Republic of the Philippines
SUPREME COURT
Manila
EN BANC
G.R. No. 162759 August 4, 2006
LOIDA NICOLAS-LEWIS, GREGORIO B. MACABENTA, ALEJANDRO A. ESCLAMADO,
ARMANDO B. HEREDIA, REUBEN S. SEGURITAN, ERIC LACHICA FURBEYRE, TERESITA A.

CRUZ, JOSEFINA OPENA DISTERHOFT, MERCEDES V. OPENA, CORNELIO R. NATIVIDAD,


EVELYN D. NATIVIDAD, Petitioners,
vs.
COMMISSION ON ELECTIONS, Respondent.
DECISION
GARCIA, J.:
In this petition for certiorari and mandamus, petitioners, referring to themselves as "duals" or dual
citizens, pray that they and others who retained or reacquired Philippine citizenship under Republic
Act (R.A.) No. 9225, the Citizenship Retention and Re-Acquisition Act of 2003, be allowed to avail
themselves of the mechanism provided under the Overseas Absentee Voting Act of 2003 1 (R.A.
9189) and that the Commission on Elections (COMELEC) accordingly be ordered to allow them to
vote and register as absentee voters under the aegis of R.A. 9189.
The facts:
Petitioners are successful applicants for recognition of Philippine citizenship under R.A. 9225 which
accords to such applicants the right of suffrage, among others. Long before the May 2004 national
and local elections, petitioners sought registration and certification as "overseas absentee voter"
only to be advised by the Philippine Embassy in the United States that, per a COMELEC letter to the
Department of Foreign Affairs dated September 23, 2003 2, they have yet no right to vote in such
elections owing to their lack of the one-year residence requirement prescribed by the Constitution.
The same letter, however, urged the different Philippine posts abroad not to discontinue their
campaign for voters registration, as the residence restriction adverted to would contextually affect
merely certain individuals who would likely be eligible to vote in future elections.
Prodded for clarification by petitioner Loida Nicolas-Lewis in the light of the ruling in Macalintal vs.
COMELEC 3 on the residency requirement, the COMELEC wrote in response:
Although R.A. 9225 enjoys the presumption of constitutionality , it is the Commission's position
that those who have availed of the law cannot exercise the right of suffrage given under the OAVL for
the reason that the OAVL was not enacted for them. Hence, as Filipinos who have merely reacquired their citizenship on 18 September 2003 at the earliest, and as law and jurisprudence now
stand, they are considered regular voters who have to meet the requirements of residency, among
others under Section 1, Article 5 of the Constitution. 4
Faced with the prospect of not being able to vote in the May 2004 elections owing to the
COMELEC's refusal to include them in the National Registry of Absentee Voters, petitioner NicolasLewis et al., 5 filed on April 1, 2004 this petition for certiorari and mandamus.
A little over a week before the May 10, 2004 elections, or on April 30, 2004, the COMELEC filed a
Comment, 6therein praying for the denial of the petition. As may be expected, petitioners were not
able to register let alone vote in said elections.

On May 20, 2004, the Office of the Solicitor General (OSG) filed a Manifestation (in Lieu of
Comment), therein stating that "all qualified overseas Filipinos, including dual citizens who care to
exercise the right of suffrage, may do so" , observing, however, that the conclusion of the 2004
elections had rendered the petition moot and academic. 7
The holding of the 2004 elections had, as the OSG pointed out, indeed rendered the petition moot
and academic, but insofar only as petitioners participation in such political exercise is concerned.
The broader and transcendental issue tendered or subsumed in the petition, i.e., the propriety of
allowing "duals" to participate and vote as absentee voter in future elections, however, remains
unresolved.
Observing the petitioners and the COMELECs respective formulations of the issues, the same may
be reduced into the question of whether or not petitioners and others who might have meanwhile
retained and/or reacquired Philippine citizenship pursuant to R.A. 9225 may vote as absentee voter
under R.A. 9189.
The Court resolves the poser in the affirmative, and thereby accords merit to the petition.
In esse, this case is all about suffrage. A quick look at the governing provisions on the right of
suffrage is, therefore, indicated.
We start off with Sections 1 and 2 of Article V of the Constitution, respectively reading as follows:
SECTION 1. Suffrage may be exercised by all citizens of the Philippines not otherwise disqualified
by law, who are at least eighteen years of age, and who shall have resided in the Philippines for at
least one year and in the place wherein they propose to vote for at least six months immediately
preceding the election. xxx.
SEC 2. The Congress shall provide a system for absentee voting by qualified Filipinos abroad.
In a nutshell, the aforequoted Section 1 prescribes residency requirement as a general eligibility
factor for the right to vote. On the other hand, Section 2 authorizes Congress to devise a system
wherein an absentee may vote, implying that a non-resident may, as an exception to the residency
prescription in the preceding section, be allowed to vote.
In response to its above mandate, Congress enacted R.A. 9189 - the OAVL 8 - identifying in its
Section 4 who can vote under it and in the following section who cannot, as follows:
Section 4. Coverage. All citizens of the Philippines abroad, who are not otherwise disqualified by
law, at least eighteen (18) years of age on the day of elections, may vote for president, vicepresident, senators and party-list representatives.
Section 5. Disqualifications. The following shall be disqualified from voting under this Act:
(a) Those who have lost their Filipino citizenship in accordance with Philippine laws;

(b) Those who have expressly renounced their Philippine citizenship and who have pledged
allegiance to a foreign country;
(c) Those who have [been] convicted in a final judgment by a court or tribunal of an offense
punishable by imprisonment of not less than one (1) year, including those who have been found
guilty of Disloyalty as defined under Article 137 of the Revised Penal Code, .;
(d) An immigrant or a permanent resident who is recognized as such in the host country, unless
he/she executes, upon registration, an affidavit prepared for the purpose by the Commission
declaring that he/she shall resume actual physical permanent residence in the Philippines not later
than three (3) years from approval of his/her registration under this Act. Such affidavit shall also state
that he/she has not applied for citizenship in another country. Failure to return shall be the cause for
the removal of the name of the immigrant or permanent resident from the National Registry of
Absentee Voters and his/her permanent disqualification to vote in absentia.
(e) Any citizen of the Philippines abroad previously declared insane or incompetent by competent
authority . (Words in bracket added.)
Notably, Section 5 lists those who cannot avail themselves of the absentee voting mechanism.
However, Section 5(d) of the enumeration respecting Filipino immigrants and permanent residents in
another country opens an exception and qualifies the disqualification rule. Section 5(d) would,
however, face a constitutional challenge on the ground that, as narrated in Macalintal, it violates Section 1, Article V of the 1987 Constitution which requires that the voter must be a
resident in the Philippines for at least one year and in the place where he proposes to vote for at
least six months immediately preceding an election. [The challenger] cites Caasi vs. Court of
Appeals 9 to support his claim [where] the Court held that a "green card" holder immigrant to the [US]
is deemed to have abandoned his domicile and residence in the Philippines.
[The challenger] further argues that Section 1, Article V of the Constitution does not allow provisional
registration or a promise by a voter to perform a condition to be qualified to vote in a political
exercise; that the legislature should not be allowed to circumvent the requirement of the Constitution
on the right of suffrage by providing a condition thereon which in effect amends or alters the
aforesaid residence requirement to qualify a Filipino abroad to vote. He claims that the right of
suffrage should not be granted to anyone who, on the date of the election, does not possess the
qualifications provided for by Section 1, Article V of the Constitution. 10 (Words in bracket added.)
As may be recalled, the Court upheld the constitutionality of Section 5(d) of R.A. 9189 mainly on the
strength of the following premises:
As finally approved into law, Section 5(d) of R.A. No. 9189 specifically disqualifies an immigrant or
permanent resident who is "recognized as such in the host country" because immigration or
permanent residence in another country implies renunciation of one's residence in his country of
origin. However, same Section allows an immigrant and permanent resident abroad to register as
voter for as long as he/she executes an affidavit to show that he/she has not abandoned his domicile
in pursuance of the constitutional intent expressed in Sections 1 and 2 of Article V that "all citizens of

the Philippines not otherwise disqualified by law" must be entitled to exercise the right of suffrage
and, that Congress must establish a system for absentee voting; for otherwise, if actual, physical
residence in the Philippines is required, there is no sense for the framers of the Constitution to
mandate Congress to establish a system for absentee voting.
Contrary to the claim of [the challenger], the execution of the affidavit itself is not the enabling or
enfranchising act. The affidavit required in Section 5(d) is not only proof of the intention of the
immigrant or permanent resident to go back and resume residency in the Philippines, but more
significantly, it serves as an explicit expression that he had not in fact abandoned his domicile of
origin. Thus, it is not correct to say that the execution of the affidavit under Section 5(d) violates the
Constitution that proscribes "provisional registration or a promise by a voter to perform a condition to
be qualified to vote in a political exercise." 11
Soon after Section 5(d) of R.A. 9189 passed the test of constitutionality, Congress enacted R.A.
9225 the relevant portion of which reads:
SEC. 2. Declaration of Policy. It is hereby declared the policy of the State that all Philippine citizens
who become citizens of another country shall be deemed not to have lost their Philippine citizenship
under the conditions of this Act.
SEC. 3. Retention of Philippine Citizenship. Any provision of law to the contrary notwithstanding,
natural-born citizens of the Philippines who have lost their Philippine citizenship by reason of their
naturalization as citizens of a foreign country are hereby deemed to have re-acquired Philippine
citizenship upon taking the following oath of allegiance to the Republic:
xxx xxx xxx
Natural-born citizens of the Philippines who, after the effectivity of this Act, become citizens of a
foreign country shall retain their Philippine citizenship upon taking the aforesaid oath.
SEC. 4. Derivative Citizenship. The unmarried child, whether legitimate, illegitimate or adopted,
below eighteen (18) years of age, of those who re-acquire Philippine citizenship upon effectivity of
this Act shall be deemed citizens of the Philippines.
SEC. 5. Civil and Political Rights and Liabilities. Those who retain or re-acquire Philippine
citizenship under this Act shall enjoy full civil and political rights and be subject to all attendant
liabilities and responsibilities under existing laws of the Philippines and the following conditions:
(1) Those intending to exercise their right of suffrage must meet the requirements under Section 1,
Article V of the Constitution, Republic Act No. 9189, otherwise known as "The Overseas Absentee
Voting Act of 2003" and other existing laws;
(2) Those seeking elective public office in the Philippines shall meet the qualifications for holding
such public office as required by the Constitution and existing laws and, at the time of the filing of the
certificate of candidacy, make a personal and sworn renunciation of any and all foreign citizenship
;

3) xxx xxx xxx.


(4) xxx xxx xxx;
(5) That right to vote or be elected or appointed to any public office in the Philippines cannot be
exercised by, or extended to, those who:
(a) are candidates for or are occupying any public office in the country of which they are naturalized
citizens; and/or
(b) are in active service as commissioned or non-commissioned officers in the armed forces of the
country which they are naturalized citizens.
After what appears to be a successful application for recognition of Philippine citizenship under R.A.
9189, petitioners now invoke their right to enjoy political rights, specifically the right of suffrage,
pursuant to Section 5 thereof.
Opposing the petitioners bid, however, respondent COMELEC invites attention to the same Section
5 (1) providing that "duals" can enjoy their right to vote, as an adjunct to political rights, only if they
meet the requirements of Section 1, Article V of the Constitution, R.A. 9189 and other existing laws.
Capitalizing on what at first blush is the clashing provisions of the aforecited provision of the
Constitution, which, to repeat, requires residency in the Philippines for a certain period, and R.A.
9189 which grants a Filipino non-resident absentee voting rights, 12 COMELEC argues:
4. DUALS MUST FIRST ESTABLISH THEIR DOMICILE/ RESIDENCE IN THE PHILIPPINES
4.01. The inclusion of such additional and specific requirements in RA 9225 is logical. The duals,
upon renouncement of their Filipino citizenship and acquisition of foreign citizenship, have practically
and legally abandoned their domicile and severed their legal ties to the homeland as a
consequence. Having subsequently acquired a second citizenship (i.e., Filipino) then, duals must,
for purposes of voting, first of all, decisively and definitely establish their domicile through positive
acts; 13
The Court disagrees.
As may be noted, there is no provision in the dual citizenship law - R.A. 9225 - requiring "duals" to
actually establish residence and physically stay in the Philippines first before they can exercise their
right to vote. On the contrary, R.A. 9225, in implicit acknowledgment that "duals" are most likely nonresidents, grants under its Section 5(1) the same right of suffrage as that granted an absentee voter
under R.A. 9189. It cannot be overemphasized that R.A. 9189 aims, in essence, to enfranchise as
much as possible all overseas Filipinos who, save for the residency requirements exacted of an
ordinary voter under ordinary conditions, are qualified to vote. Thus, wrote the Court in Macalintal:
It is clear from these discussions of the Constitutional Commission that [it] intended to
enfranchise as much as possible all Filipino citizens abroad who have not abandoned their domicile
of origin. The Commission even intended to extend to young Filipinos who reach voting age abroad

whose parents domicile of origin is in the Philippines, and consider them qualified as voters for the
first time.
It is in pursuance of that intention that the Commission provided for Section 2 [Article V] immediately
after the residency requirement of Section 1. By the doctrine of necessary implication in statutory
construction, , the strategic location of Section 2 indicates that the Constitutional Commission
provided for an exception to the actual residency requirement of Section 1 with respect to qualified
Filipinos abroad. The same Commission has in effect declared that qualified Filipinos who are not in
the Philippines may be allowed to vote even though they do not satisfy the residency requirement in
Section 1, Article V of the Constitution.
That Section 2 of Article V of the Constitution is an exception to the residency requirement found in
Section 1 of the same Article was in fact the subject of debate when Senate Bill No. 2104, which
became R.A. No. 9189, was deliberated upon on the Senate floor, thus:
Senator Arroyo. Mr. President, this bill should be looked into in relation to the constitutional
provisions. I think the sponsor and I would agree that the Constitution is supreme in any statute that
we may enact.
Let me read Section 1, Article V, of the Constitution .
xxx xxx xxx
Now, Mr. President, the Constitution says, "who shall have resided in the Philippines." They are
permanent immigrants. They have changed residence so they are barred under the Constitution.
This is why I asked whether this committee amendment which in fact does not alter the original text
of the bill will have any effect on this?
Senator Angara. Good question, Mr. President. And this has been asked in various fora. This is in
compliance with the Constitution. One, the interpretation here of "residence" is synonymous with
"domicile."
As the gentleman and I know, Mr. President, "domicile" is the intent to return to one's home. And the
fact that a Filipino may have been physically absent from the Philippines and may be physically a
resident of the United States, for example, but has a clear intent to return to the Philippines, will
make him qualified as a resident of the Philippines under this law.
This is consistent, Mr. President, with the constitutional mandate that we that Congress must
provide a franchise to overseas Filipinos.
If we read the Constitution and the suffrage principle literally as demanding physical presence, then
there is no way we can provide for offshore voting to our offshore kababayan, Mr. President.
Senator Arroyo. Mr. President, when the Constitution says, in Section 2 of Article V, it reads: "The
Congress shall provide a system for securing the secrecy and sanctity of the ballot as well as a
system for absentee voting by qualified Filipinos abroad."

The key to this whole exercise, Mr. President, is "qualified." In other words, anything that we may do
or say in granting our compatriots abroad must be anchored on the proposition that they are
qualified. Absent the qualification, they cannot vote. And "residents" (sic) is a qualification.
xxx xxx xxx
Look at what the Constitution says "In the place wherein they propose to vote for at least six
months immediately preceding the election."
Mr. President, all of us here have run (sic) for office.
I live in Makati. My neighbor is Pateros . We are separated only by a creek. But one who votes in
Makati cannot vote in Pateros unless he resides in Pateros for six months. That is how restrictive our
Constitution is. .
As I have said, if a voter in Makati would want to vote in Pateros, yes, he may do so. But he must do
so, make the transfer six months before the election, otherwise, he is not qualified to vote.
xxx xxx xxx
Senator Angara. It is a good point to raise, Mr. President. But it is a point already well-debated even
in the constitutional commission of 1986. And the reason Section 2 of Article V was placed
immediately after the six-month/one-year residency requirement is to demonstrate unmistakably that
Section 2 which authorizes absentee voting is an exception to the six-month/one-year residency
requirement. That is the first principle, Mr. President, that one must remember.
The second reason, Mr. President, is that under our jurisprudence "residency" has been
interpreted as synonymous with "domicile."
But the third more practical reason, is, if we follow the interpretation of the gentleman, then it is
legally and constitutionally impossible to give a franchise to vote to overseas Filipinos who do not
physically live in the country, which is quite ridiculous because that is exactly the whole point of this
exercise to enfranchise them and empower them to vote. 14 (Emphasis and words in bracket
added; citations omitted)
Lest it be overlooked, no less than the COMELEC itself admits that the Citizenship Retention and
Re-Acquisition Act expanded the coverage of overseas absentee voting. According to the poll body:
1.05 With the passage of RA 9225 the scope of overseas absentee voting has been consequently
expanded so as to include Filipinos who are also citizens of other countries, subject, however, to the
strict prerequisites indicated in the pertinent provisions of RA 9225; 15
Considering the unison intent of the Constitution and R.A. 9189 and the expansion of the scope of
that law with the passage of R.A. 9225, the irresistible conclusion is that "duals" may now exercise
the right of suffrage thru the absentee voting scheme and as overseas absentee voters. R.A. 9189
defines the terms adverted to in the following wise:

"Absentee Voting" refers to the process by which qualified citizens of the Philippines abroad exercise
their right to vote;
"Overseas Absentee Voter" refers to a citizen of the Philippines who is qualified to register and vote
under this Act, not otherwise disqualified by law, who is abroad on the day of elections;
While perhaps not determinative of the issue tendered herein, we note that the expanded thrust of
R.A. 9189 extends also to what might be tag as the next generation of "duals". This may be deduced
from the inclusion of the provision on derivative citizenship in R.A. 9225 which reads:
SEC. 4. Derivative Citizenship. The unmarried child, whether legitimate, illegitimate or adopted,
below eighteen (18) years of age, of those who re-acquire Philippine citizenship upon effectivity of
this Act shall be deemed citizens of the Philippines.
It is very likely that a considerable number of those unmarried children below eighteen (18) years of
age had never set foot in the Philippines. Now then, if the next generation of "duals" may
nonetheless avail themselves the right to enjoy full civil and political rights under Section 5 of the Act,
then there is neither no rhyme nor reason why the petitioners and other present day "duals,"
provided they meet the requirements under Section 1, Article V of the Constitution in relation to R.A.
9189, be denied the right of suffrage as an overseas absentee voter. Congress could not have
plausibly intended such absurd situation.
WHEREFORE, the instant petition is GRANTED. Accordingly, the Court rules and so holds that
those who retain or re-acquire Philippine citizenship under Republic Act No. 9225, the Citizenship
Retention and Re-Acquisition Act of 2003, may exercise the right to vote under the system of
absentee voting in Republic Act No. 9189, the Overseas Absentee Voting Act of 2003.
SO ORDERED.

Republic of the Philippines


SUPREME COURT
Manila
THIRD DIVISION
G.R. No. 170603

January 29, 2007

EDISON SO, Petitioner,


vs.
REPUBLIC OF THE PHILIPPINES, Respondent.
DECISION
CALLEJO, SR., J.:

Assailed in this Petition for Review on Certiorari is the Decision 1 of the Court of Appeals (CA) in CAG.R. CV No. 80437 which reversed the Decision2 of the Regional Trial Court (RTC) of Manila,
Branch 8, in Naturalization Case No. 02-102984. Likewise assailed is the appellate courts
Resolution denying the Motion for Reconsideration of its Decision.
Antecedents
On February 28, 2002, petitioner Edison So filed before the RTC a Petition for Naturalization 3 under
Commonwealth Act (C.A.) No. 473, otherwise known as the Revised Naturalization Law, as
amended. He alleged the following in his petition:
He was born on February 17, 1982, in Manila; he is a Chinese citizen who has lived in No. 528
Lavezares St., Binondo, Manila, since birth; as an employee, he derives an average annual income
of around P100,000.00 with free board and lodging and other benefits; he is single, able to speak
and write English, Chinese and Tagalog; he is exempt from the filing of Declaration of Intention to
become a citizen of the Philippines pursuant to Section 6 of Commonwealth Act (C.A.) No. 473, as
amended, because he was born in the Philippines, and studied in a school recognized by the
Government where Philippine history, government and culture are taught; he is a person of good
moral character; he believes in the principles underlying the Philippine constitution; he has
conducted himself in a proper and irreproachable manner during the entire period of his residence in
the Philippines in his relation with the constituted government as well as with the community in which
he is living; he has mingled socially with the Filipinos and has evinced a sincere desire to learn and
embrace the customs, traditions and ideals of the Filipino people; he has all the qualifications
provided under Section 2 and none of the disqualifications under Section 4 of C.A. No. 473, as
amended; he is not opposed to organized government or affiliated with any association or group of
persons who uphold and teach doctrines opposing all organized governments; he is not defending or
teaching the necessity or propriety of violence, personal assault or assassination for the success or
predominance of mens ideas; he is not a polygamist or a believer in the practice of polygamy; he
has not been convicted of any crime involving moral turpitude; he is not suffering from any incurable
contagious diseases or from mental alienation; the nation of which he is a citizen is not at war with
the Philippines; it is his intention in good faith to become a citizen of the Philippines and to renounce
absolutely and forever all allegiance and fidelity to any foreign prince, potentate, state or sovereignty,
and particularly to China; and he will reside continuously in the Philippines from the time of the filing
of the petition up to the time of his admission as citizen of the Philippines. The petition was docketed
as Naturalization Case No. 02-102984.
Attached to the petition were the Joint Affidavit4 of Atty. Artemio Adasa, Jr. and Mark B. Salcedo; and
petitioners Certificate of Live Birth,5 Alien Certificate of Registration,6 and Immigrant Certificate of
Residence.7
On March 22, 2002, the RTC issued an Order8 setting the petition for hearing at 8:30 a.m. of
December 12 and 17, 2002 during which all persons concerned were enjoined to show cause, if any,
why the petition should not be granted. The entire petition and its annexes, including the order, were
ordered published once a week for three consecutive weeks in the Official Gazette and also in a
newspaper of general circulation in the City of Manila. The RTC likewise ordered that copies of the
petition and notice be posted in public and conspicuous places in the Manila City Hall Building. 9

Petitioner thus caused the publication of the above order, as well as the entire petition and its
annexes, in the Official Gazette on May 20, 200210 and May 27, 2002,11 and in Today, a newspaper
of general circulation in the City of Manila, on May 25, 2002 and June 1, 2002.
No one opposed the petition. During the hearing, petitioner presented Atty. Adasa, Jr. who testified
that he came to know petitioner in 1991 as the legal consultant and adviser of the So familys
business. He would usually attend parties and other social functions hosted by petitioners family. He
knew petitioner to be obedient, hardworking, and possessed of good moral character, including all
the qualifications mandated by law. Atty. Adasa, Jr. further testified that petitioner was gainfully
employed and presently resides at No. 528 Lavezares Street, Binondo, Manila; petitioner had been
practicing Philippine tradition and those embodied in the Constitution; petitioner had been socially
active, mingled with some of his neighbors and had conducted himself in a proper and
irreproachable manner during his entire stay in the Philippines; and petitioner and his family
observed Christmas and New Year and some occasions such as fiestas. According to the witness,
petitioner was not disqualified under C.A. No. 473 to become a Filipino citizen: he is not opposed to
organized government or believes in the use of force; he is not a polygamist and has not been
convicted of a crime involving moral turpitude; neither is he suffering from any mental alienation or
any incurable disease.12
Another witness for petitioner, Mark Salcedo, testified that he has known petitioner for ten (10) years;
they first met at a birthday party in 1991. He and petitioner were classmates at the University of
Santo Tomas (UST) where they took up Pharmacy. Petitioner was a member of some school
organizations and mingled well with friends.13Salcedo further testified that he saw petitioner twice a
week, and during fiestas and special occasions when he would go to petitioners house. He has
known petitioner to have resided in Manila since birth. Petitioner is intelligent, a person of good
moral character, and believes in the principles of the Philippine Constitution. Petitioner has a gainful
occupation, has conducted himself in a proper and irreproachable manner and has all the
qualifications to become a Filipino citizen.
Petitioner also testified and attempted to prove that he has all the qualifications and none of the
disqualifications to become a citizen of the Philippines.
At the conclusion of his testimonial evidence, petitioner offered in evidence the following documents:
(1) Certificate of Live Birth;14 (2) Alien Certificate of Registration;15 (3) Immigrant Certificate of
Residence;16 (4) Elementary Pupils17 and High School Students18 Permanent Record issued by
Chang Kai Shek College; (5) Transcript of Record issued by the University of Santo Tomas; 19 (6)
Certification of Part-Time Employment dated November 20, 2002; 20 (7) Income Tax Returns and
Certificate of Withholding Tax for the year 2001;21 (8) Certification from Metrobank that petitioner is a
depositor;22 (9) Clearances that he has not been charged or convicted of any crime involving moral
turpitude;23 and (10) Medical Certificates and Psychiatric Evaluation issued by the Philippine General
Hospital.24 The RTC admitted all these in evidence.
The RTC granted the petition on June 4, 2003.25 The fallo of the decision reads:
WHEREFORE, judgment is hereby rendered GRANTING the petition and declaring that petitioner
EDISON SO has all the qualifications and none of the disqualifications to become a Filipino citizen

and he is hereby admitted as citizen of the Philippines, after taking the necessary oath of allegiance,
as soon as this decision becomes final, subject to payment of cost of P30,000.00.
SO ORDERED.26
The trial court ruled that the witnesses for petitioner had known him for the period required by law,
and they had affirmed that petitioner had all the qualifications and none of the disqualifications to
become a Filipino citizen. Thus, the court concluded that petitioner had satisfactorily supported his
petition with evidence.
Respondent Republic of the Philippines, through the Office of the Solicitor General (OSG), appealed
the decision to the CA on the following grounds:
I.
THE LOWER COURT ERRED IN GRANTING THE PETITION FOR NATURALIZATION DESPITE
THE FACT THAT THE TWO (2) CHARACTER WITNESSES, NAMELY: ARTEMIO ADASA, JR. AND
MARK SALCEDO WERE NOT QUALIFIED CHARACTER WITNESSES.
II.
PETITIONER IS NOT QUALIFIED TO BE ADMITTED AS CITIZEN OF THE PHILIPPINES. 27
Respondent contended that based on the evidence on record, appellee failed to prove that he
possesses all the qualifications under Section 2 and none of the disqualifications under Section 4 of
C.A. No. 473. It insisted that his two (2) character witnesses did not know him well enough to vouch
for his fitness to become a Filipino citizen; they merely made general statements without giving
specific details about his character and moral conduct.28The witnesses did not even reside in the
same place as petitioner.29 Respondent likewise argued that petitioner himself failed to prove that he
is qualified to become a Filipino citizen because he did not give any explanation or specific answers
to the questions propounded by his lawyer. He merely answered "yes" or "no" or gave general
statements in answer to his counsels questions. Thus, petitioner was unable to prove that he had all
the qualifications and none of the disqualifications required by law to be a naturalized Filipino
citizen.30
On the other hand, petitioner averred that he graduated cum laude from the UST with the degree of
Bachelor of Science in Pharmacy. He is now on his second year as a medical student at the UST
Medicine and Surgery. He avers that the requirements for naturalization under C.A. No. 473, as
amended by LOI 270, in relation to Presidential Decree Nos. 836 and 1379, had been relaxed after
the Philippine government entered into diplomatic relations with the Peoples Republic of China; the
requirements were further relaxed when Republic Act (R.A.) No. 9139 was signed into
law.31 Petitioner pointed out that the petition, with all its annexes, was published in the official gazette
and a newspaper of general circulation; notices were likewise sent to the National Bureau of
Investigation, Department of Justice, Department of Foreign Affairs, and the OSG. But none from
these offices came forward to oppose the petition before the lower court. 32 Petitioner insisted that he

has all the qualifications and none of the disqualifications to become Filipino. This was clearly
established by his witnesses.
In its Reply Brief, respondent alleged that R.A. No. 9139 applies to administrative naturalization filed
with the Special Committee on Naturalization. It insisted that even in the absence of any opposition,
a petition for naturalization may be dismissed.
In its Decision33 dated August 4, 2005, the CA set aside the ruling of the RTC and dismissed the
petition for naturalization without prejudice.34 According to the CA, petitioners two (2) witnesses were
not credible because they failed to mention specific details of petitioners life or character to show
how well they knew him; they merely "parroted" the provisions of the Naturalization Act without
clearly explaining their applicability to petitioners case.35 The appellate court likewise ruled that
petitioner failed to comply with the requirement of the law that the applicant must not be less than 21
years of age on the day of the hearing of the petition; during the first hearing on December 12, 2002,
petitioner was only twenty (20) years, nine (9) months, and twenty five (25) days old, falling short of
the requirement.36 The CA stated, however, that it was not its intention to forever close the door to
any future application for naturalization which petitioner would file, and that it believes that he would
make a good Filipino citizen in due time, a decided asset to this country.37
Petitioners motion for reconsideration38 was denied in a Resolution39 dated November 24, 2005;
hence, the present petition grounded on the sole issue:
WHETHER OR NOT THE HONORABLE COURT OF APPEALS COMMITTED REVERSIBLE
ERROR WHEN IT REVERSED THE DECISION OF THE REGIONAL TRIAL COURT OF MANILA. 40
In support of his petition, petitioner reiterates the arguments he set forth in the Brief filed before the
CA.
In its Comment41 on the petition, respondent countered that R.A. No. 9139 (which took effect on
August 8, 2001 and where the applicants age requirement was lowered to eighteen (18) years old),
refers only to administrative naturalization filed with the Special Committee on Naturalization; it does
not apply to judicial naturalization before the court, as in the present case. 42 Respondent, through the
OSG, avers that its failure to oppose the petition before the court a quo does not preclude it from
appealing the decision of the RTC to the CA; it is even authorized to question an already final
decision by filing a petition for cancellation of citizenship.43 Lastly, respondent reiterates its argument
that petitioners character witnesses are not qualified to prove the formers qualifications.
In determining whether or not an applicant for naturalization is entitled to become a Filipino citizen, it
is necessary to resolve the following issues: (1) whether or not R.A. No. 9139 applies to petitions for
naturalization by judicial act; and (2) whether or not the witnesses presented by petitioner are
"credible" in accordance with the jurisprudence and the definition and guidelines set forth in C.A. No.
473.
The petition is denied for lack of merit.

Naturalization signifies the act of formally adopting a foreigner into the political body of a nation by
clothing him or her with the privileges of a citizen. 44 Under current and existing laws, there are three
ways by which an alien may become a citizen by naturalization: (a) administrative naturalization
pursuant to R.A. No. 9139; (b) judicial naturalization pursuant to C.A. No. 473, as amended; and (c)
legislative naturalization in the form of a law enacted by Congress bestowing Philippine citizenship to
an alien.45
Petitioners contention that the qualifications an applicant for naturalization should possess are those
provided for in R.A. No. 9139 and not those set forth in C.A. No. 473 is barren of merit. The
qualifications and disqualifications of an applicant for naturalization by judicial act are set forth in
Sections 246 and 447 of C.A. No. 473. On the other hand, Sections 348 and 449 of R.A. No. 9139
provide for the qualifications and disqualifications of an applicant for naturalization by administrative
act.
Indeed, R.A. No. 9139 was enacted as a remedial measure intended to make the process of
acquiring Philippine citizenship less tedious, less technical and more encouraging. 50 It likewise
addresses the concerns of degree holders who, by reason of lack of citizenship requirement, cannot
practice their profession, thus promoting "brain gain" for the Philippines. 51 These however, do not
justify petitioners contention that the qualifications set forth in said law apply even to applications for
naturalization by judicial act.
First. C.A. No. 473 and R.A. No. 9139 are separate and distinct laws the former covers all aliens
regardless of class while the latter covers native-born aliens who lived here in the Philippines all their
lives, who never saw any other country and all along thought that they were Filipinos; who have
demonstrated love and loyalty to the Philippines and affinity to the customs and traditions. 52 To
reiterate, the intention of the legislature in enacting R.A. No. 9139 was to make the process of
acquiring Philippine citizenship less tedious, less technical and more encouraging which is
administrative rather than judicial in nature. Thus, although the legislature believes that there is a
need to liberalize the naturalization law of the Philippines, there is nothing from which it can be
inferred that C.A. No. 473 was intended to be amended or repealed by R.A. No. 9139. What the
legislature had in mind was merely to prescribe another mode of acquiring Philippine citizenship
which may be availed of by native born aliens. The only implication is that, a native born alien has
the choice to apply for judicial or administrative naturalization, subject to the prescribed qualifications
and disqualifications.
In the instant case, petitioner applied for naturalization by judicial act, though at the time of the filing
of his petition, administrative naturalization under R.A. No. 9139 was already available.
Consequently, his application should be governed by C.A. No. 473.
Second. If the qualifications prescribed in R.A. No. 9139 would be made applicable even to judicial
naturalization, the coverage of the law would be broadened since it would then apply even to aliens
who are not native born. It must be stressed that R.A. No. 9139 applies only to aliens who were born
in the Philippines and have been residing here.
Third. Applying the provisions of R.A. No. 9139 to judicial naturalization is contrary to the intention of
the legislature to liberalize the naturalization procedure in the country. One of the qualifications set

forth in R.A. No. 9139 is that the applicant was born in the Philippines and should have been
residing herein since birth. Thus, one who was born here but left the country, though resided for
more than ten (10) years from the filing of the application is also disqualified. On the other hand, if
we maintain the distinct qualifications under each of the two laws, an alien who is not qualified under
R.A. No. 9139 may still be naturalized under C.A. No. 473.
Thus, absent a specific provision expressly amending C.A. No. 473, the law stands and the
qualifications and disqualifications set forth therein are maintained.
In any event, petitioner failed to prove that the witnesses he presented were competent to vouch for
his good moral character, and are themselves possessed of good moral character. It must be
stressed that character witnesses in naturalization proceedings stand as insurers of the applicants
conduct and character. Thus, they ought to testify on specific facts and events justifying the
inference that the applicant possesses all the qualifications and none of the disqualifications
provided by law.53
Petitioners witnesses, Atty. Adasa and Salcedo, did not testify on his specific acts; they did not
elaborate on his traits. Their testimonies do not convince the Court that they personally know
petitioner well and are therefore in a position to vouch for his qualifications. As correctly found by the
CA, the witnesses testimonies consisted mainly of general statements in answer to the leading
questions propounded by his counsel. What they conveniently did was to enumerate the
qualifications as set forth in the law without giving specific details. The pertinent portion of Atty.
Adasas testimony follows:
q Do you know the petitioner Edison So?
a Yes, Sir.
q Will you please tell us how did you come to know him?
a Well I came to know him[,] the petitioner[,] when I was the legal consultant and adviser of their
family business and I used to ah (sic) me[e]t him during my visit to their place way back in 1991 to
1992.
q From that day of 1991 up to the present, is your relationship with the petitioner more or less
contin[u]ous?
a Yes, sir, because aside from the usual professional visit that I did to their family some social
function was sponsored normally and I am (sic) invited and I used to attend.
q During the birthday party of the petitioner, did you usually attend petitioners birthday?
a On several occasions I attend the birthday.
q Will you please tell us where the petitioner resides at present?

a At present the petitioner resides at No. 528 Lavezares Street, Binondo, Manila.
q Do you know for how long the petitioner resides in the Philippines?
a As far as I personally known (sic) Your Honor is that since birth.
q During all the times that you have know[n] the petitioner, what is your impression of his conduct?
a Well ah (sic) I have personally known him to be obedient and hard working individual and ah (sic)
he has a good moral character and he has been ah (sic) no adverse report concerning the character
of the petitioner.
q In your opinion does the petitioner has the qualifications necessary to become [a] citizen of the
Philippines?
a Yes.
q Can you tell us why do you say so?
a I would say Your Honor that petitioner has posses (sic) all the qualifications mandated by law and
presently he is more than 21 years old and he has resided in the Philippines particularly in the City of
Manila contin[u]ously for more than ten (10) years and that since his birth; and that he has good
moral character and I have observed that ah (sic) he has been practicing Philippine traditions and ah
(sic) those embodied in the Philippine constitution and he has been socially active and meddle (sic)
some of his neighbors and ah (sic) I am sure he has desire to embrace and learn the customs and
ideas and traditions in the Philippine[s] and as I earlier mentioned that he conducted himself in
proper and approachable (sic) manner during his entire residence in our country and he has a
gainful occupation.
q Will you please tell us what are these customs which the petitioner embraced?
a Well I have observed that ah (sic) together with his family they used to ah observed (sic) the usual
Filipino celebration during Christmas and new year and some occasions such as fiestas.
q And do you know whether petitioner is not disqualified under Commonwealth Act to become
Filipino citizen of the Philippines (sic)?
a Ah there has been no incident or occasion which I learned that would disqualify of coming (sic) the
citizen of the Republic of the Philippines. I have noticed that ah (sic) he is qualified under
Commonwealth Act 473 as amended because he is not opposed to ah (sic) organized government.
His family and himself does not believed (sic) in the use of force in the success of his ideas and ah
(sic) he is not a poligamist (sic) or believer in the practice of illegal and he has not been convicted in
any crime involving him in any crime (sic). and he is not suffering from any mental alienation or any
incurable contidious (sic) disease. as provided for.
q Will you please tell us why you know all these stage?

a Because of ah (sic) the personal attachment with his family we have continuously having ah (sic)
the usual contact with his family.54
It can thus be inferred that Atty. Adasa is close to petitioners family, but not specifically to petitioner.
Atty. Adasas statements refer to his observations on the familys practices and not to petitioner in
particular. Nothing in his testimony suggests that he was close to petitioner and knew him well
enough to vouch for his qualifications.
Salcedo, on the other hand, testified thus:
q Now do you know the petitioner in this case Edison So?
a Yes, Sir.
q Are you personally acquainted with him?
a Yes, Sir.
q How long have you known the petitioner?
a I have known him for about ten (10) years, Sir.
q Will you please inform the Honorable court under what circumstances did you come to know the
petitioner?
a I met him in a birthday party in 1991, Sir.
q And from 1991 up to the present is your relationship with the petitioner more or less contin[u]ous?
a Yes, Sir.
q How often did you see the petitioner?
a I see him twice a week, Sir.
q And during this time that you met the petitioner, what did you usually do?
a We play some games, Sir. We play Patentero (sic).
q Do you go to church together?
a Yes, Sir.
q During fiestas in your place, did the petitioner go?
a Yes, Sir.

q How about during fiestas in the place where the petitioner reside[s], did you also go during fiestas?
a Yes, Sir.
q During occasion in the house of the petitioner, are you invited?
a Yes, Sir.
q How many time[s] did you go to his (sic) residence of the petitioner?
a Twice a week, sir.
q Will you please tell us where the petitioner resides?
a The petitioner resides at 528 Lavezares Street, Tondo, Manila, Sir.
q For how long does the petitioner reside in that address?
a Since birth, Sir.
q During all the times that you have known the petitioner, will you please tell us your impression of
his conduct?
a He is a person of good moral, sir, and he believed in the principles of the Philippines (sic)
Constitution.
q Will you please cite one or two of these principles underlined the principles (sic) of the Philippines
(sic) Constitution?
a Ah the Philippines is a Republican of the (sic) state, sovereignty preside (sic) over the people and
the government authority emanate from within; and the other one is the civilian government is not
supreme over the military.
q Now in your opinion does the petitioner have all the qualifications necessary to become a citizen of
the Philippines?
a Yes, Sir.
q What are these qualifications?
a He is at least 21 years old, he is a person of good moral and has been residing in the Philippines
since birth.
q What else?
a He must be a Filipino and ah must practice the traditions and customs, Sir.

q Do you know whether the petitioner conducted himself in a proper and appraochable (sic) manner
during the period of his residence in the Philippines?
a Yes, Sir.
q Do you know if the petitioner has a gainful occupation?
a Yes, Sir.
q What is the occupation of the petitioner?
a Ah (sic) he is the secretary in a wood factory in Commonwealth, Sir.
q And aside from being the secretary, what else did the petitioner do?
a He help (sic) in the factory cargo, Sir.
q Is the petitioner still a student?
a Yes, Sir.
q Where is he studying?
a In UST, Sir.
q Is he your classmate?
a Yes, Sir.
q What was his course?
a Pharmacy, Sir.
q So when you said he was the secretary he only works as part time secretary?
a Yes, Sir.
q You said the petitioner meddle (sic) socially with the Filipinos?
a Yes, Sir.
q Will you please name at least one of those Filipinos the petitioner meddle (sic) with?
a Samuel Falmera, Sir, Marlon Kahocom, Sir.
q Who else?

a Elmer Ramos, Sir.


q Who else?
a Sharmaine Santos, Sir.
q You said the petitioner is of good moral character?
a Yes, Sir.
q Why do you know that?
a As a classmate I can see him I go with him and ah (sic) I can see that he has ah better approached
(sic) with other people and I can see that he mixed very well with friends.
q So during school days you see him everyday?
a Yes, Sir.
q When there are no classes during the vacation you see the petitioner twice a week?
a Yes, Sir.
q Does the petitioner (sic), do you think the petitioner is not disqualified to become the citizen of the
Republic of the Philippines?
a Yes, Sir, he is not disqualified, Sir.
q Why do you say that he is not disqualified?
a Because he abide [by] any law in the government, sir, ah (sic) he is not polygamus and he is not
convicted of any crime, Sir.
q Do you know ever the petitioner oppose to any organized government?
a No, Sir.
q Do you know whether he believe[s] in the use of force in any such ideas?
a No, Sir.
q Do you know if the petitioner is a believer in the practice of polygamy?
a No, Sir.
q Do you know whether the petitioner suffer[s] from mental alienation or incurable disease illnesses?

a No, Sir.
q Why do you know?
a I know him personally, sir, I have been with him as my classmate, sir and ah (sic) he is a very
intelligent person, Sir.
q Is the petitioner a member also of any organization or association in your school?
a Yes, Sir.
q What organization?
a He is a member of Wishten and a member of starget, Sir.
q What does starget means?
a Starget is an organization of Chinese community in UST, Sir.
q How about the other one which you mentioned?
a Ah (sic) these are twisting, sir he represents the ah the (sic) school intercollegiate, Sir.55
Again, Salcedo did not give specific details on petitioners qualifications.
In sum, petitioners witnesses clearly did not personally know him well enough; their testimonies do
not satisfactorily establish that petitioner has all the qualifications and none of the disqualifications
prescribed by law.
In naturalization proceedings, it is the burden of the applicant to prove not only his own good moral
character but also the good moral character of his/her witnesses, who must be credible
persons.56 Within the purview of the naturalization law, a "credible person" is not only an individual
who has not been previously convicted of a crime; who is not a police character and has no police
record; who has not perjured in the past; or whose affidavit or testimony is not incredible. What must
be credible is not the declaration made but the person making it. This implies that such person must
have a good standing in the community; that he is known to be honest and upright; that he is reputed
to be trustworthy and reliable; and that his word may be taken on its face value, as a good warranty
of the applicants worthiness.57
The records likewise do not show that the character witnesses of petitioner are persons of good
standing in the community; that they are honest and upright, or reputed to be trustworthy and
reliable. The most that was established was the educational attainment of the witnesses; however,
this cannot be equated with their credibility. In fine, petitioner focused on presenting evidence
tending to build his own good moral character and neglected to establish the credibility and good
moral character of his witnesses.58

We do not agree with petitioners argument that respondent is precluded from questioning the RTC
decision because of its failure to oppose the petition. A naturalization proceeding is not a judicial
adversary proceeding, and the decision rendered therein does not constitute res judicata. A
certificate of naturalization may be cancelled if it is subsequently discovered that the applicant
obtained it by misleading the court upon any material fact. Law and jurisprudence even authorize the
cancellation of a certificate of naturalization upon grounds or conditions arising subsequent to the
granting of the certificate.59 If the government can challenge a final grant of citizenship, with more
reason can it appeal the decision of the RTC within the reglementary period despite its failure to
oppose the petition before the lower court.
Thus, petitioner failed to show full and complete compliance with the requirements of naturalization
law. For this reason, we affirm the decision of the CA denying the petition for naturalization without
prejudice.
It must be stressed that admission to citizenship is one of the highest privileges that the Republic of
the Philippines can confer upon an alien. It is a privilege that should not be conferred except upon
persons fully qualified for it, and upon strict compliance with the law.60
IN LIGHT OF ALL THE FOREGOING, the petition is DENIED for lack of merit.
SO ORDERED.
Republic of the Philippines
SUPREME COURT
Manila
EN BANC
G.R. No. 142840

May 7, 2001

ANTONIO BENGSON III, petitioner,


vs.
HOUSE OF REPRESENTATIVES ELECTORAL TRIBUNAL and TEODORO C.
CRUZ, respondents.
CONCURRING OPINION
DISSENTING OPINION
KAPUNAN, J.:
The citizenship of respondent Teodoro C. Cruz is at issue in this case, in view of the constitutional
requirement that "no person shall be a Member of the House of Representative unless he is a
natural-born citizen."1
Respondent Cruz was a natural-born citizen of the Philippines. He was born in San Clemente,
Tarlac, on April 27, 1960, of Filipino parents. The fundamental law then applicable was the 1935
Constitution.2

On November 5, 1985, however, respondent Cruz enlisted in the United States Marine Corps and
without the consent of the Republic of the Philippines, took an oath of allegiance to the United
States. As a Consequence, he lost his Filipino citizenship for under Commonwealth Act No. 63,
section 1(4), a Filipino citizen may lose his citizenship by, among other, "rendering service to or
accepting commission in the armed forces of a foreign country." Said provision of law reads:
SECTION 1. How citizenship may be lost. A Filipino citizen may lose his citizenship in any
of the following ways and/or events:
xxx
(4) By rendering services to, or accepting commission in, the armed of a foreign
country: Provided, That the rendering of service to, or the acceptance of such commission in,
the armed forces of a foreign country, and the taking of an oath of allegiance incident thereto,
with the consent of the Republic of the Philippines, shall not divest a Filipino of his Philippine
citizenship if either of the following circumstances is present:
(a) The Republic of the Philippines has a defensive and/or offensive pact of alliance with said
foreign country; or
(b) The said foreign country maintains armed forces on Philippine territory with the consent
of the Republic of the Philippines: Provided, That the Filipino citizen concerned, at the time of
rendering said service, or acceptance of said commission, and taking the oath of allegiance
incident thereto, states that he does so only in connection with his service to said foreign
country; And provided, finally, That any Filipino citizen who is rendering service to, or is
commissioned in, the armed forces of a foreign country under any of the circumstances
mentioned in paragraph (a) or (b), shall not be Republic of the Philippines during the period
of his service to, or commission in, the armed forces of said country. Upon his discharge
from the service of the said foreign country, he shall be automatically entitled to the full
enjoyment of his civil and politically entitled to the full enjoyment of his civil political rights as
a Filipino citizen x x x.
Whatever doubt that remained regarding his loss of Philippine citizenship was erased by his
naturalization as a U.S. citizen on June 5, 1990, in connection with his service in the U.S. Marine
Corps.
On March 17, 1994, respondent Cruz reacquired his Philippine citizenship through repatriation under
Republic Act No. 2630.3 He ran for and was elected as the Representative of the Second District of
Pangasinan in the May 11, 1998 elections. He won by a convincing margin of 26,671 votes over
petitioner Antonio Bengson III, who was then running for reelection.
1wphi1.nt

Subsequently, petitioner filed a case for Quo Warranto Ad Cautelam with respondent House of
Representatives Electoral Tribunal (HRET) claiming that respondent Cruz was not qualified to
become a member of the House of Representatives since he is not a natural-born citizen as required
under Article VI, section 6 of the Constitution.4
On March 2, 2000, the HRET rendered its decision5 dismissing the petition for quo warranto and
declaring Cruz the duly elected Representative of the Second District of Pangasinan in the May
1998 elections. The HRET likewise denied petitioner's motion for reconsideration of the decision in
its resolution dated April 27, 2000.6

Petitioner thus filed the present petition for certiorari assailing the HRET's decision on the following
grounds:
1. The HRET committed serious errors and grave abuse of discretion, amounting to excess
of jurisdiction, when it ruled that private respondent is a natural-born citizen of the
Philippines despite the fact that he had ceased being such in view of the loss and
renunciation of such citizenship on his part.
2. The HRET committed serious errors and grave abuse of discretion, amounting to excess
of jurisdiction, when it considered private respondent as a citizen of the Philippines despite
the fact he did not validly acquire his Philippine citizenship.
3. Assuming that private respondent's acquisition of Philippine citizenship was invalid, the
HRET committed serious errors and grave abuse of discretion, amounting to excess of
jurisdiction, when it dismissed the petition despite the fact that such reacquisition could not
legally and constitutionally restore his natural-born status.7
The issue now before us is whether respondent Cruz, a natural-born Filipino who became an
American citizen, can still be considered a natural-born Filipino upon his reacquisition of Philippine
citizenship.
Petitioner asserts that respondent Cruz may no longer be considered a natural-born Filipino since he
lost h is Philippine citizenship when he swore allegiance to the United States in 1995, and had to
reacquire the same by repatriation. He insists that Article citizens are those who are from birth with
out having to perform any act to acquire or perfect such citizenship.
Respondent on the other hand contends that he reacquired his status as natural-born citizen when
he was repatriated since the phrase "from birth" in Article IV, Section 2 refers to the innate, inherent
and inborn characteristic of being a natural-born citizen.
The petition is without merit.
The 1987 Constitution enumerates who are Filipino citizens as follow:
(1) Those who are citizens of the Philippines at the time of the adoption of this Constitution;
(2) Those whose fathers or mothers are citizens of the Philippines;
(3) Those born before January 17, 1973 of Filipino mother, who elect Philippine citizenship
upon reaching the age of majority, and
(4) Those who are naturalized in accordance with law.8
There are two ways of acquiring citizenship: (1) by birth, and (2) by naturalization. These ways of
acquiring citizenship correspond to the two kinds of citizens: the natural-born citizen, and the
naturalized citizen. A person who at the time of his birth is a citizen of a particular country, is a
natural-born citizen thereof.9
As defined in the same Constitution, natural-born citizens "are those citizens of the Philippines from
birth without having to perform any act to acquire or perfect his Philippine citezenship." 10

On the other hand, naturalized citizens are those who have become Filipino citizens through
naturalization, generally under Commonwealth Act No. 473, otherwise known as the Revised
Naturalization Law, which repealed the former Naturalization Law (Act No. 2927), and by Republic
Act No. 530.11 To be naturalized, an applicant has to prove that he possesses all the
qualifications12 and none of the disqualification13 provided by law to become a Filipino citizen. The
decision granting Philippine citizenship becomes executory only after two (2) years from its
promulgation when the court is satisfied that during the intervening period, the applicant has (1) not
left the Philippines; (2) has dedicated himself to a lawful calling or profession; (3) has not been
convicted of any offense or violation of Government promulgated rules; or (4) committed any act
prejudicial to the interest of the nation or contrary to any Government announced policies. 14
Filipino citizens who have lost their citizenship may however reacquire the same in the manner
provided by law. Commonwealth Act. No. (C.A. No. 63), enumerates the three modes by which
Philippine citizenship may be reacquired by a former citizen: (1) by naturalization, (2) by repatriation,
and (3) by direct act of Congress.15
Naturalization is mode for both acquisition and reacquisition of Philippine citizenship. As a mode of
initially acquiring Philippine citizenship, naturalization is governed by Commonwealth Act No. 473, as
amended. On the other hand, naturalization as a mode for reacquiring Philippine citizenship is
governed by Commonwealth Act No. 63.16 Under this law, a former Filipino citizen who wishes to
reacquire Philippine citizenship must possess certain qualifications 17 and none of the disqualification
mentioned in Section 4 of C.A. 473.18
Repatriation, on the other hand, may be had under various statutes by those who lost their
citizenship due to: (1) desertion of the armed forces;19 services in the armed forces of the allied
forces in World War II;20 (3) service in the Armed Forces of the United States at any other time,21 (4)
marriage of a Filipino woman to an alien;22 and (5) political economic necessity.23
As distinguished from the lengthy process of naturalization, repatriation simply consists of the taking
of an oath of allegiance to the Republic of the Philippine and registering said oath in the Local Civil
Registry of the place where the person concerned resides or last resided.
In Angat v. Republic,24 we held:
xxx. Parenthetically, under these statutes [referring to RA Nos. 965 and 2630], the person
desiring to reacquire Philippine citizenship would not even be required to file a petition in
court, and all that he had to do was to take an oath of allegiance to the Republic of the
Philippines and to register that fact with the civil registry in the place of his residence or
where he had last resided in the Philippines. [Italics in the original.25
Moreover, repatriation results in the recovery of the original nationality.26 This means that a
naturalized Filipino who lost his citizenship will be restored to his prior status as a naturalized Filipino
citizen. On the other hand, if he was originally a natural-born citizen before he lost his Philippine
citizenship, he will be restored to his former status as a natural-born Filipino.
In respondent Cruz's case, he lost his Filipino citizenship when he rendered service in the Armed
Forces of the United States. However, he subsequently reacquired Philippine citizenship under R.A.
No. 2630, which provides:
Section 1. Any person who had lost his Philippine citizenship by rendering service to, or
accepting commission in, the Armed Forces of the United States, or after separation from the
Armed Forces of the United States, acquired United States citizenship, may reacquire

Philippine citizenship by taking an oath of allegiance to the Republic of the Philippines and
registering the same with Local Civil Registry in the place where he resides or last resided in
the Philippines. The said oath of allegiance shall contain a renunciation of any other
citizenship.
Having thus taken the required oath of allegiance to the Republic and having registered the same in
the Civil Registry of Magantarem, Pangasinan in accordance with the aforecited provision,
respondent Cruz is deemed to have recovered his original status as a natural-born citizen, a status
which he acquired at birth as the son of a Filipino father.27 It bears stressing that the act of
repatriation allows him to recover, or return to, his original status before he lost his Philippine
citizenship.
Petitioner's contention that respondent Cruz is no longer a natural-born citizen since he had to
perform an act to regain his citizenship is untenable. As correctly explained by the HRET in its
decision, the term "natural-born citizen" was first defined in Article III, Section 4 of the 1973
Constitution as follows:
Sec. 4. A natural-born citizen is one who is a citizen of the Philippines from birth without
having to perform any act to acquire or perfect his Philippine citizenship.
Two requisites must concur for a person to be considered as such: (1) a person must be a Filipino
citizen birth and (2) he does not have to perform any act to obtain or perfect his Philippine
citizenship.
Under the 1973 Constitution definition, there were two categories of Filipino citizens which were not
considered natural-born: (1) those who were naturalized and (2) those born before January 17,
1973,38 of Filipino mothers who, upon reaching the age of majority, elected Philippine citizenship.
Those "naturalized citizens" were not considered natural-born obviously because they were not
Filipino at birth and had to perform an act to acquire Philippine citizenship. Those born of Filipino
mothers before the effectively of the 1973 Constitution were likewise not considered natural-born
because they also had to perform an act to perfect their Philippines citizenship.
The present Constitution, however, now consider those born of Filipino mothers before the effectivity
of the 1973 Constitution and who elected Philippine citizenship upon reaching the majority age as
natural-born. After defining who re natural-born citizens, Section 2 of Article IV adds a sentence:
"Those who elect Philippine citizenship in accordance with paragraph (3), Section 1 hereof shall be
deemed natural-born citizens." Consequently, only naturalized Filipinos are considered not naturalborn citizens. It is apparent from the enumeration of who are citizens under the present Constitution
that there are only two classes of citizens: (1) those who are natural-born and (2) those who are
naturalized in accordance with law. A citizen who is not a naturalized Filipino, i.e., did not have to
undergo the process of naturalization to obtain Philippine citizenship, necessarily is natural-born
Filipino. Noteworthy is the absence in said enumeration of a separate category for persons who,
after losing Philippine citizenship, subsequently reacquire it. The reason therefor is clear: as to such
persons, they would either be natural-born or naturalized depending on the reasons for the loss of
their citizenship and the mode prescribed by the applicable law for the reacquisition thereof. As
respondent Cruz was not required by law to go through naturalization proceeding in order to
reacquire his citizenship, he is perforce a natural-born Filipino. As such, he possessed all the
necessary qualifications to be elected as member of the House of Representatives.
A final point. The HRET has been empowered by the Constitution to be the "sole judge" of all
contests relating to the election, returns, and qualifications of the members of the House. 29 The
Court's jurisdiction over the HRET is merely to check "whether or not there has been a grave abuse

of discretion amounting to lack or excess of jurisdiction" on the part of the latter.30 In the absence
thereof, there is no occasion for the Court to exercise its corrective power and annul the decision of
the HRET nor to substitute the Court's judgement for that of the latter for the simple reason that it is
not the office of a petition for certiorari to inquire into the correctness of the assailed decision. 31 There
is no such showing of grave abuse of discretion in this case.
WHEREFORE, the petition is hereby DISMISSED.
SO ORDERED.

EN BANC
G.R. No. 142840

May 7, 2001

ANTONIO BENGSON III, petitioner,


vs.
HOUSE OF REPRESENTATIVES ELECTORAL TRIBUNAL and TEODORO C.
CRUZ, respondents.
CONCURRING OPINION
PANGANIBAN, J.:
I concur in the ponencia of Mr. Justice Santiago M. Kapunan, holding that the House Electoral
Tribunal did not gravely abuse its discretion in ruling that Private Respondent Teodoro C. Cruz
remains a natural-born Filipino citizen and is eligible to continue being a member of Congress. Let
me just add a few points.
The Facts in Brief
It is undisputed that Congressman Cruz was born on April 27, 1960 in San Clemente, Tarlac, to
Filipino parents. He was, therefore, a Filipino citizen, pursuant to Section 1 (2), 1 Article IV of the
Constitution. Furthermore, not having done any act to acquire or perfect the Philippine
citizenship he obtained from birth, he was a natural-born Filipino citizen, in accordance with
Section 22 of the same Article IV.
It is not disputed either that private respondent rendered military service to the United States Marine
Corps from November 1958 to October 1993. On June 5, 1990, he was naturalized as an American
citizen, in connection with his US military service. Consequently, under Section 1 (4) 3 of CA No. 63,
he lost his Philippine citizenship.
Upon his discharge from the US Marine Corps, private respondent returned to the Philippines and
decided to regain his Filipino citizenship. Thus, on March 17, 1994, availing himself of the benefits of
Republic Act (RA) No. 2630, entitled "An Act Providing for Reacquisition of Philippine Citizenship by
Persons Who Lost Such by Rendering Service to, or Accepting Commission in, the Armed Force of
the United States,"4 Cruz took his oath of allegiance to the Republic and registered the same with the
Local Civil Registry of Mangatarem, Pangasinan. On the same day, he also executed an Affidavit of
Reacquisition of Philippine Citizenship.

Main Issue
The main question here is: Did the House of Representatives Electoral Tribunal (HRET) commit
grave abuse of discretion in holding that, by reason of his repatriation, Congressman Teodoro C.
Cruz had reverted to his original status as a natural-born citizen? I respectfully submit that the
answer is "No." In fact, I believe that the HRET was correct in its ruling.
1. Repatriation Is Recovery of Original Citizenship
First, repatriation is simply the recovery of original citizenship. Under Section 1 of RA 2630, a person
"who ha[s] lost his citizenship" may "reacquire" it by " taking an oath of allegiance to the Republic of
the Philippines." Former Senate President Jovito R. Salonga, a noted authority on the subject,
explains this method more precisely in his treatise, Private International Law.5 He defines repatriation
as "the recovery of the original nationality upon fulfillment of certain condition."6 Webster buttresses
this definition by describing the ordinary or common usage ofrepatriate, as "to restore or return to
one's country of origin, allegiance, or citizenship; x x x."7 In relation to our subject
matter, repatriation, then, means restoration of citizenship. It is not a grant of a new citizenship, but a
recovery of one's former or original citizenship.
To "reacquire" simply means "to get back as one's own again." 8 Ergo, since Cruz, prior to his
becoming a US citizen, was a natural-born Filipino citizen, he "reacquired" the same status upon
repatriation. To rule otherwise that Cruz became a non-natural-born citizen would not be
consistent whit the legal and ordinary meaning of repatriation. It would be akin to naturalization,
which is the acquisition of a new citizenship. "New." Because it is not the same as the with which he
has previously been endowed.
In any case, "the leaning, in questions of citizenship, should always be in favor of [its] claimant x x
x."9Accordingly, the same should be construed in favor of private respondent, who claims to be a
natural-born citizen.
2. Not Being Naturalized, Respondent Is Natural Born
Second, under the present Constitution, private respondent should be deemed natural-born,
because was not naturalized. Let me explain.
There are generally two classes of citizens: (1) natural-born citizens and (2) naturalized
citizens.10 While CA 63 provides that citizenship may also be acquired by direct act of the
Legislature, I believe that those who do become citizens through such procedure would properly fall
under the second category (naturalized).11
Naturalized citizens are former aliens or foreigners who had to undergo a rigid procedure, in which
they had to adduce sufficient evidence to prove that they possessed all the qualifications and none
of the disqualifications provided by law in order to become Filipino citizens. In contrast, as stated in
the early case Roa v. Collector of Customs,12 a natural-born citizen is a citizen "who has become
such at the moment of his birth."
The assailed HRET Decision, penned by Mr. Justice Vicente V. Mendoza, explains clearly who are
considered natural-born Filipino citizens. He traces the concept as first defined in Article III of the
1973 Constitution, which simply provided as follows:

"Sec 4. A natural-born citizen is one who is a citizen of the Philippines from birth without
having to perform any act to acquire or perfect his Philippine citizenship."
Under the above definition, there are two requisites in order that a Filipino citizen may be considered
"natural-born": (1) one must be a citizen of the Philippines from birth, and (2) one does not have to
do anything to acquire or perfect one's Philippine citizenship. 13 Thus, under the 1973 Constitution,
excluded from the class of "natural-born citizens" were (1) those who were naturalized and (2) those
born before January 17, 1973, of Filipino mothers who, upon reaching the age of majority, elected
Philippine citizenship.14
The present Constitution, however, has expanded the scope of natural-born citizens to include
"[t]hose who elect Philippine citizenship in accordance with paragraph (3), Section 1 hereof,"
meaning those covered under class (2) above. Consequently, only naturalized Filipino citizens are
not considered natural-born citizens. Premising therefrom, respondent being clearly and
concededly not naturalized is, therefore, a natural-born citizen of the Philippines. 15
With respect to repatriates, since the Constitution does not classify them separately, they naturally
reacquire theiroriginal classification before the loss of their Philippine citizenship. In the case of
Congressman Teodoro C. Cruz, upon his repatriation in1994, he reacquired his lost citizenship. In
other words, he regained his original status as a natural-born Filipino citizen, nothing less.
3. No Grave Abuse of Discretion on the Part of HRET
Third, the HRET did not abuse, much less gravely abuse, its discretion in holding that Respondent
Cruz is a natural-born Filipino citizen who is qualified to be a member of Congress. I stress that the
Court, in this certiorari proceeding before us, is limited to determining whether the HRET committed
grave abuse of discretion amounting to lack or excess of jurisdiction in issuing its assailed Decision.
The Court has no power to reverse or modify HRET's rulings, simply because it differs in its
perception of controversies. It cannot substitute its discretion for that of HRET, an independent,
constitutional body with its own specific mandate.
The Constitution explicitly states that the respective Electoral Tribunals of the chambers of Congress
"shall be thesole judges of all contests relating to the election, returns, and qualifications their
respective members."16 In several cases,17 this Court has held that the power and the jurisdiction of
the Electoral Tribunals are original and exclusive, as if they remained in the legislature, a coequal
branch of government. Their judgment are beyond judicial interference, unless rendered without or in
excess of their jurisdiction or with grave abuse of discretion. 18In the elegant words of Mr. Justice
Hugo E. Gutierrez Jr.:19
"The Court does not venture into the perilous area of trying to correct perceived errors of
independent branches of the Government. It comes in only when it has to vindicate a denial
of due process or correct an abuse of discretion so grave or glaring that no less than the
Constitution calls for remedial action."
True, there is no settled judicial doctrine on the exact effect of repatriation. But, as earlier explained,
the legal and common definition of repatriation is the reacquisition of the former citizenship. How
then can the HRET be rebuked with grave abuse of discretion? At best, I can concede that the legal
definition is not judicially settled or is even doubtful. But an interpretation made in good faith and
grounded o reason one way or the other cannot be the source of grave abuse amounting to lack or
excess of jurisdiction. The HRET did not violate the Constitution or the law or any settled judicial
doctrine. It was definitely acting within its exclusive domain.

Be it remembered that our Constitution vests upon the HRET the power to be the sole judge of the
qualifications of members of the House of Representatives, one of which is citizenship. Absent
any clear showing of a manifest violation of the Constitution or the law or nay judicial decision, this
Court cannot impute grave abuse of discretion to the HRET in the latter's actions on matters over
which full discretionary authority is lodged upon it by our fundamental law.20 Even assuming that we
disagree with the conclusion of public respondent, we cannot ipsofacto attribute to it "grave abuse of
discretion." Verily, there is a line between perceived error and grave abuse. 21
By grave abuse of discretion is meant such capricious and whimsical exercise of judgment as is
equivalent to lack of jurisdiction. Mere abuse of discretion is not enough. "It must be grave abuse of
discretion as when the power is exercised in an arbitrary or despotic manner by reason of passion or
personal hostility, and must be so patent and so gross as to amount to an evasion of a positive duty
or to a virtual refusal to perform the duty enjoined or to act at all in contemplation of law." 22
That the HRET, after careful deliberation and purposeful study, voted 7 to 2 to issue its Decision
upholding the qualifications of Congressman Cruz could not in any wise be condemned as gravely
abusive. Neither can I find any "patent or gross" arbitrariness or despotism "by reason of passion or
hostility" in such exercise.
4. In Case of Doubt, Popular Will Prevails
Fourth, the court has a solemn duty to uphold the clear and unmistakable mandate of the people. It
cannot supplant the sovereign will of the Second District of Pangasinan with fractured legalism. The
people of the District have clearly spoken. They overwhelmingly and unequivocally voted for private
respondent to represent them in the House of Representatives. The votes that Cruz garnered (80,
119) in the last elections were much more than those of all his opponents combined (66, 182). 23 In
such instances, all possible doubts should be resolved in favor of the winning candidate's eligibility;
to rule otherwise would be to defeat the will of the people.24
Well-entrenched in our jurisprudence is the doctrine that in case of doubt, political laws must be so
constructed as to give life and spirit to the popular mandate freely expressed through the
ballot.25 Public interest and the sovereign will should, at all times, be the paramount considerations in
election controversies.26 For it would be better to err in favor of the people's choice than to be right in
complex but little understood legalisms.27
"Indeed, this Court has repeatedly stressed the importance of giving effect to the sovereign will in
order to ensure the survival of our democracy. In any action involving the possibility of a reversal of
the popular electoral choice, this Court must exert utmost effort to resolve the issues in a manner
that would give effect to the will of the majority, for it is merely sound public policy to cause elective
offices to be filled by those who are the choice of the majority. To successfully challenge a winning
candidate's qualifications, the petitioner must clearly demonstrative that the ineligibility is so patently
antagonistic to constitutional and legal principles that overriding such ineligibility and thereby giving
effect to the apparent will of the people would ultimately create greater prejudice to the very
democratic institutions and juristic traditions that our Constitution and laws so zealously protect and
promote."28
5. Current Trend Towards Globalization
Fifth, the current trend, economically as well as politically, is towards globalization. 29 Protectionist
barriers dismantled. Whereas, in the past, governments frowned upon the opening of their doors to
aliens who wanted to enjoy the same privileges as their citizens, the current era is adopting a more
liberal perspective. No longer are applicants for citizenship eyed with the suspicion that they merely

want to exploit local resources for themselves. They are now being considered potential sources of
developmental skills, know-how and capital.
1wphi1.nt

More so should our government open its doors to former Filipinos, like Congressman Cruz, who
want to rejoin the Filipino community as citizens again. They are not "aliens" in the true sense of the
law. They are actually Filipino by blood, by origin and by culture, who want to reacquire their former
citizenship.
It cannot be denied that most Filipinos go abroad and apply for naturalization in foreign countries,
because of the great economic or social opportunities there. Hence, we should welcome former
Filipino citizens desirous of not simply returning to the country or regaining Philippine citizenship, but
of serving the Filipino people as well. One of these admirable Filipino is private respondent who, in
only a year after being absent from the Philippines for about eight (8) years, was already voted
municipal mayor of Mangatarem, Pangasinan. And after serving as such for just one term, he was
overwhelmingly chosen by the people to be their representative in Congress.
I reiterate, the people have spoken. Let not a restrictive and parochial interpretation of the law bar
the sovereign will. Let not grave abuse be imputed on the legitimate exercise of HRET's
prerogatives.
WHEREFORE, I vote to DISMISS the petition.
Republic of the Philippines
SUPREME COURT
Manila
THIRD DIVISION
G.R. No. 125793 August 29, 2006
JOEVANIE ARELLANO TABASA, Petitioner,
vs.
HON. COURT OF APPEALS, BUREAU OF IMMIGRATION and DEPORTATION and WILSON
SOLUREN,Respondents.
DECISION
VELASCO, JR., J.:
Citizenship is a priceless possession. Former U.S. Chief Justice Earl Warren fittingly emphasized its
crowning value when he wrote that "it is mans basic right for it is nothing less than to have
rights." 1 When a person loses citizenship, therefore, the State sees to it that its reacquisition may
only be granted if the former citizen fully satisfies all conditions and complies with the applicable law.
Without doubt, repatriation is not to be granted simply based on the vagaries of the former Filipino
citizen.
The Case

The instant petition for review 2 under Rule 45 of the 1997 Rules of Civil Procedure contests the
denial by the Court of Appeals (CA) of the Petition for Habeas Corpus interposed by petitioner
Joevanie Arellano Tabasa from the Order of Summary Deportation issued by the Bureau of
Immigration and Deportation (BID) for his return to the United States.
The Facts
The facts as culled by the CA from the records show that petitioner Joevanie Arellano Tabasa was a
natural-born citizen of the Philippines. In 1968, 3 when petitioner was seven years old, 4 his father,
Rodolfo Tabasa, became a naturalized citizen 5 of the United States. By derivative naturalization
(citizenship derived from that of another as from a person who holds citizenship by virtue of
naturalization 6), petitioner also acquired American citizenship.
Petitioner arrived in the Philippines on August 3, 1995, and was admitted as a "balikbayan" for one
year. Thereafter, petitioner was arrested and detained by agent Wilson Soluren of the BID on May
23, 1996, pursuant to BID Mission Order No. LIV-96-72 in Baybay, Malay, Aklan; subsequently, he
was brought to the BID Detention Center in Manila. 7
Petitioner was investigated by Special Prosecutor Atty. Edy D. Donato at the Law and Investigation
Division of the BID on May 28, 1996; and on the same day, Tabasa was accused of violating Section
8, Chapter 3, Title 1, Book 3 of the 1987 Administrative Code, in a charge sheet which alleged:
1. That on 3 August 1995, respondent (petitioner herein [Tabasa]) arrived in the Philippines and was
admitted as a balikbayan;
2. That in a letter dated 16 April 1996, Honorable Kevin Herbert, Consul General of [the] U.S.
Embassy, informed the Bureau that respondents Passport No. 053854189 issued on June 10, 1994
in San Francisco, California, U.S.A., had been revoked by the U.S. Department of State;
3. Hence, respondent [petitioner Tabasa] is now an undocumented and undesirable alien and may
be summarily deported pursuant to Law and Intelligence Instructions No. 53 issued by then
Commissioner Miriam Defensor Santiago to effect his deportation (Exhibit 3). 8
The pertinent portion of the Herbert letter is as follows:
The U.S. Department of State has revoked U.S. passport 053854189 issued on June 10, 1994 in
San Francisco, California under the name of Joevanie Arellano Tabasa, born on February 21, 1959
in the Philippines. Mr. Tabasas passport has been revoked because he is the subject of an
outstanding federal warrant of arrest issued on January 25, 1996 by the U.S. District Court for the
Northern District of California, for violation of Section 1073, "Unlawful Flight to Avoid Prosecution," of
Title 18 of the United States Code. He is charged with one count of a felon in possession of a
firearm, in violation of California Penal Code, Section 12021(A)(1), and one count of sexual battery,
in violation of California Penal Code, Section 243.4 (D). 9
The BID ordered petitioners deportation to his country of origin, the United States, on May 29, 1996,
in the following summary deportation order:

Records show that on 16 April 1996, Mr. Kevin F. Herbert, Consul General of the U.S. Embassy in
Manila, filed a request with the Bureau to apprehend and deport the abovenamed [sic] respondent
[petitioner Tabasa] on the ground that a standing warrant for several federal charges has been
issued against him, and that the respondents Passport No. 053854189 has been revoked.
By reason thereof, and on the strength of Mission Order No. LIV-96-72, Intelligence operatives
apprehended the respondent in Aklan on 23 May 1996.
In Schonemann vs. Commissioner Santiago, et al., (G.R. No. 81461 [sic, 81461 should be 86461],
30 May 1989), the Supreme Court ruled that if a foreign embassy cancels the passport of an alien,
or does not reissue a valid passport to him, the alien loses the privilege to remain in the country.
Further, under Office Memorandum Order No. 34 issued on 21 August 1989, summary deportation
proceedings lie where the passport of the alien has expired.
It is, thus, apparent that respondent has lost his privilege to remain in the country. 10
Petitioner filed before the CA a Petition for Habeas Corpus with Preliminary Injunction and/or
Temporary Restraining Order 11 on May 29, 1996, which was docketed as CA-G.R. SP No. 40771.
Tabasa alleged that he was not afforded due process; that no warrant of arrest for deportation may
be issued by immigration authorities before a final order of deportation is made; that no notice of the
cancellation of his passport was made by the U.S. Embassy; that he is entitled to admission or to a
change of his immigration status as a non-quota immigrant because he is married to a Filipino
citizen as provided in Section 13, paragraph (a) of the Philippine Immigration Act of 1940; and that
he was a natural-born citizen of the Philippines prior to his derivative naturalization when he was
seven years old due to the naturalization of his father, Rodolfo Tabasa, in 1968.
At the time Tabasa filed said petition, he was already 35 years old.

12

On May 30, 1996, the CA ordered the respondent Bureau to produce the person of the petitioner on
June 3, 1996 and show the cause of petitioners detention, and restrained the Bureau from
summarily deporting him. On June 3, 1996, the BID presented Tabasa before the CA; and on June
6, 1996, the CA granted both parties ten (10) days within which to file their memoranda, after which
the case would be considered submitted for decision. 13Meanwhile, the Commissioner of Immigration
granted the petitioners temporary release on bail on a PhP 20,000.00 cash bond. 14
However, on June 13, 1996, petitioner filed a Supplemental Petition alleging that he had acquired
Filipino citizenship by repatriation in accordance with Republic Act No. 8171 (RA 8171), and that
because he is now a Filipino citizen, he cannot be deported or detained by the respondent Bureau.

15

The Ruling of the Court of Appeals


The CA, in its August 7, 1996 Decision, 16 denied Tabasas petition on the ground that he had not
legally and successfully acquiredby repatriationhis Filipino citizenship as provided in RA 8171.
The court said that although he became an American citizen by derivative naturalization when his
father was naturalized in 1968, there is no evidence to show that he lost his Philippine citizenship
"on account of political or economic necessity," as explicitly provided in Section 1, RA 8171the law

governing the repatriation of natural-born Filipinos who have lost their citizenship. The affidavit does
not state that political or economic necessity was the compelling reason for petitioners parents to
give up their Filipino citizenship in 1968. Moreover, the court a quo found that petitioner Tabasa did
not dispute the truth of the April 16, 1996 letter of the United States Consul General Kevin F. Herbert
or the various warrants issued for his arrest by the United States court. The court a quo noted that
after petitioner was ordered deported by the BID on May 29, 1996, he successively executed an
Affidavit of Repatriation on June 6, 1996 and took an oath of allegiance to the Republic of the
Philippines on June 13, 1996more than ten months after his arrival in the country on August 3,
1995. The appellate court considered petitioners "repatriation" as a last ditch effort to avoid
deportation and prosecution in the United States. The appellate court concluded that his only reason
to want to reacquire Filipino citizenship is to avoid criminal prosecution in the United States of
America. The court a quo, therefore, ruled against Tabasa, whose petition is now before us.
The Issue
The only issue to be resolved is whether petitioner has validly reacquired Philippine citizenship
under RA 8171. If there is no valid repatriation, then he can be summarily deported for his being an
undocumented alien.
The Courts Ruling
The Court finds no merit in this petition.
RA 8171, "An Act Providing for the Repatriation of Filipino Women Who Have Lost Their Philippine
Citizenship by Marriage to Aliens and of Natural-Born Filipinos," was enacted on October 23, 1995. It
provides for the repatriation of only two (2) classes of persons, viz:
Filipino women who have lost their Philippine citizenship by marriage to aliens and natural-born
Filipinos who have lost their Philippine citizenship, including their minor children, on account of
political or economic necessity, may reacquire Philippine citizenship through repatriation in the
manner provided in Section 4 of Commonwealth Act No. 63, as amended: Provided, That the
applicant is not a:
(1) Person opposed to organized government or affiliated with any association or group of persons
who uphold and teach doctrines opposing organized government;
(2) Person defending or teaching the necessity or propriety of violence, personal assault, or
association for the predominance of their ideas;
(3) Person convicted of crimes involving moral turpitude; or
(4) Person suffering from mental alienation or incurable contagious diseases.

17

(Emphasis supplied.)

Does petitioner Tabasa qualify as a natural-born Filipino who had lost his Philippine citizenship by
reason of political or economic necessity under RA 8171?

He does not.
Persons qualified for repatriation under RA 8171
To reiterate, the only persons entitled to repatriation under RA 8171 are the following:
a. Filipino women who lost their Philippine citizenship by marriage to aliens; and
b. Natural-born Filipinos including their minor children who lost their Philippine citizenship on account
of political or economic necessity.
Petitioner theorizes that he could be repatriated under RA 8171 because he is a child of a naturalborn Filipino, and that he lost his Philippine citizenship by derivative naturalization when he was still
a minor.
Petitioner overlooks the fact that the privilege of repatriation under RA 8171 is available only to
natural-born Filipinos who lost their citizenship on account of political or economic necessity, and to
the minor children of said natural-born Filipinos. This means that if a parent who had renounced his
Philippine citizenship due to political or economic reasons later decides to repatriate under RA 8171,
his repatriation will also benefit his minor children according to the law. This includes a situation
where a former Filipino subsequently had children while he was a naturalized citizen of a foreign
country. The repatriation of the former Filipino will allow him to recover his natural-born citizenship
and automatically vest Philippine citizenship on his children of jus sanguinis or blood
relationship:18 the children acquire the citizenship of their parent(s) who are natural-born Filipinos. To
claim the benefit of RA 8171, however, the children must be of minor age at the time the petition for
repatriation is filed by the parent. This is so because a child does not have the legal capacity for all
acts of civil life much less the capacity to undertake a political act like the election of citizenship. On
their own, the minor children cannot apply for repatriation or naturalization separately from their
parents.
In the case at bar, there is no dispute that petitioner was a Filipino at birth. In 1968, while he was still
a minor, his father was naturalized as an American citizen; and by derivative naturalization, petitioner
acquired U.S. citizenship. Petitioner now wants us to believe that he is entitled to automatic
repatriation as a child of natural-born Filipinos who left the country due to political or economic
necessity. This is absurd. Petitioner was no longer a minor at the time of his "repatriation" on June
13, 1996. The privilege under RA 8171 belongs to children who are of minor age at the time of the
filing of the petition for repatriation.
Neither can petitioner be a natural-born Filipino who left the country due to political or economic
necessity. Clearly, he lost his Philippine citizenship by operation of law and not due to political or
economic exigencies. It was his father who could have been motivated by economic or political
reasons in deciding to apply for naturalization. The decision was his parents and not his. The
privilege of repatriation under RA 8171 is extended directly to the natural-born Filipinos who could
prove that they acquired citizenship of a foreign country due to political and economic reasons, and
extended indirectly to the minor children at the time of repatriation.

In sum, petitioner is not qualified to avail himself of repatriation under RA 8171. However, he can
possibly reacquire Philippine citizenship by availing of the Citizenship Retention and Re-acquisition
Act of 2003 (Republic Act No. 9225) by simply taking an oath of allegiance to the Republic of the
Philippines.
Where to file a petition for repatriation pursuant to RA 8171
Even if we concede that petitioner Tabasa can avail of the benefit of RA 8171, still he failed to follow
the procedure for reacquisition of Philippine citizenship. He has to file his petition for repatriation with
the Special Committee on Naturalization (SCN), which was designated to process petitions for
repatriation pursuant to Administrative Order No. 285 (A.O. No. 285) dated August 22, 1996, to wit:
Section 1. Composition.The composition of the Special Committee on Naturalization, with the
Solicitor General as Chairman, the Undersecretary of Foreign Affairs and the Director-General of the
National Intelligence Coordinating Agency, as members, shall remain as constituted.
Sec. 2. Procedure.Any person desirous of repatriating or reacquiring Filipino citizenship pursuant
to R.A. No. 8171 shall file a petition with the Special Committee on Naturalization which shall
process the same. If their applications are approved[,] they shall take the necessary oath of
allegiance to the Republic of the Philippines, after which they shall be deemed to have reacquired
Philippine citizenship. The Commission on Immigration and Deportation shall thereupon cancel their
certificate of registration (emphasis supplied).
Sec. 3. Implementing Rules.The Special Committee is hereby authorized to promulgate rules and
regulations and prescribe the appropriate forms and the required fees for the processing of petitions.
Sec. 4. Effectivity.This Administrative Order shall take effect immediately.
In the Amended Rules and Regulations Implementing RA 8171 issued by the SCN on August 5,
1999, applicants for repatriation are required to submit documents in support of their petition such as
their birth certificate and other evidence proving their claim to Filipino citizenship. 19 These
requirements were imposed to enable the SCN to verify the qualifications of the applicant particularly
in light of the reasons for the renunciation of Philippine citizenship.
What petitioner simply did was that he took his oath of allegiance to the Republic of the Philippines;
then, executed an affidavit of repatriation, which he registered, together with the certificate of live
birth, with the Office of the Local Civil Registrar of Manila. The said office subsequently issued him a
certificate of such registration. 20At that time, the SCN was already in place and operational by virtue
of the June 8, 1995 Memorandum issued by President Fidel V. Ramos. 21 Although A.O. No. 285
designating the SCN to process petitions filed pursuant to RA 8171 was issued only on August 22,
1996, it is merely a confirmatory issuance according to the Court in Angat v. Republic. 22 Thus,
petitioner should have instead filed a petition for repatriation before the SCN.
Requirements for repatriation under RA 8171

Even if petitionernow of legal agecan still apply for repatriation under RA 8171, he nevertheless
failed to prove that his parents relinquished their Philippine citizenship on account of political or
economic necessity as provided for in the law. Nowhere in his affidavit of repatriation did he mention
that his parents lost their Philippine citizenship on account of political or economic reasons. It is
notable that under the Amended Rules and Regulations Implementing RA 8171, the SCN requires a
petitioner for repatriation to set forth, among others, "the reason/s why petitioner lost his/her Filipino
citizenship, whether by marriage in case of Filipino woman, or whether by political or economic
necessity in case of [a] natural-born Filipino citizen who lost his/her Filipino citizenship. In case of the
latter, such political or economic necessity should be specified." 23
Petitioner Tabasa asserts, however, that the CA erred in ruling that the applicant for repatriation must
prove that he lost his Philippine citizenship on account of political or economic necessity. He
theorizes that the reference to political or economic reasons is "merely descriptive, not restrictive, of
the widely accepted reasons for naturalization in [a] foreign country." 24
Petitioners argument has no leg to stand on.
A reading of Section 1 of RA 8171 shows the manifest intent of the legislature to limit the benefit of
repatriation only to natural-born Filipinos who lost their Philippine citizenship on account of political
or economic necessity, in addition to Filipino women who lost their Philippine citizenship by marriage
to aliens. The precursor of RA 8171, Presidential Decree No. 725 (P.D. 725), 25 which was enacted
on June 5, 1975 amending Commonwealth Act No. 63, also gives to the same groups of former
Filipinos the opportunity to repatriate but without the limiting phrase, "on account of political or
economic necessity" in relation to natural-born Filipinos. By adding the said phrase to RA 8171, the
lawmakers clearly intended to limit the application of the law only to political or economic migrants,
aside from the Filipino women who lost their citizenship by marriage to aliens. This intention is more
evident in the following sponsorship speech of Rep. Andrea B. Domingo on House Bill No. 1248, the
origin of RA 8171, to wit:
Ms. Domingo: x x x
From my experience as the Commissioner of the Bureau of Immigration and Deportation, I observed
that there are only four types of Filipinos who leave the country.
The first is what we call the "economic refugees" who go abroad to work because there is no work to
be found in the country. Then we have the "political refugees" who leave the country for fear of their
lives because they are not in consonance with the prevailing policy of government. The third type is
those who have committed crimes and would like to escape from the punishment of said crimes.
Lastly, we have those Filipinos who feel that they are not Filipinos, thereby seeking other citizenship
elsewhere.
Of these four types of Filipinos, Mr. Speaker, the first two have to leave the country not of choice, but
rather out of sacrifice to look for a better life, as well as for a safer abode for themselves and their
families. It is for these two types of Filipinos that this measure is being proposed for approval by this
body. (Emphasis supplied.)

xxxx
x x x [I]f the body would recall, I mentioned in my short sponsorship speech the four types of
Filipinos who leave their country. And the two typesthe economic and political refugeesare the
ones being addressed by this proposed law, and they are not really Filipino women who lost their
citizenship through marriage. We had a lot of problems with these people who left the country
because of political persecution or because of pressing economic reasons, and after feeling that
they should come back to the country and get back their citizenship and participate as they should in
the affairs of the country, they find that it is extremely difficult to get their citizenship back because
they are treated no different from any other class of alien. 26
From these two sources, namely, P.D. 725 and the sponsorship speech on House Bill No. 1248, it is
incontrovertible that the intent of our legislators in crafting Section 1 of RA 8171, as it is precisely
worded out, is to exclude those Filipinos who have abandoned their country for reasons other than
political or economic necessity.
Petitioner contends it is not necessary to prove his political or economic reasons since the act of
renouncing allegiance to ones native country constitutes a "necessary and unavoidable shifting of
his political allegiance," and his fathers loss of Philippine citizenship through naturalization "cannot
therefore be said to be for any reason other than political or economic necessity." 27
This argument has no merit.
While it is true that renunciation of allegiance to ones native country is necessarily a political act, it
does not follow that the act is inevitably politically or economically motivated as alleged by petitioner.
To reiterate, there are other reasons why Filipinos relinquish their Philippine citizenship. The
sponsorship speech of former Congresswoman Andrea B. Domingo illustrates that aside from
economic and political refugees, there are Filipinos who leave the country because they have
committed crimes and would like to escape from punishment, and those who really feel that they are
not Filipinos and that they deserve a better nationality, and therefore seek citizenship elsewhere.
Thus, assuming petitioner Tabasa is qualified under RA 8171, it is incumbent upon him to prove to
the satisfaction of the SCN that the reason for his loss of citizenship was the decision of his parents
to forfeit their Philippine citizenship for political or economic exigencies. He failed to undertake this
crucial step, and thus, the sought relief is unsuccessful.
Repatriation is not a matter of right, but it is a privilege granted by the State. This is mandated by the
1987 Constitution under Section 3, Article IV, which provides that citizenship may be lost or
reacquired in the manner provided by law. The State has the power to prescribe by law the
qualifications, procedure, and requirements for repatriation. It has the power to determine if an
applicant for repatriation meets the requirements of the law for it is an inherent power of the State to
choose who will be its citizens, and who can reacquire citizenship once it is lost. If the applicant, like
petitioner Tabasa, fails to comply with said requirements, the State is justified in rejecting the petition
for repatriation.
Petitioner: an undocumented alien subject to summary deportation

Petitioner claims that because of his repatriation, he has reacquired his Philippine citizenship;
therefore, he is not an undocumented alien subject to deportation.
This theory is incorrect.
As previously explained, petitioner is not entitled to repatriation under RA 8171 for he has not shown
that his case falls within the coverage of the law.
Office Memorandum No. 34 dated August 21, 1989 of the BID is enlightening on summary
deportation:
2. The Board of Special Inquiry and the Hearing Board IV shall observe summary deportation
proceedings in cases where the charge against the alien is overstaying, or the expiration or
cancellation by his government of his passport. In cases involving overstaying aliens, BSI and the
Hearing Board IV shall merely require the presentation of the aliens valid passport and shall decide
the case on the basis thereof.
3. If a foreign embassy cancels the passport of the alien, or does not reissue a valid passport to him,
the alien loses the privilege to remain in the country, under the Immigration Act, Sections 10 and 15
(Schonemann v. Santiago, et al., G.R. No. 81461 [sic, should be 86461], 30 May 1989). The
automatic loss of the privilege obviates deportation proceedings. In such instance, the Board of
Commissioners may issue summary judgment of deportation which shall be immediately
executory. 28
In addition, in the case of Schonemann v. Defensor Santiago, et al., this Court held:
It is elementary that if an alien wants to stay in the Philippines, he must possess the necessary
documents. One of these documents is a valid passport. There are, of course, exceptions where in
the exercise of its sovereign prerogatives the Philippines may grant refugee status, refuse to
extradite an alien, or otherwise allow him or her to stay here even if he [the alien] has no valid
passport or Philippine visa. "Boat people" seeking residence elsewhere are examples. However, the
grant of the privilege of staying in the Philippines is discretionary on the part of the proper
authorities. There is no showing of any grave abuse of discretion, arbitrariness, or whimsicality in the
questioned summary judgment. x x x 29
Petitioner Tabasa, whose passport was cancelled after his admission into the country, became an
undocumented alien who can be summarily deported. His subsequent "repatriation" cannot bar such
deportation especially considering that he has no legal and valid reacquisition of Philippine
citizenship.
WHEREFORE, this petition for review is DISMISSED, and the August 7, 1996 Decision of the Court
of Appeals isAFFIRMED. No costs to the petitioner.
SO ORDERED.

Republic of the Philippines


SUPREME COURT
Manila
EN BANC
G.R. No. 177597

July 16, 2008

BAI SANDRA S. A. SEMA, Petitioner,


vs.
COMMISSION ON ELECTIONS and DIDAGEN P. DILANGALEN, Respondents.
x - - - - - - - - - - - - - - - - - - - - - - -x
G.R. No. 178628
PERFECTO F. MARQUEZ, Petitioner,
vs.
COMMISSION ON ELECTIONS, Respondent.
DECISION
CARPIO, J.:
The Case
These consolidated petitions1 seek to annul Resolution No. 7902, dated 10 May 2007, of the
Commission on Elections (COMELEC) treating Cotabato City as part of the legislative district of the
Province of Shariff Kabunsuan.2
The Facts
The Ordinance appended to the 1987 Constitution apportioned two legislative districts for the
Province of Maguindanao. The first legislative district consists of Cotabato City and eight
municipalities.3 Maguindanao forms part of the Autonomous Region in Muslim Mindanao (ARMM),
created under its Organic Act, Republic Act No. 6734 (RA 6734), as amended by Republic Act No.
9054 (RA 9054).4 Although under the Ordinance, Cotabato City forms part of Maguindanaos first
legislative district, it is not part of the ARMM but of Region XII, having voted against its inclusion in
the ARMM in the plebiscite held in November 1989.
On 28 August 2006, the ARMMs legislature, the ARMM Regional Assembly, exercising its power to
create provinces under Section 19, Article VI of RA 9054, 5 enacted Muslim Mindanao Autonomy Act
No. 201 (MMA Act 201) creating the Province of Shariff Kabunsuan composed of the eight
municipalities in the first district of Maguindanao. MMA Act 201 provides:
Section 1. The Municipalities of Barira, Buldon, Datu Odin Sinsuat, Kabuntalan, Matanog, Parang,
Sultan Kudarat, Sultan Mastura, and Upi are hereby separated from the Province of Maguindanao
and constituted into a distinct and independent province, which is hereby created, to be known as
the Province of Shariff Kabunsuan.
xxxx

Sec. 5. The corporate existence of this province shall commence upon the appointment by the
Regional Governor or election of the governor and majority of the regular members of the
Sangguniang Panlalawigan.
The incumbent elective provincial officials of the Province of Maguindanao shall continue to serve
their unexpired terms in the province that they will choose or where they are residents: Provided,
that where an elective position in both provinces becomes vacant as a consequence of the creation
of the Province of Shariff Kabunsuan, all incumbent elective provincial officials shall have preference
for appointment to a higher elective vacant position and for the time being be appointed by the
Regional Governor, and shall hold office until their successors shall have been elected and qualified
in the next local elections; Provided, further, that they shall continue to receive the salaries they are
receiving at the time of the approval of this Act until the new readjustment of salaries in accordance
with law. Provided, furthermore, that there shall be no diminution in the number of the members of
the Sangguniang Panlalawigan of the mother province.
Except as may be provided by national law, the existing legislative district, which includes Cotabato
as a part thereof, shall remain.
Later, three new municipalities6 were carved out of the original nine municipalities constituting Shariff
Kabunsuan, bringing its total number of municipalities to 11. Thus, what was left of Maguindanao
were the municipalities constituting its second legislative district. Cotabato City, although part of
Maguindanaos first legislative district, is not part of the Province of Maguindanao.
The voters of Maguindanao ratified Shariff Kabunsuans creation in a plebiscite held on 29 October
2006.
On 6 February 2007, the Sangguniang Panlungsod of Cotabato City passed Resolution No. 3999
requesting the COMELEC to "clarify the status of Cotabato City in view of the conversion of the First
District of Maguindanao into a regular province" under MMA Act 201.
In answer to Cotabato Citys query, the COMELEC issued Resolution No. 07-0407 on 6 March 2007
"maintaining the status quo with Cotabato City as part of Shariff Kabunsuan in the First Legislative
District of Maguindanao." Resolution No. 07-0407, which adopted the recommendation of the
COMELECs Law Department under a Memorandum dated 27 February 2007,7 provides in pertinent
parts:
Considering the foregoing, the Commission RESOLVED, as it hereby resolves, to adopt the
recommendation of the Law Department that pending the enactment of the appropriate law by
Congress, to maintain the status quo with Cotabato City as part of Shariff Kabunsuan in the First
Legislative District of Maguindanao. (Emphasis supplied)
However, in preparation for the 14 May 2007 elections, the COMELEC promulgated on 29 March
2007 Resolution No. 7845 stating that Maguindanaos first legislative district is composed only of
Cotabato City because of the enactment of MMA Act 201. 8
On 10 May 2007, the COMELEC issued Resolution No. 7902, subject of these petitions, amending
Resolution No. 07-0407 by renaming the legislative district in question as "Shariff Kabunsuan
Province with Cotabato City (formerly First District of Maguindanao with Cotabato City)." 9
1avvphi1

In G.R. No. 177597, Sema, who was a candidate in the 14 May 2007 elections for Representative of
"Shariff Kabunsuan with Cotabato City," prayed for the nullification of COMELEC Resolution No.

7902 and the exclusion from canvassing of the votes cast in Cotabato City for that office. Sema
contended that Shariff Kabunsuan is entitled to one representative in Congress under Section 5 (3),
Article VI of the Constitution10 and Section 3 of the Ordinance appended to the Constitution. 11 Thus,
Sema asserted that the COMELEC acted without or in excess of its jurisdiction in issuing Resolution
No. 7902 which maintained the status quo in Maguindanaos first legislative district despite the
COMELECs earlier directive in Resolution No. 7845 designating Cotabato City as the lone
component of Maguindanaos reapportioned first legislative district.12 Sema further claimed that in
issuing Resolution No. 7902, the COMELEC usurped Congress power to create or reapportion
legislative districts.
In its Comment, the COMELEC, through the Office of the Solicitor General (OSG), chose not to
reach the merits of the case and merely contended that (1) Sema wrongly availed of the writ of
certiorari to nullify COMELEC Resolution No. 7902 because the COMELEC issued the same in the
exercise of its administrative, not quasi-judicial, power and (2) Semas prayer for the writ of
prohibition in G.R. No. 177597 became moot with the proclamation of respondent Didagen P.
Dilangalen (respondent Dilangalen) on 1 June 2007 as representative of the legislative district of
Shariff Kabunsuan Province with Cotabato City.
In his Comment, respondent Dilangalen countered that Sema is estopped from questioning
COMELEC Resolution No. 7902 because in her certificate of candidacy filed on 29 March 2007,
Sema indicated that she was seeking election as representative of "Shariff Kabunsuan including
Cotabato City." Respondent Dilangalen added that COMELEC Resolution No. 7902 is constitutional
because it did not apportion a legislative district for Shariff Kabunsuan or reapportion the legislative
districts in Maguindanao but merely renamed Maguindanaos first legislative district. Respondent
Dilangalen further claimed that the COMELEC could not reapportion Maguindanaos first legislative
district to make Cotabato City its sole component unit as the power to reapportion legislative districts
lies exclusively with Congress, not to mention that Cotabato City does not meet the minimum
population requirement under Section 5 (3), Article VI of the Constitution for the creation of a
legislative district within a city.13
Sema filed a Consolidated Reply controverting the matters raised in respondents Comments and
reiterating her claim that the COMELEC acted ultra vires in issuing Resolution No. 7902.
In the Resolution of 4 September 2007, the Court required the parties in G.R. No. 177597 to
comment on the issue of whether a province created by the ARMM Regional Assembly under
Section 19, Article VI of RA 9054 is entitled to one representative in the House of Representatives
without need of a national law creating a legislative district for such new province. The parties
submitted their compliance as follows:
(1) Sema answered the issue in the affirmative on the following grounds: (a) the Court
in Felwa v. Salas14stated that "when a province is created by statute, the corresponding
representative district comes into existence neither by authority of that statute which
cannot provide otherwise nor by apportionment, but by operation of the Constitution,
without a reapportionment"; (b) Section 462 of Republic Act No. 7160 (RA 7160) "affirms" the
apportionment of a legislative district incident to the creation of a province; and (c) Section 5
(3), Article VI of the Constitution and Section 3 of the Ordinance appended to the
Constitution mandate the apportionment of a legislative district in newly created provinces.
(2) The COMELEC, again represented by the OSG, apparently abandoned its earlier stance
on the propriety of issuing Resolution Nos. 07-0407 and 7902 and joined causes with Sema,
contending that Section 5 (3), Article VI of the Constitution is "self-executing." Thus, every

new province created by the ARMM Regional Assembly is ipso facto entitled to one
representative in the House of Representatives even in the absence of a national law; and
(3) Respondent Dilangalen answered the issue in the negative on the following grounds: (a)
the "province" contemplated in Section 5 (3), Article VI of the Constitution is one that is
created by an act of Congress taking into account the provisions in RA 7160 on the creation
of provinces; (b) Section 3, Article IV of RA 9054 withheld from the ARMM Regional
Assembly the power to enact measures relating to national elections, which encompasses
the apportionment of legislative districts for members of the House of Representatives; (c)
recognizing a legislative district in every province the ARMM Regional Assembly creates will
lead to the disproportionate representation of the ARMM in the House of Representatives as
the Regional Assembly can create provinces without regard to the requirements in Section
461 of RA 7160; and (d) Cotabato City, which has a population of less than 250,000, is not
entitled to a representative in the House of Representatives.
On 27 November 2007, the Court heard the parties in G.R. No. 177597 in oral arguments on the
following issues: (1) whether Section 19, Article VI of RA 9054, delegating to the ARMM Regional
Assembly the power to create provinces, is constitutional; and (2) if in the affirmative, whether a
province created under Section 19, Article VI of RA 9054 is entitled to one representative in the
House of Representatives without need of a national law creating a legislative district for such new
province.15
In compliance with the Resolution dated 27 November 2007, the parties in G.R. No. 177597 filed
their respective Memoranda on the issues raised in the oral arguments. 16 On the question of the
constitutionality of Section 19, Article VI of RA 9054, the parties in G.R. No. 177597 adopted the
following positions:
(1) Sema contended that Section 19, Article VI of RA 9054 is constitutional (a) as a valid
delegation by Congress to the ARMM of the power to create provinces under Section 20 (9),
Article X of the Constitution granting to the autonomous regions, through their organic acts,
legislative powers over "other matters as may be authorized by law for the promotion of the
general welfare of the people of the region" and (b) as an amendment to Section 6 of RA
7160.17 However, Sema concedes that, if taken literally, the grant in Section 19, Article VI of
RA 9054 to the ARMM Regional Assembly of the power to "prescribe standards lower than
those mandated" in RA 7160 in the creation of provinces contravenes Section 10, Article X of
the Constitution.18 Thus, Sema proposed that Section 19 "should be construed as prohibiting
the Regional Assembly from prescribing standards x x x that do not comply with the minimum
criteria" under RA 7160.19
(2) Respondent Dilangalen contended that Section 19, Article VI of RA 9054 is
unconstitutional on the following grounds: (a) the power to create provinces was not among
those granted to the autonomous regions under Section 20, Article X of the Constitution and
(b) the grant under Section 19, Article VI of RA 9054 to the ARMM Regional Assembly of the
power to prescribe standards lower than those mandated in Section 461 of RA 7160 on the
creation of provinces contravenes Section 10, Article X of the Constitution and the Equal
Protection Clause; and
(3) The COMELEC, through the OSG, joined causes with respondent Dilangalen (thus
effectively abandoning the position the COMELEC adopted in its Compliance with the
Resolution of 4 September 2007) and contended that Section 19, Article VI of RA 9054 is
unconstitutional because (a) it contravenes Section 10 and Section 6, 20 Article X of the

Constitution and (b) the power to create provinces was withheld from the autonomous
regions under Section 20, Article X of the Constitution.
On the question of whether a province created under Section 19, Article VI of RA 9054 is entitled to
one representative in the House of Representatives without need of a national law creating a
legislative district for such new province, Sema and respondent Dilangalen reiterated in their
Memoranda the positions they adopted in their Compliance with the Resolution of 4 September
2007. The COMELEC deemed it unnecessary to submit its position on this issue considering its
stance that Section 19, Article VI of RA 9054 is unconstitutional.
The pendency of the petition in G.R. No. 178628 was disclosed during the oral arguments on 27
November 2007. Thus, in the Resolution of 19 February 2008, the Court ordered G.R. No. 178628
consolidated with G.R. No. 177597. The petition in G.R. No. 178628 echoed Sema's contention that
the COMELEC acted ultra vires in issuing Resolution No. 7902 depriving the voters of Cotabato City
of a representative in the House of Representatives. In its Comment to the petition in G.R. No.
178628, the COMELEC, through the OSG, maintained the validity of COMELEC Resolution No.
7902 as a temporary measure pending the enactment by Congress of the "appropriate law."
The Issues
The petitions raise the following issues:
I. In G.R. No. 177597:
(A) Preliminarily
(1) whether the writs of Certiorari, Prohibition, and Mandamus are proper to test the
constitutionality of COMELEC Resolution No. 7902; and
(2) whether the proclamation of respondent Dilangalen as representative of Shariff
Kabunsuan Province with Cotabato City mooted the petition in G.R. No. 177597.
(B) On the merits
(1) whether Section 19, Article VI of RA 9054, delegating to the ARMM Regional
Assembly the power to create provinces, cities, municipalities and barangays, is
constitutional; and
(2) if in the affirmative, whether a province created by the ARMM Regional Assembly
under MMA Act 201 pursuant to Section 19, Article VI of RA 9054 is entitled to one
representative in the House of Representatives without need of a national law
creating a legislative district for such province.
II. In G.R No. 177597 and G.R No. 178628, whether COMELEC Resolution No. 7902 is valid
for maintaining the status quo in the first legislative district of Maguindanao (as "Shariff
Kabunsuan Province with Cotabato City [formerly First District of Maguindanao with
Cotabato City]"), despite the creation of the Province of Shariff Kabunsuan out of such
district (excluding Cotabato City).
The Ruling of the Court

The petitions have no merit. We rule that (1) Section 19, Article VI of RA 9054 is
unconstitutional insofar as it grants to the ARMM Regional Assembly the power to create
provinces and cities; (2) MMA Act 201 creating the Province of Shariff Kabunsuan is void;
and (3) COMELEC Resolution No. 7902 is valid.
On the Preliminary Matters
The Writ of Prohibition is Appropriate
to Test the Constitutionality of
Election Laws, Rules and Regulations
The purpose of the writ of Certiorari is to correct grave abuse of discretion by "any tribunal, board, or
officer exercising judicial or quasi-judicial functions."21 On the other hand, the writ of Mandamus will
issue to compel a tribunal, corporation, board, officer, or person to perform an act "which the law
specifically enjoins as a duty."22True, the COMELEC did not issue Resolution No. 7902 in the
exercise of its judicial or quasi-judicial functions.23Nor is there a law which specifically enjoins the
COMELEC to exclude from canvassing the votes cast in Cotabato City for representative of "Shariff
Kabunsuan Province with Cotabato City." These, however, do not justify the outright dismissal of the
petition in G.R. No. 177597 because Sema also prayed for the issuance of the writ of Prohibition and
we have long recognized this writ as proper for testing the constitutionality of election laws, rules,
and regulations.24
Respondent Dilangalens Proclamation
Does Not Moot the Petition
There is also no merit in the claim that respondent Dilangalens proclamation as winner in the 14
May 2007 elections for representative of "Shariff Kabunsuan Province with Cotabato City" mooted
this petition. This case does not concern respondent Dilangalens election. Rather, it involves an
inquiry into the validity of COMELEC Resolution No. 7902, as well as the constitutionality of MMA
Act 201 and Section 19, Article VI of RA 9054. Admittedly, the outcome of this petition, one way or
another, determines whether the votes cast in Cotabato City for representative of the district of
"Shariff Kabunsuan Province with Cotabato City" will be included in the canvassing of ballots.
However, this incidental consequence is no reason for us not to proceed with the resolution of the
novel issues raised here. The Courts ruling in these petitions affects not only the recently concluded
elections but also all the other succeeding elections for the office in question, as well as the power of
the ARMM Regional Assembly to create in the future additional provinces.
On the Main Issues
Whether the ARMM Regional Assembly
Can Create the Province of Shariff Kabunsuan
The creation of local government units is governed by Section 10, Article X of the Constitution, which
provides:
Sec. 10. No province, city, municipality, or barangay may be created, divided, merged, abolished or
its boundary substantially altered except in accordance with the criteria established in the local
government code and subject to approval by a majority of the votes cast in a plebiscite in the political
units directly affected.
Thus, the creation of any of the four local government units province, city, municipality or barangay
must comply with three conditions. First, the creation of a local government unit must follow the

criteria fixed in the Local Government Code. Second, such creation must not conflict with any
provision of the Constitution. Third, there must be a plebiscite in the political units affected.
There is neither an express prohibition nor an express grant of authority in the Constitution for
Congress to delegate to regional or local legislative bodies the power to create local government
units. However, under its plenary legislative powers, Congress can delegate to local legislative
bodies the power to create local government units, subject to reasonable standards and provided no
conflict arises with any provision of the Constitution. In fact, Congress has delegated to provincial
boards, and city and municipal councils, the power to create barangays within their
jurisdiction,25 subject to compliance with the criteria established in the Local Government Code, and
the plebiscite requirement in Section 10, Article X of the Constitution. However, under the Local
Government Code, "only x x x an Act of Congress" can create provinces, cities or municipalities. 26
1avvphi1

Under Section 19, Article VI of RA 9054, Congress delegated to the ARMM Regional Assembly the
power to create provinces, cities, municipalities and barangays within the ARMM. Congress made
the delegation under its plenary legislative powers because the power to create local government
units is not one of the express legislative powers granted by the Constitution to regional legislative
bodies.27 In the present case, the question arises whether the delegation to the ARMM Regional
Assembly of the power to create provinces, cities, municipalities and barangays conflicts with any
provision of the Constitution.
There is no provision in the Constitution that conflicts with the delegation to regional legislative
bodies of the power to create municipalities and barangays, provided Section 10, Article X of the
Constitution is followed. However, the creation of provinces and cities is another matter. Section 5
(3), Article VI of the Constitution provides, "Each city with a population of at least two hundred fifty
thousand, or each province, shall have at least one representative" in the House of Representatives.
Similarly, Section 3 of the Ordinance appended to the Constitution provides, "Any province that may
hereafter be created, or any city whose population may hereafter increase to more than two hundred
fifty thousand shall be entitled in the immediately following election to at least one Member x x x."
Clearly, a province cannot be created without a legislative district because it will violate Section 5 (3),
Article VI of the Constitution as well as Section 3 of the Ordinance appended to the Constitution. For
the same reason, a city with a population of 250,000 or more cannot also be created without a
legislative district. Thus, the power to create a province, or a city with a population of 250,000 or
more, requires also the power to create a legislative district. Even the creation of a city with a
population of less than 250,000 involves the power to create a legislative district because once the
citys population reaches 250,000, the city automatically becomes entitled to one representative
under Section 5 (3), Article VI of the Constitution and Section 3 of the Ordinance appended to the
Constitution. Thus, the power to create a province or city inherently involves the power to create a
legislative district.
For Congress to delegate validly the power to create a province or city, it must also validly delegate
at the same time the power to create a legislative district. The threshold issue then is, can Congress
validly delegate to the ARMM Regional Assembly the power to create legislative districts for the
House of Representatives? The answer is in the negative.
Legislative Districts are Created or Reapportioned
Only by an Act of Congress
Under the present Constitution, as well as in past28 Constitutions, the power to increase the
allowable membership in the House of Representatives, and to reapportion legislative districts, is
vested exclusively in Congress. Section 5, Article VI of the Constitution provides:

SECTION 5. (1) The House of Representatives shall be composed of not more than two hundred
and fifty members, unless otherwise fixed by law, who shall be elected from legislative districts
apportioned among the provinces, cities, and the Metropolitan Manila area in accordance with the
number of their respective inhabitants, and on the basis of a uniform and progressive ratio, and
those who, as provided by law, shall be elected through a party-list system of registered national,
regional, and sectoral parties or organizations.
xxxx
(3) Each legislative district shall comprise, as far as practicable, contiguous, compact, and
adjacent territory. Each city with a population of at least two hundred fifty thousand, or each
province, shall have at least one representative.
(4) Within three years following the return of every census, the Congress shall make a
reapportionment of legislative districts based on the standards provided in this section.
(Emphasis supplied)
Section 5 (1), Article VI of the Constitution vests in Congress the power to increase, through a law,
the allowable membership in the House of Representatives. Section 5 (4) empowers Congress to
reapportion legislative districts. The power to reapportion legislative districts necessarily includes the
power to create legislative districts out of existing ones. Congress exercises these powers through a
law that Congress itself enacts, and not through a law that regional or local legislative bodies enact.
The allowable membership of the House of Representatives can be increased, and new legislative
districts of Congress can be created, only through a national law passed by Congress. In Montejo v.
COMELEC,29 we held that the "power of redistricting x x x is traditionally regarded as part of the
power (of Congress) to make laws," and thus is vested exclusively in Congress.
This textual commitment to Congress of the exclusive power to create or reapportion legislative
districts is logical. Congress is a national legislature and any increase in its allowable membership or
in its incumbent membership through the creation of legislative districts must be embodied in a
national law. Only Congress can enact such a law. It would be anomalous for regional or local
legislative bodies to create or reapportion legislative districts for a national legislature like Congress.
An inferior legislative body, created by a superior legislative body, cannot change the membership of
the superior legislative body.
The creation of the ARMM, and the grant of legislative powers to its Regional Assembly under its
organic act, did not divest Congress of its exclusive authority to create legislative districts. This is
clear from the Constitution and the ARMM Organic Act, as amended. Thus, Section 20, Article X of
the Constitution provides:
SECTION 20. Within its territorial jurisdiction and subject to the provisions of this Constitution and
national laws, the organic act of autonomous regions shall provide for legislative powers over:
(1) Administrative organization;
(2) Creation of sources of revenues;
(3) Ancestral domain and natural resources;
(4) Personal, family, and property relations;

(5) Regional urban and rural planning development;


(6) Economic, social, and tourism development;
(7) Educational policies;
(8) Preservation and development of the cultural heritage; and
(9) Such other matters as may be authorized by law for the promotion of the general welfare
of the people of the region.
Nothing in Section 20, Article X of the Constitution authorizes autonomous regions,
expressly or impliedly, to create or reapportion legislative districts for Congress.
On the other hand, Section 3, Article IV of RA 9054 amending the ARMM Organic Act, provides,
"The Regional Assembly may exercise legislative power x x x except on the following
matters: x x x (k) National elections. x x x." Since the ARMM Regional Assembly has no legislative
power to enact laws relating to national elections, it cannot create a legislative district whose
representative is elected in national elections. Whenever Congress enacts a law creating a
legislative district, the first representative is always elected in the "next national elections" from the
effectivity of the law.30
Indeed, the office of a legislative district representative to Congress is a national office, and its
occupant, a Member of the House of Representatives, is a national official.31 It would be
incongruous for a regional legislative body like the ARMM Regional Assembly to create a national
office when its legislative powers extend only to its regional territory. The office of a district
representative is maintained by national funds and the salary of its occupant is paid out of national
funds. It is a self-evident inherent limitation on the legislative powers of every local or regional
legislative body that it can only create local or regional offices, respectively, and it can never create a
national office.
To allow the ARMM Regional Assembly to create a national office is to allow its legislative powers to
operate outside the ARMMs territorial jurisdiction. This violates Section 20, Article X of the
Constitution which expressly limits the coverage of the Regional Assemblys legislative powers
"[w]ithin its territorial jurisdiction x x x."
The ARMM Regional Assembly itself, in creating Shariff Kabunsuan, recognized the exclusive nature
of Congress power to create or reapportion legislative districts by abstaining from creating a
legislative district for Shariff Kabunsuan. Section 5 of MMA Act 201 provides that:
Except as may be provided by national law, the existing legislative district, which includes Cotabato
City as a part thereof, shall remain. (Emphasis supplied)
However, a province cannot legally be created without a legislative district because the Constitution
mandates that "each province shall have at least one representative." Thus, the creation of the
Province of Shariff Kabunsuan without a legislative district is unconstitutional.
Sema, petitioner in G.R. No. 177597, contends that Section 5 (3), Article VI of the Constitution,
which provides:

Each legislative district shall comprise, as far as practicable, contiguous, compact, and adjacent
territory. Each city with a population of at least two hundred fifty thousand, or each province, shall
have at least one representative. (Emphasis supplied)
and Section 3 of the Ordinance appended to the Constitution, which states:
Any province that may hereafter be created, or any city whose population may hereafter increase
to more than two hundred fifty thousand shall be entitled in the immediately following election to
at least one Member or such number of Members as it may be entitled to on the basis of the
number of its inhabitants and according to the standards set forth in paragraph (3), Section 5
of Article VI of the Constitution. The number of Members apportioned to the province out of which
such new province was created or where the city, whose population has so increased, is
geographically located shall be correspondingly adjusted by the Commission on Elections but such
adjustment shall not be made within one hundred and twenty days before the election. (Emphasis
supplied)
serve as bases for the conclusion that the Province of Shariff Kabunsuan, created on 29 October
2006, is automatically entitled to one member in the House of Representatives in the 14 May 2007
elections. As further support for her stance, petitioner invokes the statement in Felwa that "when a
province is created by statute, the corresponding representative district comes into existence neither
by authority of that statute which cannot provide otherwise nor by apportionment, but by
operation of the Constitution, without a reapportionment."
The contention has no merit.
First. The issue in Felwa, among others, was whether Republic Act No. 4695 (RA 4695), creating the
provinces of Benguet, Mountain Province, Ifugao, and Kalinga-Apayao and providing for
congressional representation in the old and new provinces, was unconstitutional for "creati[ng]
congressional districts without the apportionment provided in the Constitution." The Court answered
in the negative, thus:
The Constitution ordains:
"The House of Representatives shall be composed of not more than one hundred and twenty
Members who shall be apportioned among the several provinces as nearly as may be according to
the number of their respective inhabitants, but each province shall have at least one Member. The
Congress shall by law make an apportionment within three years after the return of every
enumeration, and not otherwise. Until such apportionment shall have been made, the House of
Representatives shall have the same number of Members as that fixed by law for the National
Assembly, who shall be elected by the qualified electors from the present Assembly districts. Each
representative district shall comprise as far as practicable, contiguous and compact territory."
Pursuant to this Section, a representative district may come into existence: (a) indirectly,
through the creation of a province for "each province shall have at least one member" in
the House of Representatives; or (b) by direct creation of several representative districts
within a province. The requirements concerning the apportionment of representative districts and
the territory thereof refer only to the second method of creation of representative districts, and do not
apply to those incidental to the creation of provinces, under the first method. This is deducible, not
only from the general tenor of the provision above quoted, but, also, from the fact that the
apportionment therein alluded to refers to that which is made by an Act of Congress. Indeed, when a
province is created by statute, the corresponding representative district, comes into existence

neither by authority of that statute which cannot provide otherwise nor by apportionment, but
by operation of the Constitution, without a reapportionment.
There is no constitutional limitation as to the time when, territory of, or other conditions under which
a province may be created, except, perhaps, if the consequence thereof were to exceed the
maximum of 120 representative districts prescribed in the Constitution, which is not the effect of the
legislation under consideration. As a matter of fact, provinces have been created or subdivided into
other provinces, with the consequent creation of additional representative districts, without
complying with the aforementioned requirements.32 (Emphasis supplied)
Thus, the Court sustained the constitutionality of RA 4695 because (1) it validly created legislative
districts "indirectly" through a special law enacted by Congress creating a province and (2) the
creation of the legislative districts will not result in breaching the maximum number of legislative
districts provided under the 1935 Constitution. Felwa does not apply to the present case because
in Felwa the new provinces were created by anational law enacted by Congress itself. Here, the
new province was created merely by a regional law enacted by the ARMM Regional Assembly.
What Felwa teaches is that the creation of a legislative district by Congress does not emanate alone
from Congress power to reapportion legislative districts, but also from Congress power to create
provinces which cannot be created without a legislative district. Thus, when a province is created, a
legislative district is created by operation of the Constitution because the Constitution provides that
"each province shall have at least one representative" in the House of Representatives. This does
not detract from the constitutional principle that the power to create legislative districts belongs
exclusively to Congress. It merely prevents any other legislative body, except Congress, from
creating provinces because for a legislative body to create a province such legislative body must
have the power to create legislative districts. In short, only an act of Congress can trigger the
creation of a legislative district by operation of the Constitution. Thus, only Congress has the power
to create, or trigger the creation of, a legislative district.
Moreover, if as Sema claims MMA Act 201 apportioned a legislative district to Shariff Kabunsuan
upon its creation, this will leave Cotabato City as the lone component of the first legislative district of
Maguindanao. However, Cotabato City cannot constitute a legislative district by itself because as of
the census taken in 2000, it had a population of only 163,849. To constitute Cotabato City alone as
the surviving first legislative district of Maguindanao will violate Section 5 (3), Article VI of the
Constitution which requires that "[E]ach city with a population of at least two hundred fifty thousand x
x x, shall have at least one representative."
Second. Semas theory also undermines the composition and independence of the House of
Representatives. Under Section 19,33 Article VI of RA 9054, the ARMM Regional Assembly can
create provinces and cities within the ARMM with or without regard to the criteria fixed in Section 461
of RA 7160, namely: minimum annual income of P20,000,000, and minimum contiguous territory of
2,000 square kilometers or minimum population of 250,000. 34 The following scenarios thus become
distinct possibilities:
(1) An inferior legislative body like the ARMM Regional Assembly can create 100 or more
provinces and thus increase the membership of a superior legislative body, the House of
Representatives, beyond the maximum limit of 250 fixed in the Constitution (unless a
national law provides otherwise);
(2) The proportional representation in the House of Representatives based on one
representative for at least every 250,000 residents will be negated because the ARMM

Regional Assembly need not comply with the requirement in Section 461(a)(ii) of RA 7160
that every province created must have a population of at least 250,000; and
(3) Representatives from the ARMM provinces can become the majority in the House of
Representatives through the ARMM Regional Assemblys continuous creation of provinces or
cities within the ARMM.
The following exchange during the oral arguments of the petition in G.R. No. 177597 highlights the
absurdity of Semas position that the ARMM Regional Assembly can create provinces:
Justice Carpio:
So, you mean to say [a] Local Government can create legislative district[s] and pack Congress with
their own representatives [?]
Atty. Vistan II:35
Yes, Your Honor, because the Constitution allows that.
Justice Carpio:
So, [the] Regional Assembly of [the] ARMM can create and create x x x provinces x x x and,
therefore, they can have thirty-five (35) new representatives in the House of Representatives without
Congress agreeing to it, is that what you are saying? That can be done, under your theory[?]
Atty. Vistan II:
Yes, Your Honor, under the correct factual circumstances.
Justice Carpio:
Under your theory, the ARMM legislature can create thirty-five (35) new provinces, there may be x x
x [only] one hundred thousand (100,000) [population], x x x, and they will each have one
representative x x x to Congress without any national law, is that what you are saying?
Atty. Vistan II:
Without law passed by Congress, yes, Your Honor, that is what we are saying.
xxxx
Justice Carpio:
So, they can also create one thousand (1000) new provinces, sen[d] one thousand (1000)
representatives to the House of Representatives without a national law[,] that is legally possible,
correct?
Atty. Vistan II:
Yes, Your Honor.36 (Emphasis supplied)

Neither the framers of the 1987 Constitution in adopting the provisions in Article X on regional
autonomy,37 nor Congress in enacting RA 9054, envisioned or intended these disastrous
consequences that certainly would wreck the tri-branch system of government under our
Constitution. Clearly, the power to create or reapportion legislative districts cannot be delegated by
Congress but must be exercised by Congress itself. Even the ARMM Regional Assembly recognizes
this.
The Constitution empowered Congress to create or reapportion legislative districts, not the regional
assemblies. Section 3 of the Ordinance to the Constitution which states, "[A]ny province that may
hereafter be created x x x shall be entitled in the immediately following election to at least one
Member," refers to a province created by Congress itself through a national law. The reason is that
the creation of a province increases the actual membership of the House of Representatives, an
increase that only Congress can decide. Incidentally, in the present 14th Congress, there are
21938 district representatives out of the maximum 250 seats in the House of Representatives. Since
party-list members shall constitute 20 percent of total membership of the House, there should at
least be 50 party-list seats available in every election in case 50 party-list candidates are proclaimed
winners. This leaves only 200 seats for district representatives, much less than the 219 incumbent
district representatives. Thus, there is a need now for Congress to increase by law the allowable
membership of the House, even before Congress can create new provinces.
It is axiomatic that organic acts of autonomous regions cannot prevail over the Constitution. Section
20, Article X of the Constitution expressly provides that the legislative powers of regional assemblies
are limited "[w]ithin its territorial jurisdiction and subject to the provisions of the Constitution
and national laws, x x x." The Preamble of the ARMM Organic Act (RA 9054) itself states that the
ARMM Government is established "within the framework of the Constitution." This follows Section
15, Article X of the Constitution which mandates that the ARMM "shall be created x x x within the
framework of this Constitution and the national sovereignty as well as territorial integrity of
the Republic of the Philippines."
The present case involves the creation of a local government unit that necessarily involves also the
creation of a legislative district. The Court will not pass upon the constitutionality of the creation of
municipalities and barangays that does not comply with the criteria established in Section 461 of RA
7160, as mandated in Section 10, Article X of the Constitution, because the creation of such
municipalities and barangays does not involve the creation of legislative districts. We leave the
resolution of this issue to an appropriate case.
In summary, we rule that Section 19, Article VI of RA 9054, insofar as it grants to the ARMM
Regional Assembly the power to create provinces and cities, is void for being contrary to Section 5 of
Article VI and Section 20 of Article X of the Constitution, as well as Section 3 of the Ordinance
appended to the Constitution. Only Congress can create provinces and cities because the creation
of provinces and cities necessarily includes the creation of legislative districts, a power only
Congress can exercise under Section 5, Article VI of the Constitution and Section 3 of the Ordinance
appended to the Constitution. The ARMM Regional Assembly cannot create a province without a
legislative district because the Constitution mandates that every province shall have a legislative
district. Moreover, the ARMM Regional Assembly cannot enact a law creating a national office like
the office of a district representative of Congress because the legislative powers of the ARMM
Regional Assembly operate only within its territorial jurisdiction as provided in Section 20, Article X of
the Constitution. Thus, we rule that MMA Act 201, enacted by the ARMM Regional Assembly and
creating the Province of Shariff Kabunsuan, is void.
Resolution No. 7902 Complies with the Constitution

Consequently, we hold that COMELEC Resolution No. 7902, preserving the geographic and
legislative district of the First District of Maguindanao with Cotabato City, is valid as it merely
complies with Section 5 of Article VI and Section 20 of Article X of the Constitution, as well as
Section 1 of the Ordinance appended to the Constitution.
WHEREFORE, we declare Section 19, Article VI of Republic Act No. 9054 UNCONSTITUTIONAL
insofar as it grants to the Regional Assembly of the Autonomous Region in Muslim Mindanao the
power to create provinces and cities. Thus, we declare VOID Muslim Mindanao Autonomy Act No.
201 creating the Province of Shariff Kabunsuan. Consequently, we rule that COMELEC Resolution
No. 7902 is VALID.
Let a copy of this ruling be served on the President of the Senate and the Speaker of the House of
Representatives.
SO ORDERED.

Republic of the Philippines


SUPREME COURT
Manila
EN BANC
G.R. No. 176970

December 8, 2008

ROGELIO Z. BAGABUYO, petitioner,


vs.
COMMISSION ON ELECTIONS, respondent.
DECISION
BRION, J.:
Before us is the petition for certiorari, prohibition, and mandamus,1 with a prayer for the issuance of a
temporary restraining order and a writ of preliminary injunction, filed by Rogelio Bagabuyo
(petitioner) to prevent the Commission on Elections (COMELEC) from implementing Resolution No.
7837 on the ground that Republic Act No. 93712 - the law that Resolution No. 7837 implements - is
unconstitutional.
BACKGROUND FACTS
On October 10, 2006, Cagayan de Oro's then Congressman Constantino G. Jaraula filed and
sponsored House Bill No. 5859: "An Act Providing for the Apportionment of the Lone Legislative
District of the City of Cagayan De Oro."3 This law eventually became Republic Act (R.A.) No.
9371.4 It increased Cagayan de Oro's legislative district from one to two. For the election of May
2007, Cagayan de Oro's voters would be classified as belonging to either the first or the second

district, depending on their place of residence. The constituents of each district would elect their own
representative to Congress as well as eight members of the Sangguniang Panglungsod.
Section 1 of R.A. No. 9371 apportioned the City's barangays as follows:
Legislative Districts - The lone legislative district of the City of Cagayan De Oro is hereby
apportioned to commence in the next national elections after the effectivity of this Act.
Henceforth, barangays Bonbon, Bayabas, Kauswagan, Carmen, Patag, Bulua, Iponan,
Baikingon, San Simon, Pagatpat, Canitoan, Balulang, Lumbia, Pagalungan, Tagpangi,
Taglimao, Tuburan, Pigsag-an, Tumpagon, Bayanga, Mambuaya, Dansulihon, Tignapoloan
and Bisigan shall comprise the first district while barangays Macabalan, Puntod,
Consolacion, Camaman-an, Nazareth, Macasandig, Indahag, Lapasan, Gusa, Cugman, FS
Catanico, Tablon, Agusan, Puerto, Bugo, and Balubal and all urban barangays from
Barangay 1 to Barangay 40 shall comprise the second district.5
On March 13, 2007, the COMELEC en Banc promulgated Resolution No. 78376 implementing R.A.
No. 9371.
Petitioner Rogelio Bagabuyo filed the present petition against the COMELEC on March 27,
2007.7 On 10 April 2008, the petitioner amended the petition to include the following as respondents:
Executive Secretary Eduardo Ermita; the Secretary of the Department of Budget and Management;
the Chairman of the Commission on Audit; the Mayor and the members of the Sangguniang
Panglungsod of Cagayan de Oro City; and its Board of Canvassers. 8
In asking for the nullification of R.A. No. 9371 and Resolution No. 7837 on constitutional grounds,
the petitioner argued that the COMELEC cannot implement R.A. No. 9371 without providing for the
rules, regulations and guidelines for the conduct of a plebiscite which is indispensable for the
division or conversion of a local government unit. He prayed for the issuance of an order directing
the respondents to cease and desist from implementing R.A. No. 9371 and COMELEC Resolution
No. 7837, and to revert instead to COMELEC Resolution No. 7801 which provided for a single
legislative district for Cagayan de Oro.
Since the Court did not grant the petitioner's prayer for a temporary restraining order or writ of
preliminary injunction, the May 14 National and Local Elections proceeded according to R.A. No.
9371 and Resolution No. 7837.
The respondent's Comment on the petition, filed through the Office of the Solicitor General, argued
that: 1) the petitioner did not respect the hierarchy of courts, as the Regional Trial Court (RTC) is
vested with concurrent jurisdiction over cases assailing the constitutionality of a statute; 2) R.A. No.
9371 merely increased the representation of Cagayan de Oro City in the House of Representatives
and Sangguniang Panglungsod pursuant to Section 5, Article VI of the 1987 Constitution; 3) the
criteria established under Section 10, Article X of the 1987 Constitution only apply when there is a
creation, division, merger, abolition or substantial alteration of boundaries of a province, city,
municipality, or barangay; in this case, no such creation, division, merger, abolition or alteration of
boundaries of a local government unit took place; and 4) R.A. No. 9371 did not bring about any

change in Cagayan de Oro's territory, population and income classification; hence, no plebiscite is
required.
The petitioner argued in his reply that: 1) pursuant to the Court's ruling in Del Mar v. PAGCOR,9 the
Court may take cognizance of this petition if compelling reasons, or the nature and importance of the
issues raised, warrant the immediate exercise of its jurisdiction; 2) Cagayan de Oro City's
reapportionment under R.A. No. 9371 falls within the meaning of creation, division, merger, abolition
or substantial alteration of boundaries of cities under Section 10, Article X of the Constitution; 3) the
creation, division, merger, abolition or substantial alteration of boundaries of local government units
involve a common denominator - the material change in the political and economic rights of the local
government units directly affected, as well as of the people therein; 4) a voter's sovereign power to
decide on who should be elected as the entire city's Congressman was arbitrarily reduced by at least
one half because the questioned law and resolution only allowed him to vote and be voted for in the
district designated by the COMELEC; 5) a voter was also arbitrarily denied his right to elect the
Congressman and the members of the city council for the other legislative district, and 6)
government funds were illegally disbursed without prior approval by the sovereign electorate of
Cagayan De Oro City.10
THE ISSUES
The core issues, based on the petition and the parties' memoranda, can be limited to the following
contentious points:
1) Did the petitioner violate the hierarchy of courts rule; if so, should the instant petition be
dismissed on this ground?
2) Does R.A. No. 9371 merely provide for the legislative reapportionment of Cagayan de Oro
City, or does it involve the division and conversion of a local government unit?
3) Does R.A. No. 9371 violate the equality of representation doctrine?
OUR RULING
Except for the issue of the hierarchy of courts rule, we find the petition totally without merit.
The hierarchy of courts principle.
The Supreme Court has original jurisdiction over petitions for certiorari, prohibition, mandamus, quo
warranto, and habeas corpus.11 It was pursuant to this original jurisdiction that the petitioner filed the
present petition.
While this jurisdiction is shared with the Court of Appeals12 and the RTCs,13 a direct invocation of the
Supreme Court's jurisdiction is allowed only when there are special and important reasons therefor,
clearly and especially set out in the petition. Reasons of practicality, dictated by an increasingly
overcrowded docket and the need to prioritize in favor of matters within our exclusive jurisdiction,
justify the existence of this rule otherwise known as the "principle of hierarchy of courts." More

generally stated, the principle requires that recourse must first be made to the lower-ranked court
exercising concurrent jurisdiction with a higher court.14
Among the cases we have considered sufficiently special and important to be exceptions to the rule,
are petitions for certiorari, prohibition, mandamus and quo warranto against our nation's lawmakers
when the validity of their enactments is assailed. 15 The present petition is of this nature; its subject
matter and the nature of the issues raised - among them, whether legislative reapportionment
involves a division of Cagayan de Oro City as a local government unit - are reasons enough for
considering it an exception to the principle of hierarchy of courts. Additionally, the petition assails as
well a resolution of the COMELEC en banc issued to implement the legislative apportionment that
R.A. No. 9371 decrees. As an action against a COMELEC en banc resolution, the case falls under
Rule 64 of the Rules of Court that in turn requires a review by this Court via a Rule 65 petition
forcertiorari.16 For these reasons, we do not see the principle of hierarchy of courts to be a stumbling
block in our consideration of the present case.
The Plebiscite Requirement.
The petitioner insists that R.A. No. 9371 converts and divides the City of Cagayan de Oro as a local
government unit, and does not merely provide for the City's legislative apportionment. This argument
essentially proceeds from a misunderstanding of the constitutional concepts of apportionment of
legislative districts and division of local government units.
Legislative apportionment is defined by Black's Law Dictionary as the determination of the number
of representatives which a State, county or other subdivision may send to a legislative body.17It is the
allocation of seats in a legislative body in proportion to the population; the drawing of voting district
lines so as to equalize population and voting power among the districts. 18 Reapportionment, on the
other hand, is the realignment or change in legislative districts brought about by changes in
population and mandated by the constitutional requirement of equality of representation.19
Article VI (entitled Legislative Department) of the 1987 Constitution lays down the rules on legislative
apportionment under its Section 5 which provides:
Sec. 5(1). (1) The House of Representatives shall be composed of not more than two
hundred fifty members unless otherwise fixed by law, who shall be elected from legislative
districts apportioned among the provinces, cities, and the Metropolitan Manila area in
accordance with the number of their respective inhabitants, and on the basis of a uniform
and progressive ratio, and those who, as provided by law, shall be elected through a partylist system of registered national, regional and sectoral parties or organizations.
xxx
(3) Each legislative district shall comprise, as far as practicable, continuous, compact, and
adjacent territory. Each city with a population of at least two hundred fifty thousand, or each
province, shall have at least one representative.

(4) Within three years following the return of every census, the Congress shall make a
reapportionment of legislative districts based on the standards provided in this section.
Separately from the legislative districts that legal apportionment or reapportionment speaks of, are
the local government units (historically and generically referred to as "municipal corporations") that
the Constitution itself classified into provinces, cities, municipalities and barangays.20 In its strict and
proper sense, a municipality has been defined as "a body politic and corporate constituted by the
incorporation of the inhabitants of a city or town for the purpose of local government thereof." 21 The
creation, division, merger, abolition or alteration of boundary of local government units, i.e., of
provinces, cities, municipalities, and barangays, are covered by the Article on Local Government
(Article X). Section 10 of this Article provides:
No province, city, municipality, or barangay may be created, divided, merged, abolished, or
its boundary substantially altered, except in accordance with the criteria established in the
local government code and subject to approval by a majority of the votes cast in a plebiscite
in the political unit directly affected.
Under both Article VI, Section 5, and Article X, Section 10 of the Constitution, the authority to act has
been vested in the Legislature. The Legislature undertakes the apportionment and reapportionment
of legislative districts,22and likewise acts on local government units by setting the standards for their
creation, division, merger, abolition and alteration of boundaries and by actually creating, dividing,
merging, abolishing local government units and altering their boundaries through legislation. Other
than this, not much commonality exists between the two provisions since they are inherently different
although they interface and relate with one another.
The concern that leaps from the text of Article VI, Section 5 is political representation and the means
to make a legislative district sufficiently represented so that the people can be effectively heard. As
above stated, the aim of legislative apportionment is "to equalize population and voting power
among districts."23 Hence, emphasis is given to the number of people represented; the uniform and
progressive ratio to be observed among the representative districts; and accessibility and
commonality of interests in terms of each district being, as far as practicable, continuous, compact
and adjacent territory. In terms of the people represented, every city with at least 250,000 people
and every province (irrespective of population) is entitled to one representative. In this sense,
legislative districts, on the one hand, and provinces and cities, on the other, relate and interface with
each other. To ensure continued adherence to the required standards of apportionment, Section 5(4)
specifically mandates reapportionment as soon as the given standards are met.
In contrast with the equal representation objective of Article VI, Section 5, Article X, Section 10
expressly speaks of how local government units may be "created, divided, merged, abolished, or its
boundary substantially altered." Its concern is the commencement, the termination, and the
modification of local government units' corporate existence and territorial coverage; and it speaks of
two specific standards that must be observed in implementing this concern, namely, the criteria
established in the local government code and the approval by a majority of the votes cast in a
plebiscite in the political units directly affected. Under the Local Government Code (R.A. No. 7160)
passed in 1991, the criteria of income, population and land area are specified as verifiable indicators
of viability and capacity to provide services.24 The division or merger of existing units must comply

with the same requirements (since a new local government unit will come into being), provided that a
division shall not reduce the income, population, or land area of the unit affected to less than the
minimum requirement prescribed in the Code.25
A pronounced distinction between Article VI, Section 5 and, Article X, Section 10 is on the
requirement of a plebiscite. The Constitution and the Local Government Code expressly require a
plebiscite to carry out any creation, division, merger, abolition or alteration of boundary of a local
government unit.26 In contrast, no plebiscite requirement exists under the apportionment or
reapportionment provision. In Tobias v. Abalos,27 a case that arose from the division of the
congressional district formerly covering San Juan and Mandaluyong into separate districts, we
confirmed this distinction and the fact that no plebiscite is needed in a legislative reapportionment.
The plebiscite issue came up because one was ordered and held for Mandaluyong in the course of
its conversion into a highly urbanized city, while none was held for San Juan. In explaining why this
happened, the Court ruled that no plebiscite was necessary for San Juan because the objective of
the plebiscite was the conversion of Mandaluyong into a highly urbanized city as required by Article
X, Section 10 the Local Government Code; the creation of a new legislative district only followed as
a consequence. In other words, the apportionment alone and by itself did not call for a plebiscite, so
that none was needed for San Juan where only a reapportionment took place.
The need for a plebiscite under Article X, Section 10 and the lack of requirement for one under
Article VI, Section 5 can best be appreciated by a consideration of the historical roots of these two
provisions, the nature of the concepts they embody as heretofore discussed, and their areas of
application.
A Bit of History.
In Macias v. COMELEC,28 we first jurisprudentially acknowledged the American roots of our
apportionment provision, noting its roots from the Fourteenth Amendment29 of the U.S. Constitution
and from the constitutions of some American states. The Philippine Organic Act of 1902 created the
Philippine Assembly,30 the body that acted as the lower house of the bicameral legislature under the
Americans, with the Philippine Commission acting as the upper house. While the members of the
Philippine Commission were appointed by the U.S. President with the conformity of the U.S. Senate,
the members of the Philippine Assembly were elected by representative districts previously
delineated under the Philippine Organic Act of 1902 pursuant to the mandate to apportion the seats
of the Philippine Assembly among the provinces as nearly as practicable according to population.
Thus, legislative apportionment first started in our country.
The Jones Law or the Philippine Autonomy Act of 1916 maintained the apportionment provision,
dividing the country into 12 senate districts and 90 representative districts electing one delegate
each to the House of Representatives. Section 16 of the Act specifically vested the Philippine
Legislature with the authority to redistrict the Philippine Islands.
Under the 1935 Constitution, Article VI, Section 5 retained the concept of legislative apportionment
together with "district" as the basic unit of apportionment; the concern was "equality of
representation . . . as an essential feature of republican institutions" as expressed in the leading
case of Macias v. COMELEC.31 The case ruled that inequality of representation is a justiciable, not a

political issue, which ruling was reiterated in Montejo v. COMELEC.32 Notably, no issue regarding the
holding of a plebiscite ever came up in these cases and the others that followed, as no plebiscite
was required.
Article VIII, Section 2 of the 1973 Constitution retained the concept of equal representation "in
accordance with the number of their respective inhabitants and on the basis of a uniform and
progressive ratio" with each district being, as far as practicable, contiguous, compact and adjacent
territory. This formulation was essentially carried over to the 1987 Constitution, distinguished only
from the previous one by the presence of party-list representatives. In neither Constitution was a
plebiscite required.
The need for a plebiscite in the creation, division, merger, or abolition of local government units was
not constitutionally enshrined until the 1973 Constitution. However, as early as 1959, R.A. No.
226433 required, in the creation of barrios by Provincial Boards, that the creation and definition of
boundaries be "upon petition of a majority of the voters in the areas affected." In 1961, the Charter of
the City of Caloocan (R.A. No. 3278) carried this further by requiring that the "Act shall take effect
after a majority of voters of the Municipality of Caloocan vote in favor of the conversion of their
municipality into a city in a plebiscite." This was followed up to 1972 by other legislative enactments
requiring a plebiscite as a condition for the creation and conversion of local government units as well
as the transfer of sitios from one legislative unit to another.34 In 1973, the plebiscite requirement was
accorded constitutional status.
Under these separate historical tracks, it can be seen that the holding of a plebiscite was never a
requirement in legislative apportionment or reapportionment. After it became constitutionally
entrenched, a plebiscite was also always identified with the creation, division, merger, abolition and
alteration of boundaries of local government units, never with the concept of legislative
apportionment.
Nature and Areas of Application.
The legislative district that Article VI, Section 5 speaks of may, in a sense, be called a political unit
because it is the basis for the election of a member of the House of Representatives and members
of the local legislative body. It is not, however, a political subdivision through which functions of
government are carried out. It can more appropriately be described as a representative unit that may
or may not encompass the whole of a city or a province, but unlike the latter, it is not a corporate
unit. Not being a corporate unit, a district does not act for and in behalf of the people comprising the
district; it merely delineates the areas occupied by the people who will choose a representative in
their national affairs. Unlike a province, which has a governor; a city or a municipality, which has a
mayor; and a barangay, which has a punong barangay, a district does not have its own chief
executive. The role of the congressman that it elects is to ensure that the voice of the people of the
district is heard in Congress, not to oversee the affairs of the legislative district. Not being a
corporate unit also signifies that it has no legal personality that must be created or dissolved and has
no capacity to act. Hence, there is no need for any plebiscite in the creation, dissolution or any other
similar action on a legislative district.

The local government units, on the other hand, are political and corporate units. They are the
territorial and political subdivisions of the state.35 They possess legal personality on the authority of
the Constitution and by action of the Legislature. The Constitution defines them as entities that
Congress can, by law, create, divide, abolish, merge; or whose boundaries can be altered based on
standards again established by both the Constitution and the Legislature. 36 A local government unit's
corporate existence begins upon the election and qualification of its chief executive and a majority of
the members of its Sanggunian.37
As a political subdivision, a local government unit is an "instrumentality of the state in carrying out
the functions of government."38 As a corporate entity with a distinct and separate juridical personality
from the State, it exercises special functions for the sole benefit of its constituents. It acts as "an
agency of the community in the administration of local affairs" 39 and the mediums through which the
people act in their corporate capacity on local concerns.40 In light of these roles, the Constitution saw
it fit to expressly secure the consent of the people affected by the creation, division, merger, abolition
or alteration of boundaries of local government units through a plebiscite.
These considerations clearly show the distinctions between a legislative apportionment or
reapportionment and the division of a local government unit. Historically and by its intrinsic nature, a
legislative apportionment does not mean, and does not even imply, a division of a local government
unit where the apportionment takes place. Thus, the plebiscite requirement that applies to the
division of a province, city, municipality or barangay under the Local Government Code should not
apply to and be a requisite for the validity of a legislative apportionment or reapportionment.
R.A. No. 9371 and COMELEC Res. No. 7837
R.A. No. 9371 is, on its face, purely and simply a reapportionment legislation passed in accordance
with the authority granted to Congress under Article VI, Section 5(4) of the Constitution. Its core
provision - Section 1 - provides:
SECTION 1. Legislative Districts. - The lone legislative district of the City of Cagayan de Oro
is hereby apportioned to commence in the next national elections after the effectivity of this
Act. Henceforth, barangays Bonbon, Bayabas, Kauswagan, Carmen, Patag, Bulua, Iponan,
Baikingon, San Simon, Pagatpat, Canitoan, Balulang, Lumbia, Pagalungan, Tagpangi,
Taglimao, Tuburan, Pigsag-an, Tumpagon, Bayanga, Mambuaya, Dansulihon, Tignapoloan
and Bisigan shall comprise the first district while barangays Macabalan, Puntod,
Consolacion, Camaman-an, Nazareth, Macansandig, Indahag, Lapasan, Gusa, Cugman, FS
Catanico, Tablon, Agusan, Puerto, Bugo and Balubal and all urban barangays from Barangay
1 to Barangay 40 shall comprise the second district.
Under these wordings, no division of Cagayan de Oro City as a political and corporate entity takes
place or is mandated. Cagayan de Oro City politically remains a single unit and its administration is
not divided along territorial lines. Its territory remains completely whole and intact; there is only the
addition of another legislative district and the delineation of the city into two districts for purposes of
representation in the House of Representatives. Thus, Article X, Section 10 of the Constitution does
not come into play and no plebiscite is necessary to validly apportion Cagayan de Oro City into two
districts.

Admittedly, the legislative reapportionment carries effects beyond the creation of another
congressional district in the city by providing, as reflected in COMELEC Resolution No. 7837, for
additional Sangguniang Panglunsod seats to be voted for along the lines of the congressional
apportionment made. The effect on the Sangguniang Panglunsod, however, is not directly traceable
to R.A. No. 9371 but to another law - R.A. No. 663641 - whose Section 3 provides:
SECTION 3. Other Cities. - The provision of any law to the contrary notwithstanding the City
of Cebu, City of Davao, and any other city with more than one representative district shall
have eight (8) councilors for each district who shall be residents thereof to be elected by the
qualified voters therein, provided that the cities of Cagayan de Oro, Zamboanga, Bacolod,
Iloilo and other cities comprising a representative district shall have twelve (12) councilors
each and all other cities shall have ten (10) councilors each to be elected at large by the
qualified voters of the said cities: Provided, That in no case shall the present number of
councilors according to their charters be reduced.
However, neither does this law have the effect of dividing the City of Cagayan de Oro into two
political and corporate units and territories. Rather than divide the city either territorially or as a
corporate entity, the effect is merely to enhance voter representation by giving each city voter more
and greater say, both in Congress and in the Sangguniang Panglunsod.
To illustrate this effect, before the reapportionment, Cagayan de Oro had only one congressman and
12 city council members citywide for its population of approximately 500,000. 42 By having two
legislative districts, each of them with one congressman, Cagayan de Oro now effectively has two
congressmen, each one representing 250,000 of the city's population. In terms of services for city
residents, this easily means better access to their congressman since each one now services only
250,000 constituents as against the 500,000 he used to represent. The same goes true for
the Sangguniang Panglungsod with its ranks increased from 12 to 16 since each legislative district
now has 8 councilors. In representation terms, the fewer constituents represented translate to a
greater voice for each individual city resident in Congress and in the Sanggunian; each
congressman and each councilor represents both a smaller area and fewer constituents whose
fewer numbers are now concentrated in each representative. The City, for its part, now has twice the
number of congressmen speaking for it and voting in the halls of Congress. Since the total number
of congressmen in the country has not increased to the point of doubling its numbers, the presence
of two congressman (instead of one) from the same city cannot but be a quantitative and
proportional improvement in the representation of Cagayan de Oro City in Congress.
Equality of representation.
The petitioner argues that the distribution of the legislative districts is unequal. District 1 has only
93,719 registered voters while District 2 has 127,071. District 1 is composed mostly of
rural barangays while District 2 is composed mostly of urban barangays.43 Thus, R.A. No. 9371
violates the principle of equality of representation.
A clarification must be made. The law clearly provides that the basis for districting shall be
the number of the inhabitants of a city or a province, not the number of registered voters therein. We
settled this very same question in Herrera v. COMELEC44 when we interpreted a provision in R.A.

No. 7166 and COMELEC Resolution No. 2313 that applied to the Province of Guimaras. We
categorically ruled that the basis for districting is the number of inhabitants of the Province of
Guimaras by municipality based on the official 1995 Census of Population as certified to by Tomas P.
Africa, Administrator of the National Statistics Office.
The petitioner, unfortunately, did not provide information about the actual population of Cagayan de
Oro City. However, we take judicial notice of the August 2007 census of the National Statistics Office
which shows thatbarangays comprising Cagayan de Oro's first district have a total population of
254,644, while the second district has 299,322 residents. Undeniably, these figures show a disparity
in the population sizes of the districts.45 The Constitution, however, does not require mathematical
exactitude or rigid equality as a standard in gauging equality of representation. 46 In fact, for cities, all
it asks is that "each city with a population of at least two hundred fifty thousand shall have one
representative," while ensuring representation for every province regardless of the size of its
population. To ensure quality representation through commonality of interests and ease of access by
the representative to the constituents, all that the Constitution requires is that every legislative district
should comprise, as far as practicable, contiguous, compact, and adjacent territory. Thus, the
Constitution leaves the local government units as they are found and does not require their division,
merger or transfer to satisfy the numerical standard it imposes. Its requirements are satisfied despite
some numerical disparity if the units are contiguous, compact and adjacent as far as practicable.
The petitioner's contention that there is a resulting inequality in the division of Cagayan de Oro City
into two districts because the barangays in the first district are mostly rural barangays while the
second district is mostly urban, is largely unsubstantiated. But even if backed up by proper proof, we
cannot question the division on the basis of the difference in the barangays' levels of development or
developmental focus as these are not part of the constitutional standards for legislative
apportionment or reapportionment. What the components of the two districts of Cagayan de Oro
would be is a matter for the lawmakers to determine as a matter of policy. In the absence of any
grave abuse of discretion or violation of the established legal parameters, this Court cannot intrude
into the wisdom of these policies.47
WHEREFORE, we hereby DISMISS the petition for lack of merit. Costs against the petitioner.
SO ORDERED.
Republic of the Philippines
SUPREME COURT
Manila
EN BANC
G.R. No. 189793

April 7, 2010

SENATOR BENIGNO SIMEON C. AQUINO III and MAYOR JESSE ROBREDO, Petitioners,
vs.
COMMISSION ON ELECTIONS represented by its Chairman JOSE A.R. MELO and its
Commissioners, RENE V. SARMIENTO, NICODEMO T. FERRER, LUCENITO N. TAGLE,
ARMANDO VELASCO, ELIAS R. YUSOPH AND GREGORIO LARRAZABAL, Respondents.

DECISION
PEREZ, J.:
This case comes before this Court by way of a Petition for Certiorari and Prohibition under Rule 65
of the Rules of Court. In this original action, petitioners Senator Benigno Simeon C. Aquino III and
Mayor Jesse Robredo, as public officers, taxpayers and citizens, seek the nullification as
unconstitutional of Republic Act No. 9716, entitled "An Act Reapportioning the Composition of the
First (1st) and Second (2nd) Legislative Districts in the Province of Camarines Sur and Thereby
Creating a New Legislative District From Such Reapportionment." Petitioners consequently pray that
the respondent Commission on Elections be restrained from making any issuances and from taking
any steps relative to the implementation of Republic Act No. 9716.
Republic Act No. 9716 originated from House Bill No. 4264, and was signed into law by President
Gloria Macapagal Arroyo on 12 October 2009. It took effect on 31 October 2009, or fifteen (15) days
following its publication in the Manila Standard, a newspaper of general circulation. 1 In substance,
the said law created an additional legislative district for the Province of Camarines Sur by
reconfiguring the existing first and second legislative districts of the province.
Prior to Republic Act No. 9716, the Province of Camarines Sur was estimated to have a population
of 1,693,821,2distributed among four (4) legislative districts in this wise:
District
1st District

2nd District

3rd District

4th District

Municipalities/Cities
Del Gallego
Ragay
Lupi
Sipocot
Cabusao

Libmanan
Minalabac
Pamplona
Pasacao
San Fernando

Gainza
Milaor
Naga
Pili
Ocampo

Canaman
Camaligan
Magarao
Bombon
Calabanga

Caramoan
Garchitorena
Goa
Lagonoy
Presentacion

Sangay
San Jose
Tigaon
Tinamba
Siruma

Iriga
Baao
Balatan
Bato

Buhi
Bula
Nabua

Population
417,304

474,899

372,548

429,070

Following the enactment of Republic Act No. 9716, the first and second districts of Camarines Sur
were reconfigured in order to create an additional legislative district for the province. Hence, the first
district municipalities of Libmanan, Minalabac, Pamplona, Pasacao, and San Fernando were
combined with the second district municipalities of Milaor and Gainza to form a new second
legislative district. The following table3 illustrates the reapportionment made by Republic Act No.
9716:

District

Municipalities/Cities

1st District

Del Gallego
Ragay
Lupi
Sipocot
Cabusao

2nd District

Population
176,383

Libmanan
Minalabac
Pamplona
Pasacao

San Fernando
Gainza
Milaor

3rd District (formerly 2nd District)

Naga
Pili
Ocampo
Canaman

Camaligan
Magarao
Bombon
Calabanga

439,043

4th District (formerly 3rd District)

Caramoan
Garchitorena
Goa
Lagonoy
Presentacion

Sangay
San Jose
Tigaon
Tinamba
Siruma

372,548

5th District (formerly 4th District)

Iriga
Baao
Balatan
Bato

Buhi
Bula
Nabua

429,070

276,777

Republic Act No. 9716 is a well-milled legislation. The factual recitals by both parties of the origins of
the bill that became the law show that, from the filing of House Bill No. 4264 until its approval by the
Senate on a vote of thirteen (13) in favor and two (2) against, the process progressed step by step,
marked by public hearings on the sentiments and position of the local officials of Camarines Sur on
the creation of a new congressional district, as well as argumentation and debate on the issue, now
before us, concerning the stand of the oppositors of the bill that a population of at least 250,000 is
required by the Constitution for such new district.4
Petitioner Aquino III was one of two senators who voted against the approval of the Bill by the
Senate. His co-petitioner, Robredo, is the Mayor of Naga City, which was a part of the former second
district from which the municipalities of Gainza and Milaor were taken for inclusion in the new
second district. No other local executive joined the two; neither did the representatives of the former
third and fourth districts of the province.
Petitioners contend that the reapportionment introduced by Republic Act No. 9716, runs afoul of the
explicit constitutional standard that requires a minimum population of two hundred fifty thousand
(250,000) for the creation of a legislative district.5 The petitioners claim that the reconfiguration by
Republic Act No. 9716 of the first and second districts of Camarines Sur is unconstitutional, because
the proposed first district will end up with a population of less than 250,000 or only 176,383.
Petitioners rely on Section 5(3), Article VI of the 1987 Constitution as basis for the cited 250,000
minimum population standard.6 The provision reads:
Article VI

Section 5. (1) x x x x
(2) x x x x
(3) Each legislative district shall comprise, as far as practicable, contiguous, compact, and
adjacent territory. Each city with a population of at least two hundred fifty thousand, or each
province, shall have at least one representative.
(4) x x x x (Emphasis supplied).
The petitioners posit that the 250,000 figure appearing in the above-cited provision is the minimum
population requirement for the creation of a legislative district.7 The petitioners theorize that, save in
the case of a newly created province, each legislative district created by Congress must be
supported by a minimum population of at least 250,000 in order to be valid. 8 Under this view, existing
legislative districts may be reapportioned and severed to form new districts, provided each resulting
district will represent a population of at least 250,000. On the other hand, if the reapportionment
would result in the creation of a legislative seat representing a populace of less than 250,000
inhabitants, the reapportionment must be stricken down as invalid for non-compliance with the
minimum population requirement.
In support of their theory, the petitioners point to what they claim is the intent of the framers of the
1987 Constitution to adopt a population minimum of 250,000 in the creation of additional legislative
seats.9 The petitioners argue that when the Constitutional Commission fixed the original number of
district seats in the House of Representatives to two hundred (200), they took into account the
projected national population of fifty five million (55,000,000) for the year 1986. 10 According to the
petitioners, 55 million people represented by 200 district representatives translates to roughly
250,000 people for every one (1) representative.11 Thus, the 250,000 population requirement found
in Section 5(3), Article VI of the 1987 Constitution is actually based on the population constant used
by the Constitutional Commission in distributing the initial 200 legislative seats.
Thus did the petitioners claim that in reapportioning legislative districts independently from the
creation of a province, Congress is bound to observe a 250,000 population threshold, in the same
manner that the Constitutional Commission did in the original apportionment.
Verbatim, the submission is that:
1. Republic Act 9716 is unconstitutional because the newly apportioned first district of
Camarines Sur failed to meet the population requirement for the creation of the legislative
district as explicitly provided in Article VI, Section 5, Paragraphs (1) and (3) of the
Constitution and Section 3 of the Ordinance appended thereto; and
2. Republic Act 9716 violates the principle of proportional representation as provided in
Article VI, Section 5 paragraphs (1), (3) and (4) of the Constitution.12
The provision subject of this case states:
Article VI
Section 5. (1) The House of Representatives shall be composed of not more than two hundred and
fifty members, unless otherwise fixed by law, who shall be elected from legislative districts
apportioned among the provinces, cities and the Metropolitan Manila area in accordance with the

number of their respective inhabitants, and on the basis of a uniform and progressive ratio, and
those who, as provided by law, shall be elected through a party-list system of registered national,
regional and sectoral parties or organizations.
(2) x x x x
(3) Each legislative district shall comprise, as far as practicable, contiguous, compact, and
adjacent territory. Each city with a population of at least two hundred fifty thousand, or each
province, shall have at least one representative.
(4) Within three years following the return of every census, the Congress shall make a
reapportionment of legislative districts based on the standards provided in this section.
On the other hand, the respondents, through the Office of the Solicitor General, seek the dismissal of
the present petition based on procedural and substantive grounds.
On procedural matters, the respondents argue that the petitioners are guilty of two (2) fatal technical
defects: first, petitioners committed an error in choosing to assail the constitutionality of Republic Act
No. 9716 via the remedy of Certiorari and Prohibition under Rule 65 of the Rules of Court; and
second, the petitioners have no locus standi to question the constitutionality of Republic Act No.
9716.
On substantive matters, the respondents call attention to an apparent distinction between cities and
provinces drawn by Section 5(3), Article VI of the 1987 Constitution. The respondents concede the
existence of a 250,000 population condition, but argue that a plain and simple reading of the
questioned provision will show that the same has no application with respect to the creation of
legislative districts in provinces.13 Rather, the 250,000 minimum population is only a requirement for
the creation of a legislative district in a city.
In sum, the respondents deny the existence of a fixed population requirement for the
reapportionment of districts in provinces. Therefore, Republic Act No. 9716, which only creates an
additional legislative district within the province of Camarines Sur, should be sustained as a perfectly
valid reapportionment law.
We first pass upon the threshold issues.
The respondents assert that by choosing to avail themselves of the remedies of Certiorari and
Prohibition, the petitioners have committed a fatal procedural lapse. The respondents cite the
following reasons:
1. The instant petition is bereft of any allegation that the respondents had acted without or in
excess of jurisdiction, or with grave abuse of discretion.
1avvphi1

2. The remedy of Certiorari and Prohibition must be directed against a tribunal, board, officer
or person, whether exercising judicial, quasi-judicial, or ministerial functions. Respondents
maintain that in implementing Republic Act No. 9716, they were not acting as a judicial or
quasi-judicial body, nor were they engaging in the performance of a ministerial act.
3. The petitioners could have availed themselves of another plain, speedy and adequate
remedy in the ordinary course of law. Considering that the main thrust of the instant petition
is the declaration of unconstitutionality of Republic Act No. 9716, the same could have been

ventilated through a petition for declaratory relief, over which the Supreme Court has only
appellate, not original jurisdiction.
The respondents likewise allege that the petitioners had failed to show that they had sustained, or is
in danger of sustaining any substantial injury as a result of the implementation of Republic Act No.
9716. The respondents, therefore, conclude that the petitioners lack the required legal standing to
question the constitutionality of Republic Act No. 9716.
This Court has paved the way away from procedural debates when confronted with issues that, by
reason of constitutional importance, need a direct focus of the arguments on their content and
substance.
The Supreme Court has, on more than one occasion, tempered the application of procedural
rules,14 as well as relaxed the requirement of locus standi whenever confronted with an important
issue of overreaching significance to society.15
Hence, in Del Mar v. Philippine Amusement and Gaming Corporation (PAGCOR)16 and Jaworski v.
PAGCOR,17this Court sanctioned momentary deviation from the principle of the hierarchy of courts,
and took original cognizance of cases raising issues of paramount public importance. The Jaworski
case ratiocinates:
Granting arguendo that the present action cannot be properly treated as a petition for prohibition, the
transcendental importance of the issues involved in this case warrants that we set aside the
technical defects and take primary jurisdiction over the petition at bar. One cannot deny that the
issues raised herein have potentially pervasive influence on the social and moral well being of this
nation, specially the youth; hence, their proper and just determination is an imperative need. This is
in accordance with the well-entrenched principle that rules of procedure are not inflexible tools
designed to hinder or delay, but to facilitate and promote the administration of justice. Their strict and
rigid application, which would result in technicalities that tend to frustrate, rather than promote
substantial justice, must always be eschewed. (Emphasis supplied)
Anent the locus standi requirement, this Court has already uniformly ruled in Kilosbayan v.
Guingona,18 Tatad v. Executive Secretary,19 Chavez v. Public Estates Authority20 and Bagong
Alyansang Makabayan v. Zamora,21 just to name a few, that absence of direct injury on the part of
the party seeking judicial review may be excused when the latter is able to craft an issue of
transcendental importance. In Lim v. Executive Secretary,22 this Court held that in cases of
transcendental importance, the cases must be settled promptly and definitely, and so, the standing
requirements may be relaxed. This liberal stance has been echoed in the more recent decision on
Chavez v. Gonzales.23
Given the weight of the issue raised in the instant petition, the foregoing principles must apply. The
beaten path must be taken. We go directly to the determination of whether or not a population of
250,000 is an indispensable constitutional requirement for the creation of a new legislative district in
a province.
We deny the petition.
We start with the basics. Any law duly enacted by Congress carries with it the presumption of
constitutionality.24Before a law may be declared unconstitutional by this Court, there must be a clear
showing that a specific provision of the fundamental law has been violated or transgressed. When
there is neither a violation of a specific provision of the Constitution nor any proof showing that there

is such a violation, the presumption of constitutionality will prevail and the law must be upheld. To
doubt is to sustain.25
There is no specific provision in the Constitution that fixes a 250,000 minimum population that must
compose a legislative district.
As already mentioned, the petitioners rely on the second sentence of Section 5(3), Article VI of the
1987 Constitution, coupled with what they perceive to be the intent of the framers of the Constitution
to adopt a minimum population of 250,000 for each legislative district.
The second sentence of Section 5(3), Article VI of the Constitution, succinctly provides: "Each city
with a population of at least two hundred fifty thousand, or each province, shall have at least one
representative."
The provision draws a plain and clear distinction between the entitlement of a city to a district on one
hand, and the entitlement of a province to a district on the other. For while a province is entitled to at
least a representative, with nothing mentioned about population, a city must first meet a population
minimum of 250,000 in order to be similarly entitled.
The use by the subject provision of a comma to separate the phrase "each city with a population of
at least two hundred fifty thousand" from the phrase "or each province" point to no other conclusion
than that the 250,000 minimum population is only required for a city, but not for a province. 26
Plainly read, Section 5(3) of the Constitution requires a 250,000 minimum population only for a city
to be entitled to a representative, but not so for a province.
The 250,000 minimum population requirement for legislative districts in cities was, in turn, the
subject of interpretation by this Court in Mariano, Jr. v. COMELEC.27
In Mariano, the issue presented was the constitutionality of Republic Act No. 7854, which was the
law that converted the Municipality of Makati into a Highly Urbanized City. As it happened, Republic
Act No. 7854 created an additional legislative district for Makati, which at that time was a lone
district. The petitioners in that case argued that the creation of an additional district would violate
Section 5(3), Article VI of the Constitution, because the resulting districts would be supported by a
population of less than 250,000, considering that Makati had a total population of only 450,000. The
Supreme Court sustained the constitutionality of the law and the validity of the newly created district,
explaining the operation of the Constitutional phrase "each city with a population of at least two
hundred fifty thousand," to wit:
Petitioners cannot insist that the addition of another legislative district in Makati is not in accord with
section 5(3), Article VI of the Constitution for as of the latest survey (1990 census), the population of
Makati stands at only four hundred fifty thousand (450,000). Said section provides, inter alia, that a
city with a population of at least two hundred fifty thousand (250,000) shall have at least one
representative. Even granting that the population of Makati as of the 1990 census stood at four
hundred fifty thousand (450,000), its legislative district may still be increased since it has met the
minimum population requirement of two hundred fifty thousand (250,000). In fact, Section 3 of the
Ordinance appended to the Constitution provides that a city whose population has increased to
more than two hundred fifty thousand (250,000) shall be entitled to at least one congressional
representative.28(Emphasis supplied)

The Mariano case limited the application of the 250,000 minimum population requirement for cities
only to its initial legislative district. In other words, while Section 5(3), Article VI of the Constitution
requires a city to have a minimum population of 250,000 to be entitled to a representative, it does
not have to increase its population by another 250,000 to be entitled to an additional district.
There is no reason why the Mariano case, which involves the creation of an additional district within
a city, should not be applied to additional districts in provinces. Indeed, if an additional legislative
district created within a city is not required to represent a population of at least 250,000 in order to
be valid, neither should such be needed for an additional district in a province, considering moreover
that a province is entitled to an initial seat by the mere fact of its creation and regardless of its
population.
Apropos for discussion is the provision of the Local Government Code on the creation of a province
which, by virtue of and upon creation, is entitled to at least a legislative district. Thus, Section 461 of
the Local Government Code states:
Requisites for Creation. (a) A province may be created if it has an average annual income, as
certified by the Department of Finance, of not less than Twenty million pesos (P20,000,000.00)
based on 1991 constant prices and either of the following requisites:
(i) a contiguous territory of at least two thousand (2,000) square kilometers, as certified by
the Lands Management Bureau; or
(ii) a population of not less than two hundred fifty thousand (250,000) inhabitants as certified
by the National Statistics Office.
Notably, the requirement of population is not an indispensable requirement, but is merely
an alternative addition to the indispensable income requirement.
Mariano, it would turn out, is but a reflection of the pertinent ideas that ran through the deliberations
on the words and meaning of Section 5 of Article VI.
The whats, whys, and wherefores of the population requirement of "at least two hundred fifty
thousand" may be gleaned from the records of the Constitutional Commission which, upon framing
the provisions of Section 5 of Article VI, proceeded to form an ordinance that would be appended to
the final document. The Ordinance is captioned "APPORTIONING THE SEATS OF THE HOUSE OF
REPRESENTATIVES OF THE CONGRESS OF THE PHILIPPINES TO THE DIFFERENT
LEGISLATIVE DISTRICTS IN PROVINCES AND CITIES AND THE METROPOLITAN MANILA
AREA." Such records would show that the 250,000 population benchmark was used for the 1986
nationwide apportionment of legislative districts among provinces, cities and Metropolitan Manila.
Simply put, the population figure was used to determine how many districts a province, city, or
Metropolitan Manila should have. Simply discernible too is the fact that, for the purpose, population
had to be the determinant. Even then, the requirement of 250,000 inhabitants was not taken as an
absolute minimum for one legislative district. And, closer to the point herein at issue, in the
determination of the precise district within the province to which, through the use of the population
benchmark, so many districts have been apportioned, population as a factor was not the
sole, though it was among, several determinants.
From its journal,29 we can see that the Constitutional Commission originally divided the entire country
into two hundred (200) districts, which corresponded to the original number of district
representatives. The 200 seats were distributed by the Constitutional Commission in this manner:
first, one (1) seat each was given to the seventy-three (73) provinces and the ten (10) cities with a

population of at least 250,000;30 second, the remaining seats were then redistributed among the
provinces, cities and the Metropolitan Area "in accordance with the number of their inhabitants on
the basis of a uniform and progressive ratio."31 Commissioner Davide, who later became a Member
and then Chief Justice of the Court, explained this in his sponsorship remark 32 for the Ordinance to
be appended to the 1987 Constitution:
Commissioner Davide: The ordinance fixes at 200 the number of legislative seats which are, in turn,
apportioned among provinces and cities with a population of at least 250, 000 and the Metropolitan
Area in accordance with the number of their respective inhabitants on the basis of a uniform and
progressive ratio. The population is based on the 1986 projection, with the 1980 official enumeration
as the point of reckoning. This projection indicates that our population is more or less 56
million. Taking into account the mandate that each city with at least 250, 000 inhabitants and each
province shall have at least one representative, we first allotted one seat for each of the 73
provinces, and each one for all cities with a population of at least 250, 000, which are the Cities of
Manila, Quezon, Pasay, Caloocan, Cebu, Iloilo, Bacolod, Cagayan de Oro, Davao and Zamboanga.
Thereafter, we then proceed[ed] to increase whenever appropriate the number of seats for the
provinces and cities in accordance with the number of their inhabitants on the basis of a uniform and
progressive ratio. (Emphasis supplied).
Thus was the number of seats computed for each province and city. Differentiated from this, the
determination of the districts within the province had to consider "all protests and complaints formally
received" which, the records show, dealt with determinants other than population as already
mentioned.
Palawan is a case in point. Journal No. 107 of the Constitutional Commission narrates:
INTERPELLATION OF MR. NOLLEDO:
Mr. Nolledo inquired on the reason for including Puerto Princesa in the northern towns when it was
more affinity with the southern town of Aborlan, Batarasa, Brookes Point, Narra, Quezon and
Marcos. He stated that the First District has a greater area than the Second District. He then queried
whether population was the only factor considered by the Committee in redistricting.
Replying thereto, Mr. Davide explained that the Committee took into account the standards set in
Section 5 of the Article on the Legislative Department, namely: 1) the legislative seats should be
apportioned among the provinces and cities and the Metropolitan Manila area in accordance with
their inhabitants on the basis of a uniform and progressive ratio; and 2) the legislative district must
be compact, adjacent and contiguous.
Mr. Nolledo pointed out that the last factor was not met when Puerto Princesa was included with the
northern towns. He then inquired what is the distance between Puerto Princesa from San Vicente.
xxxx
Thereupon, Mr. Nolledo stated that Puerto Princesa has a population of 75,480 and based on the
apportionment, its inclusion with the northern towns would result in a combined population of
265,000 as against only 186,000 for the south. He added that Cuyo and Coron are very important
towns in the northern part of Palawan and, in fact, Cuyo was the capital of Palawan before its
transfer to Puerto Princesa. He also pointed out that there are more potential candidates in the north
and therefore if Puerto Princesa City and the towns of Cuyo and Coron are lumped together, there
would be less candidates in the south, most of whose inhabitants are not interested in politics. He
then suggested that Puerto Princesa be included in the south or the Second District.

Mr. Davide stated that the proposal would be considered during the period of amendments. He
requested that the COMELEC staff study said proposal.33
"PROPOSED AMENDMENT OF MR. NOLLEDO
On the districting of Palawan, Mr. Nolledo pointed out that it was explained in the interpellations that
District I has a total population of 265,358 including the City of Puerto Princesa, while the Second
District has a total population of 186,733. He proposed, however, that Puerto Princesa be included in
the Second District in order to satisfy the contiguity requirement in the Constitution considering that
said City is nearer the southern towns comprising the Second District.
In reply to Mr. Monsods query, Mr. Nolledo explained that with the proposed transfer of Puerto
Princesa City to the Second District, the First District would only have a total population of 190,000
while the Second District would have 262,213, and there would be no substantial changes.
Mr. Davide accepted Mr. Nolledos proposal to insert Puerto Princesa City before the Municipality of
Aborlan.
There being no objection on the part of the Members the same was approved by the Body.
APPROVAL OF THE APPORTIONMENT AND DISTRICTING OF PALAWAN
There being no other amendment, on motion of Mr. Davide, there being no objection, the
apportionment and districting for the province of Palawan was approved by the Body.34
The districting of Palawan disregarded the 250,000 population figure. It was decided by the
importance of the towns and the city that eventually composed the districts.
Benguet and Baguio are another reference point. The Journal further narrates:
At this juncture, Mr. Davide informed the Body that Mr. Regalado made a reservation with the
Committee for the possible reopening of the approval of Region I with respect to Benguet and
Baguio City.
REMARKS OF MR. REGALADO
Mr. Regalado stated that in the formulation of the Committee, Baguio City and Tuba are placed in
one district. He stated that he was toying with the idea that, perhaps as a special consideration for
Baguio because it is the summer capital of the Philippines, Tuba could be divorced from Baguio City
so that it could, by itself, have its own constituency and Tuba could be transferred to the Second
District together with Itogon. Mr. Davide, however, pointed out that the population of Baguio City is
only 141,149.
Mr. Regalado admitted that the regular population of Baguio may be lower during certain times of the
year, but the transient population would increase the population substantially and, therefore, for
purposes of business and professional transactions, it is beyond question that population-wise,
Baguio would more than qualify, not to speak of the official business matters, transactions and
offices that are also there.

Mr. Davide adverted to Director de Limas statement that unless Tuba and Baguio City are united,
Tuba will be isolated from the rest of Benguet as the place can only be reached by passing through
Baguio City. He stated that the Committee would submit the matter to the Body.
Upon inquiry of the Chair whether he is insisting on his amendment, Mr. Regalado stated that the
Body should have a say on the matter and that the considerations he had given are not on the
demographic aspects but on the fact that Baguio City is the summer capital, the venue and situs of
many government offices and functions.
On motion of Mr. Davide, there being no objection, the Body approved the reconsideration of the
earlier approval of the apportionment and districting of Region I, particularly Benguet.
Thereafter, on motion of Mr. Davide, there being no objection, the amendment of Mr. Regalado was
put to a vote. With 14 Members voting in favor and none against, the amendment was approved by
the Body.
Mr. Davide informed that in view of the approval of the amendment, Benguet with Baguio City will
have two seats. The First District shall comprise of the municipalities of Mankayan, Buguias, Bakun,
Kabayan, Kibungan, Bokod, Atok, Kapangan, Tublay, La Trinidad, Sablan, Itogon and Tuba. The
Second District shall comprise of Baguio City alone.
There being no objection, the Body approved the apportionment and districting of Region I. 35
Quite emphatically, population was explicitly removed as a factor.
It may be additionally mentioned that the province of Cavite was divided into districts based on the
distribution of its three cities, with each district having a city: one district "supposed to be a fishing
area; another a vegetable and fruit area; and the third, a rice growing area," because such
consideration "fosters common interests in line with the standard of compactness." 36 In the districting
of Maguindanao, among the matters discussed were "political stability and common interest among
the people in the area" and the possibility of "chaos and disunity" considering the "accepted regional,
political, traditional and sectoral leaders."37 For Laguna, it was mentioned that municipalities in the
highland should not be grouped with the towns in the lowland. For Cebu, Commissioner Maambong
proposed that they should "balance the area and population." 38
Consistent with Mariano and with the framer deliberations on district apportionment, we stated in
Bagabuyo v. COMELEC39 that:
x x x Undeniably, these figures show a disparity in the population sizes of the districts. The
Constitution, however, does not require mathematical exactitude or rigid equality as a standard in
gauging equality of representation. x x x. To ensure quality representation through commonality of
interests and ease of access by the representative to the constituents, all that the Constitution
requires is that every legislative district should comprise, as far as practicable, contiguous, compact
and adjacent territory. (Emphasis supplied).
This 2008 pronouncement is fresh reasoning against the uncompromising stand of petitioner that an
additional provincial legislative district, which does not have at least a 250,000 population is not
allowed by the Constitution.
The foregoing reading and review lead to a clear lesson.

Neither in the text nor in the essence of Section 5, Article VI of the Constitution can, the petition find
support. And the formulation of the Ordinance in the implementation of the provision, nay, even the
Ordinance itself, refutes the contention that a population of 250,000 is a constitutional sine qua non
for the formation of an additional legislative district in a province, whose population growth has
increased beyond the 1986 numbers.
Translated in the terms of the present case:
1. The Province of Camarines Sur, with an estimated population of 1,693,821 in 2007 is
based on the formula and constant number of 250,000 used by the Constitutional
Commission in nationally apportioning legislative districts among provinces and cities
entitled to two (2) districts in addition to the four (4) that it was given in the 1986
apportionment. Significantly, petitioner Aquino concedes this point. 40 In other words, Section
5 of Article VI as clearly written allows and does not prohibit an additional district for the
Province of Camarines Sur, such as that provided for in Republic Act No. 9786;
2. Based on the pith and pitch of the exchanges on the Ordinance on the protests and
complaints against strict conformity with the population standard, and more importantly
based on the final districting in the Ordinance on considerations other than population, the
reapportionment or the recomposition of the first and second legislative districts in the
Province of Camarines Sur that resulted in the creation of a new legislative district is
valid even if the population of the new district is 176,383 and not 250,000 as insisted upon
by the petitioners.
3. The factors mentioned during the deliberations on House Bill No. 4264, were:
(a) the dialects spoken in the grouped municipalities;
(b) the size of the original groupings compared to that of the regrouped
municipalities;
(c) the natural division separating the municipality subject of the discussion from the
reconfigured District One; and
(d) the balancing of the areas of the three districts resulting from the redistricting of
Districts One and Two.41
Each of such factors and in relation to the others considered together, with the increased population
of the erstwhile Districts One and Two, point to the utter absence of abuse of discretion, much less
grave abuse of discretion,42 that would warrant the invalidation of Republic Act No. 9716.
To be clear about our judgment, we do not say that in the reapportionment of the first and second
legislative districts of Camarines Sur, the number of inhabitants in the resulting additional district
should not be considered. Our ruling is that population is not the only factor but is just one of several
other factors in the composition of the additional district. Such settlement is in accord with both the
text of the Constitution and the spirit of the letter, so very clearly given form in the Constitutional
debates on the exact issue presented by this petition.
1avvphi1

WHEREFORE, the petition is hereby DISMISSED. Republic Act No. 9716 entitled "An Act
Reapportioning the Composition of the First (1st) and Second (2nd) Legislative Districts in the

Province of Camarines Sur and Thereby Creating a New Legislative District From Such
Reapportionment" is a VALID LAW.
SO ORDERED.

Republic of the Philippines


SUPREME COURT
Manila
EN BANC
G.R. No. 157870

November 3, 2008

SOCIAL JUSTICE SOCIETY (SJS), petitioner


vs.
DANGEROUS DRUGS BOARD and PHILIPPINE DRUG ENFORCEMENT
AGENCY (PDEA),respondents.
x-----------------------------------------------x
G.R. No. 158633

November 3, 2008

ATTY. MANUEL J. LASERNA, JR., petitioner


vs.
DANGEROUS DRUGS BOARD and PHILIPPINE DRUG ENFORCEMENT
AGENCY, respondents.
x-----------------------------------------------x
G.R. No. 161658

November 3, 2008

AQUILINO Q. PIMENTEL, JR., petitioner


vs.
COMMISSION ON ELECTIONS, respondents.
DECISION
VELASCO, JR., J.:

In these kindred petitions, the constitutionality of Section 36 of Republic Act


No. (RA) 9165, otherwise known as the Comprehensive Dangerous Drugs Act
of 2002, insofar as it requires mandatory drug testing of candidates for public
office, students of secondary and tertiary schools, officers and employees of
public and private offices, and persons charged before the prosecutor's office
with certain offenses, among other personalities, is put in issue.
As far as pertinent, the challenged section reads as follows:
SEC. 36. Authorized Drug Testing. - Authorized drug testing shall be
done by any government forensic laboratories or by any of the drug
testing laboratories accredited and monitored by the DOH to safeguard
the quality of the test results. x x x The drug testing shall employ, among
others, two (2) testing methods, the screening test which will determine
the positive result as well as the type of drug used and the confirmatory
test which will confirm a positive screening test. x x x The following shall
be subjected to undergo drug testing:
xxxx
(c) Students of secondary and tertiary schools. - Students of secondary
and tertiary schools shall, pursuant to the related rules and regulations
as contained in the school's student handbook and with notice to the
parents, undergo a random drug testing x x x;
(d) Officers and employees of public and private offices. - Officers and
employees of public and private offices, whether domestic or overseas,
shall be subjected to undergo a random drug test as contained in the
company's work rules and regulations, x x x for purposes of reducing
the risk in the workplace. Any officer or employee found positive for use
of dangerous drugs shall be dealt with administratively which shall be a
ground for suspension or termination, subject to the provisions of Article
282 of the Labor Code and pertinent provisions of the Civil Service Law;
xxxx

(f) All persons charged before the prosecutor's office with a criminal
offense having an imposable penalty of imprisonment of not less than
six (6) years and one (1) day shall undergo a mandatory drug test;
(g) All candidates for public office whether appointed or elected both in
the national or local government shall undergo a mandatory drug test.
In addition to the above stated penalties in this Section, those found to be
positive for dangerous drugs use shall be subject to the provisions of Section
15 of this Act.
G.R. No. 161658 (Aquilino Q. Pimentel, Jr. v. Commission on Elections)
On December 23, 2003, the Commission on Elections (COMELEC) issued
Resolution No. 6486, prescribing the rules and regulations on the mandatory
drug testing of candidates for public office in connection with the May 10,
2004 synchronized national and local elections. The pertinent portions of the
said resolution read as follows:
WHEREAS, Section 36 (g) of Republic Act No. 9165 provides:
SEC. 36. Authorized Drug Testing. - x x x
xxxx
(g) All candidates for public office x x x both in the national or local
government shall undergo a mandatory drug test.
WHEREAS, Section 1, Article XI of the 1987 Constitution provides that
public officers and employees must at all times be accountable to the
people, serve them with utmost responsibility, integrity, loyalty and
efficiency;
WHEREAS, by requiring candidates to undergo mandatory drug test,
the public will know the quality of candidates they are electing and they
will be assured that only those who can serve with utmost responsibility,
integrity, loyalty, and efficiency would be elected x x x.

NOW THEREFORE, The [COMELEC], pursuant to the authority vested


in it under the Constitution, Batas Pambansa Blg. 881 (Omnibus
Election Code), [RA] 9165 and other election laws, RESOLVED to
promulgate, as it hereby promulgates, the following rules and
regulations on the conduct of mandatory drug testing to candidates for
public office[:]
SECTION 1. Coverage. - All candidates for public office, both
national and local, in the May 10, 2004 Synchronized National and
Local Elections shall undergo mandatory drug test in government
forensic laboratories or any drug testing laboratories monitored and
accredited by the Department of Health.
SEC. 3. x x x
On March 25, 2004, in addition to the drug certificates filed with their
respective offices, the Comelec Offices and employees concerned shall
submit to the Law Department two (2) separate lists of candidates. The
first list shall consist of those candidates who complied with the
mandatory drug test while the second list shall consist of those
candidates who failed to comply x x x.
SEC. 4. Preparation and publication of names of candidates. - Before
the start of the campaign period, the [COMELEC] shall prepare two
separate lists of candidates. The first list shall consist of those
candidates who complied with the mandatory drug test while the second
list shall consist of those candidates who failed to comply with said drug
test. x x x
SEC. 5. Effect of failure to undergo mandatory drug test and file drug
test certificate. - No person elected to any public office shall enter upon
the duties of his office until he has undergone mandatory drug test and
filed with the offices enumerated under Section 2 hereof the drug test
certificate herein required. (Emphasis supplied.)
Petitioner Aquilino Q. Pimentel, Jr., a senator of the Republic and a candidate
for re - election in the May 10, 2004 elections,1 filed a Petition for Certiorari

and Prohibition under Rule 65. In it, he seeks (1) to nullify Sec. 36(g) of RA
9165 and COMELEC Resolution No. 6486 dated December 23, 2003 for
being unconstitutional in that they impose a qualification for candidates for
senators in addition to those already provided for in the 1987 Constitution; and
(2) to enjoin the COMELEC from implementing Resolution No. 6486.
Pimentel invokes as legal basis for his petition Sec. 3, Article VI of the
Constitution, which states:
SECTION 3. No person shall be a Senator unless he is a natural - born
citizen of the Philippines, and, on the day of the election, is at least thirty
- five years of age, able to read and write, a registered voter, and a
resident of the Philippines for not less than two years immediately
preceding the day of the election.
According to Pimentel, the Constitution only prescribes a maximum of five (5)
qualifications for one to be a candidate for, elected to, and be a member of the
Senate. He says that both the Congress and COMELEC, by requiring, via RA
9165 and Resolution No. 6486, a senatorial aspirant, among other candidates,
to undergo a mandatory drug test, create an additional qualification that all
candidates for senator must first be certified as drug free. He adds that there
is no provision in the Constitution authorizing the Congress or COMELEC to
expand the qualification requirements of candidates for senator.
G.R. No. 157870 (Social Justice Society v. Dangerous
Drugs Board and Philippine Drug Enforcement Agency)
In its Petition for Prohibition under Rule 65, petitioner Social Justice Society
(SJS), a registered political party, seeks to prohibit the Dangerous Drugs
Board (DDB) and the Philippine Drug Enforcement Agency (PDEA) from
enforcing paragraphs (c), (d), (f), and (g) of Sec. 36 of RA 9165 on the ground
that they are constitutionally infirm. For one, the provisions constitute undue
delegation of legislative power when they give unbridled discretion to schools
and employers to determine the manner of drug testing. For another, the
provisions trench in the equal protection clause inasmuch as they can be used
to harass a student or an employee deemed undesirable. And for a third, a

person's constitutional right against unreasonable searches is also breached


by said provisions.
G.R. No. 158633 (Atty. Manuel J. Laserna, Jr. v. Dangerous
Drugs Board and Philippine Drug Enforcement Agency)
Petitioner Atty. Manuel J. Laserna, Jr., as citizen and taxpayer, also seeks in
his Petition for Certiorari and Prohibition under Rule 65 that Sec. 36(c), (d), (f),
and (g) of RA 9165 be struck down as unconstitutional for infringing on the
constitutional right to privacy, the right against unreasonable search and
seizure, and the right against self - incrimination, and for being contrary to the
due process and equal protection guarantees.
The Issue on Locus Standi
First off, we shall address the justiciability of the cases at bench and the
matter of the standing of petitioners SJS and Laserna to sue. As respondents
DDB and PDEA assert, SJS and Laserna failed to allege any incident
amounting to a violation of the constitutional rights mentioned in their separate
petitions.2
It is basic that the power of judicial review can only be exercised in connection
with a bona fidecontroversy which involves the statute sought to be
reviewed.3 But even with the presence of an actual case or controversy, the
Court may refuse to exercise judicial review unless the constitutional question
is brought before it by a party having the requisite standing to challenge it.4 To
have standing, one must establish that he or she has suffered some actual or
threatened injury as a result of the allegedly illegal conduct of the government;
the injury is fairly traceable to the challenged action; and the injury is likely to
be redressed by a favorable action.5
The rule on standing, however, is a matter of procedure; hence, it can be
relaxed for non - traditional plaintiffs, like ordinary citizens, taxpayers, and
legislators when the public interest so requires, such as when the matter is of
transcendental importance, of overarching significance to society, or of
paramount public interest.6 There is no doubt that Pimentel, as senator of the
Philippines and candidate for the May 10, 2004 elections, possesses the

requisite standing since he has substantial interests in the subject matter of


the petition, among other preliminary considerations. Regarding SJS and
Laserna, this Court is wont to relax the rule on locus standi owing primarily to
the transcendental importance and the paramount public interest involved in
the enforcement of Sec. 36 of RA 9165.
The Consolidated Issues
The principal issues before us are as follows:
(1) Do Sec. 36(g) of RA 9165 and COMELEC Resolution No. 6486 impose an
additional qualification for candidates for senator? Corollarily, can Congress
enact a law prescribing qualifications for candidates for senator in addition to
those laid down by the Constitution? and
(2) Are paragraphs (c), (d), (f), and (g) of Sec. 36, RA 9165 unconstitutional?
Specifically, do these paragraphs violate the right to privacy, the right against
unreasonable searches and seizure, and the equal protection clause? Or do
they constitute undue delegation of legislative power?
Pimentel Petition
(Constitutionality of Sec. 36[g] of RA 9165 and
COMELEC Resolution No. 6486)
In essence, Pimentel claims that Sec. 36(g) of RA 9165 and COMELEC
Resolution No. 6486 illegally impose an additional qualification on candidates
for senator. He points out that, subject to the provisions on nuisance
candidates, a candidate for senator needs only to meet the qualifications laid
down in Sec. 3, Art. VI of the Constitution, to wit: (1) citizenship, (2) voter
registration, (3) literacy, (4) age, and (5) residency. Beyond these stated
qualification requirements, candidates for senator need not possess any other
qualification to run for senator and be voted upon and elected as member of
the Senate. The Congress cannot validly amend or otherwise modify these
qualification standards, as it cannot disregard, evade, or weaken the force of a
constitutional mandate,7 or alter or enlarge the Constitution.

Pimentel's contention is well - taken. Accordingly, Sec. 36(g) of RA 9165


should be, as it is hereby declared as, unconstitutional. It is basic that if a law
or an administrative rule violates any norm of the Constitution, that issuance is
null and void and has no effect. The Constitution is the basic law to which all
laws must conform; no act shall be valid if it conflicts with the Constitution. 8 In
the discharge of their defined functions, the three departments of government
have no choice but to yield obedience to the commands of the Constitution.
Whatever limits it imposes must be observed.9
Congress' inherent legislative powers, broad as they may be, are subject to
certain limitations. As early as 1927, in Government v. Springer, the Court has
defined, in the abstract, the limits on legislative power in the following wise:
Someone has said that the powers of the legislative department of the
Government, like the boundaries of the ocean, are unlimited. In
constitutional governments, however, as well as governments acting
under delegated authority, the powers of each of the departments x x x
are limited and confined within the four walls of the constitution or the
charter, and each department can only exercise such powers as are
necessarily implied from the given powers. The Constitution is the shore
of legislative authority against which the waves of legislative enactment
may dash, but over which it cannot leap.10
Thus, legislative power remains limited in the sense that it is subject to
substantive and constitutional limitations which circumscribe both the exercise
of the power itself and the allowable subjects of legislation.11 The substantive
constitutional limitations are chiefly found in the Bill of Rights12 and other
provisions, such as Sec. 3, Art. VI of the Constitution prescribing the
qualifications of candidates for senators.
In the same vein, the COMELEC cannot, in the guise of enforcing and
administering election laws or promulgating rules and regulations to
implement Sec. 36(g), validly impose qualifications on candidates for senator
in addition to what the Constitution prescribes. If Congress cannot require a
candidate for senator to meet such additional qualification, the COMELEC, to
be sure, is also without such power. The right of a citizen in the democratic

process of election should not be defeated by unwarranted impositions of


requirement not otherwise specified in the Constitution.13
Sec. 36(g) of RA 9165, as sought to be implemented by the assailed
COMELEC resolution, effectively enlarges the qualification requirements
enumerated in the Sec. 3, Art. VI of the Constitution. As couched, said Sec.
36(g) unmistakably requires a candidate for senator to be certified illegal drug clean, obviously as a pre - condition to the validity of a certificate of
candidacy for senator or, with like effect, a condition sine qua non to be voted
upon and, if proper, be proclaimed as senator - elect. The COMELEC
resolution completes the chain with the proviso that "[n]o person elected to
any public office shall enter upon the duties of his office until he has
undergone mandatory drug test." Viewed, therefore, in its proper context, Sec.
36(g) of RA 9165 and the implementing COMELEC Resolution add another
qualification layer to what the 1987 Constitution, at the minimum, requires for
membership in the Senate. Whether or not the drug - free bar set up under the
challenged provision is to be hurdled before or after election is really of no
moment, as getting elected would be of little value if one cannot assume office
for non - compliance with the drug - testing requirement.
It may of course be argued, in defense of the validity of Sec. 36(g) of RA
9165, that the provision does not expressly state that non - compliance with
the drug test imposition is a disqualifying factor or would work to nullify a
certificate of candidacy. This argument may be accorded plausibility if the drug
test requirement is optional. But the particular section of the law, without
exception, made drug - testing on those covered mandatory, necessarily
suggesting that the obstinate ones shall have to suffer the adverse
consequences for not adhering to the statutory command. And since the
provision deals with candidates for public office, it stands to reason that the
adverse consequence adverted to can only refer to and revolve around the
election and the assumption of public office of the candidates. Any other
construal would reduce the mandatory nature of Sec. 36(g) of RA 9165 into a
pure jargon without meaning and effect whatsoever.
While it is anti - climactic to state it at this juncture, COMELEC Resolution No.
6486 is no longer enforceable, for by its terms, it was intended to cover only
the May 10, 2004 synchronized elections and the candidates running in that

electoral event. Nonetheless, to obviate repetition, the Court deems it


appropriate to review and rule, as it hereby rules, on its validity as an
implementing issuance.
It ought to be made abundantly clear, however, that the unconstitutionality of
Sec. 36(g) of RA 9165 is rooted on its having infringed the constitutional
provision defining the qualification or eligibility requirements for one aspiring to
run for and serve as senator.
SJS Petition
(Constitutionality of Sec. 36[c], [d], [f], and [g] of RA 9165)
The drug test prescribed under Sec. 36(c), (d), and (f) of RA 9165 for
secondary and tertiary level students and public and private employees, while
mandatory, is a random and suspicionless arrangement. The objective is to
stamp out illegal drug and safeguard in the process "the well being of [the]
citizenry, particularly the youth, from the harmful effects of dangerous drugs."
This statutory purpose, per the policy - declaration portion of the law, can be
achieved via the pursuit by the state of "an intensive and unrelenting
campaign against the trafficking and use of dangerous drugs x x x through an
integrated system of planning, implementation and enforcement of anti - drug
abuse policies, programs and projects."14 The primary legislative intent is not
criminal prosecution, as those found positive for illegal drug use as a result of
this random testing are not necessarily treated as criminals. They may even
be exempt from criminal liability should the illegal drug user consent to
undergo rehabilitation. Secs. 54 and 55 of RA 9165 are clear on this point:
Sec. 54. Voluntary Submission of a Drug Dependent to Confinement,
Treatment and Rehabilitation. - A drug dependent or any person who
violates Section 15 of this Act may, by himself/herself or through his/her
parent, [close relatives] x x x apply to the Board x x x for treatment and
rehabilitation of the drug dependency. Upon such application, the Board
shall bring forth the matter to the Court which shall order that the
applicant be examined for drug dependency. If the examination x x x
results in the certification that the applicant is a drug dependent, he/she
shall be ordered by the Court to undergo treatment and rehabilitation in
a Center designated by the Board x x x.

xxxx
Sec. 55. Exemption from the Criminal Liability Under the Voluntary
Submission Program. - A drug dependent under the voluntary
submission program, who is finally discharged from confinement, shall
be exempt from the criminal liability under Section 15 of this Act subject
to the following conditions:
xxxx
School children, the US Supreme Court noted, are most vulnerable to the
physical, psychological, and addictive effects of drugs. Maturing nervous
systems of the young are more critically impaired by intoxicants and are more
inclined to drug dependency. Their recovery is also at a depressingly low
rate.15
The right to privacy has been accorded recognition in this jurisdiction as a
facet of the right protected by the guarantee against unreasonable search and
seizure16 under Sec. 2, Art. III17 of the Constitution. But while the right to
privacy has long come into its own, this case appears to be the first time that
the validity of a state - decreed search or intrusion through the medium of
mandatory random drug testing among students and employees is, in this
jurisdiction, made the focal point. Thus, the issue tendered in these
proceedings is veritably one of first impression.
US jurisprudence is, however, a rich source of persuasive jurisprudence. With
respect to random drug testing among school children, we turn to the
teachings of Vernonia School District 47J v. Acton (Vernonia) and Board of
Education of Independent School District No. 92 of Pottawatomie County, et
al. v. Earls, et al. (Board of Education),18 both fairly pertinent US Supreme
Court - decided cases involving the constitutionality of governmental search.
In Vernonia, school administrators in Vernonia, Oregon wanted to address the
drug menace in their respective institutions following the discovery of frequent
drug use by school athletes. After consultation with the parents, they required
random urinalysis drug testing for the school's athletes. James Acton, a high
school student, was denied participation in the football program after he

refused to undertake the urinalysis drug testing. Acton forthwith sued, claiming
that the school's drug testing policy violated, inter alia, the Fourth
Amendment19 of the US Constitution.
The US Supreme Court, in fashioning a solution to the issues raised
in Vernonia, considered the following: (1) schools stand in loco parentis over
their students; (2) school children, while not shedding their constitutional rights
at the school gate, have less privacy rights; (3) athletes have less privacy
rights than non - athletes since the former observe communal undress before
and after sports events; (4) by joining the sports activity, the athletes
voluntarily subjected themselves to a higher degree of school supervision and
regulation; (5) requiring urine samples does not invade a student's privacy
since a student need not undress for this kind of drug testing; and (6) there is
need for the drug testing because of the dangerous effects of illegal drugs on
the young. The US Supreme Court held that the policy constituted reasonable
search under the Fourth20 and 14th Amendments and declared the random
drug - testing policy constitutional.
In Board of Education, the Board of Education of a school in Tecumseh,
Oklahoma required a drug test for high school students desiring to join extra curricular activities. Lindsay Earls, a member of the show choir, marching
band, and academic team declined to undergo a drug test and averred that
the drug - testing policy made to apply to non - athletes violated the Fourth
and 14th Amendments. As Earls argued, unlike athletes who routinely
undergo physical examinations and undress before their peers in locker
rooms, non - athletes are entitled to more privacy.
The US Supreme Court, citing Vernonia, upheld the constitutionality of drug
testing even among non - athletes on the basis of the school's custodial
responsibility and authority. In so ruling, said court made no distinction
between a non - athlete and an athlete. It ratiocinated that schools and
teachers act in place of the parents with a similar interest and duty of
safeguarding the health of the students. And in holding that the school could
implement its random drug - testing policy, the Court hinted that such a test
was a kind of search in which even a reasonable parent might need to
engage.

In sum, what can reasonably be deduced from the above two cases and
applied to this jurisdiction are: (1) schools and their administrators stand in
loco parentis with respect to their students; (2) minor students have
contextually fewer rights than an adult, and are subject to the custody and
supervision of their parents, guardians, and schools; (3) schools, acting in
loco parentis, have a duty to safeguard the health and well - being of their
students and may adopt such measures as may reasonably be necessary to
discharge such duty; and (4) schools have the right to impose conditions on
applicants for admission that are fair, just, and non-discriminatory.
Guided by Vernonia and Board of Education, the Court is of the view and so
holds that the provisions of RA 9165 requiring mandatory, random, and
suspicionless drug testing of students are constitutional. Indeed, it is within the
prerogative of educational institutions to require, as a condition for admission,
compliance with reasonable school rules and regulations and policies. To be
sure, the right to enroll is not absolute; it is subject to fair, reasonable, and
equitable requirements.
The Court can take judicial notice of the proliferation of prohibited drugs in the
country that threatens the well - being of the people,21 particularly the youth
and school children who usually end up as victims. Accordingly, and until a
more effective method is conceptualized and put in motion, a random drug
testing of students in secondary and tertiary schools is not only acceptable but
may even be necessary if the safety and interest of the student population,
doubtless a legitimate concern of the government, are to be promoted and
protected. To borrow from Vernonia, "[d]eterring drug use by our Nation's
schoolchildren is as important as enhancing efficient enforcement of the
Nation's laws against the importation of drugs"; the necessity for the State to
act is magnified by the fact that the effects of a drug - infested school are
visited not just upon the users, but upon the entire student body and
faculty.22 Needless to stress, the random testing scheme provided under the
law argues against the idea that the testing aims to incriminate unsuspecting
individual students.
Just as in the case of secondary and tertiary level students, the mandatory but
random drug test prescribed by Sec. 36 of RA 9165 for officers and
employees of public and private offices is justifiable, albeit not exactly for the

same reason. The Court notes in this regard that petitioner SJS, other than
saying that "subjecting almost everybody to drug testing, without probable
cause, is unreasonable, an unwarranted intrusion of the individual right to
privacy,"23 has failed to show how the mandatory, random, and suspicionless
drug testing under Sec. 36(c) and (d) of RA 9165 violates the right to privacy
and constitutes unlawful and/or unconsented search under Art. III, Secs. 1 and
2 of the Constitution.24 Petitioner Laserna's lament is just as simplistic,
sweeping, and gratuitous and does not merit serious consideration. Consider
what he wrote without elaboration:
The US Supreme Court and US Circuit Courts of Appeals have made
various rulings on the constitutionality of mandatory drug tests in the
school and the workplaces. The US courts have been consistent in their
rulings that the mandatory drug tests violate a citizen's constitutional
right to privacy and right against unreasonable search and seizure. They
are quoted extensively hereinbelow.25
The essence of privacy is the right to be left alone.26 In context, the right to
privacy means the right to be free from unwarranted exploitation of one's
person or from intrusion into one's private activities in such a way as to cause
humiliation to a person's ordinary sensibilities. 27 And while there has been
general agreement as to the basic function of the guarantee against
unwarranted search, "translation of the abstract prohibition against
unreasonable searches and seizures' into workable broad guidelines for the
decision of particular cases is a difficult task," to borrow from C. Camara v.
Municipal Court.28 Authorities are agreed though that the right to privacy yields
to certain paramount rights of the public and defers to the state's exercise of
police power.29
As the warrantless clause of Sec. 2, Art III of the Constitution is couched and
as has been held, "reasonableness" is the touchstone of the validity of a
government search or intrusion.30 And whether a search at issue hews to the
reasonableness standard is judged by the balancing of the government mandated intrusion on the individual's privacy interest against the promotion
of some compelling state interest.31 In the criminal context, reasonableness
requires showing of probable cause to be personally determined by a judge.
Given that the drug - testing policy for employees--and students for that

matter--under RA 9165 is in the nature of administrative search needing what


was referred to inVernonia as "swift and informal disciplinary procedures," the
probable - cause standard is not required or even practicable. Be that as it
may, the review should focus on the reasonableness of the challenged
administrative search in question.
The first factor to consider in the matter of reasonableness is the nature of the
privacy interest upon which the drug testing, which effects a search within the
meaning of Sec. 2, Art. III of the Constitution, intrudes. In this case, the office
or workplace serves as the backdrop for the analysis of the privacy
expectation of the employees and the reasonableness of drug testing
requirement. The employees' privacy interest in an office is to a large extent
circumscribed by the company's work policies, the collective bargaining
agreement, if any, entered into by management and the bargaining unit, and
the inherent right of the employer to maintain discipline and efficiency in the
workplace. Their privacy expectation in a regulated office environment is, in
fine, reduced; and a degree of impingement upon such privacy has been
upheld.
Just as defining as the first factor is the character of the intrusion authorized
by the challenged law. Reduced to a question form, is the scope of the search
or intrusion clearly set forth, or, as formulated in Ople v. Torres, is the enabling
law authorizing a search "narrowly drawn" or "narrowly focused"?32
The poser should be answered in the affirmative. For one, Sec. 36 of RA 9165
and its implementing rules and regulations (IRR), as couched, contain
provisions specifically directed towards preventing a situation that would
unduly embarrass the employees or place them under a humiliating
experience. While every officer and employee in a private establishment is
under the law deemed forewarned that he or she may be a possible subject of
a drug test, nobody is really singled out in advance for drug testing. The goal
is to discourage drug use by not telling in advance anyone when and who is to
be tested. And as may be observed, Sec. 36(d) of RA 9165 itself prescribes
what, in Ople, is a narrowing ingredient by providing that the employees
concerned shall be subjected to "random drug test as contained in the
company's work rules and regulations x x x for purposes of reducing the risk in
the work place."

For another, the random drug testing shall be undertaken under conditions
calculated to protect as much as possible the employee's privacy and dignity.
As to the mechanics of the test, the law specifies that the procedure shall
employ two testing methods, i.e., the screening test and the confirmatory test,
doubtless to ensure as much as possible the trustworthiness of the results.
But the more important consideration lies in the fact that the test shall be
conducted by trained professionals in access - controlled laboratories
monitored by the Department of Health (DOH) to safeguard against results
tampering and to ensure an accurate chain of custody.33 In addition, the IRR
issued by the DOH provides that access to the drug results shall be on the
"need to know" basis;34 that the "drug test result and the records shall be
[kept] confidential subject to the usual accepted practices to protect the
confidentiality of the test results."35 Notably, RA 9165 does not oblige the
employer concerned to report to the prosecuting agencies any information or
evidence relating to the violation of the Comprehensive Dangerous Drugs
Act received as a result of the operation of the drug testing. All told, therefore,
the intrusion into the employees' privacy, under RA 9165, is accompanied by
proper safeguards, particularly against embarrassing leakages of test results,
and is relatively minimal.
To reiterate, RA 9165 was enacted as a measure to stamp out illegal drug in
the country and thus protect the well - being of the citizens, especially the
youth, from the deleterious effects of dangerous drugs. The law intends to
achieve this through the medium, among others, of promoting and resolutely
pursuing a national drug abuse policy in the workplace via a mandatory
random drug test.36 To the Court, the need for drug testing to at least minimize
illegal drug use is substantial enough to override the individual's privacy
interest under the premises. The Court can consider that the illegal drug
menace cuts across gender, age group, and social - economic lines. And it
may not be amiss to state that the sale, manufacture, or trafficking of illegal
drugs, with their ready market, would be an investor's dream were it not for
the illegal and immoral components of any of such activities. The drug
problem has hardly abated since the martial law public execution of a
notorious drug trafficker. The state can no longer assume a laid back stance
with respect to this modern - day scourge. Drug enforcement agencies
perceive a mandatory random drug test to be an effective way of preventing

and deterring drug use among employees in private offices, the threat of
detection by random testing being higher than other modes. The Court holds
that the chosen method is a reasonable and enough means to lick the
problem.
Taking into account the foregoing factors, i.e., the reduced expectation of
privacy on the part of the employees, the compelling state concern likely to be
met by the search, and the well - defined limits set forth in the law to properly
guide authorities in the conduct of the random testing, we hold that the
challenged drug test requirement is, under the limited context of the case,
reasonable and, ergo, constitutional.
Like their counterparts in the private sector, government officials and
employees also labor under reasonable supervision and restrictions imposed
by the Civil Service law and other laws on public officers, all enacted to
promote a high standard of ethics in the public service.37 And if RA 9165
passes the norm of reasonableness for private employees, the more reason
that it should pass the test for civil servants, who, by constitutional command,
are required to be accountable at all times to the people and to serve them
with utmost responsibility and efficiency.38
Petitioner SJS' next posture that Sec. 36 of RA 9165 is objectionable on the
ground of undue delegation of power hardly commends itself for concurrence.
Contrary to its position, the provision in question is not so extensively drawn
as to give unbridled options to schools and employers to determine the
manner of drug testing. Sec. 36 expressly provides how drug testing for
students of secondary and tertiary schools and officers/employees of
public/private offices should be conducted. It enumerates the persons who
shall undergo drug testing. In the case of students, the testing shall be in
accordance with the school rules as contained in the student handbook and
with notice to parents. On the part of officers/employees, the testing shall take
into account the company's work rules. In either case, the random procedure
shall be observed, meaning that the persons to be subjected to drug test shall
be picked by chance or in an unplanned way. And in all cases, safeguards
against misusing and compromising the confidentiality of the test results are
established.

Lest it be overlooked, Sec. 94 of RA 9165 charges the DDB to issue, in


consultation with the DOH, Department of the Interior and Local Government,
Department of Education, and Department of Labor and Employment, among
other agencies, the IRR necessary to enforce the law. In net effect then, the
participation of schools and offices in the drug testing scheme shall always be
subject to the IRR of RA 9165. It is, therefore, incorrect to say that schools
and employers have unchecked discretion to determine how often, under what
conditions, and where the drug tests shall be conducted.
The validity of delegating legislative power is now a quiet area in the
constitutional landscape.39 In the face of the increasing complexity of the task
of the government and the increasing inability of the legislature to cope
directly with the many problems demanding its attention, resort to delegation
of power, or entrusting to administrative agencies the power of subordinate
legislation, has become imperative, as here.
Laserna Petition (Constitutionality of Sec. 36[c], [d],
[f], and [g] of RA 9165)
Unlike the situation covered by Sec. 36(c) and (d) of RA 9165, the Court finds
no valid justification for mandatory drug testing for persons accused of crimes.
In the case of students, the constitutional viability of the mandatory, random,
and suspicionless drug testing for students emanates primarily from the
waiver by the students of their right to privacy when they seek entry to the
school, and from their voluntarily submitting their persons to the parental
authority of school authorities. In the case of private and public employees,
the constitutional soundness of the mandatory, random, and suspicionless
drug testing proceeds from the reasonableness of the drug test policy and
requirement.
We find the situation entirely different in the case of persons charged before
the public prosecutor's office with criminal offenses punishable with six (6)
years and one (1) day imprisonment. The operative concepts in the mandatory
drug testing are "randomness" and "suspicionless." In the case of persons
charged with a crime before the prosecutor's office, a mandatory drug testing
can never be random or suspicionless. The ideas of randomness and being
suspicionless are antithetical to their being made defendants in a criminal

complaint. They are not randomly picked; neither are they beyond suspicion.
When persons suspected of committing a crime are charged, they are singled
out and are impleaded against their will. The persons thus charged, by the
bare fact of being haled before the prosecutor's office and peaceably
submitting themselves to drug testing, if that be the case, do not necessarily
consent to the procedure, let alone waive their right to privacy.40 To impose
mandatory drug testing on the accused is a blatant attempt to harness a
medical test as a tool for criminal prosecution, contrary to the stated
objectives of RA 9165. Drug testing in this case would violate a persons' right
to privacy guaranteed under Sec. 2, Art. III of the Constitution. Worse still, the
accused persons are veritably forced to incriminate themselves.
WHEREFORE, the Court resolves to GRANT the petition in G.R. No. 161658
and declares Sec. 36(g) of RA 9165 and COMELEC Resolution No.
6486 as UNCONSTITUTIONAL; and toPARTIALLY GRANT the petition in
G.R. Nos. 157870 and 158633 by declaring Sec. 36(c) and (d) ofRA 9165
CONSTITUTIONAL, but declaring its Sec. 36(f) UNCONSTITUTIONAL. All
concerned agencies are, accordingly, permanently enjoined from
implementing Sec. 36(f) and (g) of RA 9165. No costs.
SO ORDERED.

LAWYERS LEAGUE FOR A BETTER PHILIPPINES vs.


AQUINO
(G.R. No. 73748 - May 22, 1986)
-----------------------(There is no "Full-Text" of this case. This is a Minute Resolution made by the SC.)
Minute Resolutions
EN BANC
[G.R. No. 73748, May 22, 1986]
LAWYERS LEAGUE FOR A BETTER PHILIPPINES AND/OR OLIVER A. LOZANO VS. PRESIDENT
CORAZON C. AQUINO, ET AL.

SIRS/MESDAMES:
Quoted hereunder, for your information, is a resolution of this Court MAY 22, 1986.
In G.R. No. 73748, Lawyers League for a Better Philippines vs. President Corazon C. Aquino, et al.; G.R.
No. 73972, People's Crusade for Supremacy of the Constitution vs. Mrs. Cory Aquino, et al., and G.R. No.
73990, Councilor Clifton U. Ganay vs. Corazon C. Aquino, et al., the legitimacy of the government of
President Aquino is questioned. It is claimed that her government is illegal because it was not established
pursuant to the 1973 Constitution.
As early as April 10, 1986, this Court* had already voted to dismiss the petitions for the reasons to
be stated below. On April 17, 1986, Atty. Lozano as counsel for the petitioners in G.R. Nos. 73748
and 73972 withdrew the petitions and manifested that they would pursue the question by extrajudicial methods. The withdrawal is functus oficio.
The three petitions obviously are not impressed with merit. Petitioners have no personality to sue and
their petitions state no cause of action. For the legitimacy of the Aquino government is not a justiciable
matter. It belongs to the realm of politics where only the people of the Philippines are the judge. And the
people have made the judgment; they have accepted the government of President Corazon C. Aquino
which is in effective control of the entire country so that it is not merely a de factogovernment but is in fact
and law a de jure government. Moreover, the community of nations has recognized the legitimacy of the
present government. All the eleven members of this Court, as reorganized, have sworn to uphold the
fundamental law of the Republic under her government.
In view of the foregoing, the petitions are hereby dismissed.

Very truly yours,


(Sgd.) GLORIA C. PARAS
Clerk of Court

* The Court was then composed of Teehankee, C.J. and Abad Santos., Melencio-Herrera, Plana, Escolin,
Gutierrez, Jr., Cuevas, Alampay and Patajo, JJ.-----------------------------------------DIGEST
FACTS:
On February 25, 1986, President Corazon Aquino issued Proclamation No. 1 announcing that she and
Vice President Laurel were taking power.
On March 25, 1986, proclamation No.3 was issued providing the basis of the Aquino government
assumption of power by stating that the "new government was installed through a direct exercise of the
power of the Filipino people assisted by units of the New Armed Forces of the Philippines."
ISSUE:
Whether or not the government of Corazon Aquino is legitimate.

HELD:
Yes. The legitimacy of the Aquino government is not a justiciable matter but belongs to the realm of
politics where only the people are the judge.

The Court further held that:

The people have accepted the Aquino government which is in effective control of the entire
country;

It is not merely a de facto government but in fact and law a de jure government; and

The community of nations has recognized the legitimacy of the new government.

LAWYERS LEAGUE FOR A BETTER PHILIPPINES vs. AQUINO


(G.R. No. 73748 - May 22, 1986)
-----------------------(There is no "Full-Text" of this case. This is a Minute Resolution made by the SC.)

Minute Resolutions

EN BANC

[G.R. No. 73748, May 22, 1986]

LAWYERS LEAGUE FOR A BETTER PHILIPPINES AND/OR OLIVER A. LOZANO VS.


PRESIDENT CORAZON C. AQUINO, ET AL.

SIRS/MESDAMES:

Quoted hereunder, for your information, is a resolution of this Court MAY 22, 1986.

In G.R. No. 73748, Lawyers League for a Better Philippines vs. President Corazon C.
Aquino, et al.; G.R. No. 73972, People's Crusade for Supremacy of the Constitution
vs. Mrs. Cory Aquino, et al., and G.R. No. 73990, Councilor Clifton U. Ganay vs.
Corazon C. Aquino, et al., the legitimacy of the government of President Aquino is

questioned. It is claimed that her government is illegal because it was not


established pursuant to the 1973 Constitution.

As early as April 10, 1986, this Court* had already voted to dismiss the petitions for
the reasons to be stated below. On April 17, 1986, Atty. Lozano as counsel for the
petitioners in G.R. Nos. 73748 and 73972 withdrew the petitions and manifested
that they would pursue the question by extra-judicial methods. The withdrawal is
functus oficio.

The three petitions obviously are not impressed with merit. Petitioners have no
personality to sue and their petitions state no cause of action. For the legitimacy of
the Aquino government is not a justiciable matter. It belongs to the realm of politics
where only the people of the Philippines are the judge. And the people have made
the judgment; they have accepted the government of President Corazon C. Aquino
which is in effective control of the entire country so that it is not merely a de
factogovernment but is in fact and law a de jure government. Moreover, the
community of nations has recognized the legitimacy of the present government. All
the eleven members of this Court, as reorganized, have sworn to uphold the
fundamental law of the Republic under her government.

In view of the foregoing, the petitions are hereby dismissed.

Very truly yours,

(Sgd.) GLORIA C. PARAS


Clerk of Court

* The Court was then composed of Teehankee, C.J. and Abad Santos., MelencioHerrera, Plana, Escolin, Gutierrez, Jr., Cuevas, Alampay and Patajo,
JJ.-----------------------------------------DIGEST

FACTS:
On February 25, 1986, President Corazon Aquino issued Proclamation No. 1
announcing that she and Vice President Laurel were taking power.
On March 25, 1986, proclamation No.3 was issued providing the basis of the Aquino
government assumption of power by stating that the "new government was
installed through a direct exercise of the power of the Filipino people assisted by
units of the New Armed Forces of the Philippines."

ISSUE:
Whether or not the government of Corazon Aquino is legitimate.

HELD:
Yes. The legitimacy of the Aquino government is not a justiciable matter but belongs
to the realm of politics where only the people are the judge.

The Court further held that:


The people have accepted the Aquino government which is in effective control of
the entire country;
It is not merely a de facto government but in fact and law a de jure government;
and
The community of nations has recognized the legitimacy of the new government.

S-ar putea să vă placă și